Sunteți pe pagina 1din 390

ABSTRACT

Complete Paediatric
Lecture Materials as at
September 2019, with,
percentile charts, Past
Questions, Basic
Paediatric information
and important
paediatric formulas.
For all those

THE
individuals who like
everything being
together in one place
UNTH, College Of
Medicine

PAEDIATRIC
Department of
Paediatric Medicine,
UNTH, Nigeria.

GUIDE
Complete Lecture Materials Compiled By Obasi. D. C. Chinedu
(2020 MBBS Class)
Compiled by Obasi. D. C. Chinedu

PREFACE
This material has been compiled for myself and those like me who are not comfortable with Soft copy materials. We love to
scribble on our books when we read, because that’s the only time we would agree that we are actually reading. Also for all those
individuals who like everything being together in one place so as not to be seeing vision during exams and start wondering
which materials the questions came from.

In these compilation, You would find some images might be missing, in order to compress the size. However, relevant images
are compiled within. But you should always refer to the original material for missing images. The materials are from 020 Class
lectures (95%) and 019 Class (5%).
Also, percentile charts and Past questions has been added to the back of the compilation, while basic information and important
paediatric formulas I do use during posting has been added to the first chapter, so that you can have an all in one material when
you print it out.

My friends always say that I’m the only person who usually gives advises about our exams, even when I am yet to sit for or pass
the exam yet, hence the call me Dr. Mallampati. When you see them, you ask them why. However, these advices comes from
the stuffs I’ve seen while doing the posting. Hence, I’d mention a few here;

1. Passing in Clinical class is so much different than that from Pre-clinical class. In clinical class (which actually beings from
the M1S1 posting), you can pass very well from going for postings and participating in demonstrations, whereas, in the
preclinical class, you can pass even when you sit at home, by just reading your books or materials. This is very important and is
an important factor while most guys don’t do well in Clinical classes, because we are usually shy, and “don’t want to fall
hand’, hence we tend to stay behind, where the consultants wont see us or ask us to demonstrate.
Please, always stay in front. Infact, drag the front with the girls, make a fool of yourselves in demonstration, because here, we
only learn by making mistakes and practicing further. I remember everyday, I’d ginger myself and say “today, we die here. I
must clerk and do all examinations before I leave. Only for me to start crying for them to sign for us once its 12 noon. Don’t be
like me. Sometimes, I do wonder how the girls get all this energy. Lemme not lie.

2. Always go for postings with the priority to learn, and not just to sign. Be a jovial person, and get familiar with all staffs,
including the house officers and nurses, and you would see that 100% attendance is a piece of cake, even If you don’t have 50%
as well as procedure book signings and Exams/End of posting allotment of patient. You wont know how important this is till the
time comes. Always strive to be prominent and participate in everything whenever you are around. Always answer questions,
even if you say rubbish. Don’t stop. Keep saying those jargons. You learn by failing. Attitude is everything.

3. Do not finish Paediatric without visit Prof. Obidike’s, Prof. (Rev) Uwaezuoke or Prof. Ikefuna’s group as well as the Asthma
and Neurology Clinic regardless of if you are posted there or not. Its just an advice. You would discover the importance later

At the end of the day, never forget that, “If you want to make a difference in the world, You have to be different from
the world.

P/S: THIS COMPILATION SHOULD NOT GO BEYOND STUDENTS OF THE FACULTY OF MEDICINE,
UNIVERSITY OF NIGERIA TEACHING HOSPITAL, ITUKU OZALLA. (for Privacy Reasons).

1
Compiled by Obasi. D. C. Chinedu

TABLE OF CONTENTS- P1
PREFACE 1
BASIC PAEDIATRIC INFORMATION & FORMULAS Obasi. D. C. Chinedu 5

GENERAL PAEDIATRIC AND EMERGENCIES


Introductory Paediatrics: History taking and Physical examination HOD 7
Concept and Investigation of fevers/Fever of unknown origin (FUO) Prof HU Okafor 78
Accidents, Envenomation, Bites and Stings Dr Edelu BO 87

NEONATOLOGY
The newborn (normal & abnormal) Examination of the newborn Prof Obu HA 23
Prenatal Screening/Diagnosis Dr Iloh KK 31
Prematurity/Low birthweight Dr Obumneme-Anyim I 76

PAEDIATRIC GASTROENTEROLOGY
Diarrhoeal diseases/ Fluid and electrolyte balance I & II Dr Ughasoro MD 8
Infant feeding (Including breast feeding and complimentary feeding) Dr Ughasoro MD 20
Childhood nutrition, Malnutrition and FTT I & II Dr Ughasoro MD 34
Anthropometry (nutrition & growth assessment) Short stature I & II Dr Ubesie AC 40
Viral hepatitis and investigation of hepatobiliary disorders Dr Ubesie AC 66
Malabsorption Syndromes Dr Ubesie AC 71

PAEDIATRIC HAEMATOLOGY/ONCOLOGY
HIV/AIDS Prof Emodi IJ 15
Malaria Prof HU Okafor 26

PAEDIATRIC CARDIOLOGY
Heart dysfunctions, cardiac arrhythmias and Heart failure Dr Arodiwe IO 51
Investigation of cardiovascular diseases Dr Ujunwa FA 82

PAEDIATRIC ENDOCRINOLOGY
Thyroid Disorders Prof Obu HA 62

SPECIAL CONSIDERATIONS
Adolescent health, puberty and growth I & II Dr Onukwili V 46
Common skin diseases in children Dr Ezenwosu OU 84
Bioethics in Paediatrics Dr Obidike EO 92

2
Compiled by Obasi. D. C. Chinedu

TABLE OF CONTENTS- P2 (From Page 98)


GENERAL PAEDIATRIC AND EMERGENCIES
Concepts in the management of Paediatrics Emergencies Prof. Oguonu 95
Investigations in paediatrics I & II Dr Ujunwa 103
Immunization and Vaccine preventable diseases I & II Dr B N Tagbo 178
Sepsis and Antibiotics stewardship Prof Oguonu 227
Procedures in paediatrics (diagnostic and therapeutic)/ instrumentation in paediatrics Dr. Igbokwe 243
Cardiopulmonary Resuscitation Dr. O. Iloh 274
The unconscious child (aetiology, Assessment and management) Dr Edelu 287

NEONATOLOGY
Overview of inherited disorders and common chromosomal anomalies Prof Obu 136
Neonatal Infections/Sepsis Prof. Obu 256
Perinatal Asphyxia and Neonatal resuscitation Dr. K. Iloh 262
Neonatal Jaundice Dr Obum-Anyim 264

PAEDIATRIC HAEMATOLOGY/ONCOLOGY
Anaemias, including SCA I & II Dr. Ezenwosu 109
Childhood leukaemias Dr Chukwu 121
Bleeding Disorders (vascular, clotting and platelet disorders) Prof Emodi 150
Hypertension and hypertensive emergencies Prof. Okafor 171
Childhood Lymphomas Prof. Ikefuna 201
Other Childhood malignancies Prof. Ikefuna 204

PAEDIATRIC NEUROLOGY
Seizure Disorders Prof. Ojinnaka 117
CNS Infections (meningitis, Encephalitis, Cerebral abscess etc) Dr NA Uwaezuoke 144
CNS tumours Dr Aronu 175
Neuro-developmental disorders I & II Dr. Aronu 207
Behavioural, Speech and language Disorders Prof Ojinnaka 221
Neuromuscular Junction disorders and Muscular dystrophies Dr Bisi-Onyemaechi 224
The Floppy Infant Dr N.A Uwaezuoke 241

PAEDIATRIC PULMUNOLOGY
Tuberculosis Dr. Muoneke 126
Respiratory disorders I & II Dr Chime 130
Pneumonias and Complications Dr J Eze 192
Obstructive Respiratory Disorders I & II Dr. Ayuk 230
Obstructive Respiratory Disorders I & II Dr. Ayuk 282

PAEDIATRIC NEPHROLOGY
Nephrotic Syndrome Dr Mbanefo 141
Acute Glomerulo-Nephritis Dr. S.N Uwaezuoke 197
Congenital disorders of the Urogenital system, enuresis, hypertension) Dr Odetunde 236
Renal Failure and Renal Replacement Therapy Prof Okafor 247
Urinary tract Infections and Obstructive uropathies Dr. Muoneke 268

PAEDIATRIC CARDIOLOGY
Acquired Heart Diseases Prof Obidike 147
Congenital Cardiac Diseases Dr Chinawa 278

PAEDIATRIC ENDOCRINOLOGY
Adrenal disorders (insufficiency, Excess, CAH), Overview of Gonadal dysgenesis Dr Chikani 156
Diabetes Mellitus and Insipidus Prof Adimora 212
DKA (risk factors, presentation and management) Dr Chikani 259

PAEDIATRIC GASTROENTEROLOGY

3
Compiled by Obasi. D. C. Chinedu

Micronutrient and Macronutrient Deficiencies Dr Eke 217

SPECIAL CONSIDERATIONS
Osteomyelitis Dr Adiele 280
Juvenile Idiopathic Arthritis (Juvenile rheumatoid Arthritis) Dr Arodiwe 285
Viral haemorrhagic fevers Dr Ujunwa 295
Helminthic and Other parasitic infections Dr Adiele 298
Allergic disorders Dr Eze 301
Child survival Strategies and SDGs Dr O. Iloh 309
Child abuse and neglect Dr Onukwuli 312
School Health Program Dr. Bisi-Onyemaechi 317
Review of Paediatrics Radiology I & II Radiation Medicine Dept. N/D

PERCENTILE CHART WHO 321


PAST QUESTIONS Various 336

4
Compiled by Obasi. D. C. Chinedu

BASIC PAEDIATRIC INFORMATION AND FORMULAS


This section was compiled myself, with information obtained from posting and some important books like: Rule of thumb, etc. They are also
contained in their various materials, but this chapter brings it out for easy accessibility. Pls verify each values again in case of typo errors.

GENERAL PAEDIATRICS/NEONATOLOGY
APGAR Score: scores are assigned at 1, 5, 10 mins and if further resuscitation, 15 and 20 mins.
1st Min score determines need for immediate resuscitation and not necessariliy survival.
5th Min score indicates responsiveness to therapy and can also identify babies at high risk for future problems in the immediate neonatal
period. It is predictive of survival. Low 1 min and high 5 min score indicates successful resuscitation. Low 10min Apgar score more
specific but less sensitive predictor of death or disability.
Score 0 1 2

Colour Pale Or Centrally Cyanosed Peripheral Cyanosis Completely Pink

Heart Rate Absent <100b/Min >100b/Min

Reflex Irritability Non Grimace Cry

Muscle Tone Flaccid Some Flexion Well Flexed

Respiration Absent Weak, Irregular Regular


Interpretation: Score > 7 -well
1 – 3@ 1min or < 5 @ 5 min is severe asphyxia and risk of sequelae is high: Delayed onset of spontaneous respiration. Signs of
multiorgan dysfunction. Clinical signs of encephalopathy may be present even with acceptable scores.
Score of 5 or less @ 10mins predicts death or major neurologic disability
Blood Transfusion Vol Calculation For EBT: 85mls x Wt of Child (Single Volume)
Prematurity: Preterm: <37 Wks. Term: 37-42 Wks. Post-term: >42 Wks. Post-date: >40wks.
Age of Viability: US: 20 wks, WHO: 22 wks, UK: 24 Wks, Nig: 28Wks.
Jaundice: Newborn bilirubin: >5mg/dl. Older Children bilirubin> 2mg/dl.

GASTROENTEROLOGY:
Energy: Term neonate require 80-90 Kcal/Kg/day. Preterm require higher.
100mls of Milk = 67Kcals of Energy (70Kcals when calculating for ease). Preterm infants require 180Kcal/Kg of energy Per day.
Weight for Age Calculation: 0-2 Months: (n-10) x 30g + BW (n= age in days)
3-12 Months: (n+9)/2 (n= age in months) 1-6 years: 2n+8 (n= age in years)
7-12 years: (7n-5)/2 (n=age in years).
Express wt in % of expected using percentile chart at the back. Small for GA: Wt less than 10 th centile. Large for GA: Wt greater than 90th
centile.
Term babies will double wt at 6 months, triple at 12 months & quadrapule at 2 years. Preterm babies will equalize with wt of term
babies by 24 months.
Height/Length for Age: At Birth 50 +/- 2cm. Infants gain 25cm in the first year of life,
1st 3 Month: 3.5cm per month. 2nd 3 Month: 2cm 3rd 3 Month: 1.5cm
4th 3 Month: 1.2cm. Subsequent Months: 0.8cm per month.
>1 year: Height: 6n+77 (n is age in years). Preterm length equalizes with term at 40 Months.
Note: Weight is used as a sign of Acute Malnutrition, while Length/Height is for Chronic.
Dehydration: Correction Fluid: Severe: 100mls/kg. Moderate: 75mls/kg
Maintainance Fluid Calculation: 1st 10kg: 100mls/kg. 2nd 10Kg: 50mls/kg. Subsequent Kg: 20mls/Kg.
Max Fluid for 24hrs: 2400mls or 100Mls/hr.
Ongoing loss: 10mls/Loose stool and 5mls/Vomitus.

CARDIOLOGY:
Normal Heart Rate for Age: Neonate-1yr: 110-160 Bpm. 2Yrs: <140 Bpm
4Yr: <120 Bpm. >6Yr: <100 Bpm
Infants <5yrs have abdominal breathing, hence, not expected to do chest excursion.

5
Compiled by Obasi. D. C. Chinedu

Hypertension: average SBP and/or diastolic BP (DBP) that is ≥95th percentile for gender, age, and height on 3 occasions.
Pre hypertension/elevated BP in children is defined as average BP >90th percentile but <95th centile.
A patient with BP levels >95th percentile in a physician ’s office or clinic, but normotensive outside a clinical setting has “white-
coat hypertension.”
Stage 1 HTN: BP is greater than the 95th percentile but less than the 95th centile plus 12mmHg, or 130/80 to 139/89mmHg
(whichever is lower).
Stage 2 HTN: BP is greater than the 95th percentile plus 12mmHg or ≥ 140/90 mmHg.
If the systolic and diastolic pressures give rise to a discrepancy with respect to classification, the child’s condition should be categorized
by using the higher value.

PULMUNOLOGY:
Respiratory Rates for Age: <2 Months: <60 Cpm 2-12 Months: <50 Cpm
1-5Yrs: <40 Cpm 5-18 Yrs: <30 Cpm >18 Yrs: <20 Cpm (15-20)
Signs of Resp Distress: Presence of Intra-nasal O2. Flaring of Alar Nasi, Subcoastal and Intercoastal Recession, Mouth Breathing, Tracheal
Tugging.
Chest Circumference: At birth: 2-4cm<OFC. At 1 Yr: equal with OFC. >1yr: CC>OFC

NEUROLOGY:
OFC For Age: At Birth: 35 +/- 2. 1st 3 Months: gains 2cm per month
2nd 3 Months: 1cm Per Month Next 6 Months: 0.5cm Per month
Next 2 years: 2cm Per year.

HAEMATOLOGY/ONCOLOGY:
Infusion Giving Sets drop rates: Soluset: 60 drops in 1 ml. Normal drip: 20 drops in 1 Ml
Blood giving set: 15 drops in 1ml.
Blood Transfusion: Packed Cells: 10ml/kg. Sedimented Cell: 15ml/Kg. Whole blood: 20ml/Kg
Paediatric Blood Bag: 250mls

NEPHROLOGY
Normal Urine Flow rate:> 0.5mls/kg/hour

ENDOCRINOLOGY
Hypoglycemia: <2.2mmol/L or ,40Mg/dl.

CHER:
ORS constituted with 1L of water.

Read Up Emergency topics, Mgt as well as Common drugs given at UNTH and dosages.

6
Compiled by Obasi. D. C. Chinedu

INTRODUCTORY PAEDIATRICS: HISTORY TAKING AND PHYSICAL EXAMINATION


Prof. Herbert A Obu, MBBS, FWACP

DEFINITION OF PAEDIATRICS
Pediatrics is the branch of medicine that deals with the health and medical care of infants, children, and adolescents
from birth up to the age of 18.
Derived from two Greek words: (pais.= child) and (iatros = doctor or healer). It is a relatively new medical specialty, developing
only in the mid-19th century and Abraham Jacobi (1830–1919) is known as the father of paediatrics.
It is dedicated to all aspects of the well-being of children, from infancy through adolescence, including their health, their
physical, mental and psychologic growth and development; and their opportunity to achieve full potential as adults.
Paediatrics aims to reduce infant and child rate of deaths, control the spread of infectious disease, promote healthy lifestyles for
a long disease-free life and help ease the problems of children and adolescents with chronic conditions.
It incorporates health promotion and protection and prevention of illness and disease.
Paediatricians are doctors who specialise in paediatrics. Their entire purpose is to advance the well-being of children. In addition
to catering for the health needs of children, they also serve as advocates for children, irrespective of culture, religion, gender,
race, ethnicity, or of local, state, or national boundaries.

HISTORY TAKING IN PAEDIATRICS


History taking in medicine is an art that is acquired through training and practice, and it requires life-long learning.
Proficiency in this art improves with experience and it is important that one acquires the habit or art correctly from the outset.
This lecture aims to help you get it right from the beginning.
The first step is to set the stage. The environment, be it a clinic, etc. should be conducive and as child-friendly as possible.
Try to win the confidence of the child/caregiver so that they can trust you and divulge personal and sensitive information to you.
Confidentiality must be maintained.
The language of communication must be as simple as possible; avoid verbosity/medical jargons.
 Note the informant - Biodata
 Chief complaints - History of chief complaints
 Past medical history - Drug/allergy history
 Pre-natal, natal and post-natal history
 Dietary history - Immunization history
 Developmental milestones - Family and Social history
 Review of Systems - Summary of the history.

PHYSICAL EXAMINATION
 General examination
 Anthropometry
 Examination of the Systems.

RECOMMENDED TEXTBOOKS
 Textbook of Paediatrics by Hugh Jolly
 Clinical methods in Paediatrics by EO Obidike
 Neurological Emergencies by SO Iloeje

OTHER MATERIALS
 Stethoscope
 Pen torch
 Tape measure
 Tendon hammer
 Diagnostic set

Thank you for your attention.

7
Compiled by Obasi. D. C. Chinedu

DIARRHEAL DISEASES AND FLUID & ELECTROLYTE BALANCE I &II


Dr Ughasoro Maduka D.

GASTROENTERITIS
Stool (Diarrhea) = Loose and Increase in frequency (≥ 3 loose stool/ 24 hours)

Enteritis
 Wide variety of enteropathogens
 Bacteria, Viruses, and parasites
Clinical manifestations depends on the organism & host response to infection
 Asymptomatic infection
 Watery diarrhea
 Bloody diarrhea (Dysentery)
 Damage of tissue and toxaemia
 Shigella, campylobacter jejuni, E.Coli (enteroinvasive), Salmonella

Acute or Chronic
Acute (< 14 days)
Persistent (>14 days)
 (excluding chronic diarrhea due to lactase deficiency, Coeliac disease)
 Shigella Flexneri, G.lamblia, strongloides stercoralis,
 Cryptosporidium, Isosporabelli, E.Histolytica, HIV enteropathy etc

Chronic/Intractable = > 4 weeks


 Coeliac diseases (Gluten-sensitive enteropathy)
 Toddler’s diarrhea (Chronic non-specific diarrhea of childhood)

Severity
 Stooling rate (>10ml/kg/stool or 5ml/kg/vomitus)
 Hydration status (dehydrated)
 Serum electrolyte (orientation, ileus)

EPIDEMIOLOGY
Afflict millions of children.
> 30% of paediatric outpatient visits. Some (%) may have infectious agents***.
80% of diarrhea mortality due to dehydration.

Predisposing factors:
Unhygienic practices (socioeconomic)
 Bottle feeding/poor weaning practices,
 poor sanitation (feacal management, overcrowding), &
poor immunity (Host)
 Malnutrition, HIV, prolonged steroid therapy,
Harmful practices
 Excessive antibiotic usage (clindamycin, etc)

PATHOPHYSIOLOGY (INFECTIVE) AND SYMPTOMS


Non-infectious
 Osmotic
Infectious (osmotic and, or secretory)
Viruses. Rotavirus. (common cause of infantile gastroenteritis)
 Infects matured enterocytes of villus tip of small intestine = induce watery diarrhea
 Absence of histological changes
 Decreases the activities of enzymes at the apical brush boarder membrane, inhibits Na+ solute cotransport systems.
 NSP4 enterotoxin (hypothesis of an additional secretary component

8
Compiled by Obasi. D. C. Chinedu

E.Coli (enterogenic) = (Secretory) = adenyl cyclase = increase intracellular cAMP = inhibit the chloride-linked neutral sodium –
absorption from the lumen = Na+ wasting.
E.Coli (Enteropathogenic) = (Osmotic) tight adherent to mucosa = effacement of microvilli. = reduce absorptive surfaces.
E.Coli (Enteroinvasive) = as above with bloody stool and systemic toxicity
E.Coli (Enterohaemorrhagic) = complicated with HUS
 O15.H7 toxin
 Fluid and rest
 Antibiotic increase the risk of complication.
 Anti-diarrhea medications slows the digestive system down prevent the body from getting rid of toxins.
 Complicated (HUS) hospitalized (supportive care =IV fluids, blood transfusion and kidney dialysis)

Shigella (shigellosis, bacillary dysentery, G-) invade (intracellular) the epithelial lining of the ileum, colon and rectum
 abdominal pain, tenesmus, blood, mucoid stool, straining.
 Beta-lactams, Quinolones, macrolides.

Vibro Cholera O1 and O139 = secretory


 O1 = Classical and El-tor biotypes
 Three serosubtypes – Ogawa, Inaba, & Hikojima
 O139 similar to O1 but differs polysaccharide surface antigen
 Diagnosis is through isolation of V.c from stool.
 Gray stool with flecks of mucus = rice water stool
 Fluid rehydration
 Beta-lactam,

Differentiation of osmotic and secretory diarrhea by stool carbohydrate and osmolar gap measurement
 Faecal Osmolar Gap (FOG) = serum osmolarity – [2 x(feacal Na+ K concentration)]
 FOG (Higher osmotic (>100mosml/l); secretory lower <100mosml/l)
 Steatorrhoea (by Sudan III test) more is osmotic diarrhea
 Higher acid faecal pH more in osmotic diarrhea
 Reducing substances (give electron & become oxidized(benedict’s test and glucose strip) positive in osmotic diarrhea
 Reducing sugar heated in the presence of an alkali converts to a more powerful reducing species enediols. Which reduce
the cupric compound (Cu2+) present in the benedict reagent to cuprous compounds (Cu+) which precipitate as insoluble
red copper(1) oxide Cu2O).
 Semi-quantitative test the colour gives idea of the quantity of sugar [Blue to brick-red]
 Greenish = 0.5g%, Yellow = 1g%, Orange = 1.5g%, Red = 2g%
Faecal osmolarity (about the same for both)

TBW = Total Body Fluid & % Deficit in body weight


Higher body content of water/unit body weight (50% - 75%)
ICF (33 – 40%) & ECF (20% - 45%)
First line of fluid loss is ECF
1kg of water = 1000mls of water. 1 kg of the BW can be translated in volumetric terms as 1000 ml of fluid volume
Higher BMR (awake & active)
Moderated dehydrate (10% BW deficit) child weighed 5.3kg at admission the pre-illness weight in Kg .

ASSESSMENT OF LEVEL OF DEHYDRATION


No Sign Mild < 5%TBW Moderate >5% - Severe >10%TBW
<10%TBW

General Condition Well & Alert Restless/ Lethargic Unconscious


Irritable

Eyes Normal Sunken Sunken Sunken

Tears Present Nil Nil Nil

Mouth Moist Dry Dry Very Dry

9
Compiled by Obasi. D. C. Chinedu

Skin Retracts Retracts Slowly Retracts very Retracts very


Immediately slowly slowly (>2
secs)

% Age Body <5% >5% - < 10% >10%


Weight Loss

MANAGEMENT
Correct
Deficit (dehydration)
 50 – 100ml/kg over 4-6 hours
 100ml/kg (isotonic crystalloid (NS, or RL) severe dehydration (monitor vital signs). But 1 st 20ml/kg and reassess.
(withhold K+ urine makes urine)
 75ml/kg (moderate) 10ml/kg (mild for ongoing losses),
Replace ongoing loss/prevention of dehydration (immediately)
 Estimate the volume loss (10ml/kg = stool, 5ml/kg = vomiting)
Ensure daily maintenance
 100ml/kg = 1st 10kg; 50ml/kg = 2nd 10kg; 20ml/kg = 3rd 10kg
What is the maintenance fluid requirement for 17kg child?
Replace loss electrolytes; Maintain adequate nutrient intake (continue feeding)

HYPEROSMOLAR DEHYDRATION
Hypertonic dehydration
 Replace the 50% - 60% fluid deficit, maintenance & ongoing losses in the within 12 -24 hours
 The remaining volume of fluid spaced over the remnant 24hours
 To avoid fluid shift into the brain = water intoxication and seizure

Fluid Management In Severely Malnourished Children Under 5 Years Of Age Without Shock
Severe Acute Malnutrition (SAM)
 Very low weight-for-height/ weight-for-length
 Signs of bilateral pitting oedema
 Very low mid-upper-arm circumference (6 – 59months of age)
SAM patients have difficulty keeping themselves properly hydrated. Difficult to assess their hydration status.
Fluid management is complex especially with diarrhea
Rehydrate slowly (oral or NG) 5-10ml/kg/h up to 12 hours (except in cholera or profuse watery diarrhea) Give standard WHO low
osmolarity ORS
ReSoMal or half strength standard WHO low osmolarity ORS with added K+, glucose

Standard ORS vs Hyposomolar ORS


A major criticism of the use of oral rehydration solution (ORS) in the treatment of diarrhea has been that the high sodium content of the
solution might predispose the development of hypernatremia in infants not allowed access to additional free water and in whom
glomerular filtration rate and urinary concentrating ability may be low. Error use of ORS and failure to use water.
Using low-sodium, low-glucose ORS formulation reduces the need for IV (33%). Reduce incidence of vomiting (30%)

10
Compiled by Obasi. D. C. Chinedu

Electrolyte Composition of stool (Mmol/L)


Aetiology Na (+) K (-) Cl (-) HCO3 Osmolality

Normal Serum

Cholera 88 30 86 23 300

Rotavirus 37 38 22 6 300

ETEC 50 - 53 37 24 18 300

WHO/UNICEF 90 (NaCl = 20 (KCl = 80 30 (Trisodium 330 Glucose (117)


ORS(Standard) 3.5g/l) 1.5g/dl) citrate 20g
Isotonic NaHCo3 = dihydrate =
2.5g/l 2.9g/l) (10 =
citrate)

WHO/UNICEF 75 mEq/L 20 (KCl = 65 2.9g/l (10 = 245 mOsm/L 75 mmol/L


ORS (Hypo- (NaCl = 1.5g/l citrate) 13.5g
osmolar) 2.6g/l

SSS (600ml of I teaspoon 10 level


water) (3ml) teaspoon
salt = 1.8 or 5 cubes
– 2.4g of sugar
(25g)

Ringers Lactate 130 4 109 28 (lactate) Ca++ = 3


****

Normal Saline 154 154


(0.9%)

Once not in shock and can tolerate oral, ORS is as effective as IV therapy in rehydration.
Antibiotic:? Not indicated routinely except evidence of systemic infection
Antidiaarrhoeal drugs: Loperamide, diphenoxylate (no proven efficacy)
Attribute of ORS
11
Compiled by Obasi. D. C. Chinedu

Isotonic with plasma;


[K+] > Plasma
[HCo3] = 30 – 48 mmol/L
Optimal Na+ Glucose
Palatable
Cheap
Easy to use. (PHC)

Barriers to the use of ORS


 Withhold of ORT in children with vomiting
 Withhold of ORT in children with moderate dehydration
 Fail to advise prompt feeding
 Failure to use spoon or syringe

Prevention
Vaccine
 Rotavrus, Cholera, Shigella, E.Coli
Pre- & Pro- biotics
Zinc supplement
 10mg – 20mg daily 10 – 14 days (younger than 6 months and older than 6 months)
Vitamin A
Sanitation

Oral Zinc Supplementation (Sulphate, Acetate, Gluconate


Effect on the length/ duration of diarrhea
Reduce severity of diarrhea via:
1. Restoration of mucosal barriers
2. Enterocyte brush-border enzyme activity
3. Improves the absorption of water
4. Promotes the production of antibodies and circulating lymphocytes against intestinal pathogens
5. Has direct effect on ion channels
o Act in as K+ channel blocker of adenosine 3-5 cyclic monophosphate mediated chlorine secretion.
6. Reduces the risk of reoccurrence of diarrhea ensuing 2 – 3 months
Adversely it increases the risk of vomiting

Probiotics in children with acute diarrhoea


Probiotics are bacterial and yeasts that are similar to the normal bacterial found in the gut.
Friendly bacterial:
 Lactobaccilus GG or reuteri
 Bifidobacterium factis
 Streptococcus thermophiles
 Saccharomyces boulardii
Prebiotics are Microbial cell preparations or components of microbial cells that have a beneficial effects on the health and well being of the
host by Prevention of antibacterial-associated diarrhoea (AAD)

Fluid & Electrolyte Therapy


Osmolarity = Water + osmotically active solute
Cell membrane is permeable to water but not freely permeable to solutes
ECF = Na+, Cl-, HCO3-, glucose & Urea
ICF = K+ & proteins
Osmolarity of ICF = ECF = 280 - 295 mOsm/kg

Control
 Thirst; ADH; Renin – Angiotensin – Aldosterone pathway
Affected
 Gastroentritis; DM; DI; Pyloric stenosis
SIADH
 Hyponatraemia; No dehydrated; No edema; Concentrated urine

Maintenance Fluid = zero water & electrolyte balance


Water loss = insensible + Urine

12
Compiled by Obasi. D. C. Chinedu

1500 – 2000 ml/m2/day & 150 mls fluid/100calore expended


Maintenance electrolytes
 Na+ = 2 – 3mmol/kg/day
 K+ = 1 – 3 mmol/kg/day
 Cl- = 3-5 mmol/kg/days
 Ca2+ = 50 -200mg/kg/day

Na+
Isotonic = [Na+] 130 – 150 mmol/l (80%)
 diarrhoea
Hypotonic = [Na+] < 130 mmol/l (15%)
 Renal causes, burns, pancreatitis, peritomitis,
Hypertonic = [Na+] > 150 mmol/l (5%)
 Diarrhoea + poorly econstiituted ORS

K+
Hyperkalamia
 Renal failure; Muscle weakness, ECG cnages, T wave tail,
Hypokalamia
 Muscle weakness, depressed S-T segment, flattened T waves, U waves, Prolonged Q-Uinterval

Ca++
Hypercalamia
 Vomiting, constipation, polyuria, polydypsia
Hypocalamia
 Hyperexcitability, spasms,seizures, Chvosteks & trousseau’s signs

Acid-Base Balance
[H+] (7.35 – 7.45) : Negative log of [H+}: inverse relationship with [H+}
Close regulation of pH is necessary for cellular enzymes & metabolic processes.
Controlled by 3 mechanisms
 Buffers
1. Intracellular = proteins
2. Extracellular carbonic acid equilibrium { CO2 + H2O = H2CO3 = H+ & HCO3}
 Respiratory (Rapid control)
 Renal (slow)
Acidosis = base or gain in buffer acid
Alkalosis = buffer acid or gain in base.
Any disturbance that is associated with PCO2 is respiratory
Any disturbance that is associated with HCO3- is metabolic
Acid = donates [H+], Base accept [H+}
Buffer = attenuates the change in pH due to addition of acid or base.
 CO2 + H2O = H+ + HCO3-
 [H+] = 24 x Pco/[HCO3-]
Normal values: [HCO3-] = 20 -28mEq/L;
Pco2 = 35 – 45 mmHg
Metabolic Acidosis = Pco2 = 1.5 x[HCO3-] + 8 ± 2
Metabolic Alkalosis = Pco2 increases by 7mm Hg for each 10mEq/L increase in serum [HCO3-]

Respiratory acidosis =
 [HCO3-] increases by 1 for each 10mm Hg increase in the Pco2 (Acute)
 [HCO3-] increases by 3.5 for each 10mm Hg increase in the Pco2 (Chronic)
Respiratory Alkalosis =
 [HCO3-] falls by 2 for each 10mm Hg decrease in the Pco2 (Acute)
 [HCO3-] falls by 4 for each 10mm Hg decrease in the Pco2 (Chronic)

Metabolic Acidosis (HCO3 < 20mmol/l)


Weakness, nausea, vomiting, & deep rapid respiration
DKA, Starvation, salicylate poisoning, renal tubular acidosis
Fall in PH ( low HCO3) = peripheral and central chemoreceptors = respiratory center = increase CO2 exhalation
13
Compiled by Obasi. D. C. Chinedu

Low PCO2 will reduce ventilation drive. Finite repair is via kidney (reduce HCO3 excretion and Increased H+ excretion).
With adequate hydration, MA tends to correct spontaneously.
Administration of IV NaHCO3 1 – 2 mmol/kg of 8.4% (double dilution)
 Requires adequate ventilation.
0.3 x weight (kg) x base deficit = mmol of HCO3 required
Otherwise (hypernatraemia or Respiratory acidosis) trihydroxymethyl amino-methane (THAM) can be used.
 Weight(kg) x base deficit (mmol/L) x 1.1 given over one hour under ECG monitor.

Anion Gap
[Na+] – [Cl] – [HCO3-] = 8 -16
Anions = cations = electrical neutrality
Difference between unmeasured cations (K+, Mg2+, Ca2+) and unmeasured anions (albumin, phosphate, urate, sulfate)

Metabolic Alkalosis(HCO3 > 25mmol/L). Base excess


Pyloric stenosis
Diuretic therapy
K deficiency conditions
 H+ moves intracellularly to replace K+
 Lost in urine due to distal tubular hydrogen ion secretion in exchange for reabsorbed Na+
Correction of hydration with fluids that contain adequate amount of sodium chloride + KCl (3-6 mmol of K /Kg/24 hours).
 So that the distal tubules can secrete K+ and conserve H+ in exchange for reabsorbed Na+

Respiratory Acidosis & Alkalosis


PCO2 > 45 mm/Hg (acidosis) < 35mmHg (Alkalosis)
Inadequate Pulmonary ventilation (carbonic acid accumulates or blow-off of CO2).

Hypokalaemia (<2.5mEq/L)
Reduce K losses
Discontinue diuretics
Treat diarrhea or vomiting
Administer H2 blockers if patient is on NG
Control hyperglycemia if glycosuria is present
For every 1mEq/l decrease in serum potassium the potassium deficit is approximately 200 – 400mEq
<Mild (2.5 – 3.5 mEq/l) often asymptomatic and require oral potassium replacement
2.5mEq/L give IV potassium 10mEq/h via small vein (can give up to 40mEq/h via central line)

HyperKalaemia
5.5 – 6.0 mEq/L = Mild
6.1 – 7.0 mEq/L = Moderate
7.0mEq/l or greater = severe
Fatigue/weakness - Paresthesia/paralysis - Palpitation/Marked QRS widening
Bradycardia, reduced DTR, - Reduce source of potassium - IV bicarbonate
Calcium gluconate (magnesium sulfate if digoxin toxicity is suspected), reduces risk of ventricular fibrillation
Insulin given together with 50% dextrose (shift K into the cell)
Beta 2-adrenergic agonist (promote cell uptake of K
Cation exchange resin promote exchange of K for Na in the gut
Discontinue any potassium sparing diuretic - Diuretics causes potassium loss via kidney
Magnesium sulphate - Dialysis

14
Compiled by Obasi. D. C. Chinedu

HIV/AIDS
Prof. IFEOMA EMODI

HIV was first reported in 1981 among homosexual men in LA. HIV is a slow growing retrovirus. Two strains have been identified HIV 1 &
2.
HIV initiates in the infected cell the production of the enzyme reverse transcriptase. This enzyme transcribes viral RNA into double helical
DNA. The viral DNA so formed remains dormant and merged with the nuclear DNA of the host cell and is transmitted to other cell lines or
transcribed back to viral RNA and leaves the cell as viral particles.
It can infect other cells including nerve cells and B lymphocytes

Epidemiology: 36.7m living with HIV/AIDS at end of 2016 including 2.1m children. Prevalence rates reach 60% in some countries with the
majority of those infected aged 15-45 yrs. An estimated 14.1m African children have lost one or both parents to AIDS by 2008. An estimated
78m have been infected since epidemic began with 35m deaths. Vast majority of ppl located in L and M income countries. East and Southern
Africa saw 44% of new HIV infections in 2016. Young women more at risk with 59% of new infections. (15-24 yrs)

Global Perspective of HIV/AIDS

Nigeria’s Contribution to the Global HIV/AIDS Burden


World Sub Saharan Africa Nigeria ( 2015 statistics)

No of adults & Children 36.7-45.3m 23.8-28.9m 3.2m


living with HIV (40.3m) (25.8m) (2.3-4.3m)

New Infections 2.5 million 1.7million 0.22m


In 2016 (one out of every thirteen
occurs in Nigeria)

Deaths 1.7 million 1.2 million 0.16m


(one out of every fourteen
occurs in Nigeria)

Source: amfAR

GLOBAL SUMMARY OF THE AIDS EPIDEMIC 2015


No of ppl living with hiv: - Total 36.7m ( 34- 39.8) - Children <15yrs 1.8m (1.5-2.0)
No of ppl newly infected: - total 2.1m (1.8-2.4) - Children <15yrs 150k (110-190)
AIDS death in 2015: - Total 1.1m (0.9-1.3) - Children <15yrs 110k (84-130)
HIV Statistics In Nigeria

15
Compiled by Obasi. D. C. Chinedu

Be nue 10.6

Nas ar aw a 10.0
FCT 9.9
Ak w a Ibom 9.7

Cr os s Rive r 8.0

Rive r s 7.3
Baye ls a 7.2

Kaduna 7.0
Adam aw a 6.8
Nige r 6.2

Sok oto 6.0

Enugu 5.8
Anam br a 5.6
Edo 5.2
Tar aba 5.2
Lagos 5.1

Kogi 5.1
Abia 5.0
te

Im o 4.6
ta
S

Gom be 4.0

De lt a 3.7
Bauchi 3.1

Ke bbi 2.9
Ebonyi 2.8
Yobe 2.7

Plat e au 2.6
Kats ina 2.6

Ondo 2.4

Kano 2.2
Oyo 2.2
Zam far a 2.1

Bor no 2.0
Kw ar a 1.8

Ogun 1.7
Jigaw a 1.6
Os un 1.2
Ek iti 1.0

0.0 2.0 4.0 6.0 8.0 10.0 12.0

Prevalence (% )

In 2004 prevalence rates for Cross River was 12.5% and Benue 10% while it was 1.8% and 1.4% for Ogun and Osun respectively. In 2015
prevalence rates are Rivers 15.2% and Taraba 10.5% while Zamfara had 0.4% and Ekiti 0.2%
The precarious health care system has also been stretched to it’s limit. Of the 2.1m children under 15yrs living HIV worldwide 90% live in
Subsaharan Africa. It is currently estimated that in developing countries 1,600 children are infected daily.
The region also accounted for 72% of the world’s AIDS related deaths in 2008.

In Nigeria 1st case reported in 1986. We have the 2nd highest number in Africa and highest in W Africa, Females account for > 50% of
cases. Very high prevalence among youth. The hardest hit states are Rivers, Taraba, Kaduna, Nasarawa, FCT.

Maternal ill health esp HIV related has a negative effect on infant survival.
Modes of transmission include MTCT (95%), Tx of blood or bld products, sexual abuse, un sterile procedures, and scarification. MTCT can
occur during pregnancy, at time of delivery, or during breastfeeding

Risk factors for MTCT include maternal and infant factors.


 Maternal factors include women with a high viral load, sever immunosupression, micronutrient deficiencies, PROM (>4hrs),
cracked nipples, breast abscesses, HIV 1.
 Infant factors include prematurity, breast feeding, oral thrush and ulcers, invasive fetal monitoring, birth order in twin pregnancy,
vaginal delivery.

Preventing paediatric HIV infection involves


 Primary prevention of HIV in men & women
 Prevention of unintended pregnancy in HIV infected women
 PMTCT
 Provision of care and support for HIV infected women their infants and families

Diagnosis of paediatric HIV


High index of suspicion needed. The diagnosis may be clinical or a combination of clinical and laboratory supported. HIV presents with
conditions that are frequently found in children who are not HIV infected making it difficult to make a clinical diagnosis.

Signs/conditions very specific to HIV: pneumocystis jiroveci pneumonia (PCP/PJP), oesophageal candidiasis, extrapulmonary
cryptococcosis,Invasive salmonella infection, lymphoid interstitial pneumonia, herpes zoster, kaposi’s sarcoma, progressive multifocal
encephalopathy

Signs/conditions common in HIV and uncommon in uninfected children: Severe bacterial infections esp if rec, persistent or rec oral
thrush, bilateral painless parotid enlargement, generalised persistent non inguinal lymphadenopathy, hepatosplenomegally, persistent and/or
recurrent fever, neurologic dysfunction, herpes zoster, persistent gen dermatitis unresponsive to treatment.
Signs/condition common in all ill children
Chronic recurrent otitis with ear discharge

16
Compiled by Obasi. D. C. Chinedu

Sever pneumonia, TB, bronchiectasis, failure to thrive, marasmus


Laboratory tests:
 Antibody tests include HIV ELISA, rapid tests and Western Blot
 Virologic tests include PCR, viral load, p24 assays

Diagnosis of HIV
Passive transfer of antibodies across the placenta means that HIV exposed babies will have circulating antibodies till 18 mths. Breastfeeding
further complicates matters
Suspect HIV in children with above conditions and determine the clinical stage

1986 WHO case definition of AIDS


Major signs:
 Weight loss or abnormally slow growth
 Chronic diarrhoea (> 1 mth)
 Prolonged fever (>1mth)
 Sever or recurrent pneumonia

Minor signs:
 Generalised lymph node enlargement
 Oropharygeal candidiasis
 Recurrent common infections (O M)
 Persistent cough in absence of TB
 Generalised rash
 Maternal HIV infection
AIDS is defined as the presence of at least 2 or more major signs and 2 or more minor signs if there is no other cause of immune
suppression

WHO paediatric staging of HIV/AIDS 2004


 Stage 1: asymptomatic, PGL hepatospleenomegaly
 Stage 2: papular pruritic eruptions, seborrheic dermatitis, angular chelitis, lineal gingival erythema, extensive molluscum
contagiosum >5% of SA, rec oral ulcerations >2/6mths, parotid enlargement, herpes zoster > 1/12mths, recurrent or chronic URI
otitis media, otorrhea, sinusitis >2 / 6mths

 Stage 3: unexplained mod malnutrition, unexplained persistent diarrhea, unexp persistent fever (int or cont >1mth), oral candidiasis
outside NN period, oral hairy leucoplakia, PTB, sever rec presumed bacterial pneumonia >2/12mths, acute necrotising ulcerative
gingivitis/periodonitis LIP, unexplained anaemia (Hb< 8gm/dl), or thrombocytopenia <30,000/mm3 for >1mth, HIV related
cardiomyopathy or nephropathy

 Stage 4 (age ,<18mths) : 2 or more of oral candidiasis, sever pneumonia, FTT, sepsis. Requires confirmation with HIV virologic
tests or antibody tests after age 18 mths
 (any age): unexplained sever wasting or sev malnutrition unresponsive, PCP, rec sever bacterial infection (> 2 / 12mths excl
pneumonia) chronic orolabial or cutaneous HSV (>1mth), extra pulmonary TB, KS, eosophageal candidiasis, CNS toxoplasmosis,
cryptococcal meningitis

Any disseminated endemic mycosis, cryptosporidiosis, or isosporiasis (diarrhoea >1mth), CMV, disseminated mycobacterial dis other than
TB, candida of trachea, bronchi or lungs, acquired recto- vesico fistula, cerebral or B cell lymphoma, progressive multifocal
leukoencephalopathy, HIV encephalopathy

See images of: Severe HIV wasting, Oral cheilitis, Acute Herpetic gingivostomatitis, Oropharyngeal candidiasis, Pseudomembranous
candidiasis, Severe Molluscum contagiosum, Severe warts, Herpes Zoster, Norwegian scabies, Flat facial warts [Epidermodysplasia
verruciformis], Herpes simplex, Chronic parotitis, Extensive cutaneous Kaposi sarcoma in the material.

PMTCT
This includes the use of ARV drugs, safer delivery practices, infant feeding (BMS, EBF) counseling and care and support
ARV drugs: pregnancy was an indication for HAART but now the policy is test & treat. The baby receives NVP for six weeks. Could
receive NVP and AZT if risk is high for 12 weeks

A mother receiving ARV for her own disease could:


 EBF from birth, Introduce complementary food at 6mths
 Continue BF till 12mths
17
Compiled by Obasi. D. C. Chinedu

 Infant receives NVP till 6 weeks only

Mother who chooses not to BF


 Exclusive Formula Feed from birth
 Introduce complimentary feed at 6mths
 Infant receives NVP for 6 wks

If mother is on HAART for her own disease, the use of EFV and ABC in first trimester not recommended.
Safe delivery practices include absence of artificial rupture of membranes, foetal scalp monitoring, vacuum extraction, episiotomy, elective
CS. Where HAART is used for PMTCT, CS has no additional effect.

Infant feeding: breast feeding increases the risk of MTCT by 10 – 45 % depending on duration and feeding method ie EBF or mixed. Lack of
BF increases the child’s risk of malnutrition and infection with other orgs

Exclusive breastfeeding by HIV positive mothers is recommended for the first 6 months of life with introduction of appropriate
complementary foods thereafter, and continued breastfeeding for the first 12 months of life. A nutritionally adequate and safe diet with-
out breast milk should be provided before stopping breastfeeding.

PCP prophylaxis
CTZ significantly reduces the severity and incidence of PCP. Any child with a history of PCP should continue dly CTZ for life (sec
prophylaxis). Dose is 5mg/kg of the TMP alternate drugs include dapsone 2mg/kg/day or pentamidine 4mg/kg/dose every 4 – 6 wks or
atovaquone

Criteria for CPT


 All infants delivered to HIV positive mothers, from 6 weeks of age or at first contact if later until the HIV status is known
 HIV-infected infants and children regardless of CD4+ % or clinical symptoms. Continue prophylaxis until they are 5 years of age
regardless of symptoms, CD4+ % or good immune response to ART
 Adolescents and children ≥ 5 years with CD4+ count < 350 or Stage 3 or 4 regardless of CD4+ cell count
 Where CD4+ count is not available, WHO Stages 2, 3 and 4.

IMMUNIZATION
Studies indicate impaired passive transfer of antibodies and response following vacntn. Administer immunisation according to NPI.
When considering BCG at later age exclude symptomatic HIV disease. Do not give YF to symptomatic children.
Measles vaccine can be given even if symptomatic at 6 and 9 mths

HIV encephalopathy
HIV is neurotropic, 40 – 70 % of HIV infected persons dev symptomatic neurological disturbances. The brain is most commonly affected in
children. HIV encephalopathy has been reported in about 21% of African children. The diagnosis is clinical and can be made in the presence
of at least 2 of the following for at least 2 months
 Failure to attain or loss of dev milestones or loss of intellectual ability
 Impaired brain growth or acquired microcephaly
 Acquired symmetrical motor deficit manifested by 2 or more of the following: paresis, pathologic reflexes, ataxia, or gait
disturbances
 CSF is normal/non specific, CT shows diffuse brain atrophy

Pneumocystis pneumonia
Caused by a fungus called pneumocystis jiroveci. It is a major cause of sev pneumonia and death in infants. Incidence is higher in 1 st yr of
life, peaking at 3 – 6 mths
Clinical features include: low grade or absent fever, marked resp distress, clear chest or fine creps, poor response to std antibiotic treatment,
severe persistent hypoxia (paO2 <90%)
Investigations include: sputum induction or broncho-alveolar lavage for fluorescent microscopy. There is no definite CXRay feature
Mgt includes: O2 therapy, maintenance of hydration, pcm for pain, IV high dose CTZ at 20mg/kg of TMP /day every 6 hrs for 21 days. Add
prednisolone 2mg/kg/day for 7 – 14 days if child is in severe resp distress , cont antibiotics for bacterial pneumonia

TB and HIV co-infection


The pandemic has led to a resurgence of TB in both adults and children. There is a higher case fatality rate.
Diagnosing TB in children was difficult even before HIV and is more difficult now as they may have other lung conditions and HIV-related
chronic lung diseases that might mimic the symptoms of TB

Symptoms/signs include:
 unexplained wt loss, failure to grow, fever (>14days), chronic cough >30 days
 Failure to respond to normal antibiotics
18
Compiled by Obasi. D. C. Chinedu

 Exposure to adult with proven/probable TB


 Plural effusion, enlarged tender C lymph nodes,
 Signs in other systems – CNS, bones, abdomen

Investigations include: ZN stain of specimens – gastric asp, ascites, CSF, CXRay, mantoux test (>5mm positive)

In most instances treatment is presumptive b/s it is difficult to make a diagnosis. National guidelines are used
It is sometimes better to treat TB completely before starting HAART or delay for 2mths. Rifampicin interacts with PIs

Lymphoid Interstitial Pneumonitis


This is common in children > 2yrs. It is often mistaken for miliary TB. Possible explanation include a co-infection of lungs by HIV and
EBV.
Diagnosis is by exclusion but pts usually are in good clinical condition despite resp distress, recurrent cough and dyspnoea, parotid
enlargement gen lymphadenopathy and hepatosplenomegally, finger clubbing, poor response to TB therapy

Radiological features include: diffuse bilat reticulonodular infiltrates similar to miliary TB, Bilateral hilar or mediastinal lymph nodes.
Mgt includes steroids, O2 therapy, antibiotics, bronchodilators, chest physio,

ART
The decision to start ART in children is complex.. Depends on a reliable care giver who understands the disease.
Any child who is HIV positive is to be started on ARTs asap, Drugs used depend on national guidelines

ART Regimens
Adolescents 10-19yrs
 TDF+3TC/FTC+EFV
3yrs to <10yrs
 ABC+3TC+EF
<3yrs
 ABC+3TC+LPV/r OR AZT +3TC+LPV/r

19
Compiled by Obasi. D. C. Chinedu

INFANT AND YOUNG CHILD FEEDING (IYCF)


Dr Ughasoro Maduka D.- Breastfeeding, Complementary feeding and Feeding in difficult circumstances.

Breastfeeding
Exclusive breastfeeding (EBF); 0 -6 months: Receives only breast milk from the mother or a wet nurse, or expressed breast milk, and no
other liquids or solids except drops or syrups consisting of vitamins, mineral supplements or medicines. Breastfeeding carries significant
health benefits & essential child survival intervention. EBF is less than 33% in Sub=Saharan Africa

Breast milk & feeding a Newborn: Mature human milk contains; 3%--5% fat, 0.8%--0.9% protein, 6.9%--7.2% carbohydrate calculated as
lactose, 0.2% mineral constituents expressed as ash. Its energy content is 60--75 kcal/100 ml.
Protein content is markedly higher and carbohydrate content lower in colostrum than in mature milk. No consistent compositional difference
between milks from the two breasts unless one is infected.

Protein: The principal proteins of human milk are a casein-Whey (20-80)


casein= slow digesting protein and anti-catabolic reduce muscle breakdown
Whey = fast digesting protein, stimulates protein synthesis, ideal recovery meal but lacks anti-catabolic property.
Many enzymes and several "minor" proteins also occur. The essential amino acid pattern of human milk closely resembles that found to be
optimal for human infants. Human milk fat is characterized by high contents of palmitic and oleic acids.

The principal mineral constituents of human milk are Na, K, Ca, Mg, P, and Cl.
Calcium concentrations reported in various studies vary from 25--35 mg/100 ml.
Phosphorus at 13--16 mg/100 ml is much more constant. Human milk is lower in proportion to casein and calcium than in milks of most
other species. Iron, copper, and zinc contents of human milk vary considerably.
All of the vitamins, except K, are found in human milk in nutritionally significant concentrations.

Benefits:
Baby
1. Protects from infection: Diarrhoea, ARI, (Secretory IgA, Bifidus growth factor, Colostrum filled with antibodies)
2. Reduces the risk of DM, Allergic, IBD
3. Improved cognitive development & intelligence (Bonding & fatty acids in breastmilk)
4. Protects from obesity: Eating until hunger is satisfied, BM stimulates less insulin than formula (Insulin stimulates the creation of fat),
More appetite regulating hormone leptin. Formula fed infants gain weight more rapidly in the first weeks of life. Rapid weight gain is
associated with later obesity.
5. Reduces risk of SIDS

Mother
1. Reduces the risk of postpartum depression: Oxytocin promotes nurturing and relaxation
2. Reduces the risk of breast cancer: Lactation suppresses amount of estrogen the body produces.
3. Post gestational amenorrhea by delaying ovulation via prolactin which keeps estrogen and progesterone at bay so ovulation is not
triggered especially night time breast feeding
4. Oxytocin help uterus to contract.
5. Empowering to see your baby grow and thrive on your breast milk alone
6. Lose pregnancy weight faster
7. Easier: Always available and always at the right temperature
8. Bonding

Environment: The dairy cows are major contributor to global warming, Belching, dung, flatulence produces enormous amount of methane.
A harmful green house gas. It is cheap and Cost effective

Actions that help protect, promote and support breastfeeding:


1. Maternity Protection Convention 183 (International Labour Organizations) and recommendation No 191
Longer duration of leave and higher benefits
2. International Code of Marketing of Breast-Milk Substitutes & World Health Assembly resolutions.
3. Ten Steps of Successful Breastfeeding (BFI)

20
Compiled by Obasi. D. C. Chinedu

Skin-to-skin contact, initiation of BF within first hour of life


Breastfeeding on demand (day & night)
Rooming-in
Not giving babies additional food or drink even water (unless medical)
4. Counseling at any contact (ANC, WCC, Immunization)
5. Community support (mother support groups, CB-health promotion, an education activities)

Complementary feeding: Around 6 months old, energy and nutrients needs starts to exceed what BM can provide. Complementary foods
are necessary to meet those needs. Infant is developmentally ready for other foods. Poorly instituted CF can lead to poor growth.

Guiding Principles: The following should be noted;


1. Frequent, on-demand breastfeeding until 2 years or beyond
2. Practice responsive feeding:
- Feed infants directly, - Assist older children - Feed slowly - Encourage to eat but no force
- Talk to the child and maintain eye contact
3. Practice good hygiene and proper food handling
4. Start at 6 months with small amounts of food and gradually increase
5. Gradually increase food consistency and variety. 6 months (Semi-solid : Porridge, mashed foods), 8 months (Finger foods (can
hold): Bananas, Mango, pawpaw, boiled egg, Porridges, Pap)
6. Increase the number of times the child is fed; 2 -3 meals/day 6 -8months; 3-4 meals/ day 9 -24 months (Additional 1-2 good snacks
/day 6 months after).
7. Use fortified complementary foods or vitamin-mineral supplements.
8. During illness, increase fluid intake including more breastfeeding and offer soft, favorite foods

Feeding sick children: Use Small amount of food frequently, Preferably soft food (gruel, mashed potatoes or soup). Plenty of fluid. Freshly
prepared foods. Target feeding when temperature is low.
Mouth clean and nose unblocked, Wick the nostrils, pre-feeding saline nasal drop.
Add little fat-rich food or sugar (easy way to increase energy without making the meal big & bulky). Add fruits and vegetables.

Encouraging young children to eat: Sit with them. Feed with the rest of the family but from their own plate and spoon (to estimate the
quantity the consumed).
The can eat with their hands if that is what they want (messy but don’t worry about table manners). Mix foods together if they pick out only
favorite foods. Allow them to play while eating.
Be patient. Allow they to drink while eating but not too much. Try to feed when hungry. Avoid feeding when sleepy. Avoid force feeding

Feeding in exceptionally difficult circumstances:


1. Low-birth weight or premature infants
2. Mothers living with HIV in setting where mortality due to diarrhea, pneumonia and malnutrition remain prevalent
3. Infants and young children who are malnourished
4. In emergencies

HIV and infant feeding: Poses challenge (Risk of transmission of the virus via breastfeeding). Avoidance or early cessation of bf seemed
logical (prior 2010). Repercussions for the health and survival of the infants were serious.
Higher mortality rates due to diarrhea, malnutrition and other diseases in non-breastfed children’

2010 recommendation based on evidence of positive outcomes for HIV-free survival through provision of ARVs to breastfed HIV
infants.
Without intervention (35% of HIV-positive pregnant women will infect their children (10 -20% via breastmilk). Risk after 6 weeks is
estimated at around 1% per month of breastfeeding. Viral load, duration of breastfeeding and condition of the breasts (sores around the
nipples) contributes to its transmission.

EBF (6months) 3-4 fold lower risk of HIV transmission compared to mixed feeding, 4% risk with EBF even in the absence of ARVs.
 HIV-infected mother + ARVs + mix-feeds = higher rate of transmission
 HIV-infected mother + ARVs + EBF = Lesser
No breastfeed more than 14 times more likely to die from diarrhea or respiratory infections than babies EBF in the 1 st 6 months.
Liquids and foods can damage already delicate and permeable gut wall.

Country specific guideline: BF + ARV or ARV – BF

21
Compiled by Obasi. D. C. Chinedu

Avoid counseling on balance of risks and the different options the mother can choose from. Avoid providing information on alternative
options.
Single national public health recommendation: ARV provided until one week after all BF has ceased.
Replacement feeding should not be used useless it is acceptable, feasible, affordable, sustainable and safe (AFASS)
AFASS translation to practical counseling messages is still a challenge.
Access to ARV: EBF even if no ARV

ART reduces the risk of postnatal HIV transmission in the context of mixed feeding. Although exclusive breastfeeding is recommended,
practising mixed feeding is not a reason to stop breastfeeding in the presence of ARV drugs.
Shorter durations of breastfeeding of less than 12 months are better than never initiating breastfeeding at all.

Picky eater: peck and poke at their food. It is part of being a toddler ( 1 – 3 years)
Respect your child’s appetite – or lack of one. Stick to the routine. Offer new things frequently from all different food groups & be patient
with new food.
Make it fun. Ask them to select and allow them to help if old enough. Keep junk food out of the house (healthful food or starvation). Observe
taste and food they dislike and avoid them. Subtly avoid force feeding, to prevent power struggle. If older enough always remind them the
next meal time.
Give vitamin. Offer a nibble tray. Cut it to sizes,
Respect tiny tummies. (a child’s stomach is approximately the size of his fist).
Appropriate sit size. Turn meals upside down. Be mindful of whom you are arguing with.

Overweight and obesity: BMI Kg/M2.


Overweight 25 – 29.9
Obese 30 & over.
These can arise from: Excessive food energy intake and poor, unbalanced diets. Reduce energy intake, increase physical activity, eat balance
diet. Explain the risk and causes of overweight. Encourage to be more physical
Advise on what to eat: Plenty of fresh fruits & vegetables, Lean meat and fish, Whole meal cereals
Avoid: Fatty & sugary foods.

Selective eating disorder (SED): Eating a highly limited range of foods associated with an unwillingness to try a new food. Eating disorder
that prevents the consumption of certain foods. Generally overcome with age. Cognitive behavioral therapy.

Keeping food safe:


Prepare in safe, hygienic way: Hand washing, Covering food to prevent flies and rodents. Use clean water.
Prevent germ form multiplying: Refrigerate, Salting, smoking (Prevent mould (aflatoxin)), Serve with clean plates.

22
Compiled by Obasi. D. C. Chinedu

THE NEWBORN (NORMAL AND ABNORMAL)


Herbert A Obu, MBBS, FWACP.

Introduction
The health of the newborn is intricately interwoven with that of the mother, and factors operating during the pre-pregnant period through
pregnancy, labor and delivery contribute to the overall well being of the newborn infant.

Effective care of the newborn, therefore, should start during this period. In the ante-natal period, efforts should be made to detect high-risk
pregnancies and make appropriate referrals to centres where such pregnancies will be well managed with a view to preventing untoward
outcomes for both the mother and the baby.

More than 50% of the high infant mortality seen in the tropics and developing countries is due to neonatal mortality.
This is a direct reflection of the quality of obstetrical and neonatal services available in a country. Improvement in the provision of these
services, will no doubt, go a long way in contributing to reduction of neonatal mortality, and thus of infant mortality.

This informs the need for routine assessment/care of the newborn with a view to determine departures from normal early enough and institute
appropriate “corrective “measures”. An understanding of the condition of the normal newborn at birth is necessary to meet the above
objective and this is the essence of this lecture.

Adaptation of the fetus to birth


The neonatal period is a very vulnerable period for the newborn baby in view of the risks and uncertainties of transition and subsequent
adaptation to extra-uterine life. The fetus occupies an aquatic environment during pregnancy and receives oxygen and nutrition from the
mother through the placenta. The foetal lungs are filled with fluid and have no respiratory function while the fetus remains in utero.
Oxygenated blood flows from the placenta via the umbilical vein and through the ductus venosus (ligamentum teres) into the inferior vena
cava and to the heart.

Because the fetal lungs are unexpanded, there is a high pulmonary vascular resistance and oxygenated blood is diverted away from the
right ventricle through the foramen ovale. In addition, the ductus arteriosus provides a further right to left shunting of blood away from the
lungs. Less than 10% of blood ejected from the heart enters the lungs. The pressure in the right heart in utero is higher than in the left
chambers. Systemic blood flow is preferentially diverted to the head and desaturated blood returns to the placenta via the umbilical arteries
which arise from the internal iliac vessels. Fetal blood is considerably less saturated than that of the newborn. The fetus is well adapted
to this low oxygen saturation by virtue of fetal hemoglobin which has a higher oxygen affinity than adult hemoglobin.

At birth, the infant gasps and expands its lungs. The pulmonary vascular resistance falls rapidly to below systemic levels and pressure in
the right side of the heart falls. As the umbilical cord is clamped/ligated, the low resistance placenta is removed from the circulation and the
systemic resistance increases.

These changes in resistance between lung and systemic circulation cause a dramatic change in blood flow through the lungs and the ductus
arteriosus. Left atrial pressure increases and functional closure of the foramen ovale occurs. With the start of breathing, arterial blood
becomes fully saturated and this is the stimulus for ductus arteriosus to close within the first few hours of life. Prostaglandins are necessary
to maintain the ductus physiologically closed and anatomical obliteration occurs by 2 weeks from birth.

In a nutshell, the newborn at birth is confronted by metabolic problems such as hypoxia, hypothermia and hypoglycaemia as a result
of cessation of placental supply of oxygen and nutrients, and cold shock from change from warm intra-uterine environment to cold
extra-uterine environment.

The first cry and active breath of air, once taken and sustained, would set in motion a cascade of events that empties the lungs of
fluid, establishes the neonatal lung volume and subsequent normal breathing and finally converts the fetal to adult circulation.
Physiological adaptation is usually complete within 24 hours of birth and is manifested by stability of vital signs, feeding,
gastrointestinal and renal functions.
Such babies exhibit hunger, cry appropriately and are able to maintain normal temperatures. Babies with delayed or compromised adaptation
tend to have delayed hunger and feeding or difficulty maintaining body temperature.

Poor transition and adaptation from intra-uterine to extra-uterine life may arise from a number of factors which may be related to the baby,
the mother or the process of delivery and will result in problems in the immediate neonatal period.

This underscores the need for routine assessment of every newborn baby with the objectives of:
1. Determining adaptation failure and need for immediate intervention or otherwise
2. Identifying potential problems with the newborn
3. Detecting congenital anomalies
4. Establishing a baseline for subsequent examination.

This assessment is done immediately after delivery and involves APGAR Scoring and general examination for gross
23
Compiled by Obasi. D. C. Chinedu

abnormalities/malformations. A more detailed examination of the baby is performed later but must be conducted within 24 hours of delivery.

Active participation of the mother and her husband/partner during the birth process and assessment of the newborn should be encouraged.
This helps them to adapt to parenting and at the same time affords one the opportunity to offer explanations to them on issues concerning the
newborn that may be of concern to them.

APGAR SCORING
This assessment is done in the first minute after birth and may be repeated at 5, 10 and more minutes from birth if the baby has not
reached optimal condition by then. Virginia Apgar introduced this scoring system in 1953. The scoring must be done accurately and
objectively for each of the five parameters used in the scoring system, notably heart rate, respiratory effort, muscle tone, reflex irritability
and colour.

Apgar scores of 8 and above indicate that the baby is making a smooth transition to extra-uterine life whereas Apgar scores of ≤ 7 at
5 minutes (and particularly up to 10 min) are linked with higher morbidity and mortality rates.

Apgar scoring:
Sign Score = 0 Score = 1 Score = 2

Activity (muscle tone) Limp Some flexion of Normal, active


the
extremeties

Pulse (Heart rate) Absent Less than 100 bpm More than 100
bpm

Grimace (Reflex None Grimace Sneeze, cough


irritability)

Appearance (Colour) All blue, pale Pink body, pale All pink
extremeties

Respiration Absent Irregular, slow Good, crying

Birth weight:
The weight should be done immediately after delivery. Normal birth weight varies from 2.5 to 4kg. Infants with birth weight less than
2.5kg are said to have low birth weight (LBW), less than 1.5kg very low birth weight (VLBW) while less than 1 kg (1000 gm)
Extreme Very Low Birth Weight (EVLBW). The weight is an indispensable anthropometric parameter in neonatal care and in child health
in general. For any given gestation, the lower the birth weight, the higher the morbidity and mortality.

Other basic measurements:


Length is measured from crown to heel; normal values are based on gestational age and should be plotted on standard growth charts. For a
term newborn, the length varies from 48 to 53.5 cm (51 cm at 50th centile).
Head circumference iNormal values are also based on gestational age and should also be plotted on standard growth charts. For a term baby
the head circumference varies from 33.5 to 37cm with a 50th centile figure of 35.

Gestational age assessment:


The gestational age of each newborn should be assessed as soon as possible after delivery. Where the mother is sure of her last menstrual
period (LMP), this is utilized in calculating the gestational age of the baby. Term infants are those born after 37 completed weeks up to
42 weeks while pre-term infants are born before 37 completed weeks. Post term infants are born after 42 completed weeks.

The Dubowitz and Ballard/modified Ballard Scores are more objective methods of assessing gestational age of newborn infants and
should be performed as soon as possible and usually within 24 hours of life. This method of assessment uses both physical and
neuromuscular maturity ratings. Refer to standard textbook of Paediatrics for this scoring system.

The rest of the examination of the newborn is conducted over the next 24 hours. Many clinicians begin with an examination of the heart and
the lungs, followed by a systemic head-to-toe examination, looking particularly for signs of birth injuries/trauma and congenital
anomalies/malformations.

Cardiorespiratory system:
The heart and lungs are evaluated when the infant is quiet.
The clinic should identify where the heart sounds are loudest to exclude dextrocardia. The heart rate should be evaluated as well and normal
24
Compiled by Obasi. D. C. Chinedu

values range from 100-160 bpm. Rhythm should be regular although an irregular rhythm from premature atrial or ventricular contractions
is not uncommon. A murmur heard in the first 24 hours is most commonly caused by a patent ductus arteriosus but this usually
would have disappeared by the 3rd day of life.

Femoral pulses should be checked and compared with brachial pulses to exclude coarctation of the aorta.
Central cyanosis may suggest congenital heart disease, pulmonary disease, sepsis and may be central in origin.
The hyperoxia test is used to determine the level of the cause of the cyanosis.

The respiratory system is evaluated by counting respiratory movements for a full minute. Normal rate is bw 40 and 60 breaths per minute.
Chest wall should be examined for symmetry and lung sounds should be equal throughout. Grunting, nasal flaring and retractions are signs of
respiratory distress.

Head and neck:


In a vertex delivery, the head is commonly molded with overriding of the cranial bones at the sutures and some swelling and ecchymosis of
the scalp (caput succedaneum).
The fontanels vary in diameter from a fingertip breadth to several cm; a large anterior fontanel may be a sign of hypothyroidism. Inspect
the head for other abnormalities such as cephalhaematoma, craniofacial abnormalities such as low set ears, Mongolian slant, orbital
hypertelorism, natal teeth, cleft lip/palate, etc.

The eyes should be examined for the red reflex; its absence may indicate cataracts, glaucoma or retinoblastoma. Sub-conjuctival
haemorrages are common and are caused by forces exerted during delivery. The neck should be examined to exclude shortness/webbing of
the neck, congenital goiters, cystic hygromas, torticollis, etc.

Abdomen and Pelvis:


The abdomen should be round and symmetric. A scaphoid abdomen may indicate diaphragmatic hernia while an asymmetric abdomen
suggests an abdominal mass.
Splenomegaly may be due to congenital infection or haemolytic anaemia. The liver is normally palpable 1-2cm below the costal margin
and the kidneys may be palpable on deep palpation; large kidneys, however, may indicate obstruction, tumour or cystic disease.

Umbilical hernia may be present and anus should be examined for normal positioning and patency. The external genitalia should be
examined in both sexes to ensure that they are normally developed and any departure from normal noted. Particular attention must be paid to
exclude any ambiguity of the external genitalia of any nature and any confusing issues resolved before assigning a sex of rearing.
Is the anus patent or are there obvious anorectal abnormalities?

Musculoskeletal system:
Examine the extremeties for deformities, amputations, contractures and maldevelopment.
The spine is inspected for neural tube defects, particularly exposure of the meninges, spinal cord or both.
Orthopaedic examination includes palpation of the long bones for birth trauma, especially clavicular fracture, and detection of hip dysplasia.
Evaluate the baby’s muscle tone, level of alertness, movement of extremeties and primitive reflexes.

Vernix caseosa covers most neonates more than 24 wk gestation.


Petechiae may occur in areas traumatized during delivery, usually the presenting part, but more diffuse petechiae need to be investigated.
Erythema toxicum is seen in many neonates; this is a benign rash with an erythematous base and a white or yellow papule. Usually appears
24 hr after birth, is scattered over the body and can last for up to 2 weeks.

Thank you for listening

25
Compiled by Obasi. D. C. Chinedu

MALARIA IN CHILDREN
Prof. HU OKAFOR

Outline: - Overview - Epidemiology - Malaria Transmission - Pathophysiology


 Clinical features - Diagnosis - Treatment - Prevention

Overview Of Malaria
Malaria is one of the commonest reasons for hospital consultation in Nigeria. 3.3 million people are at risk globally and 216 million cases
recorded in 91 countries in 2016. Accounted for 446000 deaths globally, more in children under five living in sub-Saharan Africa. Nigeria
accounted for 27%of malaria cases and 24%of malaria deaths globally.
It is a disease caused by the parasite of the genus Plasmodium. The infection is usually transmitted by the bite of an infected female
anopheles mosquito. Man is a reservoir of infection.

Epidemiology of Malaria in Nigeria


The disease is endemic in Nigeria & Transmission is stable in most parts. About 97% of the population of Nigeria are at risk. 50% of the
population will have at least one attack/year. Vulnerable groups are under-5s, pregnant women, immune compromised and the elderly. About
300,000 children die of malaria in Nigeria,EACH YEAR. 11% of pregnant women die of malaria EACH YEAR

Causative agents of Malaria: Common Plasmodium species which cause the disease in humans include
 P. falciparum - P .vivax, - P. Ovale - P. malariae.
 Recently P.Knowlesi in SE Asia.

Plasmodium falciparum- commonest specie in virtually all parts of Africa, accounting for up to 98 % of the cases of malaria in Nigeria and
is associated with significant morbidity and mortality. It is also responsible for virtually all the features of severe malaria.
P. malariae and P. ovale cause up to 2% of malaria cases in Nigeria while P. vivax is very rare in Africa except in Eritrea and Ethiopia where
they account for up to 40% of the cases.
Malaria vectors: Infected Female Anopheles mosquitoes of which there are 400 species. The predominant species in Nigeria include A.
gambiae ss complex, and A. funestus. Others are; A.arabiensis(North),A.melas found in coastal areas.

ENDEMICITY
In Africa malaria varies from country to country and sometimes from one part to another within the same country. Some of the factors
responsible for the variations in endemicity include:-
 Rainfall pattern - Altitude - Temperature
High rainfall pattern is associated with high malaria transmission while places with high altitude and low temperatures tend to be associated
with lower rates of transmission. Transmission peaks in the rainy season; April-November
Patterns of malaria endemicity:
Stable malaria: Where malaria is transmitted all year round, but may have seasonal variation. Adults living in these areas usually acquire
some protection against malaria and may be less likely to get severe malaria.
Unstable malaria: It is characterized by intermittent transmission that may be annual, bi-annual or variable epidemic due to poor immunity
against malaria.
Malaria free areas: Usually the population has no immunity whatsoever against malaria and therefore all age groups are prone to severe
malaria if exposed to the parasite.
In Nigeria malaria is stable practically every where including the far north, but the possibility in break in transmission in the Sahelian areas,
of up to 2 years, leading to epidemic transmission should be borne in mind.

Patterns of endemicity based on spleen rates: Hypo


endemic malaria: spleen rate in children (2 - 9 yrs), less than 10%.
Mesoendemic: spleen rate between 11-50%.
Hyper endemic: spleen rates are > 50% in children and 25 % in
adults.
Holoendemic: spleen rates up to 75% in children but very low in
adults.

Malaria transmission cycle (See Image on the right)

Parasite Life Cycle:


Sporozoites-30mins-some phagocytosed.
Tissue phase-sporozoites invade liver cells-undergo
schizogony (Pre-erythrocytic)
P.Falc(5.5-7dys);P.vivax(6-
8dys);P.ovale(9days);Pmalariae(12-16dys).

26
Compiled by Obasi. D. C. Chinedu

Hypnozoites; exo-erythrocytic schizogony.


Release of merozoites (36-72hrs)- erythrocytic schizogony(36-72hrs) shortest in P.falc-release of merozoites –re-invasion of red cells
Release of merozoites correspond with fever paroxysms.
P.falc-sub-tertian(<3dys),P.ovale/P.vivax-tertian(every 3 rd day);P.malariae-quartan(every 4th dy).

Pathogenesis
The parasites digest Hb leading to formation of haemozoin(pigment). The effects of malaria infection range from completely asymptomatic
infection to fatal severe disease and many factors influence the clinical manifestations.

These factors include:

 cytokines produced by the individual in response to malaria infection called tumor necrosis factor (TNF). This may contribute to
some of the symptoms and signs of severe malaria e.g coma, hypoglycemia, acidosis and adult respiratory distress syndrome
(ARDS).
 Other mechanisms like sequestration of red blood cells with maturing parasites deep in the capillaries and venules may be the cause
of altered level of consciousness.

Metabolic changes within the RBC


 Digestion of haemoglobin with utilization of glucose, oxygen and hemozoin formation.
 Hypoxia stimulating lactate formation.
 Free radical formation.
 TNF is major mediator of the changes seen in severe malaria.

Mechanism of Sequestration
 Alteration of the membrane transport system of the RBC
 Decrease deformability of the RBC
 increased susceptibility to sludging and destruction
 Formation of knobs (from the periphery of mature schizonts) and protruberances on the RBC surface leading to formation of neo-
antigens with increased immune stimulation and increased vascular stickiness (ICAM & ELAM) and cytoadherance.
 Presence of thrombospondin/CD46 which help in rosette formation.

Xteristics of Plasmodium falciparum


It infects RBCs of all ages (P vivax and ovale – young, P malariae –old)
Causes rosetting, sequestration – PfEMP1 binds to CD36, thrombospondin, VCAM-1, ICAM-1 and E selectin
TNF, IFN-γ, IL-1 which suppress production of RBC etc

CLINICAL FEATURES
May be classified as complicated severe malaria ,asymptomatic, or uncomplicated. Clinical features include:
 fever. - headache, - chills or rigors, - general weakness, - Vomiting,
 loss of appetite - profuse sweating - Myalgia/Arthralgia
 Malaise &Prostration with loss of appetite.
 In young children there may be abdominal pain, vomiting and poor feeding and diarrhea.

DEFINITION OF SEVERE MALARIA


In a patient with P. falciparum asexual parasitaemia and no other confirmed cause for their symptoms/signs, the presence of one or more of
the clinical or laboratory features in the Table below classifies the patient as suffering from severe malaria.
Clinical manifestations or laboratory findings Frequency a

Children Adults

 Prostration (i.e. generalised weakness or inability to sit, stand or +++ +++


walk without support)

 Impaired consciousness (confusion or drowsiness or coma) +++ ++

 Respiratory distress (difficulty in breathing, fast deep breath) +++ +

 Multiple convulsions +++ +

 Circulatory collapse (shock) + +

 Pulmonary oedema (respiratory distress) +/- +

27
Compiled by Obasi. D. C. Chinedu

 Abnormal bleeding (disseminated intravascular coagulopathy) +/- +

 Jaundice (yellow discoloration of the eyes) + +++

 Haemoglobinuria (Coca-Cola coloured urine) +/- +

 Severe anaemia ( Hb <5 gm/dl) +++ +

 Hyperparasitaemiab >250,0000 parasites/ul ++ +

 Hyperpyrexia >39.50C ++ +

 Hypoglycaemia <2.2mmol/l ++ ++

 Renal failure > creatinine >265 micrommole/L +/- ++

 Persistent vomiting >x4 in 30mins ++ +

Management
1. Good history
2. Investigation
3. Supportive treatment
4. Specific treatment

DIAGNOSIS
LABORATORY DIAGNOSIS: In children under 5 in areas with stable malaria a clinical diagnosis is adequate for treatment of
uncomplicated malaria but parasite based diagnosis is the current policy.
Parasitological diagnosis (blood slides or RDTs) may be required in:-
Confirming the diagnosis, Cases of treatment failure
For older children and adults parasitological diagnosis before treatment is highly recommended.
For Microscopy giemsa stained thick and thin blood films are prepared to determine the parasite density and the specie of the parasites.
Other investigations include: Hemoglobin (Hb) level or Packed Cell Volume (PCV)

Complications
1. Cerebral Malaria
2. Severe malaria anemia
3. Pulmonary edema/ARDS
4. Acute Kidney Injury
5. Bleeding diasthesis

TREATMENT
Current WHO standard for selection of antimalarial drugs
Efficacy
1. Any drug to be used should have a minimal efficacy of 95%
2. A drug should be removed when the efficacy is less than 90%.

National Treatment Guidelines


ACTs-AL & AA for uncomplicated malaria. Injection Artesunate for severe malaria
ACTs are used in children >5kg. Younger children and neonates are given amodiaquine or quinine except in cases of complicated
malaria.
Pregnant women less than 16 weeks gestation are given quinine and clindamycin for uncomplicated malaria and parenteral artesunate
for complicated malaria.

Why Artemisinin-based combinations?


Artemisinins
1. Rapidly effective
2. Reliably effective
3. Safe
4. Prevent emergence of resistance
5. Reduce transmission potential (reduction of gametocyte carriage)
6. 3 day regimens
7. Broad stage specificity (ring forms, mature and old schizonts)

28
Compiled by Obasi. D. C. Chinedu

8. No significant resistance so far reported

Treatment of severe malaria


Supportive treatment
Coma or unconscious patient: Correct hypoglycaemia, Children: 0.5 -1gm of glucose per kg (i.e. 1-2 ml / kg of 50% dextrose) diluted to
10-15%.
Convulsions: Ensure patent airway and that the patient is breathing. Correct hypoglycaemia or control temperature.
In children give rectal diazepam 0.5 mg/kg or IM paraldehyde 0.1 ml/kg. If convulsions continue, give IM phenobarbitone 10-15 mg/kg.
Severe malaria anemia: Use packed cells (10 ml/kg in children) or whole blood (plus frusemide). Where blood is not available, give pre-
referral treatment and refer urgently to a health facility with blood transfusion services. Give urgent blood transfusion to patients with
severe pallor/anaemia in heart failure. The blood must be screened to ensure that it is HIV, Hepatitis B and C negative.
If convulsions continue with no clear cause despite above treatment investigate further.
Severe dehydration or shock: In severe dehydration or hypovolaemic shock give 20-30 ml/kg of normal saline and reassess the patient
within 30 minutes to decide on the next fluid requirement according to the degree of dehydration hydrated.

Anti-Malaria Treatment for severe malaria:


The primary objective of antimalarial treatment in severe malaria is to save life. Prevention of recrudescence and avoidance of minor adverse
effects are secondary. Quinine or Artemisinin derivatives given parenterally are choice of treatment for severe malaria. Presently IV
artesunate is the drug of choice and first line treatment.

Intermittent Preventive Treatment (IPT):


IPT is based on the assumption that every pregnant woman living in an area of high malaria transmission has malaria parasites in her blood
or placenta, whether or not she has symptoms of malaria

Intermittent Preventive Treatment: WHO Recommendation


1. All pregnant women should receive at least 3 doses of IPT after quickening, during routinely scheduled ANC visits but no more
frequently than monthly (as DOT)
2. WHO recommends a schedule of four visits, three after quickening (From 20 weeks+)
3. Presently, the most effective drug for IPT is sulfadoxine-pyrimethamine (SP)
4. HIV positive pregnant women should receive at least three doses of IPT with SP at ANC visits after quickening, but no more
frequently than monthly
5. HIV positive mothers on cotrimoxazole prophylaxis are exempt.

IPT: Special target groups:


1. Women in their first or second pregnancies
2. HIV infected women
3. Adolescents (10-19 years of age)
4. Women with sickle cell disease
5. All pregnant women with unexplained anaemia

Intermittent Preventive Treatment: Dose of SP


 A single dose is three tablets of SP each containing sulfadoxine (500 mg) + pyrimethamine (25 mg)
 Healthcare providers should dispense the dose and directly observe the client taking the tablets (DOT strategy)

Remember Contraindications to SP:


1. Before 16 weeks of pregnancy
2. SP dose in last 4 weeks
3. Allergies to sulfa drugs (e.g., co-trimoxazole)
4. Currently taking other sulfa drugs
 Substitute other drug before giving SP
 No folate for one week when SP is given

Insecticide-Treated Nets
 Repel and Kill mosquitoes
 Prevent physical contact with mosquitoes
 Repel and Kill other insects: Lice, Ticks, Bedbugs, Cockroaches

Action Untreated Nets ITNs

Protection against malaria Provides some levels Provides high levels

29
Compiled by Obasi. D. C. Chinedu

Kill and repel Mosquitoes No Yes

Reduce No of mosquitoes in/outside No Yes


net

Kill other insects e.g. lice, roaches, No Yes


bedbugs

Safe for pregnant women, young Yes Yes


children and infants

Benefits of Insecticide-Treated Nets to the Community


1. ITNs cost less than treating malaria
2. Reduce number of sick children and adults (helping children grow to be healthy and helping working adults to remain productive)
3. Reduce number of deaths in the community

Environmental Management:
Malaria carrying mosquitoes breed on clean stagnant water. When you work on your surroundings, the number of mosquitoes reduces.
Things to do to bring this about are:
 Throwing away containers, broken pots, unused tyres where water collect and mosquitoes breed.
 Cutting grasses around the house and community
 Putting window nets
 Covering containers used to store water
 Filling up pot holes and pools of water that don’t flow
 Regulating water holding plants and other water reservoirs

Conclusion
Nigeria has a huge malaria burden. Malaria is preventable, treatable and curable. Proper harnessing of resources required in the fight against
malaria. We are determined to fight this scourge with the best arsenal available. Time is now right for rapid scale up to achieve impact

30
Compiled by Obasi. D. C. Chinedu

GENETIC COUNSELLING AND PRENATAL DIAGNOSIS


ILOH, KENECHUKWU K (MBBS, Dip(Theo)

What is Genetic Counselling: Genetic counseling is an educational process that involves provision of information to affected individuals or
family member at risk of a genetic disorder, the consequences of the disorder, risk (probability) of developing or transmitting it and ways in
which it may be prevented or ameliorated and means available to modify its consequences. OR
An educational process that seeks to assist affected and/at risk individuals to understand the nature of a genetic disorder, its transmission and
the options available to them in mgt and family planning.

Who provides Genetic Counselling: Medical geneticist and genetic counselors, experienced family physicians, pediatrician or nurse. These
must have a sound understanding of genetic principles
1. The ability to recognize and diagnose genetic diseases and rare syndromes.
2. Knowledge of the natural history of the disorder and its recurrence risk.
3. Awareness of prenatal diagnosis and screening programs available in a particular region and access to information about advances in
genetic disorders and medical techniques are also necessary.
4. Environmental influence on gene expression.

SITUATIONS THAT REQUIRE GENETIC COUNSELLING


Prenatal diagnosis of a congenital anomaly or genetic dx - Information is urgently needed here because a family needs to decide whether to
continue or to terminate a pregnancy. Consideration for the risk to the mother.
A CHILD BORN WITH LIFE-THREATENING CONGENITAL ANOMALY OR GENETIC DX - requires urgent information and
decisions that must be made immediately with regard to how much support should be provided for the child and whether certain types of
therapy should be attempted.

Situations that arise later in life e.g


1. When a diagnosis with a genetic implication is made.
2. A couple is planning a family and there is a family history of a genetic problem; including whether one member of a couple carries a
translocation or is a carrier of an abnormal gene for an autosomal recessive or X-linked disorder.
3. An adolescent or young adult has a family hx of an adult – onset genetic disorder (e.g. Huntington dx or breast cancer)
4. Unusual feature are present and a diagnosis is wanting or not possible.
5. There is suspected exposure to a toxic substance or teratogen.

INDICATIONS FOR GENETIC COUNSELLING


1. Advanced parental age:- maternal age >35 years, paternal age > 50 years
2. Child with congenital anomalies or dysmorphology.
3. Consanguinity or incest
4. Family hx of heritable disorders or diseases
5. Heterozygote screening based on ethnicity including; SCD, Thalessemia.
6. Pregnancy screening abnormality including: maternal serum alpha fetoprotein, maternal serum triple screen, prenatal ultrasound
examination
7. Still born with congenital anomalies.
8. Teratogen exposure or risk.

GOOD GENETIC COUNSELLING REQUIRES


Taking a careful family hx and constructing a pedigree that lists the pxt’s relatives (including, abortions, still births and deceased individuals)
with sex, age and state of health up to and including third degree relatives.
Gathering information for hospital records about the affected individuals and in some cases about other family members.
Documenting prenatal, pregnancy and delivery histories. Giving the family information about support groups.
Providing new information to the family as it becomes available (a mechanism for updating needs to be established)
Receiving the latest available medical, laboratory and genetic information concerning the disorder.
Careful physical examination of the affected individual with photographs and measurements and in apparently unaffected family members.
Establishing or confirming the diagnosis by the diagnostic tests available.

INFORMATION NECESSARY FOR OPTIMAL BENEFITS OF GENETIC COUNSELLING.


The specific condition or conditions, Natural history of the condition. Genetic aspects of the condition and recurrence risk, Prenatal
Diagnosis and Prevention. Therapies and Referrals. Support Groups. Follow – up
Nondirective Counseling. Finally, genetic counseling is nondirective to the family to decide what is right for them.

MODES OF DISEASE INHERITANCE


Autosomal Dominant Disorders
Autosomal dominant disorder are those in a single gene in the heterozygous state is sufficient to cause the phenotype
Features: They appears in vertical pattern in the pedigree which affects

31
Compiled by Obasi. D. C. Chinedu

-Any child of an affected parent has a 50% risk of inheriting the disorder
-Phenotypically normal family member do not transmit the condition to their offspring
-Males and females are equally affected
-Significant proportion of cases is due to new mutation
e.g: Neurofibromatosis (NF1 &NF2), Progressive Sensorineural deafness, Osteogenesis imperfecta, Apert syndrome and Crouzon syndrome

Autosomal recessive disorders


Those in which 2 copies of the mutual gene in the homozygous state are necessary to cause the phenotype. Thus, both parents of an affected
carrier are heterozygous carriers of the gene
Features:
 Males and females are equally affected.
 Parents of an affected child are asymptomatic heterozygous carriers of the gene.
 Recurrence risk for siblings of an affected child id 25%
e.g; Tay-Sachs disease, Gaucher’s disease, cystic fibrosis, sickle cell disease, Fanconi Anemia

X – LINKED INHERITANCE
These are inherited disorders associated with a genotypic alteration on the X-chromosome exclusively. A great majority of these disorders
demonstrate a recessive pattern of inheritance. Few of the disorders demonstrate a dominant pattern of inheritance

X- linked recessive inheritance


High incidence in males than females. Heterozygous female carriers are usually unaffected. The gene is transferred from an affected man to
all his daughters and any of his daughter’s son has a 50% chance of inheriting the gene. Gene is never transmitted from father to son. Gene
may be transmitted through a series of carrier females in which case all affected males are related through the carrier females. Significant
proportion of sporadic cases are a consequence of new gene mutations.

Males are usually the ones affected by x-linked recessive disorders. When females are affected they inherit the x-linked recessive allele in a
homozygous state from both parents. Most x-linked recessive disorders are rare. This reduces the possibility of female homozygous
inheritance. However, consanguinity is the exception to this. X-linked recessive disorder are more common in females with Turner
syndrome (45X) in whom there is hemizygozity for all the genes on the x-chromosome (which are all expressed). Eg Hemophilia A

X linked Dominant Disorders


An x-linked dominant disorder is one which is regularly expressed in the heterozygous female carriers. Characteristics are:
All the daughters and more of the sons of an affected man have the condition. Both male and female offsprings of affected females have
a 50% risk of inheriting the condition. Affected females are about twice as common as affected males, but affected females typically have
milder (although variable) manifestations of the phenotype.
e.g Hypophosphatemia rickets (vit. D resistant rickets), males are more severely affected than females. Incontinentia pigmenta, which
affects skin, hair, teeth, nails, occurs exclusively in females (b/c the hemizygous state in males in lethal)

MULTIFACTORIAL INHERITANCE
These are products of multiple genetic and environmental factors. The predisposing genetic factors are heterogeneous and largely unknown.
There is a similar rate of recurrence (3-5%) among all 1st degree relatives. However it is unusual to find a substantial increase in risk for
relatives related more distantly than the 2nd degree to the index case. The risk of occurrence is related to the incidence of the disease. Some
disorder have a sex predilection indicated by an unequal M:F; congenital hip dislocation F>M.

The likelihood that both identical twins will be affected with the same anomaly is less than 100% but much greater than the chance that both
members of a non-identical twin pair will be affected. The risk of recurrence is increased when multiple family members are affected. The
risk of occurrence may be greater when the disorder is more severe. e.g. Neural tube defects, cleft lip, DM, stroke.

NON-TRADITIONAL PATTERNS OF INHERITANCE


These are genetic disorders which do not follow the usu. Patterns of dominant, recessive, x-linked or multi-factorial inheritance. Some
times they involve specific diseases and in other instance may apply to any hereditary disorder.
Certain of these disorders result from mutations in mitochondrial DNA. Non-traditional inheritance can also be the result of genomic
imprinting. In Prader-Willis syndrome; there is de-novo microdeletions of xsome 15 on the paternally derived xsome. Similar deletions in
maternal x-some results in Angelman Syndrome

PRENATAL DIAGNOSIS
Prenatal diagnosis or prenatal screening is testing for diseases or conditions in a fetus or embryo before it is born. The aim is to detect
birth defects.
Common defects include neural tube defects, Down syndrome, chromosome abnormalities, genetic diseases and other conditions, such as
spina bifida, cleft palate, Tay Sachs disease, sickle cell anemia, thalassemia, cystic fibrosis, and fragile x syndrome.

32
Compiled by Obasi. D. C. Chinedu

Diagnostic prenatal testing can be by invasive or non-invasive methods.


An invasive method involves probes or needles being inserted into the uterus, e.g. amniocentesis (14-20weeks), and chorionic villus
sampling (9.5-12.5 weeks).
Non-invasive techniques include examinations of the woman's womb through ultrasonography, maternal serum screens (i.e. Alpha-
fetoprotein) and also genetic analysis on fetal cells isolated from maternal blood.
Some screening tests performed on the woman are intended to detect traits or characteristics of the fetus.

There are three purposes of prenatal diagnosis:


(1) to enable timely medical or surgical treatment of a condition before or after birth.
(2) to give the parents the chance to abort a fetus with the diagnosed condition.
(3) to give parents the chance to "prepare" psychologically, socially, financially, and medically for a baby with a health problem or
disability, or for the likelihood of a stillbirth.

Indications
1. Women over the age of 35
2. Women who have previously had premature babies or babies with a birth defect, especially heart or genetic problems
3. Women who have high blood pressure, lupus, diabetes, asthma, or epilepsy
4. Women who have family histories or ethnic backgrounds prone to genetic disorders, or whose partners have these
5. Women who are pregnant with multiples (twins or more)
6. Women who have previously had miscarriages

Maternal serum screen include; Beta human Chorionic Gonadotropin, Pregnancy associated plasma protein A (PAPP-A), Maternal serum
alpha fetoprotein (MSAFP), Unconjugated estriol (uE3), Inhibin A.

PRENATAL SCREENING TEST


Decreased MSAFP -Incorrect GA, Trisomy 13, Trisomy 18, Trisomy 21.
Increased MSAFP - more advanced GA, multiple gestation, NTDs, open abdominal wall defects, renal diseases eg congenital nephrosis,
infantile polycystic disease, bilateral renal agenesis

Others include
 skin diseases eg ectodermal dysplasia, aplasia cutis congenital
 cystic adenomatoid malformation of the lungs
 maternal hepatic and ovarian tumours
 fetomaternal haemorrhage
 fetal demise

PATTERNS OF TRIPLE SCREENING RESULTS AND THEIR INTERPRETATION


CONDITION MSAFP uE3 hCG
NTD increased normal normal
Trisomy 21 low low increased
Trisomy 18 low low low
Molar preg low low very high
Multiple preg increased normal increased
Fetal death increased decreased decreased

Ethical issues of prenatal testing


 The option to continue or abort a pregnancy is the primary choice after most prenatal testing. Rarely, fetal intervention corrective
procedures are possible.
 Are the risks of prenatal diagnosis, such as amniocentesis worth the potential benefit?
 Some fear that this may lead to being able to pick and choose what children parents would like to have. This could lead to choice in sex,
physical characteristics, and personality in children. Some feel this type of eugenic abortion is already underway (for example, sex
selection) .
 Knowing about certain birth defects such as spina bifida and teratoma before birth may give the option of fetal surgery during
pregnancy, or assure that the appropriate treatment and/or surgery be provided immediately after birth.
 Questions of the value of mentally or physically disabled people in society.
 How to ensure that information about testing options is given in a non-directive and supportive way.
 That parents are well informed if they have to consider abortion vs. continuing a pregnancy.

33
Compiled by Obasi. D. C. Chinedu

CHILDHOOD NUTRITION, MALNUTRITION SYNDROMES & FTT


Dr Ughasoro Maduka D.

Facts
Every infant and child has the right to good nutrition according to the "Convention on the Rights of the Child".
Infant and young child feeding (IYCF) is a cornerstone of care for childhood development.
The WHO) defines malnutrition as "the cellular imbalance between the supply of nutrients and energy and the body's demand for them
to ensure growth, maintenance, and specific functions.“

Estimated range of dietary needs


Items. Total /kg/day
Energy (Kcal/kg) 100 – 150
Protein (g/kg) 0.8 - 3
H20 (ml) 50 - 150
Na+ (mMol) 2- 4
K+ (mMol) 0.5 – 2
Ca2+ (mMol) 4–6
Mg 2+ (mMol) 0.5 – 1
Fe++ (mg) 6
Vit A (iU) 1500 – 2500
Vit D (iU) 400
Vit E (iU) 5 – 10
Folic acid (mg) 0.35.

Malnutrition comprises both: Undernutrition and Over-nutrition (obesity), but the term malnutrition is most often used for forms on
undernutrition.
Globally in 2015, statistics show that, Stunted = 156 million children under 5, Wasted = 50 million, and overweight or obese = 42 million.

Undernutrition can take the form of


 Acute malnutrition (presence & degree of any)
 Bilateral pitting oedema
 MUAC <12.5cm
 WFH z-score < -2 (WHO standard for sex) or WFH percentage of the median <80% (NCHS references)
 Stunting (<90% height for age)
 Indicate chronic malnutrition
 Child > 2yrs unable to stand make a downward adjustment of 0.5cm
 Underweight (<80% weight for age)
 Micronutrient deficiencies

Types of Acute Malnutrition


Moderate Acute Malnutrition (MAM) (Wasting): characterized by; Weight-for-height between -3 and -2 z scores of the international
standard or MUAC between 11cm and 12.5cm
Severe Acute Malnutrition (SAM): Severe wasting W4H below -3Z scores. MUAC below 11cm, Oedema
Reference for WFH: WHO child growth standard (z-score), National Centre for Health Statistics (NCHS) % of median.

The term PEM applies to a group of related disorders that include marasmus, kwashiorkor, and intermediate states of marasmus-
kwashiorkor.
Marasmus involves inadequate intake of protein and calories and is characterized by emaciation (adaptive response to starvation).
Williams first used the term Kwashiokor in 1933, and it refers to an inadequate protein intake with reasonable caloric (energy) intake.
(maladaptive response to starvation). Edema is characteristic of kwashiorkor but is absent in marasmus.

Causes of Malnutrition:
Outcome Child malnutrition, death and
disability

Immediate causes Inadequate dietary intake &


Disease

34
Compiled by Obasi. D. C. Chinedu

Underlying causes at household A) Insufficient access to food


family level B) Inadequate maternal and
childcare practices
C) Poor sanitation/water and
inadequate health services
Basic causes at social level A) Quality and quantity of A) Inadequate and/or
actual resources – human, inappropriate knowledge and
economic and discriminatory attitudes,
organizational- and the way limit household access to
they are controlled actual resources
B) Potential resources: B) Political, cultural, religious,
environment, technology, economic and social
people systems, including women’s
status, limit the utilization of
potential resources.

Expected Normal Body Response:


Reduced intake
 ↓ Ac vity & energy expenditure.
 ↑ Cor sol
 ↑ Blood glucose
↑ energy supply.
 ↑Essen al Aas
↑ protein essen al for heomostasis.
 ↑ Subcutaneous fat loss.
 ↑ Fa y acids
Synthesis of metabolites.
↑ energy
 When fat store is depleted, protein catabolism (tissue breakdown) follows.

Classical theory
 Diet (↑(CH2O)n & ↓ Protein.
↑(CH2O)n
↑ blood glucose
↑ FTA & Glycerol via lipogenesis
Fatty liver.
↓ Protein.
 ↓ Serum AAs
↓ β lipoproteins
Accumulation of Lipids in the liver (fatty liver)
↓ Albumin produc on
↓ onco c pressure
o Arterial blood pressure
 ↓ GFR => ↑ Renin-Angiotensin-Aldosterone pathway
 ↑Na+ & H2O reabsorp on => Oedema.
 Oedema.

Clinical Features/Pathophysiology
 Unhappy & irritable, apathetic,
 Hair & skin changes (skin ulceration (Flaky paint dermatosis) = Low vitamins (A), low serum levels of zinc, & essential fatty acids.)
 Diarrhea & dehydration –
 enzymopathy (lactose intolerance) &
 villus atrophy => malabsorption.
 ↓ Gastric acid secre on => bacteria overgrowth.
 Oedema = The resulting hypoalbuminemia => extravascular fluid accumulation.
 Fatty liver (abdominal distention) = Impaired synthesis of B-lipoprotein.
35
Compiled by Obasi. D. C. Chinedu

 Infections: E.coli, Proteus, Pseudomonas, measles, Herpes simplex.


 Atrophy of lymphoid tissue (thymus, spleen) => ↓ circula ng lymphocytes.
 Impaired immunity = Malnutrition ramps up arginase activity in macrophages and monocytes.
 Compromised humoral and cell-mediated immunity.
 Normal phagocytic fxn of monocytes but impaired chemotaxis & intracellular killings.
 ↓complement level.
 Anaemia:
 Nutritional (Fe deficiency, micronutrient glycine and succinyl-CoA citric acid cycle)
 worm infestation, malaria,
 Enlarged liver:
 fatty infiltration, ↓ lipoprotein (impaired transporta on of lipids from the liver; chylomicrons, VLDL, LDL,HDL)
 Chylomicrons (enterocytes)
 VLDL (Liver)
 Eye:
 Vitamin A (retinal + opsin = rhodopsin) required for low-light vision (scotopic and colour vision)
 Immunity and growth-factor for epithelial cells
 req
 Night blindness, xerophthalmia, (xerosis, Bitot’s spot & Keratomalacia.

Investigations & Rxn


 RBS, Blood film, Urine M/C/S, stool M/C/S, CXR, tuberculin test, HIV, S/E/U/Cr.

Criteria for In-patient or out-patient care:


Any medical complications? Severe oedema*, failed appetite test, ≥ 1 IMCI danger signs (Unable to drink/breastfeed, vomits, convulsion,
lethargic/unconscious.)
Any metabolic derangement that causes death (Hypoglycemia, hypothermia, dys-eletrolytemia, infections)
 No correlation between anthropometric measurements and metabolic malnutrition.
 The only sign that correlates with severe metabolic derangement is a reduction in appetite.
 Poor appetite:
 Significant infection
 Metabolic abnormality such as liver dysfunction, electrolyte imbalance, cell membrane damage or damaged biochemical
pathways.
 Immediate risk of death.

How to do appetite test (AT)


AT should be conducted in a separate quiet area. Explain to the carer the purpose of the AT and how it is done. The carer should wash his
hands. Sit the child on carer’s lap and offer the RUTF. Do not force the child to eat the RUTF only encourage the child. Offer plenty of water
to drink from a cup as he/she is taking the RUTF.

The result of the appetite test


Pass. (Take minimum qty of RUFT, Offer the carer the treatment options to choose, Commence phase 2 treatment.)
Fail (Failed to take the minimum qty of RUTF, In-patient care, phase I Rxn
Weight and Minimum Quantity of RUTF:
- 4 kg (1/8-¼); - 4 – 6.9kg(1/4- 1/3) - 7 – 9.9 kg (1/3 to ½) - 10 – 14.9kg (1/2 to 2/4) - 15 – 29kg (2/4 to 1)
Over 30kg >1.

Phase 1 Rxn
 Hypoglycaemia (4 – 6 ml/kg of 5% dextrose)
 Anaemia (Transfuse withhold haematinics)
 Infection (UTI)= Antibiotics (beta-lactams)
 1 st Oral amoxicillin, 2nd c+ chloramphenicol or genticin (do not stop amoxicilin)
 Dehydration (ORS oral or NG)
 Hypothemia: Warmth clothing, draught free area, reduce bathing, KMC, Radiant warmer,
 Hot water bottle (strongly discouraged)
 Commence feeding
 RUTF, Fortified gruel
 Vitamins, folic acid, antihelminths
 Give high dose Vit. A (50,000IU, 100,000IU [1yr], 200,000IU [>1yr] (ideally only if they are give unfortified food. Day 1, 2 & 15
 100,000IU = 30,000ug = 3mg.

36
Compiled by Obasi. D. C. Chinedu

 May not require high dose of Vit. A if receiving F-75, F-100 or RUTF.
 Counsel caregiver on care

SAM & Diarrhea:


 Prone to Sodium retention. (risk of Na+ overload).
Rehydrate with ReSoMal, 5-10ml/kg/h up to 12 hours (Delivers 45 mmol/L Sodium, 40 mmol/L potassium, Instead of
75mmol/l Na+ and 20 mmol/l of hyposmolar ORS).
 except in cholera and profuse diarrhoea.)
 Half strength the WHO low-osmolarity ORS + K + glucose.
 Dissolve in 2L instead of 1L + 1 level scoop of mineral & vitamins mix + 50g of sugar.

Ready-to-Use-Therapeutic food; F-75, F-100 & HEMIX


 High energy, fortified ready to eat food.
 Four basic ingredients in RUTF: Sugar, Dried Skimmed Milk, Oil, Vitamin and Mineral Supplement (CMV). 25% product weight: oil
seeds, groundnuts, cereals.

RUTF attribute
 Taste and texture suitable for young children.
 No need for cooking before consumption.
 Resistance to contamination by micro-organisms and long shelf-life.

Nutritionally equivalent to F-100. 500 Kcal / 92 g satchet (520 -550Kcal)


Moisture content 2.5% max.
Protein 10 -12% total energy. Lipid content 40 -60 % total energy. Vit A 0.8 to 1.1 mg/100g
Na, K, Zn, Cu, Mg, p, etc. Other vitamins and essential fatty acids.
What is the dosing for 10kg baby?

Failure to response can be due to:


 Procedure, staff, equipment & environment
 Insufficient food given
 Vitamins or mineral deficiency
 Insufficient attention
 Rumination
 Unrecognized infection
 Underlying diseases.

Criteria for discharging children from treatment.


 From Inpatient to outpatient care.
 Resolved medical complication, not based on any anthropometric outcome (MUAC, W4H/L)
 Stable and Appetite has returned.
 Discharging from treatment.
 W4H/L ≥ 2 Z score & no oedema for at least 2 weeks.
 MUAC is ≥ 125mm & no oedema for at least 2 weeks.
 15% weight gain 2 consecutive weeks
 The anthropometric indicator used to confirm SAM should also be used to assess whether a child has reached nutritional
recovery.Should not based on resolved oedema alone.
 Percentage weight gain should not be used as a discharge criterion.

Other deficiency
 Iron deficiency.
 Fever inhibits GIT absorption.
 Reducing agent (vit. C) ferric => ferrous.
 To little in
 ↓ dietary intake, anorexia 20 infection, pica 20 already ↓ Fe ++ existing, delayed weaning)
 Too much out
 Hookworm infection, GIT blood loss, aetrogenic.
 Impaired absorb
 Pyrexia, 20 bacteria colonization of Jejunum.
 ↑ requirement

37
Compiled by Obasi. D. C. Chinedu

 Rapid growth, prematurity.


 Failure to utilize.
 Infection & chronic illnesses.

Stages of Fe ++ deficiency.
 ↓ Iron stores => ↓serum ferritin levels (<10µg/L)
 No clinical manifestation.
 Impaired Haemoglobin synthesis => ↑ RBC protoporphyrins ( >0.1g/dl), ↓ transferrin saturation (serum iron/TIBC), Normal Hb.
 Sustained ↓Fe ++ supplies to the BM => Hb ↓ => Iron def. anaemia.
 Hypochromic/microcytic, abnormal cell shapes.
 MCV < 75 fl, MCHC < 30g/dl (34 – 36), MCH < 25µg.
 Rx Oral Ferrous salts (sulphate, gluconate, fumarate)
 Elemental iron content: 6mg/kg/day.
 20% sulphate syrup (Astyfer 300/60, 10ml = 275mg) fesolate (dried) 200/65 tab.
 10% gluconate.syrup (Chemiron 300/35.)
 Reticulocyte response 48 – 72 hrs, peaks within 5 – 7 days.

Vitamins
Fat soluble
Vitamin A – Vision, Vitamin D – Bone calcification, Ca2+ homeostasis, Vitamin E – Antioxidant, Vitamin K - Clotting factors
Water soluble
Vitamin C- Antioxidant and Collagen synthesis, Folate - Blood and neural development. Cobalamin(B12)-Blood, CNS.
Metabolic
- Thiamine (B1) - Riboflavin (B2) - Niacin (B3) - Pantothenic acid (B5) - Pyridoxine (B6) - Biotin (B7)

Vitamin deficiency A: Deficiency of Vitamin A (Night blindness, dry skin)


Vit. A (Retinol, Retinoic, Retinal): Function: Antioxidant, Visual Pigment, differentiation of epithelial cells.
βCarotein , opsin & iodopsin bind to retinal (oxidized retinol ) retain CIS from in dark & TRANS form in light. Change from CIS to
TRANS. Releases energy which stimulate the nerve endings.
Keratinization of hair follicles, atrophy of sebaceous gland => dry, firm papules with protruding cornified plugs (follicular
hyperkeratinization)
Bone malformation & growth retardation, damaged tooth formation. decreased resistance to infection.
Required for normal growth, visual acuity & secretary epithelia.
 blood level < 10µg/dL ( 30 – 50)
 Night vision (nytalopia), impaired dark adaptation.
 xerophthalmia,(conjunctiva dryness)
 bitots spot (small triangular foamy or silvery white patches on the sclera lateral to the cornea).
 Xerosis (cornea dryness),
 keratomalasia (softening of cornea),
 Cornea scaring or blindness secondary to infection or trauma.
 Extrusion of lens or loss of vitreous secondary to panopthalmitis
 Phthrisis bulbae (shrunken useless eyes.
 Rxn: vit. A 50 -200,000iu; days 1,2 & 15.

Vitamin C: Antioxidant, keep iron in Fe2+, hydroxylation of proline & lysine, conversion of dopamine to NE.
 Scurvy – swollen gums, bruising, haemarthrosis, anemia, poor wound healing.
Vitamin D:
Sources
 D2: Ergocalciferol (plant)
 D3: Cholecalciferol (animal) sea foods and diary products & sun-exposed skin.
 Absorbed Vit. D is carried in the chylomicrons to the liver where it is converted to 25 (OH) vit D and stored in this form.
Denovo synthesis: Skin, Liver and Kidney cortex.
 In presence of UV light wavelength 296-310 nm, 7 dehydrocholesteryl synthesized in the skin => cholechalceferol. cholechalceferol
=> 25 Hydroxylcholecalciferol (liver)
 Transported bound to protein => kidney cortex 1,25 dihydroxycholecalciferol (Calcitriol), the active form.
 Phytate bind both 25 OH-D and Ca++ inhibits its absorption.
-Vit-D ↑ Ca++ absorption and phosphate.
 ↑ synthesis of Ca++ binding protein.
 Mobilizes ca++ from bone via osteoclastic activity.
38
Compiled by Obasi. D. C. Chinedu

 ↑ Renal resorp on of ca++.

 ↓ Serum Ca++ => ↑ Secretion of PTH => activation of 1α hydroxylase => ↑ Vit. D level.
 ↓ Phosphate also induces activation of 1α hydroxylase.
 Osteomalacia: lack of mineralization of growing bone. Failure of cartilage to complete their normal cycle of proliferation &
degeneration leads to Failure of capillary penetration which ultimately leads to Failure of osteiod & cartilaginous matrix to
mineralize.
 Nutritional
 Abnormal Vit-D metabolism
 Def. of 1α hydroxylase
 CRD
 CLD
 Anticonvulsants.

Clinical features
 Thickening on costochondrial joint (Rachitic rosary).
 Prominence of wrist and ankles, Bowing of legs, Growth failure, Cranotabes.
 X-ray findings
 Widening space between metaphysis & epiphysis.
 Cupping, widening & fraying of metaphysis.
 Rxn: 100µg (4000units) Cholecalciferol daily
 Evidence of healing 2 – 4wks

Vitamin E & K
 Vitamin E: Antioxidant: increase fragility of RBC, muscle weakness, demyelination.
 Vitamin K; ɣ- carboxylation of glutamic acid residues (Factor II, VII, IX, X, protein c & s)
 ↑ PT and PTT but normal bleeding me

Failure to Thrive (FTT):


It is an observation (Symptom or sign) not a diagnosis. Weight < 3rd percentile or 2 SD below the mean
 Influence: Genetic, Nutritional or Environmental. Peak incidence: 9 months to 24 years
 Poorer communities
 Single teenage mothers, working mothers with long working hours or Children in institutions

Causes: Under-feeding = commonest cause


Normal variants: Genetic (familial), constitutional
Nutritional: Poor feeding, emotional disturbances
Abnormal losses: Persistent vomiting (mechanical & metabolic disorders (galactosaemia, hypercalcemia), Malabsorption or DM, DI, CRF
Poor utilization: Chronic infecions

Evaluation
Careful history: Birth weight, nutritional, SES, parental height & weight, siblings,
Physical examination: weight., height, OFC (serial measurements; sudden & gradual), Cardiac examinations
Investigations: FBC, urinalysis, SEUCr, CXR, HIV, Stool analysis, Blood sugar, barium meal,

Treatment
 Proper counselling
 If any underlying cause = Rx

39
Compiled by Obasi. D. C. Chinedu

ANTHROPOMETRY: (NUTRITION AND GROWTH ASSESSMENT) AND SHORT STATURE


AgozieC. Ubesie

Learning Objectives
•To understand the concept and various methods of assessment in children
•Be able to carry out basic nutritional assessment in children.
•To appreciate the common causes of short stature in our environment

Introduction: Malnutrition: Cellular imbalance between supply of nutrients/energy and demand to ensure growth, maintenance, and specific
functions.
–Short-term affects weight (wasting) while Long-term leads to poor linear growth (stunting)

Nutritional assessment
•An in-depth evaluation of both objective and subjective data related to an individual's food and nutrient intake, lifestyle, and medical
history. Includes:
•Detailed dietary history, physical examination, and anthropometric measurements
•Basic hematological and biochemical indices may be included to identify specific nutrient deficiencies
•Careful measurement of growth status and reference to standard growth charts is essential.
•Skinfold thickness measurements not often calculated in routine clinical practice

Dietary assessment: Breastfed infants


•Frequency and duration on each breast –Positioning? –Attachment?
•Supplementary and other feeds?

Breastfed infant Formula-fed infants


•Type of formula –How is it reconstituted?
•Is each feed freshly prepared?, How many feeds in 24 hours and how often?. Volume of feed each time?
•How much is taken and how long does it take?. Anything else added to the formula?

Dietary/feeding history: Older children


•Questions regarding mealtimes, food intake and difficulties with eating should be a part of routine history taking.
•Gives an accurate and rapid qualitative impression of nutritional intake.
•There may be need for a more detailed quantitative dietary assessment in conjunction with a dietician

Older children… •Number of meals and snacks per day? What is the child being fed? How the parents describe child’s appetite?
•Family meal times? –Happy and enjoyable?
•Others: milk, juice

Dietary assessment methods: - 24-hour dietary recall - Food frequency questionnaire


•Food check list - A prospective 3-or 7-day record

24 hour dietary recall: –Listing of all foods and drinks consumed in a 24 hour period
–Includes approximate weight or size of portions and recipes - –Depends on memory
–Useful in assessing larger population groups
–Limited by day to day variations in consumption and errors in recollection

FFQ: –Predetermined list in a defined time period


–Reports number of food and drink portion consumed from the list –Protocol should define portion sizes
–No precise intake information –Useful in identifying dietary patterns such as exclusion of specific food groups

Food check list: –Similar to FFQ –Uses shorter food list • Targets specific food groups or nutrients
–Comprises of a list of foods; over the course of a day – Respondent makes a check beside a food each time she or he eats it
–Does not rely on memory

Prospective 3-or 7-day record –All food or drinks consumed are registered by child or caregiver
–More informative than 24-hour recall – Should include a weekend or festive day

1.Assess barriers to adequate dietary intake:


•Physical problems: chewing, swallowing, food consistency.
•Psychological problems: only eats certain foods? In particular places? Using particular plates etc
•Socio-economic problems: how much food is available

2. Assess barriers to absorption of nutrients


40
Compiled by Obasi. D. C. Chinedu

•Physical: diarrhea, vomiting, regurgitation etc


•Dietary: what food combinations are offered
•Physical activity: inactive child? Excessive or compulsive exercise?

Physical examination: •Subcutaneous fat: loss or excessive gain •Muscle wasting •Oedema–ankle, sacral
•Skin changes •Hair change –texture, colour •Nails •spooning in iron deficiency,
•transverse lines in protein deficiency •Angle of the mouth, gums, tongue •Eyes

Anthropometry: Scientific study of the measurements and proportions of the human body. Most universally applicable non-invasive,
inexpensive method of growth assessment. Accurate measurement/charting in identifying malnutrition.
For preterm infants, adjustment for corrected GA should be made up to 2 years of age by deducting the no of weeks born early from
actual age.

Growth monitoring: Serial measurements is required to monitor growth and the effectiveness of nutritional interventions

Anthropometric measurement
ADVANTAGES DISADVANTAGES
Objective with high specificity and sensitivity Inter-observererrors in measurement
Measures many variables of nutritionalsignificance Limited nutritional diagnosis
Readings are numerical and gradable on standard growth charts Problems with reference standards i.elocal vsinternational
standards
Readings are reproducible
Inexpensive and needs minimal training

Weight: Weigh children <2 years naked. Weigh older children only in light clothing. Self or regularly calibrated scales.
Length: Ideally, use an infant measuring board, measuring mat or rod. Requires 2 people
Height: Stadiometer preferred. Remove child’s shoes. Child to look straight ahead. Ensure that the heels, buttocks, and shoulder blades
make contact with the wall
Head circumference: Correct measurement is important. Mid-forehead and occipital prominence. Performed at every visit for patients <
3 yrs.
Use a non-distensible plastic measuring tape placed, If abnormal, document the HC of the parents/siblings
Normal HC: 2cm/month in the 1st3 months, 1cm/ month in the 2nd 3 months, 0.5cm/month in the second half of the 1st year. 2cm in the 2nd
year. 80% of adult size by the end of the 2nd year.

MUAC: Mark the mid-upper arm (half way between the acromion of the should and olecranon of the elbow). Use non stretch tape
measure at the mid-point of upper arm. Ideally take average of three readings.
Use left arm while hanging down the side of the body and relaxed. Based on the observation that MUAC does not change much in
children between 6 months and five years old. Low MUAC is an indicator for wasting. Can be measured easily, quickly and is affordable.
Values < 125mm and 115mm are used to define moderate and severe acute malnutrition respectively.

Shakir strip

Anthropometric indices:
•Weight for age - Length/ height for age -Weight
for height (WFH)
•Head circumference for age(OFC) - Mid
upper arm circumference (MUAC)
•Body mass index for age (BMI) - Skin
fold thickness

Weight for age: An indicator for underweight


Length/height for age: An indicator of long-term nutritional
deprivation. Often results in delayed mental development, reduced cognitive ability, poor school performance and low productivity.
Weight for height (WFH): Assesses wasting. A measure of acute malnutrition.
Body mass index: Weight divided by the square of the body height. weight (kg)/ height (m2)
Correlates closely with direct measures of body fat. An important tool in identifying childhood obesity

Interpretation…measurements: 1. Percentiles 2. Percent of the median


3. Z score or standard deviation Scores

41
Compiled by Obasi. D. C. Chinedu

1. Percentile charts: it tells the value of a variable below which a certain percentage of observations/populations fall. Position of an
individual on a given reference distribution. Widely used percentile include; 3rd, 5th, 50th85th, 95th, 97thcentiles. Its Advantage over z-
score is that its more understandable.

Nutritional status based on percentiles


NUTRITIONAL STATUS PERCENTILE CUT POINTS
Wasting WFH < 3rdcentile
Stunting HFA <3rdcentile
Underweight BMIfor age < 5thcentile
Overweight BMI for age >85thcentile
Obesity BMI for age ≥ 95thcentile

Percentile charts
•The WHO charts: Based on the physiologic growth of healthy breast fed infants across six countries.
•The CDC charts: Based on the growth of a population of children as they exist in a given location.

2. Percentage of the median: Is the ratio of the child’s measured value to the median value of a child of the same age in the reference data,
expressed as a percentage.
• (Measured wt of child/median wt of child) x 100
• Used in modified welcome and Gomez Classifications of nutritional status.
•Drawback: Gender non-specific.

Modified Wellcome classification


NUTRITIONALSTATUS % weight for age Oedema
Normal > 80 -
Kwashiorkor > 80 -<120 +
Underweight 60 -80 -
Underweight kwashiorkor 60-80 +
Marasmus <60 -
Marasmickwashiorkor <60 +

Nutritional status based on Gomez classification%


weight for age Nutritional status
≥ 90 Normal
75 -<90 1stdegree malnutrition
60 -<75 2nddegreemalnutrtion
< 60 3rddegree malnutrition

3. Z-score or SD
•Difference between the value of an individual and the median value of the reference population for the same age or height divided by the
reference population.
–Observed value –median reference value
Standard deviation of reference population
•Tells how far a child’s value is from the median value of a child of same age and sex in the same population.
•Has valid statistical significance. The 2006 reference z-score is recommended in the assessment of under-five children.

Nutritional status based on Z-scores


NUTRITIONAL STATUS Z-SCORE VALUES
Underweight Weight forage < -2 z score & > -3
Wasting/moderate malnutrition Weight for height < -2 z score & > -3
Severe wasting/ severe malnutrition Weight for height <-3 z score
Stunting Height for age < -2 z score
Normal values > -2 z score< + 2
Overweight Weight for height > +2 z score < +3
Obesity Weight for height > +3 z score.

Laboratory indices
Serum proteins are the most widely used laboratory measures of nutritional status. Negative acute phase reactants with reduced levels
during inflammation. In the absence of inflammation, a low concentration of these proteins correlate strongly with malnutrition.

42
Compiled by Obasi. D. C. Chinedu

Serum proteins include:


•Albumin, Prealbumin, transferrin, Retinol Binding Protein (RBP)

Lab assessment –serum albumin: Albumin has a half life of about 20 days and so not very effective in acute malnutrition.
Lab assessment –pre albumin: Total body pool of prealbumin is smaller than albumin. Half life of 2-3 days, Hence more reliable as a
marker of acute malnutrition. Degraded in the kidneys and levels are increased in renal insufficiency.

Lab assessment –serum proteins: •Transferrin


•Also an iron transport protein hence inaccuracies occur in iron deficiencies.
•Retinol Binding Protein (RBP) •Degraded in the kidney; increased in kidney abnormalities.
•RBP requires co-factors –Vit A and zinc

Lab assessment –Urinary urea nitrogen: Assesses a persons nitrogen balance. A positive balance means sufficient protein metabolism
from sufficient protein intake. A negative balance shows insufficient intake.
Lab assessment: •Specific nutrient assay –vitamins, minerals, trace elements.

Technical measurements
•May be required in addition to other assessment methods. -Resting energy expenditure (REE)
-Dual energy X-ray Absorptiometry -Bioelectric impedance analyzer (BIA)
Dual energy X-ray absorptiometry: DXA is a low energy X-ray technique that measures body composition. It measures bone mineral
content and density in children at risk of bone disease. Whole body DXA estimates lean body mass, fat mass and % fat in less than 5
minsand hence used in assessment for obesity. Values are compared to reference values. normal values are -2 to +2 z score

SHORT STATURE: Height well below the 3rd percentile for the chronological age. Itcan be Normal or Pathologic
Normal variant
•Familial short stature •Constitutional delay in growth and development.

Pathologic can be proportionate or Disproportionate.


Disproportionate: Skeletal dysplasia, Rickets Proportionate: can be prenatal or postnatal
Prenatal: IUGR, Placental disease, Infections, Dysmorphic syndromes, Chromosome disorders
Postnatal: Endocrine disorders, Nutritional, Gastrointestinal disease, Cardiopulmonary disease, Renal disease, Drugs, Chronic anaemia

Normal variant
1. Constitutional delay in growth and development(CDGD): –Modest delay in skeletal maturation
–A delay in pubertal onset in an otherwise healthy child –Hereditary tendency
–Growth curve usually follows the 3rdpercentile –BA delays corresponds to HA
–BA is within 75% of CA –Adolescent growth spurt > 14yrs girls and > 16yrs boys
–Final adult height and sexual development normal

2. Familial short stature: –Family h/o of short stature, Remain parallel to growth curve, Final adult height is small

Disproportionate short stature:


Skeletal dysplasia: achondroplasia,
hypochondroplasia, osteogenessis imperfecta,
metaphyseal chondrodysplasia.

1. Achondroplasia: limbs are short, upper-


lower body ratio is increased.
2. Rickets-limb deformity + other signs of
rickets.
–Upper body segment growth is normal
–Lower segment is shortened by at least 15%

Proportionate short stature:


1. Malnutrition: Most common cause of
growth failure worldwide.
–Inadequate intake
–Malabsorption
•Infectious and inflammatory bowel diseases

43
Compiled by Obasi. D. C. Chinedu

2. Somatic syndromes associated with obesity: –Laurence-Moon-Bardet-Biedl syndrome (polydactyly, retinitis pigmentosa and obesity),
Prade-Willi syndrome (hypognadism, hypomentia, hypotonicity, growth retardation and obesity)
3. Endocrine Disorders:
•Growth hormone deficiency– Incidence is 1in 40,000 children
•Hypothyrodism-decreases growth rate and skeletal development.
•Diabetes mellitus –poorly controlled.
•Glucocorticoid Excess –in Cushing’s disease or excess adrenal glucocorticoid intake
–reduced peripheral effect of somatomedin on growing cartilage or GH secretion
•Hypogonadism-a cause of growth retardation after age 10yrs in girls and 12yrs in boys.

•Others:
1.Gastrointestinal diseases:
1.Chronic infections, malabsorbtion, tropical sprueand cystic fibrosis, celiac disease and crohn’sdisease.
2. Cardiopulmonary disease:
3. Children with stage 3-4 asthma who are on oral prednisoloneof > 4mg/m2
–Inhalation steroid therapy does not predispose child to stunted growth
4. Chronic anaemia: –Sickle cell anaemia and thalassemia cause subnormal growth and delayed sexual development.
5. Renal disease: –under-nutrition, IGF-1 inhibitors, osteodystrophy and impaired synthesis of 25-2OH D3.

Diagnosis: •Is child is pathologically short statured? •Distressed because of his or her height that is not at the 50thpercentile?
•Tests are very expensive. Don’t request unless it is necessary.

Medical history: •Pregnancy •Birth size, mode of delivery & birth injury
•Assess mental and physical development •Symptoms and signs of systemic disease
•Abnormal diet •Family heights and ages
•When did pubertal maturation occurred? •Evaluate family relationship with the child

Physical examination •Height and weight plotted on the growth chart and compared with previous data if available
•Determine height velocity and compare with normal growth rate for age.
•Calculate mid parental height and adjust

Mid-Parental Height:
Boy: Cm: (Father's Height + Mother's Height + 13) / 2
Girl: Cm: (Father's Height -13 + Mother's Height) / 2
•Rough estimate of the child's projected height, without taking skeletal maturation or pubertal tempo into account
–Determined by extrapolating child's growth along his/her own height percentile to the corresponding 20-year point
•If estimated final height is within 5 cm of the MPH, the child's current height is appropriate for the family.
•If differs from the MPH by >5 cm
–Variant growth pattern or a pathologic cause
•Measure the parents' heights rather than reported height, to avoid over-or underestimation

Physical examination
•Determine arm span, the arm span minus the height is –3cm in the 1st7 yrs, near 0 from 8-12yrs and rises by age 14yrs to =+1 for girls and
+4 for boys.
•Also head circumference & upper to lower segment ratio, which is 1.7 at birth; 1.3 at 3yrs; and 1.0 at 7yrs of age
•Look for physical signs of syndromes
•Neurologic examination important

Diagnosis/Laboratory
•FBC •Urinalysis •Serum electrolytes Blood glucose •Serum calcium
•Serum phosphate •Alkaline phosphatase •Thyroid function test
•Serum prolactin •Cortisol •17-OH •Skeletal age determination
•CT-scan •karyotyping

Treatment
•Identify those that do not need treatment: Familial short stature, Constitutional growth delay. Reassure & follow up
•Treat those that can be treated e.g GH deficiency-SC recombinant GH daily, Hypothyroidism-L-thyroxine
•Support those that cannot be treated e.g Achondroplasia

Supportive care
Problems -
•Object of ridicule by classmates -school refusal
•Lack of satisfaction in life -depression
44
Compiled by Obasi. D. C. Chinedu

•Vocational achievement -difficulty in getting a job


•Marital status may be compromised

Supportive Measures-
•Normal IQ -encourage to go to mainstream school. May require special desk / chair
•Clothing -specially sewn especially for disproportionate short stature
•Light switches, sinks-may be out of reach
•Help with mobility-release contractures
•Advise about pregnancy-high risk of CPD=C/S
•Counsel parents -emotional deprivation will worsen case

•A person’s worth is not related to his stature but to the way he uses his talents
…After all, short stature ISNICE
•I Idiopathic (constitutional), Intrauterine
•S Skeletal dysplasia. Spinal -scoliosis
•N Nutritional, Nurturing -pyschosocial
•I Iatrogenic -steroids, radiation
•C Chronic diseases -CRF,CHD,SCD
•E Endocrine -Cushing’s, IDDM, GH deficiency, hypothyroidism.

45
Compiled by Obasi. D. C. Chinedu

ADOLESCENT HEALTH , PUBERTY AND GROWTH


BY DR ONUKWULI V.O

 What is Adolescence?
 Who is an Adolescent?
Adolescence is a developmental period through which normal human beings pass from childhood to adulthood. It is a period of rapid
biological, intellectual and psychosocial development.

An adolescent is a young person undergoing these developmental processes. Usually between the ages of 10-20 years. However, for
administrative purposes the age range of 10-24 years has been found functionally descriptive. Statistically, this group of people constitutes
40-60% of any population Cardinal goal of adolescence is preparatory for a more functional adulthood.

ENDOCRINOLOGICAL BACKGROUND OF ADOLESCENCE


There are 3 important groups of hormone:
-Adrenal hormone
-Ovarian oestrogen and progesterone and testicular hormone
-Growth hormone

The adrenal gland maturation occurs as early as 5-8 years. This results in the secretion of increased amounts of androstenedione.
Peripherally, this hormone is converted to a weak hormone, the oestrone(E1). Both hormones are functionally active and are responsible
for the initiation of the sexual maturation changes. Androstenedione is responsible for the growth and development of pubic hair
(adrenarche) in both sexes. oestrone(E1) is responsible for initiating thelarche(breast development) up to Tanner stage 2 or 3.

Subsequently, the more potent and more abundant ovarian hormone oestradiol(E2) takes over and completes thelarche in the female. The
Gonadotropin Releasing Hormone(GnRH) is secreted by the medial-basal hypothalamic neurons. GnRH is a decapeptide which is
responsible for the release of both follicle stimulating hormone(FSH) and luteinizing hormone(LH) from the pituitary gland.

Both FSH and LH are glycoproteins and are responsible for the growth and maturation of the gonads - ovaries in the female and testes in the
males. The release of these gonadotropins from the pituitary is initially pulsatile and sleep augmented. Later in adolescence, they get
released throughout wakefulness. The FSH level rises at about 9-10 years while the LH rises later at about 11-12 years in the female
and about 18 months later in the male.

The FSH is more active in the initiation of the granulosa cell development as well as the progression of the development of the secondary
sexual characteristics while the LH is responsible for luteinizing the glands in preparation for the reproductive function of adulthood. It
prepares the ovaries and uterus for menstruation and testes for ejaculation.

The ovaries produce three major hormones, the sex hormones: Oestradiol(E2), Progesterone and Inhibin.
Oestradiol(E2) is responsible for the completion of the growth and development of the female breasts as well as the other feminine
appearances. Progesterone is responsible for maintaining pregnancy. Inhibin selectively inhibits FSH
The testes produce testosterone which is responsible for the maturation of the penile shaft in addition to other muscular features.
Both Oestradiol and testosterone contribute to linear growth in either sex but at high levels, just before puberty ends, oestrdiol enhances
fusion of epiphyseal plates, halting growth.

Growth hormone is secreted in a pulsatile pattern in response to the interplay of two regulatory peptides which are produced by the
hypothalamus. Growth hormone releasing hormone (GHRH) serves to stimulate growth hormone production while somatostatin acts to
inhibit GH release.
A sleep-augmented rise in GH levels mediates the increase in plasma somatomedin-c (Sm-c) at puberty.
It is important to note that the stage and progress of adolescence relate more with bone age and secondary sexual characteristics
rather than chronological age. At the onset of puberty, the bone age is usually 11 years for girls and 12.5 years for boys. An increase in
physical size is a universally recognised event of puberty.
Average growth velocities decrease from the first year of life until puberty from 25cm/year during the first year of life to 5-6cm/year
during years 5-10. During puberty, the height velocity increases and peaks during the adolescent growth spurt

The physical changes that occur during the development of secondary sexual characteristics has been well documented by Marshall and
Tanner. There is a staging of what is otherwise a continuous change in adolescence into five using some constant somatic criteria.
These criteria are pubic hair quantity and configuration and breast size and configuration in the female.
In the male, use is made of both the pubic hair and testicular size with penile growth and development.
Each of these events is staged into 5 known as the tanner stages

Classification of Sexual Maturity rating in Girls (Tanner staging)

46
Compiled by Obasi. D. C. Chinedu

SMR Pubic Hair Breast


stage
1 Pre adolescent Pre adolescent
2 Sparse, lightly pigmented, straight medial border Breast and papilla elevated as small mound; alveolar
of labia diameter increased
3 Darker, beginning to curl, increased amount Breast and areolar enlarged, no contour seperation
4 Coarse, curly, abundant but amount less than in Areola and papilla form secondary mound
adult
5 Adult feminine triangle, spread to medial surface Mature, nipple projects, areolar part of general breast
of thighs contour

Classification of Sexual Maturity rating in Boys (Tanner staging)


SMR Pubic Hair Penis Testis
stage
1 None Pre adolescent Pre adolescent
2 Scanty, long, slightly pigmented Slight enlargement Enlarged scrotum, pink,
texture altered
3 Darker, stats to curl, small amount Longer Larger
4 Resembles adult type but less in Larger, glans and breadth increase Larger, scrotum dark
quantity, coarse, curly in size
5 Adult distribution, spread to medial Adult size Adult size
surface of thighs

Stage 1 generally represents the preadolescent stage.


Stage 2 of all these events represents the onset of puberty which heralds adolescence:
o in males, it is always the testicular enlargement that initiates puberty
o in females, either the breast bud (thelarche) or pubic hair (adrenarche) appearance can be the first sign.
Whichever appears first in the female depends on end-organ sensitivity. This can be from genetic or environmental differences as the
stimulus for both events is adrenal hormone.

The regularity with which these stages of maturation are associated with other observations in adolescence including behaviours gave rise to
the use of SMR in grouping adolescence into early, middle and late adolescence.
Early adolescence corresponds to SMR1 and 2, middle adolescence – SMR 3 while late adolescence corresponds to SMR 4 and 5.

In females, onset of puberty-Tanner stage 2, occurs at a mean age of 10.5 years (range 6-16). Completion of puberty - Tanner stage 5 is
at 15.5 years (range 11-18 years). In the male, it is usually about 18 months after similar event in the female. Mean duration of this
maturational process is 5 years.

Menarche
This refers to the onset of menstruation. The most dramatic and most easily recalled event of puberty in girls. Occurs in about 80% of
females in middle adolescence, 20% in SMR 3 and 60% in SMR 4. It marks the beginning of female reproductive life. A weak correlation of
this in the male is the 1st ejaculation, which signifies spermache (sperm production).

Age at menarche has shown some individual variability which may be related to genetic and environmental factors. It usually occurs only
when a critical weight of 22% occurs as a result of increase in fat deposit. Menarche signals that reproductive capacity is almost complete.
Progesterone activity is not always present at menarche, thus menstrual periods are often irregular. Most often, about 3 anovular
cycles within the 1st 18 post-menarche months can be recorded before it becomes regular. Normal cycles ranges from 3-7 days flow every
21-45 days.

Sexual Maturation
Adrenal oestrogen in both sexes gives rise to the embryonic differentiation of the internal organs and appendages as early as 14 weeks of
gestation. At adolescence, the ovarian oestrogen in the female is responsible for the adolescent sexuality. Testosterone does the same in the
males.
The vaginal mucosa thickens, and there is increased pigmentation and vascularisation of the clitoris. Body of the uterus enlarges, outgrowing
the cervix. The endometrium thickens in preparation for pregnancy or menstruation. The uterus is lined by columnar cells while the vaginal
mucosal cells are cuboidal. The cervical lining is left with transitional cells before full maturation.
Increased glycogen deposition with increased pH in the vaginal mucosa favours growth of Doderlein bacillus making the environment more
susceptible to yeast infections. Early sexual activity and STIs especially chlamydia organism can predispose girls to precancerous states,
cervicitis and PIDs – infertility in later years.

47
Compiled by Obasi. D. C. Chinedu

In males, testosterone and LH are responsible for the enlargement of the seminiferous tubules, epidydimis, seminal vesicles and the prostate
gland. This results from increase in the leydig and sertoli cells. Externally, the first manifestation is testicular enlargement (Tanner 2). This
occurs at about 12.5 years of age when it has a volume of 4ml; with the left usually lower than the right. Final adult volume of 25ml is
achieved at about 24 years.

Growth and development of the pubic hair and the penile shaft (in length 1st and then in breath follows). About 40-65 % of boys have mild
degree of breast hypertrophy called gynaecomastia. This results from relative excess oestrogen stimulation. Gynaecomastia sufficient to
cause embarrassment and social disability occurs in fewer than 10%. It is usually transient, tends to occur at SMR4. Breast swelling less
than 4cm in diameter has a 90% chance of spontaneous resolution within 3 years. Bigger enlargements may require further evaluation.
Other features include breaking of the voice, acne, growth of facial and chest hairs. Spermache (first ejaculation) occurs at about 14
years. Full physical maturity with full reproductive capability is often achieved by 18-20 years.

Physical Growth
Increase in the rate of physical growth occurs concurrently. Starts with rapid growth of hands and feet before the increase in linear growth
(legs before the trunk). This gives rise to the typical clumsiness of early and middle adolescence. Linear growth is mainly truncal with the
rate being almost twice the pre-adolescent rate constituting the growth spurt.

The growth spurt peaks at SMR4 in males and at SMR3 in females just before menarche. High oestrogen level at this time accelerates
epiphyseal fusion, retarding linear growth till it eventually ceases. Peak height velocity (PHV) can be as high as 8cm/year in the female and
10 cm/year in the male. Skin fold thickness shows an increase in fat content in the female at each stage of puberty/sexual maturation
with a total of 8-25% gain.

Factors that influence the onset of puberty


Socioeconomic class: The important factor here is nutrition, Children of the upper class tend to be bigger, taller and mature earlier
than their lower class counterparts
Chronic ill health: e.g Sickle cell anaemia, tuberculosis, chronic renal failure, severe mental retardation etc will delay onset and completion
of puberty
Intensive or strenuous physical exercise tend to delay puberty

Puberty is delayed if there is no sign of pubertal development by age 13 years in the female or 14 years in the male. It is precocious if
it occurs before 8 years in the female and 9 years in the male.

Statistically, the events of puberty are said to fall outside the normal if the chronological age of each event fall outside two standard
deviations of the normal for that group of people at that time.

Other factors such as stress in the child which could come from disturbed home environment, culture conflict, school, other sources of
isolation, discrimination or negative influences may either enhance or delay the onset of puberty.

Secular trend; a worldwide observation that children are growing taller, bigger and maturing earlier than their parents did. This is
attributed to better nutrition esp. protein intake as well as other environmental factors that provide better influence and expression of the
genetic component.

ASSOCIATED PROBLEMS IN ADOLESCENCE


Problems associated with adolescence:
These are conditions or circumstances seen either solely or largely at adolescence. They may also include those that originate at adolescence
and are problematic to them.
Adolescence is not necessarily a problem in itself. These conditions are peculiar to their stage of development. It is the physical changes or
behaviour towards these changes that characterise these problems especially during early adolescence. During middle adolescence, problems
revolve around independence and global sense of identity, thus high risk behaviour consequent on experimentation are common.

The spectrum of problems at adolescence include: - Developmental variants of normal


 Functional variants of normal - Psychosocial disequilibria
 Medical illness in adolescence - Miscellaneous issues

a. Developmental variants of normal: These are normal variants of physical or physiological development and includes

Early sexual maturation: Early breast development may be seen as “swellings” on the chest of young girls. Early scrotal enlargement
may also be termed “abnormal swelling”. Spontaneous ejaculation(wet dreams) in boys during sleep may be seen as bed-wetting or signs
of infection. Physical examination usually shows a normal child most often having an improved social environment than the parents.
Breast asymmetry: Common at puberty. Ensure there are no masses or discharge with or without tenderness. Masses when present are
usually benign fibro adenoma or cyst. Breast cancer is not common at this stage . Biopsy not indicated except when this is an obvious mass.
48
Compiled by Obasi. D. C. Chinedu

Physiologic leucorrhoea: Vaginal discharge usually scanty, clear, non offensive. Results from endometrial granular proliferation
following stimulation by oestrogen. Occurs just before menarche, by SMR 3. Microscopic examination shows no pathogen. Clinically,
there may be erythema resulting from conscious rubbing or masturbation. Usually requires no treatment. However, presence of abrasions
or tears may raise a suspicion of sexual abuse.

Masturbation: The art of fondling and rubbing ones genitalia or breasts to arouse sexual feeling and sometimes reach orgasm. Not unusual
during early and middle adolescence. Usually perpetrated by same sex partners. Both males and females may practise it. Normally transient
though some may continue into adulthood as homosexuals.

Irregular Menstruation: Immaturity of the hypothalamic-pituitary-ovarian pathway results in anovulatory periods. Irregular menstruation
are thus common in the immediate Post-menarcheal period. Oestrogen therapy is not needed as it normalises over time and periods
become more regular.

Gynaecomastia: Transient breast enlargement seen in up to 40-60% of male adolescents. In response to excess adrenal oestrogen
stimulation. Usually at SMR 4. Breast does not usually grow beyond Tanner stage 3 in size. Usually benign and resolves within 3
years. If it persists, other possible causes e.g. Klinefelters syndrome, liver cirrhosis or drugs may be considered.
Normally self-limiting but may occasionally require treatment with bromocriptine or surgical reduction.

Short stature: Complaints often come from male late developers. May have been growing normally with preadolescent peers. But had a
later growth spurt especially in relation to their same age females. May affect their ego and self-esteem with associated feeling of inferiority
complex. May also be familial/genetic or constitutional as in cases of chronic malnutrition.
Good history and physical examination is important. Investigations especially radiological determination of bone age. Reassurance is the
mainstay of treatment.
Orthopaedic Problems: Are mainly postural problems acquired during the rapid growth spurt period. May present as kyphosis, scoliosis,
or lordosis. Poor sitting position can also give similar problems. If uncorrected, may become permanent and give rise to functional
disturbance. Management will be based on patient’s age and curve progression which depends on age, sex, menarcheal status, curve location
and curve magnitude.
Investigation is mainly radiography with measurement of cobb angle. Curves with cobb angle of 25 degrees or less can be managed with
exercise, 25-45 degrees will require orthoses while above 45 degree surgery is indicated.

Acne: One of the most common physiological condition in adolescence. A dermal manifestation of the action of pubertal hormones on
sebaceous glands. Usually seen as papular or pustular lesions on the face and upper chest.
Exact pathogenesis is not clear. Action is known to take place in the pilosebaceous units ie the sebaceous glands, ducts and rudimentary hair
follicles found on the face, upper chest and upper back.
In patients with acne, the hair follicles are lined normally by sticky epidermal cells which cause faulty keratinisation. This predisposes the
follicle to plug formation. Under androgen stimulation, secretion of sebum from sebaceous glands with interaction of normal skin bacteria -
Propionibacterium acnes will incite inflammatory reaction. Excess androgen secretion, genetic predisposition, stress and some drugs like
steriods may exacerbate it.
Treatment include counselling and guidance on contributory factors as well as correct medications. Careless picking or popping may
exacerbate inflammation and lead to further scarring.
Topical steroids are not recommended. Pustular acne may require antibiotics.

b) Functional variants of normal


Amenorrhoea: This refers to the absence of menstruation. May be primary or secondary
Primary at the age of 16 years, in the presence of breast development or at the age of 14, in the absence of breast development.
Secondary amenorrhoea, if after regular and normal menstrual period, there is cessation of menses for more than 3 consecutive
periods. It is the commoner type and until proven otherwise, pregnancy is the likely diagnosis especially in sexually active females.
Other common causes include hormonal contraceptives, stress, exercise, weight loss/eating disorder (anorexia nervosa) and polycystic
ovarian syndrome.
Primary amenorrhoea is relatively rare and can be complicated to evaluate and manage.
Structural abnormality as in haematocolpos or congenital malformations, may also cause primary amenorrhoea.

Dysfunctional uterine bleeding(DUB): Normal menstrual flow lasts for 3 to 7 days every 21-45 days. Any abnormal pattern of endometrial
shedding not caused by an underlying pathological process. It is a diagnosis of exclusion. Anovular, painless, prolonged or heavy enough
to cause clinical symptoms is the Hallmark of primary DUB.
Immature hypothalamic-pituitary-ovarian axis is a common cause. Secondary DUB usually has significant pain associated with the heavy
bleeding. PCOS, hypothyroidism, vaginitis, cervicitis, salpingitis and coagulation disorders are other causes

Polycystic ovarian syndrome(PCOS): Fundamental problem is hyper insulinism and increased ovarian production of androgens
particularly testosterone. Excess testosterone causes premature follicular atresia, anovulation and systemic effects. Manifests clinically as
hirsutism, anovulation and either oligomenorrhoea or amenorrhoea, obesity and virility.
First line treatment is use of combination oral contraceptives to produce regular cycles. Weight reduction in obese adolescents. Anti-
androgens like spironolactone for hirsutism.Metformin enhances peripheral sensitivity to insulin and decreases insulin requirement.
49
Compiled by Obasi. D. C. Chinedu

Dysmenorrhoea: A common menstrual disorder seen in nearly three quarters of all female adolescents. Elaboration of prostaglandins from
falling progesterone levels produces uterine ischaemia and contractions. May be mild, moderate or severe.
Pains are typically sharp, spasmodic, supra pubic/abdominal pains and may radiate to upper thighs or back. Severe cases may be
accompanied by vomiting, diarrhoea, headache or fatigue.
Primary cases has no pelvic lesion. Secondary cases may result from PIDs, endometriosis, ovarian cysts and congenital anomalies. Treatment
is mainly with NSAIDs

Impotence: Normal young adolescents males can have unaroused erection especially at the early morning hours. Erection may also follow
masturbation and may lead to seminal emission (ejaculation). Absence of this experience can be a cause of anxiety and infertility later in life.
Appropriate investigation and treatment should be offered

C. Psychosocial Disequilibrium
Psychosocial development involves the entire range of personal development and social adaptability in adolescence. Parents/caregivers have
the duty of protecting the adolescent, to be able to survive this period and prepare them to thrive on their own. They should imbibe core
values of courage, sense of responsibility, cooperation and positive and high self esteem. They should develop an independent identity, a
philosophy of life, see the need to work and appreciate their emerging sexuality.
Achieving these through certain goals may be in positive or negative ways. Can determine adolescent health or risk behaviour which has
biomedical consequences. Can also eventuate in social and personal or psychological outcomes.
Complete psychological development is usually achieved towards the end of adolescence or even in adulthood long after physical and
reproductive development. This gap leaves the vulnerable adolescent with the potential for psychosocial disequilibria. A triad of family,
school/religion and peer group linked by the media presents the adolescent to the society.
The role of the family is very important and where it is difficult, peer pressure becomes the strong wind or storm. Problem arises when
wrong peer group influences a confused adolescent and association with gangs, cults or religious fanaticism may be unavoidable. This
negative influence on the unstable adolescent manifests itself with negative behaviours which will be injurious to the person and health of the
adolescent as well as the society. May be seen as youth risk behaviour which include alcohol consumption, smoking of cigarette and
marijuana, illicit use of drugs or sexual activities, poor school performance, delinquency etc.
One major problem here is that once established these behaviours are unlikely to be abandoned without alternatives that can
provide similar satisfaction.

d. Medical illness in adolescence: Endemic diseases are common in adolescence just like other members of the society e.g. malaria etc.
Chronic diseases like sickle cell disease, bronchial asthma, cardiac diseases - congenital or acquired, cerebral palsy, mental retardation,
seizure disorders etc. Some develop from childhood and continue into adolescence.
Others like feeding problems (anorexia nervosa and bulimia) tend to be peculiar to the adolescent or start in the late teenage years.

e. Miscellaneous issues: Legal issues considering a number of uncertainties.


Careful and thorough documentation is very important. Obtain a second opinion where necessary.
Protecting the adolescent’s interest should be primary. Confidentiality is very crucial and the adolescent needs the assurance at the onset to
volunteer sensitive information.
Parental consent is important for an adolescent below 18 years, since they are still dependent e.g. on issues such as finance.
Special legal issues on things like use of contraception, sexual abuse/rape, abortion, choice of profession/career should be handled carefully.
Some of them have strong legal implications.

Prevention/intervention strategies:
 These should be comprehensive and promise to yield greater success than those that are more limited in scope.
 The root cause of any problem should be identified and addressed.
 There should be effective programmes to reduce risk and promote protection
 Programs to be organised and patterned for multiple risk behaviours rather than focused on specific behaviour alone.
 Programs should acknowledge the importance or salience of social environment and incorporate it.
 Individuals alone should not be blamed nor held responsible for not removing the risk in their lives.
Health care for the adolescent
This should be holistic with strong family support. Health is not just the absence of disease or infirmity but the state of physical, mental,
emotional and social well being(WHO). Proper parent-child relationship will lead to stable character formulation and less effective
neighbourhood/peer influences.
Need for enlightenment of the adult caretakers and adolescents via
 Seminars, workshops, trainings etc. - Media-radio, magazines, television etc.
 Adolescent continuing education in schools, organisations. - Play therapy like songs, drama/play of film.
 Literature: short pictorials and to-the-point literature materials.

Youth centre under the supervision and management of a youth friendly body. Adolescent clinic to provide a specialised form of care. To be
run by an adolescent paediatrician. May be a referral clinic in a youth friendly setting.
Establishment of enterprises/gainful employment to help one establish a stable source of income and plans toward adult life

50
Compiled by Obasi. D. C. Chinedu

BY Dr Arodiwe Ijeoma

Outline
 Definition
 Pathophysiology
 Signs and symptoms
 Causes by age
 Management – Various options
 Communication
 Summary

Learning Objectives
1. Understand what HF is.
2. Identify a child with ccf
3. Apply the best treatment option

Definition
Congestive heart failure (CHF) refers to an inability of the heart to pump as much blood as required for the adequate metabolism of the body
tissues leading to a clinical state of systemic and pulmonary congestion.

Complex clinical syndrome: Structural or functional cardiac disorder that impairs the ability of the ventricle to fill with or eject blood

The clinical picture of CHF results from a combination of “relatively low output” and compensatory responses to increase it. It is a medical
emergency that should be looked for in every acutely ill child.

The cardinal manifestations are; 1. Dyspnea and fatigue. 2. Fluid retention: Pulmonary/peripheral edema. And 3. Cardiomegaly

Cardiac insufficiency state:


Prof. Eichna in 1959: Distinction between Circulatory Congestion (CC) & CCF
CC is 2° to Noncardiac causes and CCF is 2° Cardiac causes (Myocardial failure).
CC is the Hemodynamic disturbance associated with CCF; Removal of the Congestion relieves the symptoms. Includes so-called high output
failure; Aneamia, Thyrotoxicosis, Infections. AKA Circulatory Failure.
The term CCF is reserved for Circulatory congestion in which there is myocardial failure.” Basically, The implication is that in heart
failure there must be something structurally or functionally abnormal with the heart.

CCF is a clinical diagnosis. There is no “gold standard” laboratory test. A careful history is how one makes the diagnosis. There should be
some direct evidence of structural heart disease.

Classification:
1. Duration: Acute or Chronic HF
2. Ventricular involvement: RV HF, LV HF or Biventricular (congestive) HF. The latter is the commonest in infants and young children.
3. Cardiac function affectation: Systolic HF or Diastolic HF
CLASSIFICATION
NYHA Heart Failure Classification is not applicable
Ross Heart Failure Classification was developed for global assessment of heart failure severity in infants. Modified to apply to all pediatric
ages.
Modified Ross Classification incorporates: 1. Feeding difficulties, 2. Growth problems and 3. Symptoms of exercise intolerance

MODIFIED ROSS HEART FAILURE CLASSIFICATION FOR CHILDREN


Class I: Asymptomatic
Class II: Mild tachypnea or diaphoresis with feeding in infants, Dyspnea on exertion in older children
Class III: Marked tachypnea or diaphoresis with feeding in infants, Marked dyspnea on exertion and Prolonged feeding times with growth

51
Compiled by Obasi. D. C. Chinedu

failure
Class IV: Symptoms such as tachypnea, retractions, grunting, or diaphoresis at rest

Stages of Heart Failure


This newer staging scheme is very clinically oriented. Target therapy in a more focused manner toward specific subsets of patients. Patients
only progress forward in this schema, although occasionally patients may go from D to C.
Stages Definition

Stage A. Patients at risk of CCF. Eg: HTN, DM, CAD.

Stage B Pt has structural heart disease associated with the development of CCF but asymptomatic. Eg CHD,
DCM. Asymptomatic valvular heart disease.

Stage C Pts has current or prior sympt of CCF associated with underlying structural heart disease.

Stage D Marked symptoms of heart failure at rest despite maximal medical therapy and who require
specialized interventions. Eg: Mechanical circulatory device

NYHA Functional classification


Class I- asymptomatic. No limitation to ordinary physical activity.
Class II- mild limitation of physical activity. Unable to climb up stairs.
Class III- moderate/marked limitation. Shortness of breath on walking on flat surface.
Class IV- severe limitation. Orthopnoea, breathlessness even at rest. No physical activity possible.

Pathophysiology:
The Frank-Starling law, With ↑ Preload (LV EDV), healthy heart ↑ COP until a max. is reached and cardiac output can no longer be
augmented (Fig. 1 ). There is ↑ SV in the failing heart with ↑ed preload, But not the same level of maximal COP as normal heart. At this
point, Pulmonary Congestion develops: Tachypnea and dyspnea and Systemic congestion: Hepatomegaly. Incr JVP

There are three major determinants of Heart function


1. Preload: Volume of blood stretching the LV just prior to contraction . A measure of this Left Ventricular End Diastolic Volume
(LVEDV) is left ventricular filling pressure. (LVFP). According to the Starlins law of the heart, with increasing LVFP, stroke volume
increases until the critical LVFP (25mmHg) is reached, beyond which stroke volume will no longer increase. ( Fig 1 )
2. Afterload: This is the resistance or impedance against which the heart must contract. A measure of this is the peripheral vascular
resistance (PVR). Stroke volume decreases with increasing PVR , unlike with preload.
3. Contractility or inotropic state of the heart: This refers to the intrinsic ability of the heart muscle to contract independently of its
external load.
Positive inotropic agents - digitalis. Isoproterenol Adrenalin.
Negative inotropic agents- propranolol. cardiac insult, hypoxia, hypercapnia , acidosis
In response to increased load, Hypertrophy occurs that tends to normalize the load per cell. Myocytes elongate only and rarely divides.
Reprogramming of the cardiac myocytes occurs, resulting in a more fetal-like response leading to an increase in the size of the cardiac
myocyte, Renders the surviving myocytes a short-term structural and functional advantage.

Compensatory Responses (CR):


The failing heart activation 2 important neurohormonal mechanisms:
 The Sympathetic Nervous System (SNS) .
 The Renin-Angiotensin-Aldosterone System (RAAS).

Attempt to preserve COP and thus are beneficial initially, However Chronic stimulation is injurious to myocardial function.
1. SNS: ↑ in sympathetic tone: 2° to ↑ adrenal secretion Epinephrine and ↑ neural release of Norepinephrine.
Initial beneficial effects: ↑ HR and myocardial contractility with resulting ↑ in COP. However chronic stimulation leads to adverse
52
Compiled by Obasi. D. C. Chinedu

myocardial effects, Eg :↑ afterload, Hypermetabolism, Arrhythmogenesis, and Direct myocardial toxicity Catecholamines Toxicity
Possible Mechanism:
 Produce calcium overload
 Inhibition of the synthesis of contractile proteins.
 High levels depletes β-adrenergic receptors on Myocardial surface.
→ functional loss of the catecholamine-mediated positive inotropic response.
Clinically, Reduction of adrenergic stimulation by the use of β-adrenergic blockers Leads to Improvement

2. RAAS: ↓ blood flow to the kidneys leads to ↑ in Renin and ↑ Angiotensin II/ Aldosterone. This results in reabsorption of both water and
salt from the renal tubules. Angiotensin II may cause a trophic response in vascular smooth muscle (vasoconstriction) and myocardial
hypertrophy. Angiotensin II also promotes myocardial fibrosis.
Although adaptive by attempting to restore wall stress to normal. In CHF Angiotensin II plays a maladaptive role by initiating fibrosis and
altering ventricular compliance. Thus, Using β-adrenergic blockers and (ACE) inhibitors in the treatment of CHF are to block its maladaptive
roles.

When these determinants are deranged adaptive mechanisms are called into play. These include :
 Adrenergic mechanisms.
 Renal mechanism and Atrial natriuretic protein. (ANP & BNP).
 Stimulation of IGF and GH
 Ventricular dilatation/hypertrophy

ANP and BNP are hormones secreted by the heart/Brain in response to volume and pressure overload that increase vasodilation and diuresis,
Prevent inflammation, cardiac fibrosis and hypertrophy. But when these fail, myocardial decompensation result.

In the decompensation; Stroke volume is decreased, LVEDP is increased, Atrial pressure increased, Systemic venous congestion &
Pulmonary venous congestion occurs.

CARDIAC REMODELING: This is a Maladaptive cardiac hypertrophy as a result of expansion of the Myofibrillar components of
individual myocytes (new cells rarely form) leading to an increase in the myocyte/capillary ratio. This gives rise to activation and
proliferation of abundant nonmyocyte cardiac cells, some of which produce cardiac scarring and eventually produce a poorly contractile and
less compliant heart.

What is the Etiology of CCF? (Causes by age)


Etiology in Cong. Heart Dz: CCF by itself is not a diagnosis but a manifestation of an underlying anatomical or pathological cause affecting
the heart.

53
Compiled by Obasi. D. C. Chinedu

AT BIRTH: 1. Arrhythmia (SVT, VT, CHB) or 2. Critical MS, TA,HLHS.


NEONATES & INFANTS:
A. Ventricular Volume Overload: Eg: ASD, ECD, PDA, VSD , TGA , TAPVR.
B. Systemic outflow obstruction (Pressure overload): Eg: PS, AS, CoA.
C. Systemic inflow obstruction: Eg: MS, Pulm Veins obst, Cor-triatriatum
CHILDHOOD: AR, MR, MS, Pulm Veins obst.

Etiology in Normal Heart


AT BIRTH Eg: Anaemia, & Large AV fistula
NEONATES & INFANTS: Eg: Anaemia, Infection / Sepsis, Fluid Overload, Hypoglyceamia, Hypothyroidism, Asphyxial
Cardiomyopathy, DCM, Carnitine Deficiency, Hypertension, Arrhythmia, AV fistula,
CHILDHOOD: Eg: RF/RHD, IE, Anaemia, Arrhythmia, Renal failure, Cancer therapy, Cardiomyopathy, Hypertension. Kawasaki synd

Summary causes;- Severe anaemia*


- Severe pneumonia*
- Septicemia *
- CHD- ECD, VSD, PDA, TGA, TA *
- Acute hypertension e.g. AGN
- Supra ventricular tachycardia (Tachyarrhythmias)
- Congenital heart block
 - Endocrine disorders e.g. hypothyroidism, hyperthyroidism
 Arterio-venous fistulae.
*commonest underlying causes in Nigeria and most parts of the tropics.

CAUSES OF HF IN CHILDREN
CARDIAC
CHD: ● Excessive Preload ● Excessive Afterload ● Complex CHD
NO STRUCTURAL ANOMALIES: ● Cardiomyopathy ● Myocarditis ● Acquired valve disorders
● Hypertension ● Kawasaki syndrome ● Arrhythmia (Bradycardia or tachycardia)

NONCARDIAC
● Anemia ● Sepsis ● Hypoglycemia ● Diabetic ketoacidosis ● Hypothyroidism
● Other endocrinopathies ● Arteriovenous fistula ● Renal failure ● Muscular dystrophies

Highlights on Etiology: CHD


Onset of CCF varies predictably with the type of defect
1. Vol. OL lesions Eg: VSD, PDA, and ECD. causes of CHF in the first 6 mo. NOT before 6 to 8 wks because the ↑PVR.
2. ASD rarely causes CHF , BUT OP ASD may causes CHF in adolescent.
3. TOF do not cause CHF unless they have received a large a Gore-Tex interposition shunt (modified Blalock-Taussig shunt).

The age at onset of CHF is not as predictable as with CHD, but the following generalities apply:
1. Viral myocarditis tends to be more com < 1 year. occasionally in the NB period, with a fulminating clinical course with poor prognosis.
2. Kawasaki disease is seen in 1 - 4 yrs.
3. Acute Rheumatic Carditis causes CCF school- age children.
4. RHD usually Vol OL lesions MR, AR cause CHF in older children and adults.
These diseases are uncommon in industrialized countries.

Anaemia & DCM:


At any age, Low conc. of Hb may lead to a relative inability to degrade nitric oxide (NO) hypoxia leading to the vasodilation that is so
typical of high-output heart failure. Low blood pressure may in turn activate neuroendocrine activity. DCM may cause CHF at any age.
Cause is idiopathic.
Others: Infectious, Endocrine, or Metabolic disorders or Antineoplastic treatment (e.g., Anthracycline & Doxo).

54
Compiled by Obasi. D. C. Chinedu

What are the clinical manifestations (signs & symptoms) of CCF?


Clinical presentations is age dependent
In neonates, the earliest clinical manifestations may be subtle

Clinical Manifestations In Infants With HF:

 Feeding difficulties - Rapid respirations* - Tachycardia* - Cardiac enlargement*


 Gallop rhythm (S3) - Hepatomegaly* - Pulmonary rales - Peripheral edema*
 Easy fatigability.* - Sweating - Irritability - failure to thrive.
*common in older children

DIAGNOSTIC CRITERIA FOR CCF


1. Significant tachycardia – resting HR, no fever
> 160/min in infancy
> 140/min at 2 years
> 120/min at 4 years
> 100/ min at 6 years and above.

2. Significant Tachypnoea –resting RR, no fever


> 60 cycles/min, 0- 1 month
> 40 cycles/min, 1mth- 24 mths
>30 cycles/min, 2-5 years
> 28 cycles/min, 5-10 years
>25 cycles/min, 10 years

3. Cardiomegaly: Displaced AB in the presence of a normally located trachea. Normal location of AB-4th left intercostal space, mid-
clavicular line in < 4yrs, 5th LICS, MCL in > 4yr. Cardio-thoracic ratio above the upper limit of normal for age:
 60% for children under 5 years
 50% for children over 5 years.
4 Tender hepatomegaly of at least 3cm below the right subcostal margin in the mid- clavicular line.
N.B—A clinical diagnosis of CCF can be made on the basis of presence of at least 3 of the criteria, one of which must be criterion 3.

Clinical Features of HF:


Systemic venous congestion results in:
- elevated jugular venous pulsation - Hepatomegaly: tender and rapidly enlarging.
- Pedal oedema usually absent in infants and young children . Why?
- Peripheral cyanosis
Pulmonary venous congestion result in:
- cough, tachypnoea, dyspnoea, grunt/ wheeze if severe , basal crepitations , recurrent apnoea in the newborn
Low cardiac output and increased adrenergic drive leads to :
- tachycardia, apnoeic spells in new born,
- restlessness, cold extremities, cloudy sensorium
- Poor peripheral pulses / pulses alternans
Fluid retention causes
-abnormal weight gain
- decreasing urinary output
- Feeding difficulties & increased fatigability

Important clue in detecting CHF in infants: Often it is noticed by mother, Interrupted feeding (suck- rest -suck cycles). Infant pauses
frequently to rest during feedings. Inability to finish the feed, taking longer to finish each feed (> 30 minutes). Forehead sweating
during feeds –due to activation of sympathetic nervous system –a very useful sign. Increasing symptoms during and after feedings.

55
Compiled by Obasi. D. C. Chinedu

Tachycardia: Rate is difficult to evaluate in a crying or moving child. Tachycardia in the absence of fever or crying when accompanied
by rapid respirations and hepatomegaly is indicative of HF. Persistently raised heart rate > 160 bpm in infants. > 100 bpm in older
children. Consider SVT if heart rate > 220 bpm in infants and > 180 bpm in older children.

Cardiomegaly: Consistent sign of impaired cardiac function, secondary to ventricular dilatation and/or hypertrophy. On CXR a CT ratio of
> 60% in the newborn and > 55% in older infants with CHF is the rule. May be absent in early stages, especially with myocarditis,
arrhythmias, restrictive disorders. Noted as a displaced AB.

Hepatomegaly: Lower edge of the liver is palpable 1 to 2 cms below right costal margin normally in infancy. In the presence of respiratory
infection increased expansion of the lungs displace liver caudally. Usually in such circumstances the spleen is palpable. Hepatomegaly is a
sign of CHF . Decrease in size is an excellent criterion of response to therapy

Pulmonary crepitations (rales): Of not much use in detecting CHF in infants. Rales may be heard at both lung bases. When present
difficult to differentiate from those due to the pulmonary infection which frequently accompanies failure

Peripheral edema: Edema is a very late sign of failure in infants and children. Presacral and posterior chest wall edema in young infants. It
indicates a very severe degree of failure. Daily wt monitoring is useful in neonates -- rapid increase in wt > 30 gm/day may be a clue to
CCF and is useful in monitoring response to treatment.

Severe CCF: Cold extremity, low blood pressure, skin mottling are signs of impending shock. Pulsus alternans (alternate strong and
weak contractions of a failing myocardium). Pulsus paradoxus (decrease in pulse volume and blood pressure with inspiration) are frequently
observed in infants with severe CHF

Investigations:
1. PCV. WBC total and differential: baseline, exclude anaemia and sepsis possibly.
2. Electrolyte and urea: baseline and to exclude hypokalaemia
3 Serum calcium: to exclude hypercalcemia
4. Chest radiograph; probably diagnostic e.g. from typical heart shape, Underlying lung pathology, Cardiomegaly

Principles of Treatment of HF: Aim of treatment is to reduce the load on the failing heart and to restore myocardial contractility to normal.
A. To reduce preload
 Diuretics : fast acting diuretics e.g. frusemide. Ethacrynic acid plus supplemental potassium, if more than one dose of diuretic
required.
 Reduce water intake 2/3 maintainance.
B. To reduce after load – only in intractable cases
- arteriolar vasodilator e.g. hydralazine,
- ACEI: Enalapril 0.1-0.5 mg/kg/day
- Captopril 6 mg/kg/day in divided doses.

C. To improve cardiac performance


- Digitalis: Prototype of which is Digoxin: +Ionotropic & Neg Chronotropic
Note: Safety precautions and side effects, ECG features of effect and of toxicity. NOT indicated in Anaemia HF?
- Dopamine preferred in infants with severe distress or those with renal compromise.
- Recently, low-dose β-adrenergic blockers has been added, with encouraging results.

Digitalis: oral: 0.04-0.06 mg/kg/day, I.V.: 70-80% of oral dose. Because of varying metabolism, appropriate dose varies by age.
Rapid digitalization
- May be performed over 12-24 hours.
- Calculate TDD administer 1/2 of TDD, followed by 1/4, then 1/4 of TDD
- Maintenance digoxin is approximately 1/4 of TDD, divided b.i.d., or 0.1 mg/kg per dose b.i.d.
Case example: patient weight is 5.5 kg. What is the TDD

Digitalis effect: Prolongation of PR, shortening of QTc, sagging of ST segment, slowing of heart rate

56
Compiled by Obasi. D. C. Chinedu

Digoxin toxicity:
- GI symptoms are common presenting symptoms: nausea, vomiting, anorexia, colour blindness (older child)
- Most common sign of cardiac toxicity is arrhythmia: bradycardia, AV block, PVCs
- Treatment includes withholding doses. Atropine for sinus bradycardia.

Digoxin toxicity: prolongation of PR, sinus bradycardia, 2nd degree heart block, supraventricular arrhythmias, ventricular bigeminy/
trigeminy (rare in children), premature ventricular contractions, ventricular tachycardia.

MEDICATIONS DOSAGES
Diuretics: Furosemide (Lasix); 0.5-1.0 mg/kg/dose, Chlorothiazide (Diuril); 20-50 mg/kg/day, Spironolactone (Aldactone); 1-2
mg/kg/day
Afterload reduction: Captopril (Capoten); 1-6 mg/kg/dose t.i.d, Enalapril (Vasotec); 0.1 – 0.5 mg/kg/day
Beta-blocker: Propranolol 2-4mg/kg/day, Carvediolol 3.125-6.25mg/day

D. Treat specific underlying cause(s) e.g pneumonia, Aneamia, or CHD


E. Provide general supportive measures:
1. Bed rest in A cardiac position (semiupright position) to relieve respiratory distress.
2. Intranasal oxygen and 40-50% humidity. Oxygen tents are ideal and far more effective than nasal catheters.
3. Daily weighing?
4. Nasogastric tube feeding of fortified feeds
5. Prostaglandin EI- to ensure patency of PDA in duct-dependent CHD e.g. HLHS, CoA.

When surgically feasible, OHS for underlying CHD and valvular heart disease is the best approach for complete cure.

SURGICAL AND DEVICE THERAPY:


- Pacemaker and implantable defibrillator therapy
- Biventricular pacing
- Ventricular assist devices
- Heart transplantation
GENERAL MEASURES: Calories
Adequate calories should be provided to permit appropriate weight gain. Infants in CHF need significantly higher caloric intakes than
recommended for average children. The required caloric intake may be as high as 150 - 160 kcal/kg/day for infants in CHF.

?? Adequate calories: Cannot take in needed calories: tachypnea, increased work of breathing, diminished strength of sucking, and
difficulty with coordination of sucking and swallowing.
a. Increasing caloric density with fortification of feeding
b. Frequent small feedings are better tolerated than large feedings in infants.
c. If oral feedings are not well tolerated continuous (NGT) feeding is indicated.
Communication: In dealing with parents, it is preferable to use words like “pulmonary congestion”, “liver congestion” rather than ‘heart
failure”, since “heart failure”, is likely to be misunderstood by the parents and this may hamper useful interaction.

Prognosis: Depends on
1. The underlying cause(s): Acute HF due to severe Anaemia or Pneumonia if well treated has a better prognosis than that in a child with
CHD/AHD which tends to recur.
2. Early diagnosis and treatment: Non-recognition or delayed diagnosis of HF is associated with increased mortality in emergency paed
units.

Summary:
- Heart failure is a complex clinical syndrome but relatively straightforward to diagnose.
- Heart failure implies underlying structural and functional changes in the heart that contribute importantly to the clinical
syndrome.
- The pathophysiologic principles such as reduced preload, afterload and augmentation of contractility is employed in its treatment.

57
Compiled by Obasi. D. C. Chinedu

Review Questions:
1a. List 6 possible causes of heart failure in childhood.
b. Outline the pathophysiology of heart failure.
c. Discuss the management of heart failure in an 18mo old child.

- Dr Arodiwe ijeoma
OUTLINE: - Introduction - Brief review of ECG - Abnormality of Rate - Abnormality of Rhythm
 Conduction abnormality - Summary

Learning Objectives:
 Recognize common pediatric cardiac arrhythmias
 Recognize early signs of clinical decompensation/hemodynamic instability
 Initiate management of arrhythmias in the inpatient setting

Introduction: Arrhythmias refers to the Abnormality of Rate, Rhythm & Conduction of the electrical activity of the heart. It can be:
- Physiological or Pathological,
- Congenital (WPW) or Acquired,
- Self-limited or life threatening.
For all of which ECG is essential in the diagnosis (Physiology of
conduction system)

Categories:
Sinus node (Rate) disorders: Sinus arrhythmia, Sinus Brady & Sinus
Tachy.
Disorders of Rhythm: Ectopic Premature beat (PAC, PVC), Atrial
flutter, Atrial Fib & Ventricular Fib
Conduction disorders: Heart blocks (1st -3rd degree)

CONVENTIONAL LEADS:
Limb leads (frontal plane): I, II, III, aVR, aVL, aVF
Chest (Precordial) Leads (horizontal plane): Adults: V1, V2, V3, V4, V5 and
V 6.
Children: V1 to V6, + V3R and/or V4R

Placement of precordial leads: V1


: 4th R intercostal space, parasternal
- V2 : 4th L “ “ “
- V3 : exactly mid way between V2 and V4
- V4 : 5th L intercostal space, MCL
- V5 : same transverse level as V4, AAL
- V6 : “ “ “ “ “ V4, MAL
- V3R : corresponds to V3 on the Right side
- V4R : “ “ V4 “ “ Right side

- V1, V2, V3R and V4R - R ventricular activity


- V3, V4 (transitional) - septal activity
- V5, V6 - L ventricular activity

COMPONENTS OF THE STANDARD ECG


In the standard ECG recording:

58
Compiled by Obasi. D. C. Chinedu

- Paper speed = 25mm/sec


- 1 small square (horizontal) = 0.04 sec
- 1 large square (horizontal) = 0.2 sec
- 10mm (vertical) = 1 mV
- (Standardization)

STANDARD ECG RECORDING (See Image)

COMPONENTS OF ECG
- P wave = atrial depolarisation
- QRS complex = ventricular depolarisation
- T wave = ventricular repolarisation
- PR interval = atrioventricular (AV) conduction
- QRS interval = ventricular depolarisation time
- QT interval = duration of ventricular systole (depolarisation + repolarisation)
ECG analysis for arrhythmias: Analyse for Rate, Rhythm, PR Interval.
What are the Sinus node (Rate) disorders?
Normal ranges of HR
AGE Range(beat/min )

Birth 120 - 180

1-6mos 120 - 140

6-12mos 110 - 130

1-5years 100 - 120

6-10 years 90 - 120

10 – 15 years 60 -100
RATE: Age dependent – faster at birth, gradually slowing with increasing age thereafter.
How do we Calculate Rate?
1. Divide 300 by the number of large squares in successive R – R interval.
2. Divide 1500 by the nos of small squares
3. Multiply the number of R-R cycles in 6 large squares (1.2 sec.) by 50.

Sinus Arrythmia: It is the most common irregularity of heart rhythm seen in children. Normal variant of beat to beat variation. Reflects
healthy interaction between autonomic, respiratory and cardiac control activity in CNS. Heart rate increases during inspiration and decreases
during respiration. It requires no treatment.

Sinus Bradycardia – Symptoms: HR is lower than the expected for age. Normal in Athletes/during sleep.
General: Fatigue, lightheadedness, dizziness, syncope.
Hemodynamic instability: Hypotension, poor end-organ perfusion, respiratory distress/failure, sudden collapse/death

Sinus bradycardia – Causes:


1º Abnormal pacemaker/conduction system (congenital or postsurgical injury), cardiomyopathy, myocarditis
2º: Reversible Hs & Ts:

59
Compiled by Obasi. D. C. Chinedu

Hypoxia, Hypotension, H+ ions (acidosis), Hypothyroidism, Hypothermia, Hyperkalemia


Trauma (to the head) ↑ ICP, Toxins/drugs (Ca++ channel blockers, β-adrenergic blockers, digoxin, opioids)
MGT:
Asymptomatic: No treatment
Symptomatic: Chronotropic drugs (Atropine 0.1 – 0.5ug/kg, Isoproterenol). And Cardiologist evaluation

Sinus Tachycardia (ST): HR > upper limit for age but not more 220 bpm (SVT)
Causes –
- Physiological compensation rapid discharge from S.A: Crying, Pain, Anxiety and Exercise.
- Pathological events: Fever, shock, fever, hypoxia, H.F, anemia.
-Drugs: Atropine, adrenaline or theophylline.

ECG shows a discernable P wave followed by a QRS. While in SVT the P wave is not recognizable. Treatment is directed towards the
cause.

Supraventricular tachycardia (SVT): Most common in pediatric practice. Requiring treatment. HR > 220bpm. Commonest has an
assessory pathway in the atria. (Retrograde activation of the atria). Impulse comes from above the ventricles and conducted normally.
Presentation: 1st Infancy , 2nd peaks @ in adolescense
Rapid, regular, normal narrow QRS, P wave not discernable but 1:1 with QRS.
Causes: Unknown, WPW Syndrome, CHD: Eg Ebstein”s anomaly, cTGA

SVT Symptoms:
Older kids can describe a sensation of a fast heart rate, palpitations, or chest tightness. Rarely present in CHF?
Infancy: Poor feeding, irritability, tachypnea excessive sweating, CHF.

SVT Treatment: The goal is to identify unstable patient and terminate the rhythm. Options includes:
1. Trial of Vagal maneuvers in stable patients: Massage of carotid sinus (one side at a time), Gag reflex or using ice park on the face 15-
20s. Parasympathetic stimulation ↓ HR
2. DOC: IV Adenosin: stops conduction through AV node. Give 0.1 mg/kg (max 6 mg), repeat 0.2 mg/kg (max 12 mg) with Saline flush.
Digoxin give TDD stat. IV preferable (Slow onset)
3. Critically ill: DC Cardioversion with 1-2 j/kg

SVT – Treatment: WPW syndrome ( 25 % pts with SVT)


Radiofrequency catheter ablation of the accessory pathway: Frontline treatment, Very effective (total cure). Cutoff points usually are 5 yrs
and 15 kg, unless severe SVT.
Long term Medications: Digoxin 0.05mg /kg for about 1 yr. and beta blockers as first line. Amiodarone

Electrophysiology(EPs)/PMs: for Arrhythmias Unresponsive to drug.


1. 1°or 2°(postop) arrhythmias.
2. SVT: High-frequency ablations of aberrant /Automatic ectopic atrial foci.
3. Highly sophisticated miniaturized PMs with long life span for the smallest infants with heart block.

What are disorders of Rhythm?


They are Ectopic (Premature) beat that Originate from other site other than the SA. Occur earlier than expected on ECG. Presents as regular
irregular/ irregular irregular beat
Types: They include;
a. Premature atrial contraction (PAC) b. Premature Vent. contraction (PVC)
c. Atrial flutter/ d. Atrial Fibrillation e. Ventricular Fibrillation

a. PAC: P wave is different from a sinus P wave followed by a normal PR & QRS complex. Benign: Stop with exercise. Some caused by
structural heart dz (SHD)/ Digoxin. No treatment needed except caused by SHD
b. PVC: No P wave with Abnormally wide ORS and inverted T wave. Could be Unifocal or Multifocal. Unifocal has same ORS

60
Compiled by Obasi. D. C. Chinedu

morphology. Bigeminy: Sinus beat followed by PVC, repeating as a pattern. Treatment: Unifocal requires none. Multifocal: IV
Lidocaine, Propranolol
c. Atrial flutter: characterized by Atrial rate of 200-300bpm, with regular and undulating wave forms stimulating “sawtooth”. Variable AV
conduction 2:1,3:1,4:1 block resulting in slow ventriclar
Causes: DCM, Myocarditis, Digoxin toxicity Post Surgery: Fontan, Atrial Switch (TGA)
Treatment: Digoxin (toxicity) Withhold drug, use DC cardioversion to NSR. Add Propranolol after TDD. Maintain with Digoxin

Atrial fibrillation: Rare in children. Atrial rate so fast > 450bpm irregular irregular. The P wave is not easy to discern but the ORS complex
is normal
Causes: Severe MS with Dilated LA, & Hypertrophic subaortic stenosis
Treatment: Digoxin Add Propranolol after TDD. Maintain with Digoxin . DC cardioversion to NSR

Ventricular Fibrillation: Bizarre ORS Complexes. With waxy lines of varying sizes. Rapid Rate with no palpable peripheral pulses:
Terminal event in patient with cardiac arrest
Causes: long QT syndrome, Brugada syndrome – inherited Arrhythmia, Autosomal Dominant. Cardiomyopathies, SHD causing ventricular
dysfunction. Child goes into V-fib, faints, sudden death
Treatment: Immediate DC Defibrillation, Cardioversion to NSR. Start CPR. IV Lidocaine or Procainamide. Careful screening

What are the conduction disorders? (CDs)- Heart blocks (1st -3rd degree)
In CDs there is a Delay or Interruption in transmitting the impulse from the SA node to the AV – node or to the terminal branches
ECG Shows: Prolonged PR interval. They can be:
a) 1st degree heart block
b) 2nd degree heart block: - Mobitz type I (Wenckebach). - Mobitz type II
a) 3rd degree (complete) heart block (AV Dissociation)

PR INTERVAL: The normal is age and heart rate dependent. Usually


0.08 – 0.16 sec Infancy - 1 year of age ECG = (2- 4 small squares)
0.10 - 0.18 sec older child ECG = 2.5 – 4.5 small sq

First degree Block: PR interval > normal for age. The ECG is otherwise normal, No heamodynamic consequence.
Causes: ARF, Myocarditis, CHD: ASD, Ebstein’s anomaly and digoxin toxicity
Treatment: Resolves with treating the cause.

Second Degree HB: can be Mobitz I (Wenckebach Phenomenon) or Mobitz II.


Mobitz 1: There is a progressive prolongation PR, Untill the P wave is not conducted = dropped ventricular beat. Benign
Mobitz type II: There is an unpredictability about the dropped P wave. So called “All or none” phenomenon. The ECG has normal PR
interval & QRS complex or it is completely blocked. It is serious than type one and can progress to 3rd dgr. Rare in children & needs
Cardiologist evaluation

Third Degree HB: Complete HB. There is a complete dissociation between the P wave and the QRS complex. The atrial rate and the
ventricular rate are regular but different with the latter being slower.
Causes: L-TGA, Infant of mother with SLE, Surgically induced HB,
Presentation: Asymptomatic or in CCF
Treatment: Treat the cause, Epicardial/Transvenous Pacemaker insertion

61
Compiled by Obasi. D. C. Chinedu

THYROID DISORDERS
Prof Herbert A Obu, MBBS, FWACP (Paed)

Outline
 Introduction
 Aetiology of thyroid disorders
 Hypothyroidism
 congenital
 acquired
 symptomatology, diagnosis, treatment and prognosis
 Endemic goitre/cretinism: epidemiology, symptomatology, treatment and control
 Hyperthyroidism: causes, presentation, diagnosis, treatment.

Introduction: The main function of the thyroid gland is the synthesis of thyroid hormones, T4 & T3. They are formed in the colloids
(lumen) of the thyroid follicles by iodination tyrosine. The thyroid hormones play essential role in metabolism through; increase oxygen
consumption, stimulate protein synthesis, influence growth and differentiation, affect carbohydrate, lipid, and vitamin metabolism.

Of the thyroid hormones, T3 is the most metabolically active thyroid hormone, metabolic potency 3-4 times that of T4. However, its level
in blood is about 1/50th that of T4. Only 20% of circulating T3 is secreted by the thyroid gland; the remainder is produced by de-
iodination of T4 in the liver, kidney and other peripheral tissues by type 5'-deiodinase. Iodine is necessary for synthesis of these
hormones;

Thyroid tissue has avidity for iodine - able to trap (with a gradient of 100:1), transport, and concentrate it in the follicular lumen for synthesis
of thyroid hormones. Trapped iodide only reacts with tyrosine when in the oxidized form; this reaction (oxidation of trapped iodide) is
catalyzed by thyroidal peroxidase.
The thyroid cells elaborate a specific thyroprotein, a globulin with approximately 120 tyrosine units (thyroglobulin). Iodination of tyrosine
forms monoiodotyrosine (MIT) and diiodotyrosine (DIT); coupling of these result in T3 or T4 as the case may be. Once formed, the
hormones are stored as thyroglobulin in the colloid until needed. T4 and T3 are liberated from thyroglobulin by activation of proteases and
peptidases.

About 70% circulating T4 is bound to thyroxine-binding globulin (TBG). Less important carriers are thyroxine-binding prealbumin,
called transthyretin, and albumin. Only 0.03% of T4 in serum is not bound and comprises free T4.
Approximately 50% of circulating T3 is bound to TBG, and 50% is bound to albumin; 0.30% of T3 is unbound or free T3. Because the
concentration of TBG is altered in many clinical circumstances, its status must be considered when interpreting T4 or T3 levels.

The thyroid is regulated by TSH, a glycoprotein produced by the anterior pituitary. TSH activates adenylate cyclase in the thyroid gland
to effect release of thyroid hormones. TSH synthesis and release stimulated by TRH synthesized in the hypothalamus and secreted into the
pituitary. TRH is found in other parts of the brain besides the hypothalamus and in many other organs; aside from its endocrine
function, it may be a neurotransmitter. In states of decreased production of thyroid hormone, TSH and TRH are increased. Exogenous
thyroid hormone or increased thyroid hormone synthesis inhibits TSH and TRH production. Except in the neonate, levels of TRH in
serum are very low.

Further control of thyroid hormone levels occurs in the periphery. In many nonthyroidal illnesses, extrathyroidal production of T3 decreases;
factors that inhibit thyroxine-5'-deiodinase include fasting, chronic malnutrition, acute illness, and certain drugs. Levels of T3 may be
significantly decreased, whereas levels of free T4 and TSH remain normal. Presumably, the decreased levels of T3 result in decreased rates
of oxygen production, of substrate use, and of other catabolic processes.

Although thyroid hormones are essential for normal post-natal growth, the situation in foetal life is less clear. Active T3 is not found in the
human foetus until about 30 weeks gestation. Paradoxically, although the foetus with thyroid aplasia might have mild skeletal immaturity,
foetal growth appears normal despite the fact that thyroid hormones do not cross the placenta in significant amounts. Thus the role of thyroid
hormones in foetal growth remains a subject of speculation.
Note that the amount of T4 that crosses the placenta is not sufficient to interfere with a diagnosis of congenital hypothyroidism in the
neonate.

Aetiology of thyroid disorders


A. Hypothyroidism: May be congenital or acquired.
Congenital causes include:
1. Dev. anomalies eg. aplasia, hypoplasia or ectopia. Occurs sporadically, about 80% of children with congenital hypothyroidism.
2. Defect in thyroid hormone synthesis 10-15%. This can be from; Thyroid oxidase mutations, Iodide transport defect, Thyroid peroxidase
defect, Thyroglobulin synthesis defect, Deiodination defect.
3. Iodine deficiency (endemic goiter/endemic cretinism): Can be of the Neurologic type or Myxedematous type

62
Compiled by Obasi. D. C. Chinedu

4. Maternal antibodies: e.g: Thyrotropin receptor–blocking antibody (TRBAb) (or thyrotropin binding inhibitor immunoglobulin)
5. Maternal medications like Radioiodine, iodides, Propylthiouracil, methimazole, Amiodarone.
6. Hypothalamic-Pituitary Disorders: Eg. Septo-optic dysplasia, Isolated TRH deficiency, Cleft lip and palate midline syndromes

Acquired causes Includes:


1. Thyroiditis: Hashimotos, Post-viral, Sub-acute
2. Hypothalamic-pituitary disorders: Trauma, Post-irradiation, Tumours
3. Iodine deficiency
4. Post-thyroidectomy
5. Drugs eg. Carbimazole
6. Associated syndromes: down’s, diabetes mellitus, pendred’s

Hypothyroidism: This is one of the commonest endocrine diseases of infancy and childhood.
Congenital hypothyroidism - a primary disorder of the thyroid gland or secondary to hypothalamic-pituitary abnormalities. Affected
children, when untreated, are left with mental retardation with/without short stature in later life. neonatal screening programme
detects problem early and provides for early Rx to prevent development of the long term effects. Unfortunately neonatal screening
programme which are readily available in advanced nations of the world are but a “luxury” in Nigeria and other resource poor countries.

Incidence is about 1:4000 children, females 2-4x more than males. Most newborns with condition have no signs at birth, even in cases with
complete agenesis.* Many a time, the deficiency is severe and clinical features develop within a few weeks while in some with mild disorder
symptoms are delayed for months.

Clinical features:
Prolonged neonatal jaundice, feeding difficulties, respiratory difficulties , poor cry , sleeping excessively, decreased activity, constipation,
pallor, cold mottled and dry skin, coarse facies, large protruding tongue, hoarse cry, goiter (occasionally), umbilical hernia , hypotonia, slow
pulse, and heart murmurs, cardiomegaly, asymptomatic pericardial effusion, refractory anaemia .

Retardation of physical and mental development becomes greater during the following months, and by 3–6 mo of age, the clinical picture is
fully developed: child's growth is stunted, the extremities are short, and the head size is normal or even increased.

Wide fontanels, eyes appearing far apart, depressed nasal bridge, narrow palpebral fissures, swollen eyelids, large tongue, delayed dentition,
short and thick neck, deposits of fat above the clavicles and between the neck and shoulders, broad hands, short fingers, dry, scaly skin,
absence of/ or poor sweating.
Myxedema is manifested, particularly in the skin of the eyelids, the back of the hands, and the external genitals.
Because symptoms appear gradually, the diagnosis is often delayed except with neonatal screening programme.

Diagnosis
Thyroid hormone assays: TSH, T4. Serum T4 levels are decreased in primary hypothyroidism while TSH is elevated. Serum T3 may
be normal or low and is not particularly helpful in making a diagnosis.
Thyroglobulin levels in serum are low in thyroid agenesis and elevated with ectopic glands and goiter.
X-rays of skeletal structures may show retardation of osseous development in about 60% of affected newborns at birth.
Radionuclide studies, using 123 I-sodium nuclide to evaluate I-trapping/concentrating mechanism of the thyroid may be helpful in
determining the underlying cause. Technetium pertechnate can be used in place of 123Iodine.
Ultrasound may be helpful in determining location, size and shape of the thyroid gland.
ECG changes in congenital hypothyroidism include low voltage P and T waves and diminished amplitudes of QRS complexes.

Neonatal hypothyroid index:


Without treatment affected infants become profoundly mentally deficient dwarfs. This underscores the need for early diagnosis and prompt
Rx. Routine newborn screening for hypothyroidism and other conditions is offered to all newborns within the first few days of life in
advanced nations of the world. Unfortunately, this is not available in resource-poor nations (such as ours). A scoring system has been
developed that can be utilized in selecting babies that need further evaluation; scores of 3 or more should be screened further.

Neonatal hypothyroid index


CLINICAL FEATURES SCORE
Typical facies 3
Dry skin 1.5
Open posterior fontanel (>0.5cm) 1.5
Feeding problem 1
Constipation 1
Poor activity 1
Poor muscle tone 1

63
Compiled by Obasi. D. C. Chinedu

Large tongue 1
Skin mottling 1
Umbilical hernia (>0.5cm) 1
TOTAL 13

Treatment:
Replacement therapy with Levo-thyroxine orally for life is the treatment of choice. Initial starting dose in the neonate is 10-
15µg/kg/day; tapered to 4 µg/kg/day b/w 1yr -10yr; further tapered to 2µg/kg/day in adult life.
Careful monitoring of patient clinically and laboratory-wise is required to ensure adequacy of treatment and prevent overtreatment.

PROGNOSIS:
Early diagnosis and adequate treatment from the first weeks of life results in normal linear growth and intelligence.
Delay in treatment beyond 3 months of life, inadequate treatment and poor compliance result in variable degrees of brain damage with
attendant learning disability. Without treatment at all, affected infants are mentally retarded and dwarfed.

Juvenile hypothyroidism (acquired hypothyroidism):


Occurrence: 1 per 1250children of school age. Symptoms usually appear after the first year of life or in adolescence. Twice commoner in
females than males. Usually caused by auto-immune thyroiditis. Other auto-immune disorders such as diabetes mellitus may develop, esp
in children with Down’s syndrome or Turner’s syndrome. Addison’s disease may also occur in some families.

Clinical features include the following: Short stature/growth failure, Cold intolerance, Dry skin, Cold peripheries, Bradycardia, Thin dry
hair, Pale, puffy eyes with loss of eyebrows, Goitre, Slow-relaxing reflexes, Constipation, Delayed puberty, Obesity, Slipped upper femoral
epiphysis, Deterioration in school work, Learning difficulties.

Treatment of juvenile hypothyroidism: L-thyroxine. 3-5µg/kg/day as a single daily oral dose and is for life.

Endemic goiter:
Most common cause of thyroid disease world-wide. Underlying cause is iodine deficiency. Commoner in girls than boys. Hyperplasia and
hypertrophy of the gland occurs as a compensatory mechanism in overcoming moderate deficiency and ensuring thyroid hormone
synthesis resulting in the development of a goiter. In areas with severe iodine deficiency, decompensation occurs and the gland is unable to
produce enough thyroid hormones with attendant development of hypothyroidism.

About 2 billion people in developing countries live in areas of iodine deficiency. Areas affected include: the Pacific West, the Great Lakes
area of the USA, the Himalayas, the Andes and the highlands of Papua New Guinea.
Endemic Goitre is seen in Nepal, Northern India, Bangladesh, Thailand, Vietnam, Malaysia, the Philipines, Middle East and parts of Central
and West Africa. In Nigeria, areas around Obubra in Cross River State and parts of Nsukka in Enugu State are involved.
However, in the US and other advanced nations of the world, the wide use of iodine fortified salt/other foods make endemic goiter rare.

Clinical features:
Vary depending on the degree of iodine deficiency.
In mild deficiency, thyroid enlargement seen esp. during periods of rapid growth eg. adolescents & pregnant women.
In severe deficiency, about 1/2 population has large goiters and endemic cretinism is common.

Endemic cretinism: neurologic & myxoedematous type.


Neurologic type xtized by: Mental retardation, Deaf mutism, Gait/standing disturbances, Pyramidal signs, Goitre, Normal pubertal
development, Adult stature
Myxoedematous type: Features include: Mental retardation, Deafness, Delayed sexual development, Short stature, Myxoedema, Absence of
goiter, Low T4 levels.

Treatment/control:
Single im injection of poppy seed oil to women prevents iodine deficiency during future pregnancy for about 5 years. Can be given to
children less than 4 years of age with myxoedematous cretinism to maintain a euthyroid state for about 5 months. Not helpful in older
children and adults with the disorder.
Iodination of salt, cooking oil, various foods and irrigation water can be used to supplement iodine intake and prevent endemic goiter.

HYPERTHYROIDISM: Results from excessive secretion of thyroid hormones.


Causes include:
 Auto-immune over-production of T4 and T3 (Grave’s Disease) (Commonest cause)
 Some patients with McCune-Albright syndrome
 Toxic Uninodular goiter (Plummer disease)

64
Compiled by Obasi. D. C. Chinedu

 Hyperfunctioning thyroid carcinoma


 Sub-acute thyroiditis
 Acute Suppurative thyroiditis
 Thyrotoxicosis fatitia

THYROTOXICOSIS (GRAVE’S DISEASE)


Usually results from an auto-immune phenomenon secondary to the production of thyroid stimulating immunoglobulins (TSIs). Most
commonly seen in teenage girls. T4 and T3 levels are elevated while TSH levels are suppressed to very low levels. Anti-thyroid
peroxisomal antibodies may also be present; spontaneous remission may occur; hypothyroidism also may follow.

Clinical features: Insidious onset, Clinical course highly variable


Anxiety and restlessness, Emotional lability, Deteriorating school performance, Goitre
Increased appetite, Sweating, Diarrhoea / hyperdefeacation
Weight loss, Rapid growth in height, Advanced bone maturity, Tremor,
Tachycardia and wide pulse pressure, Warm, vasodilated peripheries, Goitre (with/out bruit)
Learning difficulties/behavioural problems, Psychosis
Eye signs: exophthalmos, ophthalmoplegia, lid lag and lid retraction. Eye signs are uncommon in children.

Treatment: Medical treatment involves the use of anti-thyroid drugs and ancillary treatment.
Propylthiouracil and methimazole (carbimazole) are the major antithyroid drugs used.
Propylthiouracil dose: 5-10mg/kg/24 hr given 3 times daily
carbimazole dose: 0.25-1mg/kg/day given 1-2 times daily.
Duration of therapy is about 5 years or longer. Surgery or radio-iodine treatment is required when medical treatment fails.
Careful follow-up and reduction of drugs may be required to avoid over treatment. Thyroxine may be added to prevent glandular
enlargement. Propranolol is also given to block the systemic effects of thyrotoxicosis.

THYROID QUESTIONS
1. The thyroid gland synthesizes T4 and T3. True or False?
2. Where are the thyroid hormones formed?
3. How are the thyroid hormones formed?
4. Thyroid hormones play essential role in metabolism. Mention the specific roles.
5. Which of the thyroid hormones is more metabolically active, T3 or T4?
6. Which has a higher concentration in the blood/serum?
7. Peripheral de-iodination of T4 to form T3 is catalysed by---------enzyme.
8. What micronutrient is required for synthesis of these hormones?
9. Is the thyroid gland able to trap iodine against a concentration gradient? (Yes/No)
10. Iodine can only react with tyrosine in the oxidized state. (True or false).
11. What enzyme caltalyses oxidation of trapped iodine in thyroid follicles?
12. A specific thyroprotein elaborated by thyroid cells with approx. 120 tyrosine units is--------.
13. Thyroid hormones are formed in this protein.
14. Name the stages involved. In formation of thyroid hormones
15. Once formed, the hormones are stored in the colloid as thyroglobulin until needed (True/False).
16. Name the enzymes involved in liberation of these hormones from thyroglobulin.
17. The major protein that binds thyroid hormones is -------
18. Only 0.03% of T4 is free T4 (ie unbound) while only 0.3% of T3 is unbound (True of False)
19. Regulation of thyroid hormone synthesis is by ------hormone from the anterior pituitary

65
Compiled by Obasi. D. C. Chinedu

VIRAL HEPATITIS
Agozie C. Ubesie
Paediatric Gastroenterology, Hepatology and Nutrition Unit

What is Viral Hepatitis?


•Yellowness of the eyes???
•Infection of the liver???
•Bilirubin in the urine???
•Deranged liver function test???
•Enlarged liver???

Introduction
Viral hepatitis is infection of the liver, marked by diffuse hepatic cell necrosis & inflammation, usually caused by agents whose primary
tissue tropism is the liver.
Organisms: 90% Hepatotropic Viruses
–HBV -DNA Virus –HAV -RNA Virus –HCV -RNA Virus
–HDV -RNA Virus –HEV –RNA Virus
•HFV, HGV and TTV considered to relate to viral hepatitis

Organisms: 10% other viruses


–Herpes simplex virus (HSV) –Cytomegalovirus (CMV) –Epstein-Barr virus (EBV)
–HIV –Rubella –Entero Viruses –Adeno Viruses –Yellow Fever

All hepatitis viral infections are acute but hepatitis B, C, D and G can also result in chronic infections. In 20% HBV and 80% HCV
cases, it is chronic and may progress to CLD

Hepatitis A Viral Infection


Hepatitis A is an RNA (picorna virus group). Incubation period of 15 to 50 days (30). There are Four genotypes and one serotype.

Mode of transmission:
1. Ingestion of contaminated food and water– Fecal-oral transmission
2. Direct contact with an infectious person: Community, restaurant, and school outbreaks due to contaminated water or food.
3. Child care setting: Day Care and Nursery, Nosocomial spread in NB units

Pathophysiology: Viral replication occurs in the liver, leading to hepatic injury and necrosis. Liver injury is represented in 3 ways:
–Cellular injury (cell-mediated immune mechanism): Elevated serum liver enzyme levels
–Cholestasis: Jaundice and hyperbilirubinemia
–Inadequate liver function: lowers serum albumin levels and prolongs the PT

Clinical features: Many are asymptomatic. Only 30% of children are symptomatic,
•Prodromal symptoms: low-grade fever, nausea, vomiting, ↓ appetite, abdominal pain. RUQ pain in older children.
•GI symptoms common, Jaundice in 10-15%. Anicteric infections are common in young children. Diarrhea may be the only symptom in
young children. Mild hepatomegaly and RUQ tenderness Spleen in 10-20% of patients.

Diagnosis
•Specific antibody tests for HAV
–Anti-HAV IgM present at onset of symptoms, high for 4-8 weeks, usually disappear 4-6 mo
–Anti-HAV IgG: detectable shortly after the IgM appears, persists for life, provides immunity against re-infection
•Elevated ALT, AST, and GGT levels.
–usually 4-100 x; may precede symptoms onset by ≥ one week, peak 3-10 days after onset of illness.
•Serum bilirubin levels, usually remain below 10 mg/dL and peak after 1-2 weeks of illness.
•Prolonged PT and a significant ↓in the albumin level suggest a more severe course.

Complications: Does not cause chronic disease


•Rarely: Fulminant hepatic failure, Relapsing hepatitis, Prolonged cholestasis for months
•Complications include:
–AHF, myocarditis, pancreatitis, etc
–ALF more in underlying Liver Dz eg HCV

66
Compiled by Obasi. D. C. Chinedu

•Extra-hepatic manifestations: rash and arthralgias


•Less common: vasculitis, arthritis, optic neuritis, transverse myelitis, encephalitis, and bone marrow suppression

Treatment: Generally supportive (analgesics, anti emetics, vaccines, and Ig). Admit if severe dehydration or FH. Monthly follow up till
liver enzymes normalize

Prevention:
General measures: Improved sanitary conditions and practices (eg, hand washing), heating foods appropriately, Avoid water and foods
from endemic areas.
Prophylaxis (Pre and post-exposure): Vaccine/IM Ig within 2 weeks of exposure
–household and other intimate contacts
–institutions (e.g., day care centers)
–common source exposure (Food handlers)
–Vaccine: Monovalent (HAVRIX)/Combined (Twinrix/Hepatyrix)
•IM Hepatitis A vaccine (inactivated) ≥ 12 months in a two-dose schedule

Hepatitis B Virus:
Major health problem. About 2 billion people infected worldwide. 350 million suffer from chronic HBV infection. 10th leading cause of
death worldwide. SSA has a high prevalence of chronic HBV infection. High perinatal transmission.

Endemicity: Four levels of endemicity of HBV infection have been identified using HBs Ag Markers:
• Low(0-1%), • Intermediate(2-4%), • High(5-10%) • Hyper endemic(above 10%).
• Nigeria is a (super) hyper endemic area with frequency as high as 37-46%.

Mode of transmission:
• Through blood or body fluids (wound exudates, semen, cervical secretions, saliva, blood). HBV is 100 times more infectious than HIV
•Percutaneous and permucosal exposure, needle sharing, sexual, perinatal and blood transfusion
•Person to person spread within households

Peri-natal transmission
• Usually during labor and delivery • In utero (<2%) • Risk is 70-90% if mom is HBsAg and HBeAg +ve and 5-10% if HBeAg –
ve. Breastfeeding is not a risk factor for HBV infection.

Clinical relevance
Factor Use
HBsAg Acutely or chronically infected people, in HB vaccine
Anti-HBs Those with resolved HBV infection or immunity after
vaccination
HBeAg Infected people at increased risk of transmitting HBV
Anti-HBe Infected people with lower risk of transmitting HBV
Anti-HBc People with acute, resolved or chronic HBV infection (not
present after immunization)
IgM anti-HBc Acute or recent HBV infection (including HBs Ag–ve
people during “window period” phase of infection.

67
Compiled by Obasi. D. C. Chinedu

Clinical features: Incubation period of 45 to 160 days (90 days). 90% recover from acute phase. 1% progress to fulminant hepatic failure.
Others to chronic carriers: progress to persistent hepatitis or CAH (will end in CLD). Chronic HBV is presence of HBsAg in serum for at
least 6 months or –ve to anti-HBc IgM

Clinical features: Acute HBV: Usually asymptomatic


• Non specific symptoms: (anorexia, nausea, malaise)
• Clinical hepatitis: Jaundice, RUQ discomfort
• Fulminant fatal hepatitis: Asymptomatic sero-conversion is common
• Extra-hepatic: Arthralgia, arthritis, macular rashes, thrombocytopenia, papularacrodermatitis (Gianoti-Crostisyndrome) may precede
jaundice.

Treatment of Acute HBV:


• Consider NA if Fulminant hepatitis, severe acute hepatitis and protracted acute hepatitis
• Lamivudine, adefovir, entecavir and tenofovir: Until HBsAg clearance, or at least 3 months after HBsAg seroconversion, or 1 year after
HBeAg seroconversion without HBsAg
•IFN is contraindicated

Management (chronic): Initial evaluation:


History: risk factor, alcohol, family
Physical exam: for signs of CLD: spider telangiectasias, palmar erythema, hepatosplenomegaly, jaundice, ascites, edema, wasting, and
gynecomastia in boys) as well as growth parameters.
Lab: CBC with platelets, LFT (AST, ALT, total bilirubin, alkaline phosphatase), albumin, prothrombin time, and tests for HBV replication
(HBeAg, anti-HBe, HBV DNA), HCV. Screen for HCC (US) and alpha fetoprotein
•Liver biopsy if criteria for chronic hepatitis: HBsAg + for >6 months, serum HBV DNA >10(5) copies/mL, persistent elevation in
ALT/AST levels) and who are being considered for treatment.

Risk factors for chronicity:


Peri-natal infection infants (>90%): 25% of children with chronic HBV infection develop cirrhosis or HCC

Immunological Phases
1. Immune tolerant: Normal or mildly elevated serum ALT. Active HBV replication (HBV DNA >20,000 IU/mL or 10(5) copies/mL).
HBsAg and HBeAg are positive, normal liver histology
Variable duration: Continues for months, years, or even decades. Treatment in this phase is not recommended
2. Immune reactive (clearance): Elevated (ALT>1.5 to 2 times UL). Active HBV replication (HBV DNA is typically >20,000 IU/mL or
10(5) copies/mL). HBsAg/HBeAg +; Clear HBeAg spontaneously (sero-conversion) or to respond to treatment. At high risk for progressive
liver disease.
3. Low replicative or latent phase: Inactive carrier
Follows HBeAg seroconversion to anti-Hbe. Potential for disease flare-ups and complications. normal levels of ALT/ low or UD HBV.
HBsAg is positive but HBeAg is negative.
68
Compiled by Obasi. D. C. Chinedu

4. Reactivation: (anti-HB-+ve chronic hepatitis B). 20 to 30 %, Viral DNA ↑, elevated or fluctuating ALT. HBeAg remains negative. More
virulent liver disease. Antiviral indicated.

Resolution: Clearance of HBeAg, HBV infection, HBsAgand appearance of anti-HBs

Immunological Phases: •Based on persistence of HBsAg > 6months;


•IgG anti-HBc is positive, while IgM anti-HBc is negative •Most children asymptomatic
•Grow and develop normally •Vague R upper quad discomfort/ fatigue
•Extrahepatic manifestations ocassionally
•polyarteritis nodosa, glomerulonephropathy; membranous nephropathy membranoproliferative GN.

Treatment: Treat/Consider if : 1.ALT (Persistently abnormal levels)


2.HBeAg (positive vs negative) 3.Cirrhosis (compensated and decompensated)
4.Liver biopsy (Moderate/severe inflamm/fibrosis 5.HBV DNA
6.Patient's values and preferences

•Key goal of treatment is Inhibition of viral replication: Loss of HBeAg in HBeAg+ patients and suppression of HBV DNA levels.
•Secondary goals are to reduce symptoms, prevent or delay progression to cirrhosis or HCC.
•Agents currently used include PEG-IFN alfa-2a and the oral nucleoside or nucleotide analogues. PEG-IFN for 48 weeks for both
HBeAg+and –C/Hep. Oral agents may be used for as short as 1-2 years.
–Most HBeAg +ve and almost all HBeAg –ve patients require indefinite therapy
–Withdrawal of oral agents results in virologic relapse

Complications:
Cirrhosis: infrequent complication during childhood. Co-infection (HDV or HCV) likely
Hepatocellular carcinoma: Duration of dz, histologic injury, replicative state (HBV DNA levels). Higher in HBeAg + for extended periods.
Increased in presence of cirrhosis or concomitant infection with HCV or HIV.

Prevention: HB Vaccination (preferred method). Screening of blood donors and pregnant women. Sterilized needles and surgical
equipment. Avoidance of sexual contact with carriers. Vaccination of household contacts, medical personnel and other high risks.

Prevention:
Post exposure prophylaxis: HBV vaccine alone or with HBIg. Combination reduces vertical transmission.
•>90% chance of chronic hepatitis B, >90% chance of being protected
•Must be given within 12 hours

Prognosis:
Mortality due to fulminant hepatitis is 30%, > with co-infection/super infection with HDV. Chance of chronicity is 90% if infected at birth,
25 -50% at 1-5yrs, 5-10% in case of adult/older children.

Hepatitis C infection
Hepatitis C is an RNA virus and member of Flavivirus family. 170 million infected worldwide. Significant risk factor for both ESLD and
HCC.
Risk factors include Intravenous Drug Abuse, inadequately sterilized equipment and injection, cultural practices, blood transfusion, sharing
of shaving kits, sexual contact.

Hepatitis C genotypes: Six distinct genotypes and multiple subtypes based on the genetic material in the RNA strands of the virus.
1. Genotype 1a 2. Genotype 1b 3. Genotype 2a, 2b, 2c & 2d
4. Genotype 3a, 3b, 3c, 3d, 3e & 3f 5. Genotype 4a, 4b, 4c, 4d, 4e, 4f, 4g, 4h, 4i & 4j
6 Genotype 5a 7. Genotype 6a
Generally, patients are only infected with one genotype, but each genotype is a mixture of closely-related viruses called quasi-species.
quasi-species have the ability to mutate very quickly, become immune to current treatments.

Diagnosis and treatment:


Diagnosis: HCV antibody using ELISA (IgG) or HCV RNA viral load testing (NAA)
Treatment: Interferon alfa-2b (Peg-IFN) + ribavirin or Newer drugs: Victrelis, simeprevir, sofosbuvir
•Immunized against hepatitis A and B
•Monitor the liver enzymes: 80% turn into chronic infection

69
Compiled by Obasi. D. C. Chinedu

Hepatitis D virus (HDV)


HDV (Delta hepatitis virus) is a kind of defective virus. Replication on HBV or other hepadnavirus. Anti-HDV detected by RIA or ELISA in
serum. HBV and HDV co-infection or superinfection result in exacerbation and fulminant hepatitis.
Treatment is supportive. Hepatitis B immunization

Hepatitis E
Features similar to HAV. Transmission is more Faeco-oral. Person to person transmission less efficient than HAV. IgM and IgG testing.
Treatment is supportive

Fulminant Hepatitis: Severe impairment of hepatic functions or severe necrosis of hepatocytes in the absence of preexisting liver disease.
Features includes:
Encephalopathy: Absent, late, or apparent in children only at the terminal stages.
Significant coagulopathy in the absence of sepsis or DIC not correctable by parenteral vitamin K within 8 hours.

Treatment:
General and supporting therapy: Rest: strict bed rest, Diet and Supporting therapy
Symptomatic therapy
• Hemorrhage: fresh blood, prothrombin complex, platelet
• Hepatic encephalopathy
–Liver transplantation
–Anti-virals
INVESTIGATIONS FOR HEPATO-BILIARY DISORDERS
A. Biochemical Tests (Predominantly Hepatocellular)
•Alanine aminotransferase (ALT) •Aspartate aminotransferase (AST) •Bilirubin (Unconjugated, Conjugated)

AST/ALT ratio:
• Ratio < 1: acute and chronic hepatitis.
• Ratio >1: alcoholic liver disease (mitochondria), advancing non alcoholic disease
However, significant disease may be present with very mild or normal values!!!

• Very high values in: Viral hepatitis, Paracetamol OD, Ischaemia/shock


• Not very high values: Metabolic disorders (WD, Tyrosenemia)
• No correlation to severity of disease or liver necrosis

B. Predominantly Biliary Epithelium


• Gamma glutamyl transferase (GGT) • Alkaline phosphatase (ALP) • 5-nucleotidase (5-NT)
• Bilirubin • Bile acids blood and urine • Urine urobilinogen

Bilirubin:
Conjugated (normally none in the blood, >20% of total). Seen in the blood in Hepatocellular and biliary disease. Not neurotoxic.
Unconjugated (almost all bili is unconjugated): Seen in Newborn (important neurotoxicity), Crigler Najjar Syndrome (neurotoxicity),
Gilbert’s Syndrome.

C. Synthetic Function:
• Prothrombin time • Albumin • Ammonia • Cholesterol

Hepatitis B infection Sample Question


a. Transmission is through parenteral route
b. In acute HBV infection, HBsAg and IgM anti-HBc are positive
c. In chronic HBV infection, HBsAg and IgM anti-HBc are positive
d. HbsAg, anti-HBc and anti-HBs are positive in children with immunity due to natural infection
e. Breastfeeding is contra-indicated in infants whose mothers have HBV infection.

70
Compiled by Obasi. D. C. Chinedu

MALABSORPTION SYNDROMES
Agozie C. Ubesie
Paediatric Gastroenterology, Hepatology and Nutrition Unit

Malabsorption implies Impaired absorption of nutrients. It is not a diagnosis but several distinct conditions. It can be congenital (primary)
and acquired (secondary). Can result to chronic diarrhea, abdominal distention, and FTT. Affect one or more of the different steps in the
intestinal hydrolysis and transport of nutrients

Physiology:
GI tract digest and absorb nutrients: (fat, carbohydrate, protein, and fiber), micronutrients (vitamins and trace minerals), water, and
electrolytes.
Mechanical & enzymatic breakdown of food: Mechanical processes include chewing, gastric churning, and the to-and-fro mixing in the
SI. Enzymatic hydrolysis requires gastric, pancreatic, and biliary secretions.
The final products of digestion are absorbed through the intestinal epithelial cells.

Steps in nutrient absorption:


•Luminal events ("Luminal" phase)
•Processing at the brush border and absorption into the intestinal mucosa ("Absorptive" or "Mucosal" phase)
•Processing within the enterocyte and transport into the circulation ("Post-absorptive" or "Processing" phase)

Pathophysiology:
Caused by a disorder in the intestinal processes of digestion, transport, or both across the intestinal mucosa into the systemic circulation. Can
be Congenital or Acquired/secondary (more common)

Carbohydrates: Includes starches, sucrose, lactose (most abundant digestible CHO in the human diet). Constituent monosaccharides prior to
absorption. luminal digestion of starch by salivary pancreatic amylases.
Final hydrolysis at the BB of the enterocytes as follows; Sucrase-isomaltase= maltose, isomaltose (to glucose), and sucrose (to glucose and
fructose); Glucoamylase= glucose polymers to glucose; Lactase splits lactose into glucose and galactose.

CHO absorption:
•Monosaccharaides → enterocytes through the BB via carrier molecules.
Glucose, galactose share same carrier, SGLT-1, Transports 1 molecule of the monosaccharide and 1 molecule of Na in a secondarily active
transport. Energized by Na-activated and potassium (k)-activated adenosine triphosphatase (NaK ATPase).
Fructose uses carrier that allows entry only down a concentration gradient (Facilitated diffusion).
71
Compiled by Obasi. D. C. Chinedu

Undigested /unabsorbed Carbohydrate Undergo bacterial degradation in the colon.

Undigested/unabsorbed CHO:
The terminal phase is fermentation via Formation of short-chain fatty acids (butyrate, propionate, acetate, lactate), CO2, H2, methane. Short-
chain FA are available as an additional energy source (absorbed in colon). Excessive bacterial fermentation gives rise to acidic stools,
abdominal distension, and flatulence in pts with CHO malabsorption.

Causes of CHO Mal-absorption:


Congenital
–Cystic fibrosis, Shwachman-Diamond syndrome: May cause amylase deficiency
–Congenital lactase deficiency
–Glucose-galactose malabsorption
–Sucrase-isomaltase deficiency
–Adult-type hypolactasia

Acquired
–Lactose intolerance (most common): Secondary to a damage of the mucosa, such as a viral enteritis or celiac disease.

Lactose Intolerance: can be of different forms.


•Congenital or developmental LI: Extremely rare, autosomal recessive. Complete absence of lactase expression
•Primary or childhood-and adult-onset LI: Extremely common, autosomal recessive. Sharp fall in lactase production
•Acquired or secondary LI: Transient phenomenon, Due to damage of intestinal mucosa by an infectious, allergic, or inflammatory process.
Resolves once disease process is corrected. Healing of intestinal mucosa restores BB enzymes.

Protein Mal-absorption:
Protein digestion begins in the stomach through the enzyme, pepsinogen, released as proenzymes (pepsinogen 1 and 2). undergo
autoactivation at low pH to pepsin. In the duodenum, Pancreatic proteases completes breakdown to AA, dipeptides, and tripeptides. BB–
bound peptidases further hydrolyze peptides to release a mixture of free AA and small peptides. absorbed through highly efficient Na-
dependent AA co-transporters at the BB membrane.

Protein Mal-absorption: Can be congenital or acquired.


Congenital
–Cystic fibrosis: Defect in CFTR, responsible for salt excretion. Results in viscous secretions in multiple organ systems
–Shwachman-Diamond syndrome: Rare, autosomal recessive, exocrine pancreatic insufficiency, bone marrow dysfunction, skeletal
abnormalities
–Enterokinase deficiency
Acquired (non-specific)
Damage to the absorptive intestinal surface: Extensive viral enteritis, MPA enteropathy, and celiac
Food Protein Induced Enteropathy (FPE): Affects formula fed infants (cow’s milk or soy). Results in non-bloody diarrhea, malabsorption,
PLE, hypoalbuminemia, FTT. Strict cow and soy milk diet. Extensively hydrolyzed or AA based formulae.

Fat Mal-absorption
Fat digestion begins in the mouth via the Lingual lipase = 1st partial hydrolysis of triglycerides. Largest part of triglyceride digestion is in
the DuoD_jejunal lumen because of a complex of pancreatic enzymes. Develop slowly, low capacity of babies to absorb lipids,
(Physiologic steatorrhea of the newborn).
Adequate conc of intraluminal conjugated bile salts needed to form micelles, and the secretion of bile acids may also be partially inadequate
in very young patients

Fat Mal-absorption: may be


Congenital
–Cystic fibrosis, Shwachman-Diamond syndrome, Isolated deficiency of lipase and colipase; Congenital primary bile acid malabsorption
Acquired (secondary)
–Mostly disorders of liver and biliary tract or chronic pancreatitis: CLD: biliary atresia, chronic viral hepatitis,
–Causing loss of SI absorptive surface (SBS)

Most dietary lipids are absorbed in the proximal two-thirds of the jejunum. Normally, >94 % dietary fat is absorbed. In 100 g of fat/d
diet, >6 g of fecal fat in a 24-hour collection = fat malabsorption.

72
Compiled by Obasi. D. C. Chinedu

Causes: Global
• Celiac disease • Cow’s milk allergy • Microvillous inclusion disease • Tufting enteropathy
• Lymphangiectasia • Short bowel syndrome • Chronic malnutrition • Congenital immunodeficiency disorders • HIV •
Parasitic infections • Tropical sprue/Tropical enteropathy
• Bacterial overgrowth

Causes: Specific nutrients


Carbohydrate malabsorption: lactase deficiency (congenital, secondary), Congenital sucrase-isomaltase deficiency, Glucose-galactose
malabsorption
Protein malabsorption: Enterokinase deficiency, Amino acid transport defect (eg;Hartnup disease )
Fat malabsorption: Pancreatic exocrine insufficiency (cystic fibrosis, shwachman diamond syndrome, chronic pancreatitis), liver and
biliary disorders, abetalipoproteinemia
Mineral and vitamin malabsorption: Congenital chloride diarrhea, Congenital sodium absorption defect, Acrodermatitis enteropathica,
Menke disease, Vitamin D dependent rickets, Vitamin B12 malabsorption

Management: History
• Dietary history: Amt/type of fluids, solid foods, & formula ingested. Fat slows the movement of food thru the intestine. Juice may cause
osmotic diarrhea
• GI tract symptoms: Common, Ranges from mild abdominal gaseous distention to severe abdominal pain and vomiting. Chronic or
recurrent diarrhea most common symptom.
• Abdominal distention and watery diarrhea, ±mild abdominal pain, skin acidic irritation in the perianal area characterize CHO MA synd
• Periodic nausea, abd distention/pain, diarrhea common in chronic Giardia infections.
• Vomiting, abdominal pain, bloody stools; Protein sensitivity syndromes, Other causes of intestinal injury (eg,IBD).
• Some items may cause symptom when taken alone or with other specific dietary items

Stool characteristics:
–loose stools with undigested food particles in TD: Not true malabsorption.
–Frequent loose watery stools in CHO intolerance.
–Pasty or loose foul-smelling in fat malabsorption, or steatorrhea.
–Bloody stools in protein sensitivity syndromes

Other symptoms:
–Systemic symptoms: weakness, fatigue, FTT: Malabsorption of CHO, fats, or proteins can cause FTT. Folate, B-12 malabsorption result in
macrocytic anemia.
–Patients with abetalipoproteinemia develop retinitis pigmentosa and ataxia: chronic fat-soluble vitamin malabsorption and deficiency
(vitamins A and E)

Physical Examination:
• FTT, malnutr, poor wt gain, delayed
puberty
• ↓ muscle/fat mass, atrophic tongue
changes, enlarged liver/spleen: Low
weight and weight-for-height
percentiles
• Eczematous rash in protein sensitivity
• Dehydration from diarrhea
uncommon: Assess for signs,
symptoms & severity of dehydrat
• Borborygmi: associated with ↓
intestinal transient time.
• Constant wet diaper in young
children: erythema, skin irritation, and
skin breakdown,

Investigations:
Stool analysis

73
Compiled by Obasi. D. C. Chinedu

–Reducing substances in CHO malabsorption: Test fresh stools b/c of bacterial hydrolysis and removal of the reducing substances
–Acidic stool (pH level <5.5 in CHO malabsorption
–Quantitative stool fat and the amount of fat
–Serum proteins in the stool: a1 -antitrypsin, in PLE
–Ova and parasites or antigen in Giardiasis: Other chronic intestinal infections

CBC count
–megaloblastic anemia in folate and vitamin B-12 malabso
–Neutropenia in Shwachman-Diamond syndrome,
–blood smears may reveal acanthocytosis in abetalipoproteinemia
Serum protein and albumin levels
–Protein-losing enteropathy and pancreatic insufficiency or enterokinase deficiency
Elevated ESR, C-reactive protein level: –IBD
LFT:
–Elevated enzymes in hepatitis, GGT ALP in cholestatic liver disease
•IgG and IgA antigliadin and IgA antiendomysial antibodies, tissue transglutaminase antibodies,
–gluten-sensitive enteropathy

Substance tolerance test:


• Isolate suspected substance causing malabsorption: Resolution of diarrhea when removed and resumption when reintroduced are specific
signs
• If diarrhea does not resolve when removed: Not necessarily an indication of intolerance to the substance. Malabsorption of certain nutrients
can result in secondary intestinal damage and secondary malabsorption

Hydrogen breath test: CHO malabsorption results in bacterial fermentation. Releases H2 gas absorbed into the blood and excreted by the
lungs. An increase in the exhaled H2 conc following ingestion of an oral carbohydrate load indicates CHO malabsorption. Antibiotic
administration within the 2 weeks prior to the test may cause false-negative results in this test.

Lactose tolerance test: Oral administration of a 50g test dose, blood glucose levels are monitored at 0, 60, and 120 minutes. An increase in
blood glucose by less than 20 mg/dL (1.1 mmol/L) plus the development of symptoms is diagnostic. False negative in DM and BOO.

D-xylose absorption test: (pentose passively absorbed by jejunal mucosa). In normal individuals, a 25 g oral dose of D-xylose will be
absorbed and excreted in the urine at approximately 4.5 g in 5 hours. A decreased urinary excretion of D-xylose is positive test. A positive
test result suggests malabsorption due to proximal small bowel mucosal lesion (enteropathy).
•False-positive results: delayed gastric emptying, small bowel bacterial overgrowth, accelerated transit time

Mucosal biopsy: Definitive diagnosis


–Moderate villous atrophy in protein-sensitive enteropathies, Giardia infection, or bile acid malabsorpt.
–Histologic exam: mucosal inclusions seen in abetalipoproteinemia, eosinophilic gastroenteritis, Wolman disease, or congenital microvillous
atrophy,
–Functional assays of the biopsy tissue assess carbohydrate disaccharidase enzymes

Treatment:
•Depends on the specific entity being considered
–Gluten-free diet in celiac disease (diet completely devoid of wheat, barley, and rye
–Chronic diarrhea due to proximal small bowel BO is treated with oral broad-spectrum antibiotics, eg, metronidazole. That due to bile acid
malabsorption, is treated with the use of cholestyramine.
–Replace pancreatic enzymes with oral supplements.
–Elimination diet if secondary to food allergic enteropathy

Treatment
•Parenteral Nutrition in SBS till gut adaptation: Most children with SBS are weaned off PN and do not require surgery
•If disease is refractory to enteral feeding, or develop ESLD from the prolonged PN
•Fat malabsorption/SBS
–Fat-soluble vitamin, linoleic/linolenic FA supplements
–Consider liver, gut, or multivisceral transplantation in IF
–Gut lengthening procedures in IF: STEP and BIANCHI

74
Compiled by Obasi. D. C. Chinedu

PREMATURITY AND LOW BIRTH WEIGHT


DR.OBUMNEME-ANYIM IJEOMA

Outlines: - Background - Definition of terms/classification - Epidemiology


 Assessment - Immediate complications - Management - Long term complications

Background: Prematurity refers to a broad category of newborns delivered before 37 completed weeks. Globally, prematurity is the leading
cause of death in children under the age of 5(one million deaths per year). Preterm birth is responsible for more deaths during the first year of
life than any other single factor, and in almost all countries with reliable data, preterm birth rates are increasing.
Premature newborns have many physiologic challenges when adapting to the extra uterine environment. An estimated 15 million babies are
born too early every year. That is more than 1 in 10 babies.
Preterm infants have a higher percentage of birth defects. Many survivors face a lifetime of disability, including learning disabilities and
visual and hearing problems.
Prematurity and low birth weight are often concomitant, and both factors are associated with increased neonatal morbidity and mortality. The
less an infant weighs at birth, the greater the risks to life during delivery and immediately thereafter.
Inequalities in survival rates around the world are stark. In low-income settings, half of the babies born at or below 32 weeks (2 months
early) die due to a lack of feasible, cost-effective care, such as warmth, breastfeeding support, and basic care for infections and breathing
difficulties. In high-income countries, almost all of these babies survive.

Definition of terms and classification.


Term gestation: 37 – 42 weeks Preterm: less than 37 completed weeks
Posterm: more than 42 completed weeks. Extreme low birth weight(ELBW) :<1000g(1kg)
Very low birth weight(VLBW) :<1500g. Low birth weight(LBW): <2500g.
Normal birth weight: 2500g - 4000g. Macrosomia birth weight >4000g.
Chronologic age: based on date of birth Corrected age: based on gestational age. (post menstrual age)
WHO classification of prematurity is based on gestational age:
extremely preterm (<28 weeks) very preterm (28 to <32 weeks)
moderate to late preterm (32 to <37 weeks).

Epidemiology: More than 60% of preterm births occur in Africa and South Asia, but preterm birth is truly a global problem. In the lower-
income countries, on average, 12% of babies are born too early compared with 9% in higher-income countries. No racial differences. High
mortality especially in sub Saharan Africa.
Nigeria is ranking 3rd among 10 countries with the highest number of preterm birth (773,600 per year)

Risk factors for premature birth/LBW include: Low socioeconomic status, Tobacco use, Preterm labour, Preterm premature rupture of
membrane, Chorioamnionitis, Intrauterine
growth restriction, Maternal diabetes,
Hypertension, Multiple gestation, Maternal
age < 16 or >40 years. Commoner in non-
Hispanic blacks

Other factors include: Obstetric factors like


uterine malformation, uterine trauma,
placenta previae, abruptio placentae, cervical
shortening, cervical surgery. Previous
preterm delivery.
Fetal conditions like IUGR, severe hydrops,
non-reassuring test of fetal wellbeing.
Inadvertent early delivery due to incorrect
GA estimation.

Assessment of Preterms
Ballard Maturational Assessment: The
scoring relies on the intra-uterine changes
that the fetus undergoes during its
maturation. The neurological criteria
depend mainly upon muscle tone while the
physical ones rely on anatomical changes.
The neonate that is less than 37 weeks of
age is in a state of physiological hypotonia. This tone increases throughout the fetal growth period, meaning a more premature baby
would have lesser muscle tone.

75
Compiled by Obasi. D. C. Chinedu

Immediate problems of preterm


babies
Respiratory: Perinatal depression at
birth due to poor transition to breathing.
Respiratory distress syndrome from
surfactant deficiency and pulmonary
immaturity. Apnea due to immaturity of
the respiratory center. Chronic lung
disease of prematurity.

Neurologic: Perinatal depression


Intracranial haemorrhage: Germinal
matrix/intraventricular haemorrhage
predominates in preterms < 32weeks.
Bleeding originates from the fragile
involuting vessels of the sub
ependymal germinal matrix in the
caudothalamic grove. It is usually silent
in the preterm and usually
demonstrated by cranial ultrasound.
Management is largely supportive.

Cardiovascular: Hypotension which could result from hypovolemia, cardiac


dysfunction and sepsis. PDA common among preterms and may lead to pulmonary
over-circulation and diastolic hypotension.

Haematologic: Anaemia, Thrombocytopaenia, Hyperbilirubinemia


Nutritional/Digestive: Poor or absent sucking and swallowing. Feeding intolerance.
Necrotising enterocolitis
Metabolic: Hypoglycemia, Hyperglycemia, Hypocalcemia, Hypermagnesemia.
Renal: Low GFR, Inability to handle water and solutes

Temperature regulation: Preterms are susceptible to hypothermia and hyperthermia.


There is increased mechanism of heat loss combined with poor heat production
capability as a result of; Higher skin surface area ratio to weight, highly permeable skin, decreased subcutaneous fats, less developed brown
fat stores, poor caloric intake for thermogenesis and limited oxygen consumption when pulmonary problems exist.

Immunologic: Deficiencies in both cellular and humoral response predispose preterm infants to infections
Ophthalmologic: Retinopathy of prematurity may develop in the immature retina of infants less than 32 weeks or with birth weight less than
1500g.

Management the preterm infant


Prenatal: Transport in utero to appropriately equipped & staffed hospital. Antenatal steroid to hasten lung maturity
At delivery: Immediate availability of qualified personnel and equipment to resuscitate and stabilize.
Post natal: Thermal regulation by incubator care, radiant warmer and kangaroo care.

Respiratory support: Oxygen therapy, Continous positive airway pressure(CPAP for an infant with spontenous respiration). Ventilatory
support. Use of exogenous surfactant for infants less than 32 weeks. Use of caffeine citrate for apnea of prematurity

Fluid and electrolyte therapy: For ELBW infants use 5-10% Dex. and for VLBW and LBW use 10% dex. ELBW and VLBW require
higher amount of fluids compared with LBW. Electrolytes like Na is not required in the first 24-48 hours. Why?
Nutrition: Gavage feeding or TPN may be needed Hyperbilirubinemia: phototherapy and EBT
Anaemia: Red cell transfusion Thrombocytopaenia: Platelet concentrates
Antibiotics for infections

Long term complications of prematurity


Developmental disability: Cerebral palsy, Mental retardation, Hearing loss, Visual impairment/ROP, Language disorder, Learning
76
Compiled by Obasi. D. C. Chinedu

disability. Others are: ADHD, Chronic lung disease and Poor growth.

A, The preterm newborn. This infant evidences the


extended posture of the arms and legs characteristic of
the preterm infant. The skin is thin and transparent, and
the labia are open and gaping. (See Ballard scale, Figure
13-2.)

B, The full-term newborn. Compare the completely


flexed arms & legs evidencing good muscle tone in this
full-term newborn. The flexed position of the newborn
limits the loss of body heat, as less skin surface is
exposed to the air.

C, Popliteal angle. This heel-to-ear maneuver


demonstrates the easy extension of the leg consistent
with a 30-week gestation. A full-term infant would
show muscle resistance to this maneuver. (See Ballard
scale, Figure 13-2.)

D, When the arm is pulled across the chest of the full-


term infant, the elbow goes only as far as the chin in the
midline. This is called the scarf sign. E, When the arm
is pulled across the chest of the preterm infant, it can be
pulled into a straight line, with the elbow passing the chin at the midline (abnormal scarf sign).

INTRAUTERINE GROWTH RETARDATION(IUGR):


This is a condition in which an unborn baby is smaller than it should be because it is not growing at a normal rate inside the womb.
Defined as weight less than the 10th centile for age.
Delayed growth puts the baby at risk of certain health problems during pregnancy, delivery, and after birth. It complicates 3-10% of
pregnancies. Contributes 50% of preterm stillbirth and 25% of term stillbirth. IUGR is the 2nd leading cause of perinatal mortality.

Causes of IUGR: They can be Maternal or Fetal.


Maternal factors: They include: Advanced diabetes mellitus, High blood pressure or heart disease, Infections such as rubella,
cytomegalovirus, toxoplasmosis, and syphilis. Kidney disease or lung disease, Malnutrition or anemia, Sickle cell anemia, Smoking, drinking
alcohol, or abusing drugs.
Fetal factors: Chromosomal anomalies, Multiple gestation, Vertically transmitted intrauterine infections.

Types of IUGR
Asymmetric: More common, about 70-80% of all IUGR. Occurs as a result of restriction of nutrient supply inutero (uteroplacental
insufficiency). There is restriction weight, followed by length but the head continues to grow at near normal rate. Growth usually occurs
at a normal rate in the Ist and 2nd trimester but encounters difficulties in the 3rd trimester. At birth, the neonate appears disproportionate, has
dry peeling and loose skin, thin and dull umbilicus. Factors extrinsic to the fetus are usually implicated. Ponderal index is low {PI =
Birth weight*100
(crown-heel length)3

Symmetric: Is less common about 20-25%. Growth restriction is global indicating an early insult. The head circumference of such a
newborn is in proportion to the rest of the body. The fetus with symmetrical IUGR is more likely to have permanent neurological
sequelae since most neurons develop by the 18th week of intrauterine life. Factors intrinsic to the fetus are usually the cause. Ponderal
index is normal.

Complications of IUGR: - Low birth weight - Difficulty handling the stresses of vaginal delivery
 Decreased oxygen levels - Hypoglycemia - Low resistance to infection
 Low Apgar scores - Meconium aspiration - Trouble maintaining body temperature
 Abnormally high red blood cell count - Stillbirth in most severe cases.
 Long term complications include obesity, diabetes, metabolic syndrome, heart diseases

77
Compiled by Obasi. D. C. Chinedu

FEVER IN CHILDREN
Dr HU Okafor

Outline
 Learning Objectives - Definitions - Methods of measurement - Pathophysiology
 Epidemiology - Causes - Evaluation/Treatment

Learning Objectives: At the end of this, the students should be able


 To be able to define fever - To be able to measure and interpret temperature readings.
 To have some knowledge of its pathogenesis - To know the common causes of fever in children.
 The approach towards evaluation & treatment of the child with fever.

Introduction: Fever is the commonest symptom of childhood illnesses. It usually cuts across many disease conditions. Main goal is to
identify the cause.

Definition: Fever is an abnormal elevation of body temperature that occurs as part of a specific biologic response that is mediated and
controlled by the central nervous system. Clinically Fever is defined as elevated temperature beyond the normal range: The standard
definition of fever is a rectal temperature of 100.4°F (38°C) or higher. Normal temperature range varies with method and site of
measurement. Core temperature may vary 1-1.5°C in 24hrs. Normal temperature generally varies between 36.5°C-37.2°C.

Methods of Measurement: Site of Measurement /Normal temperature range


 Rectum: 36.6°C to 38°C (97.9°F to 100.4°F) - Mouth: 35.5°C to 37.5°C (95.9°F to 99.5°F)
 Armpit: 34.7°C to 37.2°C (94.5°F to 99.1°F) - Ear: 35.8°C to 38°C (96.4°F to 100.4°F)
Oral temperature is typically 0.6ºC (1.0ºF) lower than rectal temperature because of mouth breathing, which is particularly
important in patients with tachypnea.

Pathophysiology: In the face of pathology, pyrogens (low-molecular-weight proteins produced by leukocytes and some external agents like
bacteria) are produced. There is peripheral synthesis and release of interleukin (IL)-1, IL-6, tumor necrosis factor (TNF), interferon (IFN)-
alpha, and other endogenous pyrogenic cytokines by phagocytic cells in the blood or tissues. These cytokines enter the blood and are carried
to the anterior hypothalamus, where they induce an abrupt increase in the synthesis of prostaglandins, especially prostaglandin E2
(PGE2).
The induction of PGE2 in the brain raises the hypothalamic set-point for body temperature. After the set-point is raised, the
thermoregulatory center recognizes current body temperature to be too low and initiates a series of events to raise body temperature
to the new set-point. This involves augmentation of heat production by increased metabolic rate and increased muscle tone and activity, and
decreased heat loss through diminished perfusion of the skin.
Body temperature rises until a new equilibrium is achieved at the elevated set-point. The upper limit of temperature due to fever appears
to be 42ºC (107.6ºF), but it is unusual for temperature to exceed 41ºC (106ºF) without some element of concomitant hyperthermia.

In addition to causing fever, pyrogenic cytokines increase the synthesis of acute-phase proteins by the liver, decrease serum iron and
zinc levels, provoke leukocytosis, and accelerate skeletal muscle proteolysis.
IL-1 also induces slow-wave sleep, perhaps explaining the somnolence frequently associated with febrile illnesses.
The increase in peripheral PGE2 may account for the myalgias and arthralgias that often accompany fever. Increased heart rate is a normal
physiologic response to fever. Their effects act on the hypothalamus and affect the body's response to heat by altering vascular constriction
and other heat production and/or release mechanisms.

Epidemiology: USA 20-25% of presenting complaints. In 10-15% fever is an associated sign. In Nigeria more than 60% of hospital
consultations are because of malaria which is synonymous with fever in our setting.
No race, sex or age bias. Can occur at any age.
When presenting with fever, neonates (<28 d) and very young infants (28-60 d) are considered at risk for sepsis until proven otherwise. The
neonate has been traditionally described as being at greater risk than older children for 2 reasons:-
1. their bacterial pathogens may be different from those in older children.
2. their immune systems may be less capable than those of older children to opsonize and compartmentalize infection.
Note: Not all septic neonates present with fever. Septic neonates may present to the ED with a lower than normal temperature or other
systemic signs of illness.

Causes of fever:
 Parasitic - Bacterial -Viral
 Autoimmune - Malignancies - Increased activity - Metabolic causes

Definitions/Abbreviations:
Pyrexia of Unknown Origin(PUO): A temperature greater than 38.3°C (101°F) on several occasions with more than 3 weeks' duration of

78
Compiled by Obasi. D. C. Chinedu

illness and failure to reach a diagnosis despite one week of in patient investigation. (Petersdorf and Beeson 1961).

Definition (children): The term fever of unknown origin (FUO) is applied to children with fever >101ºF (38.3ºC) of at least eight days'
duration, in whom no diagnosis is apparent after initial outpatient or hospital evaluation that includes a careful history, physical
examination and initial laboratory assessment.

Fever without focus: fever with no obvious source of infection.


Serious Bacterial Infection(SBI): is a term used to describe the various conditions caused by bacterial infection that were major sources of
morbidity and mortality during the period of time before routine use of Haemophilus influenzae type b (HIB) and pneumococcal conjugate
(PCV) vaccinations. Studies have shown that infants <3months are at higher risk.

Occult bacteremia: Defined as bacteremia not associated with clinical evidence of sepsis (shock or purpura) or toxic appearance, underlying
significant chronic medical conditions, or clear foci of infection upon examination.

Sepsis: Generally, sepsis is considered to comprise a spectrum of disorders or SIRS that result from infection by bacteria, viruses, fungi,
or parasites or the toxic products of these microorganisms.
The spectrum of sepsis ranges from microbial invasion of the bloodstream or intoxication with early signs of circulatory compromise,
including tachycardia, tachypnea, peripheral vasodilatation, and fever (or hypothermia), to full-blown circulatory collapse with multi-
organ system failure and death .

Systemic Inflammatory Response Syndrome (SIRS): SIRS in children includes two or more of the following, one of which must be an
abnormal temperature or leukocyte count:
 A rectal temperature > 38.5°C or < 36°C;
 Heart rate more than two standard deviations (SD) above the normal, or bradycardia in children older than 1 year of age (< 10th
percentile for age);
 Respiratory rate more than two SD above normal (or pCO2 < 32 mmHg);
 Leukocyte count > 12,000 cells/mm3, < 4,000 cells/mm3, or > 10% band forms

SEPTIC SHOCK: Septic shock is sepsis with fluid refractory hypotension and signs of hypo perfusion. Shock can be cold or warm.
Table 1. Definitions of Shock12
Characteristics
Type of Shock
Central Capillary Refill Peripheral Pulse Skin Pulse Pressure
Cool
Cold Shock > 3 seconds Decreased Narrow
Mottled
Warm Shock < 3 seconds Bounding Warm Wide

Bacteremia, viremia, fungemia, and parasitemia: refer to bloodstream invasions that may be associated with fever but have no other
signs or symptoms of circulatory compromise or end-organ malperfusion or dysfunction.

Serious bacterial illnesses (causes): Causes of elevated temperature include serious as well as less serious illnesses.
Historically the classic serious bacterial illness (SBI) include following:
- Meningitis, or encephalitis - Bacteremia and sepsis - Urinary tract infection (UTI)
- Bacterial or viral pneumonia - Septic arthritis - Osteomyelitis - Cellulitis
Less serious causes of fever: includes
- Otitis media - Oral infections, including pharyngitis due to Streptococcus pyogenes (group A Streptococcus species) and viral
herpetic gingivostomatitis
- Apthous ulcers - Generalized viral illness - Upper respiratory tract infection (URTI)
These are usually more commonly occurring conditions

Non infectious causes of fever:


Not all temperature elevations are caused by an infectious disease. Temperature elevations may occur without infectious etiology.
These include environmental factors, such as the following:
High external temperature (especially in the warmer weather months), over bundling of children in colder weather months.
Diseases or conditions that present with fever: Appendicitis, Malignancy, Rheumatoid diseases.

Other Causes of fever


Recent immunization administration: Routine administration of childhood vaccinations carry the risk of temperature elevation as a
common adverse effect. Drugs (drug fever).

79
Compiled by Obasi. D. C. Chinedu

Administration of the diphtheria, tetanus, and pertussis (DPT) vaccine may cause fever within a few hours after administration and may
persist up to 48 hours.
Administration of live-virus vaccinations, such as the measles, mumps, and rubella (MMR) vaccine, may result in temperature elevations up
to 7-10 days after its administration.

Evaluation of The older child with fever –History


- What is the timing of the current illness?
- When did the fever start?
- How long has the fever been present? Are there any related symptoms?
- What has been done at home to help control the fever?
- Has the correct dose of antipyretic been given at home?
- What is the patient's medical history? The history may not be applicable in all cases, but it must be explored to reveal potential high
risk or complicating factors.
- Has there been exposure to illness through babysitters, daycare contacts, or other caregivers? Are others at home sick?
- Have the sleep patterns changed? Has the patient been snoring more at night than usual?
- Has there been any recent travel that might have exposed the child to illnesses?
- Take a good immunization history & find out if there was recent immunization.

Evaluation of the neonate with fever- History


The history of the neonate is explored for possible evidence of poor feeding, vomiting, poor social interaction, changes in the quality of
crying, and possible apneic episodes. Any of these findings are reasons to consider serious bacterial infection and may warrant laboratory
evaluation.

The birth history is explored to ascertain risk factors for underlying pathology, such as prematurity, maternal infections, and congenital
or chronic disease states. Neonates at risk for congenital herpes are those born to mothers with recent genital infection, high-risk
sexual activity, and rupture of membranes >6 hours.

Neonates who present with irritability, seizures, respiratory distress, jaundice, or a characteristic vesicular rash should be considered at
risk for neonatal herpes. Note that 10-50% will not develop skin lesions during the course of their illness. Likewise locally, history of
maternal malaria esp. in the last trimester, should increase suspicion of congenital malaria.
The history is also explored for previous diagnostic studies and their results.

Other less common etiologic agents are Neisseria meningitides and (especially in patients with sickle cell disease) Salmonella species.
Herpes and community-acquired methicillin-resistant Stahylococcus aureus (MRSA) are now emerging as more common pathogens in
neonate.

Children older than 24 months:


The febrile child older than 24 months (who has been previously fully immunized) is primarily evaluated by obtaining a history and
performing a physical examination. Specific workup and/or treatment is based on the clinical findings and suspicion of disease.

Evaluation of the child with fever- Temperature measurement:


Approach depends on age of the child and underlying conditions eg immune status. The evaluation of any child in the emergency department
should include documentation of the presence or absence of temperature changes.
Thermometer use varies between oral, rectal, or axillary. Ear-probe thermometers may not be as accurate as rectal thermometers in the
neonate. Some study results suggest that operator error is the main reason.
A rectal-probe thermometer is probably most likely to result in an accurate assessment of a neonate's temperature.
Temperature elevation may not be the only sign of sepsis in neonates and infants. Other potential signs and symptoms of sepsis unique to
infancy should also be assessed.

Physical examination: Full general examination, Examination of the various systems. ENT examination

Work up
Neonates: Guidelines have been applied to neonatal emergency medicine. Traditionally, a febrile neonate (temperature 100.4°F [>38°C])
undergoes a full sepsis workup including CBC, blood culture, urinalysis, urine culture (obtained by using a catheter), diagnostic lumbar
puncture (LP) for CSF analysis and Surface swabs.
Older child:
Routine investigations such as: CBC & Blood film & ESR, Urinalysis, Urine culture.
Specific investigations like: Blood culture, LP or Serology, etc depending on findings

Before routine use of the HIB vaccine, HIB accounted for 20% of occult illness, but this cause has also decreased in frequency after the

80
Compiled by Obasi. D. C. Chinedu

vaccination became routine in the 1990s.


An increase in the total band count or erythrocyte sedimentation rate (ESR) is not more predictive of occult pneumococcal bacteremia than
the elevated WBC count alone. Before the routine use of the pneumococcal vaccine, a WBC count >15,000/mm3 had been reported to be
70% sensitive for predicting occult bacteremia from pneumococcus.
Children with fever and no other signs may have an enterovirus infection. Some studies report the incidence as high as 50% in febrile
children in the ED. Enteroviral infection is a clinical diagnosis for the emergency physician. No specific laboratory testing is indicated.

Urinalysis and urine culture: Boys younger than 6 months with fever have an increased rate of UTIs. Several groups report an
incidence of approximately 7%. For this reason, urinalysis and urine culture (obtained by catheterization or suprapubic tap) are
recommended in male infants without another source of fever.

Girls younger than 12 months have about an 8% incidence of UTIs. Urinalysis and urine culture (obtained by catheterization) should be
performed in febrile girls younger than 12 months if no other focus of fever is discovered. Fecal and skin contamination make the urine
bag inadequate for obtaining specimens for culture. Bladder catheterization is required for culture.

Stool studies: Diarrhea in children is commonly caused by viral organisms and usually not considered a major source of fever(rotavirus).
Diarrhea with blood or mucus is an indication for further testing for fecal leukocytes, which suggests invasive bacterial etiologies.
If fecal leukocytes (>5 per high-powered field) are present, a bacterial etiology is suggested and cultures are indicated. This finding may
indicate infection with species of Salmonella, Campylobacter, Shigella, Yersinia, and toxic strains of Escherichia coli. The final diagnosis
can be made only by obtaining stool cultures.
During last months of the year, children presenting with low-grade fever, vomiting, and diarrhea should be considered possibly infected
with rotavirus.

Treatment:
- Aggressive treatment is indicated when pt is at risk of febrile seizures.
- Otherwise tepid sponging and antipyretics
- Main focus should be on cause of fever.
- For cases with confirmed bacterial infections treatment would be based on lab. results or epidemiological evidence.

81
Compiled by Obasi. D. C. Chinedu

INVESTIGATION OF CARDIOVASCULAR DISEASES


Ujunwa FA

Learning objectives.
 To understand the various steps needed to evaluate a cardiac patient.
 To understand the basic approach to confirming or eliminating specific cardiovascular diseases.
 To learn the common invasive and non invasive investigations in cardiology.

Outline: - Introduction - Approach to clinical diagnosis of cardiovascular diseases.


 Non invasive cardiovascular investigations - Invasive cardiovascular investigations.
 Current trends in diagnosis. - References.

Introduction: Diseases and conditions that affect the heart and blood vessels are collectively known as cardiovascular diseases.
They can be acquired or congenital. The quality of patient care depends on a thorough bedside assessment. A good history and physical
examination enables a physician to entertain a list of differential diagnosis.
Order relevant investigations. The ability of not over investigating a patient and still manage the patient is a hallmark of a good
clinician. Always remember that both the invasive and non invasive investigations are sources of concern to parents.

In investigating a cardiovascular disease, the importance of history and physical examination cannot be overemphasized. It is the richest
source of information concerning the patient’s illness. It also enables the clinician establishes bond with the patient and evaluates the impact
of the disease.
A cardiac examination enables the clinician to guide the echocardiographic and further investigations. Adequate history and examinations
helps to avoid unnecessary and expensive investigations. Helps in reassuring the caregivers and placing unwarranted restrictions to the child.

History: History should follow the standard format. Emphasis should be focused on symptoms of cardiovascular diseases. This should be
modified according to the age of the patient and suspected nature of the disease. Acquired or congenital heart disease.
Current illness: usually age dependent
Most common presentations include; Cyanosis, Respiratory difficulty, Feeding difficulties, Cough, Exercise intolerance/ limitation of
activities, Dyspnoea/breathlessness, Orthopnoea and PND.
Others are Edema, Chest pain (unusual cause but commonest reason for referral to cardiologist), Palpitations, Syncope, fever.

Gestational and natal history: History of Maternal illness eg rubella in the first trimester., SLE, Maternal history of CHD.
Maternal diabetes, drug and alcohol abuse., Exposure to teratogens. Prematurity, Perinatal asphyxia. Birth weight
Family history: History of sudden death, Cardiac abnormality in first degree relatives such as hypertrophic cardiomyopathy, Duchenne’s
dystrophy. Myocardial infarction, hypercholesterolemia. Consanguineity.

Physical examination: Physical examination should be carried out in a systematic format: IPPA
On Inspection, check for: Respiratory distress, Cyanosis (central, acrocyanosis, differential), Clubbing, Plethora, Evidence of growth failure,
Tachypnoea, Physique/posture/webbed neck, Dysmorphic features, Cataract, Precordial bulge, Pedal edema, Ascitis/abdominal distention,
Engorged neck veins, Palor, Harrison’s groove, Splinter haemorrhages (Roths spot, Jane way lesions).
On Precordial inspection: Check Precordial bulge, Harrison’s groove, Pectus carinatum and excavatum, Visible parasternal impulse,
Hyperdynamic precordium

On palpation: Assess for


Vital signs; temperature, capillary refill time, anthropometry, extremities (palms and feet), heart rate (Bradycardia and tachycardia)
Pulses; peripheral pulses, rate rhythm, volume, synchrony, state of arterial wall and other pulses; brachial, femoral, popiletal, dorsalis pedis.
Carotid, tibia. Pulse pressure
Blood pressure: use sphygmanometer, use appropriate cuff, patient must be relaxed. Be wary of white coat hypertension. Flush method
and oscillometric method can be used in infants younger than 2years.
Venous examination: raised JVP in right heart failure pericardial tamponade, tricuspid atresia SVC obstruction, tricuspid regurgitation.
constrictive pericarditis. Usually examined with patient at 450. Abdomino-jugular reflux.
Congested liver: rounded, firm and tender.

Precordium palpation:
Apex beat; determine site /point of maximal intensity. Location, character (tapping or heaving)
Thrills: grade 4 palpable murmur. Palpable hear sounds-p2 in severe pulmonary hypertension also in TGA. Palpate the suprasternal notch
and neck for bruits-aortic stenosis. Faint in pulmonary stenosis

Percussion of precordium: This does not contribute much to diagnosis of heart disease. Used in checking area of cardiac dullness.
Increased resonance in air trapping
Auscultation: Areas of auscultation includes Cardiac apex, right and left sternal borders interspace by interspace pulmonary and
aortic area areas. Both sides of the neck and at the back. Be systematic. Listen to one sound at a time

82
Compiled by Obasi. D. C. Chinedu

Listen for: heart sounds, S1-4, ejection clicks, opening snap, pericardial rub, murmurs.
Time the sounds with palpation of the carotid artery (to distinguish between systolic and diastolic).
Note the cardiac cycle using S1 and S2
Note S3; Ventricular filling sound heard best at the apex. Heard best with bell, represents transition from rapid filling to slow filling
phases. S4; an atrial filling sound which is heard when ventricular compliance is reduced.

Characterize Murmur in terms of intensity (grade 1-6), Timing (systolic, diastolic or continuous), location (point of maximal intensity),
transmission, radiation and quality (harsh or rumbling or vibratory).

Classification: Innocent- no abnormality Organic- both physiologic and anatomic abnormalities present.
Functional: physiologic abnormality present but anatomic absent, correction of the physiology leads to the disappearance of murmur.
Note venous hum –continuous innocent murmur, diagnosed in sitting position but disappears on supine or by turning the patients head to the
left side.

Laboratory investigation: could be invasive or Non-invasive


Non invasive; includes Pulse oximetry, hyperoxia test ambulatory bold pressure monitoring,
ECG: Graphic representation of electrical activity of the heart; Arrhythmias, chamber hypertrophy, ischemia, drug toxicity eg digoxin. Stress
ECG and Holter monitoring, exercise ECG

Echocardiography: this has improved the practice of cardiology. it has M mode, 2D, and 3D components with Doppler functions. Stress
ECHO
Echocardiography helps in; evaluation of cardiac structures, quantitate function of the muscles, chambers and blood vessels, determine the
direction of flow of a shunt, integrity of arteries, presence of vegetation's, cardiac tumors and thrombi. It also assists in performance of
pericardiocentesis, endocardial biopsy, Estimate intracardiac pressures.
TEE is used intraoperatively in surgical repair of congenital heart diseases.
Doppler flow studies help in assessment of flows and pressures in the valves and arteries.
Fetal echo is used in fetal diagnosis as early as 17-19wks of gestation and in diagnosing arrhythmias

ECHO has different components: M-Mode and 2 D compononent and 3D components


M-MODE is used in assessment of cardiac functions and dimensions especially percent Fractional shortening(%FS)
2D echocardiography provides real time images of cardiac structures
Doppler echo provides information intracardiac shunt flows, pressure across shunts, direction of shunt flows, vascular communications,
flows across valves semilunar and atrioventricular valves, assessment of diastolic dysfunction.

Trans oesophageal echo (TEE): Invasive, shows clearer images of smaller lesions like vegetations. Also used in identifying posterior
structures like aortic roots, atria, atrioventricular valves.
Used intraoperatively for monitoring cardiac function. to monitor residual defect or regurgitation after surgery.

Invasive investigations Includes;


Chest X-ray; invaluable in diagnosis of cardiac diseases; Gives information about the cardiac size, lung fields, ribs (rib notching), Rib cage,
aorta and chamber sizes, Situs.
Characteristic shapes in certain congenital heart diseases. Shows associated lung pathology. Chamber dilatation
Arterial blood gases; particularly useful in assessment of cyanotic newborns to determine the oxygen tension in the blood. Note hyperoxia
test should be done first.
Haematological profiles: PCV, esp to diagnose Polycythemia. Others include Prothrombin time and coagulation profile, iron studies, full
blood count,
Blood culture; infective endocarditis
Pericadiocentesis (diagnostic)
Electro physiologic studies (intracardiac electro physiologic testing): rhythm disorders
Cardiac catheterisation/angiocardiography: this is both for diagnostic and therapeutic purposes- helps in diagnosing complex heart
diseases, monitoring chamber oxygen saturation, and pressures. It can be used for therapeutics in catheter based closure of ASD, PDA,
Atrial septostomy balloon valvuloplasty. It still remains the definite procedure to diagnose cardiac defect in any patient.
CT SCAN: for cardiac tumors
MRI AND MRA: extremely helpful in diagnosis and management of congenital heart diseases. MRA is used to follow patient up especially
after repair of complex congenital heart diseases.
Magnetic resonance spectroscopy-research tool: demonstrates relative concentration of metabolites in the myocardium.
Electron beam CT: used for performing rapid respiration gated cardiac imaging esp in studying vascular abnormalities.
Radionuclide angiogragraphy: used to detect and quantify shunts and to analyze the distribution of blood flow to each lung.

83
Compiled by Obasi. D. C. Chinedu

COMMON SKIN DISEASES IN CHILDREN


By Dr. O.U. Ezenwosu

Outline: - Morphology of the skin - Functions of the skin - Introduction - Definition of terms
 Classification - Common skin lesions in children
At the end of the lecture: Students should be able to recognize and deal with some of the more common paediatric skin conditions that you
are likely to encounter.

Functions of the skin


 Protection from UV radiation, dehydration, micro-organisms, mechanical/physical injuries
 Sensation- cold/heat, pressure/touch, pain
 Temperature regulation
 Immunity- interaction with body immune system via langerhans cells, epidermal dendritic cell, phagocytic cells
 Excretion- water, ammonia, urea, uric acid via sweat
 Endocrine- synthesis of vitamin D (7DHC reaction with UV light)

Introduction: Recognition of common skin conditions is a skill that is extremely valuable in all areas of medicine. In paediatrics in
particular, it is important to have the ability to identify skin lesions, as such knowledge will enable you to; recognize potentially significant
systemic diseases, such as meningococcemia or chicken pox, as well as reassure concerned parents.
As the skin serves as a critical barrier to infection and dehydration, it is important to bear in mind that disease or impairment of the skin’s
normal function can lead to significant morbidity and mortality.
Although many skin disorders are easily recognized by simple inspection, history and physical examination are necessary for accurate
assessment. Examine: the entire body surface, mucous membrane, conjunctiva, hairs, and nails.

Lesions: Inspect, Palpate, Classify (Morphology, Size, Colour, Location, Distribution, Texture, Firmness). Lesions can be Primary,
Secondary or Special.
Primary lesions: The original lesions are known as primary lesions which are as follows: Macules, Papules, Patches, Plaques, Nodules,
Vesicles, Bullae, Pustules, Wheals and cysts.
Secondary lesions: the primary lesions may continue to full development or may be modified by a secondary factor such as infection,
trauma or therapy. They are as follow: Scales, Ulcers, Erosions, Excoriations, Fissures, Crusts and scars.
Special lesions: lesions that are produced by special circumstances/conditions of the body system. They are: Erythema (erythema
multiforme, erythema nodosum), Telangiectasia, Purpura, Petechiae, Ecchymosis, Haematoma.

Definition of Terms (Primary lesions)


Macule – an alteration in skin colour <1cm and cannot be felt Patch - alteration in skin colour >1cm
Papules – palpable solid lesions smaller than 1cm Nodules – palpable solid lesions >1cm
Vesicles – raised, fluid-filled (clear fluid) lesions 0.1-1cm in diameter
Bullae – raised, fluid-filled (serous/sero-purulent) lesions >1cm in diameter
Pustules – eruptions/elevations containing purulent material Plaques – aggregations of papules and pustules
Wheals – flat-topped, palpable lesions of variable size and configuration that represent dermal collections of oedema fluid.
Cysts – circumscribed, thick-walled lesions located deep in the skin, covered by a normal epidermis and contain fluid or semi-solid material

Definition of terms (secondary lesions)


Scales – compressed layers of stratum corneum cells retained on the skin surface
Erosions – focal loss of the epidermis; usually heal without scarring
Ulcers – extend into the dermis and tend to heal with scarring
Excoriations – ulcerated lesions inflicted by scratching, often linear or angular in configuration
Fissures – splitting or cracking, usually occurring in diseased skin
Crusts – matted, retained accumulations of blood, serum, pus, and epithelial debris on the surface of a weeping lesion
Scars – end-stage lesions that can be thin, depressed and atrophic, raised and hypertrophic, or flat and pliable

Definition of terms (special lesions)


Erythema – redness of skin due to dilatation of blood vessels which may be localized or widespread near the skin surface
Telangiectasia – Permanent dilatation of blood vessels
Purpura – bleeding into the skin and mucous membrane from the capillary at the arterio-capillary junction which does not blanch on
pressure and has progressive colour change
Petechiae – superficial pin-head sized (<3mm), round haemorrhagic macules, initially bright red and later brownish or rust-coloured
Ecchymoses – signify a deeper and extensive interstitial haemorrhage, which forms a flat, irregularly shaped bluish-purplish patch
Haematoma – collection of extravasated blood in a dead space in a tissue that is of sufficient size to produce swelling that fluctuates on
palpation

84
Compiled by Obasi. D. C. Chinedu

Classification of Skin Diseases in Children: Grouped aeliologically:


- Bacterial - Fungal - Viral - Infestations by animals
- Allergic reactions - Eczema or dermatitis - Nutritional disorders
- Pigmented disorders - Papulo-squamous eruptions - Skin tumours

Bacterial skin infections: They include; Impetigo (Non-bullous, Bullous), Cellulitis, Scalded skin syndrome, Pyoderma, Folliculitis,
Sycosis, Furuncles and carbuncles, and Tb of the skin.
Fungal skin infections can be
Superficial: Tinea capitis, Tinea corporis (T. faciale, T. axillaris, T. cruris, T. pedis, T. manum, T. unguium), Tinea versicolor (ptyriasis
versicolor), Candidiasis.
Deep: Actinomycosis, Blastomycosis, Coccidiodomycosis, Histoplasmosis, aspergillosis.

Viral skin infections such as; Warts (verrucae), Herpes simplex, Varicella (chicken pox), Herpes zoster (shingles) and Molluscum
contagiosum
Infestations by animal parasites like; Scabies, Pediculosis (lice infestation), Larval migrans (creeping eruptions), Tumbu fly, Jiggers,
Papular urticarial, Bed bug bites, Filarial infestations, Onchocerciasis, Loaiasis (loa loa) and Dracunculiasis (guinea worm).

Eczema or dermatitis; Endogenous eczema, Atopic eczema, Infantile eczema, Seborrheic eczema, Nappy (diaper) dermatitis.
Allergic reactions: Primary irritant dermatitis, Allergic contact dermatitis, Photo-allergic dermatitis, Generalized exfoliative dermatitis.
Papulo-squamous (scaly) eruptions: Pityriasis rosea, Lichen planus, Psoriasis.
Nutritional dermatosis: Acrodermatitis enteropathica, Kwashiokor, Pellagra, Scurvy, Vitamin A deficiency.
In Neonates: Impetigo neonatorium, Milia, Sebaceous hyperplasia, Harlequin colour change, Salmon patch, Mongolian spots, Erythema
toxicum, Transient neonatal pustular melanosis

IMPETIGO CONTAGIOSUM: More common on exposed parts. Vesicles or bullae develop on erythematous bases, rapidly enlarge and
rupture releasing yellowish exudates. Satellite lesions quickly develop where the exudate touches. The ruptured vesicle or bullae are covered
by exudates which dry to form crusts or scabs. Commonly affects children <5 years. Predilection sites are face, neck, scalp, hands and
feet.
Diagnosis: Microscopy and culture of swab from the lesion. Most common organisms are S. aures and GAβHS
Differentials: Herpes simplex, Varicella, Tinea corporis
Prognosis: Mild cases may resolve spontaneously within 1-2 weeks. Prompt and adequate treatment results in ready resolution. Untreated
may result in cellulitis, bacteraemia, AGN
Treatment: Mild cases respond to topical 1% GV or topical antibiotics (mupirocin, fucidin, neomycin). Moderate cases – oral Flucloxacillin
30-50mg/kg/day in 3-4 divided doses; Erythromycin 30-50mg/kg/day in 3 divided doses X 10/7. -Severe cases may require parenteral
antibiotics

TINEA CAPITIS: Dermatophyte infection of the scalp and scalp hairs. Found commonly in school age and regress toward adolescence.
Presents as dry, scaly, itchy, annular patches with weak and broken hair shafts
Organisms: Microsporium audouini, M. canis, T. tonsurans.
85
Compiled by Obasi. D. C. Chinedu

Frequent scratching may lead to secondary bacterial infection and cellulitis. Host’s cellular immune response may lead to development of
inflammatory or infected mass (kerion). Untreated/inadequately treated cases may develop scarring alopecia.
Diagnosis: Clinical picture and Wood’s lamp examination (fluorescence). Confirm by microscopic exam of KOH preparation. culture of
scrapings in Sabouraud’s dextrose agar
Differentials: alopecia areata, psoriasis, discoid lupus erythematosus
Treatment: oral/topical Ketoconazole, tioconazole, itraconazole/ Oral Griseofulvin 10mg/kg/day or Whitfield’s ointment.

Others:
Tinea corporis is similar to T. capitis
Tinea versicolor (Pityriasis versicolor): Caused by Malassezia furfur. Seen commonly in childhood and adolescence. yellowish-brown
macules in patches of dry scales. common in upper trunk, shoulder, neck and face.
Diagnosis is same as other tinea infection.
Treatment: Selenium sulfide 5-10 mins each day x2wks/ 4-6% salicylic acid bd x 2/52/ miconazole, clotrimazole, ketoconazole, or
terbinafine bd x 2/52, fluconazole.

Varicella (Chicken pox): Caused by varicella zoster virus (VZV). Spread by respiratory route. Incubation period is about 2 weeks.
Clinical features include: malaise, itchy papules and clear vesicle. papules & vesicles become pustular, break down and crust. lesions appear
in crops. centripetal in distribution.
Treatment: Severe & immunocompromised (Acyclovir 10mg/kg 4-6hrly. Broad spectrum antibiotics against systemic involvement). Mild
and moderate (self-limiting. Use antihistamine and broad spectrum antibiotics).

Scabies: Caused by the itch mite, Sarcoptes scabiei. Highly contagious and transmitted by close personal contact
Presentation: Group presentation – from school, family, compound, barracks. itching is worse at night. frequent epidemics in refugees and
camps.
Common lesions: isolated or lines of papules.
-pustules, vesicles, bullae, crusts, scabs, erosions, excoriations
-cellulitis, pyoderma
NB: In infants = bullae and pustules common. In older children = thread-like burrows.

Common sites of lesion: In infants- palm, soles, face and scalp are often affected
In older children- webs, wrist flexors, anterior axillary folds, ankles, elbows, buttocks, umbilicus, waist, genitals, areola of the breasts, thighs,
natal cleft
Diagnosis: History and examination. Confirmed by identification of the mite in blunt scrappings from broken papules or vesicles examined
on a slide with a hand lens

Treatment: -Treat children with their groups (preferably at night) -Scrub with soap and sponge
-apply 25% Benzylbenzoate emulsion (from neck to toes) -repeat for 2-3 consecutive nights
-repeat course after 1 week -Can use soap containing 10% tetraethyl monosulphide (tetmosol)
-Relieve itching with chlorpheniramine -Antibiotics for secondary bacterial infection

Nappy (Diaper) Dermatitis: A form of irritant dermatitis. The skin around the nappy area may be erythematous and scaly. Sometimes with
papulo-vesicular or bullous lesions. Fissures and erosions may be present
Due to reaction to: overhydration of the skin, prolonged contact with urine and faeces, friction, retained diaper soaps, topical
preparations.
Characteristically, the genito-crural folds are spared
Treatment: -Frequent changing of the diapers - meticulous washing of the genitals
-try to maintain a relatively dryness -frequent topical application of a bland protective barrier agent (zinc
oxide or petroleum gel)
-if not responding, topical 0.5-1% hydrocortisone ointment
Complication: May be secondarily infected by candida

Impetigo neonatorium: Presents as thin flaccid bullae on any part of the body. Appears within hours or a few days and may spread
extensively. Caused by S.aures; sometimes Grp A Streptococci
Investiagtion: swabs; blood culture for diagnosis
Treatment: isolate infected newborn. Treat with an antistaph agent or broad-spectrum antibiotics
Complications: septicaemia, G/enteritis, pneumonia, meningitis

NOW, Can you be able to recognize and deal with some of the more common paediatric skin conditions that you are likely to encounter.

86
Compiled by Obasi. D. C. Chinedu

CHILDHOOD ACCIDENTS, POISONINGS


Including Bites, Stings and Envenomation
LECTURE BY DR. B.O. EDELU

Introduction: A common, but preventable cause of morbidity and mortality in children. Occur more in the homes than outside. Most home
accidents occur in the living/dinning room, but most serious accidents occur in the kitchen and staircase. More accidents occur during
school holidays and weekends. Children 1 – 4 years more affected. Boys more affected than girls. Type and nature of accidents are often
related to the age and level of development
 Types of accidents in children include: RTA, falls, burns, drowning and near drowning, animal bites, insect sting, accidental
poisoning, etc.
 Types and Nature of injury include: Fractures, dislocations, head injuries, bruises, lacerations, puncture wounds, burns, scalds,
asphyxiation, etc.

Risk factors for accidents in the home


1. Lack of or poor supervision of children
2. Family stress like death in the family, chronic illness, homelessness or change of home,
3. Parental carelessness and ignorance
4. Marital discord
5. Living with elderly relatives
6. Overcrowding
7. Lack of outdoor play facility
8. Alteration of home routine
9. Unfamiliar environment

Innate factors that predispose children to accidents and poisonings


1. Children are often absorbed in their immediate interest and can be oblivious of their surrounding.
2. They lack experience and have limited perceptions of their environment
3. Unaware of the consequences of the many new situations they encounter daily
4. They are usually curious and adventurous
5. Their small size may prevent them from seeing an obstruction or being seen
6. Unstable gait of toddlers predispose them to falls
7. They like to show off and exceeding their abilities especially among friends
8. Children like to imitate adults

Accidental versus Non-accidental


Child abuse is very common. Suspect intentional injury if:
- Child not a biological child of care giver - Injury is unexplained or unexplainable
- History is incompatible with type of injury - Injury is not compatible with child’s age
- Tell-tale signs of previous abuses - Emotional disconnect
- Any reason for being unwanted
Before reporting as a case of abuse, determine if the caregivers have the understanding, emotional, physical and financial resources needed
to provide for the child.
Management: Depends on the type, nature and severity of the injury

Accidental Childhood Poisoning: Accidental poisonings involve a person, usually a young child, ‘accidentally’ poisoning themselves
without wanting to cause harm to their body (as opposed to intentional poisoning or overdose). Children < 3 years are mostly involved.
Boys > Girls. Intentional poisoning is more common in adolescents.

This is most common in young children as they are keen to explore their world and often learn about new things by putting them in their
mouth. Most children do not know that swallowing poison can be harmful. A sour or bitter taste does not deter a child from swallowing a
poison, especially medications.

Common sources of poisoning:


Medications: Paracetamol, Cough syrups, Multivitamins, Haematinics, Antimalarials etc
Cleaning Agents: Bleach, detergents, turpentine, deodorants etc
Cosmetics: Lotions, ointments, shampoo, aftershave, perfumes
Insecticides and Pesticides
House hold fuel: Kerosene, Diesel
Poisonous plants

87
Compiled by Obasi. D. C. Chinedu

Clinical Manifestation: depends on the nature and quantity of poison ingested. Also, the immediate intervention instituted. May include
nausea, vomiting, abd pain, drowsiness, respiratory distress etc.

Toxic Syndromes
Syndrome CNS Pupils Others

Cholinergic (Organophosphates, Restlessness, Depression, Pinpoint Nausea, Vomiting, Diarrhoea,


Carbamates) Seizures, Coma Hyper secretion, Muscle
fasciculation

Anticholinergics (Antihistamines, Delirium, Hallucination, Dilated Hyper secretion, Urinary


TCA) Hyperthermia retention, Thirst, Flushed
skin

Extrapyramidal (Metoclopromide) Occulogyric crises, Unremarkable Dystonia, Torticollis,


Conscious Dysphonia

Sympathomimetic Psychosis, Seizures, Dilated Tachypnoea, Tachycardia,


(Cocaine, Theophyllines, Hyperthermia, Tremors, Hypertension
Amphetamines) Restlessness

Narcotic/Opioid (morphine, Depression, Hypothermia Pinpoint Hypotension, Resp.


Codeine) depression

Management: Follow initial ABCDE protocol


GIT decontamination if presentation is early
Emesis - Usually with Syrup of Ipecac. Not advisable at home and in infants <6mths. Onset of emesis is usually 20 – 30 mins. Do not use if
unconscious, caustic or hydrocarbon ingestion.
Gastric lavage: - Aspirate with a large bore tube and flush severally using normal saline. Do not use with caustic, hydrocarbon ingestion.
Activated charcoal – Adsorbs the poison, but must be given within one hour for better efficacy. Given usually in a solution of sorbitol, a
cathartic to enhance elimination. It does not work with substances like heavy metals, hydrocarbons and alcohols.
Whole Bowel irrigation – Useful with heavy metals like iron, lead and lithium and enteric coated tablets. Large volumes of polythene
glycol (colonic lavage solutions): Colyte, Go-LYTELY. Do not use in unconscious child or suspected bowel obstruction.
Antidote – Can be given for some poisons to reverse the effects.
Observation – Some poisons have a delayed effect and the child may have to be admitted in hospital, possibly overnight. Monitoring of
heart rhythms and checking other vital signs such as blood pressure and oxygen levels.
Toxin screen – To check the level of poison in the blood. This helps decide further treatment. In most cases the level is very low and no
problems are expected.
Dialysis – Haemodialysis and Peritoneal dialysis in selected poisoning e.g. methanol, salycilate, theophylline.
Counseling

Further Reading: Specific antidotes for Common poisoning agents

Prevention of Accidents and Poisoning in Children


1. Safety education
2. Improved supervision of small children
3. Improved home design
4. Keep small children out of the kitchen as much as possible
5. Choose age-appropriate toys
6. Always store household chemicals in their original containers, Label appropriately.
7. Keep all medicines and chemicals out of sight and reach of children.
8. Never refer to medicines as ‘sweet’
9. Keep children off the streets as much as possible
10. Simple safety measures

SPECIFIC POISONING: Kerosene and organophosphate poisoning


Kerosene poisoning: Most common agent involved in childhood poisoning in our environment. Kerosene is a large molecular weight
hydrocarbon commonly found in the homes. Highly volatile and of low viscosity and surface tension and thus easily aspirated even when
there is no vomiting. Most manifestations are as a result of aspiration (inhalation) as it is minimally absorbed in the GIT. However, if

88
Compiled by Obasi. D. C. Chinedu

a large amount is ingested, significant concentration can be found in the various organs. Aspiration causes chemical pneumonitis which
can manifest with cough, dyspnoea, wheeze, rhonchi and crepitations on auscultation. Symptoms can occur within minutes or may be
delayed for as much as 6 hours.
Pathological changes in the lungs are bronchospasm, atelectesis, emphysema, bronchiolar oedema and necrosis.
Radiological changes may be present in children who are asymptomatic. Other symptoms include nausea, vomiting, diarrhoea from the GIT.
In large ingestions, headache, irritability, blurring of vision, convulsions and coma may occur. There may be secondary bacterial infection.
If no symptom, observe for at least 6 hours and discharge. Monitor O2 saturation. Chest radiograph may be necessary.
Treatment depends on the presentation and it’s mainly supportive. Oxygen administration if indicated, and in severe cases artificial
ventilation. Antibiotics, only if secondary infection is suspected. Steroids are contraindicated, likewise attempt at lavage.

Organophosphates/ Carbamates Poisoning: Common cause of poisoning worldwide and are frequently used in suicides.
Organophosphates are phosphoric acid or thiophosphoric acid esters. Examples include; malathion, parathion, diazinon, fenthion, dichlorvos,
chlorpyrifos, ethion used as insecticides. Can also be found in herbicides (tribufos [DEF], merphos), nerve gases, ophthalmic agents and
antihelmintics (trichlorfon). Carbamates are esters of N-methyl carbamic acid. Examples: Aldicarb, carbaryl, propoxur, oxamyl and
terbucarb.
Although, they differ chemically, they act similarly, hence they are both used as insecticides and pesticides in homes and farms. Marketed
under a wide variety of trade names eg, Otapiapia, Sniper.

Poisoning can occur via ingestion, cutaneous absorption or inhalation. They inhibit acetylcholinesterase in nerve synapses and on
red-cell membranes, resulting in accumulation of Acetylcholine (ACh). Also inhibit butyrylcholinesterase in plasma. Carbamates are
cleared spontaneously within about 48 hrs after exposure but Organophosphates, however, can bind irreversibly.
Although most patients rapidly become symptomatic, the onset and severity of symptoms depend on the specific compound, amount, route of
exposure, and rate of metabolic degradation

Clinical Features
MUSCARINIC:
SLUDGE (S= Salivation, L=Lacrimation, U=Urination, D=Defecation, G=GI symptoms, E=Emesis.
DUMBELS (D=Diarrhea and diaphoresis, U=Urination, M=Miosis, B=Bronchorrhea, bronchospasm, and bradycardia, E=Emesis,
L=Lacrimation, S=Salivation)
Nicotinic eg, Muscle fasciculations, twitching, tremors, Resp. & Skeletal muscle weakness. In severe cases, Flaccid paralysis.
General CNS Effects eg, drowsiness, mental confusion, anxiety, ataxia, convulsion and coma

Management
Airway /ventilation support: Endotracheal intubation for those with weak or no breathing effort.
Decontamination; Remove all clothings and wash with saop, bath skin, nails, hair properly with soap.
Gastric decontamination with activated charcoal if ingested.
Atropine: Blocks central and muscarinic receptors. NO effect on nicotine receptors. Large dose of atropine is repeated every few minutes
till signs of atropinisation- (warm dry flushed skin dilated pupil, increased HR)
Pralidoxime (2-PAM), a cholinesterase re-activator (reverses nicotinic effects) is also given in severe cases and repeated every 6 – 12 hrly
Measure RBC cholinesterase level. Less than 25% of normal indicate significant exposure
Benzodiazepines are used for seizures. Monitor and correct hypoglycaemia
Prognosis depends largely on the type and quantity of poison as well as the route. Also, the time of presentation and availability of
appropriate support therapy. Carbamates are less toxic than organophosphates. Death is usually from respiratory failure due to pulmonary
oedema and respiratory paralysis. Chronic neurologic sequelae can occur

BITES AND STINGS


Snake Bite: Epidemiology
There are over 3,500 snake species worldwide but only about 200 are poisonous to man. Most bites do not result in envenomation. Peak
period between May and October since snakes are poikilothermic. 97% of all snakebites, are on the extremities, with two thirds on the
upper extremities and one third on the lower extremities. Males are bitten nine times more frequently than females. Only about 8.5 percent of
snake bite victims attend hospitals in Nigeria.

Classification of Venomous snakes


The Elapidae: Cobras, Mambas, Coral snakes
The Viperidae (True vipers): Russell’s viper, the puff adder, gaboon viper, saw-scaled viper, and European viper
The Crotalidae (Pit vipers): Rattlesnakes, water moccasins, copperheads, bushmaster.
The Hydrophidae: Water snakes

Examples of Non venomous snakes


- Pythons - Boa constrictors -Anacondas - Corn snake - Rat snakes
89
Compiled by Obasi. D. C. Chinedu

- King snakes - Gopher snakes - Four-lined snakes - Smooth snakes - Grass snake

How to identify venomous snakes


Most venomous snakes have triangular shaped heads while non- venomous mostly have round heads. A venomous snake normally has
a heat-sensitive pit there to locate warm-blooded prey. Most venomous snakes have one row of scales at the underside of the tip of the tail
while non venomous have two rows. The bottom of the tail (behind the anus) looks the same as the rest of the belly but if the snake has a
cross pattern (like a diamond shape) it is non-venomous. A snake with a rattle on its tail must be a rattlesnake, which is venomous.
Only venomous water snakes swim with their entire bodies visible on the water. Two close-set puncture marks at bite site indicate
that the snake has fangs and is venomous. By contrast, a ragged bite mark caused by non-venomous snakes. Colour of snake is not a good
discriminator of non venomous snake

Symptoms of snake bite: Symptoms can generally be classified as local or systemic.


The local effects are generally caused by enzymatic action on the various cellular and non-cellular structures in the victim’s tissues. These
enzymes can cause coagulation, anticoagulation, cell lysis, hemorrhage, hemolysis, and the destruction of nucleic acid,
mitochondria, and other organelles.

The signs and symptoms of a venomous snakebite vary considerably and depend on several factors.
1. Size and species of the snake (relative toxicity)
2. Amount of venom injected, number of bites, location and depth eg head and trunk or blood vessels more severe than extremities.
3. Time elapsed before presentation
4. Age, size and underlying medical condition
5. Fangs; broken or intact
6. Venom glands: empty or full

Elapidae/Hydrophidae: Primarily neurotoxic. They act by blocking neuronal transmission at the neuromuscular junction. In addition to being
neurotoxic, the Hydrophidae venom are also myotoxic causing muscle cell injury. Many of these species’ venoms contain compounds that
block neuromuscular transmission at acetylcholine receptor sites and have direct inhibitory effects on cardiac and skeletal muscle. Ptosis is
common and often the first outward sign of envenomation. Other signs and symptoms include vertigo, paresthesias, fasciculations,
slurred speech, drowsiness, dysphagia, restlessness, increased salivation, nausea, and proximal muscle weakness.
Little pain & swelling may occur. Death from envenomation is usually by respiratory depression.

Viperidae/crotalidae: Primarily cytolytic and haemotoxic. Acts by causing cellular necrosis, vascular leak, hemolysis and coagulopathy.
Petechiae, ecchymosis, and serous or hemorrhagic bullae are other local signs. Necrosis of the skin and subcutaneous tissue is noted later.
Spontaneous bleeding and prolonged bleeding from fang and venipuncture sites occur.
Non specific systemic symptoms, such as weakness, nausea, fever, vomiting, sweating, numbness and tingling around the mouth, metallic
taste in the mouth, muscle fasciculations, and hypotension, often occur. Death is associated with a disruption of the coagulation
mechanism and increased capillary membrane permeability.
Ultimately these two processes lead to massive pulmonary edema, shock, renal failure and death

Emergency Department Care


Ensure ABCDE: Pulse oximetry, Supplemental oxygen if indicated, Cardiorespiratory monitoring, IV access with at least 2 lines.
Manage for shock, acute respiratory failure and acute renal failure. Assess need for administration of antivenom

Patient’s History: Specific historical information should include the time elapsed since the bite, circumstances surrounding the bite, number
of bites, whether first aid was administered and what type, location of the bite, and any symptoms.
A brief medical history should include the last tetanus immunization, medications, and cardiovascular, hematologic, renal, or
respiratory problems. Allergy history

Examination: The bite area should be examined for signs of fang marks or scratches and local envenomation (e.g., edema, petechiae,
ecchymosis, bullae). The area distal to the bite should be checked for pulses.
A general physical examination should be performed with emphasis on the cardiorespiratory system, Haematologic, respiratory and
neurologic systems. If the bite involves an extremity, the circumference of the extremity at the site of the bite and approximately 5
inches proximal to the bite should be measured and recorded.
These data aid in objectively estimating both the spread of the venom and the effect of antivenin.

Basic investigations that should be done include:


- Blood grouping and cross matching
- FBC + platelets (↓PCV,↓platelets, ↑WBC) in haemotoxic envenomation.
- Clotting profile
- 20 minute Whole Blood Clotting Time

90
Compiled by Obasi. D. C. Chinedu

- S/E/U/Cr (evidence of renal failure)


- Urinalysis (blood, hemoglobin, myoglobin, protein)
- Serum Glutamine Oxaloacetate Transaminase and Creatinine Phosphokinase (↑ indicates skeletal muscle damage).
- ECG – patients with arrhythmias.
If a compartment syndrome is suspected, a pressure monitor should be inserted and surgical consultation obtained. Pressures greater than
30 mm Hg may require fasciotomy.

Antivenin: Usually horse or sheep derived. Snake antivenom can be classified by which antigens (venoms) were used in the production
process:
If the neutralizing antibodies is obtained from a single species, then it is considered a Monovalent antivenom. (fewer side effects)
If the antivenom contains neutralizing antibodies raised against two or more species of snakes - Polyvalent.
A skin test should be performed ideally in every patient before each administration of any foreign antiserum. Children are given the same
dose as adults as envenomation does not discriminate size.

The initial dosage of antivenin is prepared based on assessment of severity (see chart below). Dissolve each vial as instructed by
manufacturers in normal saline (≈5-10 ml/kg) and administer as an infusion over an hour
In case of anaphylactoid rxn /anaphylaxis:
- Interrupt administration of antivenom
- Give IM Adrenaline 0.01ml/kg stat
- IV Hydrocortisone 5mg/kg stat.
- IV Chlorpheniramine 0.2mg/kg stat. (alt. Promethazine)
- Manage for shock if patient is in shock.
- Tepid sponge and antipyretic, if febrile.

Estimation of antivenin based on Classification of Envenomation


Severity of Clinical features Amount of antivenom
envenomation

Mild Progressive local swelling +/- lymphadenitis and local 5 vials


purpura/ecchymoses

Moderate (2) Mild systemic signs or coagulation defects/ haematological 5-15 vials
changes / nausea, vomiting

Severe (3,4) Rapidly progressive swelling with extensive local effects, systemic 15-20 vials
signs and symptoms, shock, paralysis, encephalopathy

Dog Bite: Domestic dogs are the main source of human rabies, contributing up to 99% of all rabies transmissions to humans. Infection
causes tens of thousands of deaths every year, mainly in Asia and Africa. 40% of people bitten by suspect rabid animals are children under
15 years of age. Rabies elimination is feasible through vaccination of dogs and prevention of dog bites.

Further Readings: - Dog bite - Bee sting - Scorpion sting - Submersion injuries (Drowning and near drowning)

91
Compiled by Obasi. D. C. Chinedu

BIOETHICS
Dr Obidike EO (Professor of Paediatrics)

OUTLINE.
 Introduction: - Definition - Origin of bioethics - Scope.
 Principles of Ethical considerations - Issues in Bioethics
 Informed consent/assent. - Counselling.
 Ethical Situations in Paediatrics - Conclusion

INTRODUCTION: Field of allied health care that is inter- or multi- disciplinary - law, philosophy, theology, medicine, the life sciences,
nursing and social science - and is evolving.

DEFINITION: Generally - a study of philosophy as well as a study of biology.


Philosophy – study of proper behaviour and search for wisdom – the understanding of ‘how’ and ‘why’ people do the things they do and
what is involved to live a good life.
Biology – study of life and living organisms – physical structure, chemical composition, function, development and evolution.
Specifically - the application of ethics to the field of medicine and healthcare.
Ethics - philosophical discipline concerned with the idea of good and bad, right and wrong i.e. our moral life in community. In other words, it
is concerned with the perceived risks and benefits of advances and technologies in medicine and healthcare, and their impact on society.

ORIGIN OF BIOETHICS
Bioethics - branch of Applied Ethics originated from the investigation and study of ways in which decisions in medicine and science affect
the health and lives of people, society and environment and this has been with us for over 20 years.

SCOPE.
o Assisted procreation
o Human cloning
o Biodiversity [number, variety and variability of life on earth – terrestial, marine and other aquatic ecosystems – at the genetic (within
species), species (between species) and ecosystem (between ecosystems) levels]
o Protecting the integrity of participants in therapeutic trials
o Organ trafficking
o End of life
o Biotechnology (exploitation of biological processes through genetic manipulation of microorganisms for production eg antibiotic,
hormones)
o Nanotechnology (working at atomic, molecular or submolecular levels to produce small particles capable of delivering anticancer drugs
or turning polluted water into drinking water).
o Genetically modified organisms.
o Access to medicines and health care devices.
o Environmental ethics

PRINCIPLES OF ETHICAL CONSIDERATIONS (excerpted from Beauchamp and Childress (2008).


o Principle of respect for autonomy
o Principle of beneficence
o Principle of non-maleficence
o Principle of justice.
o and additional
o Confidentiality
In conducting researches, it is mandatory that the methodology of any such work will be subjected to a review by the appropriate Ethical
committee for such institutions to make sure subjects for the reasearch are ethically considered.

ISSUES IN BIOETHICS
1. Doctor-patient relationships
2. Medical decision making

92
Compiled by Obasi. D. C. Chinedu

Medical Resource Allocation/Healthcare rationing.


Futility of medical care in certain patient groups
Eugenics – set of beliefs and practices that aim to improve the human population genetic quality
Euthanasia/Physician-assisted suicide
Organ Donation
End-of-Life Care/palliative care
3. Patients’ rights Involvement in cases that require unbiased patient advocacy

VALUES AT STAKE
a) Human life b) Just healthcare c) Bodily integrity and d) Ability to make reasonable decisions

INFORMED CONSENT/ASSENT
The concept of informed consent stems from the fundamental ethical principle of the right of self determination. This principle recognizes
that patients are autonomous, independent agents with the right to make decisions regarding their well-being without coercion from others
Medically it is based on ethical doctrines and is the process of communication by which a patient learns about and understands the purpose,
benefits, and potential risks of a medical or surgical intervention, including clinical trials, enabling the patient to make an informed and
voluntary decision to agree to receive or decline the treatment or participate in the trial.

The ethical doctrines include


a) Autonomy
b) Beneficence
c) Justice or sometimes as equality and occasionally
d) Non-malfescience

Obtaining it is a legal and ethical necessity before treating a patient. Being an act of reason, the person consenting must be sane and of
sufficient age to be capable of giving consent. Documentation of the contents of this conversation as part of the permanent medical record by
the physician is also critical. Three elements involved are;
1. Full disclosure of the nature of the research and the participant's involvement,
2. Adequate comprehension on the part of the potential participant, and.
3. The participant's voluntary choice to participate.

THINGS TO DO
a) Give subject adequate information concerning the treatment/study
b) Provide adequate opportunity for the subject to consider all options
c) Respond to the subject's questions
d) Ensure that the subject has comprehended this information
e) Obtain the subject's voluntary agreement.

TYPES OF CONSENT
Verbal – here, that subjects are read a verbal version of a consent form (i.e., an information sheet - see below section) and give their verbal
consent in place of written consent to participate eg explanations for patient to accept before starting treatment or carrying out procedures
and sometimes in researches
Written – Is an agreement in writing after reading through/being read the contents of a consent form eg at surgery, some procedures,
researches.

CONSENT FORM: A form signed by a patient prior to a medical procedure (inclusive medical research) to confirm that he or she agrees to
the procedure and is aware of any risks that might be involved. The primary purpose of the consent form is to provide evidence that the
patient gave consent to the procedure in question. Very important in cases of litigation!
Note: Consent may be waived in certain situations eg a critically ill patient.

CHALLENGES IN OBTAINING INFORMED CONSENT


- Language Barriers. - Religious Influence. - Cultural. - False Expectations.
- Patient Perceptions. - Children. - Vulnerable People and Groups.
- Risk of worsening of illness. - Use of placebo and - Validity of informed consent

ASSENT: "Assent" is a term used to express willingness to participate in research by persons who are by definition too young to give
informed consent but who are old enough to understand the proposed research in general, its expected risks and possible benefits, and
the activities expected of them as subjects.
Assent by itself is not sufficient, however. If assent is given, informed consent must still be obtained from the subject's parents or guardian.
AGE OF ASSENT: Seven to Twelve

93
Compiled by Obasi. D. C. Chinedu

COUNSELLING: The advent of Bioethics in healthcare made health workers appreciate the patient as a person better and also enrich their
abilities in caring for the patient. The consequence was that health workers were now able to follow an ethical code when working with
patients (Consent) which was once a problem and by so doing, improve healthcare significantly. Patient counselling therefore formed the
bedrock of these.
Generally speaking, counselling in itself is a talking therapy that helps a trained therapist in listen/communicate with a patient/client and
help such to find ways to deal with emotional or other social issues.

COUNSELLING SKILLS
- Counselor Self-Disclosure. - Multicultural Competency - Empathy. - Genuiness.
- Rapport-Building Skills - Social and Communication Skills. - Flexibility
- Concreteness. - Focusing. - Clarifications. - Critical Thinking.
- Problem-Solving Skills. - Summarising.
- Open Questions - A questioning process to assist the client in clarifying or exploring thoughts.

ETHICAL SITUATIONS IN PAEDIATRIC PRACTICE


Occasionally, ignorance, religion, culture, poverty and social issues create ethical issues in managing sick children where their parents are
concerned! Some examples:
a) A very sick child whose life or healthy future depends on the use of blood and the parents objecting on the grounds of religion.
b) A sick child on or needing admission and the parents for financial reasons objecting to the admission or further treatment/investigation.
Can also be in the form of mother not being able to take decisions because child’s father is not around.
c) A child that convulses and needs regular hospital visits for assessment and medication but parents believe it is not a problem for
hospital.
d) Sometimes one can confront a situation where as in HIV infection, a child and the mother are positive but father negative.
e) Challenge of determining the sex in a case of ambiguous genitalia.
f) Parents with very poorly developing or sickly child/children e.g. Downs syndrome, Congenital Heart Defects, sickle cell disease
requiring continuous hospital attention.
g) Immunization of subsequent child/children following previously failed protection.
h) Failed outcome – morbidity or mortality – of a previously thought safe intervention – medicines or surgery.
i) In vitro fertilisation using stored or another man’s semen.
j) A need for post mortem on a dead child being refused by child’s parents.

AVAILABLE MEANS OF HANDLING THESE ISSUES


1. Generally, and pre-emptively, effective use of counselling as one makes contact with the patient to give necessary information on the
disease process and expectations and risks and benefits of intended interventions.
2. Where poverty is the issue, efforts to secure a waiver of fees or drastic reduction in such if such facility is available in the hospital.
3. In hospitals with the Social Welfare department, they are often useful in helping to resolve some of these issues.
4. In climes where it is available, resort to use of dedicated court processes to obtain an order for the state to take custody of the child for
the period that the treatment will be necessary and after that restore child to the parents and of course, with the parents repaying the
hospital expenses particularly if capable.
5. Where all else fails or non availability of courts designated to handle such matters, such parents are asked to sign for the hospital a
Discharge Against Medical Advice form after being informed about the consequences of such a step.
6. If the ambiguous genitalia is detected early, the determined actual biologic sex of the child in most cases will be acceptable to the
parents except where sexes of already born children creates its social issues. If later however when parents would have given the child a
social sex identity which may not agree with the biologic sexuality, it then becomes more intricate – discussions with parents here will
involve the role of the paediatrician especially a paediatric endocrinologist, the paediatric surgeon, the psychologist, the social worker
and sometimes the priest.
7. A HIV negative father with a HIV positive child and mother throws up one of the most difficult ethical dilemmas in practice because of
the societal stigma attached to HIV and the fact that people still think it is not curable – to tell the father or not to tell! The standard of
care is that he has to be told. However, that should be after counselling in sessions for him to be prepared for the news and also
determining his psychological mindset as that benefit may turn out with unbelievable consequences.

CONCLUSION:
Bioethics is a necessary and timely intervention in the practices and developments in medicine that, applied correctly, will improve
acceptability of patient care delivery and participation in researches by patients and subjects respectively as well as be fulfilling for the
physicians/researchers.

94
Compiled by Obasi. D. C. Chinedu

CONCEPT IN MANAGEMENT OF PAEDIATRIC EMERGENCIES


Prof Oguonu

95
Compiled by Obasi. D. C. Chinedu

METHODS IN ASSESSMENT

96
Compiled by Obasi. D. C. Chinedu

97
Compiled by Obasi. D. C. Chinedu

Case 4: 16 Month old girl is taken to the ER with increasing lethargy and dehydration with 5 days of profuse diarrhea and
poor P.O. What will be the immediate step?

98
Compiled by Obasi. D. C. Chinedu

99
Compiled by Obasi. D. C. Chinedu

100
Compiled by Obasi. D. C. Chinedu

101
Compiled by Obasi. D. C. Chinedu

102
Compiled by Obasi. D. C. Chinedu

INVESTIGATIONS IN PAEDIATRICS I & II


Dr. Ujunwa

Introduction:
In Paediatrics and medical science the diagnosis of diseases rely mainly on history physical examination and investigations. The clinician
decides the investigations that should be done whether it is diagnostic or supportive for the said diagnosis. The investigations ordered is
influenced by technique facilities, personnel, expertise and cost.

There is a continuous improvement in the range of investigations used in diagnosis of diseases in children. Clinicians need to know the
options available and determine the ones that are relevant to their practice in their environment and in particular disease condition. The
quality of patient care depends on a thorough bedside assessment.

A good history and physical examination enables a physician to entertain a list of differential diagnosis and order relevant investigations.
Good history is the richest source of information concerning a patient and helps to avoid unnecessary and expensive investigations. The
ability of not over investigating a patient and still manage the patient is a hallmark of a good clinician. Always remember that both the
invasive and non invasive investigations are sources of concern to parents.

Investigations used in management of different systemic diseases in children are vast. The clinician should be aware of the rationale for the
tests.
In ordering the tests the following factors must be considered in addition: Reliability of the test (sensitivity and specificity),
Reproducibility(precision) and Interpretation, adverse reaction, cost benefit and cost effectiveness.

Aims of investigations:
 Confirm a clinical diagnosis or assist in diagnosis
 To assist in assessment of disease severity, and the prognosis.
 Monitor disease progression
 Assist in planning treatment
 To exclude or eliminate other co-founding diagnosis or alternative diagnosis
 Measuring of concentration of substance in the body
 Screening for diseases.

Steps in investigating a patient:


Following adequate history, physical examination and clinical diagnosis, it is important that the doctor before investigating should;
Do patient education and consent. Ensure Proper specimen with proper technique. Ensure proper patient preparation, timing of the
investigation, amount and source of sample needed, container, label. Transportation in the appropriate medium. Adequate specimen
processing. Safety of personnel and self and proper disposal of waste
Quality assurance; checking the procedures periodically and Confidentiality in reporting and disclosure of results.

With the plethora of investigations available it is the duty of the clinician to use these tests judiciously either singly or in combination in the
most appropriate way that will help in management of the specific or group of diseases in children. when more than one test is available
for one investigation the test with the higher and cost effective diagnostic value should be chosen.

Source of error in investigation: They include


- Misdirected clinical diagnosis - Improper technique - Poor sample handling
- Non adherence to protocol in terms of timing, preparation , transportation, storage and processing
- Contamination of specimen - Variations in technique.
- Biological variations: age, sex, race etc - Cross reactions and drug interactions.

Generally investigative procedures in Paediatrics can be invasive and non invasive. The term non invasive connotes a technique which
presents no discomfort, trauma, pain or significant risk to the patient when properly performed a can easily be done in the lab, bed side
or office. Inherent safety and repetitiveness are added advantage.eg blood pressure monitoring, pulse oximetry, ultrasound, ECG etc

Invasive techniques` connotes that the body is entered either by a needle, tube, device scope which results in some discomfort, tissue damage
or may cause temporary or permanent pathologic complication. Eg are abdominal paracentesis, bone marrow biopsy, arterial blood sampling,
venous blood sampling pleural tap, pericardiocentesis etc. These procedures can be used for diagnostic and therapeutic purposes in
Paediatrics.

Invasive haematological investigations:


Blood tests: this is usually obtained via venous sampling or arterial blood sampling, it is the commonest specimen used for tests. It has use in
various areas such haematology, microbiology, chemistry and immunology. Safety is paramount importance because it is highly infectious.
The test can be done using serum, plasma or whole blood.

103
Compiled by Obasi. D. C. Chinedu

Blood is usually drawn from site without intravenous infusion and tourniquet not applied too tightly in order to avoid microclots. Test
done has implications in various systemic illness.
Examples are Haemoglobin estimation; anaemia, polycythaemia. Packed cell volume

Full blood count; this is the most requested blood investigation in paediatrics practice. It usually gives a picture of blood and it is
component. Iit involves complete red blood cell count, Haemoglobin estimation, total white blood count, differential white cell count,
Platelets, retic count, MCH, MCHC, RDW, MCV, platelet distribution width, MPV, (mean platelet volume). It can help in diagnosing wide
range of diseases. Review treatment, monitor disease severity, and progression.
It is used in estimation of WBC, HB, Platelet, differential count of WBC is used as a supportive investigation in certain type of infections.
Derangement in Neutrophilic count associated with bacteria infections, inflammation. Lymphocytes raised in viral, protozoal
infections, Tuberculosis, malignancy. Eosinophils and basophils in allergic reactions. Monocytes in protozoal infections.

Blood film microscopy: this is done by making a thin and thick film. It helps in assessment of RBC morphology, abnormalities, presence of
blasts eg leukaemia. Malaria parasitaemia. Malaria parasite can also be detected using Rapid diagnostic test strip(MRDT) which detects
specific malaria antigens or enzymes. Other rapid test include HIV rapid tests; detecting antibodies to the virus.

Anaemias
Sickle cell test (sickling test and solubility tests); screening test for sickle cell disease. Sickling test detects the presence of HbS. Sickling of
RBC occurs under deoxygenated conditions.
Solubility tests- deoxygenated HbS is insoluble in solutions of high molarity
Hb electrophoresis used for definitive diagnosis of sickle cell.
Other tests done include Enzyme assays such as G6PD assay, pyruvate kinase assay. Quantitation of haemoglobins by liquid
chromatographic method for A, A2, F and S. Cord blood sampling-Hb Bart in thalasaemia.
Iron studies: serum ferritin, transferrin, total iron biding capacity, serum iron.
Micronutrient assays eg serum folate, red cell folate, serum vit b12, Schilling‘s test, lead assay, other heavy metal assays. Serum bilirubin
in hemolytic anaemia. Bone marrow assay in aplastic anaemia, bone marrow failure.
Direct and indirect (antiglobulin testing): Direct Coomb’s test in immune hemolysis cross-matching, hemolytic tranfusion reaction, HDN,
drug induced immune hemolytic anaemia, SLE. Indirect coombs test in Rhesus isoimmunisation.
ABO blood group typing

BMA which is the process by which blood or cells are removed from the marrow of the patient; used in investigating a child with abnormal
peripheral blood, diagnosis of malignant haematological disorders, anaemias, infilterative and storage diseases.
In leukemia, further tests like immunohistochemistry, monoclonal antibodies, immunophenotyping and cytogenetic tests are done to confirm
diagnosis. Microbiological cultures in PUO. RNA or DNA analysis for gene abnormalities. Follow up after chemotherapy or stem cell
transplantation.

Inflammation:
Erythrocyte sedimentation rate; the rate at which RBCs sediment in one hour. it is raised in diseases associated with changes in plasma
protein. It is a non specific test eg anaemia, tuberculosis, infections, collagen diseases kidney diseases. It is reduced in sickle cell CCF,
leukocytosis, low plasma protein, polycythaemia.

Other non specific markers of inflammation include C-reactive proteins, complement assays.

Tests for HIV: ELISA, enzyme linked immunoassay detects antibodies or combined antigen –antibody to HIV
Western blot technology, Antibody differentiation test and Nucleic acid tests : RNA or DNA –PCR.

Blood culture tests: this is done whenever a systemic infection is suspected. Isolating the offending organism in the patient blood is gold
standard in diagnosis of sepsis. Blood culture is also the investigation of choice in the diagnosis of infective endocarditis. It has use in all
the systemic infective illness. Viral culture can also be done. Viral infections are diagnosed by light and electron microscopy, serology,
ELISA, RIA.

Coagulation tests:
These are done in patients suspected to have bleeding or clotting disorders: the tests include platelet count, bleeding time, prothrombin
time, thrombin time, activated partial thromboplastin time.
Prothrombin time; extrinsic pathway and common pathway of coagulation Factors I, II V, VII, X Liver disease and Vit K deficiency,
warfarin therapy, DIC. prothrombin time greater than 35-43 seconds indicates the abnormality in the liver, vit K Deficiency.
INR –used to monitor patient on anticoagulation
Activated partial thromboplastin time: measures the intrinsic pathway deficiency in factors II, V, VIII, IX, X, XI & XII, fibrinogen.
Fibrinogen measurement
Assay of coagulation factors eg in haemophilia VIII, IX and XI, von Willebrand factor levels. D dimer assay in disseminated intravascular
coagulation

Chemistry:
104
Compiled by Obasi. D. C. Chinedu

Other blood based test include serum electrolytes, Urea and creatinine usually done to assess kidney functions, but used in monitoring of
therapy especially in chemotherapy in order to detect tumor lysis syndrome.
Blood sugar –hypoglycaemia and hyperglycaemia
Glycosylated hemogloblin –to measure patients average blood glucose in the preceding 3months
Arterial blood gas analysis: assessment of partial pressure of oxygen, carbon dioxide and acidosis.
Blood tests are not limited to the above mentioned tests but depends on the clinical condition.

Non invasive test: Non invasive tests in haematology but not limited to the system include:
Pulse oximetry: detection of oxygen saturation of the blood. It is also useful in diagnosing respiratory, cardiovascular and central nervous
system disorders.
Ultrasound: the principle of ultrasound is the ability of sound waves to be reflected off surfaces producing an echo it is one of the most
widely used diagnostic tool in Paediatrics practice. It is used in haematology in detection of solid organ abnormalities. Trans cranial Doppler
ultrasound is used in identifying sickle cell patients at risk of stroke.
In cardiology it is one of the most cost effective diagnostic tool of choice (echocardiography). Nephrology used in assessment of kidneys
ureters and bladder. Pulmonology for assessment of pulmonary edema and intrathoracic masses. In the Digestive system for detection of intra
abdominal masses, obstruction, gases, fecal impaction, abscess collections. In neurology, Trans cranial ultrasound is used in diagnosing
intraventricular haemorrhage, obstructive hydrocephaly, visualization of vessels in the brain.
Disadvantage include skilled personnel, poor bone penetrance, impedance by air and fat.

Computed tomography scan: uses xray beams to present cross-sectional image. It has found use in haematology in the diagnosis of
lymphoma and other malignancies. Gives better detail and visual data as compared to the ultrasound however it is more expensive, high
radiation exposure and it is not very good in identifying soft tissue pathology.
In Neurology, brain tumors, hydrocephalus, abscess, trauma, subdural hemorrhage. In cardiology it is used in the study of cardiac vessels,
complex cardiac anomalies in CT angiography. Pulmonology, it is used in study of intrathoracic masses, abnormal respiratory connection,
pulmonary malignancies.

Magnetic resonance imaging: MRI is used to investigate the anatomy and physiology of the body, it is a non invasive radiology technique.
A magnitude field is generated by highly excitable hydrogen ions and this produces the image. It is very sensitive in detecting tumors in the
brain, localizing tumor in all parts of the body. However, cost and unavailability are the limiting factors

Positron emission tomography: it is a newer technology and provides functional imaging and shows areas of dysfunctional metabolism. A
combination of PET–MRI is a hybrid imaging that combines the soft tissue imaging and functional imaging and labeling of cell types and
receptors of PET scan. This has found clinical use in oncology, cardiology (complex congenital heart diseases) and neurology.

Respiratory system: Respiratory tract spans from the nasopharynx to the lungs. it is a continuum. Infection in one segment is unlikely to be
limited by anatomic boundaries. Investigations in the respiratory system are usually done to confirm, support a diagnosis and to monitor the
treatment.

Non invasive: They include


Pulse oximetry: monitoring of oxygen saturation.
Capnography: measurement of cardondioxide in expired air, useful in apnoea monitoring, asthma, bronchopulmonary dysplasia, information
about ventilation-perfussion mismatch.
Spirometry measure lung function; asthma, chronic obstructive diseases, bronchitis, emphysema pulmonary fibrosis. it measures FVC and
FEV1.
Fraction of exhaled nitric oxide testing- non invasive marker of airway inflammation. Guides in adjustment of dose of corticosteroids.
Polysomnography sleep study used in diagnosing sleep disorders. information collected include brain waves, blood oxygen level eye and
leg movement, heart rhythm and respiratory efforts or events.
6minute walk test: (simple pulmonary stress test) measure the rate of breathlessness after 6 minute walk.
Cardiopulmonary exercise test measures breathing variations as well as blood and oxygen flow to the muscles during exercise.
Others are: Sweat test – for cystic fibrosis, Sputum culture, Sputum AFB for tuberculosis, Gene xpert test, Throat swab.

Imaging:
Xray: chest X ray provides images of the lungs, soft tissue used in the diagnosis of pneumonia, pleural effussion, emphysema. Other Xray
forms include paranasal sinuses, post nasal space.
CT scan provides crossectional view, often used to identify pulmonary embolism.
MRI: 3 dimensional images. Ultrasound: pulmonary edema, pneumothorax.

Invasive: Such as
Broncholavage: obtaining specimen from the bronchial tree for analysis. Lung biopsy not usually done.
Pleural tap for fluid assay culture and sensitivity, used in tuberculosis treatment, empyema thorasis.
ABG in patients with acidosis, used to determine Pa02, PaCo2, HCO3 PH, Oxygen content and saturation
Microbiological investigations such as throat swab, blood culture.
105
Compiled by Obasi. D. C. Chinedu

Fibre optic bronchoscopy: evaluating of airway, vocal cords trachea vocal cords.
Trans bronchial biopsy. Larygoscopy.

In the respiratory system infection, offending organisms can be identified using cultures, nasopharyngeal aspirates, nasal swabs, blood
cultures and serology. Fungal studies are used for respiratory fungal infections such aspergillosis.

Digestive system: The digestive system spans from the oral cavity to the anus and involves Oesophagus, GIT and liver, gall bladder. Like
every other system diagnosis involves proper history and physical examination. The investigations include invasive and non invasive
investigations.

Invasive investigations:
Blood test for markers of inflammation example ESR, FBC, CRP. Specific tests for disease entities such as Hepatitis B surface antigen,
Hepatitis C and hepatitis panel. Liver function tests. Serum biluribin in suspected jaundice. Widal test for enteric fever. H pylori screening
test.
Stool tests: gross, microscopic and chemical examination and culture. This helps in detection of appearance if formed or semi formed.
Colour and consistency (pale and bulky in cholestasis, black in Melaena). Mucus flakes or rice water in cholera. Content: pus, blood, worms
Microscopy: examine for parasite, protozoa cysts, egg and ova of helminthes.
Stool PH: in persistent diarrhoea and lactase deficiency. Stool reducing substances, fat content and stool occult blood in suspected cases of
PUD.
Urine test: Urobilinogen in hemolytic disease, and liver disease using dip stick. Bilirubin can also be detected in the urine in obstructive
jaundice.

Differentiation of liver disease using urobilinogen and bilirubin


normal Hemolytic disease Biliary obstruction Liver disease

urobilinogen normal increased Low or absent increased

Bilirubin negative negative positve Positve

Others are: Alpha feto protein assay in liver malignancy, Enzyme assays such serum amylase in pancreatic disease. Glucose tolerance
test. Fasting blood sugar. Gastric lavage in childhood tuberculosis. Abdominal paracentesis for ascitic fluid and peritoneal fluid analysis.
Rectal snip for helminthes. Liver biopsy for chronic liver disease, evaluation of abnormal liver result, liver mass amoebic abscess.
Malignancy and evaluation of trace elements.

Imaging
Imaging techniques is of high diagnostic value in digestive system pathologies. Non invasive includes
abdominal ultrasound –evaluates abdominal organs, cysts, malignancy, obstruction, fluid, inflammation.
X-ray; plain Xray to contrast studies, Plain abdominal X-ray in intestinal obstruction, peritonitis,
Contrast studies –Barium swallow, meal, follow through and Barium enema for GIT disorders
Invertogram for congenital anorectal disorders eg hirschprung disease.
CT Scan for cross sectional abdominal images in intraabdominal malignancy
Endoscopic procedures such as oesophagastroduodenoscopy for upper gastrointestinal disorders. Biopsy can also be obtained using
endoscopy.
Electrogastrogram – a non invasive test used to measure gastric myoelectrical activities.

Nutritional assessment: This helps in early detection of growth and development disorders, such malnutrition, failure to thrive
malabsorption disorders. Anthropometric measurements, weight, height, MAC, Head circumference BMI, derived variables. Measurement of
body mass composition (fat free mass), using bioelectrical impedance analysis, densitometry. Assay of micronutrients: vit A, D, Calcium,
zinc, phosphrous, total protein, cholestrol, iodine etc.

Central Nervous system: Like in other system appropriate use of investigative technique improves the diagnostic capabilities of the
clinician in management of neurological disorders. The investigations can be invasive and non invasive

Invasive procedure:
Lumbar puncture: for Cerebrospinal fluid analysis. Indications are both diagnostic and therapeutic. Diagnostic indications are; Diagnosis
of meningitis, Encephalitis tuberculous meningitis, Subarachnoid hemorrhage, Gullian Barre syndrome and CNS malignancies.
The therapeutic indication include; Anaesthesia, Chemotherapy.
In performing a lumbar puncture, an informed consent must be obtained including the benefits of the procedure. Asepsis must be
106
Compiled by Obasi. D. C. Chinedu

maintained. Appropriate needle inserted into a space between L3 and L4 or L5 and S1. Once in the space there is usually a give and stylet
is withdrawn. And CSF collected for both chemistry and microbiological analysis. The normal CSF has a pressure of 90-160 mmH20. It
is clear and colourless. Slightly xanthochromic in neonates, no redcell,. no neutrophils, white cell in the neonates 7-8/mm3 monocytes. A
Normal CSF contains about 5 cell/mm3. CSF of neonates may contain as many as 15cell/mm3. Normal CSF Protein 10-40mg/dl in
children but may be higher in 120-200mg/dl in neonates gets to normal by 3months. CSF sugar is about 50-60% of blood sugar. In
disease state like meningitis CSF colour is cloudy, turbid or pus like, elevated neutrophils are seen in pyogenic meningitis. Bloody CSF in
traumatic tap and subarachnoid hemorrhage. Xanthochromic in high protein yield, jaundice, (neonates)supernatant in subarachnoid
hemorrhage. Spiderweb formation on standing and high protein count –Tb meningitis. High lymphocytes count in viral meningitis.

Contraindications: Should not be attempted in cases of; Deep coma, Signs of raised intracranial pressure, Cardiopulmonary instability,
Coagulopathy, Suspected cerebral herniation.
Others are: Local infection at the site, focal neurological seizures, Purpura in a febrile suspected to have meningococcal infection.

Complications: They are; post Dural headache, Local pain, Infection spinal hematoma, Arachnoiditis, Apnea , Respiratory arrest, Tonsillar
herniation and ocular palsy.

Subdural puncture: subdural tap is a procedure performed to remove fluid or blood from the subdural space. Indications acute or chronic
blood or fluid accumulation (subdural effusion or haematoma). Samples are analyzed for chemistry and microbiology.

Imaging:
Trans fontanels ultrasound: in suspected cases of intraventricular haemorrhage, aneurysm hydrocephalus. Subdural effusion.
Skull X-ray: certain pathological conditions can be revealed by skull X-ray eg fractures, calcifications, indentations in raised intracranial
pressure, widening of sella trucica and erosion of cliniod process in pittuitary fossa tumors, sutural diastesis in Intracranial hypertension,
hydrocephalus, erosion of the calvarium in dermoid cysts and early fusion in craniosynostoses.
Spinal X-ray in Potts diseases, kyphosis, spinal bifida. Etc
Ventriculography- used in diagnosis of hydrocephalus and certain tumors.
Myeography: mass lesion encroaching on compressing the spine, intramedullary tumors.
Pneumoencephalography: this is done when delineation of the ventricles and cisterns is required.
Cerebral angiography: used in studying cerebral blood vessels, vascular neoplasm aneurysm and malformations.
CT scan: this very valuable in the diagnosis of intracranial disorders, tumors, abscess malformations, trauma, hydrocephalus, infarcts
calcifications.
MRI: Just like CT, a superior investigation, indications are similar to CT but does not resolve bone lesions but better in white matter lesions
(leukoencephalopathies). It does not have ionizing radiation.
Electroencephalography: This technique of recording electrical activity of the cerebral neurons; Used in management of epilepsy, cerebral
abscess, SSPE (Sub acute sclerosing panencephalities). Specific pattern are noted in certain encephalopathies, Eg hypsarrhythmia in infantile
spasms, Burst suppression pattern in SSPE, Paroxysmal episodes of generalized bilateral and synchronous 3per second spikes wave.
Generalised spikes, polyspikes wave discharges or spikes slow wave complexes of irregular frequency separated by normal background.
Electromyography: used to assess muscle function in patients suspected to have muscle diseases eg muscular dystrophies, spinal muscle
atrophy.
Electroretinogram: to assess visual function and more specifically retinal function eg in retinitis pigmentosa, congenital retinal dysfunction,
congenital optic atrophy, albinism.

Other test done in diagnosing nervous system disorders include; nerve conduction velocity; rate of conduction of impulses along a nerve-
functional integrity of the nerve. Nerve conduction velocities are reduced in neuropathies.
Measurement of intracranial pressure: in trauma, hydrocephalus, encephalopathies and comatose patients using a Rickham reservoir
connected to the lateral ventricle and connected to pressure transducer via a butterfly needle.
Assessment of auditory pathway and visual pathway using brainstem auditory evoked responses and visual evoked responses
respectively.
Blood tests: which include biochemical studies, calcium assay, serum electrolytes, blood glucose, serum creatinine kinase, and other
enzymes in suspected in born errors of metabolism.
Nerve biopsy and muscle biopsy for protein assay and histologic studies.

Nephrology: The investigation necessary for diagnosis of renal disorders are many but judicious use either singly or in combination help in
diagnosis. The investigation for evaluation of renal function can be classified in
1. Urine examination: which include urinalysis, macroscopy, microscopy, dipstick, culture and sensitivity used in diagnosis of UTI,
PYELONEPHRITIS, Abnormal glomerular function, haematuria, proteinuria, glucosuria.

2. Blood tests: to detect presence of renal dysfunction, blood chemistry, SEUC, lipids, Vit D, Ca, phosphorous, parathormone alkaline
phosphatase.
FBC: anaemia of CKD
Arterial blood gases in Renal tubular acidosis
Immunological tests in ASO TITRE and complement factors anti DNAase, AGN, anti nuclear antibodies in SLE, anti neutrophil
cytoplasmic antibody in Wegener granulomatosis. Hormone profile in Paediatrics hypertension.
107
Compiled by Obasi. D. C. Chinedu

3. Estimation of GFR; Glomerular filtration rate, it is used to detect and classify the severity of renal diseases different substances can be
used to estimate GFR. Creatinine and more recently cystatin C are used. Creatinine clearance is used and this is usually obtained by
obtaining mid serum creatinine in a timed urine collection for creatinine clearance.

Schwartz formula is used: GFR (ml/min/1.73 m2) = (0.413x height(cm))/serum creatinine (mg/dl)
Normal for newborn -20ml/min/1.73m2. 1 year =100ml/min/1.73m2. Adult male 120 and female 100ml/min/1.73m2

Assessment of tubular function: Tubular function is involved in reabsorption and excretion of fluid and electrolytes and acidification of
urine. Investigation include; Fractional excretion of sodium (normal value is equal or less than 1%. In diseased state it is more than 1% in
impaired tubular transport, value of 2% in AKI due to acute tubular necrosis). Other tests include water deprivation test in diagnosis of
Diabetes insipidus.
Diluting ability test in adrenal insufficiency, hypothyroidism, potassium depletion.
Urine acidification tests using ammonium chloride challenge test.

Imaging: Imaging modalities help in diagnosis of renal functional and structural abnormalities. It includes
Plain X-ray: of the abdomen –(KUB) nephrolithiasis, sacral agenesis as a preliminary film in IVU.
Intravenous urography (IVU): use is on the decline. Delineates the anatomy of the kidney, duplex kidney, renal scars, site of pelvi-ureteric
junction obstruction. Contraindicated in neonates, young infants and renal failure at any age.
Voiding cysto urethrography: Method of choice in grading reflux and posterior urtheral valve and in cases of repeated UTI, ureteric
dilatation, voiding difficulties and thick walled bladder.
Antegrade pyelography and retrograde pyelography; determination of level of obstruction in the collecting system.
Renal arteiography: this highly invasive procedure gives direct visualization of the renal arterial system-reno vascular hypertension,
preliminary work up in transplantation.
Abdominal ultrasound; very common investigation in assessment of kidneys ureter and bladder, it is used in evaluation of obstruction,
nephrolithiasis, renal parenchyma diseases. Renal size in CKD polycystic kidney disease, reflux hydroureters, renal masses, malignancy,
congenital abnormalities of the urinary tract. Limited in focal renal scarring or renal arterial abnormality.
Radioistope used in the urinary tract are dynamic renography, static renal scan and cystography. TC DTPA (Technicium labelled diethylene
triamine penta-acetic acid) and TC MAG 3. Useful in investigation of obstruction of upper urinary tract, calyces, renal pelvis and renal blood
flow symmetry of excretory functions. Evaluation of post transplant state.
Static renal scan – TC dimercaptosuccinic acid. Commonest use. Reveals renal parenchyma and demonstrate areas of filling defect.
Sensitive in detecting renal cortical scars, Size and shape of the kidneys, Parenchymal abnormalities, ectopic kidneys, Renovascular diseases
and dribbling urinary incontinence
Cystography; this is for bladder function, rate of emptying. Residual urine and MAG3 iv or 99 TCc pertechnetate directly into the bladder
are used.
CT scan: indication as are similar to ultrasound but gives clearer resolution of anatomy of kidneys and surrounding structures eg tumors, fine
calcifications in the kidneys.
MRI- provides high quality anatomic details and evaluation of complex urinary system disorders

Renal biopsy –obtaining tissues from the kidney. Open biopsy or percutaneous. Commonly percutaneous biopsy, using real time ultrasound
guided or CT guided.
Indications; Nephrotic syndrome; steroid resistance, atypical presentation, family history suggestive of hereditary disease. Acute renal
failure with rapid deterioration, poor response. Acute nephritic syndrome SLE with renal involvement. CKD in absence of shrunken kidneys.
Transplantation .to determine indications for allograft rejection
Contraindications: Single kidney, Shrunken kidneys, large or multicystic kidney, Ectopic kidney, horseshoe kidneys. Uncontrolled bleeding
diastasis, Severe hypertension, Severe anaemia, Severe ureamia.

Complications: Bleeding, hematuria, Transient hypertension, Infection, pain, Arteriovenous fistula, complications of analgesia, Bladder
function.

Urodynamic studies are investigation used to evaluate anatomy and physiology of the lower urinary tract; neurogenic bladder, urinary
incontinence, voiding dysfunction, VUR, anorectal malformations. Quality control after surgery to relieve obstruction.

Uroflowmetry: urine speed and volume


Post void residual urine estimation: use of ultrasound to estimate post void volume in the bladder. Values more than 100ml is
significant.
Invasive urodynamic studies
Cystometry studies cystometry measures the relationship between intra vesical pressure changes during micturition and detrusor pressure
needed to void to force the urethra open.
Pressure flow study: the bladder pressure required to void and the flow rate a given pressure generates it helps detect bladder outlet
blockage.
Cystoscopy: direct visualization of the lower urinary tract. Eg non glomerular haematuria, congenital abnormalities, recurrent UTI.
Electromyography: used to measure the electrical activity of the pelvic musculature
108
Compiled by Obasi. D. C. Chinedu

ANAEMIA
By Dr Ezenwosu, O.U.

Definition
Anaemia refers to an abnormal reduction/decrease in the number of circulating red cells, circulating haemoglobin (Hb) concentration
and in the haematocrit. It is not a disease itself but a manifestation of other conditions. With initial reduction, few clinical disturbances
occur till the Hb level falls below 7-8g/dl.

Age-related normal values of Hb:


Age Mean (g/100mls) Range

Birth 17 14-20
1-2 weeks 16 13-20
3-11 months 12 10-14
1-5 years 12 11-14

Neonates: Peculiar. Born with relatively high Hb concentration. During first 6-8wks after birth, the Hb conc falls gradually to reach a
nadir of abt 9-10g/dl (Physiologic anaemia of infancy). Thereafter, it begins to rise. Hardly requires treatment.
Suggested reason: Sudden drop of erythropoietin following onset of respiration soon after birth or other reasons like: Shortened red cell
survival time at this age, Def of folic acid, Fe and Vit E or Rapid expansion of the bld vol accompanying rapid growth in early infancy.

Anaemia of Prematurity: An exaggeration of Physiologic anaemia of infancy. The decline in Hb level is more rapid and more severe in the
premature (esp VLBW & ELBW) because Postnatal suppression of erythropoiesis is rapidly accentuated by greater expansion of blood vol
due to far more rapid growth, less iron stores and more deficient folic acid & vit E.
Preventive measures include: Nutritional supplementation (Folic acid, Vit E, Fe). If Hb <9-10g/dl, transfuse ±symptoms. If symptomatic,
transfuse earlier. Recombinant EPO- a suitable alternative

Aetiological Conditions/Disorders in Anaemia:


a) Neonates: Antepartum haemorrhages, Haemorrhagic disease of newborn, Maternal-fetal blood grp incompatibility, IU infections,
Birth trauma or Malaria.
b) Infants: Dietary Fe def, Malaria, Infections, Haemoglobinopathy, Lead Poisoning, Malignancy.
c) Toddlers: Malaria, Malnutrition, Helminthiasis, Infections, Haemoglobinopathy, Dietary Fe def, Malignancy.
d) School Age: Malaria, Fe def, Hookworm infestation, GI bleeding, Malabsorption, Malignancy, Chronic diseases, Infections (acute
and chronic).

Degrees of Anaemia: Mild anaemia (Hb 9 - 11g/dl), Moderate anaemia (Hb 5 - <9g/dl), Severe anaemia: (<5g/dl)

Classification of Anaemia: Clinically, classified according to the morphology (appearance) of rbc on peripheral smear and mean
corpuscular volume (MCV)
1.Hypochromic microcytic – Small pale rbc & low mcv
2.Macrocytic – Large rbc and high mcv
3.Normocytic normochromic – Normal sized rbc and normal mcv

1. Hypochromic microcytic anaemia: can arise as a result of; Impaired synthesis of Heme (due to Fe deficiency, Cu deficiency, Lead
poisoning, Chronic diseases like Inf, Ca,Inflamm, Renal diz.) or Impaired synthesis of globin (as seen in Thalassaemia syndrome).

2. Macrocytic anaemia (megaloblastic anaemia): can be seen in; Deficiency of Vitamin B12 (Cobalamin) or Deficiency of folic acid.

3. Normocytic normochromic anaemia: seen in; Impaired survival (Haemolysis) (which includes Intracorpuscular abnormalities like
Enzyme defects: G6PD def, PK def, Membrane defects: Hereditary spherocytosis, Elliptocytosis, Poikilocytosis, Hb abnormality:
Haemoglobinopathies and Extracorpuscular abnormalities like Autoimmunity: warm AIHA, cold AIHA, PCH, Microangiopathy: DIC,
HUS, severe HTN, Toxic agents, Infections).
Other causes are; Impaired production (Aplasia, BM Replacement), Blood loss (Haemorrhage) or Sequestration.

IRON DEFICIENCY ANAEMIA: Anaemia results from insufficient Fe for synth of Hb. The most common haematologic disease in the
world. Fe-The most common micronutrient def in the world. The commonest cause of anaemia in children. Can arise due to:
 Reduced intake (commonest)
 Bleeding from hookworm infestation, menstruation, trauma, surgery, GIT (PUD, IBD)
 Perinatally- fetomaternal, twin-twin transfusion, APH

Pathogenesis: Newborn has abt 0.5g Fe while adult is 5g. To make up, an average of 0.8mg Fe must be absorbed daily in the first
15years of life. In addition, small amt of Fe is lost by cell shedding. Since abt 10% of dietary Fe is absorbed, 8-10mg of dietary Fe is

109
Compiled by Obasi. D. C. Chinedu

required for optimal nutrition. Nutritional Fe def occurs when rapid growth of child puts excessive demand on Fe store. Seen in infancy and
adolescence.

Clinical Features: Common b/w 6-24 months. In mild cases, the patient are usually asymptomatic. In severe cases, they present with
irritability, anorexia, lethargy, easy fatiguability. Can complicate features of heart failure. There is an associated hx of pica, geophagia,
pagophagia.
On exam: Pale, tachycardia, haemic murmur, splenomegaly. Signs of heart failure. Ridged nail, koilonychia

Laboratory findings
 Hb – mild to severe anaemia
 Peripheral smear: small, pale rbc
 MCV, MCH, MCHC – Reduced proportionate to degree
 Reticulocyte count: Normal or slightly elevated
 Fe studies – Low serum Fe, ↓serum ferritin, ↓transferrin saturation, elevated TIBC
 Bone marrow exam – Not routine. Micronormoblastic hyperplasia. Reduced or absence of stainable Fe

Treatment
Mild to moderate without decompensation: Oral elemental Fe 4-6mg/kg/day in 3divided doses. Best absorbed in b/w meals.
Simultaneous ascorbic acid aids Fe absorption. If Fe def is the only cause of anaemia, resolution within 4-6weeks. Treatment should be
continued for additional 2-3months to replenish Fe store.
No room for blood transfusion except there is anaemic heart failure. (Packed cells 5-10ml/kg slowly)

ANAEMIA OF CHRONIC DISORDERS:


Anaemia of Chronic Inflammation: Chronic conditions with inflammation e.g. JRA, SLE, osteomyelitis, ulcerative colitis may show
anaemia usually mild to moderate in severity with hypochromic microcytosis and low reticulocyte count.
Pathogenesis: inflammatory cytokines inhibit erythropoiesis & also impair Fe release by reticuloendothelial cells
Investigations: Low serum Fe, ↑serum ferritin, TIBC not elevated
Treatment: with treatment of underlying disease, anaemia resolves. Fe is unnecessary.

Anaemia of Chronic Renal Failure


Pathogenesis: deficiency of erythropoietin production by the kidney; bleeding & haemolysis from uremia.
Investigation: Bone marrow shows hyperplasia of the erythroid series while WBC and platelet productions are normal
Treatment: Recombinant erythropoietin

FOLIC ACID DEFICIENCY: This may be caused by: Inadequate dietary intake, Impaired absorption (malabsorption states),
Abnormal/impaired metabolism (anticonvulsant, antimetabolites), ↑Folate demand (malignancy, haemolytic anaemia, pregnancy).
Inadequate dietary intake is common in the tropics. Cooking destroys folate in green vegetables.

Clinical features: Palor, mild jaundice, progressive anaemia. Heart failure in severe anaemia, LBW, pale with FTT- suspect.
Investigations:
↓Red cell folate level – better reflection of folate store
Homocysteine- may be elevated
Hb – Anaemia, ↑MCV, ↑MCH
Peripheral film – macrocytes with anisocytes & poikilocytes; Large hypersegmented neutrophils

Treatment: 5-10mg folic acid daily for at least 4weeks. Folic acid prophylaxis in pts on drugs with antifolate property. Also treat underlying
cause/infection

VITAMIN B12 DEFICIENCY: Rare- vit B12 found in many foods. Mostly seen in strict vegetarians. Combines with intrinsic factor to be
absorbed in terminal ileum. Its deficiency could be caused by:
-lack of secretion of intrinsic factor -atrophy of gastric mucosa -achlorrhydria
-surgical resection of terminal ileum -bacterial overgrowth of small intestine
-D. latum infection -crohn’s diz, regional ileitis
Clinical features: includes Palor,
Severity – irritability, anorexia, listlessness
GIT – smooth tongue, red, painful, - diarrhea, weight loss
CNS – paraesthesia, weakness, ataxia, hyporeflexia, coma
Investigations: Hb – Anaemia, Peripheral film – macrocytes, Serum Vit B12 – Low. Schilling test – demonstrates mechanism of absorption
using radioactive Vit B12
Treatment:
Most - im inj of loading dose of 1000mg Vit B12 followed by monthly maintenance im 100mg for life

110
Compiled by Obasi. D. C. Chinedu

In neurologic involvement – 1000mg dly x 2wks b/4 maintenance

GLUCOSE 6 PHOSPHATE DEHYDROGENASE (G6PD) DEFICIENCY: RBC depends on anaerobic glycolysis for maintenance of
ATP for hemostasis (80-95% glucose). 5-10% follow HMP/PP shunt. Generates sufficient reduced glutathione to protect rbc against
oxidative damage. G6PD enzyme is required for HMP shunt. Its def causes oxidant compds formed during rbc metabolism to
accumulate and denature Hb. Affected rbc are haemolysed and removed by the spleen →Anaemia
G6PD def is the most important red cell enzymopathy worldwide and affect abt 200million people worldwide. X-linked recessive
abnormality. In Africa, affects abt 25% males and 2-3% females.
The normal G6PD worldwide is G6PD B+ and in Africa G6PD A+. Mutant type in Africa is G6PD A- (<5-15% activity). Mutant type
elsewhere G6PD B- (<1% normal activity). G6PD def in people from African ancestry is less severe.

Substances that cause oxidative damages: Naphthalene balls, Sulphones (Dapsone), Sulphosalazine
Antimalarials – chloroquine, primaquine, quinine, quinidine
Antimicrobials – sulphonamides, nalidixic acid, nitrofurantoins, chloramphenicol, niridazole
Other drugs – Aspirin, vit k analogues, dimercaprol, phenacetin, doxorubicin, methylene blue
Others – fava beans, tinitrotoluene (TNT), benzene
Illness – diabetic acidosis, hepatitis

Clinical features:
Neonatal jaundice – impt cause. May require phototherapy or EBT. If severe, may cause kernicterus
Drug-induced haemolysis – palor, jaundice, haemoglobinuria. severe anaemia → failure
*Not all oxidant drugs cause haemolysis in Africa esp when taken in recommended doses e.g. Chloroquine, chloramphenicol, quinine,
quinidine, vit k analogue

Investigations
In b/w haemolysis (in absence of oxidants): normal Hb, reticulocytes and periph bld
In haemolysis: Hb - ↓, Reticulocyte count - ↑, Film – spherocytes, G6PD status – reduced activity in rbc – confirmation. (done when retic
is normal)
Treatment: Avoidance of drugs associated with haemolysis. Self limiting but in CVS compromise – transfuse red cells. Neonates –
phototherapy or EBT depending on level of unconjugated bilirubin.

HEREDITARY SPHEROCYTOSIS: Most common familial and congenital abnormality of the red cell membrane resulting in haemolysis
and haemolytic anaemia with splenomegaly and small spherical shaped cells. 80% of cases inherited as autosomal dominant. Remainder
autosomal recessive or by new mutation (sporadic). Rare in Africa

Pathogenesis: Partial deficiency of spectrin (structural protein of the red cell membrane skeleton). This weakens the attachment of the cell
membrane to the underlying membrane skeleton and causes loss of membr surface area.
Creates spherocytes that are poorly deformable and are trapped in the microvasculature of the spleen and engulfed by splenic macrophages
→ haemolysed

Clinical features: Significant Neonatal jaundice in 50% affected children requiring EBT.
Infant & children – palor, jaundice (variable), weakness, malaise, splenomegaly. Severe cases → cardiac failure

Investigation
Hb – mild anaemia (most have mild chronic haemolysis)
Reticulocyte count – Reticulocytosis
Periph film – micro spherocytes
Serum bilirubin - ↑unconj fraction
Coomb’s test – negative
Diagnosis – confirmed by ↑osmotic fragility test
Treatment: Acute exacerbation of anaemia may require bld transfusion
Splenectomy: significant improvement in many cases. Prior pneumococcal vaccine & penicillin prophylaxis afterwards

AUTOIMMUNE HAEMOLYTIC ANAEMIA: May be:


a) Primary (Idiopathic)
b) Secondary to
-Infection (URTI, CMV, Infectious mononucleosis, Hepatitis, EBV)
-Immunologic disease (SLE, Rheumatoid arthritis)
-Immunodeficiency states (Agammaglobulinaemia)
-Malignancy (Lymphoproliferative disorders)
-Drugs (Methyldopa, Levodopa)

Classified into: Warm AIHA, Cold AIHA, PCH


111
Compiled by Obasi. D. C. Chinedu

Warm AIHA: RBC antibodies which are active b/w 35-40oC. Often belong to IgG class. Does not require complement for activity.
Antibodies from serum and from RBC react with the RBC to cause haemolysis.

Cold AIHA: RBC antibodies that are more active at low temperatures and agglutinate RBCs at temp <37oC is called “cold” antibodies.
Primarily of IgM class. Require complement for activity

Clinical Features: Acute onset- Palor, Weakness, Dark urine, Jaundice, Splenomegaly.
Features of underlying condition

Investigations: Hb – Reduced, Periph film – spherocytes, nucleated red cells, Reticulocytes – Raised. Serum Bilirubin - ↑ total & indirect,
Urine – Haemoglobinuria (indicates intravascular hemolysis)
Coomb’s test – Direct & indirect +ve:
Presence of IgG in rbc, max activity at 37oC with extravascular destruction by RES – Warm AIHA
Presence of complement on rbc, max activity at 4oC with intravascular haemolysis – Cold AIHA
Treatment:
Warm- Prednisolone 2mg/kg/day tapered slowly after initial response. May respond to IVIG 1mg/kg/day x2/7
Cold- Not responsive to steroid or IVIG. Supportive care. Plasma exchange in severe AIHA.
PCH- Remove from cold to warm environment

112
Compiled by Obasi. D. C. Chinedu

HAEMOGLOBINOPATHIES
DR. O.U. EZENWOSU

Haemoglobinopathies are structural abnormalities of haemoglobin (Hb). Structurally, Hb is a tetrameric protein with 4 subunits each with
haeme grp and globin chain. The haeme grp is identical in all the 4 while the globin consists of a pair of α chains (or α-like e.g. ζ) and a pair
of β chains (or β-like e.g. γ,δ, and ε).
In Foetal life: α2γ2 (HbF). Postnatal : α2 β2 (HbA) and α2 δ2 (HbA 2). α chain have 141 amino acids controlled by genes encoded on
chromosome 16 while β chain have 146 amino acids controlled by genes on chromosome 11.

Structural abnormalities of haemoglobin result from changes in the amino acid sequences of the globin chains.
This change may be due to:
-single base substitution (point mutation)
-substitution of 2 amino acids
-deletion or insertion
-fusion of haemoglobins
-frame shifts
More than 700 structural variants of Hb have been identified. Many, which show electrophoretic changes, are readily detected. Those with
similar electrophoretic pattern need further means of specific identification.

Single base mutation: HbS: At the 6th position of the β-globin chain, the normal codon GAG (in HbA) is replaced by GUG leading to
substitution of glutamic acid by valine.
Abnormal Hb β-globin position Normal amino acid Abnormal amino acid

HbS 6th Glutamic acid Valine

HbC 6th Glutamic acid Lysine

Hb D 121st Glutamic acid Glutamine

Hb O Arab 121st Glutamic acid Lysine

The HbS gene is found in high frequency in those living in areas where P. Falciparum malaria is endemic

SICKLE CELL DISEASE:


A variety of pathological conditions resulting from the inheritance of sickle haemoglobin either in a homozygous state (SS) or in a
heterozygous state with another abnormal Hb such as SC, Sβthal, SD, SO Arab. The most common and most severe is SS known as Sickle
Cell Anaemia (SCA) from inheritance of HbS from each parent.
AS is not part of SCD.

SICKLE CELL ANAEMIA (SCA)


Epidemiology: SCA is commonly found among people of African ancestral origin. Also found in India, Saudi Arabia, the Mediterranean
countries, Southern Europe (Italy, Turkey, Greece). Prevalence of the trait in W/Africa is 10-40%, North Africa 1-2% and <1% in Southern
Africa. Prevalence of the trait in Nigeria is about 25%. The prevalence of SCA in Nigeria is 2-3%.

Pathogenesis of SCA: The substitution of glutamic acid (a water soluble amino acid) with valine (a fat soluble amino acid) causes a change
in the surface charge of the Hb molecule. In oxygenated state, HbS functions normally.
With reduced O2 tension, an interaction b/w complementary valines on β chains of adjacent Hb molecules results in formation of highly
ordered polymers. These polymers elongate forming filamentous structures, which aggregate into rigid, crystal-like rods (tactoids).
(POLYMERIZATION). These crystals form a viscous gel which deforms the red cells into rigid sickle cells.
Diameter of RBC = 7μ; Diameter of capillaries = 3μ. Unsickling ↔ Sickling → ISC (Irreversibly Sickled)

The rigidity of sickled RBCs increases their mechanical fragility and reduces their life span (abt 20 days) as they are destroyed in the RES.
=>(Recurrent Haemolysis).
Recurrent inflammation and vasculopathy in SCA activate (via cytokines) leukocytes and endothelial cells.
The activation also leads to upregulation of selectin receptors which mediate adhesion of leukocytes to the endothelium and to each other. →
bonding resulting to vaso-occlusion → thrombosis, ischaemia and infarction.

Inheritance: Simple mendelian fashion. Autosomal recessive pattern. The simple mendelian inheritance is true for each pregnancy,
irrespective of previous ones.

Factors enhancing sickling


1.↓ O2 Tension 2.↑ WBC 3. Presence of bacteria 4. Fever 5. Dehydration 6. Metabolic acidosis

113
Compiled by Obasi. D. C. Chinedu

Clinical features of SCA:


< 6 months Delay manifestation due to HbF protection
6months – 2yrs: Pallor/anaemia, jaundice, Hepatosplenomegaly
*Dactylitis – painful swelling of the digits
*hand-foot syndrome – bilateral, painful, warm, non-pitting swelling of dorsa of hands and feet.
*Due to ischaemic necrosis of the small bones of the extremeties. The earliest specific manifestation
> 2yrs: Anaemia/jaundice, Hepatomegaly ±splenomegaly (above 7years), infections, Sickle cell habitus: bossing, gnathopathy, athenia.

CRISES IN SCA: Recurrent, intermittent, episodic events of acute illness.


-Vaso-occlusive crises - commonest
-Anaemic crises – ‘aplastic’, ‘sequestration’, ‘hyperhaemolytic’ crises.

Vaso-occlusive crises: Also called painful or thrombotic crises. Due to occlusion of blood vessels by sickled cells leading to pain from
ischaemic injury. May affect any part of the body but particularly common in:
Long bones of limbs & back – Bone pain crisis
Abdomen – Abdominal voc
chest (pulmonary) – Acute chest syndrome
Penile shaft – Priapism
Brain – CVA
Precipitating factors: physical exertion, extremes of weather, fever, infection, dehydration, emotional disturbances

Bone pain crisis:


In infants- manifest as dactylitis, hand-foot syndrome. In young children- commonly extremities. In older children- commonly the back. Pain
may be symmetrical, asymmetrical.
Diff Diagnosis: Acute osteomyelitis, septic arthritis

Abdominal VOC: Occlusion of mesenteric vessels


Diff Diagnosis: Acute surgical, medical or gynaecological conditions.

Acute chest syndrome: Presence of new pulmonary infiltrates with chest pain, a temperature of >38.5oC, tachypnea, dyspnea, wheezing or
cough in a pt with SCA.
Causes- infection (pneumonia), fat emboli, pulmonary infarction.

Priapism: Sustained painful penile erection due to obstruction of venous outflow from the corpora cavernosa. May be spontaneous or
following sexual intercourse or masturbation. Risk of partial or complete impotence.

CVA: Usually results from partial or complete occlusion of major cerebral vessels especially one or both internal carotid arteries, anterior or
middle cerebral arteries. May present with convulsions, drowsiness, coma, brain stem dysfunction. May progress to hemiplegia (from
cerebral infarction). Hemiplegia may resolve within few weeks but usually slow and partial. Stroke is usually recurrent, usually within
3years.

Treatment of VOC:
 Treat underlying cause esp infection
 Hydration- liberal oral fluid intake in mild; parenteral infusion (2-2.5L/m2/24hrs)
 Analgesics- Non-narcotics (ASA, pcm) in mild; NSAIDs in moderate; Narcotics in severe pains.
o Other modalities: controlled analgesia, epidural infusion
 Others: Na2CO3 if acidosis; O2 if hypoxaemia
Priapism- additionally: Sedation to reduce apprehension; ICI adrenergic agonists like etilefrine;
Surgery-direct aspiration of corpora and cavernous-spongiosum shunts
EBT or TEA (therapeutic erythrocytapharesis)
CVA- additionally: Red cell exchange using EBT or TEA
Hypertransfusion programme / Hydroxyurea (due to recurrence)
ACS- additionally: EBT or TEA

Anaemic crises
Aplastic crisis: Temporary/transient bone marrow failure xtized by sudden reduction of ≥30% in steady state Hb level with significant
reticulocytopaenia. Usually follow vague URTI. Parvovirus B-19 has been implicated, which infects maturing erythroblasts, causing
cessation of production of matured red cells for 1-2wks.
Presents with weakness and increasing pallor with recovery in 2-3wks. In severe cases, steroids and blood transfusion may be indicated.

Hyperhaemolytic crisis: Exacerbation of chronic haemolysis of SCA following infections – bacterial (UTI, pneumonia, acute
osteomyelitis), parasitic (malaria).
Presents with severe pallor, jaundice, hepatosplenomegaly. There is ↓Hb/pcv with reticulocytosis
114
Compiled by Obasi. D. C. Chinedu

Treat the underlying cause (esp if presented early) to restore Hb to steady state. Severe cases may need blood transfusion.

Sequestration crisis: Xtized by sudden enlarging spleen, rapidly decreasing Hb, an elevated reticulocyte count and in severe cases signs of
circulatory collapse. Due to sequestration/ trapping of a significant proportion of red cells within the spleen. Cause is unknown and tend
to recur.
Treatment of severe case is immediate bld transfusion. A significant volume will re-enter the circulation (others are lysed). So packed cell
transfusion is needed. Prevent recurrence by regular transfusion or splenectomy.

Complications of SCA: Due to combined effects of chronic hypoxia, recurrent infections and infarctions (resulting from chronic, severe
haemolytic anaemia and vaso-occlusion), pathologic changes occur in almost all body organs

Musculoskeletal system complications:


Osteomyelitis: Staph (commonest) and Salmonella infx may complicate bone infarction. Usually bilateral and symmetrical. Clinically point
tenderness. X-ray at 10-14 days. Rx- antibiotics 4-6 wks (at least 2wks iv)
Avascular necrosis of femoral head: mainly in adolescents and older children.
Pathological fracture: from severe bone infarction and destruction.
Digital clubbing
Chronic leg ulcer: common in adolescents and affects the malleolus. Ulcers are punched out with raised edge.

Hepatobiliary system complications:


Hepatomegaly: seen in 40-80% in Nigeria
Sickle cell hepatopathy (Hepatic crisis): Occurs with tender hepatomegaly, deep jaundice and deranged liver enzymes. It is transient, with
unknown cause but prognosis is good.
Hepatic coma: Rare. Unexplained fulminant hepatic failure.
Haemosiderosis: Transfusion associated
Cholelithiasis: Uncommon in Africa. Due to low consumption of refined, fibre-depleted foods and cholesterol.

Genito-urinary system complications:


Polyuria, Nocturia and enuresis: Due to impaired urinary concentrating ability. Begins at 4-5yrs
Haematuria: Due to papillary necrosis. Usually from left kidney.
Proteinuria: early manifestation of SC nephropathy.
Bacteriuria: found in 22% of febrile children with SCA
Pyelonephritis: Dilute urine favors bacterial growth.
Renal tubular acidosis: acidification defect

Cardiovascular system complications:


Cardiomegaly: functional adaptation to chronic anaemia results in hypertrophy and dilatation.
Myocardial infarction: Rare due to short transit time.
Congestive cardiac failure: from severe anaemic crisis.
Cor pulmonale: 2o to pulmonary thrombotic disease and pulmonary hypertension.

Respiratory system complication: Pulmonary hypertension, Fat embolism


Hypoxaemia: hallmark of pulmonary function abnormality in SCA
Reduced lung volume: Frequent pneumonia and reduced thoracic size

CNS complications: CVA, Febrile seizures, Sensorineural hearing loss


Ocular lesions:
Anterior chamber- corneal oedema, keratic precipitates, necrotic lens, low IOP, dilated pupils.
Posterior chamber- dilated tortuous retinal vessels, vitreous haemorrhage, retinal detachment, visual impairment.

Susceptibility to infections:
SCA are more susceptible to sepsis and other bacterial infections esp by Pneumococci, H. influenza and Salmonella.
Due to: Altered splenic function from progressive infarction and fibrosis, Functional asplenia. Defective alternate pathway of complement
activation, Elevated serum iron, Deranged pulmonary alveolar macrophage function due to chronic hypoxia.

Impaired physical & sexual development:


Lower weight and height (Emodi & Kaine). Some studies documented catch-up during adolescence. Lower BMI. Delayed sexual
maturation: Mean menarcheal age 16.1yrs (13.4yrs in AA), Low mean testicular volume. Delayed appearance of pubic hairs.
All attributed to abnormal endocrine function, chronic anaemia and subnormal nutritional factors

Impaired psychosocial function: Psychosocial disturbances may come from:


- Physical, skeletal and sexual maturation problems - Parents and sibling’s psychological problems
- Societal attitude/discrimination towards them - Frequent ill health and hospitalization
115
Compiled by Obasi. D. C. Chinedu

- School absenteeism - Academic performance - Role limitations


- Loss of job - Persistent jaundice - Neurocognitive impairment

Laboratory findings:
FBC: Hb: 5-9g/dl, Normal MCV, Retic count: 5-15%
WBC: 12,000-20,000/mm3 (Diff:-normal or predominance of neutrophil)
Platelets: normal or increased
Nucleated RBC
Blood film: hypochromic, normocytic cells, target cells, poikilocytes, sickled cells.

Diagnosis: Clinical – 80%


Laboratory diagnosis:
Electrophoresis: migration of proteins in an electric field.
i) cellulose acetate (pH 8.6): Hb S, D & G migrate at same rate.
ii) citrate agar (pH 6.3): clearly differentiates Hb
iii) iso-electric focusing: superior to above 2 but used in newborn screening b/c of good seperation of HbF.
Sickling test: induce deoxygenation using 1 drop of blood + 1 drop of 2% Na metabisulphite and read under microscope after 24hrs
Solubility test: solutions of high molarity (phosphate buffers, Na dithionate) are mixed with blood. HbS is poorly soluble in solution.
total pink solution = HbAA, CC, DD
ppt + pink solution = AS, SC
ppt + clear solution = SS

Mgt during steady state: Not in crisis and has no other acute illness for at least 4weeks
Emphasis is on prevention of crises by removing encouraging factors:
- Avoid exposure to cold and high altitudes
- Avoid dehydration (liberal fluid intake)
- Folic acid supplementation
- Regular malaria prophylaxis and prompt Rx.
- Administration of pneumococcal and Hib vaccines
*Hepatitis B vaccine and Fe chelators (regular transfusion)
 Health education and genetic counselling
 Baseline steady state physical and laboratory findings
 Regular follow-up.

New therapeutic approaches:


Anti-sickling agents: urea and cyanate but are toxic. Local chewing stick in Nigeria- fagara zanthoxyloides (NIPRISAN, NICOSAN)
Modulation of HbF synthesis: tried hydroxyurea, Na butyrate, EPO, Rasveratrol (dietary polyphenol). Most successful is hydroxyurea-
↑HbF and ↓adhesion molecules.
Prevention of RBC dehydration: Oral mg and clotrimazole inhibit k-cl cotransporter and k+ pump which control rbc hydration.
Na cromoglycate inhalation- ↓ sickled cells.
Red cell exchange: therapeutic erythrocytapharesis (TEA)
Bone marrow transplantation: has potential for cure and reserved for pts with severe clinical course.
Stem cell transplantation
Gene therapy: has potential future benefit but still in its infancy
Role of anti-oxidants: prevention of peroxidative damage e.g. Vit c.
Recombinant Hb: genetically engineered recombinant Hb may in long future replace transfusion with donor blood.

Prevention of SCA
Genetic counselling: Premarital counselling and screening
Prenatal diagnosis
?? Selective abortion
Routine neonatal screening: early Dx and ff-up

116
Compiled by Obasi. D. C. Chinedu

SEIZURE DISORDERS IN CHILDHOOD


PROF NGOZI C. OJINNAKA

LEARNING OBJECTIVES
To understand the definition of seizure and be able to identify a seizure in any age group
To have a good knowledge of possible causes of seizure in any age group
To have a good knowledge of the current definition and classification of epilepsy
To be able to identify the common epileptic seizures in children

INTRODUCTION:
DEFINITION: Seizures are abnormal cortical/subcortical neuronal discharges involving the grey matter. They are sudden, excessive and
rapid in nature. These discharges cause alterations in behavior, sensation, or consciousness.
A seizure (convulsion) can also be defined as a paroxysmal involuntary disturbance of brain function that may manifest clinically as an
impairment or loss of consciousness, abnormal motor activity, behavioral abnormalities, sensory disturbances, or autonomic dysfunction.
Some seizures are however characterized by abnormal movements without loss or impairment of consciousness.
Childhood seizures are the most common neurological problem encountered worldwide. They can occur alone or in association with other
chronic neurological problems

CAUSES OF SEIZURES IN CHILDHOOD: Certain seizures in the pediatric population are age specific.
a. Neonatal period: Convulsions in the neonatal period suggest hypoxic-ischemic encephalopathy as a result of
Birth trauma resulting in birth asphyxia and intracranial hemorrhage
Metabolic causes: Hypoglycaemia (blood sugar <2.2mmol/L), hypo or hypernatraemia (Na level <120mEq/l or >150mEq/l),
hyperbilirubinaemia (inc. level of unconjugated bilirubin), hypocalcaemia, Pyridoxine def. or dependency.
Infections: Meningitis (common organisms are E.coli, Staph. aureus, GBS etc), Encephalitis (mainly from HSV type II). This is a diffuse
process and encephalomalacia may result.
Drug withdrawal (eg. anticonvulsant)
Congenital malformations (mainly vascular)
Familial seizures.

Seizures in the neonatal periods are usually very subtle


In premature infants, for instance, are associated with abnormal eye or facial movements; the motor component is often that of tonic
extension of the limbs, neck, and trunk. Term infants may have focal or multifocal, clonic or myoclonic movements but may also manifest
more subtle seizure activity. Apnea may be the first manifestation of seizure activity, particularly in a premature infant.

b. Post neonatal Period


Febrile Seizures (FS): Most common convulsive disorder in children. Defined as convulsion asso. with fever without evidence of
intracranial infection or defined cause for the seizure occurring b/w 6months to 6years.
It is strikingly age –dependent. Most FS occurs between 6 months and 6 yrs. Very rare after 6 years. Peak incidence is at 18 months.
Approx. 6-15% occur after 4 years. Onset after 6 years is unusual. May occur early in the illness and may be the presenting feature.
Others occur during or after the onset of fever though relationship to the 0º of temp. not definite. Peak of fever may be related to the
recurrence of seizures. Children convulsing at temperature < 38.9º may present with a focal seizure initially or seizure that is repeated within
the same illness.

Common causes in our environment include: Malaria, respiratory tract infections, UTI, sepsis. FS have a genetic predisposition.

Types of febrile seizures: Two main types, Simple and complex FS.
Simple: Gen. tonic-clonic without focal features. duration <10minutes. no recurrence in the next 24hours and resolves spontaneously.
Complex FS (*complicated): Focal, prolonged duration (>10-15min), reoccurrence within 24hrs or within the
febrile illness, May be asso. with a neurological deficit. Higher tendency to develop epilepsy later in childhood.
Incidence: Population incidence is 2-5% in the Western world and 5-10% or more in developing countries. Proportion of initial FS being
complex is 9-33%. Febrile status epilepticus accounts for 25% of all status epilepticus.

RISK FACTORS: What are the risk factors for developing a febrile seizure?
- Age
- Sex: more common in male (M:F =2:1)
- Presence of a close relation (1st degree relation) with FS. The risk of siblings having an FS is 10-45%.
- An underlying brain disorder
- Viral infection esp. human HSV

Risk factor: What are risk factors for recurrence?


- Only about 33.3% of children that had a 1st FS will experience another
- Age: single, strongest and most consistent risk factor for recurrence. 1st FS at age less than 18 mnths
- A family history of FS in 1st º relative
117
Compiled by Obasi. D. C. Chinedu

- Temperature: seizure at a lower temp, and a short duration of fever


- Multiple initial seizure occuring within the same febrile episode

Risk of later epilepsy: Only 2-4% of children with one febrile seizure develop epilepsy. 2% of children with SFS & 12% of those with CFS
develop epilepsy. 15-20% of pts. with epilepsy have a history of FS. Strong asso. between CFS and temporal lobe epilepsy (TLE). MRI
showing hippocampal oedema within 48 hrs of seizure & later sclerosis and atrophy. Retrospective studies have shown that 40% of adult
with TLE give a hx of CFS in childhood.
Longitudinal study is required on this.

Management: Evaluation of a child with febrile seizure


Detailed history: Was the seizure really febrile? occasionally fever follows an afebrile tonic-clonic seizure though this does not exceed 38ºC.
Is there a possibility of meningitis, encephalitis or cerebral malaria (Need for thorough history).
Investigations: LP for children under 2-3years with particular attention to history and mode of presentation. Look out for signs of meningitis
in the older child. FBC & film for MP & urinalysis
What about an EEG? EEG is not indicated in the workup of a simple FS. 39.43% of those with CFS will have abnormal EEG & often >
3years old
The main challenge posed upon presentation is whether a lumbar puncture is indicated. Despite several authoritative guidelines over the
past 15 years, there is still no consensus, with seemingly contradictory recommendations. AAP and National guidelines from UK suggest LP
in infants because of poor clinical signs of meningitis in this age group. However with changing epidemiology of bacterial meningitis over
the past two decades in its overall prevalence in the western world, few authors have their reservation on this issue.

Treatment: Manage the fever: Expose, fan, tepid sponge and Antipyretics
What is the role of anticonvulsants: Used to stop seizures immediately: Diazepam, Paraldehyde in our environment.
Is prophylaxis indicated? PHB has been used extensively but has its limitation. Few studies found phenytoin & carbamazepine ineffective.
Valproate was found effective in few reported studies. Treat the underlying cause of fever.
Outcome of febrile seizures: 30-40% of at least one recurrence and a 10% chance of 3 or more recurrence. More recurrence within 6-12
months of initial seizure. No evidence of brain damage is documented. Mismanagement in our environment may lead to neurological
deficits.

EPILEPSY: Most common chronic neurological disorder in the developing countries. Most chronic disease among school children. Pt who
have had seizures in the last 2-5 yrs or are currently on anti-convulsant therapy are known as active epileptics.

DEFINITION: Epilepsy is defined as a disorder of the brain characterized by any of the following conditions:
(1) At least two unprovoked (or reflex) seizures occurring >24 h apart,
(2) One unprovoked (or reflex) seizure and a probability of further seizures similar to the general recurrence risk (at least 60%) after two
unprovoked seizures, occurring over the next 10 years and
3) Diagnosis of an epilepsy syndrome

CLASSIFICATION
Old:
a. Generalized Seizures
b. Partial Seizures
c. Unclassified

NEW: (See Image)

GENERALIZED SEIZURES: are characterized by widespread involvement of bilateral cortical regions at the outset and are usually
accompanied by impairment of consciousness. The familiar tonic-clonic seizure (grand mal) is often preceded by a cry. The patient suddenly
falls to the ground and exhibits typical convulsive movements, sometimes with tongue or mouth biting and urinary incontinence. Other
subtypes of generalized seizures include absence, myoclonic, clonic, tonic and atonic seizures.

Absence Seizures: Simple absence seizures usually last 5-10 seconds and commonly occur in clusters. They manifest as sudden onset of
staring and impaired consciousness. Common about 8 -12 years and remits in adolescence. Commoner in females. The EEG typically
shows a 3Hz spike and wave pattern. There is a strong genetic component for the seizures as well as for the EEG abnormality. Absences
may remit during adolescence in around 40% of patients.
Atypical absence seizures usually begin before 5 years of age in conjunction with other generalized seizure types and mental retardation.
Predisposing factors can perinatal events. They last longer than typical absence seizures and are often associated with changes in muscle
tone. EEG pattern varies and can present with generalized spikes, sharps and 2-4.5 spike-wave complexes.

Myoclonic seizures: consist of sudden, brief muscle contractions, either singly or in clusters, that can affect any muscle group. Juvenile
118
Compiled by Obasi. D. C. Chinedu

myoclonic epilepsy: starts in adolescence and persists. It is a lifelong condition.


Clonic seizures: are characterized by rhythmic or semi-rhythmic muscle contractions, typically involving the upper extremities, neck and
face.
Tonic seizures cause sudden stiffening of extensor muscles, often associated with impaired consciousness and falling to the ground.
Atonic seizures (drop attacks) produce sudden loss of muscle tone with instantaneous collapse, often resulting in facial or other injuries.
Infantile spasms: very common in infancy. Has characteristic manifestations. Hypsarrythmia on EEG

PARTIAL SEIZURES can be simple or Complex.


Simple Partial: No loss or impaired consciousness, Benign. Can occur at any age group. Motor activity is the most common symptom of
SPS. The movements are characterized by asynchronous clonic or tonic movements, and they tend to involve the face, neck, and extremities.
Limited to one limb or side.
Complex partial Seizure: Common in all age group. Impaired consciousness. Limited to a limb or side. Symptomatic epilepsy.
CPS with Secondary generalization

Epileptic syndromes: Rolandic epilepsy (benign in nature), West syn., Lennox-Gastaut syn.
Refractory seizures
Others: Nocturnal, Gelastic (laughing) epilepsy, Reflex epilepsy, Morphiec epilepsy.

MANAGEMENT:
Detailed history: from a reliable observer and/or patient. Questions to ask
 Onset: When did it occur. How did the attack(s) start? What was the first thing that happened?
 What was the duration, time of occurrence (e.g., on awakening, when drowsy, during sleep) and frequency of seizures.
 What were the events before, during and after the seizure?
 Was patient aware or not aware of the surrounding?
 Was there any warning?
 Was there a change in behaviour?
 Could the event be interrupted?
 Any motor involvement? If yes what type?
Past attack /or current occurrence of other seizure types (especially myoclonic, or absence) should be obtained.
Medical history. Pregnancy, birth and neonatal history. Development, current school performance. Family history of epilepsy and learning
difficulties and social history.

Physical Examination: Clinical, neurological examination and fundus examination. signs to look out for:
 Gross motor dysfuction
 Sensory abnormalities
 Dysmorphic features
 Signs of other co-morbidities
 Response to hyperventilation (if necessary)

EEG Findings in Epilepsy: Absence – 3/sec spike-wave complexes, Infantile spasms- Hypsarrhythmia

Treatment: The optimal goal in seizure treatment is to achieve complete seizure control with no adverse effect from modality of
treatment. This helps in improving the quality of life of the child with epilepsy.

Treatment options: could be pharmacological or Non-pharmacological.


Pharmacological: preferred and more efficacious

Anticonvulsant Therapy: Bromide, Phenobarbitone, Phenytion, Carbamazepine, Ethosuximide, Sodium Valproate.


Gabapentin: This anticonvulsant is used as an add-on drug for patients with refractory complex partial and secondarily generalized tonic-
clonic seizures. The mechanism of action results from binding of the drug to neuronal membranes (glutamate synapses) and an increased
brain GABA turnover.
Corticosteriods: ACTH, Prednisone.
Benzodiazepines: Diazepam, Lorazepam, Clonazepam, Nitrazepam.
NEWER ANTICONVULSANTS: Levetiracetam (Keppra), Lamotrigine, Topiramate, Vigabatrin, Zobisamide.

ADVERSE EFFECTS OF MAJOR ANTICONVULSANTS:


- Valproate has a high risk for congenital malformation and neurodevelopmental impairments (learning difficulties and autism) if used
during pregnancy. Highly restricted in the adolescent female.
- Phenytoin has protean side effects. Carbamazepine can cause severe dermatological disturbances.
- Phenobarbitone has profound sedative effect. Affects cognitive function and can cause hyperactivity
- A class of anticonvulsants known as carbonic anhydrase inhibitors (Topiramate, zonisamide) are known to increase the risk of
kidney stones.

119
Compiled by Obasi. D. C. Chinedu

Seizure type First-line AED Daily maintenance dose Remark


range

Myoclonic seizures sodium val, 0–30mg/kg Do not offer carbamazepine,


levetiracetam 15–50mg/kg gabapentin, phenytoin
topiramate 2–12mg/kg vigabatrin

Infantile spasms Steroid (prednisolone 1mg/kg vigabatrin 1st-line in infants


+vigabatrin with infantile spasms due to
tuberous sclerosis

MAE (Doose syndrome) sodium Val 10–30mg/kg Rarel responds to


clobazam 0.2–1.0mg/kg monotherapy
ethosuximide 20–30mg/kg
lamotrigine 1–15mg/kg

Lennox-Gastaut syndrome Sodium valproate 10-30mg/kg Avoid carbamazepine

Epilepsy of the centro-temporal oxcarbazepine 25–35mg/kg Carbamazepine drug of


spikes carbamazepine 10–25mg/kg choice
levetiracetam 15–50mg/kg
lamotrigine 1–15mg/kg

JME sodium Val levetiracetam 10–30mg/kg Avoid CMZ, Phenytoin.


15–50mg/kg lamotrigine
exacerbate seizures

Gen Epilepsy with febrile seizure Sodium Val 10-30mg/kg Carbamazepine exacerbates
+

Focal seizures carbamazepine 10–25mg/kg Monotherapy is


levetiracetam 15–50mg/kg preferred.
lamotrigine 1–15mg/kg Levetiracetam often
oxcarbazepine 25–35mg/kg used as adjuvant

Newly-diag sodium Val 10–30mg/kg


GTC seizures lamotrigine 1–15mg/kg
clobazam 0.2-1mg/kg

Absence Ethosuximide 20–30mg/kg high risk of GTCs, offer


Sodium Valproat 10–30mg/kg sodium val first
NON-PHARMACOLOGICAL: Approximately 20–30% will develop medically refractory (drug-resistant) epilepsy. For this population,
non-pharmacological treatments can be highly efficacious.
Surgery: Epilepsy surgery is highly successful in achieving seizure freedom in carefully selected children with drug-resistant focal epilepsy.
Ketogenic diet
Vagal stimulation: Very important in refractory seizures

Other Non-pharmacological
Counselling: very important in seizure management especially in our environment
Psychological support and educational assessment often contribute enormously to ongoing management.

120
Compiled by Obasi. D. C. Chinedu

CHILDHOOD LEUKAEMIAS
DR. B.F. CHUKWU

Learning objectives: At the end of this lecture, students are expected to:
 Define leukaemia
 Have a knowledge of the Risk factors for leukaemia
 Classify leukaemia
 Know: common clinical features
 Basic relevant investigations
 Prognostic factors
 Differential diagnosis
 Modalities of treatment

Introduction: Leukaemias are malignant progressive disease of haemopoietic stem cells in which genetic abnormalities of these cells result
in:
Increased and unregulated proliferation of immature white cells (blasts)
Decreased rate of spontaneous cell death (apoptosis) of the blasts.
Diminished production of normal blood cells e.g., mature neutrophils, erythrocytes and platelets
Disruption of normal marrow function, Ultimately, resulting in marrow failure

Epidemiology: Leukaemias are the most common childhood neoplasms worldwide (31% of all malignancies in children < 15 years). ALL
represents about 75% of all cases. AML -20% of all childhood leukaemias. CML (3-5%). CLL- Very rare in children.
In Nigeria and other developing countries, leukaemias are less common than Lymphomas

Aetiology: Like other cancers, the aetiology of leukaemia is unknown. Alterations/mutations in genetic code or DNA underlies cancer
development including leukaemias. Majority of leukemias in children initiated by acquired mutations within the haemopoietic tissue while
some mutations occurred in utero. Certain factors are associated with increased predisposition (risk factors).

Risk factors: can be genetic or Environmental.


Genetic factors (gene mutation): They include; Fanconi anaemia, Bloom syndrome, Neurofibromatosis (Loss of both alleles of NF-1 in
bone marrow cells), Ataxia-telangiectasia, severe combined immunodeficiency, Turner syndrome, Down syndrome (10-20 fold increase
before 10 yrs), Diamond-Blackfan syndrome.
Environmental factors: They include; Alkylating agents e.g. cyclophosphamide, Benzene, Ionizing radiation (prenatal X-ray, postnatal
radiation), Viruses e.g. Epstein-Barr viral infection, Maternal alcohol consumption during pregnancy, parental smoking, pesticide exposure.

Classification of leukaemias: Acute or chronic leukaemia based on biological behaviour of disease.


Acute: survival in weeks or months without treatment, malignant cells mostly blasts.
Chronic: proportion of blasts usually low and survival may be up to several months or years.

Can also be: Lymphoid, myeloid or mixed lineage


lymphoid: B-cell or T-cell
Myeloid: malignant white cells other than lymphoid lineage.
Mixed lineage: features of both lymphoid and myeloid cells.

ALL: Peak age incidence 2-5 years in whites, 7-10 in blacks. male > females (M:F = 1.3:1) of all ages. whites > blacks. High/middle
income > lower income countries.

Classification of ALL:
Immunophenotype using surface markers
B-cell lineage (80-95%), predominantly early B-precursor. About 1% are mature B-cells
T-cell lineage (10-15%); common in males, older age, mediastinal mass, high WBC count, CNS disease at presentation e.g. cranial nerve
palsies
Morphology (FAB):
 L1: small with little cytoplasm and absent or inconspicuous nucleoli (85% of cases)
 L2: larger, increased cytoplasm, irregular nuclear shape, prominent nucleoli (14%)
 L3: deeply basophilic and vacuolated cytoplasm, prominent nucleoli, “Burkkit leukaemia”

Chromosomal abnormalities: influence prognosis and type of treatment

121
Compiled by Obasi. D. C. Chinedu

Clinical features: Features result from: Progressive bone marrow failure and Bone marrow and extra-medullary infiltration by leukaemic
blasts.
Initial symptoms usually nonspecific, including: Anorexia, lethargy, irritability, Weight loss, Paleness of the body.
Recent history of viral infection e.g. URTI, exanthema. Fever due to infection or mediated by cytokines
Easy fatigability due to anemia
Bone pain- direct leukemic infiltration of periosteum or expansion of marrow cavity by leukemic cells. Pathologic fracture may also cause
pain
Others include: Easy bruising or bleeding, Epistaxis, Painless lymphadenopathy, Hepatomegaly, Splenomegaly, Scrotal enlargement (usually
unilateral).
Tachypnea, orthopnea, resp. distress due to mediastinal mass in T-cell ALL
Neck rigidity due to CNS infiltration

Investigations:
FBC: Low Hb, normocytic normochromic RBCs, decreased retic count. WBC (low, normal or high), neutropenia.
Thrombocytopenia.
Peripheral blood film: blast cells in peripheral blood (absence does not rule out diagnosis).
Bone marrow aspiration: at least 25% blasts in bone marrow confirms diagnosis (marrow may be hypocellular).
S/E/U/Cr, LDH, Uric acid, calcium, phosphorous.
Liver function test, LDH
CXR, CT scan: chest, abdominopelvic, Abdominal ultrasound (enlarged LNs, Kidney infiltration)
Cultures –blood and other suspected sites of infection
Lumber puncture and CSF examination (CNS involvement has both prognostic and therapeutic implications)
ECG, echocardiography

Other investigations:
 Chromosomal studies for leukaemia-associated chromosomal translocations e.g. BCR-ABL [t(9;22)], TEL-AML 1 [t(12;21)], detected by
PCR or FISH (fluorescence insitu hybridization).
 Chromosomal abnormalities identified in approximately 80-90% of childhood acute leukemia.
 Immunophenotyping- use of immunological markers to detect cluster of differentiation (CD) antigens expressed by cells of a particular
lineage, e.g. B-cell lineage ALL is positive for CD10, CD19, CD22.
 T-cell lineage ALL express CD3, CD7, terminal deoxynucleotidyl transferase (TdT).
 CD10, common ALL antigen (cALLa).

Diagnosis: Clinical features from history and physical examination. Peripheral blood film showing blasts. Bone marrow aspiration or
biopsy showing >25% blasts in bone marrow, 5 to <25% suspected, < 5% normal.

Differential diagnoses:
- Chronic infections e.g. EBV, CMV ( LNs, hepatosplenomegaly, anaemia, fever)
- Aplastic anemia (pancytopenia, no lymphadenopaty or hepatosplenommegaly)
- SCA (anaemia, bone pain, hepatosplenomegaly) - ITP (petechie, purpura)
- Acute myeloid leukemia - JRA ( fever, joint pain, hepatosplenomegaly)
- Marrow infiltration by other malignancies (neuroblastoma, rhabdomyosarcoma, retinoblastoma).

122
Compiled by Obasi. D. C. Chinedu

Treatment:
Combination chemotherapy
A. Induction phase: aims to induce remission of disease and resumption of normal haematopoiesis. 3-4 drugs including steroid for at least 4
weeks.
B. Consolidation phase: Aims at eradicating remaining leukaemic cells and prophylaxis against cells of “sanctuary” sites. IT, component of
consolidation.
C. Maintenance or continuation phase: for maintaining remission while bone marrow recovers.
D. Intrathecal chemotherapy for CNS prophylaxis or therapy

Standard risk regimen

Intrathecal chemotherapy

Other combinations: COAP,


Immunotherapy- use of monoclonal antibodies.
- Supportive treatment:
- Tumor lysis syndrome (hyperuricaemia, hyperkalaemia, hyperphosphatemia)- prophylaxis or treatment.
- Fluid, Sodium bicarbonate, allopurinol.
- Antimicrobials (prophylaxis or treatment) - Blood and blood derived products
- Nutritional support - Psychosocial support - Nursing care. - Drug adverse effects/Toxicity

Before chemotherapy: Hb-----10g/dl, TWBC----2 x 10^9/L, ANC-----1 X 10^9/L, Platelets-----100 x 10^9/L.


Relapse: High rate of relapse. Sites include; Bone marrow, CNS, Testes.

Management of chemotherapeutic side effects:


- Bone marrow suppression - Alopecia - Hemorrhagic cystitis
- Cardiotoxicity - Neuropathy - Impaired sexual development or fertility
- Hyperpigmentation - Secondary malignancy—long term

Prognostic factors:

123
Compiled by Obasi. D. C. Chinedu

Acute myeloid leukaemia (AML): 20% of childhood leukaemias. More in infants and adolescents. No gender or racial differences as in
ALL. Eight morphological subtypes:
FAB classification of AML

M1, M2, M4, M5 accounts for 80% AML in children. M3 accounts for 10%, associated with t(15;17) chromosomal translocation and high
risk of bleeding.

Clinical features: Similar to ALL due to marrow infiltration and failure: They include Anaemia, Bone pain, Hepatomegaly, Splenomegaly,
Lymphadenopathy, Gingival hypertrophy in 10-15% of cases.
Others are: Parotid gland swelling, Proptosis due to retro-orbital leukaemic deposits (chloroma). Paraplegia due to epidural chloroma

Investigations: As in ALL
Bone marrow aspiration biopsy: 20% blasts in bone marrow
Cytochemical and morphological analysis differentiates AML from ALL: e.g.
- Myeloperoxidase (enzyme in granulocytes) test, + for AML, - for ALL
- Sudan black stain (stains polymorph granules), + for AML, - for ALL
- Periodic Acid-Schiff stain (stains glycogen deposits), + for ALL and erythro-leukemia.

Treatment: Regimen for treatment of AML (See table in the image below)
Others are: Immunotherapy e.g. Gemtuzumab is anti-CD33 monoclonal antibody. Also HSCT following remission.

124
Compiled by Obasi. D. C. Chinedu

Chronic myeloid leukaemia (CML): 3-5% of childhood leukaemias. Two types exist, viz:
- Philadelphia chromosome t(9;22) positive (90-95% )
- Juvenile myelocytic leukemia, del (7q)
Philadelphia chromosome t(9;22, bcr-abl protein with excessive Tyrosine kinase activity. TK phosphorylates cells involved in cell division.

Clinical features: Insidious onset, Anorexia, Weight loss, Night sweat, Massive splenomegaly, Priapism and Visual disturbances usually
due to increased viscosity from WBC.

Philadelphia positive (adult CML): 3 phases:


- chronic phase (3-4 years)- leukocytosis can be controlled with HU, Busulphan, TK inhibitor
- Blastic crisis phase and Accelerated phase: resembles ALL, AML, refractory to therapy. BMT curative.

Investigations:
1. FBC- leukocytosis: hyperleukocytosis
2. Blood film: all forms of myeloid cells
3. Thrombocytosis
4. Bone marrow ASB: hypercellular marrow, normal myeloid cells in all stages of differentiation
5. Chromosomal studies (Southern blot or PCR) shows t(9;22).

Treatment:
Hydroxyurea and Busulphan for chronic phase
Interferon-α suppress Philadelphia chromosome completely
Imatinib, nilotinib: TK/BCR-ABL protein inhibitors
BMT

Supportive treatment:
- Transfusion of blood products: RBC, platelets
- Antimicrobials
- Allopurinol (xanthine oxidase inhibitor)
- Rasburicase (uric acid oxidase)
- Hyper-hydration

Questions
 What is leukaemia?
 What % of childhood leukemia is ALL
 What is the pathogenesis of leukemia symptoms and signs?
 What are the hematological baseline values before commencing chemotherapy of leukemia?

125
Compiled by Obasi. D. C. Chinedu

TUBERCULOSIS (TB)
Dr. Muoneke

Introduction: An air-borne infectious disease 10arily affecting the lungs. Major cause of morbidity and mortality esp in people in
resource-poor settings, esp.in Africa and Asia. Poses significant challenges to developing economies as it affects more people in their most
productive years. Transmitted via aerosols mainly and is both preventable and curable.

Epidemiology: 1/3rd of the world’s population (~ 2 billion people) carry the TB bacteria. More than 9 million of these become sick each year
with active TB that can be spread to others. New infections occur in about 1% of the population each year. In 2014, there were 9.6 million
cases of active TB which resulted in 1.5 million deaths. More than 95% of deaths occurred in developing countries.

Aetiology: Caused by a small, aerobic, non-motile bacillus known as Mycobacterium tuberculosis.

Other types include:


M. africanum- a significant cause of TB in parts of Africa.
M. bovis- Results from drinking unpasteurized milk
M. canetti- rare and limited to the Horn of Africa and in some African emigrants.
M. microti- also rare and mostly seen in immunodeficient people.
Other known pathogenic mycobacteria include: M. leprae, M. avium and M. kansasii.

The latter two species are classified as "nontuberculous mycobacteria (NTM). NTM neither causes TB nor leprosy but can cause
pulmonary diseases that resemble TB.

Classification of TB:
TB can be classed into 2 important types: Pulmonary and Extra-Pulmonary.

Pulmonary: Active tuberculosis infection commonly involves the lungs (in about 90% of cases). Produces such symptoms as chest pain and
a prolonged/chronic cough producing sputum. About 25% of people may remain "asymptomatic”.

Extra-pulmonary: Spread of infection to other parts of the body apart from the lungs in 15–20% of active cases. These are collectively
noted as "extrapulmonary tuberculosis". Occurs more commonly in immunosuppressed persons (>50% of HIV pts) and young children.

Common sites include: The pleura, The CNS (tuberculous meningitis), The lymphatic system (scrofula of the neck), The GUS, The bones
and joints (in Pott disease of the spine). When it spreads to the bones, it is also known as "osseous tuberculosis", a form of
osteomyelitis.
A more serious, widespread form of TB called "disseminated tuberculosis", or miliary tuberculosis. Miliary TB makes up about 10% of
extrapulmonary cases.

Risk factors for TB: A number of factors make people more susceptible to TB infections.
1. HIV - 13% of all people with HIV are infected by the mycobacterium.
2. Overcrowding 3. Malnutrition

Those at high risk also include:


 People who use illicit drugs (HIV and later TB)
 Inhabitants and employees of locales where vulnerable people gather (e.g. prisons and homeless shelters)
 Medically underprivileged and resource-poor communities, high-risk ethnic minorities
 Children in close contact with high-risk category patients
 Health care providers.
 Chronic lung disease is another significant risk factor. eg Silicosis increases the risk about 30-fold.
 Cigarette smoking have nearly twice the risk of TB than nonsmokers.
 Alcoholism and Diabetes mellitus (threefold increase).
 Drugs such as corticosteroidsand infliximab (an anti-αTNF monoclonal antibody) are becoming increasingly important risk factors,
especially in the developed world.
 There is also a genetic susceptibility for which overall importance remains undefined

Pathogenesis:
About 90% of those with M. tuberculosis are asymptomatic, 10% may progress to active tuberculous disease. 10% of HIV patients are at risk
of developing active TB.
Mycobacteria on reaching the pulmonary alveoli, invade and replicate within endosomes of these alveolar macrophages. Macrophages
attempt to eliminate it by formation of phagosomes and phagolysosomes but this fails due to the presence of a thick, waxy mycolic acid
capsule that protects the bacteria from these toxic substances.
M. tuberculosis reproduces inside the macrophages and will eventually kill the immune cell.

126
Compiled by Obasi. D. C. Chinedu

The 10 site of infection in the lungs, known as the "Ghon focus” is generally located in either the upper part of the lower lobe, or the
lower part of the upper lobe. “Simon focus” typically found on the top of the lung results from infection from the blood stream.
Granulomas are formed following aggregation of Macrophages, T lymphocytes, B lymphocytes, and fibroblasts forming around the
mycobacteria engulfed by the alveolar macrophages - Classified as a granulomatous inflammatory disease.

The plight of the bacteria within the granuloma:


 Can become dormant, resulting in latent infection
 The granuloma may result in abnormal cell death or caseous necrosis in the center of tubercles.
 Can spread through the blood stream to affect multiple areas of the body => Miliary TB common in young children and those with HIV
and resultant high fatality rate of about 30% even with treatment.
 Formation of scar tissues in the lungs and cavities filled with caseous necrotic material.
 Some of these cavities are joined to the bronchi and this material containing live bacteria can be coughed up enhancing further spread of
the infection.

The infective droplets: The infective droplet nucleus is very small, measuring 5 µm or less, and may contain approx. 1-10 bacilli. About 5-
200 inhaled bacilli are usually required for infection. Droplets are suspended in the air for a long time due to their small size.
On inhalation, they cause 10 infection of the lungs with the risk increasing in small enclosed areas and in areas with poor ventilation. The bacilli
then get deposited (usually in the midlung zone) into the distal respiratory bronchiole or alveoli, which are subpleural in location. Subsequently,
the alveolar macrophages try to kill the inhaled bacilli but are unable, hence the bacilli continue to multiply unimpeded.

THE NATURAL HISTORY OF UNTREATED 10 TB


Initial infection is followed by a period of incubation during which The 10 focus forms, leading to enlargement of regional nodes and
subsequently, the organisms escape into the blood stream. This stage is symptomless, however 6-8 weeks after infection symptoms like
fever, malaise may develop.
The infection in most children, however, is never clinically manifest. Phlyctenular conjunctivitis is seen occasionally, and should always
alert the clinician to the probability of tuberculosis.

After this initial phase, the clinical syndromes can be divided into
A) Those arising from the 10 focus,
B) Those arising from the regional lymph glands
C) The dissemination.

Progressive enlargement of the 10 focus leading to cavitation sometimes, especially in young children (a progressive 1 0 lesion).
The regional lymph nodes always enlarge in 10 TB, usually bigger in young children.
Symptoms and signs are related to compression by the enlarged Nodes on a bronchus => obstructive emphysema, or collapse with
consolidation (a segmental lesion). Bronchial erosion is commoner in early childhood; it usually occurs between 3 and 9 months after infection.
Disseminated disease is more frequent in young children. The liver and spleen are often enlarged. 20% of children under 1 year, and 4% of
children infected under 5 years old will develop tuberculous meningitis or miliary TB each developing within 12 months of infection in 90%
of cases. Bone and joint lesions occur mainly within 3 years of infection. Renal and skin tuberculosis are late complications, mostly
occurring five or more years after infection.

Complications from each of these three situations can occur simultaneously, especially in young children and the risk of serious dissemination
is related inversely to age from birth to 10 years, with a rise again at puberty.

Clinical Manifestations:
General signs and symptoms include: Fever, chills, Night sweats, loss of appetite, weight loss and fatigue. Significant finger clubbing may
also occur.
Pulmonary: Chest pain, Prolonged cough producing sputum. Hemoptysis in small amounts, and in very rare cases, may cause massive bleeding
when the infection erodes into the pulmonary artery (Rasmussen's aneurysm). May cause extensive scarring with fibrosis in the upper
lobes of the lungs.

Extrapulmonary: Occurs more commonly in immunosuppressed persons and young children.


The pleura- TB pleurisy +/- Pleural effusion The CNS- TB meningitis,
The lymphatic system- in scrofula of the neck The GUS (in urogenital TB)
The bones and joints (in Pott's disease of the spine).

The "disseminated" form (miliary TB) may sometimes present as a space occupying lesion- an intracranial tuberculoma. It may manifest
subacutely with low-grade fever, malaise, weight loss, and fatigue. History of cough and respiratory distress may be obtained

Tuberculosis of the CNS may present in 3 different ways:


1. Meningitis: In the young child, the onset of meningitis is often insidious; there is neck stiffness, and Kernig’s sign is usually absent. The

127
Compiled by Obasi. D. C. Chinedu

rest may present with fever, headache, vomiting, and neck stiffness. CSF examination shows mainly lymphocytes, the sugar is reduced and
the protein raised.
2.Encephalitis: the pathology is essentially massive caseation, with progressive changes in one or both cerebral hemispheres with little or no
reaction in the meninges. There is fever, headache and vomiting, but no meningeal signs. The CSF contains only a few lymphocytes, but the
protein is raised.
3. A space occupying lesion with headache, vomiting, papilloedema with or without localizing neurological signs - tuberculoma (in up to
50% of the cases)

Investigations: Diagnosing active tuberculosis based merely on signs and symptoms is difficult but considered when lasting >2wks.

Conventional diagnostic tools


1. Symptoms and signs (History) – None specific
2. Tuberculine Skin Test (TST) or Mantoux often used to screen people at high risk for TB. The result could be False positive in those who
have been previously immunized or False negative in those with Sarcoidosis, Hodgkin's lymphoma, malnutrition, or most notably, in those
who truly do have active TB & in those with poor immunity.
3. Radiograph – Xrays of the chest and other parts of the body indicated.
4. Bacteriology; AFB Culture- Multiple sputum cultures for acid-fast bacilli. (6-8 wks)
5. Haematology; ESR, FBC
(Gram stain is not done- mycobacterium is unable to retain dyes due to the high lipid and mycolic acid content of its cell wall but is able to
retain certain stains (Ziehl–Neilsen stain) even after being treated with acidic solution thus the name- Acid-fast bacillus (AFB).
(A definitive diagnosis of TB is made by identifying M. tuberculosis in a clinical sample e.g. sputum, pus, or a tissue biopsy usually
taking 2-6wks to develop).
6. Gene Xpert MTB/RIF (GeneXpert multi-disease platform): Simplified form of molecular testing, Fully integrated and automated sample
preparation, Detects MTB and RIF resistance-conferring mutations (rpoB) directly from sputum.

Limitations of Gene Xpert: Expensive, Technical, Need for Power, Detects only RIF resistance. Some cases may have false positive RIF
resistance, Does not preserve DNA for further testing, Selective sensitivity in extrapulmonary samples.

Management of Childhood TB, what’s new?


In 1995 DOTs introduced and changed the face of TB Mgt and control. 22 million lives were saved through DOTS & Stop TB Strategy
between 1995-2012. The TB death rate dropped 45% between 1990 and 2012. Realization for need for evidence based guidelines.

Directly Observed Treatment (DOT): DOT ensures that the TB patient takes the right drugs, in the right doses at the right times. The
treatment supervisor watches the patient swallow the tablets throughout the whole course of treatment. Supervisors could be either a
health worker or “treatment supporter” (which could be a trained; volunteer, member of the family or the community or guardians). A
patient-centred approach with proper communication between the patient and treatment supporter promotes patient education, good
adherence and early identification of challenges with treatment (including side effects and clinical worsening). All treatment supporters
should be chosen and be acceptable to the patient.
The need for good adherence and follow-up should be reinforced at all times. Patients should be reminded about the duration of treatment and
common side effects. DOT allows the prompt detection and mgt of adverse drug reactions and clinical worsening of TB.

There are 2 phases in the DOTs treatment regimen


The initial intensive phase of directly observed daily administration of drugs is 2 months for both regimen.
The continuation phase of treatment for the six months regimen is 4 months and that of the twelve months regimen is 10 months of drug
administration (-2).

Six Months Regimen(2RHZ+E/4RH): A 6 months Short Course treatment under supervision for a child with pulmonary TB or any form of
extra – pulmonary TB other than TB meningitis, miliary TB and TB of the bones (osteo-articular TB)
Twelve Months Regimen (2RHZ+E/10RH): A 12 months Short Course therapy for a child diagnosed with TB meningitis, Miliary TB, TB
of the bones (osteo-articular TB).

Drug Treatment
The recommended treatment of new-onset PTB is 6 - months of a combination of Rifampicin + Isoniazid, + Pyrazinamide + Ethambutol
for the first 2months then, Only Rifampicin + Isoniazid for the last 4 months.

Recurrent disease: In recurrent TB, testing to determine which antibiotics it is sensitive is important before determining treatment. If
multiple drug-resistant TB (MDR-TB) is detected, treatment with at least 4 effective antibiotics for 18 to 24 months is recommended.

Medication resistance can be 10 or 20. 10 resistance occurs when a person becomes infected with a resistant strain of TB. 20 (acquired)
resistance during therapy because of inadequate treatment, Non compliance, or using low-quality medication.
Drug-resistant TB is a serious public health issue in many developing countries, as its treatment is longer and requires more expensive
drugs.

128
Compiled by Obasi. D. C. Chinedu

Differential Diagnoses: TB should always be considered in the differential diagnosis of a pyrexia of unknown aetiology (or PUO). Other
conditions that may cause PUO include:
Typhoid, Malaria, Urinary tract infections, Bacterial endocarditis, Bronchiectasis
HIV infection and other malignant diseases. Actinomycosis Aspergillosis
Bronchopulmonary Dysplasia, Brucellosis Coccidioidomycosis Failure to Thrive
Histoplasmosis Legionella Infection Meningitis, (Aseptic & Bacterial)
Pneumonia, Pleural Effusion

Complications
 Miliary disease and tubercular (TB) meningitis are the earliest and most deadly complications of primary TB.
 Pulmonary complications: Pleural effusions, Pneumothorax, Atelectasis due to complete obstruction of a bronchus by extruded
caseous material. Bronchiectasis, stenosis of the airways, bronchoesophageal fistula, and endobronchial disease caused by penetration
through an airway wall. Massive Haemoptysis in very rare cases, due to erosion into the pulmonary artery or a Rasmussen's aneurysm.
 Digestive system complications: Perforation of the small bowel, obstruction, enterocutaneous fistula, and the development of severe
malabsorption may complicate TB of the small intestine.
 Cardiovascular system complications: Pericardial effusion can be an acute complication or can resemble chronic constrictive
pericarditis.
 Genito-urinary complications: Hydronephrosis and autonephrectomy usually do not occur in children.
 MSK System: Paraplegia may complicate Pott’s disease of the spine (ie, TB spondylitis).

Prevention: TB prevention and control efforts rely mainly on the vaccination of infants. The only currently available vaccine as of 2011 is
Bacillus Calmette–Guérin (BCG) which is effective against disseminated disease in childhood, but confers inconsistent protection against
contracting pulmonary TB. The detection and appropriate treatment of active cases as well as Public health Education are important
preventive measures.

Prognosis: The prognosis of tuberculosis (TB) varies according to the clinical manifestation. Poor prognosis is associated with
disseminated TB (miliary disease), and TB meningitis. Progression from TB infection to overt TB disease occurs when the bacilli
overcome the immune system defenses and begin to multiply. The risk of reactivation increases with immunosuppression, seen in cases of
co-morbidity with HIV infection usually increasing to 10% per year. The chance of death from a case of tuberculosis is about 4% as of 2008,
down from 8% in 1995.

Follow-up:
 A regular follow-up appointment every 4-8 weeks should be scheduled to ensure compliance and to monitor the adverse effects of and
response to the medications administered.
 Adherence to the regimen is of vital importance to its success, hence the practice of DOTs.
 Monitoring of liver function test results is not indicated routinely but is important in the treatment of patients with miliary TB, TB
meningitis, and coexistence of other hepatic disorders or with concomitant hepatotoxic drug therapy.
 In the event the patient has symptoms of hepatitis, discontinue the regimen and evaluate liver function. If the tests are now normal, then
a decision to restart the medications may be made. Reintroduce the drugs one by one.
 Follow-up CXRs may be performed after 2-3 months of therapy to observe the response to treatment in patients with pulmonary TB.
 However, note that hilar lymphadenopathy may take several years to resolve. Thus, a normal chest radiography finding is not required
for termination of therapy.

Discuss – Absent BCG scar. Implications and further management.

129
Compiled by Obasi. D. C. Chinedu

RESPIRATORY DISORDERS
Dr. Chime Paschal. U;

Learning Objectives
To know the burden, etiology and diagnosis of respiratory tract infections in children
To know how to manage and treat them

Outline
Introduction: Overview of the anatomy of the respiratory system
: Overall burden of respiratory disease in children
Etiology, Pathophysiology, Diagnosis, Evaluation, and Management of:
Rhinosinusitis
Acute pharyngitis and its complications
Acute epiglottitis
Croup
Chronic Obstructive Pulmonary Disease (COPD): Emphysema and Chronic Bronchitis
Conclusion

Overall burden of respiratory disease in children


Preschool children in urban areas have 6-8 episodes of acute respiratory infection (ARI) per year. Repeated clinic visits, hospitalization and
medications & significant missed school days. Pneumonia is a major contributor of under-5 deaths especially in East and South Africa, West
and central Africa, and South Asia. In Nigeria, Viral respiratory infections accounted for up to 65% of admissions in CHERs. Bacterial
respiratory infections notably pneumonia, accounted for more than 30% admission rate in tertiary health facilities in south east Nigeria.
Mortality is rare except for complicated community-acquired pneumonia.

Table 1: Classification of RTIs


Upper respiratory infection Lower respiratory infection

Uncomplicated Complicated Uncomplicated Complicated

Rhinitis Pharyngitis Croup Atelectasis

Bacterial sinusitis Tracheo-bronchitis Pleural effusion

Epiglotitis Bronchiolitis Pneumothorax

Otitis media Pneumonia

Pyogenic adenitis

Pharyngeal abscess

Risk FACTORS FOR ARI


Demographic factors – age and gender. Incidence decreases with increase in age. Males > Females
Host factors –nutrition; immunisation
Socio-economic factors: family income, family size, housing, parental literacy
Environmental factors: poor housing, indoor pollutants, day care attendance, close contact
Geographical factors: ARI tend to occur during the wet season and cold harmattan season.

Pathogenesis of ARIs: Organisms are spread by small-particle aerosols, large-particle aerosols, and direct contact. Host defence
mechanisms are invaded. Viruses are obligate intracellular parasites. They achieve intracellular invasion through their specific properties.
Re-infection in viral RTIs occurs due to lack of protective immunity after an infection. Interplay between the risk factors is important in the
individual’s ability to avert microbial evasion of the resp. tract
UPPER RESPIRATORY TRACT INFECTIONS
A. Rhinosinusitis (Common Cold or Coryza): Usually viral illness. Symptoms include nasal discharge and obstruction. Systemic
symptoms such as myalgia and fever may be absent or mild.

130
Compiled by Obasi. D. C. Chinedu

Common pathogens associated are rhinoviruses; other viruses such as Coronaviruses, Respiratory syncytial viruses, Human
metapneumovirus, Influenza viruses, Parainfluenza viruses, Adenoviruses. Enteroviruses are Uncommon

Sinusitis is common in children and adolescence as Paranasal sinuses do not develop before the age of 3 years. Has potential for serious
complications. There are 2 types of acute sinusitis: viral and bacterial.
Approximately 5–13% of viral upper respiratory tract infections in children are complicated by acute bacterial sinusitis. ***The common
cold produces a viral, self-limiting rhinosinusitis.***

Clinical Presentation: The onset of common cold symptoms typically occurs 1–3 days after viral infection. The 1st symptom is usually
sore or “scratchy” throat, followed closely by nasal obstruction and rhinorrhea. The sore throat usually resolves quickly and, by the 2nd
and 3rd day of illness, nasal symptoms predominate. Examination of the nasal cavity may not be revealing.
Cough is associated with ≈30% of colds and usually begins after the onset of nasal symptoms. The usual cold persists for about 1 wk to 2 wk.

Physical Examination: Increased nasal secretion is frequently obvious to the examiner. A change in the colour or consistency of the
secretions is common during the course of the illness.
Diagnosis is mainly historical

Treatment: TREATMENT IS PURELY SYMPTOMATIC


FEVER – antipyretic treatment is generally not indicated
NASAL OBSTRUCTION – Saline nasal drops; Physical decongestion
Adrenergic agents– (Draw back: attendant side effects CNS stimulation , hypertension, palpitation, rebound congestion etc)
RHINORRHEA: The first-generation antihistamines are useful . Draw back: associated sedation
SORE THROAT- Acetaminophen; Warm beverages – soothing effect plus extra fluid needed
ASSOCIATED COUGH: Cough suppression is not recommended; antihistamines may be helpful but cough mixtures are not recommended
FLUIDS AND FEEDS should be increased
INEFFECTIVE TREATMENTS: warm humidified air, zinc based lozenges, herbal treatments.

B. Bacterial Sinusitis: Sinusitis is common in children and adolescence. Has potential for serious complications.
There are 2 types of acute sinusitis: viral and bacterial. Viral Rhinosinusitis (common cold) occur more commonly in children.
Approximately 5–13% of viral upper respiratory tract infections in children are complicated by acute bacterial sinusitis.
ACUTE BACTERIAL SINUSITIS: typically follows a viral upper respiratory tract infection. Inflammation and oedema block sinus
drainage and impair muco-ciliary clearance of bacteria.

Causes of Acute bacterial sinusitis:


- Streptococcus pneumoniae (≈30%) - Non typable Haemophilus influenzae (≈20%)
- Moraxella catarrhalis (≈20%)
*About 25% of S. pneumoniae may be penicillin resistant*
Staphylococcus aureus, other streptococci, and anaerobes are uncommon causes in children.

Chronic sinus disease: H. influenzae, α- and β-hemolytic streptococci, M. catarrhalis, S. pneumoniae, and coagulase-negative staphylococci
are implicated.
Historical Information: Persistent nasal congestion, purulent nasal discharge (unilateral or bilateral) following acute URI lasting more than
10 days without improvement. Worsening or new onset fever, day time cough or nasal discharge after an initial improvement
Less commonly: Bad breath (halitosis), Decreased sense of smell, and Periorbital swelling.
Complaints of headache and facial pain are uncommon (more in adolescents). Maxillary tooth discomfort, pain or pressure exacerbated by
bending forward.
Findings on Physical examination: Erythema and swelling of the nasal mucosa with purulent nasal discharge. Periorbital oedema. Sinus
tenderness may be detectable in adolescents. Trans-illumination reveals an opaque sinus.

Diagnosis: Clinical diagnosis of acute bacterial sinusitis is based solely on history. Severe respiratory symptoms with fever of at least 39°C
and purulent nasal discharge for 3–4 consecutive days suggest possible complicated acute bacterial sinusitis.
Sinus aspirate culture- only accurate method of diagnosis but routine use in children not practical. Bacteria are recovered from maxillary
sinus aspirates in 70%. Transillumination of the sinus cavities may demonstrate the presence of fluid.
Radiographic findings (sinus plain films, CT scans): opacification, mucosal thickening, or presence of an air-fluid level are suggestive.

Important differential diagnostic considerations: viral upper respiratory tract infection, allergic rhinitis, and non-allergic rhinitis. Nasal
foreign body.

Treatment: Treatment is recommended to prevent suppurative complications. Initial therapy with standard dose amoxicillin (45mg/kg/day
in 2 divided doses) is adequate for uncomplicated acute bacterial sinusitis or High dose amoxicillin (80-90mg/kg/day) with clavulanate
(6.4mg/kg/day) for resistant cases, and failure to respond within 72 hours of initiation of therapy. Treatment should be continued for up to 7
days following resolution of symptoms. Intavenous ceftriaxone at 50mg/kg/day is a good alternative.

131
Compiled by Obasi. D. C. Chinedu

Nasal decongestants, antihistamines, mucolytics, are useful in children with allergic rhinitis who also have acute sinusitis. Intranasal
corticosteroids have been found beneficial in some RCTs involving children.
Frontal sinusitis can rapidly progress to serious intracranial complications and necessitates initiation of parenteral antibiotics

Complications of Sinusitis:
Orbital complications: periorbital cellulitis and intra-orbital cellulitis. Evaluation: CT scan of the orbits and sinuses with ophthalmology
and otolaryngology consultations.

Intracranial complications: epidural abscess, cavernous sinus thrombosis, subdural empyema, brain abscess, and meningitis. Children with
altered mental status, nuchal rigidity, or signs of increased intracranial pressure (headache, vomiting) require urgent CT scan of the brain,
orbits, and sinuses. Treatment with broad-spectrum intravenous antibiotics should be initiated immediately, pending culture and
susceptibility results.
Choices of drugs: vancomycin with either ceftriaxone or ampicillin- sulbactam to cover for MRSAs or penicillin- resistant S.pneumoniae.
Metronidazole should be considered in intraorbital and should be given in all cases of intracranial abscess. Surgical drainage of the ethmoidal
sinuses and brain abscesses may be required.
Other complications: Pott puffy tumour, Mucoceles.
Pott puffy tumour is a non-neoplastic complication of acute sinusitis. It is characterized by a primarily subgaleal collection, subperiosteal
abscess and osteomyelitis. Usually related to the frontal sinus

C. Acute Bacterial Pharyngitis: Pharyngitis involves Inflammation of structures of the pharynx. Pharyngitis and tonsillitis may occur
together as pharyngotonsilitis. Most are viral; However, streptococcal pharyngitis is of clinical importance. Major Bacterial cause: Group A
β-haemolytic streptococcus (GABHS). Other organisms include- Group C streptococcus, Mycoplasma pneumoniae, Neisseria gonorrhoeae,
Corynebacterium diphtheriae.

Pathogenesis: acute bacterial sinusitis typically follows a viral upper respiratory tract infection. Inflammation and edema block sinus
drainage and impair muco-ciliary clearance of bacteria.
STREPTOCOCCAL PHARYNGITIS: Colonization of the pharynx by GABHS can result in either asymptomatic carriage or acute
infection. The M protein is the major virulence factor of GABHS and facilitates resistance to phagocytosis by polymorphonuclear
neutrophils. GABHS Serotypes implicated: 1, 3, 5, 6, 14, 18, 19, and 24. Specific immunity develops during infection and provides
protective immunity to subsequent infection with that particular M serotype

Clinical Manifestation: The incubation period is 2–5 days; Usually present as Pharygotonsillitis. Onset is often rapid with prominent sore
throat and fever. Headache and GI symptoms (abdominal pain, vomiting) are frequent.
Pharynx is hyperaemic; tonsils may be enlarged, hyperaemic ± exudates.
There may be petechiae on the soft palate and posterior pharynx, uvula may be red, stippled, and swollen.
The anterior cervical lymph nodes are enlarged and tender.

Diagnosis: The clinical presentations of streptococcal and viral pharyngitis show some overlap. Throat swab culture remains an imperfect
gold standard for diagnosing streptococcal pharyngitis. False-negative cultures are attributed to a variety of causes (inadequate throat
swab specimen and surreptitious use of antibiotics).
Rapid diagnostic tests (RDTs) detect group A streptococcal antigen with a high specificity. RDTs are generally less sensitive than culture;
confirmation of a negative rapid test with a throat culture is recommended where there is high index of suspicion.

Treatment: Most untreated episodes of streptococcal pharyngitis resolve uneventfully in a few days. The primary benefit of treatment is
the prevention of acute rheumatic fever. Antibiotic treatment has no effect on the development of acute glomerulonephritis.
GABHS remains susceptible to penicillin. Penicillin V is inexpensive; given bid or tid for 10 days: 250 mg/dose for children and 500
mg/dose for adolescents. A single intramuscular dose of benzathine penicillin (600,000 U for children <27 kg [60 lb]; 1.2 million U for
children weighing more than 27kg)
Other effective antibiotics include amoxicillin, cephalosporins , and macrolides- Erythromycin ; Azithromycin gives the benefit of shorter
duration of therapy and convenience (once daily)

Nonspecific/symptomatic treatment
Oral antipyretic/analgesic agent (acetaminophen or ibuprofen) may relieve fever and throat pain.
Gargling with warm salt water (soothing effect); anaesthetic sprays and lozenges (often containing benzocaine, phenol, or menthol) may
provide local relief.

Complications
SUPPURATIVE
- Retropharyngeal abscess - Peritonsillar abscess - Cervical adenitis
- Otitis media - Sinusitis - Mastoiditis
- Bacteraemia and metastatic infection
132
Compiled by Obasi. D. C. Chinedu

NON SUPPURATIVE
- Acute rheumatic fever - Acute glomerulonephritis
- Reactive arthritis - Toxic shock syndrome

Retropharyngeal Abscess: Not common. Commonly occurs in <2 year olds. Breathing is rapid, unresolving fever, drooling saliva, mouth
open. Neck is extended and stiff, head bent backwards
Treatment: Intravenous antibiotics. Urgent ENT consult to drain the abscess to avoid rupture and aspiration into the lungs.

Acute Otitis Media (AOM): Acute ear infection occurs in up to 30% of URIs. Peak incidence seen in children 6 to 20 months of age.
Bacterial Pathogens most frequently found in AOM include: Streptococcus pneumoniae: up to 60% of AOM in children < 1 year. Non
typable H. influenza, Moraxella catarrhalis.
Others pathogens include: Group A Streptococcus, Staph aureus and gram negative organism such as Pseudomonas aeruginosa

Diagnosis: Clinical diagnosis depends on:


History: ear pain, ear tugging, fever
Systematic examination of the ear for tenderness at the tragus, colour, transparency and mobility of the tympanic membrane (TM).
Diagnosis is marked by: Recent, usually abrupt onset of symptoms and signs
Findings best visualized with pneumatic otoscope showing: Presence of middle ear effusion indicated by at least one of the following:
Bulging of the tympanic membrane or Air/fluid level behind the TM or TM colour (whitish yellow) and opaque or Otorrhea or absent or
limited mobility of the TM.
And middle ear inflammation as indicated by at least one of the following: Marked or diffuse erythema of the TM or Otalgia (discomfort
clearly referable to the ear interfering with or precluding normal activity or sleep).
Treatment: Penicillin is recommended. Amoxicillin-clavulanate has a broader coverage than amoxicillin and may be a better initial
antibiotic. Definite therapy should be based on culture results.
Complications: perforation of eardrums, chronic ear discharge and hearing impairment or deafness later in childhood. Chronic ear infection
following repeated episodes of AOM may lead to mastoiditis, meningitis, cerebral abscess.

Peritonsillar Abscess (Quincy): This is a complication of tonsillopharyngitis following the spread of infection and abscess formation
beyond the tonsillar capsule to the contiguous peritonsillar tissue. Common in the late school-aged child and adolescents. GABHS and
several mouth anaerobes are associated with quincy.
The typical clinical scenario is that of a child /adolescent just recovering from a bout of tonsillopharyngitis…… who suddenly develop
unilateral throat pain, high fever, trismus and difficulty with swallowing.

Physical findings include: Toxicity and pyrexia. A visible asymmetrical bulge over the tonsillar area. The uvula may be displaced to the
contralateral side.
Treatment: Empirically chosen antimicrobial agents effective against GABHS and common oral anaerobes. Then diagnostic/therapeutic
aspiration. Incision and drainage cum tonsillectomy.
Spontaneous rupture and aspiration of the abscess may result in aspiration pneumonia.

Acute Epiglottitis: Inflammation of the epiglottis usually is caused by H. influenza type b. Affects school aged children. Incidence now less
common due to increase immunization coverage against H. influenza. Causes significant toxicity with high fever, sore throat, drooling of
saliva, refusal of feeds, stridor, and difficulty in breathing.
Salient examination findings: Child is usually anxious, toxic, sitting upright in a tripod position with hyperextended neck and open mouth.
Throat visualization quite hazardous; preferable in a theatre setting (characteristic cherry-red and swollen epiglottis). Lateral cervical
radiography is informative shows thump print appearance suggesting grossly swollen epiglottis.
May confirm Hib bacteremia on blood culture (yield is low). FBC (leucocytosis) plus ESR- usually raised.
NB: phlebotomy may trigger laryngeal spasm. Its better to secure airway first with either nasotracheal intubation or tracheostomy.
Recommended antibiotic treatment: penicillin (ampicillin -sulbactam), or cephalosporins pending culture results.
High case fatality up to 12% in hospitalized patients

133
Compiled by Obasi. D. C. Chinedu

LOWER RESPIRATORY TRACT INFECTIONS


CROUP (Laryngotracheobronchitis):
ETIOLOGY AND EPIDEMIOLOGY: The parainfluenza viruses (types 1, 2 and 3) account for ≈75% of cases. Other viruses include
influenza A and B, adenovirus, respiratory syncytial virus (RSV) and measles. Most patients with croup are between the ages of 3 mo and 5
yr, with the peak in the 2nd year of life. Incidence is higher in males;
Recurrences are frequent from 3–6 yr of age and decrease with growth of the airway.

Clinical Manifestations: Combination of rhinorrhea, pharyngitis, mild cough, and low-grade fever for 1–3 days before the signs and
symptoms of upper airway obstruction become apparent. The characteristic “barking” cough, hoarseness, and inspiratory stridor
subsequently develop. Symptoms are characteristically worse at night and often recur with decreasing intensity for several days and resolve
completely within a week. The child may prefer to sit up in bed or be held upright. Usually progress to stridor and slight dyspnea before
recovery

PHYSICAL EXAMINATION may reveal a hoarse voice, coryza, evidence of inflammation of pharynx
Respiratory distress varies substantially between patients. Hypoxia and low oxygen saturation are seen only when complete airway
obstruction is imminent.

Diagnosis:
Croup is a clinical diagnosis and may not require a radiograph of the neck. Radiographs of the neck may show the typical “steeple sign” or
“wine bottle sign”. The radiographs do not correlate well with disease severity. Radiographs should be considered only after airway
stabilization. Radiographs may be helpful in distinguishing between severe laryngotracheobronchitis and epiglottitis, but airway management
should always take priority.
Treatment: Mainstay of treatment for children with croup is airway management. There is no evidence to support the effectiveness of mist
therapy. Nebulized epinephrine is an accepted treatment for moderate or severe croup.
**mechanism of action --constriction of the precapillary arterioles, causing fluid resorption from the interstitial space and a decrease in the
laryngeal mucosal edema**. Epinephrine at a dose of 0.5ml (of 1:1000) diluted. Single dose of 0.6mg/kg dexamethasone IM, Orally.
Antibiotics are not indicated in croup.

Chronic Obstructive Pulmonary Disease (COPD): Characterized by presence of airflow obstruction. Caused by emphysema or chronic
bronchitis. Generally progressive. May be accompanied by airway hyperreactivity. May be partially reversible.

Emphysema: Abnormal permanent enlargement of the airspace distal to the terminal bronchioles. Accompanied by destruction of
bronchioles.

Chronic Bronchitis: Presence of chronic productive cough for 3 or more months in each of 2 successive years in a patient whom other
causes of chronic cough have been excluded

COPD: causes
Cigarette smoking: Nicotine stimulates sympathetic nervous system resulting in:
-increased HR
-peripheral vasoconstriction
-increased BP and cardiac workload
 Decrease ciliary activity, Possible loss of ciliated cells.
 Abnormal dilation of the distal air space
 Alveolar wall destruction
 Carbon monoxide: reduces oxygen carrying capacity
 Cellular hyperplasia: production of mucus, reduction in airway diameter and increased difficulty in clearing secretion
Secondhand smoke exposure associated with: reduced pulmonary function
Infection: Major contributing factor to the aggravation and progression of COPD
Hereditary: α-Antitrypsin (AAT) deficiency (produced by liver and found in lungs; accounts for <1% of COPD cases.

Emphysema: clinical manifestations


Minimal coughing with no to small amounts of sputum. Overdistension of alveoli causes diaphragm to flatten and AP diameter to increase.
Patient becomes chest breather, relying on accessory muscles. Patient is underweight despite adequate calorie intake

Chronic bronchitis: clinical manifestations


Earliest symptoms: Frequent, productive cough during cold wheather, Frequent respiratory infections.
Others: Bronchospasm at end of paroxysms of coughing, Cough, Dyspnea on exertion, Normal weight.
Bluish-red appearance: polycythaemia and cyanosis
Hypoxia and hypercapnia: results from hypoventilation and increased airway resistance

134
Compiled by Obasi. D. C. Chinedu

Pathophysiology of Chronic Bronchitis and Emphysema

COPD: complications
Pulmonary hypertension: pulmonary vessel constriction
Cor pulmonale: Right heart hypertrophy with or without RV failure
Pneumonia
Acute respiratory failure

COPD: diagnostic studies


Chest x-ray early in the disease may not show abnormalities. History and physical exam
Pulmonary function studies: reduced FEV1/FVC and increase residual volume and total lung capacity
ABGs: Decreased PaO2, Increased PaCO2, Decreased pH, Increase bicarbonate level found in late stages of COPD

COPD: management
Smoking cessation: Most significant factor in slowing the progression of the disease
Bronchodilators: as maintenance therapy viz Β-adrenergic agonists: MDI or nebulizer preferred, Anticholinergics.
Oxygen therapy: Raises PO2 in inspired air, Treats hypoxia, Titrate to lowest effective dose.
Chest physiotherapy: to bring secretions into larger, more central airways. Postural drainage, Percussion, Vibration
Surgical therapy: Lung transplant
Nutritional therapy: Full stomachs press on diaphragm causing dyspnea and discomfort. Use bronchodilator before meals. 5-6 small
meals to avoid bloating.
Avoid gas-forming foods. High-calorie, high-protein diet is recommended. Supplements
Health promotion: Stop smoking!!!!, Avoid environmental pollutants and irritants. Early detection of small-airway disease. Early diagnosis
of respiratory tract infections.
Home care: Nasal saline sprays, Decongestants, Nasal steroid inhalers. Long-acting theophylline (decreases bronchospasm and airway
obstruction).

135
Compiled by Obasi. D. C. Chinedu

OVERVIEW OF INHERITED DISORDERS AND COMMON CHROMOSOMAL ANOMALIES


Prof Herbert A Obu, MBBS, FWACP (Paed)

Outline
 Introduction
 The human genome
 Genetic (inherited) disorders
 Major categories of genetic disorders
 Identifying human genetic disorders
 Treatment of genetic disorders
 Common chromosomal abnormalities
 Inherited metabolic disorders (Inborn errors of metabolism (IEMs))

Introduction:
Inheritance: characters or qualities transmitted from parent to offspring by encoded cytologic data. Offsprings receive a mixture of genetic
information from both parents. This process contributes to the great variation of traits seen in nature. Genetics is the science of heredity and
variation in living things and seeks to understand the process of inheritance.
Genetics is important in appearance and behaviour of organisms but an interplay with the environment determines the ultimate outcome.
Genes are the basic unit of inheritance. Genetic information is encoded and transmitted from generation to generation in DNA. DNA is a
coiled molecule organized into structures called chromosomes within cells. Segments along the length of a DNA molecule form genes.
Genes direct the synthesis of proteins, the molecular laborers that carry out all life-supporting activities in the cell. The structure and function
of human beings is determined by about 30, 000 genes.

Human genome: Human genes reside on 23 pairs of chromosomes found in the nucleus of every body cell except gamete cells. In each pair,
one of the chromosomes is inherited from the mother and the other is passed down from the father. All but one of these 23 pairs are
composed of chromosomes nearly identical in shape. Each of these 22 chromosome pairs, known as autosomes, contains the same genes
(although they likely carry different alleles). The 23rd pair of chromosomes are the sex chromosomes and determine the sex of an individual.
Females inherit two X chromosomes, a matched pair carrying the same genes; one from the mother and the other from the father. Males
inherit an X chromosome from their mother and a Y from the father. Humans produce gamete cells for sexual reproduction. These gametes
contain a haploid number of chromosomes—23 chromosomes instead of the full complement of 46. During fertilization egg and sperm join
with restoration of the diploid number of chromosomes

Genetic (inherited) disorders: For human beings, indeed all organisms, to grow and function properly, cells must constantly divide to
produce new ones which will replace old, worn-out cells. During cell division, it is essential that DNA remains intact and evenly distributed
among the cells. It is equally important that reproductive cells such as eggs and sperm contain the right number of chromosomes and that
these chromosomes have correct structure.
Problems occurring during these processes lead to a variety of genetic disorders.
Thousands of inherited diseases caused by altered genes and chromosomal abnormalities affect humans. These disorders cause problems
such as physical deformities, metabolic dysfunction, and developmental problems.
As many as 1 baby in 200 is born with a chromosomal abnormality serious enough to produce physical defects or mental retardation. Eg:
cystic fibrosis (CF)- autosomal recessive disorder, Huntington’s disease- autosomal dominant disorder, Hemophilia – x-linked recessive
disorder, trisomies 21, 18 and 13, etc.

Major categories of genetic disorders: Genetically determined diseases result from one of the following:
1. Chromosomal abnormalities
2. The action of a single gene (Mendelian disorders)
3. Unusual genetic mechanisms
4. Interaction of genetic and environmental factors (multifactorial or polygenic disorders)

Chromosomal abnormalities: Chromosomal abnormalities are either numerical or structural; occur in approx. 25% of spermatozoa and
10% of mature oocytes. Estimated incidence in live-born infants is about 1 in 150; often cause multiple congenital anomalies and learning
difficulties. Acquired chromosomal disorders are involved in carcinogenesis and tumour progression.
Common chromosomal abnormalities include Down’s syndrome (trisomy 21), Edward’s syndrome (trisomy 18), Patau’s syndrome
(trisomy 13), Turner’s syndrome (45 XO), Klinefelter’s syndrome (47, XXY), Cri du chat syndrome (5 p-), Beckwith Weideman
syndrome (duplication of a portion of the chromosome resulting in extra genetic material), Prader Willi syndrome (microdeletion of
paternal p15), Angelman syndrome (microdeletion of maternal p15), etc.

Mendelian inheritance (single gene disorders): Over 15000 single gene traits or disorders of this nature have been described. Mendelian
pattern of inheritance is known for many of these disorders. When diagnosis is uncertain, pedigree analysis (family tree) may help in
resolving the pattern of inheritance.

Mendelian (single gene) disorders may be:

136
Compiled by Obasi. D. C. Chinedu

Autosomal dominant: most common form of Mendelian inheritance. Affected individuals carry the abnormal gene on one of a pair of
autosomes. 1 in 2 chance of inheriting the abnormal gene from affected parent. Phenotype may be affected by variation in expression, non-
penetrance, no family history (new mutation, gonadal mosaicism, non-paternity) or homozygosity (rare). Examples: Achondroplasia, Ehlers-
Danlos syndrome, Familial hypercholesterolaemia, Marfan’s syndrome, Neurofibromatosis, Tuberous sclerosis, Osteogenesis imperfecta,
etc.

Autosomal recessive: Affected individuals are homozygous for the abnormal gene; each parent is a heterozygous carrier. 1 in 4 chance of
having an affected child for 2 carrier parents. Risk of these disorders increased by consanguinity and within specific racial groups. Often
affect metabolic pathways. Examples: Sickle cell anaemia, Tay-Sachs disease, Thalasaemia, Werdnig-Hoffmann disease, Phenyketonuria,
Oculocutaneous albinism, Hurler’s syndrome, Gycogen storage diseases, Congenital adrenal hyperplasia, Cystic fibrosis, Friederich’s ataxia,
Galactosaemia.

X - linked recessive disorders: Over 400 disorders of this nature described. Abnormal recessive gene is carried on the X chromosome.
Males affected; females are carriers but are usually healthy or have mild disease. Family history may be negative (new mutations or
mosaicism). Identifying female carriers is important in order to be able to offer genetic counseling. Examples of X–linked recessive
disorders: G6PD Deficiency, Haemophilia A and B, Hunter’s syndrome, (mucopolysaccharidosis II), Fragile-X syndrome, Colour blindness,
Dutchenne’s muscular dystrophy, Becker’s muscular dystrophy.

X-linked and Y-linked dominant disorders: X-linked dominant and Y-linked traits are rare. An example of X-linked dominant disorder is
incontinentia pigmenti.

Polygenic or multifactorial inheritance: Whereas some diseases arise from purely genetic or environmental causes, many disorders result
from the additive effect of several genes with or without the influence of the environment or other unknown factors, hence the terms
polygenic or multifactorial. Normal traits such as height and intelligence are also inherited in this fashion. These parameters show a Gaussian
or normal distribution in the environment. Similarly, the liability of an individual to develop a disease of polygenic or multifactorial etiology
also has a normal distribution. Examples of conditions with multifactorial inheritance: Diabetes mellitus, Asthma, Epilepsy, Hypertension,
Atherosclerosis and coronary heart disease, Hypospadias, Talipes, Congenital dislocation of the hip, Pyloric stenosis. Cleft lip and palate.
Neural tube defects.

Identifying human genetic disorders:


Analysis of a family medical history (pedigree analysis) is used to track the transmission of a condition through generations. Blood tests that
identify specific DNA sequences can reveal carriers of a disease-causing gene who have no symptoms of the disease. Pre-
symptomatic testing can identify DNA abnormalities in a person before health problems develop. In the case of certain inherited heart
conditions, for example, these tests enable a person to make healthy lifestyle changes or take other preventive measures, such as medications,
to lower the risk of illness or death.

DNA Analysis: Many single gene disorders are now amenable to molecular analysis. Most molecular analysis are done using polymerase
chain reaction (PCR). PCR involves the amplification of specific DNA sequences, enabling rapid analysis of small samples, and is
particularly important in ante natal diagnosis.
Main impact of DNA analysis:
 Confirmation of clinical diagnosis
 Detection of female carriers in X-linked disorders
 Carrier detection in autosomal recessive dieorders
 Pre-symptomatic diagnosis in autosomal dominant disorders
 Antenatal diagnosis of an increasing number of Mendelian conditions.

Treatment of genetic disorders: Treatment approaches in genetic disorders include the following:
- health surveillance - supportive measures
- medical therapy - surgical procedures
- dietary manipulations - replacement of deficient gene products or enzymes
- bone marrow transplantation

Gene therapy: Gene therapy involves the repair, suppression or artificial introduction of genes into genetically abnormal cells with a view to
curing the disease. Still at an experimental stage; many technical and safety issues yet to be resolved. Has been initiated in malignant
melanoma and cystic fibrosis with clinical benefit noted in a few. At present it is generally accepted that gene therapy should be restricted to
somatic (not germ line) cells so as to minimize the risk of adverse outcomes for future generations.

Common chromosomal abnormalities


DOWN’S SYNDROME (Trisomy 21): Most common and best known chromosomal disorder in humans. Also most common cause of
intellectual disability. Incidence in live-born infants is about 1 in 650; 75% of conceptions with down syndrome abort spontaneously.
137
Compiled by Obasi. D. C. Chinedu

May result from non-disjunction (94%), translocation (5%) or mosaicism (1%).


Occurrence is strongly associated with maternal age; 1/15,000 births in women aged 15-29 years and 1/50 live births in those older than 45
years. Women who have Down syndrome have a 50% chance of having a child with the disorder. Many affected fetuses abort spontaneously.
Men with Down syndrome are usually infertile.

Clinical features: Down syndrome is characterized by structural and functional defects, viz:
- Intellectual disability - Dysmorphic facial features - Congenital anomalies
Other distinctive phenotypic traits (Note that wide phenotypic variation occurs; not all defects seen in every patient.)
- Round face & flat nasal bridge - Upslanted palpebral fissures - Prominent epicanthic folds
- small mouth & protruding tongue - low set ears/small ears - flat occiput & third fontanel
- Brushfield (pigmented) spots in iris - short neck - single palmer crease
- incurved fifth finger - wide “sandal” gap between the big and second toes
- Hypotonia - Congenital heart defects - Duodenal atresia & hirschprung’s disease

Down syndrome: later medical problems includes; delayed motor milestones, learning difficulties; small stature, increased susceptibility
to infections, hearing impairment from secondary otitis media, visual impairments from cataracts, squints, myopia. Increased risk of
leukaemia and solid tumours, hypothyroidism and coeliac disease, epilepsy and alzheimer’s disease.

Down syndrome: diagnosis: Mainly clinical but cytogenetic chromosomal studies needed to confirm diagnosis. Fluorescent in situ
hybridization (FISH) can be used for rapid diagnosis. Karyotyping essential to determine recurrence risk. Antenatal screening tests available
in adv. societies for foetal diagnosis: low maternal serum alpha-feto protein (MSAFP), low unconjugated estriol (eU3), and elevated
human chorionic gonadotropin (hCG) are markers of Down syndrome.

Down syndrome: management: Counseling (including genetic counseling) and Medical/ surgical treatment of associated conditions.

Down syndrome: prognosis: Varies from person to person. Over 80% of affected infants live up to one year of age; severe congenital heart
defects (A-V SEPT. defect) major cause of early mortality. At least 50% of affected individuals live > 50 years.

Edwards syndrome (trisomy 18): Among live-born children, trisomy 18 is the second most common autosomal trisomy after trisomy 21.
Incidence is 1 in 8000 live births. Most babies IUGR & dysmorphic.
Characterized by severe psychomotor and growth retardation, microcephaly, microphthalmia, malformed ears, micrognathia or
retrognathia, microstomia, distinctively clenched fingers, and other congenital malformations.

Clinical features: prominent occiput, narrow forehead, small mouth and jaw, short sternum, clenched hands with over-lapping digits,
prominent heels and rocker-bottom feet.
Major malformations: heart defects, neural tube defects, omphalocoele, oesophageal atresia & radial defects.

Most children with Edwards syndrome die shortly after birth. Diagnosis is confirmed by chromosome analysis. Many affected fetuses are
detected by ultrasound in second trimester and diagnosis can be confirmed by amniocentesis and chromosome analysis.

Patau’s syndrome (Trisomy 13): Incidence: 1 in 14000. Dysmorphic features: scalp defects, broad nasal tip and polydactyly.
Major malformations: holoprosencephaly, microcephaly, ocular malformations, cleft lip and palate, heart defects and renal abnormalities.
Most affected babies die within one month.

Turner’s syndrome (45, XO): Incidence in live-born females is about 1 in 2500


Clinical features: Lymphoedema of hands and feet in neonate, Spooned shaped nails, Short stature, Webbing of the neck or thick neck, Wide
carrying angle (cubitus valgus), Widely spaced nipples, Congenital heart defects (aortic coarctation), Delayed puberty, Ovarian dysgenesis
resulting in infertility, Hypothyroidism, Renal anomalies, Pigmented moles, Recurrent otitis media, Normal intellectual function in most.

Antenatal diagnosis: Ultrasound may detect foetal oedema of the neck, hands or feet or cystic hygroma.
Treatment: Growth hormone therapy. Oestrogen replacement therapy for the development of secondary sexual xtics at time of puberty.
Incidence does not increase with maternal age and risk of recurrence is very low.

Klinefelter’s syndrome (47, XXY): Occurs in about 1-2 per 1000 live-born males.
Clinical features: Infertility, Hypogonadism with small testes, Tall stature, Gynaecomastia in adolescence.
Pubertal development apparently normal but some may require hormonal therapy. Intelligence is usually in the normal range but may have
educational and psychological problems.

CRI DU CHAT SYNDROME (5 P-): Deletion of the tip of the short arm of chromosome 5, hence the name 5 p- or monosomy 5. Affected
babies have high-pitched mewing cry in early infancy. Affected babies succumb within a few days of delivery.

Inherited Metabolic Disorders (Inborn Errors Of Metabolism/Congenital Metabolic Diseases)


A heterogeneous group of rare genetic diseases that generally result from a defect in an enzyme or transport protein which results in a block
138
Compiled by Obasi. D. C. Chinedu

in a metabolic pathway (involving the carbohydrates, proteins, lipids or other complex molecules). Effects are due to toxic accumulations
of substrates before the block, intermediates from alternative metabolic pathways, defects in energy production and use caused by a
deficiency of products beyond the block, or a combination of these metabolic deviations. Often the central nervous system (CNS) is affected,
leading to neurological disease.
Incidence, collectively, is estimated to be as high as 1 in 800 live births, but this varies greatly between populations. Phenylketonuria (PKU)
and medium-chain acyl-CoA dehydrogenase (MCAD) deficiency with respective incidences of 1 in 10,000 and 1 in 20,000 are among the
most prevalent.
Presentation is usually in the neonatal period or infancy but can occur at any time, even in adulthood. A high index of suspicion is most
important in making the diagnosis.

IEM: categories
Disorders that result in toxic accumulation: Disorders of protein metabolism (eg, amino acidopathies, organic acidopathies, urea cycle
defects); disorders of carbohydrate intolerance; lysosomal storage disorders.
Disorders of energy production, utilization: Fatty acid oxidation defects; disorders of carbohydrate utilization, production (ie, glycogen
storage disorders, disorders of gluconeogenesis and glycogenolysis); mitochondrial disorders; peroxisomal disorders

IEM aetiology: Caused by mutations in genes coding for proteins that function in metabolism. Most of the disorders are inherited as
autosomal recessive, whereas autosomal dominant and X-linked disorders are also present.
Initially thought to be caused by single-gene mutations, but environmental, epigenetic, and microbiome factors as well as additional genes
are potential modifying etiologic factors in individual cases.

IEM clinical presentation: In the neonatal period: IEMs should be considered in all critically ill neonates. Features:
- Neurologic abnormalities (seizures, encephalopathy, abnormalities of tone)
- Disorders of acid-base status (metabolic acidosis, respiratory alkalosis, hyperammonemia syndromes)
- Hypoglycemia - Liver dysfunction - Dysmorphic features - Ambiguous genitalia
- Cardiomyopathy - Abnormal body and urine odours

Infants and older children may present with: history of recurrent episodes of vomiting, ataxia, seizures, lethargy, coma. Also fulminant
hepatoencephalopathy. Poor feeding/failure to thrive, Fussiness/decreased activity, developmental delay, sometimes with loss of milestones.
Undiagnosed metabolic disease should be considered in older children, adolescents, or even adults with subtle neurologic or psychiatric
abnormalities.

IEM: diagnostic evaluation:


- FBC: neutropenia, anemia, and thrombocytopenia, etc.
- Serum electrolytes, urea and creatinine (including HCO3 and blood gases).
- Liver function tests (+ PT and activated PTT).
- Lactate dehydrogenase, aldolase, creatinine kinase, and urine myoglobin levels.
- Ammonia levels
- Blood glucose, urine pH, ketones and reducing substances to evaluate for hypoglycemia.
- Plasma quantitative amino acids
- Urine organic acids, acylglycine, and/or orotic acid (5-10 mL, freeze immediately).
- Serum lactate and pyruvate levels
- Cerebrospinal fluid (CSF) lactate, pyruvate, organic acids, neurotransmitters, and/or disease-specific metabolites
- EEG, nerve conduction studies, evoked potential studies, and/or EMG

IEM: treatment: Treatment is targeted at


prevention of further accumulation of harmful substances,
correction of metabolic abnormalities,
elimination of toxic metabolites.

Principles of Tx:
Emergency resuscitation (ABC of resuscitation, hypoglycaemia, adequate hydration, etc).
Discontinue intake of offending agent and ensure adequate glucose to prevent catabolism.
Dialysis for removal of toxic substances
Administration of deficient metabolite and/or enzyme/co-factor or co-enzyme when possible
Others: bone marrow transportation, liver transportation, gene therapy

Carbohydrate metabolic defects: Autosomal recessive inheritance. 3 main types:


a) Galactosaemia
b) Glycogen storage disorders, type I (von Gierke’s) & type II (Pompe’s)
c) Congenital lactase deficiency.

139
Compiled by Obasi. D. C. Chinedu

d) Mucopolysaccharidoses

Galactosaemia: Hereditary galactosemia is among the most common carbohydrate metabolism disorders and can be a life-threatening in the
newborn period. Galactose-1-phosphate uridyltransferase (GALT) deficiency is the most common enzyme deficiency that causes the
disorder. Lactose in food (eg. diary products) is broken down by lactase into glucose and galactose. In galactosemia, further metabolism of
galactose is impaired leading to toxic levels of galactose-1-phosphate in various tissues
Clinical features in severe forms include: Poor growth, persistent jaundice, hypoglycaemia, vomiting, hepatomegaly, cirrhosis, liver failure,
cataracts, bleeding from coagulopathy, ascites, seizures, sepsis (esp. E. coli)
Unless treated, most patients with this condition die in the newborn period.
Diagnosis: reducing substances in urine, albuminuria, confirmatory GALT analysis in RBCs via biochemical genetic screening
Treatment: Avoidance of lactose and galactose containing feeds. Should not be breastfed; fed on soy based formula.

Phenylketonuria(PKU) (example of aminoacidopathic disorders): Autosomal recessive inheritance. Xs dietary phenylalanine is normally
converted to tyrosine by phenylalanine hydroxylase. Deficiency of this enzyme results in xs accumulation of phenylalanine, especially in
the brain, & xs formation of phenylketones which are excreted in urine. Patients are healthy at birth, but if untreated, symptoms appear at 6-
12 months with infantile spasms, intellectual disability and a mousy odour (sweat & urine)
Tx: restriction of phenylalanine in diet.

Familial hypercholesterolaemia: An autosomal dominant disorder of lipoprotein metabolism. About 1/500 of the population affected. LDL
is markedly elevated. Associated with premature coronary heart disease and skin and tendon xanthomata may be present.

Conclusion:
I have taken you through introduction to basic genetics, the human genome, genetic disorders (major categories, identification and treatment),
common chromosomal abnormalities and some inherited metabolic disorders (inborn errors of metabolism).

140
Compiled by Obasi. D. C. Chinedu

NEPHROTIC SYNDROME
BY DR MBANEFO N R

Objectives
1. Have a good understanding of the clinical entity
2. Identify a child with such syndrome
3. Have a knowledge of the requisite investigations for correct diagnosis
4. Aware of the treatment modalities

Outline
 Definition - Aetiology - Epidemiology - Pathophysiology
 Clinical manifestation - Investigations & Diagnosis - Treatment
 Complications

Definition
Nephrotic syndrome(NS) is a clinical condition characterized by massive proteinuria, hypoalbuminaemia and generalized oedema with
or without hypercholesterolaemia. The disease occurs due to alterations at the glomerular wall resulting in its inability to restrict the
urinary loss of protein. Presently, NS is recognized globally as a common chronic kidney disease in childhood.

Definition of terms:
Nephrotic range proteinuria is defined as proteinuria exceeding: 40mg/m2/hr or 1000mg/m2/day or spot urinary protein to creatinine ratio
exceeding 2mg/mg. Hypoalbuminaemia is defined as a serum albumin less than 25g/L.
Hypercholesterolemia is defined as serum cholesterol level greater than 230mg/dl or 5.2mmol/L
Generalized oedema is an excessive accumulation of fluid in the interstitial space throughout the body.

Aetiology: Depends on the type, which are: 1. Primary also known as idiopathic nephrotic syndrome (INS)
2. Secondary 3. Congenital

PRIMARY NS: caused by diseases intrinsic to the kidney and not related to any systemic cause. The glomerular diseases involved are
described based on histological descriptions. These include
- Minimal change nephrotic syndrome (MCNS); commonest, 90% Steroid sensitive
- Focal segmental glomerulosclerosis (FSGS) 20% responds to steroids
- Membranoproliferative glomerulonephritis (MPGN)
- Membranous nephropathy (MN) and the
- Diffuse mesangial

SECONDARY NS: caused by aetiologies extrinsic to the kidney. These include


Autoimmune and vasculitic diseases e.g SLE, HSP
Infections e.g Post Strept.Glomerulonephritis, hepatitis b and c, HIV/AIDS
Malignancies e.g lymphoma
Drug exposure such as gold, mercury, lead, lithium
Non-steroidal anti-inflammatory drugs e.g Brufen, naproxen
Infestations e.g Quartan malaria, TB, Schistosomiasis

CONGENITAL NS: described as NS occurring within the first year of life. Whereas the congenial NS presents before age three months,
the Infantile typically presents between the ages of four and twelve months.
Several genetic abnormalities have been implicated in both types which are defects in the: Nephrin gene (NPHS1), Podocin gene (NPHS2),
Phospholipase C epsilon 1 gene (PLCE1) and Wilms tumor suppressive gene (WT1), Alpha-actinin- 4- gene (ACTN4), Troponin c-6 gene
(TRPC6).
Mutations in the podocin gene (NPHS2) are associated with a familial, autosomal recessive form of FSGS. Mutations in wilms’ tumor gene
(WT1) are associated with Denys-Drash and Frasier syndromes. Mutations in the alpha-actinin-4-gene (ACTN4) and the gene TRPC6 are
associated with autosomal dominant forms of FSGS. The PLCE1 has been identified as the main gene causing diffuse mesangial sclerosis.

Epidemiology: Incidence of NS varies according to genetic, ethnic and/or environmental factors. Found commonly among the south Asian,
Arabian, Indian and African ancestry. It is commoner in males than females with a male to female ratio of 2:1. Approximately 70% to 80%
of cases of NS occur in children less than six years. The rest are found in older age groups. NS is also commoner in children from low socio
economic class.

Pathophysiology: NS develops as a result of an interplay of several factors causing increased basement membrane permeability. A strong
evidence suggests a cytokine mediated immune dysregulation particularly involving the cell-mediated (T-cell) immunity. This is supported
by the ability of the disease to manifest after a viral infection or an atopic episode.
Defects in the podocyte structure as a result of mutations in genes contained within. The most implicated are the nephrin and the podocin

141
Compiled by Obasi. D. C. Chinedu

genes. The underfill theory is as a result of hypoalbuminemia that causes a marked reduction in the plasma colloid (oncotic) osmotic
pressure. The overfill theory suggests that the abnormality leading to oedema is a result of a primary defect in sodium excretion leading to
its retention. it is thought to be mediated by resistance to atrial natriuretic peptide.
In an attempt by the liver to synthesize proteins as they are being lost in urine, hyperlipidemia occurs as a result of concurrent increase in the
synthesis of low density lipoprotein, very low density lipoprotein, and total cholesterol and reduced synthesis of high density lipoprotein.
Hypercholesterolemia is supported by up-regulation of hepatic acyltransferase enzyme involved in the synthesis of cholesterol and down-
regulation of high density lipoprotein receptors.
There is also urinary loss of lecithin-cholesterol acyltransferase and apoproteins responsible for the extrahepatic uptake of cholesterol

Clinical manifestation: Hallmark is generalized body swelling


--face (facial swelling) & around the eyes (periorbital)
--legs (bilateral pitting pedal oedema)
--abdomen (demonstrable ascites)
--scrotum (scrotal oedema) or labia in females
Others are: Respiratory system (cough, breathlessness), Skin (striae), Digestive system (abdominal pain, anorexia, vomiting, diarrhea), Fever
(from infections).

Diagnosis and investigations:


Detailed history: aetiologies, urine colour & output, recurrence. PMHx. Allergy; approx. 30% will have allergy.
Clinical examination: extent/degree of body swelling, face, neck, legs. Lungs, abdomen, sacrum, scrotum in males. Blood pressure: very
important.
Anthropometry: measure weight, abdominal girth

Laboratory investigations
Urinalysis: 3+ or 4+ of proteinuria, haematuria, WBCs, nitrites
Serum protein: Total ↓, albumin < 25g/L
Spot urine protein creatinine ratio: > 2mg/mg
Lipid profile: ↑triglycerides, ↑cholesterol, ↑LDL, ↓HDL
SEUCr, calcium and phosphorus: assess renal function
Renal ultrasonography: structural abnormalities & sizes

Kidney biopsy: Indications include; age < 1 or > 10, steroid resistant, steroid dependent, frequent relapse, secondary NS.

Other investigations: ASO titre, C3, C4. Viral screening: HBsAg, HCV, HIV. Immunological screening: ANA, ANCA, antidsDNA.
Genetic studies.

Treatment:
 Admit patient
 Steroid therapy; tabs prednisone at 2mg/kg/day for 6-8weeks, then taper
 Pooled plasma if albumin is very low
 Diuretics e.g frusemide
 Antihypertensives e.g ACEIs (lisinopril)& ARBs (losartan) because of proteinuria lowering effect

142
Compiled by Obasi. D. C. Chinedu

 Dietary modifications: no added salt, protein of high biologic value


 Treat UTI or RTI with antibiotics if present
 Monitor weight loss: alternate daily weighing
 Patient education; adherence, urine testing
 For those who fail to respond to steroids, treat with cytotoxic agents such as
-- alkylating agents e.g cyclophosphamide
-- calcineurin inhibitors e.g tacrolimus, cyclosporin
-- anti CD20 monoclonal antibodies e.g Rituximab
For those who progress to end stage renal disease (ESRD), renal replacement therapy (RRT) such as dialysis or transplant will be offered

Definition of Terms:
Remission: Nil or trace proteinuria by dipstick for three consecutive days
Steroid sensitive nephrotic syndrome (SSNS): Achieving remission within four weeks of daily oral prednisolone
Steroid resistant nephrotic syndrome (SRNS): Failure to achieve remission after eight weeks of appropriate daily dose prednisolone
Relapse: Recurrence of proteinuria by dipstick for three consecutive days after achieving remission
Steroid dependent nephrotic syndrome (SDNS): Two consecutive relapses during alternate day steroid therapy or within two weeks after
cessation of steroid.
Frequent relapse (FR): Two or more relapses within six months of initial response or four or more relapses in any twelve month period.

Complications: Could either be due to the disease process or following treatment


Complications due to disease process include:
1. Infections: commonest. The reasons why affected children are prone to infections are
-- urinary loss of proteins involved in opsonization and phagocytosis such as IgG, factors B & I
-- steroid therapy
-- depressed T-cell function
-- presence of oedema
Examples of such infections are UTI, RTI, cellulitis, sepsis, peritonitis
Peritonitis is usually caused by the Gram positive organisms particularly streptococcus pneumoniae, the reason why pneumococcal vaccine
is recommended for all children with NS. Escherichia coli is mostly implicated in UTI
2. Hypertension; commoner in those with the non-MCNS
3. Thromboembolism; reasons include
-- ↑ synthesis of prothrombotic factors such as factors V, VII, VIII, X & fibrinogen
-- urinary loss of antithrombin III, protein C & S
-- immobilization from oedema

Complications from treatment


Steroid therapy; moon face, HTN, cataracts, glucose intolerance, striae, infections, osteoporosis
Cyclophosphamide; hemorrhagic cystitis, sterility, myelosuppression
Cyclosporin; nephrotoxicity, HTN

EVALUATION Q1
A 13 year old male presented to the clinic with complaints of recurrent body swelling. He has normal urine output and blood pressure. The
most important investigations will include
A. FBC, SEUCr, urinalysis & renal USS
B. SEUCr, urinalysis, FBC & serum protein
C. Lipid profile, FBC, SEUCr, & spot UP/Cr ratio
D. Lipid profile, spot UP/Cr ratio & biopsy
E. Urinalysis, spot UP/Cr ratio, SEUCr & biopsy

Q2
A 5 year old presented to the CHER with a history of recurrent body swelling, fever and reduced urinary output. Examination findings
showed anansarca, demonstrable ascites by fluid thrill and normal blood pressure. Serum albumin was 12g/l.
A. He will require ascitic fluid tap
B. The fluid will most likely be exudative
C. Commencement of steroid therapy is vital
D. Patient will need a transfusion with fresh pooled plasma
E. SEUCr & FBC should be done.

143
Compiled by Obasi. D. C. Chinedu

CNS INFECTIONS IN CHILDREN


acute bacteria meningitis, encephalitis, cerebral abscess, ventriculitis
Dr Uwaezuoke N.A

Students are to
1. diagnose CNS infections with ease 2. understand its severity 3. known its complications
4. Treatment and its duration with NHIS code 5. apply such knowledge on the bedside.

Outline for ABM


 Introduction - Aetiology - Epidermiology - Pathogenesis - Clinical features
 Diagnosis - Treatment - Prevention - sequelaes
 Questions; clinical scenarios

Introduction
CNS is over protected by 1. skull 2. meninges 3. blood brain barrier 4. poor in host defense.
Once breeched severe infection follows with high morbidity & mortality.
3 meninges protects the brain. The dura (pachymeninx), arachnoid & pia (collectively known as the leptomeninges). The space between
them, the Subarachnoid space contains CSF.
Inflammation is caused by bacteria, including Tb, fungal, viruses, protozoa and occasionally helminths. CSF contains few lymphocytes if
any, low compliments, neutropil proteases, no capillaries hence low neutrophil in CSF. CSF imunoglobulin is also lower than required to
fight infections.

Acute Bacterial Meningitis: Inflammation of the leptomeninges, vessels, brain surface. Vasculitis lead to ischaemia and leaks hydrocepalus.
Sources of infection include the nasopharynx, blood, ear infections, breech in skull/ meninges, maternal genital flora, nosocomial infection.
Severe infection and should always be RX as such.

Aetiology:
Neonatal ABM: Ecoli, grp B Streptococcus, enterobacteriaecae eg proteus. Gram negatives. Others listeria monocytogenes. Staph aureaus.
3 months -3 years; Haemophilus influenza type B (HIb) , Niseria meningitidis (meningococcus), strep pneumo(pneumococcus), M Tb.
(NHIS)
> 3 years. Meningococus, pneumococus, Mtb.
Anatomic defect/immune suppression; pseudomonas aeruginosa, staph aureus, staph. Epidermidis, listeria Monocytogene, salmonela

Haemophilus Influenza Type B (HIB): HIB fastidious, non motile, pleomorphic gram negative coccobacillus. 2 types; typable and non
typable. Typable are listed a,b,c,d,e,f based on polyribosyl-ribitol-phosphate.
Hib is most virulent causing meningitis, bacteraemia, septic arthritis, cellulitis, pneumonia, pericarditis, epiglotitis. Seen in throat of
children <4yrs. non-typable seen in adults as normal flora but may cause infection.
Vaccine preventable by protein conjugation even. Contact with secretion, aerosol.

Neisseria Meningitides: G –ve diplococus, usually encapsulated and pilated. Cultured in rich medium because of fragility. Capsular
serotypes A,B,C,D X,Y,Z, 29E, W135.(9). ET electrophoretic typing eg B ET5.
Meningoccosaemia. Epidermic meningitis A&C in africa. Meningitis belt including northern Nigeria (25 states and FCT), nov to may, high
temp, low humidity. 760 cases reported oct 2018 –apr 2019. 58 deaths. Type C in 29%.
Epidemic when attack rate >15 cases /100,000 persons for two weeks. Increased risk in C5-C9 deficiency, splenectomy, SCD.

Pneumococcus (Streptococcus pneumonia): G +ve smooth encapsulated strains are pathogenic. Only cleared by opsonisation (c3) not
phagocytosis alone. 10 serotypes account for 80% of disease in the temperate regions of world. 6,19,23 less freq. 1,4,9,11,14,15,18. In
Nigeria 1, 3 accounts for 70% of disease. Antibody production is more common after 2 years.

Pathogenesis of ABM: Bacterial proliferation leads to formation of polymorphs & fibrin. This results in inflm of brain, lepto-meninges,
blockage of aqueduct of sylvius and basal cistern. This causes development of hydrocephalus & cerebral oedema. Also Vascular thrombosis
leads to cortical necrosis.
Pus may become encapsulated. CNS may be raped in pus at brain base. Subdural effussion esp with H. influenza.
Bact get to the CSF from any source multiplies freely due to poor immuno defect of CSF.
G+ve peptidoglycans, teichoic acid fragment.
G-ve: lipopolysaccaride fragment all attach endothelium, epithelial cells & macrophages
Cytokine productn; IL-Beta, TNF beta, chemotactic neutrophil factors, neutrophil diapedesis. Inflm products damage vessels, brain cells,
increased permeability of vessels and poor BBB

Intense inflm leads to 3 pathologies:


 Focal inflm results in; ventriculitis, cerebritis, hydrocephalus, nv root damage both CN & SN (meningeal signs)

144
Compiled by Obasi. D. C. Chinedu

 Cerebral edema (cytogenic & vasogenic) leads to ICP, bulging AF.


 Vascular thrombosis leads to ischaemia & infarction leading to focal neurological signs eg focal seizures

Lets see what u have learnt so far! - Explain the mechanism of a +ve kernig’s sign
- Facial nerve palsy in meningitis - What will be the benefit of using an anti inflammatory drug in ABM
- Give two reasons why bacteria multiplies rapidly in the CSF. - List 5 risk factors for developing pyogenic meningitis.
- Why is CSF glucose reduced in ABM - Why is CSF protein increased.

Risk factors:
 Anatomic defects: strep pneu, HIB,Ecoli, pseudomonas, proteus, anareobes, staph
 Immunocompromise: organisms of low virulence + Sp,HIB, NM,
 Opsonisation dysfxn; SCD, Fxnal asplenia,properdin def, acq compliment def
 Iatrogenic eg ventricular shunts; stph. Epididimis, pseudomonas, G-ve

Clinical features: - Fever, headache, neck pain and back pain - Seizures are usually repetitive and more generalised.
- Others: Projectile vomiting, anorexia - HIB may be insidious for 2-3 days while others are sudden.
- The older child with NM maybe mobile walking around with locked jaw for the first two days.
- Kernigs, brudzinski, lassagnes, neck stiffness, petechia, shock - Neck stiffness other signs post 1 year

Diagnosis:
Lumbar puncture and CSF analysis: Normal CSF pressure is 70-180 mmH20, colourless, 0-4 lymphocytes, no polymorph, no RBC,
protein 15-40mg/dl, glucose >66% of RBS.
ABM pressure >200, turbid, protein >40mg/dl and up to 1500mg/dl, glucose is decreased.
Partially treated bact. Meningitis. Clear or cloudy, usually raised, elevated protein, normal or reduced glucose.
Treat once u see pus cell. It means there are poly morphs engulphing organisms.
Gram stain. 60% detection rate

Other tests are: - Fbc, Immunologic tests - Counter current immuno electrophoresis; 80% detection rate.
Coaglutination tests; 2x as sensitive as CIE, rapid diagnostics and cheaper.
Latex agglutination; 2x more sensitive than CIE when detecting meningococal type A, and NM
ELISA better than all above but used for many samples.
Mab ELISA. Better than ELISA but very expensive for muti sample analysis. Not common.

ABM Treatment for infant & older child:


 Note meningitic doses due to BBB
 Use of steroids IV dexa 0.15mg/kg 6hrly for 2 to 4 days started hours before or immediately b4 antibiotics
 3rd generation cephalosporin; ceftriaxone (rocephine) 100mg/kg/day. Ceftazidime(fortum) 150mg/kg/day 8hrly. Cefotaxime 200-
300mg/kg/day q6hrly
 Penicillin G + chloramphenicol Penicillin 250,000 -400,000 Units/kg/day, chloramphenicol 100mg/kg/day
 Ampicillin+ chloramphenicol
 Amikacin/genticin/kanamycin can be added in G-ve meningitis

Duration of Rx: NHIS; N= NM rxed 7 days, HI=HIB Rxed for 7 to 10 days, Strep pneumonia 10 days
Neonatal meningitis 21 days treatment cause of the g-ve organisms eg E. coli. No oral medication allowed all medication are IV. NEVER
GIVE ORAL ANTIBIOTICS WHEN TARGETING CNS infections.

Supportive treatment
- Nursing esp when unconscious - Use of drugs to reduce ICP - Cortical blindness/deafness
- Physiotherapy post treatment when they loss motor milestones - DIC give platete, FFP, FWB
- Feeding. Tubes - Pneumonias may need Oxygen - Fluid restriction to prevent SIADHS
- Seizures use diazepam for breakthrough seizures, paraldehyde, iv phenytoin in status. AED can be used.

Neurological sequelaes: include Deafness, Blindness (cortical), Hydrocephalus, Ataxia, CP, MBD, Epilepsy, Speech and language
impairment, MR, Hyperactivity syndrome.

Prevention of meningitis: NM epidemic meningitis; anti A&C, for those 4 yrs and older. Protects 10 days after a shot, contains an outbreak
in 21 days.
Chemoprophylasis in NM, used within 48hrs of diagnosis, for all patients in admission, house hold/ school mates and all health staff.
Rifampicin, amoxicillin. Routine immunisation; conjugated HIB vaccine, PCV 7.
Questions:
1. Ola is a 2 year old male who developed fever and excessive cry. The emergency doctor thought of meningitis and did an Lp. CSF gam
stain showed diplococci g+ve. He gave 1g of ceftazidine once daily for 5 days.

145
Compiled by Obasi. D. C. Chinedu

Ola weighed 12 kg when he was reviewed. While in the ward the CSF culture yielded Strep. Pneumonia sensitive to ceftriaxone and
ampicillin. Should the doctor continue with the ceftazidime or revert to ceftriaxone or ampicillin and if so should the ceftazidime
which already had been given for 3 days count?

2.Nembe is 1 years old today, while preparing for his birthday party he developed fever and within an hour was already convulsing. At the
emergency room he was noted to have a status. His ofc incresed by 3 cm the next day and gram stain showed gram negative cocco
bacillus. CSF chemistry and culture are suggestive of ABM
What is the causative organism? Why is there a rapid head size how can this be managed.

ENCEPHALITIS: Inflammation of the brain tissue. Encephalopathy mean similar symptoms as encephalitis but without
inflammation.eg hepatic encephalopathy. There could be a combination of meningitis and encephalitis giving a meningo-encephalitis.
Encephalitis is usually caused by viruses and shows that the infection is diffused.

Aetiology:
1) Viruses. Arboviruses are the most common cause others are mumps virus, influenza A&B, measles, herpes group: vz, herpes simplex,
cytomegalovirus.
Other virus spread from animals eg Rabies, equine, lymphocytic chorio meningitis from rat excreta.
2) NON VIRAL infections. rickettsia, mycoplasma pneumonia, protozoa, metazoans, spirochetes.
3) post infectious. Measles, rubella, rickettsia, hepatitis
4) human slow viruses; SSPE, PML progressive multifocal leukoencephalopathy

Clinical Features: Convulsions are more difficult to handle. Low grade fever but occasionally high grade fever.
Involves many parts of the brain eg the cerebellum and cerebrum same with radiculopathy etc.
Coma is usually more pronounced and last longer. Sequeles are more often present and may last a lifetime
Diagnosis: CSF pressure is normal or slightly raised. Viral cultures using CSF. PCR

Treatment: Most virus have no treatment and use of anti viral agent in paediatrics is not popular cos of narrow theraupetic index.
Herpes simplex virus: iv acyclovir 30mg/ kgday in three divided doses for 14 days. HAART in HIV encephalitis.
Supportive care. Seizure management.
Prevention: Vaccination with MMR, measles vaccine

Cerebral Abscess: Occurs in all ages but most common 4-8 yrs. 20% located in brainstem, occipital region, cerebellar. 30% are multiple.
Abscess in frontal lobe is more likely from sinuses or orbital cellulitis. Those on temporal lobe and cerebellum are most likely from
chronic otitis media & mastoiditis. Those from penetrating injury are single and staph aureus while those from septic emboli are have many
organisms as cause.
Aetiology: S. aureus, Strep. Viridians, Anaerobic organism, Proteus, Pseudomonas auriginosa, Citrobacter diversus.
One organism is cultured in 70%, two in 20% and more in 10%
Pathophysiology: Septic embolisation in CHD cyanotic type. Infections in structures around brain eg sinusitis, dental abscess, mucositis of
mouth, chronic otitis media, orbital cellulitis, mastoiditis, soft tz infectn of face and scalp. Imuno deficiency state. Infections of shunts VP.
Unknown source in 10% of cases

Diagnosis (ring enhancement): CSF is usually clean unless the abscess is ruptured. CBC may be normal or elevated.
Only 10% of blood cultures yield any growth. Large abscesses give SOL giving raised ICP. Be careful with LP
EEG shows slowing. CT Scan ring enhancement with contrast. MRI capsule seen with gadolinium

Treatment:
Initially emperical and to cover for gram positive anarobes gram negative. We use iv cephtriaxone, iv metronidazole and iv genticine.
Penetrating head injury use vancomycin or flucloxacillin and metronidazole. RX for 4 to 6 weeks IV. Drainage is preferred but surgery when
ring is >2.5cm diameter. Good prognosis.

146
Compiled by Obasi. D. C. Chinedu

ACQUIRED HEART DISEASES


Prof. Kunle Obidike

Outline
 Introduction –
 What makes a disease an acquired heart disease?
 Types
 Infective Endocarditis
o What is it and classification
o Aetiology and pathogenesis
o Clinical features
o Investigations
o Treatment
o Prognosis
 Rheumatic Heart Disease
o Aetiology and types
o Pathophysiology
o Clinical features
o Investigations
o Treatment.

Introduction:
There are many heart diseases that could ordinarily be considered as acquired heart disease. However, this topic is limited to infective
endocarditis and rheumatic heart disease. These mentioned heart diseases tend to
 Not have a genetic cause even though they may have a genetic predisposition
 Have clear aetiologies/risk factors
 Have immunology complicit in their evolution and
 All the layers of the heart are often involved in the inflammation in the early phase.

Infective Endocarditis: Involvement of the endocardium in an infective process. Classified into


- acute bacterial - subacute bacterial - non bacterial

Infective agents; Not dependent on age of child, type of congenital defect or duration of illness. Certain organisms however are certain
procedures dependent;
- staphylococcus aureus where there is no underlying problem
- viridans group strept after dental procedures
- group d enterococci after lower bowel or genitourinary manipulations
- fungal after open heart surgeries
- coagulase neg staph in indwelling central venous catheter
- ps aeruginosa and serratia marcescens in iv drug users.

These procedures are predispositions.


Note:
In only about 30% of cases can above predispositions including that of a child with cyanotic heart disease and poor oral hygiene be isolated.
This condition is not common in infancy except following an open heart surgery.
It often complicates a congenital heart disease where preferably there is a high velocity lt to rt shunt through a hole or stenotic opening and in
rheumatic heart disease
The valves of a normal heart can occasionally be affected.

Pathophysiology: The turbulent flow causes erosion of the endothelial or intimal layer and formation of vegetations on the valves and areas
where the flow jet hit.

Clinical features: Requires a high index of suspicion especially in a child with predisposing factors – initial features not very specific though
late manifestations will improve diagnostic capabilities.
Generally, the clinical features vary depending on if it is: Acute bacterial, Subacute bacterial or Non bacterial – fungal etc. The later two
present in similar ways.

Acute bacterial: Clinical features – symptoms and signs – are acute and severe with High intermittent fever,
Prostration, meningismus, focal neurologic signs, altered sensorium, stroke, seizures, headaches and heart block.

The pathologies associated include


 Embolic strokes, increased intracranial pressures, cerebral abscesses

147
Compiled by Obasi. D. C. Chinedu

 Mycotic aneurysms, haemorrages


 Myocardial abscesses, damage to conducting system, purulent pericarditis.

Subacute bacterial endocarditis/non bacterial endocarditis: Insidous in onset with early and late manifestations
Early manifestations: Prolonged fever with afternoon elevations (could last for months).
Fatigue, night sweats, chills- cns - Arthralgia, myalgia, weight loss- mss
 Dyspnoea- rs - Nausea, vomiting - ds
New or changing heart murmur associated with heart failure – cvs - Splenomegaly, petechiae - hs

Late manifestations – thought to be due to vasculitis from circulating antibody-antigen complexes incude;
- skin and appendages lesions
-osler nodes – tender pea sized intradermal nodules in pads of fingers and toes
-janeway lesions – painless small erythematous or haemorrhagic lesions on the palms and soles
-splinter haemorrhages – linear lesions beneath the nails
- glomerulonephritis

Investigations:
Blood tests: Blood culture positivity (3 -5 cultures with 90% positivity in 1st 2 samples). Done at peak of fever.
Findings: Anaemia, Elevated esr, Low serum complements, Leucocytosis and Serologic evidence of infection.
Chest Xray: Bilateral infiltrates, nodules, pleural effusions.
Echocardiography: Myocardial abscesses – acute. Vegetations on valves or any other site – subacute. New onset valve insufficiency.
Prosthetic valve dysfunction or leak.

Treatment: urgent antibiotic therapy for 4 – 6 weeks consisting of;


For acute bacterial endocarditis:
- nafcillin or oxacillin; cephazolin in penincillin allergic; vancomycin for oxacillin resistant.
- gentamicin
For subacute bacterial endocarditis
- crystalline penicillin or cetriazone
- gentamycin
- vancomycin
They have to be used as much as possible based on culture and sensitivity results. Fever, if responsive resolves in 5 – 6 days.
- Digitalis and diuretics for failure
- Surgery if condition intractible or failure does not improve

Prognosis: Mortality is 20 -25% while sequellae is seen in about 50 – 60% 0f suvivors.


Prevention: Antimicrobial prophylaxis for certain procedures in at risk individuals. Proper dental care and oral hygiene in at risk children.
Careful asepsis during open heart surgery and catheterization.

Rheumatic heart disease: Follows occasionally on a rheumatic fever. Commonly affects the left chambers and valves (atrioventricular and
semilunar) related to them. Two variants;
Acute – due to effects of immunologic inflammation of the layers of the heart resulting in weakening of these structures, an occurrence in
rheumatic fever.

Chronic – from structural sequellae following resolution of rheumatic fever, that could have been recurrent. Two effects: Incompetence and
Stenosis from fibrosis of the valve ring, contractures of the valves and adhesions of the commissures late onset (10 or more years)
Pathophysiology:
Acute – verrucae on edges of valves, mitral valve Incompetence and weakened myocardium result in cardiac failure.
Chronic – resolution of inflammation and the verrucae (often repeated) and damage to the valves resulting in; Mitral incompetence and
Later stenosis.

All these processes result in cardiac failure through different mechanisms and cause effects on the heart that could point to the particular
lesion.

Clinical manifestations: Depends on whether it is the incompetence lesion the stenotic or a mixed one.
Incompetence lesion – depends on the severity of the incompetence;
- For mild degrees: Symptoms may not be marked. Examination may only reveal a regurgitant systolic murmur at the apex that radiates
into the axilla
- For severe degrees; Symptoms and signs of heart failure with additional; Heaving apex beat and apical thrill, Accentuated 2 nd heart
sound in the presence of pulmonary hypertension.
Also, Regurgitant systolic murmur that radiates into the axilla and Presence of 3rd heart sound.

148
Compiled by Obasi. D. C. Chinedu

For stenotic lesions– features correlate very well with degree of obstruction;
- Mild obstructions are often without symptoms
- Severe obstructions; Exercise intolerance, Dyspnoea
- Critical obstructions; Orthopnoea, Paroxysmal nocturnal dyspnea, Atrial arrythmias.

Examination may show


- Elevated jugular venous pressure and features of pulmonary oedema
- Pulmonary hypertension
- Tricuspid insufficiency
- Atrial arrythmias
- Apical mid diastolic murmur with a presystolic accentuation.
- Hepatomegaly; ascites
- Oedema

Investigations
Incompetent lesions
For mild lesions: Cxr and ecg – normal
For significant lesions; Cxr: Cardiomegaly, Pulmonary venous congestion
Ecg: Lt ventricular hypertrophy; Bifid p-wave
Stenotic lesions
For mild lesions: Cxr and ecg – normal
For significant lesions: Cxr: Lt atrial enlargement, Prominent pulmonary artery and rt sided heart chambers, Calcification in the
mitral valve
Ecg – prominent p waves, rt ventricular hypertrophy, atrial fibrillation in late stages

Treatment
Acute lesion: Diuretics, Digoxin, Prednisolone and then acetyl salicylate. Antibiotics.
Chronic lesion
Medical: Diuretics, Digoxin, Ace inhibitors, Prophylactic antibiotics, Anti-arrythymic drugs.
Surgery: Valvotomy and valvuloplasty most often used before severe manifestations appear.
Valve replacement is not usually advocated but patients must be seen before condition becomes critical.

Thank you

149
Compiled by Obasi. D. C. Chinedu

BLEEDING DISORDERS
PROFESSOR IFEOMA EMODI

Introduction
Bleeding disorders(BD) are conditions which affect the way blood clots. It causes abnormal bleeding both outside and inside the body.
Majority of BD are inherited or develop as a result of other medical conditions and some medications.
Normal haemostasis is defined as a consequence of tightly regulated processes that maintain blood in a fluid clot-free state in normal vessels
while introducing the rapid formation of a localized haemostatic plug at the site of vascular injury.
Mechanism include: Vasoconstriction, formation of plt plug, coagulation cascade fibrinolysis
Normal haemostasis requires the integrity of three elements – blood vessels, platelets and soluble clotting factors.
A significant bleeding disorder is indicated by 1) petechiae, purpura or both, 2) severe rec epistaxis, 3) prolonged bleeding after tooth
extraction, surgical procedures, or major trauma 4) recurrent haemarthrosis 5) menorrhagia.

Causes of Bleeding

Screening the patient requires a battery of laboratory tests which include


a) FBC, platelet count and morphology
b) aPTT measures integrity of intrinsic & common pathways
c) PT measures integrity of extrinsic and common pathways
d) Bleeding time assesses vascular integrity and platelet function

150
Compiled by Obasi. D. C. Chinedu

Clotting Disorders: These are a group of disorders involving the body’s clotting process. Such conditions can lead to heavy and prolonged
bleeding. Normal blood clotting involves as many as 20 different plasma proteins which are known as coagulation factors. Problems occur
when these factors are low or absent.

Causes of Bleeding Disorders: Some of these can be inherited or acquired. They Include
a) Qualitative or quantitative abnormalities of plasma coagulant factors
b) Qualitative or quantitative abnormalities of platelets
c) Vascular abnormalities
d) Accelerated fibrinolysis

Blood coag is a host defense system which maintains the integrity of the closed circulation. Coagulation factors are grouped into (a) vit K
dependent zymogenes– Fs VII, IX, X, protein C (b) soluble factors—Fs V, VIII, VW and © others XI, cell assoc co-factors, fibrinogen,
plasma coag PI.
Coagulation Pathway:
151
Compiled by Obasi. D. C. Chinedu

Two mechanisms of activating clotting involve the intrinsic and extrinsic pathways. The intrinsic pw is initiated when factor XII is
activated by contact with damaged epithelium. When TF is released from damaged cell, in conjunction with FVII and Ca ++ converts
Factor X to Xa in the extrinsic system.
Both pws activate factor X to generate XA which complexes with Factor V, platelet phospholipid and Ca to activate prothrombin II to
thrombin IIA. IIA converts fibrinogen to fibrin. IIA also activates fac XIII which through clot retraction and cross linking of the fibrin clot
forms the stable fibrin-platelet plug.

In the intrinsic pathway, factor XII, high mol wt kininogen, prekallikrein, and activated factor XI (factor XIa) in response to
endotoxin, antigen-antibody complexes, fatty acids from fat embolism, burns, and extracorporeal circulation, interact to produce factor IXa
which then combines with factor VIIIa and procoagulant phospholipid (present on the surface of activated platelets and tissue cells) to form a
complex that activates factor X. Factor XIIa leads to the activation of the complement system and generation of bradykinin (responsible for
DIC, hypotension).

In the extrinsic pathway, in response to trauma, factor VIIa and tissue factor or thromboplastin, which is the primary activating moiety for
the extrinsic pathway of coagulation, directly activate factor X (the factor VIIa/tissue factor complex). TF also activates factor IX. TF is
expressed by cells of the subendothelium (smooth muscle cells, fibroblasts) and leukocytes. Excessive release of TF is the primary
mechanism involved in DIC resulting from trauma, especially head injury (rich TF content of brain tissue), and obstetric complications.

Common Pw: Prothrombin to fibrin monomer, fibrin polymer and finally advanced cross linked fibrin polymer
12 -12a: Cofactors – PK, HMWK. Inhibitors – Heparin, Antithrombin III
11 to 11a: Cofactors – Factor VIII, (vWF) carries Factor VIII. Inhibitors - Protein C, Protein S
Intrinsic Clotting Pathway- Initiated by Negatively charged surface
Extrinsic Clotting Pathway- Initiated by disrupted cell membranes
Common Clotting Pathway follows above initiators, Antagonized by Clotting Pathway Inhibition

For the bleeding pt obtain a detailed personal and family history. Thorough physical exam with special attention to the skin, oral and nasal
mucosa, liver, spleen and joints. Screening tests have been detailed above

PLATELETS: Causes of thrombocytopenia


a. Destruction
 (ab mediated) ITP, infection imm diseases
 (coag) DIC, sepsis, NEC, thombosis, Carv H
 (other) HUS, hypersplenism, RDS

Purpura: This is the appearance of red or purple discolorations on the skin that do not blanch on applying pressure. Caused by bleeding
under the skin. Purpura measures 0.3-1cm, Petechiae less than 3mm, Ecchymosis greater than 1cm.

b. Decreased production: (Normal count is 150,000 – 300,000/mm3).


Congenital: Fanconi anaemia, Wiskott- Aldrich syn, Thrombocytopenia with absent radii, metabolic disorders, osteoporosis
Acquired: aplastic anaemia, leukemia and other malignancies, vitamin B12 and folate deficiencies, hypersplenism

152
Compiled by Obasi. D. C. Chinedu

Acute Idiopathic Thrombocytopenic Purpura: The most common bleeding disorder of childhood. Frequent at 2 – 5 yrs. Follows
infection with viruses such as rubella, varicella, measles, immunization or EBV. Most recover spontaneously. Chronic ITP (> 6 mths) seen
in 10 -20%. Results from immune clearance of platelets. Spleen plays a major by forming antibodies and sequestering damaged platelets.
Onset is acute with app of multiple eccymoses, petechiae esp on lips and buccal mucosa, epistaxis is common.
Generally, no other physical finding. Platelet count usually 50.000 – 10,000/μL.
Bone marrow shows ↑ megakaryocytes which may be larger than norm. Erythroid myeloid cellularity is normal.
PTT, PT normal, bleeding time is prolonged. Differential diagnosis as in the list above
ITP is a diagnosis of exclusion. Complications include Severe hemorrhage and bleeding into vital organs.

Most children require no therapy, avoid aspirin, and trauma. In cases of sig bleeding or platelet count<10,000 can be treated with pred 2 -4
mg/kg orally in divided doses for 1 -4 wks and then taper.
As an alternative or adjunct to above IVIG may be used in both acute and chronic. It is the drug of choice for sever life threatening
bleeding. 1g/kg/d for 1 – 4wks in the usual dosage.
Rho (D) immune globulin can be used only in Rh+ patients. Takes longer time for response.
Splenectomy is reserved for those who do not recover after treatment with above options and still have significant thrombocytopenia for at
least a yr
In the newborn, most common cause of ↓plts are congenital infections (TORCHES) bacterial sepsis, immune mediated causes and DIC
Qualitative platelet disorders may be congenital or acquired.
- Bernard-Soulier synd, Glanzmann’s thrombastenia
- Drug induced (aspirin), myeloproliferative dis

Von Willebrand’s disease/Haemophila A: Both involve different regions of factor VIII molecule. Can be distinguished frm each other by a
number of clinical and laboratory features. Factor VIII is a complex of two proteins:
Factor VIII procoagulant protein (antiheamophilic factor; VIII:C)
Factor VIII-related protein (von Willebrand factor; VIII:R)
Features of common inherited bleeding disorders
characteristic Haemophilia A Haemophilia B Von Wilebrand’s

inheritance X-linked X-linked Auto Domin


bleeding Jts, mus, tra As haem A Epi trauma
aPTT ↑ ↑ ↑ or Normal
Bleeding Tim Normal As Haem A ↑
Deficiency F VIII F IX VWF
PT Normal Normal Normal
Platelet Normal Normal May be low
Risocetin-ind P Normal Normal Abnormal
aggregatn
Haemophilia A: Results frm deficiency of VIII:C that acts as cofactor in the intrinsic PW of coagulation. Defective gene located on
chromosome X. Characterized by spontaneous or traumatic heamorrhage (subcut, IM) and heamathrosis. In infants bleeding after

153
Compiled by Obasi. D. C. Chinedu

circumcision and + history in maternal brothers.


Classified into mild 5 ≤ 40%, moderate 1 -5%, severe <1% activity. The bleeding into joints is painful, and leads to long-term inflammation
and deterioration. There is permanent deformities, misalignment, loss of mobility, and extremities of unequal length.
The worldwide incidence is about 1 case per 5000 males. About 1/3rd of affected individuals may not have a family history. Mortality rate is
twice that of normal males. Viral complications became a problem during the replacement era.

Severely affected boys (<1% activity) show easy bruising and are prone to heamathrosis frm the time they start to walk. Later joint, soft
tissue & muscle bleeding. Life threatening int bleeding may follow trauma.
The aPTT is prolonged.
Treatment includes general supportive care and replacement therapy with Factor VIII concentrate or cryoprecipitate.
Recombinant factor VIII and IX produced through DNA technology epitomises progress in hemophilia care

Von Willebrand Disease: This is a family of bleeding disorders caused by an abnormality of VWF (quantitative or qualitative). VWF is a
L glycoprotein that fns as a carrier protein for FVIII. It is also required for platelet normal plt adhesion. It is the most common hereditary
bleeding disorder. Characterized by a lifelong tendency towards easy bruising, frequent epistaxis and menorrhagia.

Transmission is variable AD, AR. Severity varies with degree of deficiency. Can be classified into 3 main types. 3 is most severe. Bleeding
is mild, may follow trauma. Lab findings include prolonged bleeding time and aPTT time. VIII:R ag is low. Supportive trt and
replacement. cyroprecipitates as factor concentrate lacks sufficient VIII:R
Desmopressin effective in type I (iv or intranasal).

Haemophilia B (IX) (Christmas Disease): X – linked recessive. Bleeding similar to haemophilia A, aPTT is prolonged and activity of
Factor B is ↓. General supp measures, replacement with ffp or prothrombin complex concentrate (mix of factors II, VII, IX, and X).

Prophylaxis:
The best treatment for Haemophilia A (severe)& B is prophylaxis. This was started in some countries like Canada and Sweden in the 1960s.
Barriers to this include compliance and cost.
Prophylaxis begins at 1 – 2 yrs – for FVIII 25 -30U/kg every other day, FIX 50 – 60 U/kg twice a wk

VITAMIN K def:
Vitamin-K dependent factors include: Factors II, VII, IX, X, Protein C and S. The factors are synthesized in the liver and this process is
Vitamin K dependent. Etiology includes malabsorption states, other GIT disorders and use of drugs (coumarin) that are Vitamin K
antagonists. Haemorrhagic disease of the NB can result from omission of routine use of Vitamin K at birth.
Liver Disease: All coagulation factors may be deficient with exception of VIII. Hepatic clearance of activated clotting factors may be
impaired. Both aPTT and PT may be prolonged. Fresh frozen factor is the therapy of choice.

Hemolytic Uremic Syndrome(HUS): Related to TTP, no of plts suddenly dec, RBCs are destroyed and kidneys stop functioning. HUS is
rare but can occur with certain bact inf (E.coli, shigella) drugs (quiniine cyclospo).
154
Compiled by Obasi. D. C. Chinedu

Toxin producing org such as E coli cause endothelial damage that activates localized clotting, leading to plt aggregation and consumption.
Common in young children and pregnant women.

Thrombotic Thrombocytopenic Purpura (TTP): Rembles HUS but occurs more commonly in adults than in children. Spontaneous
aggregation of plts and activation of coagulation in small blood vessels.
In TTP, plt consumption, precipitated by a congenital or acquired def of metalloproteinase that cleaves vWF

QUESTIONS: Should children with severe clotting disorders undergo circumcisions?


Can we infuse bovine or porcine products in medical treatment?

Answers:
Von Willerbrand Disorder
Circumcision is not compulsory, can if only procedure is monitored by specialist and prophylaxis is available
Can use porcine or bovine products

155
Compiled by Obasi. D. C. Chinedu

ADRENAL DISORDERS
Ugo Chikani MBBS, FMCpaed, FESPE

Outline
 Learning objectives - Anatomy/function - Forms of adrenal disorders
 Adrenal insufficiency (addison`s crisis) - Congenital adrenal hyperplasia - Adrenal excess

Learning objectives: At the end of this presentation, Students are expected to:
1. Know the pathophysiology of Adrenal insufficiency, CAH and Adrenal excess
2. Identify the causes and the clinical manifestation of each condition
3. Explain and manage Addison crisis
4. Know the different treatment modalities

Anatomy: Adrenal cortex is divided into three anatomic parts viz; zona glomerulosa, zona fasiculata and zona reticularis. A fetal zone
which is seen only in primates involutes within the first few months of postnatal life.
Aldosterone is regulated by RAS and increase serum potassium concentration. Cortisol is regulated by CRH-ACTH. Cortisol has a negative
effect on both CRH and ACTH to prevent excessive secretion of cortisol from the glands
ACTH in addition to other unknown factors regulates the secretion of adrenal androgens. ACTH also promotes growth of the adrenal cortex
in conjunction with growth factors such as insulin like growth factor (IGFT) and IGF-2

Forms of adrenal disorders: They can be Adrenal Insufficiency or Adrenal Excess

Definition of AI: Adrenal insufficiency (AI) is the inability of the adrenal glands to produce enough corticosteroid hormones to maintain the
body`s need.
Adrenal insufficiency can be classified according to the site affected as; Primary or Secondary.
AI can also be classified as Congenital or acquired

Besides CAH, many other conditions cause adrenal insufficiency, including ACTH deficiency(central) and primary adrenal disorders.
Primary adrenal insufficiency is commonly termed Addison disease, a vague term that encompasses many disorders (see table).

Causes of adrenal insufficiency(AI): Primary adrenal insufficiency


ACQUIRED: - Autoimmune adrenalitis - Autoimmmune polyglandullar syndromes (type1 and 11)
- Adrenal hemorrhage or infarction - Infections ( e.g. TB, HIV, fungal ,meningococcal meningitis)
- Neoplastic destruction - Unresponsiveness to ACTH - Administration of anaesthetic agent
- Metabolic disorders (eg various forms of adrenal Leucodystrophy, wolman disease, smith-lemil- opitz syndrome)
- Hermochromatosis, may cause either primary or secondary AI
- Iron deposition in the pituitary &/or adrenal glands may also cause AI seen in patients with multiple transfusion
CONGENITAL: Congenital adrenal hyperplasia/hypoplasia

Secondary adrenal insufficiency: - Withdrawal from glucocorticoid therapy


- Hypopituitarism (central AI) - Hypothalamic tumors - Irradiation of the CNS

1. Auto immune Destruction of the Adrenal cortex: It can be Type 1 Autoimmune polyglandular disorder (APECED (PGAD)- is
isolated or part of a syndrome: an AR disorder that involves the AIRE gene on chr 21
Usually occur in the first decade of life.
Schmidt syndrome – Addison disease and Hashimoto Present with all or some of the following features: Chronic
thyroiditis. mucocutaneous candidiasis, Hypoparathyroidism, Gonadal failure
Vitiligo, Alopecia

Type 2 PGAD consists of TIDM, autoimmune thyroid disease, adrenal


failure.
Manifest in 2nd or 3rd decades of life
Although some components of the syndrome maybe be present in
pediatric age group
Transmitted as an AD disorder with variable penetrance
Addison disease should be considered in patients with type1 DM,
hypotension, hypoglycemia, hyponatremia and hyperkalemia

156
Compiled by Obasi. D. C. Chinedu

Adrenal hemorrhage or infarction caused by Infections (e.g. WATERHOUSE –FRIDERICHSEN SYNDROME


TB, HIV, fungal ,meningococcal meningitis)

CENTRAL AI: Aldosterone secretion may not be affected IATROGENIC CENTRAL AI


Most cases of central AI are iatrogenic, caused by long term administration
of glucocorticoids
A 2 week exposure to pharmacologic does of glucocorticoids can suppress
the CRH- ACTH- adrenal axis
The suppression can be so great that sudden withdrawal or stress may

157
Compiled by Obasi. D. C. Chinedu

prevent the axis from responding with sufficient cortisol production to


prevent an acute adrenal crisis.
The suppression is also seen in drugs that have glucocorticoid properties.
Aldosterone secretion may not be affected

Signs and symptoms of adrenal insufficiency


Features shared by acute and chronic insufficiency: Hypoglycemia, Nausea and vomiting, Dehydration, fatigue
Postural hypotension, Hypovolemia and tachycardia, Altered mental status (even without hypoglycemia)
Apathy and confusion, Anorexia and loss of appetite, weakness, lack of energy
Hyponatremia and hyperkalemia, salt craving.

Features of acute insufficiency (adrenal crisis):


In infants, acute AI may occur in the context of serious illness (eg sepsis and birth asphyxia from prolonged labour or difficult delivery),
Abdominal pain, Fever.

Features of chronic insufficiency (Addison disease): Decreased pubic and axillary hair, Diarrhea, Hyperpigmentation (increase MSH
activity of ACTH). Low- voltage ECG, Small heart on X-ray, Weight loss.

Diagnostic consideration: High index of suspicion


Any patient that presents with chronic fatigue, anorexia, nausea, vomiting, diarrhea, unexplained weight loss, dehydration, hypoglycemia and
hypotension.
Serum electrolytes-Hyponatriaemia and hyperkalemia.
A plasma renin activity(PRA) to aldosterone ratio of more than 30 is suggestive of inadequate mineralocorticoid production

ACTH STIM TEST (See image on the Right)

Other tests include: Insulin tolerance test, Metyrapone stimulation

Management of acute AI: Acute crisis is characterized by hypotension, hypovolemia, hypoglycemia, hyponatriaemia or hyperkalaemia
Fluid resuscitation: In a hypotensive patient- rapidly give 20mls-30mls/kg NACL or 450ml/m2 over ASAAP
Repeat if the patient remains hypotensive. maintain with typical continuous infusion of 3200ml/m/d or 200ml per 100 cl calories of estimated
energy expenditure at rest to restore intravascular volume.

158
Compiled by Obasi. D. C. Chinedu

Perioperative consideration: Give stress dose (50-75mg/m2) IM or IV. When patient is being transported to operating room or in advance of
planned surgery. Fludrocortisone maybe withheld on the day of surgery while the patient is still on stress dose.
Intraoperative period: During surgery, administer additional doses of hydrocot, either hydrocortisone infusion at a dosage of 2-4 mg/m2/h or
IV Boluses of 10-25mg/m2 every 6 hour throughout the procedure. After surgery, continue the administration of hydrocortisone in the
immediate post operative period.
In the 2nd and third post-op days, decrease the dose by 50% each day to a minimum of the patient’s usual daily requirement if the patient is
recovering well and has no complications
Postoperative period: By the 4th day, the usual daily dosage of steroids maybe resumed, if patient is recovering satisfactorily. If
complications occur, stress dose of glucocorticoids must be continued. Also stress dose should be continued for those who cannot take oral
fludrocortisone.

Correction of hypoglycaemia: 4mls/kg of 10% of D/W. Maintain with 5% DW to prevent initial OR further hypoglycaemia
Glucocorticoid administration: Administer stress dose of glucocortoid (50-75mg/m2) given IV, followed by 50-75mg/m2) in a 24 hour
infusion, followed by 50-75mg/m2/ in 4 divided doses.
Hydrocotisone can be given IM, if no IV access. Comparable stress doses of methypred are 10-15mg/m2 and dexamethasone 1-1. 5mg/m2.
Dexamethasone is preferable for patients with suspected but unproven AI because the physician can treat the child while performing
cosyntropin stimulation test. Methypred and dexamethasone have negligible mineralocorticoid effects.
Large dose of hydrocot (2 fold to 3 fold stress doses) are preferred if the patient is hypovolemic, hypona+ and hyperK+. Due to the
mineralocorticoid effects of hydrocortisone
No parenteral form of mineralocorticoid. Oral fludrocortisone 0.1-0.2mg can be administered
Iatrogenic AI can be prevented by giving the patient dosages below his or her physiological requirements
Treatment with alternate-day oral prednisolone or with tropical or inhaled glucocorticoids can reduce the risk

Complications:
Hypertension, shock, hypoglycemia and death
In addition, daily oral glucocorticoid therapy may provide latrogenic suppression of the (HPA) axis within weeks
Effects can last for weeks to months, depending on the duration of exposure to pharmacologic does of glucocorticoids
Complications of excessive glucocorticoids include growth failure, hypertension, obesity, osteoporosis, muscle weakness, hyper glycemia,
cataracts.
Complications of excessive administration of mineralorcoticoids include hypertension and hypokalaemia

Long term monitoring


Growth velocity is the most sensitive indicator for monitoring long term treatment. Hydrocortisone is the drug of choice. Symptoms of salt
craving, blood pressure, plasma renin activity and electrolyte will help in adjusting the dose of fludrocortisone. Those with autoimmune
disease should watch out for other autoimmune disorders.
Re-evaluate patient`s caloric intake and glucocorticoid dosing if there is excessive weight gain since glucocorticoid stimulates appetite.

FOR EXERCISE: No stress dose is required.


Those in warm climate should receive extra salt to prevent hyponatriaemia through sweating.

Patient education: Potential for death if adequate replacement is not provided. Seek medical help immediately they are sick. Wear bracelet
or carry card or tag in case they found unconscious.
Patient and caregiver should be taught how to administer glucocorticoid in times of illness of traumatic stress, esp how to give injectable
steroids during vomiting or when patient cannot take orally. Periodically, reinforce this information.
1m injection of hydrocortisone at does of 25mg for infants, 50mg for children, 100mg for adults can be life saving in the interim before the
patient receives professional medical care. Rectal steroid can be used
Patients should eat an unrestricted diet and should have ample access to salt because of the salt wasting that occurs if their condition is
untreated. Infants with primary AI should be given 2-4g NACL per day

Congenital Adrenal Hyperplasia (CAH): a group of AR disorders involving deficiency of an enzyme in the synthesis of cortisol,
aldosterone or both. The common functional defect in each disorder is impaired cortisol secretion. Resulting in hyper secretion of
corticotrophin-releasing hormone (CRH) and adrenocorticotropic hormone (ACTH) and consequent hyperplasia of the adrenal glands.

Epidemiology: Incidence ranges from 1:10,000-1:20,000 births. Prevalent in some ethnic groups particularly in remote geographic groups
e.g Alaskans, yupiks. Most common form is caused by CYP21 A2 gene. Non classic 21OHD may be the most common AR disorder in
humans. Its prevalence rate is approximately 3.7% (1 in 27) in Ashkenazi Jews and 0.1% (1in1,000) in white populations.

Pathophysiology: The clinical manifestations of each form of CAH relate to the degree of cortisol deficiency ± the degree of aldosterone
deficiency or accumulation of precursor adrenocortical hormones. When present in supra physiologic concentrations, these precursors lead to
excess androgen production with resultant virilisation or cause sodium retention and hypertension because of mineralocorticoid properties.
The phenotype depends on the degree or type of gene deletion or mutation and the resultant deficiency of the steroidogenic enzyme.
Two copies of an abnormal gene are required for disease to occur, and not all mutations and partial deletions result in disease.

159
Compiled by Obasi. D. C. Chinedu

The phenotype can vary from: clinically inapparent disease (occult or cryptic adrenal hyperplasia), a mild form of disease that is expressed
in adolescence or adulthood (non classic adrenal hyperplasia), severe disease that results in adrenal insufficiency in infancy with or without
virilization and salt wasting (classic adrenal hyperplasia).

Types: Five major enzyme deficiencies are clinically important:


- 21-Hydroxylase(21-OHD) - 11β-Hydroxylase (11β-OHD) - 17α- Hydroxylase(17α-OHD)
- 3β-Hydroxysteroid dehydrogenase(3β-HSD) - 20, 22 Desmolase deficiency(StAR)

More than 90% of cases of CAH are caused by a defect in the enzyme 21-hydroxylase (21-hydroxylase deficiency [21OHD]).

21- Hydroxylase deficiency: Depends on the severity of the enzyme deficiency, 21OHD is defined as:
Classic (severe form): early virilisation, characterized by subnormal cortisol. If enzyme deficiency is severe, low aldosterone production,
accumulation of cortisol precursors causing elevation of androgen levels (Androstenedione, Testosterone, DHT). Salt wasting: ~75% of
patients*, simple virilising.
Non-classic (mild form): Late onset virilisation. These patients maintain enough glucocorticoid and mineralocorticoid to escape diagnosis at
birth. The moderately elevated androgen production will eventually cause symptoms later in life.

11β-OHD: 5-10% of cases of CAH, Characterised by low plasma renin activity, Increased serum 11-deoxycortisol, Increased 11-Deoxy
corticosterone, Salt retention and hypertension, Hypokalaemic alkalosis. The increased androgen causes virilisation of the female fetus.

17α-OHD: Occur with block in glucocorticoid and androgen production. Undervirilised male with female genital appearance in both sexes.
Hypertension, Hypokalaemia.
Presentation often in later childhood or adolescence with genital ambiguity and hypertension

3β-HSD: Very rare disorder. Result in accumulation of DHEA which is converted to testosterone in peripheral tissues. Undervirilised male
with perineo-scrotal hypospadias and/or cryptorchidism. Or a slightly virilised female.
If severe, presents in the neonatal period. If partial, can present with androgenism in an adolescent girl

21-OH HYDROXYLASE: Excessive androgen production is the hallmark of this disorder.


In the severe classic form: Genital ambiguity is present in affected female infants. The phenotypic virilisation varies from simple
clitoromegaly, with or without partial fusion of the labioscrotal folds, to the appearance of a penile urethra.

Clinical features OF 21OH: The genitalia of a female born with the severe form of the disease may be indistinguishable from that of a
male. The important differentiating points are the absence of testes and presence of normal uterus and ovaries. The high
concentrations of androgens accelerate growth, resulting in tall and muscular features in early childhood. Affected children’s final adult
heights are compromised due to advanced bone age associated with premature epiphyseal closure.
Approximately 75% of patients who have classic CAH due to 21OHD have a deficiency of aldosterone (classic salt-wasting form). these
patients present with salt-wasting and acute adrenal crisis, often within the first few weeks after birth.(1-4wks)* Hyperpigmentation of the
skin creases and genitalia may be early signs of adrenal insufficiency.
Of note, affected infants initially demand frequent feedings, possibly due to dehydration or salt craving. Poor feeding is a late sign of CAH
and severe adrenal crisis.

Non classic type: Milder symptoms due to partial deficiency of 21OH enzyme. Genital ambiguity is not present at birth. Androgen excess
manifests later in life for both males and females. In the early ages, presentations include premature adrenarche and an advanced bone age. In
adolescent females, hirsutism, male-pattern baldness, oligomenorrhea or amenorrhea, and polycystic ovary disease may occur. In both males
and females, short adult stature, insulin resistance, and severe cystic acne are common.

Diagnosis: Relevant history and physical examination. Needs alertness for the possibility of CAH in all babies with diarrhoea, vomiting,
hypoglycaemia and increased B.P.
Diagnosis is confirmed by measurement of ACTH, Cortisol, Aldosterone, 17-OHP, Testosterone and urinary ketosteroids.

Prenatal diagnosis: Done by Chorionic villus sampling @ 8-12wks. Amniocentesis @ 18-20wks.


HLA typing. 17-OHP & androstenedione in amniotic fluid.
Neonatal screening: Permits early identification of newborns with CAH. Prevents salt wasting crisis in boys who are unrecognised @ birth.
Also recognize completely virilized girls with ambiguous genitalia who maybe mistaken for boys with cryptorchidism. Done by assay of 17-
OHPfrom heel blood samples on filter paper

Initial Important Laboratory evaluation: classic CAH


- Blood glucose, Serum electrolytes, Arterial blood gases/ serum PH, Cortisol, ACTH, 17-OHP, Pelvic USS, Karyotype

Other Investigations: Non classical CAH

160
Compiled by Obasi. D. C. Chinedu

- Bone age, DHEAS, Androstenedione


Short ACTH stimulation test for cortisol, 17OHP, 11-DOC @ 0, and 60mins (if 17-OHP<30nmol/l 60min after ACTH injection, 21OHD is
unlikely. CT-scan/MRI of adrenals (r/o adrenal tumor)

Treatment: Is life long. The goals of treatment include: Replace deficient cortisol secretion without signs of over or under treatment.
Suppress production of excess adrenal androgen and their precursors. To provide adequate available treatment for emergency needs during
stress(e.g illness). Acute medical therapy. Maintenance therapy

Modes of treatment:
Steroid replacement- 10-15mg/m2/day in 3dd is the Rx of choice. 2-3x dose during crisis. Supportive therapy when needed. Plastic
surgery for ambiguous genitalia at an early age. Genetic counselling. Psychological support.
Monitoring of treatment: Monitor adequacy of treatment: Clinically or lab
Prenatal treatment: Prenatal treatment of 21-OHD prevents in utero virilisation of female foetuses. Maternal treatment with
Dexamethasone 20U/kg/day in 3dd started as soon as pregnancy is recognised. This suppresses fetal ACTH secretion and prevent
overproduction of adrenal androgens by fetal adrenal gland.

New Trends in Treatment: A new approach therapy is combined use of 4 drugs


- Glucocorticoid: suppress ACTH, Androgen - Mineralocorticoid: reduce angiotensin 2 secretion
- Aromatase inhibitor: to slow skeletal maturation - Flutamide: androgen blocker to reduce virilization

Prognosis: is good with early adequate therapy. Patients with CAH have an Increased risk of developing mesodermal tumors (Osteogenic
sarcoma, Pulm liposarcoma, Uterine leiomyoma and brain tumors)
Late presentation may cause: Death of newborn with salt losing types and lack of stress doses of glucocorticoid during illness, trauma and
surgery. Psychological problems in females with ambiguous genitalia. Short stature and infertility.

Summary: CAH represents a family of disorders of the cortisol synthesis pathway in which more than 90% of cases are caused by 21OHD.
The hallmark of 21OHD is excessive androgen production that varies in degree among its different subtypes. The severe classic form of
CAH is the most common cause of ambiguous genitalia in a genetically female fetus. Experimental treatment of mothers with
dexamethasone in early gestation results in significantly less virilization in female fetuses affected with classic CAH. Newborn screening has
had an important impact on identifying the salt-wasting subtype in male infants and preventing adrenal crisis.
Lifelong treatment with steroids is required for most patients.

Adrenal Excess (Cushing syndrome (CS), Cushing Disease (CD))


Introduction: Cushing syndrome (CS)- any form of glucocorticoid excess. Cushing disease (CD)- hypercortisolism due to pituitary
overproduction of ACTH. Rare in childhood and adolescence, poses serious diagnostic challenge and is associated with morbidity. Early
diagnosis and treatment is vital for long-term outcome.
Most common cause is iatrogenic, with administration supraphysiological doses of exogenous GC. A careful history of medications,
including atypical topical treatments and dietary supplements, are vital in the assessment of CS.

Classification
ACTH-independent: can arise from
i. Exogenous glucocorticoid administration (tablets, nose drops, nasal spray, skin cream
ii. Adrenocortical tumour: adenoma or carcinoma
iii. Primary adrenocortical hyperplasia: primary pigmented adrenocortical disease (PPNAD), macronodular adrenal hyperplasia, McCune–
Albright syndrome (MAS).

ACTH-dependent from: Cushing’s disease (ACTH-secreting pituitary adenoma) or Ectopic ACTH syndrome.

Epidemiology: Adrenocortical tumours: 0.3–0.4% of neoplasms in children, mostly in infancy.


Ectopic ACTH syndrome: extremely rare, branchial and thymic.
75 – 80% of CS, almost always caused by a pituitary microadenoma (only 1 macroadenoma in 34 cases¹).
Commonest in adolescence (youngest patient = 6.2 years). Prepubertal period- predominantly male. Puberty-similar incidences of males and
females. Postpubertal- increasing predominance of females.
Clinical features: Growth failure with associated weight gain: one of the most reliable and earliest indicators of hypercortisolism in
children with CS. Clinical features may occur gradually over a period of time and go unrecognised by parents and carers (eg: change in facial
appearance, which is almost always present, can often be missed).
Accurate auxological assessment is vital and serial photographs can be helpful to document physical changes.
Obesity, initially generalized rather than centripetal. Moon facies, hirsutism & facial flushing (seen more in adults).
Striae, hypertension, muscular weakness, back pain, buffalo hump fat distribution, psychological disturbances (compulsive overachieving
behavior), acne and easy bruising. →→→Advanced Cushing disease.

161
Compiled by Obasi. D. C. Chinedu

The classic cushingoid appearance: usually not the initial picture seen.
Recently: comparison of height and BMI, # simple obesity (height ↑) and CD (height ↓)¹.

Diagnostic Approach:
CS: biochemically characterize by the loss of normal feedback suppression of the HPA axis and loss of the normal circadian rhythm of
cortisol secretion.
The following tests, particularly when combined, have high sensitivity and specificity.
i. Urinary free cortisol excretion (24 h urine collection) daily for 3 times, high sensitivity but relatively low specificity.
ii. Serum cortisol circadian rhythm study (09.00 h, 18.00 h, midnight [sleeping]), normal should be <50nmol/l.
iii. Low-dose dexamethasone suppression test (LDDST): Dose: 0.5mg 6 hourly [09.00h, 15.00h, 21.00 h, 03.00h]x 48h. Dose for patients
weighing less than 40 kg: 30µg/kg/day. Serum cortisol measured at 0 and 48 h (or 0, 24h and 48h).
Results: Normal individuals will suppress their serum cortisol levels to < 50 nmol/l by 48 hours. A small proportion of patients with CD
suppress normally during LDDST but patients with CS due to other etiologies tend not to.

Salivary cortisol: Salivary & sweat glands excrete only free cortisol. Samples can be collected at home in a non-stressful condition.
Cortisol is metabolized to cortisone in salivary glands. Best method to screen cushing syndrome

Treatment:
Treat the underlying cause: Resection of the primary lesion -adrenal tumour, lung tumour or PPNAD. Selective removal of the pituitary
lesion. Tapering and gradual withdrawal of steroids
Medical therapy: Ketoconazole. Metyrapone-to lower serum cortisol levels, short-term measure, not for long term therapy.

Summary: Pediatric CS is a challenging condition to manage and requires a multidisciplinary team (surgeons, pediatric endocrinologists,
biochemists, radiologists, oncologists and radiotherapists).
Weight gain with associated growth failure is always abnormal in childhood and requires investigation.

162
Compiled by Obasi. D. C. Chinedu

INTRODUCTION TO DSD
Dr. Ugo Chikani MBBS, FMCPaed

Outline
1) Introduction 2) definition 3) Classification 4) Syndromes associated with DSD
5) Management 6) Conclusion

Introduction: Disorders of Sex Development formerly termed Intersex conditions are seen in infants who are born with ambiguous or
abnormal genitalia and may have indeterminate phenotypic sex. DSDs are referred to as congenital conditions in which the development of
the chromosomal, Gonadal or the Anatomical sex is atypical.

Definitions: DSDgender dysphoriasexual orientation. The different types of ”sex“:


a) ”prenatal echography sex“/”delivery room sex“ b) genetic sex: 46 XX or 46 XY
b)
gonadal sex: presence of testes or ovaries c) phenotypic sex: 1 ary/2ary sexual characteristics
c) gender identity/role: psychosocial component
Ambiguity = difficulty in determining sex of baby at birth = ~ 1: 10,000 live births

Embryology of Sex Development: Adequate understanding of normal and abnormal sexual diffrenciation is essential in understanding
DSDs. Chromosomal sex determines Gonadal sex which determines the phenotypic sex. The type of gonads present determines the
differentiation or regression of the internal ducts and ultimately the phenotypic sex.
At the 2nd month of fetal life, the indifferent gonad is guided to develop into a testis by the genetic information on the short arm of the Y
chromosome. This is the Testis-Determination Factor (TDF), a kilobase pair sequence in the 11.3 sub band of the Y chromosome. This area
is called the Sex determination Region on the Y chromosome (SRY). When this region is absent or altered the indifferent gonad develops
into an ovary and phenotypic female sex

SRY: Sex determining gene on the Y chromosome. The single most important gene to direct gonadal differentiation into a testis

Nature 1991, 351:117-21: Male development of chromosomally female mice transgenic for Sry.
Koopman P, Gubbay J, Vivian N, Goodfellow P, Lovell-Badge R.
The initiation of male development in mammals requires one or more genes on the Y chromosome. A recently isolated gene, termed SRY in
humans and Sry in mouse, has many of the genetic and biological properties expected of a Y-located testis-determining gene. It is now
shown that Sry on a 14-kilobase genomic DNA fragment is sufficient to induce testis differentiation and subsequent male development when
introduced into chromosomally female mouse embryos.

Revised nomenclature
In 2006, the LWPES and the ESPE published proposed changes to the previously used nomenclatures in DSDs, this was done to reflect
advances in understanding the pathophysiology of these disorders while being sensitive to the needs and concerns of the patients affected by
them
Previous Proposed
Intersex Disorders of sexual development (DSD)
Male pseudohermaphrodite 46,XY DSD
Under virilisation of an XY male
163
Compiled by Obasi. D. C. Chinedu

Under masculinisation of an XY male

Female pseudohermaphrodite 46,XX DSD


Over virilisation of an XX female
Masculinisation of an XX female
True hermaphrodite Ovotesticular DSD
XX male or XX sex reversal 46,XX testicular DSD
XY sex reversal 46,XY complete gonadal dysgenesis

Classification of DSDs: DSDs are categorized under main subgroups according to karyotype:
1. XX 2. XY 3. Sex Chromosomes

A. 46, XX DSD
1.Disorders of gonadal(ovarian) development: (a)Ovotesticular DS. (b)Gonadal dysgenesis (c)Testicular DSD
2.Adrogen Excess: (a)Fetal: Congenital adrenal hyperplasia, 21-hydroxylase deficiency, 11-hydroxylase deficiency
(b) Fetoplacental: Aromatase deficiency (c)Maternal: Luteoma, Exogenous(iatrogenic)

B. 46, XY DSD
1. Disorders of gonadal(testis) development: (a)complete gonadal dysgenesis(swyers syndrome)
(b) partial gonadal dysgenesis (c)gonadal regression (d)ovotesticular DSD
2. Disorders in androgen synthesis or action:
(a) Androgen biosynthesis defect: 17-hydroxysteriod dehydrogenase deficiency, 5alpha reductace deficiency
(b) Defect in androgen action: CAIS, PAIS. (c) LH receptor defects: leydig cell hypoplasia,
(d) Disorders of AMH and AMH receptor (persistent mullerian duct syndrome)

C. Sex chromosome DSD: 1. 45,X: Tuners syndrome and variants 2. 46,XXY: Klinefelter syndrome and variants
3. 45,X/46XY: Mixed gonadal dysgenesis, ovotesticular DSD 4. 46,XX/46,XY: chimeric, ovotesticular DSD

46,XX DSD
1. Disorders of gonadal development
Gonadal Dysgenesis: A type of female hypogonadism in which no functional ovaries are present to induce puberty.
With nonfunctional Streak Ovaries, they have low Estrogen levels with high levels of FSH and LH.
2. Androgen Excess
Congenital Adrenal Hyperplasia (CAH): 50% of cases (~ 1:30,000 live births). autosomal recessive. 90-95%: 21-hydroxylase. Rare: 3ß-
HSDehydrogenase or 11ß-hydroxylase.
Cortisol is an adrenal steriod hormone required for normal endocrine function, its production starts in the second month of fetal life.
Inefficient cortisol production is the hallmark of most forms of CAH as ACTH level rises inducing overgrowth and over activity of
the steroid producing cells of the adrenal cortex at birth which may result in mineralocorticoid and sex hormones activities.

Case: S.W 3yr old female, who was managed for infective AGE 10/12 ago but was also found to have ambigious genitalia with a large
phallus, no vaginal orifice, also was found to be hypertensive.

Case 2: One year old raised as a female. Ambiguous genitalia. Most likely 46xx DSD ? CAH
46, XY DSD
1. Disorders of gonadal development
Complete gonadal dysgenesis (swyers syndrome): Estimated incidence: 1 in 80,000 – 100,000 births.
Is thought to be due to a deletion or mutation involving the SRY chromosome.
Clinical features: Female appearance (external genitalia, but the uterus and fallopian tubes are underdeveloped).
No functional gonads (ovaries or testes), instead they have streak gonads. Streak gonads may undergo neoplastic changes (gonadoblastomas
and dysgerminomas), should be removed shortly after diagnosis, regardless of age (risk: 20% - 30%). Height (helpful in distinguishing patients
with pure 46,XY gonadal dysgenesis from those with 45,XO/46,XY mosaicism but without the classic features of Turner syndrome).
They classically present as sexually infantile phenotypic females with primary amenorrhoea.

CASE 3: A 3 Year old , raised as a girl. First child of non-consanguineous parents. Birth history-
unremarkable. No hx suggestive of cortisol deficiency at birth.
No use of medications. Palpable gonads.
NO VAGINAL OPENING, Urethral opening beneath. Phallus of 2cm
No Mullerian structures are absent.
Most likely 46XY DSD. PAIS, 5alpha reductase, 17bHSd
164
Compiled by Obasi. D. C. Chinedu

Case 4: 14 year old raised as a female.


No breast, no vaginal opening
Normal bp
u/s –no uterus, hypoplastic gonads /testes in the abdomen
Most likely 46XY DSD gonadal dysgenesis
PAIS, 5alpha reductase deficiency, 17bHSd

2. Partial gonadal dysgenesis: Characterized by: Genital ambiguity due to variable degrees of testicular dysgenesis;
Absence of syndromic picture; and 46,XY karyotype

2. Disorders in androgen synthesis or action


A. 17β-hydoxysteroid dehydrogenase deficiency: HSD17β3 gene: expressed in the Leydig cells of the testes and catalyzes conversion of
androstenedione to testosterone. Loss of its function or mutation: autosomal recessive → testosterone deficiency and undervirilization of
46,XY fetuses.
Clinical features: External female genitalia with a blind-ending vaginal pouch.
A Variable degrees of ambiguity: palpable gonads in the labio-scrotal folds, labio-scrotal fusion and hypospadias
Internal male ducts present
B. 5α-reductase deficiency: SRD5A2 gene: expressed in androgen target tissues (prostate, primordial external genitalia tissue). Converts
testosterone to DHT (responsible for virilization of the external genitalia).
Clinical features: - Small phallus - Urogenital sinus with perineal hypospadias
- Blind vaginal pouch
Internally: partially differentiated male structures (Wolfian), female (Müllerian) ducts are absent.
Reference to case 3
Lab: Testosterone ↑, DHT ↓, consistent elevated of basal or stimulated testosterone-to-DHT ratios

Sex reversal: - 5alpha reductase - 17bHSD

ANDROGEN INSENSITVITY SYNDROME: When a person who is genetically male is resistant to male hormones as a result the person
has some or all the physical traits of a woman.
AI: X-linked, recessive, caused by mutations in the androgen receptor gene (proximal long arm X chr).
Affects 1 in 20,000 – 64,000 live male births. Infertile.

Types: Complete AIS: prevents male traits from manifesting


Partial AIS: different numbers of male traits present

Clinical aspects of androgen insensitivity


Genotype 46, XY
Inheritance X-linked recessive, mutations of AR gene
External genitalia female - ambiguous - infertile man
Wolffian ducts absent - rudimental - normal
Mulleriani ducts absent
Gonads testis (cryptorchidism is sometimes present)

Complete androgen insensitivity syndrome (CAIS): Morris´ syndrome


Clinical features: External genitalia resembles normal female genitalia
- Inguinal or labial gonads
- Inguinal herniae (discovered sometime at the time of inguinal hernia repair)
- 1% - 2% of girls with bilateral inguinal herniae have androgen insensitivity
- Primary amenorrhea (adolescence)

Image beside: 46, XY individual with complete androgen insensitivity.


Note ample breast development but absent/sparse auxillary and public hair.

165
Compiled by Obasi. D. C. Chinedu

Partial androgen insensitivity syndrome (PAIS):


Clinical features: Ambiguous genitalia with perineo-scrotal hypospadias, micorphallus, bifid scrotum, and testes located in the srotum or
undescended. - Absence of Müllerian structures.
Difficulties to make a diagnosis in very mild forms where external genitalia are almost normal.
In two studies on azoospermic or oligoospermic patients, 9 out of 22 and 6 out of 9 respectively have been found to be affected by partial
AIS.

Case 5: A 33 year old raised as a female


No breast, abdominal testes
Phallus of about 5cm, no vaginal opening
Confirmed case of PAIS
On follow up with paed endo$ surgeons

Ovotesticular D.S.D. : Pathogenesis obscure, except if XX/XY chimera. Most have a 46, XX karyotype
Dx of XY ovotesticular DSD debatable (Mendonca et al, Clin Endo 70: 173-187, 2009)
Rare, except in Bantu. Gonads often asymmetrical. Gonadal malignancy not increased
1 ovary and 1 testis, or ovotestis (2 in 1).
Müllerian structures on the side where there is no testis and on the midline.

MANAGEMENT: Multidisciplinary approach


- Paediatric endocrinologist - Psychologist/ Psychiatrist - Paediatric urologistist
- Paediatric gynaecologist - Genetics / Genetic counselling - Nursing and social worker

46 XX/46XY
Left side: Right side:
• Hemiuterus No uterus or tube
• Fallopian tube Normal vas and epididymis
• Ovary Ovotestis
Perineal hypospadias

Step 1: If one or two testes are palpable: less urgent, but more difficult!
Differential diagnosis: - gonadal dysgenesis (46,XY/45,X)
- androgen insensitivity syndrome - 5-reductase deficiency
- testosterone synthesis defects

166
Compiled by Obasi. D. C. Chinedu

Check history again


Pregnancy: -acne or hirsutism? medications (progestins)?
Family history: -Consanguinity, genital anomalies, infertility, spontaneous abortions, stillbirths or neonatal deaths, pubertal anomalies
(asymmetrical axillary hair)

Clarify anatomy
Physical examination: - Urethral opening - Vaginal opening. (using fine, soft probe)
Cysto-vaginoscopy: - Find vaginal opening - Is there a portio uteri?
Ultrasound (by skilled personel!): - Gonads? - Uterus? (size, shape)
Urethro-genitogram (x-ray with contrast): - Vagina? Openings? Presence and length of urogenital sinus’
MRI
Laparoscopy (mini lap)
Clinical diagnosis (Prader’s stages of virilization)

167
Compiled by Obasi. D. C. Chinedu

Investigations and surveillance


 First line exams: - 17OHP - Testosterone, LH, FSH, AMH
 Pelvic ultrasound: THE UTERUS IS LARGE IN NEWBORNS: IS IT SEEN?. - Gonads and adrenals
 FISH or PCR for SRY (Barr bodies)
 Karyotype (can be obtained in 3 days)
 Plasma testosterone before 24h
 Plasma 17 OHP (day 3), T, Δ4AD, DHEAS

Weigh daily, plasma & urine Na+/K+ (end of 1st w)
 Plasma testosterone, precursors : (17-OHP, DHEAS, androstenedione) and metabolites (DHT)
 LH, FSH, PRA, aldosterone, cortisol, AMH?
 -hCG stimulation test (T, precursors, DHT)
 DNA to search for mutation in genes encoding receptors, enzymes or transcription factors

46, XY DSD: Classical approach: precursor/product ratio. Modern approach: mutation detection
Reason: - 2 isoenzymes for 3ßHSD, 5R - 3 isoenzymes for 17ßHSD
- Only the one in steroidogenic tissues or genital skin has a mutation
- precursor/product ratio also reflects activity of the other isoenzymes, which are normal
If salt loss: 3ßHSD deficiency

Other diagnostic tests:


- For diagnosis and to orient surgeon: Genitogram (Foley catheter in urogenital sinus, injection of contrast)
Rarely needed: - Pelvic CT scan, MRI - Gonadal biopsy

General Concepts of Care: Optimal clinical management of individuals with DSD should comprise the following:
1. gender assignment must be avoided before expert evaluation in newborns;
2. evaluation and long-term management must be performed at a center with an experienced multidisciplinary team
3. All individuals should receive a gender assignment
4.Open communication with patients and families is essential, and participation in decision-making is encouraged
5. patient and family concerns should be respected and addressed in strict confidence. The initial contact with the parents of a child with a
DSD is important, because first impressions from these encounters often persist.

Principles of managing babies with DSD (1)


Minimising physical risk to the child: - preventing adrenal crisis - Reducing risk of gonadal malignancy
- Genitourinary obstruction
Minimising pyschosocial risk to the child: - wrong gender assignment leading to later gender dysphoria
- Risk that baby will be unacceptable to parents leading to impaired bonding
- Risk of socio/cultural disadvantages to the baby - Risk of social isolation and embarrassment

"Sex of rearing"
Factors to consider: Anatomy, diagnosis, age at diagnosis, fertility potential, gonadal and genetic sex, parents, socio-cultural context,
androgenization of fœtal brain
Prevailing opinion-female sex of rearing advised if:
 Newborn girls with CAH regardless of degree of masculinization
 Ambiguous newborns with mixed gonadal dysgenesis (uterus present assisted reproduction possible)

168
Compiled by Obasi. D. C. Chinedu

46, XY DSD
 If sensitivity to androgens normal (testicular dysgenesis, enzymopathies): in principle, male
 If partial androgen resistance:
 «Classical» attitude: easier to create vagina?
 Few studies of sexual function in patients with artificial vagina
 Examples of request for change to male sex as adult
 Alternative: reconstructing the penis?
 Gynecomastia ++, very disturbing at adolescence

Medical management
Glucocorticoids: These agents have anti-inflammatory properties and cause profound and varied metabolic effects. Children with congenital
adrenal hyperplasia (CAH) require corticosteroid replacement for survival. Replacement also reduces the production of corticotropin and,
therefore, the overproduction of androgens

Sex-Steroid Replacement: Hypogonadism is common in patients with dysgenetic gonads, defects in sex-steroid biosynthesis, and resistance
to androgens. Patients with PAIS may require supraphysiologic doses of testosterone for optimal effect. Hormonal induction of puberty
stimulates replication of normal pubertal maturation to induce secondary sexual characteristics, a pubertal growth spurt, and optimal bone
mineral accumulation.
Intramuscular depot injections of testosterone esters are commonly used in males; Patients with 5-alpha-reductase deficiency who are raised
as male, testosterone or dihydrotestosterone (DHT) therapy may increase penile length.
Two doses of testosterone ester (125 mg per dose), given 3 weeks apart has been used in prepubertal children and should be considered prior
to hypospadias repair.
Females with hypogonadism require estrogen supplementation to induce pubertal changes and menses. A progestin is usually added after
breakthrough bleeding develops or within 1 to 2 years of continuous estrogen

Follow-up in childhood; It is essential that all infants are followed up both in the short and long term.
Monitor growth, assess the outcomes of surgery and to facilitate transition at the appropriate time.

Surgical management of DSD: The surgeon has a responsibility to outline the surgical sequence and subsequent consequences from infancy
to adulthood. Only surgeons with expertise in the care of children and specific training in the surgery of DSD should perform these
procedures.
Emphasis is on functional outcome rather than a strictly cosmetic appearance. It is generally felt that surgery that is carried out for cosmetic
reasons in the first year of life relieves parental distress and improves attachment between the child and the parents (Rink RC, Adams MC.
Feminizing genitoplasty: World J Urol 1998;16:212–18)

Surgical management:
 In a virilized female, Surgery should only be considered in cases of severe virilization (Prader III–V) the surgical procedure is termed
feminizing genitoplasty and includes vaginoplasty and clitoroplasty
 Undervirilized males typically have hypospadias requiring surgical reconstruction
 It is anticipated that surgical reconstruction in infancy will need to be refined at the time of puberty.
 Vaginal dilatation should not be undertaken before puberty

Feminizing Genitoplasty: Several techniques


according to anatomy
- phallectomy - clitoroplasty
- labioplasty - vaginoplasty
Complications: stenosis, dyspareunia, fistulae,
incontinence if sphincter damaged

Masculinizing genitoplasty
- Phalloplasty (chordee+hypospadias repair
- Orchidopexy
- Scrotoplasty (for bifid scrotum)
- Mastectomy (if gynecomastia develops)

Before Surgery After Surgery

169
Compiled by Obasi. D. C. Chinedu

Psychosexual outcome: 46,XX DSD with CAH


 No dramatic increase in psychological morbidity, good social adjustment and no deficit in self esteem
 The claim that clitoral surgery has no long-term effect on sexual function is wrong

Psychosexual outcome: 46, XY DSD raised as males or females


 half of the patients were neither well informed about their medical and surgical history nor satisfied with their knowledge
 Overall dissatisfaction with sex life:
Female (N=30) Male (N=10) Fem. Ctrls

41.4 % 50 % 11.5 %

What to do with testes in 46, XY children with female phenotype?


Streak gonads are removed at diagnosis
Complete AIS: gonadectomy at diagnosis or after puberty (discussed)
Partial AIS: gonadectomy at diagnosis, or in early puberty, to prevent further virilization during puberty and cancer later on (in patients
reared as females)
Short-term testosterone therapy is used in 46,XY infants with DSD to investigate the sensitivity of external genitalia to androgens (if
possible before 6 months of age).
Testosterone depot: 100 mg/m2 IM every month, for 3 consecutive months.
Usually in small infants: 25 mg IM/dose, but a higher dose is proposed by some authors (50-100 mg/dose).
Topical DHT is associated.

A mother’s perspective: With such a diagnosis, words or gestures that hurt are common. One of the nurses to whom I announced the first
name that we had chosen said these words that shocked me: “so you have decided that it would be a girl“. Shock, too, to see that our daughter
had a small white ID card, while all the other babies had a blue or pink one, or to hear a pediatrician talk about a "small penis“.

Conclusions: Knowledge of embryology. Systematic approach for prompt Dx. Interdisciplinarity, humility, collegiality. One contact person
for the parents. Consider the situation as urgent
A decision needs to be made: "3d sex" option not acceptable by parents.

170
Compiled by Obasi. D. C. Chinedu

HYPERTENSION IN CHILDREN
Prof HU Okafor

Outline
 Definition
 Classification
 Epidemiology
 Etiology and risk factors
 Pathophysiology
 Clinical features of HTN in Chld.
 Management
 Hypertensive Crisis

Learning objectives
To Define hypertension. Identify hypertension (BP measurement). Causes of hypertension
Classify hypertension. Treatment of hypertension

Definition: Hypertension is defined as average SBP and/or diastolic BP (DBP) that is ≥95th percentile for gender, age, and height on 3
occasions. Charts and Nomograms based on auscultatory methods are available. Hypertension maybe primary or secondary.
Pre hypertension/elevated BP in children is defined as average BP >90th percentile but <95th centile.
A patient with BP levels >95th percentile in a physician ’s office or clinic, but normotensive outside a clinical setting has “white-coat
hypertension.” Ambulatory BP monitoring (ABPM) is usually required to confirm this diagnosis. As with adults, adolescents with BP levels
>120/80 mm Hg should be considered pre-hypertensive

Classification of Hypertension:
- Pre-Hypertension - Stage 1 HTN - Stage 2 HTN
- Hypertensive emergencies - Hypertensive Urgency

Staging of HTN in children: HTN in children is staged as in adults. Elevated BP b/w >90th & <95th centile
Stage 1 HTN is diagnosed if a child’s BP is greater than the 95th percentile but less than the 95th centile plus 12mmHg, or 130/80 to
139/89mmHg (whichever is lower).
Stage 2 HTN is diagnosed if a child’s BP is greater than the 95th percentile plus 12mmHg or ≥ 140/90 mmHg.
If the systolic and diastolic pressures give rise to a discrepancy with respect to classification, the child’s condition should be categorized
by using the higher value.

Do Children have hypertension (HTN)? Epidemiology


3-9% globally; Canada 1-2%; 20% of Pediatric HTN is attributable to CKD. In Africa 0-22.3%.Various definitions in the studies. There has
been an increase in the prevalence of childhood high blood pressure, including both HTN and persistently elevated BP. In the US prevalence
is higher in Non-Hispanic Afro-Americans and Hispanics. Higher rates among adolescents than younger children. 0.2-3% prevalence in
neonates /infants and 0.81% incidence in infants.

Risk Factors: Obesity, Sleep disordered breathing (Obstructive sleep apnea). Positive family history, Prematurity, CKD, Male gender.

Causes of HTN in children:


Primary: this is now the predominant type seen in the USA. General characteristics include; Older age (> 6yrs), Positive family history
(parent and/or grand parent), Overweight and/or obesity.
Note: DBP elevation appears to be more predictive of secondary HTN; whereas Systolic HTN appears to be more predictive of
primary HTN.

Causes of secondary HTN: Vascular, Renal /Reno vascular eg segmental renal hypoplasia(Ask Upmark kidney -curable).
Drug induced e.g. steroids etc, Malignancies, Endocrine causes.
Renal and Reno vascular disease: most common. Renal parenchymal disease and structural abnormality accounted for 34 to 79% of
patients while Reno vascular disease(RAS) was present in 12% to 13%. In children < 6years of age with HTN; 63% to 74% had renal
disorders.

Cardiac abnormalities including coarctation of the aorta (CoA). Even after early and successful repair, children with CoA remain
hypertensive or develop HTN with a reported prevalence ranging from 17% to 77%. In children with a history of CoA, approx. 45% have
Masked HTN at 1 to 14 years after repair.

Endocrine causes: accounts for a relatively small proportion of children with a prevalence ranging from 0.05% to 6%. Examples include;
Congenital adrenal hyperplasia ;17α & 11ß hydroxylase deficiency.
Causes: Pheochromocytoma, Cushing’s syndrome, Adrenocortical carcinoma, Hyperthyroidism, Familial hyperaldosteronism,

171
Compiled by Obasi. D. C. Chinedu

Hyperparathyroidism, McCune Albright syndrome.

Neurofibromatosis (von Recklinghausen disease) Type 1 – NF-1;An increased incidence of HTN as high as 6.1% has been reported in
affected children

Environmental causes:
Heavy Metals: Cadmium, Mercury, phthalates, Lead. These are nephrotoxic and cause elevation of BP
Pharmacologic agents that ↑BP
OTC: decongestants, NSAIDs, Caffeine, herbal and nutritional supplements
Prescription drugs: steroids, stimulants for ADHD, TCAs,
Illicit drugs: amphetamines, cocaine

Causes of HTN in Neonates: Bronchopulmonary dysplasia, PDA, Intra ventricular hemorrhage, In dwelling umbilical artery catheters.
Antenatal steroids, Maternal hypertension & Reno vascular or renal pathology eg PKD

Pathophysiology: Blood pressure (BP) is determined by the balance between cardiac output and vascular resistance. A rise in either of these
variables, in the absence of a compensatory decrease in the other, increases mean BP, which is the driving pressure.
- Factors that affect cardiac output include the following: Baroreceptors, Extracellular volume, Effective circulating volume - Atrial
natriuretic hormones, mineralocorticoids, angiotensin. Sympathetic nervous syndrome
- Factors that affect vascular resistance include the following:
Pressors: Angiotensin II, calcium (intracellular), catecholamines, sympathetic nervous system, vasopressin
Depressors - Atrial natriuretic hormones, endothelial relaxing factors, kinins, prostaglandin E2, prostaglandin I2
Changes in electrolyte homeostasis, particularly changes in sodium, calcium, and potassium concentrations, affect some of these factors.
Under normal conditions, the amount of sodium excreted in the urine matches the amount ingested, resulting in near constancy of
extracellular volume. Retention of sodium results in increased extracellular volume, which is associated with an elevation of BP. By means
of various physical and hormonal mechanisms, this elevation triggers changes in both the glomerular filtration rate (GFR) and the tubular
reabsorption of sodium, resulting in excretion of excess sodium and restoration of sodium balance.
A rise in the intracellular calcium concentration, due to changes in plasma calcium concentration, increases vascular contractility. In
addition, calcium stimulates release of renin, synthesis of epinephrine, sympathetic nervous system activity.
Increased potassium intake suppresses production and release of renin and induces natriuresis, decreasing BP.

The complexity of the system explains the difficulties often encountered in identifying the mechanism that accounts for hypertension in a
particular patient. These difficulties are the main reason why treatment is often designed to affect regulatory factors rather than the cause of
the disease. In a child who is obese, hyperinsulinemia may elevate BP by increasing sodium reabsorption and sympathetic tone.

Evaluation: Perinatal history, Nutritional history, Psychosocial history, Family history


Thorough physical exam to exclude secondary causes. Anthropometry & BMI calculation, BP measurement.

BP Measurement in Children: Children > 3 years old who are seen in a medical setting should have their BP measured. <3yrs should be
measured in special circumstances. The preferred method of BP measurement is auscultation. Correct measurement requires a cuff that is
appropriate to the size of the child’ s upper arm.

Best BP Measurement Practices


For an auscultatory BP, the bell of the stethoscope should be placed over the brachial artery in the antecubital fossa, and the lower end of the
cuff should be 2–3 cm above the ante cubital fossa. The cuff should be inflated to 20–30 mm Hg above the point at which the radial pulse
disappears. Over inflation should be avoided.
Elevated BP must be confirmed on repeated visits before characterizing a child as having hypertension. Measures obtained by oscillometric

172
Compiled by Obasi. D. C. Chinedu

devices that exceed the 90th percentile should be repeated by auscultation.

BP measurement: Patient rested for 5 mins. Rt arm preferred and supported with cubital fossa at level of the heart.
Appropriate cuff size should have a bladder width at least 40% of arm circumference half way between olecranon and acromion
process. Bladder length 80-100% of arm circumference. So bladder width to length ratio is 1:2.
Stethoscope 2cm above cubital fossa superior and medial to it. Systolic BP-K1. Diastolic BP-K5
Serial measurements (at least 3) over weeks unless in severe HTN.

SOP for BP measurement in neonates: Measure with oscillometric device (standardized for pediatric use).
1.5h post feeding. Infant lying prone or supine. Appropriately sized cuff. Right upper arm. After cuff placement, wait 15mins. Infant should
be asleep or in quiet awake state. 3 successive BP readings at 2 min intervals.
The gold standard technique for BP measurement in neonates remains direct measurement by intra-arterial analysis of the pulse
pressure wave form.
Doppler assessment has been replaced by oscillometric devices.

BP measurement in children: Cuff sizes

Using BP Charts for children


Use standard height charts e.g. WHO or CDC Charts to identify child's height percentile for age.
Measure BP (SBP and DBP). Use correct gender chart
Find child’s age on left, follow horizontally to height centile column. Find values for the various centiles
BP<90th centile is normal.
≥90th but ≤95th centile= elevated BP; in adolescents BP>120/80 is considered elevated even if its <90 th centile.
Ambulatory BP measurement(ABPM): Indications: White coat HTN, CKD, DM.
Measures Mean arterial BP over 24hrs. Calculates BP load (degree of BP over upper limit).
BP measurement in infants: As in older children, use of the proper cuff size is important and has been determined in neonates to be a cuff
width to arm circumference ratio in the range of 0.45–0.55. Factors that affect BP readings in neonates:
Gestational age, body size, post conceptual age, prematurity, size for gestational age. Level of activity at the time

Investigations: Urinalysis, SEUCr, Lipid profile, Renal USS, HbA1c, LFT, FBS, Sleep study, Drug screen, TSH.
Radiological Investigations: Echocardiography, Imaging, Doppler, CTA & MRA.

STEPS TO TAKE
A. If the BP is normal, then no additional action is needed except routine measurements during well clinic visits if >3yrs. If the BP is at the
elevated level, life style interventions should be recommended which include healthy diet (DASH), exercise, sleep. Nutrition and/or weight
management referral should be considered.
Repeat measurement should be done after 6mo by auscultation

173
Compiled by Obasi. D. C. Chinedu

B. If ↑BP continues after 12mo and 3 auscultatory measurements, then Ambulatory Blood Pressure Monitor (ABPM) should be ordered if
available. At this point, diagnostic evaluation should be conducted.
Consider subspecialty referral esp. nephrology and cardiology

Treatment: can be Non-Pharmacological or Pharmacological


Non-Pharmacological treatment:
Weight reduction
Dietary: low salt, low calories, low fat, avoid potassium depletion.
Exercise: aerobics, yoga
Sleep therapy-adenoidectomy

Indications for pharmacological treatment: Symptomatic hypertension, Secondary hypertension, Hypertensive target organ damage,
Diabetes. Persistent hypertension non responsive to non pharmacological measures.

Drug Treatment:
Approved drugs include: ACE inhibitors, Calcium channel blockers, Diuretics, Vasodilators (for recalcitrant cases) & Beta blockers

Best Practice Guideline


In hypertensive children and adolescents who have failed lifestyle modifications (particularly those who have LV hypertrophy on
echocardiography, symptomatic HTN, or stage 2 HTN without a clearly modifiable factor [eg, obesity]), clinicians should initiate
pharmacologic treatment with an ACE inhibitor, ARB, long-acting calcium channel blocker, or thiazide diuretic.
Flynn JT, Kaelber DC, Baker-Smith CM, et al. Clinical Practice Guideline for Screening and Management of High Blood Pressure in
Children and Adolescents. Pediatrics.2017;140(3):e20171904
Hypertensive Crisis: Hypertensive crisis is defined as a severe elevation in BP and classified into:
Hypertensive emergencies: severely elevated BP with end organ dysfunction.
Hypertensive urgency: severely elevated BP without end organ dysfunction.

Hypertensive Emergencies: Hypertensive encephalopathy is a form of hypertensive emergency characterized by an abrupt or prolonged
elevated BP that overcomes the auto regulatory capacity of the cerebral vasculature.
Pathophysiology: children with very high BP levels could have early development of cardiovascular changes.
In the acute phase of hypertensive crisis, rapid increases in systemic vascular resistance could be precipitated by increases in the circulating
vasoconstrictor substances, e.g. norepinephrine, angiotensin II, or anti-natriuretic hormone.
Arteriolar fibrinoid necrosis may induce severely elevated BP precipitating endothelial damage with resultant end organ ischemia. Ischemia
could trigger the further release of vasoactive substances causing further vasoconstriction and myointimal proliferation. Activation of the
renin-angiotensin system could also be highly involved.

Clinical features: No Peculiar features:


symptoms/signs reflect target organ dysfunction & may include;
Headaches
Vomiting
altered mental status
Seizure
visual disturbances
Stroke
Features of left ventricular failure.

Management: Cases of hypertensive crisis are usually managed in the ED.


Ideally after initial stabilization the patient should be referred to either the cardiologist or nephrologist.
Hypertensive urgencies are managed with oral short acting anti hypertensive while a hypertensive emergency should receive initial parenteral
antihypertensive then switch to oral.

Conclusion: The goal in management of HTN is to prevent end organ damage. For parenteral treatment of hypertensive crisis the goal is
to reduce to < or 25% in the first 8 hrs.

174
Compiled by Obasi. D. C. Chinedu

CENTRAL NERVOUS SYSTEM TUMOURS


DR ANN EBELE ARONU

Aim: To improve learners’ knowledge and competence with regards to Paediatric Brain Tumours.
Objectives:
Student would have acquired basic knowledge on the symptoms and signs suggestive of CNS tumours in children.
Have competence related to evaluation and management of Paediatric brain tumours.

Introduction: CNS tumours are the third most common type of neoplasm in children. Exceeded only by leukaemias and lymphomas.
Account for 25% of all paediatric malignancies. Metastatic tumours are not common in children as they are in adults. Peak age incidence is
5yrs to 10yrs, though tumours may occur at any age. Primary spinal tumours are rare.
Majority of intracranial tumours occuring in the first year of life are supratentorial in origin and account for 2/3 of all cases. Tumours in
young children are often large and highly malignant with a poor overall prognosis.
About 75% of childhood brain tumours are glial in origin: astrocytoma, medulloblastoma, ependymomas, glioblastoma. Neonatal
brain tumours are rare, mostly supratentorial and mainly teratomas.

Classification:
Pathological classification: WHO internationally agreed classification based on the tissue of origin
Primitive neuroepithelial cells: Astrocytes- astrocytoma, oligodendrocytes- oligodendroglioma, ependymal cells- ependymoma.
Meninges- meningioma Nerve sheaths- schwannoma
Tumours of maldevelopmental origin- Craniopharyngioma, epidermoid/dermoid cysts

Classification according to site:


Cerebral hemispheres- meningioma, dermoid/epidermoid cyst , astrocytoma, glioblastoma.
Hypothalamus- astrocytoma
Ventricular system- choroid plexus papilloma, colloid cyst.
Sellar/suprasellar region- craniopharyngioma, pituitary adenoma.
Pineal region- pineal cytoma/blastoma, astrocytoma.
Posterior fossa- medulloblastoma, astrocytoma.
Skull base and sinuses- carcinoma (nasopharyngeal, sinuses)

Supratentorial tumours and infratentorial tumours classification:


Supratentorial tumours (40%): Craniopharyngioma (commonest), Cerebral astrocytoma, Gliomas of the optic pathways and
hypothalamus. Ependymoma, Papilloma of the choroid plexus of the lateral ventricles.
Infratentorial tumours (60%): Cerebellar astrocytoma, Medulloblastoma, Brainstem glioma and Ependymomas of the 4 th ventricle

Tumours of the cerebellum account for 40%. Brain stem gliomas and ependymomas of the 4th ventricle account for 15% of all CNS tumours.
Hydrocephallus is a common feature of posterior fossa tumours with the exception of brainstem glioma. Virtually all children with
cerebellar tumours have hydrocephalus.
Brainstem gliomas manifest insidiously-ataxia, long tract signs and cranial neuropathies

Epidemiology/Pathogenesis: Incidence in US is 47/1 million chn under 20yrs and 52/1 million chn ≤ 5yrs.
For 0-14yr age gp, common tumours are; Pilocytic Astrocytoma(PA), Medulloblastoma/PNETs
For 15-19yr gp, common tumours are; Pituitary tumours and PA
Primary location of tumour is age-related.
1st yr of life: supratentorial predominate (choroid plexus complex, Teratomas)
1-10yrs- Infratentorial predominate (PA,Medulloblastoma)
After 10 yrs- Supratentorial commoner (diffuse astrocytomas)

Aetiology: Cause is unknown but some predisposing factors are recognized:


Cranial irradiation- increases incidence of astrocytoma, meningioma.
Immunosuppression- increases incidence of lymphoma and lymphoreticular tumours.
Neurofibromatosis- optic glioma, meningioma.
Tuberous sclerosis- subependymal astrocytoma

Clinical features: Summarized under 2 major headings: 1.Mass effects. 2.Focal damage.
Supratentorial tumours
Mass effect: - Signs and symptoms of raised intracranial pressure and brain shift. -Tentorial herniation.
Focal damage: - focal epilepsy -Disturbed function (lateralized deficits), (Higher cortical dysfunction, Cranial nerve damage (I-VI))
- Focal motor deficits - Focal sensory changes - Language disorder,
Reflex asymmetry
Optic pathway tumor: visual disturbances
Suprasellar/3rd vent: neuroendocrine deficits such as: dev of obesity, abnormal linear growth velocity, DI, galactorrhea, precocious/delayed
175
Compiled by Obasi. D. C. Chinedu

puberty, hypothyroidism. Diencephalic Synd.

Infratentorial tumours:
Mass effect: -Signs and symps of raised intracranial pressure. -Tonsillar herniation
Obstruction to CSF outflow-hydrocephalus.
Focal damage: -Disturbed function: Brainstem/Midbrain dysfunction & Cerebellar dysfunction
Brainstem and midbrain dysfxn: Gaze palsy, UMN deficits, Cranial nerve damage(III-XII).
Cerebellar dysfunction: disorders of equilibrium, gait, coordination. Torticolis (tonsillar herniation), Blurred vision, diplopia, nystagmus.

Diagnosis: Evaluation is emergency. Diagnosis is based on Complete history, P/E (Ophthalmologic, neurologic) and neuroimaging.
Investigation
1. Neuroimaging:
Skull X-ray: Calcification (meningioma, craniopharyngioma). Osteolytic lesions (metastatic carcinoma)
-Signs of raised intracranial pressure: Sutural diastasis, ’Beaten brass’ appearance, Erosion of posterior clinoids.
CT scan, MRI- Pituitary/suprasellar, optic pathway, infratentorium)
Others are: Angiography, Isotope scanning.
2. Neuro-Endocrine Dysfxn Evaluation: midline tumours, Pituitary/suprasellar, optic chiasmal region
3. CSF Analysis: Beta-human chorionic gonadotropin, Alpha- fetoprotein. For germ cell tumours common in suprasellar and pineal regions.
Cytology for tumours that spread to leptomeninges eg Medulloblastoma, Ependymomas, germ cell tumours.

Differential diagnosis:
Vascular: Giant aneurysm, A-V malformation
Trauma: Haematoma, contusion.
Infection: abscess, tuberculoma, sarcoidosis, encephalitis.
Cysts: arachnoid cyst, parasitic (hydatid)

Treatment: Treatment modalities include: - Steroid therapy (dexamethasone) - Surgery - Radiotherapy


- Chemotherapy - Monoclonal antibodies
Generally, treatment is multimodal.

CRANIOPHARYNGIOMA: Most common tumour of the sellar and suprasellar region in childhood. 7-10% of childhood tumours. Age gp
5-10yrs. Arises from remnants of the Rathke’s pouch. Often large cystic masses separated by connective tissue (solid and cystic component).
Slow growing tumour, symps and signs insidious.
More than 50% of them get calcified.

Clinical features: Depend on site and size of tumour. Raised ICP (intracranial mass and/or CSF obstruction at foramen of Munro. Mild to
severe dementia (frontal lobe extension)
Optic atrophy, bitemporal hemianopia (optic chiasma involvement)
Panhypopituitarism, pituitary dwarfism, delayed sexual maturation, DI (hypothalamic and pituitary involvement).

Investigation: Skull X-ray- calcification above or within the pituitary fossa in most children and 25% of adults.
CT scan- may show calcifications. MRI- solid tumour with cystic structures.
Pit. Function studies (demonstrates need for hormone repalcement)

Treatment: Surgery- cure for small tumours. Radiotherapy- Surgery and Radiotherapy for large tumours.
No place for chemotherapy.

Prognosis: Significant morbidity from visual loss/neuroendocrine complications.

MEDULLOBLASTOMAS: Primitive neuroectodermal tumours. Occur mainly in childhood, with a second peak in adulthood. Usual site of
origin:
-Childhood---cerebellar vermis in the midline-growth into 4th ventricle.
-Later life----more lateral in the cerebellar hemisphere.
A midline tumour. Readily disseminates via the subarachnoid space. Rapidly growing and seeds along the entire cerebrospinal axis. Cells are
small, round, blue and undifferentiated. Age incidence is 3yrs-10yrs with median age at 5-7yrs. Occurs more in males than Females. Solid,
homogenous contrast enhancing mass in Post. Fossa. Causes obstruction of 4th Vent. with Hydrocephalus.

Clinical features: Features of raised ICP (headache, nausea, vomiting, mental status changes, HBP) from blockage of CSF drainage
including severe papilloedema with visual loss. Ataxia (truncal and gait ataxia), poor balance, dysmetria, (from destruction of cerebellar
vermis) develops over a few wks.
Investigation: CT scan, MRI, CSF analysis.
Treatment: is multimodal.
Surgery: Remove as much tumour as possible. Possible CSF shunt in some pts.
176
Compiled by Obasi. D. C. Chinedu

Radiotherapy: Medulloblastoma is radiosensitive. craniospinal irradiation covers undetected CSF seeding.


Chemotherapy (effective supplementary treatment)

Sequelae of radiotherapy in children: Micocephaly, Learning disabilities, Cognitive impairment, Secondary malignancies,
Neuroendocrine dysfunction: (growth failure, hypothyroidism, hypogonadism, absence or delayed puberty)

Prognosis: 85% survival- non-metastatic, gross total tumour resection.


56% survival- bulky residual tumour - 38% survival- metastatic tumours.

Pilocytic Astrocytoma/Cerebellar Astrocytoma: Common in children and has excellent prognosis. 20% of brain tumours. Low metastatic
potential. A cystic tumour arising near the midline but usually extends into one cerebellar hemisphere. MRI –nodule within a cystic mass.
Age incidence 2yrs-8yrs.

Clinical features: Cerebellar signs and symps tend to dev gradually over many months. Signs of raised ICP.
Unilateral cerebellar dysfunction, pt tilting to one side.
Investigation: CT scan/ MRI.
Treatment:
Surgery: -Tumour resection -Possible CSF shunt for persistent hydrocephalus.
Radiotherapy : Only when all the tumour mass is not excised.
Surgery, Radiotherapy and Chemotherapy in other forms of astrocytoma.

Brain stem glioma: Tumour expands the pontine region diffusely. Can be malignant. Devs mainly in children and young adults.
Clinical features: Cranial nerve palsies and long tract signs dev gradually as tumour progresses.
More malignant gliomas have a rapid course with signs of raised ICP.

Investigation: CT scan, MRI (gives best results).


Treatment: Surgery rarely indicated. Radiotherapy
Prognosis: Low-grade gliomas have a favourable outcome. Median survival for diffuse pontine glioma is 12 months.

Metastatic Brain Tumours: Uncommon in children. 2 types are described:


1. Metastatic spread from other childhood malignancies to the brain.
ALL, Non-Hodgkin lymphoma –leptomeninges—Comm.hydroceph.
Chloroma(AML)—Entire neuraxis
Spread to brain parenchyma results from: Lymphoma, Neuroblastoma, Rhabdomyosarcoma, Ewing sarcoma, Osteosarcoma, Clear cell
sarcoma of the kidney
2. Extraneural spread from brain tumours: Medulloblastoma, Ependymoma, PNET, Malignant glioma
-Spread can also occur via V-P shunt to peritoneal cavity and also systemically.

Complications/long term mgt of brain tumours


In developed countries: >70% long term survivors. 50% of these have chronic problems which include:
Neurologic deficits (focal motor/sensory abnormalities, seizure disorders.
Neurocognitive deficits: (Dev delay, learning disability)
Neuroendocrine deficiencies: Hypothyroidism, Growth failure, Delay or absence puberty

Question: A 5yr old male presents to the clinic with 5 weeks history of headache, nausea, vomiting, and poor vision. He also has difficulty
in walking with frequent falls.
On examination, he had hypertension, papilloedema, dysmetria, truncal and gait ataxia.

1. What is the most likely diagnosis?

177
Compiled by Obasi. D. C. Chinedu

IMMUNIZATION/VACCINATION
Dr Beckie Tagbo

Immunization
Definition: Refers to a procedure that increases an individual’s level of immunity against a particular infectious agent or toxin.
Explanation: It is the process of protecting an individual against communicable diseases by injecting weakened or killed infectious
organism or part of their structure into the body to cause the immune system to produce antibodies against the organism without cuasing the
full-blown disease. Immunization is the process by which an individual’s immune system becomes fortified against an agent (known as the
immunogen)
WHO: Immunization is the process whereby a person is made immune or resistant to an infectious disease, typically by the administration of
a vaccine. Vaccines stimulate the body’s own immune system to protect the person against subsequent infection or disease. That is, to treat
the organism such that it looses its pathogenicity while maintaining its immunogenicity.
Pathogenicity = ability to cuase disease Immunogenicity = ability to stimulate/increase immunity
Vaccination Vs Immunization
Vaccination = introducing vaccines into an individual Immunization = protection against the disease

Basic immunological principles/facts- Innate & adaptive immunity


Innate immunity: first line of defence, present from birth. non-specific.
Adaptive immunity: second line of defence, learned, specific.
There are basically two lines of defense that can prevent our bodies from being sick, in other words to prevent infection and disease.
Our first line of defense is the innate or non-specific system, meaning it is not “antigen”-specific immunity. It is comprised of the skin,
which is the most important organ to prevent bacteria and viruses from entering our bodies. And there are also barriers such as enzymes, like
lysozyme in tear secretions or fatty acids on the skin which can help to damage microorganisms before they can enter our body. Also the
low pH of mucus linings (very acid environment) and of the stomach and the high pH (alkaline) of the upper intestine and natural flora
(helpful bacteria) of the large intestine that can outcompete invading bacteria…..fecal-oral transmission route such an easy entrance into the
body. Other non-specific mechanisms exist in the blood such as phagocytosis….the eating of bacteria. Complement proteins which can
target and collect on bacteria to punch holes in their surface and destroy them. And the release of substances such as IFN-g which can inhibit
the replication of viruses in the body.

Adaptive immunity: While its non-specific defences are trying to cope with an invading pathogen, the body will already prepare its second
line of defence: adaptive immunity. Adaptive immunity has three key features: specificity, diversity and memory. It has been known for a
long time that people who recover from certain infectious diseases are unlikely to contract the same disease again. They have acquired
immunity. It is also appreciated that the immunity acquired after one infection does not protect against other diseases as immunity is specific
(with very few exceptions of cross-protection as in the case of the cowpox virus inducing immunity to the smallpox virus).
A substance that is recognised by the body as ‘non-self’ and is able to trigger a specific immune response is referred to as an antigen. The
body recognises and reacts to an antigen by mechanisms that depend on the behaviour of specialised white blood cells called lymphocytes.

Immune response mechanisms


Specific immune response mechanisms can be divided into antibody and cell-mediated immune responses:
Cell-mediated immune responses depend on the proliferation and differentiation of T lymphocytes able to recognise antigen(pathogen)-
containing cells and prepare for their destruction; some of the T cells produce biologically active substances such as cytokines or interleukins
(orchestrating cell interactions), while others act as helper (Th1 or Th2) or natural killer cells (CTL cytotoxic lymphocytes)
Antibody-mediated immune responses comprise humoral (general circulation) and local (mucosal) immune responses during which
proteins called antibodies are produced by B lymphocytes, able to react specifically with the antigens that stimulated their production;
antibody-mediated immunity is effective against pathogens circulating freely in the blood and tissue fluids or that are accessible at mucosal
surfaces. Often B lymphocytes will require help from T lymphocytes (notably Th2 helper cells) in order to induce an effective and lasting
response and immune memory to pathogens; in this case the immune response is referred to as a T-cell dependent response. On occasions,
B lymphocytes are triggered to produce antibodies, independently from T cells (by bacterial polysaccharide antigens); in this case the
response is not very effective, often of short duration (as no adequate memory is induced) and referred to as a T-cell independent response.

Passive & active immunity


Naturally acquired immunity: We may acquire immunity naturally, as a consequence of infection; naturally acquired immunity is very
effective, but during the process we may develop disease, complications or even die.
Passive immunity can also be acquired naturally when maternal antibodies are transferred across the placenta, providing passive
protection to babies during their first few months of life; these antibodies can interfere with the take of some vaccines (especially live-
attenuated injectable vaccines if given too early in life).

Artificially acquired immunity: Antibodies can be transferred from an immune to a non-immune, or susceptible subject providing short-
lived protection against pathogens accessible in the blood circulation.
In passive immunisation there is no trigger or involvement of the subject’s immune system. It is possible to stimulate immunity actively,
by exposure to an antigen, so as to induce an active and lasting immune response in the form of both antibodies and also immune memory
mechanisms. This is how effective vaccines work.
Note that the terms immunisation and vaccination are often used to mean the same thing. For clarity, use the word immunisation to refer to
178
Compiled by Obasi. D. C. Chinedu

the passive transfer of pre-formed antibodies and vaccination to mean the administration of an antigen or a vaccine.

Passive immunisation: The main advantage of passive immunisation is that it can provide more or less immediate protection against extra-
cellular pathogens. If administered soon after exposure, it can be a life-saving therapy in diseases such as diphtheria and tetanus where toxins
are causing disease. But the technique has important drawbacks:
• the protection provided by passively transferred immunoglobulins only lasts for a few weeks or months at the most. The antibodies
are used up by combination with antigen or are broken down by the normal metabolic processes. And, since the immune system is not
activated, there are no memory cells to trigger an immune response and produce more of the same antibodies in the future
• from a practical point of view, the production of specific high concentration immunoglobulins can be an expensive procedure and there
may be difficulty in finding suitable donors
• there is a theoretical risk that infectious agents in the donor’s blood may be accidentally transferred to the recipient during
immunisation if the preparations are not stringently tested. The spread of HIV and hepatitis B and hepatitis C through the transfusion of
contaminated blood and blood products has increased public awareness of this possibility.

What is a vaccine? An inactivated or attenuated pathogen (toxin) or, a component of a pathogen that when administered to the host,
stimulates a protective response of the cells in the immune system.
Vaccination: In vaccination, the person to be protected is exposed to an antigen or a mix of antigens (in the case of a combined vaccine such
as DTPw-HepB/Hib) in order to induce B lymphocytes to secrete antibodies and immune memory (if T cells are also activated). This process
of immune response activation upon the first contact with a given antigen is referred to as the primary immune response.
When the pathogen from which the antigen is derived is encountered in the course of a natural infection later in life, it will be rapidly and
effectively neutralised before it can cause any damage. Some T-cells and antibodies will be available to counter the infection immediately.
There will also be a secondary response in which much greater numbers of sensitised lymphocytes and specific antibodies will be formed to
attack the invading pathogen.

Herd Protective Effects of Vaccines: Are demonstrated by a protective impact of a vaccine in a population that exceeds an impact expected
on the basis of:
 The proportion of the population vaccinated
 The protective efficacy of the vaccine

Herd Protective Effects of Vaccines can result from:


 Transmission of a live vaccine from vaccinee to neighboring non-vaccinee (“herd immunity”)
 Reduction of transmission of the target pathogen in a population in which a proportion become immune due to vaccination with either a
live or an inactivated vaccine (“herd protection”)

Principle of vaccination: The purpose of vaccination is to produce an antigen-specific immune response against a virulent infectious agent
without actually causing disease.
First scientifically demonstrated famously by Edward Jenner in the late 1700s… Inoculated a young boy with cowpox virus material, less-
virulent than the deadly smallpox virus. When the boy was subsequently inoculated with fully virulent material from a smallpox lesion, the
child did not develop disease
Vaccines effect their immune protection primarily by stimulation of B lymphocytes, rarely, CD4 and CD8

Types of vaccines - live and non-live vaccines


live vaccines: Live vaccines involve the attenuation of disease-causing viruses by serial passage in cells or tissues from nonhuman species,
rendering them less virulent to human cells. This attenuated virus is able to infect and replicate in human cells, but results in a mild or
subclinical infection. Such live attenuated vaccines effectively trigger strong activation of the innate immune system, generally resulting in
higher immunogenicity
Non-live vaccines: Non-live vaccines come in several different forms, and can include "killed" whole virus or bacteria, proteins,
polysaccharides, glyco-conjugates, or toxoids. These non-live vaccines frequently require the use of adjuvants, or agents that increase
immune system stimulation by enhancing antigen presentation. Aluminum salts are frequently used as adjuvants.
Killed vaccines involve inactivation of microorganisms through exposure to chemical or physical agents that render the organism
noninfectious and unable to replicate
Toxoids, eg, current tetanus vaccine involves detoxification of toxin with formaldehyde, which retains immunogenicity without toxicity.

Vaccination: After a vaccine has been administered, active immunity usually takes a few weeks to develop. It may be necessary to give
further doses of some vaccines, at intervals of a few weeks (generally 1 month or more), to amplify the immune response and ensure an
adequate level of long-term immunity. Once established, the protection provided by vaccination usually lasts for several years or even for
life. This makes vaccination a highly effective method of conferring specific, long-lasting immunity.
Vaccination is very cost-effective. It costs a few dollars to protect a child against the seven or eight most common vaccine-preventable
diseases covered by the World Health Organisation’s Expanded Programme on Immunisation (measles, polio, diphtheria, tetanus, pertussis,
hepatitis B, Hib and yellow fever in risk areas). Vaccines generally account for <50% of the costs of a vaccination programme – which
include the organisation of vaccination campaigns, vaccine administration (auto-disable syringes, safety boxes, adverse event monitoring…),
storage (cold chain & its monitoring) and transport.
179
Compiled by Obasi. D. C. Chinedu

Vaccination is generally safe. Every day, millions of doses of vaccine are administered all over the world. Serious adverse reactions are very
rare - much, much rarer than the complications associated with the diseases they prevent.

The ideal vaccine: Vaccination is only of benefit if it provides a significant degree of protection against a disease to the majority of
vaccinees with a minimum of side effects.
Vaccines must be immunogenic, that is, they should provoke a good immune response in a high proportion of vaccinees. If possible, the
immune response should mimic the response which occurs during a natural infection, with comparable cell-mediated and antibody-mediated
responses but without the symptoms.
Protection provided by vaccination should be long-lasting, ideally lifelong. In practice, with some vaccines, re-vaccination at regular
intervals with so-called boosters may be necessary to sustain protection. If boosters are necessary, it will be more convenient for the vaccinee
if the intervals between them are several years apart
A vaccine should not be reactogenic, that is, it should not induce too many local (at injection site) nor systemic effects (general effects such
as fever, vomiting…) nor rare, serious adverse events (AEs). As Adverse Events may be extremely rare, post-marketing surveillance has to
be put into place in order to identify and monitor them
A vaccine must be stable, so that it retains its potency during transport as well as in field conditions. Some vaccines are sensitive to heat
(thermolabile), whilst others are destroyed by freezing (cryolabile). Some other vaccines do not suffer significantly if accidentally exposed to
higher than recommended storage temperatures (more thermostable vaccines) and again others show almost no potency loss at very low
freezing T°Cs down to -70°C (cryostable vaccines).

Benefits of vaccination: As well as individual benefits, there are important community benefits – herd immunity and in some cases, the
possibility of complete eradication of the pathogen and hence the disease (as in the case of smallpox).
To achieve a satisfactory level of herd immunity, it is usually necessary to vaccinate at least 75 to 80% of the susceptible population; for
some pathogens such as the measles virus, much higher coverage rates are required (close to 100%).
Smallpox was eradicated from the world in the 1970s by an intensive vaccination campaign supported by comprehensive surveillance
programmes. Today, polio is close to being eradicated and there are realistic hopes that measles too will have disappeared in the early
decades of the 21st century.

Benefits of vaccination
Vaccination is an obvious benefit to those individuals who have been vaccinated. It provides them with specific individual protection against
illness, disability and possible death from the disease.
Vaccination against some diseases can also benefit entire communities. The vaccination of a large proportion of members of a community
against a disease makes it difficult for the pathogen to spread through that community. The resistance of the community as a whole, its herd
immunity, is increased.
If the pathogen that causes disease is only capable of surviving in a human host, it’s theoretically possible to completely eradicate that
disease through vaccination.
The potential benefits of vaccination are enormous. At comparatively little cost, it’s possible to prevent many infectious diseases and so
improve quality of life, break the poverty cycle, enhance economic growth and free resources for other health care priorities.
Vaccines Given in First Year Vaccines Given in Second Vaccines Given to Adolescents Vaccines Given in the First Year of
of Life (US) Year of Life (US) or Adults Life (Nig)
Diphtheria 3 Measles 1 dose TdAcP 1-2 BCG 1
doses Mumps 1 Meningococcal 1 OPV 4
Tetanus 3 Rubella 1 Human Papilloma Virus 3 IPV 1
Pertussis (acellular) 3 Varicella 1 Influenza 1+ HBV 4
IPV 3 Hepatitis A 1 Zoster 1 Pertussis 3
Hepatitis B 3 Boosters for DPT, Hib, Pneumococcus 1 Diphtheria 3
H. Influenzae b 2-3 Polio, Pneumo, Influenza Total (8-9) Tetanus 3
Pneumococcal Conjugate 3 Total 12 doses Hib 3
Influenza 2 PCV 3
Rotavirus 3 Measles 1
Total (25 – 26) doses Yellow fever 1
Total (27)

20th Century Impact of Vaccines (US)


Vaccine Preventable Diseases Peak cases Cases 2006 % Reduction

Diphtheria 30 508 0 100 0

Measles 763,094 55 99.9

Mumps 212 932 6 584 95 9

180
Compiled by Obasi. D. C. Chinedu

Pertussis 265,269 15,632 92.2

Polio (acute) 42 033 0 100 0

Polio (paralytic) 21,269 0 100 0

Rubella 488 796 11 99 9

Congenital rubella 20,000 1 99.3

Smallpox 110 672 0 100 0

Tetanus 601 41 92.9


Vaccination not properly valued: We’re healthy, so why do we need vaccination?
Invasive interventions My child is not a pin-cushion Vaccines ‘weaken the immune system’
Vaccines ‘are expensive’ Vaccines ‘are dangerous’
Yet high immunization rates are necessary to keep diseases like measles and even polio from making a dangerous comeback. Here are some
myths about vaccines -and the truth behind them.

Myths: Getting so many vaccines will overwhelm my child's immune system


As long as other children are getting vaccinated, mine don't need to be.
Now that major illnesses have largely disappeared, we really don't need vaccines anymore
Vaccines cause autism and other disorders. - My baby might get the disease it's supposed to prevent.
You shouldn't give a vaccine to a child who has a cold. - Vaccines can provide 100 percent disease protection
It's best to wait until children are older before starting to give them vaccines.

Why such a crisis? despite regular exposure to pathogens & millions of victims, vaccines are considered artificial & potentially hazardous
interventions
- as disease visibility decreases, the “risk” of vaccines is perceived as increasingly unacceptable
Immune overload? “Vaccines – raindrop in the ocean of what an infant’s immune system successfully encounters every day”

Why Vaccines are Different than Drugs 1) Given to healthy people, high safety required
2) Larger governmental role 3) Low efficacy unacceptable
4) Often used in infants 5) Given once or a few times
6) Manufacturing larger part of cost 7) High regulatory / quality control burden
8) Supposed to be cheap

Routine immunization/schedule
Routine Immunization: This refers to immunization given to eligible age group of children(<1year) and Women of Child bearing Age
(WCBA) during regular visit to the Health Facility. It ensures the reduction of mobidity(diseases) and motality (deaths) arising from Vaccine
Preventable Diseases (VPDs). It protects not only individuals but also community (herd effects) against VPDs.
It is a sure means of protecting our children from killer diseases. It is cost effective.
The GOAL of RI is to reduce morbidity and mortality due to vaccine preventable disease through the provision of potent vaccines, technical,
cold chain and logistics to States and LGAs.
The TARGET is to control vaccine preventable diseases through the use of vaccines
To reduce disease burden due to VPDs, a minimum vaccination coverage of 80% is necessary

Elements of RI: They include; Vaccines Supply, Logistics/Cold Chain, Service Delivery, Data/Records, Management, Advocacy/Social
mobilization/Communication. Surveilance, Monitoring & Supervision. Co-ordination and collaboration.

Strategies for RI Service Provision:


Fixed Post: Regular Vaccination at the Health Post on specific days as scheduled
Outreach: Service Delivery to Clients who can not get to the Health Facility. Could be done monthly
Service delivery to people living in remote areas has high resource implication
Nigerian nat routine imm schedule (by age) Immunization schedule (by disease / vaccine)
Age Antigens (vaccines) given NPI Target Disease Period of Administration
Birth BCG, OPV0, HepB0 Tuberculosis / BCG At birth
6 weeks OPV1, Pentavalent1, PCV1, Poliomyelitis / OPV At 14 weeks
Rota1
10 weeks OPV2, Pentavalent2, PCV2, Poliomyelitis / IPV At birth, 6, 10 & 14 weeks
Rota2
14 weeks OPV3, IPV, Pentavalent3, Pentavalent (DPT, HBV, Hib) 6, 10 & 14 weeks
PCV3

181
Compiled by Obasi. D. C. Chinedu

9 months Measles, Yellow Fever Hepatitis B / HBV At birth


Yellow Fever 9 months
Rotavirus* 6 & 10 weeks
Measles 9 months
Meningitis* 12 months

Vaccines and their administration


Contact Minimum target Type of Dosage Route Site
age vaccine

1st At birth HBV0 0.5ml Intramuscular Outer thigh


BCG 0.05ml Intradermal Upper left arm
OPV0 2 drops Oral

2nd At 6 weeks Penta1 0.5ml Intramuscular Outer thigh


OPV1 2 drops Oral Mouth
PCV 0.5ml Intramuscular Outer thigh
Rota 1ml Oral Mouth

3rd At 10 weeks Penta2 0.5ml Intramuscular Outer thigh


OPV2 2 drops Oral Mouth
PCV 0.5ml Intramuscular Outer thigh
Rota 1ml Oral Mouth

4th At 14 weeks Penta3 0.5ml Intramuscular Outer thigh


OPV3 2 drops Oral Mouth
PCV 0.5ml Intramuscular Outer thigh
IPV 0.5ml Intramuscular Outer thigh

5th 9 – 12 months MV 0.5ml Subcutaneous Upper left arm


YF 0.5ml Subcutaneous Upper right arm

Dose When to give Percentage protection Duration of protection

TT-1 At 1st contact or as early as possible in pregnancy NIL None

TT-2 At least 4 weeks after TT-1 80 1-3 years

TT-3 At least 6 months after TT-2 or during subsequent 95 At least 5 years


pregnancy

TT-4 At least one year after TT-3 or at subsequent 99 At least 10 years


pregnancy

TT-5 At least one year after TT-4 or during subsequent 99 For all childbearing years &
pregnancy possibly longer

Vitamin A Supplementation: Not a vaccine. Essential for growth and development; strengthening immune system, limiting severity of
illness & increases chances of curability.
Deficiency causes reduced resistance to infection, blindness in children & anemia. PREVENTION by consuming enough vitamin A rich
food in addition to taking vitamin A drops (Vitamin A supplementation)
Vitamin A supplement During routine immunization & SIAs e.g. During NIPDs which is one of the quickest & least expensive ways of
reaching a large no of children in high risk group.
Age Dose Capsule
6 months 100,000 IU* (30 mg RE**) for 6-11 month 1 blue capsule or half red capsule (4 drops)
old
12 months 200,000 IU (60 mg RE) for > 12 month old 1 red capsule or 2 blue capsule

Barriers to Vaccination:
 Knowledge Deficits (patients and providers)
 Poor Access to Health Care: Financial access, Physical access.
 Vaccine Shortages / poor storage
 Lack of Mandatory Vaccination Policies (e.g. for school attendance)

182
Compiled by Obasi. D. C. Chinedu

Vaccines
BCG Vaccine: Bacillus Calmette Guerin. Protects against tuberculosis. Comes in powdered form. Must be reconstituted before use.
Reconstituted vaccine is even more sensitive to heat than the powder and therefore must be used within 6 hrs or discarded. Wrapped in foil or
paper to protect it from light. Stored with the diluents at 2-8 0C. Given at birth or as soon as possible. One dose of 0.05ml for < 1yr and
0.1ml for > 1yr given intradermally at the upper left arm. Do not rub or apply anything to injection site
Side effects: small sore for 2 wks, will leave a small scar. Swollen glands-refer to doctor. Abscess – refer to doctor
Not given to HIV positive children

Hepatitis B Vaccine: Protects against Hep B. Comes in a cloudy liquid in a vial or prefilled syringe. Does not have to be reconstituted. Must
be mixed by shaking before administration. May come as combination vaccine. Stored at 2-8 0C. Both heat and freeze sensitive. Given at
birth (not later than 2 wks). 0.5ml at outer part of the thigh, intramuscular. Side effects-mild fever-Paracetamol.

Measles vaccine: Prevents measles. Comes in powdered form. Must be reconstituted before use and discarded after 6 hrs. Stored at 2-8 0C.
Not damaged by freezing. Given at 9mths or as soon as possible after 9mths, regardless of whether the child has had measles or not. Given
subcutaneously at upper left arm.
Side effects-mild fever one week later, give paracetamol. Mild rash may appear-no treatment

Yellow fever vaccine: Prevents yellow fever. Comes in powder form and must be reconstituted before use. Discard reconstituted vaccine
after 6hours. Stored at 2-80C. Given at 9 months. Not given before 6 months (encephalitis more common). Not given to children with
clinical AIDS. Dose is 0.5 ml, subcutaneous, upper right arm.
Side effects-fever, mild muscle/joint pain, treatment is paracetamol

Pentavalent Vaccine: The Hib + DPT + Hepatitis B vaccine is called the “Pentavalent” or five in one vaccine. The Pentavalent vaccine
protects children against five diseases. These diseases are: Hib diseases, Diptheria, Tetanus, Pertussis, Hepatitis B.
Pentavalent vaccines come in two separate formulations: Liquid & Liquid + Lyophilized (freeze dried).
Nigeria is currently using the Liquid form of the vaccine which is a 10 dose vial and does not require reconstitution
Pentavalent Vaccine: Stored at 2-80C, Diphtheria and Tetanus toxoids are damaged by freezing (do not freeze)
Pertussis is damaged by heating. Given at 6, 10 and 14 wks. Interval between doses not less than 4 wks. Dose is 0.5ml. Site – outer thigh,
Intramuscular (IM)-never in the buttocks
Side effects may include: About 25% of patients report mild redness, swelling and soreness at injection site.
Rarely, a fever may develop which should subside within 48 hours. Paracetamol can be given to reduce symptoms.
Serious adverse events (e.g. anaphylactic shock) may occur, though these are extremely rare.

Oral Polio Vaccine (OPV): bOPV protects against 2 types of virus that cause poliomyelitis (types 1 & 3)
Type 2 and tOPV have been phased out in 2016 aimed at eradication. Liquid that comes in small plastic bottles that work like droppers or
glass vials with droppers in a separate plastic bag.
bOPV is a sabin live attenuated vaccine, that comes in 20 doses vials. It is given as 2 drops in the mouth (Orally).
Since 1996, OPV vials supplied by WHO/UNICEF have had vaccine vial monitor (VVM) attached. It tells the health worker whether the
vaccine is safe for use. Stored at 2-80C. Damaged by heat. Not harmed by freezing
Given at birth, 6, 10 and 14wks. Interval between doses must be at least 4wks
In case of diarrhoea, give OPV as usual, then a 5th dose 4wks after last dose. Adverse events extremely rare

Vaccine Vial Monitors (VVM)


The inner square darkens irreversibly with exposure to heat over time.
Extended exposure to heat is determined by comparing the color of the inner square to the color of the surrounding reference circle!
USE VIAL WHEN THE SQUARE IS WHITE (stage 1) OR LIGHTER THAN THE CIRCLE (stage 2).
DISCARD VIAL WHEN THE SQUARE IS AS DARK AS THE CIRCLE (stage 3), OR DARKER THAN THE CIRCLE (stage 4).
VVM is found on the cover or label.

OPV Reaction Rate with VVM


Number of days before the VVM reaches the discard point (stage 3) at different ambient temperatures


+ 37°C = 1.5 - 2 days + 25°C = 7 days

+ 4°C = 180 days (6 months) - 20°C = 2 years

OPV AND IPV: Both are effective against polio


OPV = Oral polio vaccine (live attenuated)
183
Compiled by Obasi. D. C. Chinedu

IPV = Inactivated polio vaccine (inactivated)

Comparison of Advantages and Disadvantages of OPV and IPV


Attributes OPV IPV
Potency Low (needs 4 or more doses) High (needs 2 or 3 doses)
Intestinal immunity High (community protection) Low (individual protection)
Secondary immunization Yes No
Extra injection No Yes
Intestinal or mucosal immunity prevents intestinal shedding or silent shedding and circulation of the virus must stop even if every
one is immuned for eradication to take place.

Route & Dose of IPV Administration


IPV stand-alone is a fully liquid vaccine that comes in 10 doses vials and is administered with an auto-disable 0.5 ml syringe and needle.
IPV is administered by intramuscular injection (IM) in a dose of 0.5 ml into the outer part of the right thigh.

Contraindications
Known or documented allergy to streptomycin, neomycin, or polymyxin B, which are inactive components of the vaccine, or history of an
allergic reaction following a previous injection of IPV.
Note: However, IPV can be administered to a premature infants (<37 weeks gestation) and children with immunodeficiency (e.g. HIV,
Congenital or acquired immunodeficiency, sickle cell disease.)

Side effect of IPV administration: Mild side effects are; redness, swelling and soreness at the injection site.
Rare instances: transient fever and vomiting. Resolve within 48 hours and are not serious.
Caregivers should be advised to give the child paracetamol to treat these symptoms.

PCV: Pneumococcal Conjugate Vaccine. Introduced in phases


Phase I – 2014 Phase II – 2015 Phase III – July 2016 (Enugu – Oct 2016)
Pneumococcal vaccines protect against several severe forms of pneumococcal diseases, such as meningitis, pneumonia and bacteraemia etc.
PCV 10 protects against 10 common types of pneumococcal serotypes and is part of the current EPI schedule administered in 3 doses.
PLEASE NOTE: Even after full immunization, a child may still get other types of pneumonia, meningitis or viral infections, such as
the flu.
Characteristics: Two types of Pneumococcal Conjugate Vaccines (PCV) are currently available globally, namely: PCV10 and PCV13.
Nigeria currently uses PCV10 (Synflorix™), a liquid formulation without preservative in a 2-dose vial.
VVM on cap: VVM30 present on the cap of the vial. No preservative
Contraindications: PCV10 should not be given to infants who had severe prior allergic reactions to the vaccine
- Infants with a moderate or severe illness (temperature ≥39°C) should not be vaccinated until their condition improves.
Mild illness such as an upper respiratory tract infection is not a contraindication for PCV 10 administration

Rotavirus vaccines: Rotarix (Monovalent- 2 doses) or Rotateq (Pentavalent- 3 doses).


Disease Surveillance: The process of systematic collection, orderly consolidation and evaluation of pertinent data with prompt
dissemination of the results to those who need to know, particularly those who are in a position to take action. (Adapted from Report of the
Technical Discussions at the twenty-first World Health Assembly on National and Global Surveillance of Communicable Diseases, 18 May
1968 – A21/Technical Discussion/5)

Disease surveillance: The ongoing systematic collection and analysis of data and the provision of information which leads to action being
taken to prevent and control a disease, usually one of an infectious nature.

Why was it called 'surveillance'? Close observation of exposed persons to identify onset of disease
Definition changed in the 1950s. What is the definition of public health surveillance?
What are the key elements of public health surveillance?
- Ongoing - Systematic - Collection, analysis, interpretation, and dissemination
- Data regarding a health-related event
- For use in public health action to reduce morbidity and mortality and to improve health
Types: National, sub-national, hospital sentinel. - Active versus passive
Reporting: Case-based versus aggregated
Examples: - Polio - Measles - New vaccines

Surveillance for new VPDs- When and why?


Before vaccine introduction: Demonstrate disease burden. Justification to introduce vaccine. Establish system to measure vaccine impact.
Identify circulating strains.
After vaccine introduction: Monitor vaccination program impact. Monitor any change in circulating strains: needs strong laboratory

184
Compiled by Obasi. D. C. Chinedu

support. Platform to evaluate safety.

Sentinel vs. National Surveillance


Sentinel: 1 Children's Hospital National: Every HCF, Every Case
Easier, cheaper More expensive, harder
Likely good lab Transport samples to lab
Better quality data (active vs. passive) All disease in the country (mild, severe)
Monitor trends over time Monitor trends over time
More severe cases, so not representative of all disease Identify outbreaks, epidemics

Surveillance for New VPDs: Who? Children <5 years of age admitted to the hospital with syndromes:
For treatment of acute gastroenteritis - diarrhoea
With signs & symptoms of bacterial meningitis
With signs & symptoms of invasive bacterial disease (e.g. sepsis)

WHO Rotavirus Surveillance Site in Nigeria: Institute of Child Health (ICH), Univ of Nig Teach Hosp, Enugu is the main site. Sub-sites-
Mother of Christ Spec Hosp, Tender Care Specialist Hospital, etc.
Surveillance started 1st Sept 2010. As at 31/12/2015, total 2315 samples collected with 1077(47%) ELISA.
Our excellent performance in rotavirus surveillance resulted in WHO support to commence Paediatric Bacterial meningitis on 1/12/11

Nigeria Sentinel site surveillance for PBM and Rotavirus


Existing sites: 1. Nigeria 2 sites – LUTH (PBM site) 2. UNTH, Enugu – integrated Rota and PBM sites
WHO Technical Mission: 21 May – 1 June 2012 assessment of potential sites
3. Ilorin University Teaching Hospital (Kwara State) 4. Benin University Teaching Hospital (Edo State)
5. Abubakar Tafawa Belewa University Teaching Hospital, ATBUTH, Bauchi state).
6. Ahmadu Bello University Teaching Hospital Zaria
National Measles & Polio surv ongoing with 2 WHO accredited labs in Ibadan & Maiduguri.

Conclusion: Immunization prevents between 2-3 million deaths every year. 5 visits will get the child fully immunized. A fully immunized
child is strong & healthy. this remains the greatest asset to the family & nation
Immunization provides an opportunity to deliver other life-saving measures. Vaccines are imperfect but life saving
In view of the >/=4 months affinity maturation period and that antibodies eventually wane to non protective levels, if not booted. It has
become necessary to (and there are already strong calls for) review the EPI schedule and indeed, other country – specific immunization
schedules.To extend intervals and incorporate boosters (see PAN paper on RI). The Vaccine industry is thriving: interest in vaccines is high
and technology is exploding.
However, there is insufficient competition; successful development is complicated and expensive; and the Gates Foundation, has largely
contributed to the provision of vaccination to large populations of the world.
Surveillance should accompany immunization.

185
Compiled by Obasi. D. C. Chinedu

VACCINE PREVENTABLE DISEASES (019)


Introduction: Despite decades of health interventions in the reduction of child mortality, the leading causes of death are still communicable
diseases especially the vaccine preventable diseases (VPDs). These diseases are major contributors to child morbidity and mortality
especially in the Sub-Saharan Africa and Nigeria in particular and the threat from them remains as long as the immunization coverage rates
against these diseases remain low. This implies that appropriate deployment of relevant vaccines would significantly reduce mortality and
speed up the achievement of Millennium Development Goal 4 (MDG 4). Although the Expanded Programme on Immunization (EPI) has
been in place in Nigeria for up to 35years (since 1979), the under five mortality has only decreased from 192 in 1990 to 157 in 2008 and 117
btw 2010-2014.

Factors influencing this slow trend in the reduction of mortality rates in childhood include:
Few available vaccines: Addition of very limited number of ‘new’ vaccines to the original ones started with.
Low immunization coverage: Routine immunization coverage has remained poor over the years.
Low coverage+ Frequent stock outs:
Absent Booster doses: The children who get to be immunized according to our compromised schedule do not receive booster doses.
Advantages of booster immunization: Booster antibody responses are faster and stronger than the 10 series,
Have more prolonged and higher neutralizing capacity.
Refusal by parents for their children to receive these vaccines:

Advantages of Vaccines
Disease prevention is key to public health ”prevention better than cure”.
Protection to both recipient and those they come in contact.
They are responsible for the control of many infectious diseases that were once common.
Over the years vaccines have prevented countless cases of infectious diseases and saved literally millions of lives.
Prevents cross continent, country etc transmission by a highly mobile world .

Burden / challenges of Vaccine-preventable diseases


Factors that may hinder the administration of these vaccines include:
Wrong attitude and mal-orientation of health workers. Poor political commitment and bureaucratic bottle-necks
Low level of awareness. Others are poor global donor interest in routine immunization
The overshadowing influence of supplemental immunization activities (SIAs) over routine immunization activities.

Inability to administer these vaccines usually result in:


Increased societal and economic costs that may result following missed time from school and work because of increased number of physician
office visits, and hospitalizations.
Increased premature, unnecessary and preventable deaths etc.

Many diseases which can be prevented by vaccination exist including the following:
Diphtheria, Haemophilus influenzae type b (Hib), Hepatitis A, Hepatitis B, Influenza, Measles, Meningococcal, Mumps, Pertussis, Rubella,
Rotavirus, Pneumococcal disease, Polio, Tetanus, Varicella.

Tuberculosis(TB): TB is an air-borne infectious disease caused by bacteria, which 1 0arily affects the lungs and is both preventable and
curable. It may also spread via the blood stream. Remains one of the world’s major causes of morbidity and mortality esp in people in
resource-poor settings, particularly in Africa and Asia.
1/3rd of the world’s population (~ 2 billion people) carry the TB bacteria. More than 9 million of these become sick each year with active TB
that can be spread to others. TB is caused by a small, aerobic, non-motile bacillus known as Mycobacterium tuberculosis. Other types
include:
M. africanum - not widespread, a significant cause of TB in parts of Africa.
M. bovis - once a common cause of TB but curtailed by the introduction of pasteurized milk.
M. canetti - rare and limited to the Horn of Africa and in some African emigrants.
M. microti - also rare and mostly seen in immunodeficient people

Other known pathogenic mycobacteria include: M. leprae, M. avium and M. kansasii.


The latter two species are classified as "nontuberculous mycobacteria (NTM). NTM cause neither cause TB nor leprosy but do cause
pulmonary diseases that resemble TB.
Risk Factors for TB: A number of factors make people more susceptible to TB infections.
1. HIV - 13% of all people with TB are infected by the mycobacterium.
2. Overcrowding 3. Malnutrition

Those at high risk also include:


 People who inject illicit drugs (HIV and later TB)
 Inhabitants and employees of locales where vulnerable people gather (e.g. prisons and homeless shelters)
 Medically underprivileged and resource-poor communities

186
Compiled by Obasi. D. C. Chinedu

 Children in close contact with high-risk category patients


 Health care providers.
 Chronic lung disease is another significant risk factor. eg Silicosis increases the risk about 30-fold.
 Cigarette smoking have nearly twice the risk of TB than nonsmokers.
 Alcoholism and Diabetes mellitus (threefold increase).
 Drugs such as corticosteroidsand infliximab(an anti-αTNF monoclonal antibody) are becoming increasingly important risk factors,
especially in the developed world.
 There is also a genetic susceptibility for which overall importance remains undefined

Pathogenesis: About 90% of infected patients with M. tuberculosis are asymptomatic, Only approx. 10% may have a lifetime chance of
progressing to overt, active TB disease. An increased risk of approx. 10%/year in those with HIV, developing active TB 10%.
Mycobacteria reach the pulmonary alveoli, invade and replicate within endosomes of alveolar macrophages.
This 10 site is known as the "Ghon focus” and located in either the upper part of the lower lobe, or the lower part of the upper lobe. TB may
affect any part of the body although PTB remains the most commonly occuring type.
Extrapulmonary TB occurs when tuberculosis develops outside the lungs.

Clinical Manifestations: General signs and symptoms include


Fever, chills, night sweats, loss of appetite, weight loss and fatigue Significant finger clubbing may also occur.

Pulmonary: Chest pain, Protracted cough often with sputum. Hemoptysis in small amounts, and in very rare cases, the infection may erode
into the pulmonary artery, resulting in massive bleeding (Rasmussen's aneurysm). May cause extensive scarring with fibrosis in the upper
lobes of the lungs.
Extrapulmonary: In 15–20% of active cases, the infection spreads outside the lungs, causing other kinds of TB esp in immunosuppressed
persons and young children.
Common sites include the pleura (in TB pleurisy), the CNS (TB meningitis), the lymphatic system (in scrofula of the neck), the GUS (in
urogenital TB), and the bones and joints (in Pott's disease of the spine).
A widespread form of TB is called "disseminated" TB, commonly known as miliary TB.

Investigations: Diagnosing active tuberculosis based merely on signs and symptoms is difficult but considered when lasting >2wks. Some of
the tests that may be useful in diagnosing TB include:
1. Chest X-ray 2. Mantoux test 3. FBC, ESR
4. Multiple sputum cultures for acid-fast bacilli.

(A definitive diagnosis of TB is made by identifying M. tuberculosis in a clinical sample e.g. sputum, pus, or a tissue biopsy usually
taking 2-6wks to develop- Mycobacterium is a slow- growing organism).

Latent tuberculosis: Mantoux tuberculin skin test is often used to screen people at high risk for TB. Could be
 False positive in those who have been previously immunized
 False negative in those with Sarcoidosis, Hodgkin's lymphoma, malnutrition, or most notably, in those who truly do have active TB.

Treatment: Antiobiotics are used for treatment.


2 antibiotics most commonly used are isoniazid and rifampicin and given over a number of months.
Active TB disease: best treated with combinations of several antibiotics to reduce the risk of developing antibiotic resistance.
Directly observed therapy, (DOT) is recommended by the WHO in an effort to reduce the number of people not appropriately taking their
drugs.
The recommended treatment of new-onset PTB is 6 - months of a combination of rifampicin, isoniazid, pyrazinamide and ethambutol for the
first two months, and only rifampicin and isoniazid for the last four months.

Medication resistance can be primary or secondary.


10 resistance occurs when a person becomes infected with a resistant strain of TB.
20 (acquired) resistance during therapy because of inadequate treatment, Non compliance), or using low-quality medication.
Drug-resistant TB is a serious public health issue in many developing countries, as its treatment is longer and requires more expensive drugs.

Prevention
TB prevention and control efforts rely mainly on the vaccination of infants. The only currently available vaccine as of 2011 is bacillus
Calmette–Guérin (BCG). Usually effective against disseminated disease in childhood, but confers inconsistent protection against contracting
pulmonary TB.
The detection and appropriate treatment of active cases. Public health Education
Prognosis: Progression from TB infection to overt TB disease occurs when the bacilli overcome the immune system defenses and begin to
multiply.
These dormant bacilli produce active tuberculosis in 5–10% of these latent cases, often many years after infection.
The risk of reactivation increases with immunosuppression, such as that caused by infection with HIV.

187
Compiled by Obasi. D. C. Chinedu

In people co-infected with M. tuberculosis and HIV, the risk of reactivation increases to 10% per year.
The chance of death from a case of tuberculosis is about 4% as of 2008, down from 8% in 1995.

Measles: Measles (also known as English measles or morbilli) is an infection of the respiratory system caused by a paramyxovirus of the
genus Morbillivirus. They are enveloped, single-stranded, RNA viruses.

Epidemiology: Humans are the natural hosts of the virus which is highly contagious—90% of people without immunity sharing living space
with an infected person will become infected.
Asymptomatic incubation period occurs 9 to 12 days from initial exposure. Infectivity lasts from 2-4days prior to rash, until 2 to 5days
following the onset of the rash (i.e. 4 to 9 days infectivity in total)
Measles 1st gains access to the body via the UR tract or the conjunctiva. The virus quickly spreads to the lymph nodes with destruction of the
lymphoid tissues leading to a profound leucopenia.

Aetipathogenesis: A 10 viraemia ensues which is responsible for spreading the virus throughout the rest of the R-E system and the
respiratory system.
A 20 viraemia follows and the virus is further spread to involve the skin, the viscera, kidney and bladder.
The Koplik's spots and the rash in measles are thought to result from a delayed hypersensitivity reaction, the virus antigen being absent from
the lesion itself.

Clinical Presentations: The classical signs and symptoms of measles include:


 4-day fevers which may reach upto 40°C , the 3Cs — cough, coryza (head cold), conjunctivitis (red eyes), Anorexia, and Rashes.
 Koplik's spots seen inside the mouth are pathognomonic (diagnostic) for measles. They are rarely seen, are transient and may disappear
within a day of arising.
 The xteristic measles rash is classically described as a generalized, maculopapular, erythematous rash . Beginning several days after the
fever starting from the back of ears and, spreading to the head and neck before spreading to cover most of the body, often causing
itching.
 The measles rash appears 2 to 4 days after the initial symptoms and lasts for up to 8 days.
 Clinical dx of measles requires a hx of fever of at least 3 days, with at least one of the three C's (cough, coryza, conjunctivitis).
Koplik's spot are also diagnostic of measles. Hx of contact also very impt.

Lab investigations: Confirmation is positive measles IgM antibodies. Isolation of measles virus RNA from respiratory specimens.
Saliva can be collected for salivary measles-specific IgA testing.
Treatment: No specific Rx. Pts recover with rest and supportive treatment, medical advice can be sought if the pt becomes more unwell, as
they may be developing complications.
While there is no specific Rxt for measles encephalitis, antibiotics are required for bacterial pneumonia, sinusitis, and bronchitis that can
follow measles. Symptomatic tx also given.

Complications include: Pneumonia, Otitis, bronchitis, and encephalitis.


Acute encephalitis (rarely SSPE--subacute sclerosing panencephalitis),
Corneal ulceration (leading to corneal scarring ). Complications are usually more severe in adults.

Prognosis: A vast majority of pts survive measles while complications occur fairly frequently.
Risk of death is much higher in immune-compromised patients, due to complications such as giant cell pneumonia.
Acute measles encephalitis is another serious risk of measles virus infection often occurring 2-7days after the breakout of the measles rash
beginning with very high fever, severe headache, convulsions, and altered mentation. Pt may become comatose, and death or brain injury
may occur.

Prevention: Measles vaccination occurs at 18mths of age since children younger than that tend to retain their maternal anti-measles Igs
antibodies.
A 2nd dose is usually given to children b/w the ages of 4 and 5 yrs, to increase rates of immunity.
In developing countries where measles is highly endemic, WHO doctors recommend 2 doses of vaccine be given at 6 and 9 months of age.
Measles vaccination programs are often used to deliver other child health interventions, as well, such as
Bed nets to protect against malaria, Anti-helminthics, Vitamin A supplements, and so contribute to the reduction of child deaths from other
causes.

Tetanus: An intoxication caused by Clostridium tetani which produces 2 exotoxins; tetanolysin( a haemolysin) and tetanospasmin(causes
increased muscle tone and spasms). It occurs in several clinical forms, including generalized, cephalic, localized, and neonatal disease.

Pathophysiology: Results from direct contamination of wounds (low oxidation-reduction potential) with clostridial spores. Toxin migrates
across the synapse to presynaptic terminals and blocks the release of the inhibitory neurotransmitters glycine and gamma-aminobutyric
acid (GABA). This reduced inhibition results in an increase in the resting firing rate of the motor neuron, which is responsible for the
observed muscle rigidity.

188
Compiled by Obasi. D. C. Chinedu

Epidemiology: C. tetani is found worldwide in soil, on inanimate objects, in animal faeces, and, occasionally, in human faeces. The disease
is common where soil is cultivated, in rural areas, in warm climates, during summer months, and among males (adventurous). Tetanus
predominantly develops in neonates and young children who are not immunized.

Clinical manifestations: Generalized tetanus


 Sustained trismus may result in the xteristic sardonic smile (risus sardonicus).
 Persistent spasm of the back musculature may cause opisthotonus.
 With progression, the extremities become involved in episodes of painful flexion and adduction of the arms, clenched fists, and
extension of the legs.
 Noise or tactile stimuli may precipitate spasms and generalized convulsions.
 Involvement of the autonomic nervous system may result in severe arrhythmias, oscillation of the blood pressure, profound diaphoresis,
hyperthermia, rhabdomyolysis, laryngeal spasm, and urinary retention.
Age: Most severe disease develops in elderly people and this age predilection is mostly governed by soil exposure patterns.

Tetanus neonatorum is most common in countries without comprehensive vaccination programs. Generalized tetanus that results from
infection of a neonate. Occurs in underdeveloped countries and accounts for up to 1/2 of all neonatal deaths. The usual cause is the use of
contaminated materials to sever or dress the umbilical cord in newborns of unimmunized mothers. Incubation period after birth is 3-10
days, which is why it is sometimes referred to as the disease of the seventh day. The newborn usually exhibits irritability, poor feeding,
rigidity, facial grimacing, and severe spasms with touch. The mortality rate exceeds 70%.
Diagnosis is based entirely on history/clinical findings.
Wounds should be cultured in suspected cases. The leukocyte count may be high. CSF examination yields normal results.
Serum muscle enzyme levels (eg, creatine kinase, aldolase) may be elevated
Serum antitoxin levels of 0.01 or higher are considered protective and make tetanus unlikely, although rarely cases have been reported
despite the presence of protective antitoxin levels.
Electromyography may show continuous discharge of motor subunits and shortening or absence of the silent interval normally observed after
an action potential. Nonspecific changes may be evident on electrocardiography.

The goals of treatment in patients with tetanus include: Initiating supportive therapy
Sedation/Use of muscle relaxants.
Magnesium infusion may help relieve muscle spasm and diminish sedation requirements
Debriding the wound to eradicate spores and alter conditions for germination,
Stopping the production of toxin within the wound. Neutralizing unbound toxin, Controlling disease manifestations
Use of anti microbials (Penicillins, metronidazole etc) and Managing complications.

Prevention: Tetanus is completely preventable by active immunization. Immunizations for children include tetanus toxoid(TT), usually in
the form of DPT vaccine. Booster dose given at 11-12 years and thereafter every 10 years.
For 10 immunization of adults, TT is administered as 2 doses 4-6 weeks apart, with a third dose 6-12 months later.
Booster doses are administered every 10 years or at the time of major injury if it occurs more than 5 years after a dose. Passive immunization
with TIG should be used in non-immunized and in those whose immunization status is uncertain.

Prognosis: High mortality rates are associated with the following:


Short incubation period, Early onset of spasms, Delay in treatment
Contaminated lesions of the head and the face. Neonatal tetanus.
Clinical tetanus does not induce immunity against future attacks; therefore, all patients should be fully immunized with tetanus toxoid during
the convalescent period

Complications: Airway obstruction is common. Seizures can occur. Dehydration can develop.
Urinary retention and constipation may develop because of sphincter spasm.
Respiratory failure and cardiac failure are late life-threatening events.
Fractures and muscle tears may result from sustained spasms. Autonomic dysfunction has been described.
Pulmonary emboli may develop. Bacterial superinfections are possible complications.

Poliomyelitis: Often called polio or infantile paralysis, is an acute, viral infectious disease spread from person to person, primarily via the
fecal-oral route. Caused by an Enterovirus (poliovirus). An RNA virus that colonizes the GIT- the oropharynx and the intestine. The
incubation time ranges from 3 to 35 days.

Aetiopatho-genesis: Poliovirus enters the body through the mouth, infecting - the pharynx and intestinal mucosa.
Gains entry by binding to a receptor, known as the poliovirus receptor from where the virus takes over the host cell's own machinery, and
begins to replicate.
Two basic patterns of polio infection are described: a minor illness which does not involve the CNS, called abortive poliomyelitis, and a
major illness involving the CNS, which may be paralytic or non-paralytic.

189
Compiled by Obasi. D. C. Chinedu

Clinical Presentation
 Usually poliovirus infection is asymptomatic but may produce minor symptoms which include URTI, GI disturbances but rarely,
diarrhea.
 CNS involved in about 3% of infections presenting mostly as non-paralytic aseptic meningitis, with symptoms of headache, neck, back,
abdominal and extremity pain, fever, vomiting, lethargy and irritability.
 About 1-5 in 1000 cases progress to paralytic dx, where the muscles become weak, floppy and poorly controlled, and, finally,
completely paralyzed; this condition is known as acute flaccid paralysis.

Laboratory investigations
 A lab dx is usually made based on recovery of poliovirus from a stool sample or a swab of the pharynx.
 Antibodies to poliovirus can be diagnostic, and detectable in the blood of infected pts early in the course of infection.
 Analysis of the pt's CSF, (reveals leucocytosis (10arily Lymphocytes).
 There is also mildly elevated protein level.
 Detection of virus in the CSF is diagnostic of paralytic polio, but rarely occurs.

Treatment: There is no cure for polio. Focus of treatment involves: Providing relief of symptoms,
Speeding recovery and preventing complications.
Supportive measures include antibiotics to prevent infections in weakened muscles, analgesics for pain, moderate exercise and a balanced
diet.
Treatment of polio often requires long-term rehabilitation, including physical therapy, braces, corrective shoes and, in some cases, orthopedic
surgery.
Portable ventilators may be required to support breathing.

Complications: Residual complications of paralytic polio often occur following the initial recovery process.
Muscle paresis and paralysis can sometimes result in skeletal deformities, tightening of the joints and movement disability.
Osteoporosis and increased likelihood of bone fractures may occur.
Extended use of braces or wheelchairs may cause compression neuropathy, as well as a loss of proper function of the veins in the legs, due to
pooling of blood in paralyzed lower limbs.
Complications from prolonged immobility involving the lungs, kidneys and heart include pulmonary edema, aspiration pneumonia, urinary
tract infections, kidney stones, paralytic ileus, myocarditis and Cor pulmonale.

Prognosis: Patients with abortive polio infections recover completely. In those who develop only aseptic meningitis, the symptoms can be
expected to persist for 2-10 days, followed by complete recovery.
In cases of spinal polio, with complete neural destruction, paralysis will be permanent; Cells that are not destroyed, but lose function
temporarily, may recover within 4 to 6 weeks after onset.
1/2 the pts with spinal polio recover fully; 1/4 recover with mild disability, and the remaining 1/4 are left with severe disability. Spinal polio
is rarely fatal

Prevention: 2 types of vaccines are used throughout the world to combat polio and both induce immunity to polio.
The Salk vaccine, or inactivated poliovirus vaccine (IPV), confers 90% immunity after two doses and 99% are immune after 3 doses.
Another vaccine made of live, oral polio vaccine (OPV) is also used. A single dose of oral polio vaccine confers 50% immunity to all three
poliovirus serotypes and 95% after 3 doses.
Because OPV is inexpensive, easy to administer, and produces excellent immunity in the intestine (which helps prevent infection with wild
virus in areas where it is endemic), it has been the vaccine of choice for controlling poliomyelitis in many countries

Pertussis: Pertussis, commonly known as whooping cough, is a respiratory tract infection caused by Bordetalla pertussis, xterized by a
paroxysmal cough.

Epidemiology: Occurs between June and Sept. There is equal sex distribution among pts with pertussis esp in those < 20 years. Pertussis has
two peak periods; in pts < 1 year and in those between 10-19 years.
Whites are more affected in pts <20yrs. Incubation period of pertussis ranges from 3-12 days.
B pertussis is a gram-negative pleomorphic bacillus that spreads via aerosolized droplets from coughing by infected individuals.

Clinical Manifestations: It is a 6-week disease divided into


Catarrhal stage: Nasal congestion, rhinorrhea, and sneezing, low-grade fever, tearing, and conjunctival suffusion.
Paroxysmal Stage: Paroxysms of intense coughing lasting several minutes + whoop in older pts.
Convalescent stages: Chronic cough, lasting for weeks, headaches feelings of suffocation or strangulation.
Each stage may last from 1-2 weeks

Laboratory Investigations: Hx and clinical presentation very impt.


Lab. confirmation of pertussis is difficult. Finding of lymphocytosis on laboratory exam may be helpful.

190
Compiled by Obasi. D. C. Chinedu

Deep naso-pharyngeal aspiration sample inoculated in a special media (Regan-Lowe or Bordet-Gengou agar and modified Stainer-Scholte
media).

Treatment: The mainstay of therapy in pts with active pertussis is supportive.


The goals of the therapy include limiting the no of paroxysms, observing the severity of cough, providing assistance when necessary, and
maximizing nutrition, rest, and recovery.
Antimicrobial agents (Macrolides mostly) given during the catarrhal phase may ameliorate the disease.

Complications
 Complications of pertussis are usually minimal, and most patients make a gradual full recovery with supportive care and antibiotics.
 Major complications are more common in infants <6mths which include pneumonia, encephalopathy, seizures, failure to thrive, and
sudden death.
 Minor complications Include : Epistaxis, Nausea and Vomiting, Ulcers of the frenulum and Subconjunctival Hemorrhages.

Prognosis: Prognosis for full recovery is excellent; High risk of morbidity/mortality with co-morbid conditions.
 More severe dx in infants <6 months, more likely to develop complications, and require admission.
 WBC > 100,000, has been associated with fatalities from pertussis. Also WBC counts of >55, 000 and pertussis complicated by
pneumonia were independent predictors of fatal outcome.

Prevention: Prevention through immunization remains the best. Evidence shows that parents and older siblings are the 1 0 source of infection
in young infants. The incidence of Pertussis in pre-adolescents, adolescents, and adults has increased, and may be responsible for the
increasing number of cases observed in young infants in some countries.

Diphtheria: An acute and toxic infection caused by Corynebacterium diphteriae. There are 3 known isolated strains including mitis,
intermedius and gravis, each able to cause diphtheria.
Epidemiology: C. diphterium found exclusively in the mucous membranes and skin of humans.

Mode of transmission: Droplet inhalation or direct contact with exudates from infected skin lesions.
Incubation period of 2-5 days and Age range: 6-12mths. Initially Symptoms are general and nonspecific, resembling a typical viral upper
respiratory infection (URI).
Cutaneous diphtheria is a disease xterized by indolent, nonhealing ulcers covered with a gray membrane, often co-infected with Staph.
aureus and group A Strep.
Gram stain shows club-shaped, non-motile, non-encapsulated, bacilli found in clusters. Culture yields C.diphteriae.
ECG may demonstrate valvular vegetations; however, this systemic manifestation of diphtheria is rare.

Treatment: Initial Treatment is with antibiotics prior to outset of results of confirmatory tests.
Erythromycin or penicillin is used for eradication of organisms, thus limiting the amount of toxin produced.

Complications: 1) Respiratory system: Respiratory failure due to pseudo-membrane formation, tissue edema, aspiration, 2 0bacterial
pneumonia.
2) CVS: Myocarditis, cardiac dilatation and failure, mycotic aneurysm, endocarditis, Rhythm disturbances - Heart block, including
dysrhythmias and AV dissociation.
3) CNS: Cranial nerve dysfunction, peripheral neuropathy/Total paralysis, Optic neuritis.
4) Haematological: Septicemia/shock (rare), Metastasis of infection to distant sites such as spleen, myocardium, or CNS (rare), Septic
arthritis/Osteomyelitis (rare).

Prognosis: Very poor prognosis with Cardiac involvement esp AV and left bundle-branch blocks (mortality rate 60-90%). Bacteremic
disease carries a mortality rate of 30-40%.

Prevention: Widespread awareness of the need for universal immunization is indicated. Stress the importance of seeking medical attention
in all cases of contact with suspected diphtheria cases. Isolate all cases promptly and use universal and droplet precautions to limit the
number of possible contacts.

191
Compiled by Obasi. D. C. Chinedu

PNEUMONIAS AND COMPLICATIONS


Dr. Joy N. Eze

What We Need to Know!!


Definition, Epidemiology and Aetiopathogenesis and Classification of Pneumonia
Symptoms and Signs of Pneumonia in Children
Identify and evaluate a child with Pneumonia
Rationale for empiric and targeted antimicrobial therapy
Complications of Pneumonia

Case Review: 2 month old female presented with 4 day history of fever, fast breathing and cough. Had fluid filled (pustular) skin rash 2
weeks before onset of illness. Immunization up to date. On exclusive breast feeding
Examination findings: well nourished, in resp distress, pyrexic, RR-100b/min, HR 154b/min, equal chest excursion, resonant percussion
note, good air entry, bibasal coarse crepitations posteriorly; liver 3cm below costal margin.
 Admitting diagnosis was severe bronchopneumonia
 Empirical treatment: IV Ceftriazone and IV Genticin; then IV Vancomycin was introduced and Genticin was discontinued. Made
remarkable improvement

Definition: Pneumonia is an acute pathogen-driven inflammation of the lung parenchyma. Pneumonia can be Community or Hospital
acquired.
Community-acquired pneumonia (CAP) refers to a pneumonia in a previously healthy person who acquired the infection outside a hospital
Hospital-acquired pneumonia (HAP) is pneumonia that occurs more than 48 hours after admission and without any antecedent signs of
infection at the time of hospital admission.
The distinction of HAP from community-acquired pneumonia is important, as patients with HAP are susceptible to pneumonia from a
different and potentially more virulent spectrum of organisms.

Who is at risk, and Why, how, when, what ?


Global burden (GDB 2015)- Children under the age of 5 years, ~155million cases/year
~ 17% of under-5 deaths ; more deaths in children younger than 2 years; developing countries worse hit
~ In 2015, pneumonia accounted for 1 in 6 childhood deaths globally.
~ 6 out of 10 pneumonia deaths occurred in 10 countries located within sub-Saharan Africa and Southeast Asia
Definite Risk Factors Likely Risk Factors Possible Risk Risk Factors for
Factors Neonates

Malnutrition (WAZ <-2) Parental Smoking Mother's education Premature rupture of


membrane

Low birth weight Zinc deficiency Day care Low birth weight
attendance

Non exclusive breast Mother's experience as a Outdoor air Preterm delivery


feeding caregiver pollution

Lack of immunization Concomitant Disesase Lack of exclusive


(measles etc) (Heart Disease, breastfeeding,
Household air pollution; SCA, concomitant
Overcrowding Immunodeficiency heart disease
states
Hospital Acquired Pneumonia
The pathogenesis of HAP is multifactorial. The concomitant illnesses of hospitalized patients place them at risk for nosocomial infections.
Alterations in patient immune function allow pathogens to cause invasive infections that would not occur in healthy individuals. Poor
nutrition increases their risk of infection.
Severe illness and hemodynamic compromise have also been associated with increased rates of HAP.
Aspiration of oropharyngeal secretions plays a significant role in the development of HAP.
**The combination of depressed immune function, impaired muco-cilliary clearance of the respiratory tract, and the presence of more
pathogenic organisms makes aspiration a significant contributor to HAP**

Organisms implicated in Hospital acquired pneumonia: Early-onset HAP in patients with no prior antibiotic exposure tends to mirror
community-acquired pneumonia.
The most common pathogens include Enterobacteraciea, Haemophilus influenzae, Streptococcus pneumonia, and methicillin-sensitive
Staphyloccous aureus. Patients with recent antibiotic exposures are susceptible to the above organisms in addition to non–lactose fermenting
gram-negative bacilli.

192
Compiled by Obasi. D. C. Chinedu

Late-onset HAP in patients with no prior antibiotic exposure presents with similar bacteria. Patients may also present with gram-negative
bacilli resistant to first-generation cephalosporins.
**Patients in the above categories generally are exposed to antibiotic-sensitive organisms**
Late-onset HAP with prior antibiotic exposure presents a greater problem.
**As many as 40% of these patients present with potentially multidrug-resistant pathogens, including Pseudomonas aeruginosa,
Acinetobacter baumannii, and methicillin-resistant Staphylococcus aureus (MRSA).

Age Common Organisms Causes of CAP (Community)

Birth to 20 days Bacteria: Escherichia coli, Group B streptococci, Listeria monocytogenes

3 weeks to 3 months Bacteria: Chlamydia trachomatis, S. pneumoniae


Viruses: Adenovirus, Influenza virus, Parainfluenza virus 1, 2, and 3, Respiratory syncytial virus

4 months to 5 years Bacteria: Chlamydia pneumoniae, Mycoplasma pneumoniae, S. pneumoniae


Viruses: Adenovirus Influenza virus Parainfluenza virus Rhinovirus Respiratory syncytial virus

5 years to Bacteria: Chlamydia Pneumoniae, Mycoplasma. Pneumoniae, S. pneumoniae


Adolescence
Pathophysiology and Pathogenesis
Key players: Body defense mechanism and microbial entry/ multiplication
Body defense mechanisms
Non specific: nasal hair, nasal turbinates, vocal cord, glottis, mucociliary clearance and cough reflex; humidification, cells -neutrophils,
alveolar macrophages, airway secretion, complements, surfactants etc.
Specific: coordinated activation of B and T lymphocytes leading to activation of cytotoxic T cells and specific antibodies

Classification of Pneumonia:
Base d on source of infection: Community or Hospital acquired pneumonia
Based on anatomical site/area of involvement: broncho/lobar /interstitial pneumonia
Based on duration: acute or chronic
Microbiological classification: based on organism isolated/identified

Based on Severity (World Health Organization): Mild or Severe


Mild: mild chest indrawing, decrease breast sounds, dull percussion note, crackles
Severe: features of mild plus central cyanosis, SaO2 <90% on room air, severe respiratory distress (grunting, severe chest indrawing,
decrease or absent breath sounds;
danger signs: inability to feed or drink, lethargy or unconsciousness, convulsions

Clinical Evaluation:
The strongest predictors of pneumonia in children are: Fever, cyanosis, and any of the following: tachypnea, cough, nasal flaring,
retractions, crackles/rales, and decreased breath sounds
History of chest pain, focal rales, duration of fever, oximetry levels at triage as significant predictors. Hypoxia (oxygen saturation 92%) was
the strongest predictor of pneumonia (NEUMAN et al. Pediatrics 2011)
Suspected Pneumonia if: a child < 2 years has tachypnea + a temperature >38°C (100.4°F). ***Measurement of tachypnea requires a full
one-minute count while the child is quiet***
Others features: general malaise, lethargy
The World Health Organization’s age-specific criteria for tachypnea are the most widely used:- Respiratory rate:
>50 breaths per minute in infants two to 12 months of age;
>40 breaths per minute in children one to five years of age; and
> 30 breaths per minute in children older than five years

Table 4: Diagnostic Evaluation summarized


Clinical facts Suggested interpretations

History Viral infection


Day care attendance Pneumococcal or Haemophilus influenzae infection,
Missing immunizations pertussis
Antibiotic therapy within previous month Infection with resistant bacterial strain
Recent travel Fungal infection, influenza

Physical examination Pneumonia is unlikely without fever and more than one

193
Compiled by Obasi. D. C. Chinedu

Elevated temperature, Respiratory signs Grunting, Nasal respiratory sign.


flaring, Chest Retractions Tachypnea, Wheezing, pulse *If there is respiratory signs and no fever, consider
oximetry (decrease oxygen saturation on room air). asthma, aspiration of foreign body, chemical ingestion, or
an underlying cardiac or pulmonary disorder.

Laboratory studies Not helpful in distinguishing etiology


Complete blood count, ESR, C-reactive protein level, May detect specific antigens/antibodies
Gram stain and culture.
Polymerase chain reaction, Rapid viral antigen
testing, Serologies

Imaging Currently found useful


Chest ultrasound scan Not routinely done, Not helpful in distinguishing etiology
Chest radiograph

Table 5: Distinguishing Chest Signs of Lobar and Bronchopneumonia


Signs Lobar Bronchopneumonia

Chest deformity None None


Chest movement Diminished or absent on affected side Normal

Mediastinal shift None None

Vocal fremitus Increased Normal

Percussion note Dull Resonant

Breath sound Bronchial Vesicular

Added sound Bronchial breath sounds or Crepitation Crepitations

Vocal resonance Increased Normal

Management/ Rationale for appropriate antimicrobial therapy: Management can be in the community or hospital setting.
In the community: the goal is to prevent progression to severe pneumonia and subsequent hospitalization.
**Health workers need to keep abreast with signs of severe pneumonia and refer to appropriate health facility where indicated**
At the hospital: important evaluation of anthropometry, Resp rate, Pulse rate, pulse oximetry, chest signs
Recommended management steps
- Count the respiratory rate for one full minute - Look for evidence of increase work of breathing
- Check for cyanosis, document oxygen saturation - Palpate the position of the trachea
- Percussion the chest for dullness or hyperresonance - Look for complications
Auscultate for bronchial breath sounds and crepitations or rhonchi - Look for signs of systemic involvement
- Classify the severity of pneumonia - Decide who needs hospitalization
- Decide on relevant investigation - Give antibiotics

Indication for chest radiograph in CAP: Failures to respond to initial course of antimicrobial therapy at 48hours
- Suspected complications such as pleural effusion, pneumothorax
- Presence of significant chest retraction
- Progressive symptom despite antibiotic therapy
**chlamydia and mycoplasma pneumonias show more extensive radiographic features than the presenting features may suggest**

Antimicrobial choice ---Rationale


Strept pneumoniae and Staph aureus are major causative agents for bacterial CAP and responds to Penicillins.
Cephalosporins are effective alternatives. Vancomycin is good where MRSA is suspected.
Macrolides indicated where atypical organisms are suspected or proven
Note: Macrolide resistance and 2nd generation cephalosporin resistance are associated with treatment failure for non-CNS S. pneumoniae
infections.
Special considerations for HIV infected/exposed children

Revised World Health Organisation Treatment recommendations for CAP


- Children with fast breathing, no chest indrawing or general danger sign: oral amoxicillin: at least 40mg/kg/dose twice daily (80mg/kg/day)
for 5 days (In areas with low HIV prevalence, give amoxicillin for three days).

194
Compiled by Obasi. D. C. Chinedu

failure on first-line treatment with amoxicillin- Refer/treat with 2nd line


- Children age 2–59 months with chest indrawing pneumonia:oral amoxicillin: at least 40mg/kg/dose twice daily for five days
- Severe pneumonia: treated with parenteral ampicillin (or penicillin) and gentamicin as a first-line treatment
Ampicillin: 50 mg/kg, or benzyl penicillin: 50 000 units per kg IM/IV every 6 hours for at least five days
Gentamicin: 7.5 mg/kg IM/IV once a day for at least five days
****Ceftriaxone should be used as a second-line treatment in children with severe pneumonia having failed on the first-line
treatment****
- Ampicillin (or penicillin when ampicillin is not available) plus gentamicin or ceftriaxone as first-line antibiotic regimen for HIV-infected
and -exposed infants and for children under 5 years of age with chest indrawing or severe pneumonia.
- Failed response to ampicillin or penicillin plus gentamicin, ceftriaxone alone is recommended for use as second-line treatment.
- Empiric cotrimoxazole treatment for suspected Pneumocystis jirovecii recommended for infants 2 months up to 1 year
- Empirical cotrimoxazole treatment for Pneumocystis jirovecii pneumonia (PCP) is not recommended for HIV-infected and -exposed
children over 1 year of age with chest indrawing or severe pneumonia.

…a flash back to the case review: Admitting diagnosis was severe bronchopneumonia
Empirical treatment: IV Ceftriazone and IV Genticin; then IV Vancomycin was introduced and Genticin was discontinued. Made remarkable
improvement
Billed for discharge on the 5th day but noted to have increasing respiratory distress, bulging right anterior chest wall.
What is the likely diagnosis?- Complicated Pneumonia- Pneumothorax

What are the complications of pneumonia?


Parapneumonic pleural effusion: is a collection of fluid in the pleural space in association with an underlying pneumonia. Pleural space
normally contains only 0.3 ml/kg of body weight of pleural fluid. An imbalance between pleural fluid formation and its subsequent drainage
more fluid accumulation.
May progress to become empyema. most common complication of bacterial pneumonia. Predominately exudative and occur in as many
as 50-70% of patients admitted with a complicated pneumonia.
Mechanism of fluid accumulation in the pleural space:
Increased drainage of fluid into the space (capillary permeability)
Increased production of fluid by cells in the space - Decreased drainage of fluid from the space
A combination of the above

Empyema: Pus in the pleural space; It occurs in 5-10% 0f children with bacterial pneumonia.
Common causes are Staphylococcus aureus, Streptococcus pneumoniae, and mycobacterium tuberculosis.
Less commonly caused by streptococcus pyogenes, H. influenza type B (if under-immunized); and anaerobes (e.g. Fusobacterium species,
Bacteroides melaninogenicus) in aspiration pneumonia.
Stage 1 (exudative stage) reflects the vascular permeability that accompanies the inflammatory response associated with infection.
Neutrophils and other inflammatory cells migrate into the pleural space and release oxygen metabolites, which further increase capillary
permeability and allow protein-rich fluid to enter the pleural space.
This increases the oncotic pressure in the space and draws even more free-flowing fluid into the area.
Stage 2 (fibrinopurulent stage) is characterized by frank pus and fibrin formations that begin to cover the pleura, causing loculations. The
coagulation cascade is activated causing decreased fibrinolysis and procoagulant activity.
The pleural fluid pH and glucose levels fall while LDH levels increase.
In Stage 3 (organizing stage), a fibrinous peel is deposited on the pleura, which restricts lung expansion and fluid reabsorption further

Symptoms: Small pleural effusion: asymptomatic


Large pleural effusion: pleuritic chest pain, abdominal pain, pain during inspiration or coughing
The child may prefer to lie on the affected side (to decrease respiratory excursions).
Cough, Fever, Respiratory distress, dyspnea, orthopnea, or cyanosis

Signs: Tracheal deviation to the opposite side. Bulging chest wall on the affected side with reduced movement
Decreased vocal fremitus. Dullness to percussion. Decreased or absent breath sounds. Decreased vocal fremitus
Pleural friction rub: Inflamed parietal & visceral pleurae rub against each other. leathery, rough in character
Heard in both inspiration and expiration. Disappears rapidly as the size of effusion increases
***If a child remains pyrexial or unwell 48 hours after admission for pneumonia, parapneumonic effusion/empyema must be
excluded***

Diagnosis
Chest radiograph (x-ray): -able to distinguish >200ml of fluid (blunted costophrenic angles). Chest radiographs acquired in the lateral
decubitus position are more sensitive and can pick up as little as 50 ml of fluid.
Pleural fluid analysis: Exudative cloudy/turbid & often clots if not heparinized. Pleural fluid/serum protein ratio, Pleural fluid/serum LDH
ratio, Glucose level.
Chest ultrasound: locates small amounts or isolated loculated pockets of fluid. able to give precise position of accumulation
Computed Tomography (CT) scan: Differentiates between fluid collection, lung abscess, or tumor
195
Compiled by Obasi. D. C. Chinedu

Cytology and culture

Chemical analysis

Decreased pleural fluid glucose below 60 mg/dL (3.33 mmol/L) or a pleural fluid/serum glucose ratio less than 0.5, is most consistent with
grossly purulent parapneumonic exudates
Pneumatoceles: are intrapulmonary air-filled cystic spaces
Organisms implicated: Staphylococcus aureus (most common), Streptococcus pneumonia, Haemophilus influenza, Escherichia coli, Group
A streptococci, Klebsiella pneumonia, Adenovirus, Primary pulmonary TB.
Typically asymptomatic. Occasionally become large enough to compress adjacent lung and the mediastinum
Rupture of a pneumatocele may cause a pneumothorax. May also be secondarily infected.
Other Complications: Lung abscess, Bronchopleural Fistula

Management of Complications
Pneumothorax, Pleural effusion/empyema: Tube thoracostomy
Refractory pleural empyema: Open thoracotomy with lung decortication, Thoracoscopy with adhesiolysis and pleural debridement
Pneumatocele: Antibiotic therapy with or without surgical intervention.

Strategies for Prevention of Pneumonia:


Specific vaccines: conjugate vaccines, S. pneumoniae, & H. influenza type b; Influenza vaccine
Other vaccines: measles containing vaccines, BCG, Pertussis
Measures to reduce risk: improve housing (good ventilation and avoid overcrowding and indoor air pollution;
-improve nutrition; exclusive breast feeding; micronutrient supplementation (Vitamin A & Zinc)
- HIV prevention

196
Compiled by Obasi. D. C. Chinedu

ACUTE (POST-STREPTOCOCCAL) GLOMERULONEPHRITIS


Dr. SAMUEL.N UWAEZUOKE, MB;BS. FWACP (Paed), Dip Th.

OUTLINE: - Introduction - Definition - Aetiology - Clinical features - Laboratory evaluation


 Treatment - Differential diagnosis - Prognosis

LEARNING OBJECTIVES: - To understand the pathogenesis of acute post-streptococcal glomerulonephritis


- To acquire the competence to make a diagnosis of the disease - To understand the treatment plan

CASE REPORT- 1: A.O is a six year-old boy who presented to the clinic with a week history of peri-orbital swelling which is more
prominent on waking up in the morning. He also had mild swelling of both feet. His urine output had reduced prior to the onset of the peri-
orbital swelling and the colour had changed to that of ‘coca cola drink’.
Major signs on examination were peri-orbital edema and pitting bilateral pedal edema, blood pressure of 140/90 mm hg and healed papules
on both lower limbs.

CASE REPORT- 2: C.N is a ten year old girl whose mother noticed that her face was getting puffy with peri-orbital edema since a week
ago. Her urine output was reduced but the urine colour was normal. Three weeks ago, she had odynophagia and was treated with antibiotics
at a chemist shop
On examination, she had peri-orbital edema and bilateral pitting pedal edema. She was mildly pale. Her blood pressure was 150/100. She had
no ascites

INTRODUCTION: Each kidney has about one million nephrons- the functional unit. The nephron receives blood through the afferent
arteriole. The afferent arteriole forms the capillary tufts or GLOMERULUS- reunite to form the efferent arteriole.
The major elements of renal function:
- Glomerular ultra-filtration - Tubular reabsorption - Tubular secretion
Glomerular capillary hydrostatic pressure generates an almost protein-free filtrate of plasma into the bowman’s capsule.
Main determinant of glomerular filtration rate (GFR): relative degree of constriction of afferent and efferent arteriole

Constriction of afferent arteriole leads to ↓ GFR while constriction of efferent arteriole leads to ↑ GFR.

DEFINTION OF AGN: A renal pathology characterized by abrupt onset of: Haematuria (variable degrees), Oedema, Hypertension,
Oliguria, following infection with a variety of organisms especially beta-haemolytic streptococci.

Aetiology of acute glomerulonephritis:


INFECTIOUS CAUSES:
Streptococci, staphylococci ,treponema pallidum, salmonella typhi, leptospirosis.
Plasmodium malariae, toxoplasma.
Hepatitis B and C , cytomegalovirus, parvovirus, Ebstein Barr virus
Infections of shunts, prostheses, bacterial endocarditis

PATHOGENESIS: post streptococcal acute glomerulonephritis (PSGN)


As the prototype of agn, remains the commonest cause in developing countries. Often follows pharyngitis, impetigo or rarely a middle ear
infection caused by group A beta-hemolytic streptococci. The nephritogenic strains of streptococci are capable of producing AGN. Host
factors, genetically determined, are important in the formation of antibodies to streptococcal antigens (e.g. Streptolysin O). Glomerular injury
in PSGN results from deposition of immune complexes in the glomerular capillaries.
Nephritogenic antigens derived from streptococci may bind directly to sub-epithelial glomerular sites. Antibodies formed against these
antigens combine (immune complexes) following
activation of complement and results in an
inflammatory response. Infiltration of neutrophils,
proliferation of glomerular cells and expansion of
mesangial matrix follow (glomerular injury).

CLINICAL FEATURES: History of sore throat


or pyoderma (impetigenous lesions) is noted in most cases
Latent period in the history of sore throat is 7 to 14 days while that of pyoderma is 2 to 4 weeks.
Peak age incidence: 5 to 12 years. Rare below the age of 3 years. Male preponderance.
Gross hematuria and mild facial edema are the most common presenting features. Urine usually cola coloured or reddish brown.
Oliguria (less than 0.5ml-1ml/kg/hour) or sometimes anuria
Hypertension (from volume overload) may lead to headache

Atypical presentations (complications of AGN) include:

197
Compiled by Obasi. D. C. Chinedu

- Acute pulmonary edema - Hypertensive encephalopathy - Acute renal failure


- Nephrotic syndrome (so-called nephritic nephrotic syndrome)

LABORATORY INVESTIGATIONS: Urinalysis- mild proteinuria


 Urine microscopy- dysmorphic red cells, red cell casts, and neutrophils. Hyaline and granular casts may also be seen.
 Serum electrolyte, urea and creatinine- elevated urea/creatinine, hyperkalemia, metabolic acidosis (in patients with ARF).
 Hematology- anemia due to hemodilution
 Imaging study (Chest X-ray)- may show cardiomegaly and pulmonary congestion
 Serology (ASO titre and C3 levels)- elevated ASO titre within 3 to 5 weeks after streptococcal infection, and decreased serum C3 levels
 Renal biopsy- not required in typical cases but indicated under special conditions

Indications for renal biopsy in AGN: Associated systemic features like fever, rash, joint pain, heart disease.
- Normal ASO titre and C3 levels - Mixed picture of AGN and Nephrotic syndrome
- Delayed cases of resolution (oliguria, hypertension and/or azotemia beyond 2 weeks, gross hematuria past 3-4 weeks, low C3 levels past 6-
8 weeks and microscopic hematuria/proteinuria beyond 6-12 months)

TREATMENT:
GENERAL MEASURES: Fluid balance: strict input/output if oliguria is present, daily weight measurement.
Diet: restriction of sodium intake in all children with edema or hypertension, restriction of foods high in potassium until oliguria resolves
Bed rest: if hypertension, edema or cardiac failure are present
Drug treatment: Eradication of streptococcal infections using penicillin or alternatively erythromycin.
Intravenous furosemide(1mg/kg) for edema and circulatory congestion
For hypertension, the use of vasodilators (hydralazine, nifedipine, ACEI) may be effective

Differential Diagnosis: - Acute interstitial nephritis - Shunt nephritis - Nephritis in SBE


- Glomerulonephritis associated with hepatitis B or C - Ig A nephropathy
- Hemolytic uremic syndrome (HUS)

Hemolytic Uremic Syndrome: Micro-angiopathic hemolytic anemia. Thrombocytopenia. Azotemia. Oliguria. Preceding diarrheal disease.
Usually occurs in children less than 4 years.

PROGNOSIS: Most cases resolve within the first week. Gross hematuria rapidly clears but microscopic hematuria may be detected for 6 to
12 months. The long-term prognosis of PSGN in children is excellent. Even those with severe disease completely recover

CASE 3: K. N, is a seven year old boy with recurrent history of generalized body swelling for three months. He has had normal urine output
and no change in urine colour
Major findings on examination were massive peripheral and central edema with blood pressure of 80/50 mm hg. Ascites was demonstrable
by fluid thrill, and patient also had bilateral chest dull percussion notes

NEPHROTIC SYNDROME: The clinical and biochemical features of nephrotic syndrome result from heavy proteinuria (40mg/m2/hour or
1g/m2/24hours).
A syndrome characterized by massive proteinuria (3+ to 4+ protein), hypoalbuminemia, edema and hypercholesterolemia (hyperlipidaemia).
Microscopic hematuria, hypertension and raised blood urea levels are occasional findings

Not a distinct renal disease. A clinical and biochemical state that may develop during the course of several different renal diseases of known
and unknown aetiology,

BIRD’S EYE VIEW ON CAUSES OF NEPHROTIC SYNDROME

CAUSES/ CLASSIFICATION IN CHILDREN


CONGENITAL:
- Finnish Type (Autosomal Recessive)
- Microcystic Kidney Disease (Congenital Nephrosis)
198
Compiled by Obasi. D. C. Chinedu

- Intrauterine Infections Such As Syphilis, Cmv, Toxoplasma


ACQUIRED: Idiopathic or Primary (>90% of cases) and Secondary

PRIMARY/IDIOPATHIC: Minimal change nephropathy (MCN), Mesangial proliferative GN, Focal segmental glomerulosclerosis (FSG),
Membranoproliferative GN (MPGN), and Membranous nephropathy
SECONDARY: Acute post-streptococcal GN, Systemic lupus erythematosus(SLE), Henoch-Schonlein purpura (HSP), amyloidosis,
hepatitis B, HIV, P. malariae, SCD, Bee stings, Gold salts/ heavy metals such as mercury etc.

Minimal change nephrotic syndrome (MCNS): Occurs in about 85% of children with idiopathic nephrotic syndrome. Often steroid
sensitive(SSNS). Onset usually between the age of 2-6 years. More common in boys
Hypertension, hematuria and raised urea levels are rare
Light microscopy shows little or no abnormalities

CURRENT CONCEPT ON PATHOGENESIS


 Glomerular filtration barrier as a trilaminate structure: glomerular basement membrane, fenestrated endothelium (inner surface),
podocytes (outer surface)
 Disruption of the podocyte structure and function as the basis for nephrotic syndrome (podocytopathy)
NEPHROTIC SYNDROME AS A PODOCYTOPATHY
The podocyte: maintains the structural integrity of the glomerular filtration barrier
Podoctye injury or loss: proteinuria
Podocyte injury occurs in many immunologic (MCN, FSGS) and non-immunologic diseases of the kidney

Clinical features of MCNS: Insidious onset with periorbital swelling and facial puffiness
Swelling gradually increases to involve the extremities and abdomen and if untreated may become massive resulting in anasarca.
May occasionally be associated with gross hematuria and oliguria (mixed picture of nephrotic syndrome and acute nephritis).

LABORATORY EVALUATION
Urinalysis: dipstick test (3+/4+), spot urine test or protein/creatinine conc (ratios <0.5 in children <2 years and <0.2 in older children are
normal. A ratio >2 suggests nephrotic range proteinuria), 24-hr urine protein estimation.
Urine microscopy and culture: microscopic hematuria, exclude UTI
Blood chemistry: serum urea, creatinine and electrolytes, cholesterol Serology: Hbsag, C3 levels, HIV
Imaging: CXR, Abdominal US Renal biopsy- not usually indicated for MCNS

RENAL BIOPSY- EARLY INDICATIONS: - Age at onset <1yr or >16 yr.


- Gross haematuria, persistent microscopic haematuria or low serum C3 - Sustained severe hypertension
- Renal failure not attributable to hypovolemia - Suspected secondary causes

TREATMENT:
Initial episode: Control of massive edema and infection before starting steroid therapy. Oral prednisolone 2mg/kg in single or 2 divided
doses for 6weeks and single morning dose alternate days for the next 6 weeks.
Prolonging the treatment for 6 months may result in a longer remission and fewer relapses

PATTERNS OF RESPONSE TO STEROID THERAPY


Early responders- 1-2 wks. Late responders- 4-6 wks.
Remission: Protein-free urine (urine protein negative or trace) for 3 consecutive days
Relapse: Significant proteinuria (urine protein 1+ or more) for 3 consecutive days
Frequent relapser: 2 or more relapses within 6 months of initial episode or more than 3 relapses within any 12 month period.
Steroid dependent: 2 consecutive relapses during alt. day pred. or within 2 weeks of stopping therapy

Common Infectious Triggers of Relapse: Upper Respiratory Tract infection, Urinary Tract infection

Steroid sparing agents used in frequent relapses and steroid dependence include;
- Levamisole - Cyclophosphamide - MMF- mycophenolate mofetil
- Calcineurin inhibitors- tracolimus, cyclosporine (Cs A) - Rituximab

SUPPORTIVE TREATMENT: Use of ACEI: Enalapril, Lisinopril (anti- proteinuric effect)


Management of edema: loop diuretics +/- thiazides, intravenous infusions of 20% albumin (0.5-1g/kg over 2-4 hrs.) or pooled plasma
transfusion
Dietary management: high biological value protein, salt restriction, supplements of vitamins and micronutrients
Management of complications

Complications of Nephrotic syndrome: Infections: spontaneous bacterial peritonitis, cellulitis, pneumonia


- Hypovolemia - Thromboembolism - Hyperlipidemia
199
Compiled by Obasi. D. C. Chinedu

- Iatrogenic: hypertension, fungal sepsis, hypovolemia

DIFFERENTIAL DIAGNOSIS: - Angioneurotic edema - Protein-losing enteropathy


- Chronic liver disease - Malnutrition with edema - Congestive heart failure

PROGNOSIS: The final outcome of steroid sensitive nephrotic syndrome (SSNS) is excellent. Most patients stop getting relapses between
the ages of 14 to 20 years. Fully recover without any residual dysfunction. Some may continue to have relapses into adulthood.

POST TEST (MCQ)


The following infectious agents are implicated in the etiology of acute nephritis
a. Plasmodium vivax b. Treponema pallidum c. Toxoplasma gondii d. Ebstein barr virus
e. Clostridium difficile

Concerning acute post-streptococcal glomerulonephritis


A. A female preponderance is the rule
B. Peak age incidence is between 1 to 3 years
C. Latent period in the history of sore throat is 2 to 4 weeks
D. It rarely occurs above the age of 3 years
E. Acute pulmonary edema can be an atypical presentation

The following are true about the outcome in acute post-streptococcal glomerulonephritis
A. Most cases resolve within the first week
B. Microscopic hematuria may persist for 6 months
C. The long term prognosis in children is poor
D. Nephrotic syndrome may occur during the disease
E. Low c3 levels after 6 to 8 weeks is an indication for renal biopsy

Causes of secondary nephrotic syndrome include


a) Idiopathic thrombocytopenic purpura b) Viperine snake bite c) Gold salts d) Hemosiderosis
e) Amyloidosis

In minimal change nephrotic syndrome


A. The usual age of onset is between 2 to 6 years B. Girls are more commonly affected
C. Hypertension and azotemia are rare D. The urine protein/creatinine ratio is less than 2
E. There is usually a high protein selectivity index

The following histo-pathological patterns are seen in primary or idiopathic nephrotic syndrome
A. Focal segmental glomerulosclerosis B. Membranoproliferative glomerulonephritis
C. Mesangio-capillary glomerulonephritis D. Lupus nephritis
E. Minimal change nephropathy

A 3 year old girl presents with oliguria and facial puffiness. Major signs include pallor, normal blood pressure and petechial rash.
The diagnosis is characterized by
A. Antecedent history of vomiting B. Auto-immune hemolytic anemia
C. Micro-angiopathic hemolysis D. Prolonged prothrombin time
E. Thrombocytopenia

The management protocol in acute post-streptococcal glomerulonephritis comprises


A. Oral prednisolone to reduce inflammation B. Oral penicillin to eradicate streptococcal infections
C. Iv mannitol for circulatory overload D. Dietary restriction of high biological value protein
E. Dietary restriction of sodium intake

Children with nephrotic syndrome who present with frequent relapses


A. Usually have 2 or more relapses in 12 months B. Usually have 2 or more relapses within 6 months
C. May require adjunct drugs like piperazine D. May respond to cyclosporin A
E. Requires only oral prednisolone for complete remission

An 8 year old boy who presents with microscopic hematuria for more than 12 months requires the following tests
A. Renal biopsy B. Serum c3
C. Platelet count D. Clotting profile
E. Genotype

200
Compiled by Obasi. D. C. Chinedu

Dr Ikefuna

Introduction: Leukaemia is the presence of 25% malignant haematopoetic cells(blasts) on bone marrow aspirate. Is a childhood
emergency commoner among the caucasians. Age incidence is 2-10yrs with a peak of 4yrs and a M:F 1.3:1. May be associated with some
syndromes eg Down syndrome, Fanconi anaemia. ALL is an important model around which the concept of chemotherapy for other
malignancies was developed.

Aetiology: ALL arises from uncontrolled proliferation of immature lymphocytes. Aetiology not fully understood. However, known
predisposing factors include
-radiation injury -chemicals ie benzene -alkylating agents ie cyclophosphamide
-congenital abnormalities eg Down syndrome, Fanconi anaemia, Wiskott –Aldrich syndromes

Pathology: Involves severe disruption of bone marrow function by leukaemic cells and can be classified into
-acute lymphoblastic lymphoma-80%
-acute non lymphoblastic lymphoma-17%
-chronic leukaemias -3%-this is rare in children especially CLL

Classification
Using the morphology of the blasts as well as their surface markers (FAB), ALL can be classified into
L1—small cells ,regular nuclei and scanty cytoplasm
L2—large heterogenous cells with one or more nucleoli
L3—large homogenous cells, regular nuclei and prominent cytoplasmic vacuoles. This has the worst prognosis

For ANLL, classification ranges from M1 to M7


-M1—myeloblastic with maturation -M2---myeloblastic with some differentiation
-M3—acute promyelocytic -M4---myelomonocytic with differentiation
-M5a—monoblastic undifferentiated -M5b---monoblastic differentiated
-M6---erythroleukaemia-erythroid dominance with 30% blasts -M7---megakaryocytic leukaemia

Classification by risk/prognostic factors


Factors Good risk High risk
Age >1yr, <10yrs <1yr, >10yrs
WBC <50000cells >50000cells
Sex F M
Race white negroid
Cell type L1 L2, L3
CNS disease absent present
Hb <10gm >10gms
Platelet count >100000 <100000
Lymphoma Syndr absent present

Standard risk patient at diagnosis include: Wbc count <100000, normal C X Ray, No CNS involvement as well as No B or T cell features.

Clinical features: About 67% of patients will have symptoms and signs of their illness for <6wks at time of presentation. Features include
palor, petechiae, purpura, epistaxis, haematuria, intracranial bleeding. Others include bone pains, fever, generalised lymphadenopathy,
hepato splenomegaly(60%)
In ANLL( e.g AML), there may be chloroma – masses which are greenish on section ; usually found in the orbit(due to myeloperoxidase
present in d tumor mass).
Testis may be unilaterally or bilaterally enlarged due to leukaemic infiltration. Superior vena cava syndrome may arise from mediastinal
adenopathy. Adenopathies may cause tachypnoea, orthopnoea and respiratory distress
Leukaemic infltration of cranial nerves may lead to cranial nerve palsies, nuchal rigidity may occur

Investigations: FBC, ESR.


Bone marrow aspiration- if more than 25% of marrow cells are blasts, diagnosis is confirmed
LFT—baseline values and in hepatic involvement
C X Ray- mediastinal adenopathies
X Ray of long bones and spine may show demineralization, periosteal elevation, growth arrest lines or compression of vertebral bodies.
Brain CT—in brain involvement

Treatment-ALL: Treatment is divided into


Induction of remission: 4 – 6 weeks ( to achieve bone marow remision) (MOAP). And consists of Vincristin (Oncovin) 1.4mg/m2,
prednisolone 40 -60mg/m2, L asparginase. IT methotrexate is given wkly
201
Compiled by Obasi. D. C. Chinedu

Consolidation phase: 4-8 wks (to eradicate cels that survived induction) (6CM). Drugs used include cytosine arabinoside, 6 mecarptopurine
and intrathecal methotrexate
Maintanance phase: lasts for 2-3 years and aimed at preventing reocurence. (6M+MO). Achieved with 6MP, oral methotrexate as well as
monthly pulses of vincristin and IT methotrexate. Other supportive measures include blood transfusion, treatment of infections and
antipyretics [don’t use NSAIDS].

ANLL: Cells of ANLL are positive for peroxidase and Sudan black stains. Features are same as ALL:
Gingivial swelling –due to leukaemic cell infiltration. Chloroma-presenting as proptosis
Defective coagulation hence the need for coagulation profile, especially in M3.
Megaloblastic features may mimic folate or Vit B Co deficiency.

Treatment-ANLL
Various drug combinations have been used such as (AC6/ACT) Adriamycin or Actinomycin on day 1, Cytosine Arabinoside on days 1 -5, 6
MP or Thioguanine on days 1 -5 (orally).
Cycles are repeated 2wkly from Day 1, a total of 6cycles are given i.e on day 6-14 (9days)drug free days.
Other modes of treatment include: Radiotherapy, Bone marrow transplant

Complications: These could be drug related or tumour related eg


cyclosphosphamide-(HANG): haemorrhagic cystitis, alopecia, gonadal dysfunction, nausea, vomiting.
Methotrexate (DHMM): mucositis, myelosuppression(same as cyclophosphamide), dementia, hemiparesis.
Vincristin (FANS-CAP): Foot drop, Cramps Cranial nerve palsies, Abd pains, Ataxia, Neurotoxicity, Paralytic ileus, SIADHS.

Tumour related complications


Tumour lysis syndrome: arises from rapid breakdown of tumour cells. Patient at risk include those with Non Hodgkins lymphoma esp
Burkitts Lymphoma, leukaemia. Features include increased incr potasium, phosphate hyperuriceamia and decreased calcium.
consequence is the occurrence of renal failure
Prevention; allopurinol, alkalinization of urine, increased fluid intake.
Complications include azotaemia, oliguria, renal faikure.
Management: dialysis

LYMPHOMAS: Two broad classes are recognized ie Hodgkins lymphoma & non Hodgkins lymphoma
Hodgkins lymphoma: commoner in older children and rare before 5yrs of life. Has two peak ages -15 to 34 and after 50 years. commoner in
males. aetiology include viruses of low virulence and infectivity as well as prolonged use of hydantoin drugs
histological types: four histological types are recognized with their implications for diagnosis. central histologic feature in all the types is the
Reed Sternberg cell.
Reed sternberg (diagnostic when seen in a lymph node) contains two nuclei each with a prominent nucleolus, has a distinct nuclear
membrane, cytoplasm is relatively abundant.
Other cells namely lymphocytes, plasma cells and tissue mononuclear cells are present. Types includes;
- Lymphocyte predominant- mature lymphocytes with scanty R-S cells. Has the best prognosis
- Nodular sclerosis: broad bands of collagen divide the lymph nodes - Mixed celularity
- Lymphocyte depleted- worst prognosis with frequent anaplastic R-S cells

Clinical features: Cough, dysphagia, bronchial compression from mediastinal node involvement
Cervical, axillary, supraclavicular lymphadenopathy- nodes are firm, non tender and discrete
Systemic symptoms ie wt loss, night sweats and periodic fever may occur. Bone pains have been reported
Jaundice has been observed in extra hepatic biliary tract obstruction by enlarged nodes
Cellular immunity is impaired –increased susceptibility to infections

Clinical staging: Using the Ann Abbor Staging System, four stages are recognized viz
Stage I--- involvement of a single nodal or extra nodal site on one side of the diaphragm
Stage II: Two distinct nodal sites or one nodal site plus an extra nodal site on one side of the diaphragm
Stage III: involvement of lymph on both sides of the diaphragm but confined to the RES including the spleen
Stage IV: diffuse organ involvement with or without lymph node involvement
Each stage is sub divided into A(absence) or B (presence) of night sweats, weight loss of at least 10% in previous 6 mths, poor weight gain,
pruritus, fever or bone pains

Investigations: C X Ray—lung or mediatinal lymph nodes


Lymph node biopsy –diagnostic and to determine the histological type
Lymphangiography – determines lymph node involvement and enables staging
CBC, LFT, Serum E/U /Cr - CCS/LLL
202
Compiled by Obasi. D. C. Chinedu

Treatment:
For stages I & II – radiotherapy, surgery - Stages III & IV -- chemotherapy

Common chemotherapeutic regimen include


A: MOPP (mustine(mechlorethamine), oncovin, procarbazine -100mg/m2, prednisolone)
b: COPP (cyclophosphamide, oncovin, procarbazine, prednisolone)
O,C given on days 1 & 8. Pro, P orally on day 1 -14.
No drugs are given on days 15-28, In all 6 courses are given

Non-Hodgkins lymphoma (Burkitt’s lymphoma): Commonest childhood malignancy in this environment. Fastest growing tumour known
to man. Commoner in males and derived from the B lymphocytes. Cells are often of the high grade small non cleaved variety with a doubling
time of <24hrs. Age incidence -5 to 9 years with a peak of 7 years.
Aetiology: None is specific but some factors have been associated with,
- repeated malaria infection as this causes lymphocyte hyperplasis - malnutrition
- Epstein Barr virus as patients are known to have high levels of EBV antibodies in their serum

Clinical features: Interval between onset of symptoms and presentation is short and may be in form of a
Jaw mass: involves the maxilla &/or mandible. Proptosis may occur. Loose or loss of tooth. Bleeding from the gum.
Abdominal mass: involves the retroperitoneal organs ie pancreas, kidney etc. Liver with/without ascites may occur.
CNS involvement—flaccid paralysis, multiple cranial nerve palsies/ other neurological deficits.
Other organs affected include the ovaries, thyroid, testis, breast

Clinical staging: Using the Zeiglar Staging System, the following stages are recognized
Stage A—single extra abdominal focus Stage B---multiple extra abdominal foci
Stage C –abdominal involvement Stage D—CNS, bone, bone marrow
Stage AR –abdominal tumour with >90% resectable at surgery

Investigations: Radiography- Jaw X Ray (AP & oblique) shows dental anarchy and loss of lamina dura (a radiolucent halo around the
unerupted tooth)
Others: C X Ray, IVU, Ultrasonography, Cytology. FBC, LFT, Serum EUCr.
Biopsy: shows the ‘starry –sky’ appearance ( the star represents vacuolated macrophages washed off during tissue preparation ; the sky are
the Burkitts cells

Treatment: Surgery for biopsy and de bulking especially in Stage AR.


Chemotherapy –COMP, COAP. IT methotrexate in all cases. - Radiotherapy, Immunotherapy
Regimen: Two weekly treatment with COMP (6 - 7courses)
Each course is made up of: IV cyclophosphamide, IV Vincristine, IV methotrexate, oral prednisolone.
Weekly IT methotrexate for the 1st 6 weeks of treatment.
(Adriamycin,Vinblastin,Daunorubicin, Bleomycin new regimen).

203
Compiled by Obasi. D. C. Chinedu

OTHER CHILDHOOD MALIGNANCIES


DR IFEOMA EMODI

RETINOBLASTOMA: Commonest primary intra-ocular malignancy of infants & children. Commonest in children aged b/w 1-3yrs peak
@ 1yr can have bimodal occurrence at 9-11yrs. May be sporadic (>in Africans) or familial. The R gene is located on the long arm of
chromosome 13. Inherited as AD with variable penetrance can skip generations.

PATHOLOGY: The tumor arises from the neural crest of the retina. It is a slow growing tumor made up of small round or polygonal cells.
In 30% of cases it is bilateral (AD). Tumor may spread via the optic nerve to the CNS or to the other eye via the optic chiasm.

CLINICAL FEATURES: Varies according to stage at which T is detected. Most common mode in this environment is an orbital mass.
Early stages present as: Leukokoria, Strabismus, Decrease or loss of vision.

DIFFERENTIAL DIAGNOSIS: Visceral larva migrans, Bacterial panendophthalmitis, Cataract, Retinopathy of prematurity.
The Reese-Ellsworth staging system is used for intra-ocular T and is done at fundoscopy. A simple staging system for extra-ocular tumor is
done clinically
Stage I: confined to retina Stage II: confined to globe
Stage III: confined to orbit Stage IV: distant metastasis

INVESTIGATIONS: Fundoscopy: done under GA with pupils widely dilated


Skull Xray: may show orbital erosion or calcification of tumor. C T scan: with or without contrast
Others: Orbital US, CSF cytology (lumbar puncture). Biopsy: , Baseline investigations: FBC, LFT, SEUC etc
ECG as Adriamycin will be used

DIAGNOSIS: Can be made with indirect ophthalmoscopy when ocular medium is clear. If haemorrhagic ultrasonography and C T will
show calcification within the tumor. Histology of biopsy specimen

TREATMENT: Radiotherapy for localised tumors but can induce cataract and 2nd malignancy.
Cryotherap, Photocoagulation. Enucleation or exenteration with at least 10mm of optic nerve.
Chemotherapy: (VAC+It MTX) vincristine, adriamycin and cyclophosphomide used in combination with intrathecal methotrexate
Other drugs used: carboplatin,etoposide, cisplatin

PROGNOSIS: Prognosis is related to size and extent of tumor. It can be cured if confined to the eye. Poor prognosis if spread to orbit or
optic nerve. Persons with bilat T have high risk of 2 nd malignancy and extrabeam radiation increases the risk of osteogenic CA,
rhabdomyosarcoma, leukemia, glioma, etc

NEPHROLASTOMA: Also known as Wilm’s tumor. Commonest primary intra-renal malignancy in childhood. Originating from primitive
renal mesenchymal tissue. Majority present before the age of 5 with a median age of 3-4 yrs. It is a solitary growth that may occur in any
kidney. It is sharply demarcated and invariably encapsulated

CLINICAL FEATURES: Presents as a slow growing abdominal mass, with or without painless haematuria
It does not extend beyond the midline. Other cong abnormalities that may be found include: aniridia, hypospadias, hemi-hypertrophy, ectopia
vesica, horse-shoe kidney, double ureters, and NS. Hypertension may be present in 25% of pts. Metastasis usually to lungs, liver and lymph
nodes but painful bone secondaries may be present.

NATIONAL WILM’S STUDY TUMOR STUDY STAGING SYS


Stage I: confined to kidney, within the capsule, completely removable
Stage II: capsule penetrated completely removable
Stage III: residual, non-haematogenous spread confined to abdomen (+LN)
Stage IV: haematogenous spread to the lungs, liver bone and brain
Stage V: Bilateral renal involvement

INVESTIGATIONS: Urinalysis: microscopic or gross haematuria. Abdominal ultrasonography


Plain abdominal X – ray: enlarged renal shadow. Chest X-ray: metastasis to lungs
IVU: gross distortion of pelvis and calyces. Kidney may be non-functional
Other routine investigations

TREATMENT: Nephrectomy if tumor still intact. Radiotherapy started 10 days after surgery (Adjuvant Radiotherapy). If bilateral tumor,
wedge resection of better and total removal of the other.
Anti-hypertensives where necessary. Chemotherapy mainstay of treatment for (VAC + It MTX) disseminated tumor with vincristine,
Actinomycin D cyclophosphamide and IT

204
Compiled by Obasi. D. C. Chinedu

PROGNOSIS: Depends on histology and stage of disease. Age < 2yrs at diagnosis appears to improve over all prognosis. Disease free
interval of 2 yrs appears to correlate well with complete cure

NEUROBLASTOMA: Median age is 3 – 4 yrs, usually < 5 yrs. Tumor consists of primitive neuroblastoma cells.
Occurs in the adrenal medulla or along the sympathetic chain. It is fast growing and can extend beyond the midline (unlike wilm’s). Mass is
firm to hard craggy and irregular may be painful.

CLINICAL FEATURES: In cases of bone metastasis, pt may present with bone pain, multiple swellings in the scalp or flat bones. In the
vertebral column, it extends into the spinal canal in a dumb-bell fashion causing paraplegia
In the cervical ganglia it can give rise to Horner’s syndrome.
Other signs include encephalopathy, diarrhoea or hypertension. Frequently there is fever malaise, anorexia, weight loss and cachexia. Staging
is with the Evan’s staging system

INVESTIGATIONS: Investigations include: abdominal U/S. IVU: will show the kidney displaced downwards and laterally on affected
side. Plain abdominal X ray may show stipple calcifications in 50%.
Urinary 24 hrs Valinyl Mandelic Acid (VMA) is elevated in about 70% of cases.
Skeletal survey may show lytic lesions. Bone marrow aspirate or biopsy/ Biopsy of tumor
Other routine investigations plus ECG

TREATMENT: Surgery for localised tumor. Chemotherapy for tumors that has spread drugs used include Vincristine, Adriamycin and
cyclophosphamide (VAC). Most tumors are radiosensitive

RHABDOMYOSARCOMA: This is a soft tissue sarcoma (embryonal mesenchymal tissue) arising from striated muscles. Common
primary sites include the head neck (parameningial, orbit, pharyngeal), GUT, extremities, vagina and trunk. There are 3 histologic subtypes:
embryonal, alveolar, pleomorphic

CLINICAL FEATURES: Age range is from 2 – 20 yrs. Males are more affected than females. The most common feature is a mass at the
site of the tumor which may or may not be painful.
Sign and symptoms depend on site involved and +/- metastasis: Proptosis, ptosis, chemosis, cranial nerve palsies, dysuria, haematuria, rectal
or vaginal bleeding can occur depending on tumor site.
25% of patients have metastasis at the time of diagnosis to BM, liver and later the brain
The Intergroup Rhabdomyosarcoma Study (IRS) is used to stage the patients

INVESTIGATIONS: biopsy is most important. Other will depend on site of mass. Routine investigations
TREATMENT: Depends on the stage. Includes; Excision, Radiotherapy, Chemotherapy.
Regimen is as for wilm’s tumor with Vincristine, Actinomycin D Cyclophosphamide, cisplatin, ifosfamide

BONE NEOPLASIA: Most primary tumors are osteosarcoma and Ewing’s Sarcoma. Most occur in the second decade of life with a male
predominance.

OSTEOSARCOMA: Seen in periods of rapid bone growth. Found in metaphysial end of bones. Common primary sites are distal femur,
proximal tibia and humerus. Occasionally seen in flat bones.
Predisposing factors include hereditary retinoblastoma, Fraumeni syn, Paget’s disease and local irradiation
It is a highly malignant spindle cell neoplasm. Typically breaks through the cortex & invade surrounding soft tissue
CLINICAL FEATURES: Most common presenting signs are localised swelling & pain. Often there is a history of trauma. The limb shows
localised swelling, tenderness and limited range of movement. Metastasis is to the lungs or other bones

INVESTIGATIONS: X-ray of the affected bone will reveal sclerosis of bone and periosteal new bone formation
Biopsy is required for definite diagnosis and should be done by the surgeon who will do the definite surgical procedure.
C T scan of lungs to rule out metastasis, MRI to assess tumor extent. Bone scan to rule out bone metastasis

TREATMENT: Chemotherapy and complete resection. Drugs include: cisplastin, ifosfamide, methotrexate, doxorubicin, etoposide
Pre operative chemo may allow salvage surgery. Extensive rehabilitation post op is necessary b/c of phantom limbs
Prosthesis and gait training. Most are not radiosensitive

LIVER TUMORS: 90% of hepatic malignancies are either Hepatoblastoma or Hepatoma (HCC). Hepatoblastoma is most common primary
malignant liver in children aged 0 – 3 yrs with a median age of 1 yr. Peak age for hepatoma is 5 – 18 yrs with median of 12 yrs. HCC may be
preceded by hepatitis B virus infection esp when perinatally acquired. HCC may also be preceded by conditions leading to cirrhosis.
Hepatoblastoma may occur with hemi hypertrophy Beckwith Weidermann syn
Clinical features include: Painless right sided or midline upper abdominal mass (pain in HCC). Anorexia, weight loss vomiting,

INVESTIGATIONS: Elevated alpha – fetoprotein and serum ferritin


LFT may be normal with 30% having increased transaminases and 25% increased bilirubin level
205
Compiled by Obasi. D. C. Chinedu

Leukocytosis and anaemia may be present


Abdominal Xrays may show hepatic enlargement and calcifications (less common in HCC)
CT scan of abdomen may reveal T commonly in R lobe but may be multicentric esp in HCC
Abdominal ultrasound. Radionuclide bone scan for metastasis. Needle biopsy

TREATMENT: Gross surgical resection of either is nec before any hope of cure. Chemotherapy may be nec initially to shrink T. Agents
used include cisplatin, adriamycin, dactinomycin, cyclophosphamide, vincristine, 5-fluororubicin. Tumor shrinkage can be monitored by
radiography and decrease in α feto protein
Radiation and liver transplantation is being investigated for pts whose T cannot be completely resected following chemo

PROGNOSIS: For HCC complete surgical excision is the only effective treatment. This is possible in about 33% of cases. Response to
chemo is seen in 30 – 40%. This is short lived. Overall prognosis is poor

TUMOR LYSIS SYNDROME: A syndrome of; Hyperuricaemia, Hyperkalemia, Hyperphosphatemia, Hypocalcemia, Acute renal
failure. Occurs following tumor cell lysis especially in lymphoma and leukemia. The abnormalities are especially severe in B cell
lymphoma/leukemia as a result of rapid T cell lysis & extreme sensitivity to chemotherapy/radiotherapy. Lysis following chemo can result in
renal failure and sudden death from >K+ or < Ca.
Clearance of products from T lysis depends on renal excretion, hepatic meta, and phagocytosis by the RES. Renal clearance is the primary
mech for excretion of uric acid, K and PO4. Optimum mgt for Pts at risk depends on evaluation of renal fn prior to starting chemo, adequate
hydration (maintenance fluid x 1.5) and close monitoring.

MANAGEMENT: Prevention therapy. Hydration of pt begins at least 24hrs before chemotherapy 3 L / M 2


Allopurinol 100mg tds as above. Alkalinization of urine with IV NaHco3, 2-3ml/kg.
Frequent monitoring of K, Ca, Po4 levels as well as renal fn when treatment starts

COMPLICATIONS of TLS: Renal failure with azotemia, oliguria, hyperuricaemia, hyperK, hypoCa,
Treat hyperkalemia acutely when serum levels exceed 6mEq/L or ECG changes. Use ion exchange resin – kayexalate (and monitor Na+
level). Consult nephrology unit – dialysis.
Lowering PO4 with oral phosphate binders aluminum OH, and oral Ca supplementation can treat hypoCa
If pt is highly symptomatic slow diluted IV Ca can be used with cardiac monitor for bradycardia

CANCER TREAMENT: Cytotoxic therapy


Surgery – debulking, biopsy
Radiotherapy
Hormonal therapy
Most chemo agents are non-selective and destroy a good fraction of rapidly dividing normal cells of BM, gut epithelium, lymphoid tissues,
hair follicles, foetus, spermatozoa as well as abnormal cells

Dose limiting factors include BM failure and destruction of normal tissue


Administration is “pulsed” to allow for recovery
Combination chemo is preferred to limit resistance and reduce optimal dosages
Complications of cytotoxic drugs may be general or specific

General adverse effects include nausea and vomiting, myelosuppression and alopecia
Major specific complications include
 Cyclophosphamide – haem cystitis, sterility
 Cisplain – renal toxicity, peripheral neuropathy
 Actinomycin-D – tissue necrosis, stomatitis
 MTX – mucositis,diarrhoea
 Vincristine – constipation, neuropathy, foot drop
 Adriamycin – tissue necrosis, red urine, cardiomyopathy (>450mg/m 2)
 Cortico-steroids –peptic ulceration, iatogenic Cushing’s syndrome, ↑BP, infec ons etc

206
Compiled by Obasi. D. C. Chinedu

NEURODEVELOPMENTAL DISORDERS- CEREBRAL PALSY


DR ARONU ANN EBELE

AIM: To acquire basic and current knowledge on Cerebral Palsy (CP). Apply acquired knowledge in the identification and management of
CP.
OBJECTIVES: Define Cerebral Palsy (CP). Recognize a child with CP and identify the risk factors for the CP
Recognize comorbidities in a child with CP. Draw a treatment plan with reasons

OUTLINE:
Defition/introduction Risk factors/Causes
Clinical features/Classification of motor function Diagnosis
Investigations Management
Prevention

DEFINITION: A disorder of movement and posture resulting from a non-progressive injury to the developing brain. CP describes a
group of disorders of development of movement and posture, causing activity limitation that are attributed to non- progressive disturbances
that occurred in the developing fetal or infant brain.
The motor disorder of CP are often accompanied by disturbances of sensation, cognition, communication, perception, and/or behaviour
and/or by seizure disorder” (Bax et al,2005)

INTRODUCTION: Commonest motor disability in childhood. Globally occurs in 1.2-3.6 per 1000 live births. In Africa, accounts for 10-
90% cases in Neurology clinics in Low and Middle Income countries(LMICs). CP accounted for 16.2% of new referrals to child neurology
clinic in UNTH. Associated with poor health related Quality of Life. Increased risk of other morbidities such as Epilepsy, UTI, and
malnutrition.
Increased risk of mortality. Leading risk factor in High Income countries (HICs) is prematurity while in LMICs, severe perinatal
Asphyxia and bilirubin encephalopathy are dominant.

RISK FACTORS-General: Prematurity, Male sex, Black race.


Multiple pregnancies (Twins5-8x, Triplets 20-47x): death of a twin in utero carries higher risk. infertility treatments asso with increased risk.
Extremes of birth weight. Infants of mothers with low Socio-Econ Status.

RISK FACTORS-specific:
PRENATAL-maternal infections (UTI, chorioamnionitis), IUGR, Toxin exposure, congenital Torch infections.
Developmental abnormality: Brain malformations, genetic abnormalities, metabolic abnormality
PERINATAL brain injury: Perinatal Asphyxia/HIE, neonatal stroke, traumatic brain injury, intracranial haemorrhage. Brain injury related
to prematurity: Periventricular leukomalacia, Intraventicular haemorrhage
POSTNATAL brain injury: Kernicterus/bilirubin encephalopathy, CNS infections.

CLASSIFICATION: Physiologic, topographic, aetiologic, functional characteristics are used.


1. Functional:
- GMFCS (Gross Motor Function Classification system)
- MACS (Manual Ability Classification system)
- CFCS (Communication function Classification system)
- EDACS (Eating and drinking ability Classification system)

2. Edinburgh:
Spastic 60-70%: Spastic hemiplegia, Spastic Diplegia and Bilateral hemiplegia
Dyskinetic 10-20%
Ataxic 10-15%
Mixed 10% --- mixture of diff types, eg spastic & dyskinetic.

3. SWEDISH
-Spastic (hemiplegia, tetraplegia,diplegia)
-Dyskinetic
-Ataxic
-Unclassified/Mixed
Pathophysiology: Insults resulting in neuronal loss can be
Cortical (pyramidal), resulting in spasticity. Basal ganglial (extrapyramidal), resulting in abnormal movements such as choreoathetosis.
Cerebellar, resulting in hypotonia, or Mixed

207
Compiled by Obasi. D. C. Chinedu

CLINICAL FEATURES-General: Delayed achievement of developmental (motor) milestones. Abnormal/unusual motor movts—hand
preference, commando crawl.

Pathogenesis/Clinical features
Spastic CP – most common, up to 75%. Based on topographic distribution of motor involvement the following
Spastic Diplegia – more of lower limb involvement; most common; associated with prematurity
Spastic Hemiplegia – more of arm involvement Spastic Monoplegia – result of very mild hemiplegia
Spastic Quadriplegia – all four limbs Bilateral hemiplegia – quadriplegia with more of arm involvement
Triplegia – rare, results from milder and very asymmetric double hemiplegia (sparing one leg) or milder asymmetric diplegia (sparing one
arm)

Spastic Hemiplegia – results from stroke-like events with focal cortical infarction (haemorrhagic infarction, infection, thrombophilic
disorders):
--↓ spontaneous ac vity on involved side with hand preference, --difficulty in hand manipulation by 1yr,
-- walking is delayed to between 18–24 months, with circumductive gait
--Growth arrest esp in hand and thumb nail (involvement of contralateral parietal lobe)
--Spasticity of affected extremity, ↑tendon reflexes, ankle clonus, posi ve Babinski response.
--One third have seizure disorder, --approx 25% have cognitive impairment

Spastic Diplegia – usually results from periventricular leukomalacia (PVL; seen in prematurity, ischaemia)
--1st noticed when infant begins to crawl (uses arms more--commando crawl, dragging feet along)
--Scissoring posture when suspended at axillae
--Spasticity in legs with brisk reflexes, ankle clonus and bilateral Babinski reponse
--Good prognosis for normal intellect

Dyskinetic (Dystonic/Choreoathetoid) CP: follows birth asphyxia, kernicterus; pathology is in basal ganglia. Less common than spastic CP.
Features includes;
- Hypotonia initially with Head lag; - later tone increases with rigidity and dystonia;
- involuntary movts - Speech is typically affected (involvement of oropharyngeal muscles),
- also sensorineural hearing loss - UMN signs are absent,
- seizures are uncommon, - intellect is usually preserved

Spastic Quadriplegia: most severe form of CP. All four limbs affected;
- associated with severe PVL and cortical encephalomalacia - Decreased spontaneous movements
- Mental retardation and seizures common - Swallowing difficulty.
- increased tone and spasticity all limbs, brisk reflexes and positive Babinski response
- Speech and Visual problems - May have evidence of athetosis

Ataxic/Hypotonic CP: Occurs rarely. Results from cerebellar insult. most children progress to other CP subtypes.
Mixed CP: Occurs when the child displays a combination of features, such as spasticity and choreoathetosis.

Accompanying impairments in CP:


Cognitive/intellectual impairment (Mental retardation) – not in all CP patients. Spastic diplegia is characterized by normal cognition
while Spastic quadriplegic CP has greater MR. Factors associated with MR include: severe motor deficit, epilepsy, and cortical
abnormalities on neuroimaging.
Epilepsy – seen in 36% of pts with 70% manifesting in 1st yr of life (more in quadriplegic and hemiplegic than diplegic CP). Usually focal
seizures ± secondary generalization.
Poor growth – due to difficulty in feeding from pseudobulbar palsy/oromotor dysfunction.
Bladder and Bowel dysfunctions: Bladder: urinary incontinence with UTIs.
Bowel: constipation due to poor feeding, reduced water intake and immobility
Visual and Hearing impairment
Secondary musculo-skeletal abnormalities: Orthopaedic problems e.g. equinovarus deformity (tip-toe walking), hip dislocation (from
severe spastic diplegia), flexion contracture of knees and elbows
Speech abnormalities, Behavioural problems

Differential diagnosis: Inherited metab disorders (CHO, A.acid, Lysos), Intellectual disability, Metab myopathies, Metab neuropathy,
Traumatic peripheral nerve lesions, Vascular malformations (aneurysms angioma).
208
Compiled by Obasi. D. C. Chinedu

MANAGEMENT OF CP: History is important. Detailed physical/neurological exam


Investigation: EEG;FBC + Bld film; Urinalysis & urine m/c/s; CXR, CT Scan; MRI;
Hearing & Visual Assessments

DIAGNOSIS: difficult clinically in the first year: Limited variety of volitional movt for evaluation.
Substantial myelination takes months to evolve delaying the clinical picture of hypertonia/hyper reflexia

TREATMENT: Not Curable. Treatment is symptomatic and entails multidisciplinary approach:


Primary care physician Behavioural therapist Special education teachers
Physical therapist Psychiatrist or psychologist Speech therapist
Occupational therapist Neurologist Others.
Various forms of therapy help affected persons function and live more effectively. The earlier therapy begins the better.
Nutritional support: About 30% are undernourished. Many show growth delays. Early NG feeding & later Gastrostomy tube feeding may
lead to improved growth.
Give antiepileptic drugs for seizures eg carbamazepine. If there is generalized spasticity, give baclofen, diazepam or dantrolene and if
localized give injection botulinium toxin (Botox) for 3 – 6 mths
Physical therapy[Physiotherapy]: Aimed at encouraging the patient to build a strength base for improved gait & volitional movt. Stretching
program to limit contractures.
Occupational therapy: Helps maximize their function, and adapt to their limitation. To live as independently as possible.
Speech therapy: Usually starts before the child begins school. Helps to control the muscles of the mouth & jaw.
Helps to improve communication.
Orthotic intervention: using ankle-foot orthoses aimed at prevention and/or correction of deformities. Provision of support & facilitation of
skill development. Improvement of gait.
Surgery: usually the last resort: Lengthening of tendons. Rhizotomy[cutting of nerve roots].

PREVENTION: Most of the causes of CP [62%] are preventable and the 5 levels of prevention are involved.
 Mothers should be taught to ensure optimal wellbeing prior to conception
 Good obstetric care should be encouraged. Anti-D immunoglobulin for Rh-ve mothers
 Vaccination against some IU infections. Proactive treatment to minimize the effects of perinatal brain injury.
 High risk pregnant women should be identified early and referred for appropriate management
 Screening children at birth for G6PD deficiency
 Good NICU [Neonatal Intensive Care Unit] for those children who have developed birth asphyxia, severe jaundice and LBW.
Maintaining a good referral pediatric service. Physiotherapy. Rehabilitation of pts.

PROGNOSIS: Actual brain damage does not worsen but symptoms can become worse overtime. The collaborative perinatal project found
that 50% of individuals diagnosed with CP and 66% of children diagnosed with spastic diplegia “outgrew” findings by 7yr of age. The ability
to sit independently at 2yrs of age is predictive of future ambulation. Regarding life expectancy and mortality rates, the type and severity of
disability and feeding skills are major determinants. Overall, the probability of reaching the age of 20yrs in a child with severe CP is 50%.
Respiratory infections, aspirations and epilepsy are the leading causes of death. Newer therapeutic advances and the degree of parental care
have a strong influence on the length of survival of these children

QUESTION: A 20 month-old male presents with inability to sit, stand or walk compared to elder sibling who achieved all before age 12
months. He has repeated convulsions and is unable to speak. He was born at term following a prolonged labour and did cry at birth.
1.What clinical findings are possible in this child?
2.What is the treament plan for this child?

NEURODEVELOPMENTAL DISORDERS II- HYDROCEPHALUS


Definition: Dilatation of cerebral ventricles caused by an increase in CSF volume usually resulting from disturbance in formation, flow or
absorption of CSF. Results more from impaired absorption, and rarely from excessive secretion.

CSF formation and absorption: CSF forms at a rate of 20ml/hr i.e approx 500ml/day. Secreted predominantly by choroid plexus of the
lateral, 3rd and 4th vents. Flows in caudal direction via hydrostatic gradient [—pressure in vent=180mmH20 and in sagg sinus 90mmH20]
thru the vent syst. It then exits thru the foramina of Luschka and Magendie into the subarachnoid space. Absorption then occurs thru the
arachnoid granulations into the venous syst.

Classification: Non-communicating /Obstructive-: obstruction within the vent syst.


Communicating-: obstruction outside the vent syst i.e. vent CSF communicates with the sub arachnoid space.

Aetiology: Obstructive- Congenital or Acquired


Congenital causes: -- aqueductal stenosis or forking -- Vein of Galen aneurysm.
209
Compiled by Obasi. D. C. Chinedu

-- Dandy Walker synd [Atresia of foramina of Magendie and Luschka]


-- Arnold Chiari malformation [type 2-progressive hydroceph + myelomeningocele]
Acquired causes: -- acquired aqueductal stenosis [from adhesions sec to infections and haemorrhage]
-- supratentorial masses causing tentorial herniation -- intraventricular haematoma
-- Tumours [post fossa, vent, pineal region] -- Abscess/granuloma
-- Arachnoid cyst

Communicating Hydrocephalus: Results from thickening of the leptomeninges and/or involvement of the arachnoid granulations. This can
be as a result of;
Infections: -intrauterine infects, pyogenic [pneumococcal], TB, fungal
Subarachnoid haemorrhage[spontaneous,trauma] - Leukaemic infiltrations
Increased CSF viscosity e.g. high protein content. Excessive CSF production [choroid plexus papilloma] rare.

Pathological Effects: ↑CSF volume → vent dilat → CSF permeates into the perivent white matter → neuronal death, gliosis and scarring.
Vent dilat can also lead to ↑ICP which can lead to spontaneous arrest or Thinning of cerebral mantle and then lead to Death.

Clinical Features: Depend on: age of onset, nature of the lesion causing it and duration and rate of rise of ICP and include:
Symptoms:
Infant: Accelerated rate of enlargement of the head. - Irritability - Lethargy - Poor appetite
Vomiting - Delayed motor and cognitive milestones
Older child: Headache [prominent in older child] - Gradual change in personality
Deterioration in academic productivity

Signs: - Wide open, bulging and tense anterior fontanelle - Thin scalp with dilated veins - Lid retraction
- Impaired upward gaze [from pressure on midbrain tectum] - Broad fore head
- Cracked pot’ sound on skull percussion [macewen sign] due to sutural diastasis.
- Transillumination of skull may be positive [Dandy Walker, grossly enlarged vents] - Impaired consciousness
- Long tract signs [brisk tendon reflexes, spasticity, clonus, Babinski sign] - Cerebellar ataxia

Diagnosis: Hx of familial cases -X-linked hydroceph(Bicker Adam’s synd).


 Hx of prematurity [with intracranial haemorrhage, meningitis] and mumps meningoencephalitis-aqueductal stenosis.
 Macrocephaly. Café au lait spots [neurofibromatosis]--aqueductal stenosis.
 Midline lesions at the back [myelomeningocele, hair tuft,lipoma,or angioma]-spinal dysraphism
 Cranial bruit-Galen A-V malformation
 Transillumination of the skull [from massive dilatation of vents]-Dandy Walker malformation.
 Chorioretinitis on fundoscopy--Toxoplasmosis [intra-uterine infection],papilloedema in the older child-raised ICP.
 Plain X-ray of the skull: signs of raised ICP (separation of sutures, erosion of posterior clinoid process, increased convolutional
markings [silver or copper beaten appearance]
 CT, MRI and U/S --identify specific cause

Differential Diagnosis:
Enlargement secondary to thickened cranium: -chronic anaemic conds [SCA, THAL]
-Rickets -osteogenesis imperfect -epiphyseal dysplasia
-Chronic subdural collections-bilat parietal bone prominence
Megalencephaly: abnormal storage of subs within the brain eg lysosomal dis [mucopolysacch, Tay Sachs], aminoacidurias [maple syrup
urine dis], leucodystrophies [metachromatic, canavan dis]
--cerebral gigantism--increase brain mass --Neurofibromatosis
Hydranencephaly [cerebral hemispheres represented by membranous sacs. Midbrain and brain stem intact]
Achondroplasia.

Treatment: 1. Medical: Not alternative to surgery. 2. Surgical:

Medical: 4 Modalities
1.Removal of CSF (serial LP) 2. Reduce CSF production (Acetazolamide, Frusemide)
3.Decrease brain water content (Osmotic diuretics eg mannitol, hypertonic saline, urea, glycerol)
4.Increase CSF absorption (hyaluronidase, streptokinase, urokinase, tissue plasminogen)

Surgical: Ventriculo-peritonial shunt - Ventriculo-atrial shunt - Lumbo-peritoneal shunt


- External ventricular drainage - Others.

Complications of shunting: Infection- meningitis, peritonitis, shunt nephritis. Common orgs include staph epidermides, staph aureus
Malfunction (obstn, disconnection, migration). Shunt obstruction [from choroid plexus, debris, bld clot, omentum, high CSF protein].

210
Compiled by Obasi. D. C. Chinedu

Subdural haematoma [from vent collapse] Low pressure state: manifests as headache and vomiting on sitting or standing
Others include: Shunt overdrainage, Seizures, Pneumocephalus.
Cardiac complications asso with vasc shunts(endocarditis,arrythmias), Pulm complications(pulm thromboembolism)
Peritonial shunt compl: Ascites, inguinal hernia, hydrocele, bowel perforation, peritonitis).

Prognosis: Depends on the cause and associated co-morbidities. Good with most children attaining normal IQ if treatment precedes
irreversible brain damage. Repeated complications carry significant morbidity.

CRANIOSYNOSTOSIS: Premature fusion of single or multiple cranial sutures leading to deformity of skull and face. 2 types:
Primary: due to abnormalities of skull development.
Secondary: results from failure of brain growth and expansion.

Primary: Incidence 1:2000 births.


Aetiology: Unknown in majority of cases. 10-20% associated with genetic syndromes like Crouzon synd, Apert synd, Carpenter synd.
Occipital and frontal plagiocephaly may be caused by deformational forces.

Clinical manifestations: Evident at birth; asymmetric craniofacial appearance. Skull deformity. Prominent bony ridge over suture and
premature closure of fontanelles. X-ray and bone scan confirm fusion of sutures.

Depending on involved suture, the following result:


Scaphocephaly: [saggital suture] Plagiocephaly: [coronal and/or lambdoid suture]
Oxycephaly or Turricephaly: all sutures cone-shaped head or pointed head with severe MR and microcephaly.
Brachycephaly: (coronal synostosis)

Premature closure of only one suture causes no neurological problem.


Complications such as hydrocephalus and raised ICP are more likely to occur if two or more sutures are involved.
Treatment: Surgery (+/- Helmet)
Indications for Surgery: Cosmesis. - Neurological complications [raised ICP, Hydrocephalus]

Deformational plagiocephaly: Cranial flattening and asymmetry as a result of extrinsic molding forces. Incidence increasing with infants
sleeping on their backs to prevent SIDs. Head is normal at birth. Noticed at 6wks but greatest at 4mths, the deceases with age. Resolves
completely by 2-3yrs of age.

Causes: Prenatal: Uterine compression/intrauterine constraint e.g Oligohydramnios, multifetus gestation.


Postnatal: Infant sleeping position, Congenital muscular torticollis.

Treatment: Repositioning and physiotherapy (RPPT). Molding therapy (Orthotic Helmet).


Helmet treatment should start b/w 4 and 8 mths and contd for 7-8mths.

MICROCEPHALY: Head circumference that is more than 3Std below the mean for age and gender OR HC that is below the 2 nd centile for
age and gender. Common among dev delayed children. It is due to failure of normal brain growth.
Types: Primary (Genetic) and Secondary (Non-genetic)

Primary: Freq genetically determined. Associated dev delay. Associated with specific genetic syndromes. May be familial: Dev is normal.
Common types are Familial, Autosomal dorminant, & those asso with Chromosomal synds.

Causes: Primary (genetic) -autosomal recessive, -autosomal dominant


Syndromes: Down synd, Edward synd, Cri-du-chat, Cornelia de Lange etc.

Features: 1. Small head present at birth; Xtic shape: narrow forehead, slanting frontoparietal areas, pointed vertex, flat occiput, ears often
large and abnormally formed
2.Gen hypertonia common. 3.Convulsions freq dev. 4.Profound learning disorder(MR)
Primary micro also found in some recognizable synds like Trisomy 21,18,13 and non-chromosomal synds like Cornelia de Lange synd.

Secondary: Results from severe brain damage during preg or the first 2yrs of postnatal life. Developing brain is vulnerable to:
-cong infections(TORCHES) -Drugs (incl alcohol) -Radiation -Hypoxia (HIE)
-Metab disorders (maternal DM, maternal hyperphenylalaninaemia) -Neonatal meningitis -AIDS etc.

Evaluation: History: Exposure to above factors Family and birth hx Associated dysmorphic conds
Head small or normal at birth

Treatment: Genetic and Family counselling. Placement in appropriate program for maximal development.

211
Compiled by Obasi. D. C. Chinedu

CHILDHOOD DIABETES
Dr. Ugo Chikani

Outline:
Introduction Classification Epidemiology Pathogenesis. Pathophysiology
Clinical presentation/natural history Diagnosis Treatment Complications.

Introduction: Diabetes mellitus is a common chronic metabolic syndrome characterized by hyperglycaemia as a cardinal biochemical
feature. It is not a single entity but a heterogeneous group of disorders in which there are distinct genetic patterns as well as other aetiologic
and pathophysiologic mechanism that lead to impairment in glucose tolerance.

Classification: The American Diabetes Association (ADA) and the WHO have classified it into:
Type 1 DM: Those caused by deficiency of insulin secretion due to pancreatic beta cell damage. >90% in children;
- T1A: immune mediated - T1B: non-immune mediated
Type 2 DM: Those that are a consequence of insulin resistance occurring at the level of the skeletal muscle, liver and adipose tissue with
various degree of beta cell impairment. 10-50% depending on ethnicity and race and increasing prevalence of obesity.
Atypical course or presentation gives suspicion of other less common types.
Diagnostic criteria is the same regardless of aetiology: Obesity, minority pubertal children and adolescent, parent with type2 DM and
evidence of insulin resistance (acantosis nigricans, hypertension and hyprelipidaemia) most likely to have type 2DM.
Gestational diabetes.
Other specific types which include:
- Those due to genetic defect in beta – cell function:- maturity onset diabetes in the young (MODY)
- Genetic defect of insulin action:- Leprechaunism, Rabson – Mendenhall syndrome, lipoatrophic diabetes.
- Disease of the exocrine pancreas:-pancreatitis, cystic fibrosis. Trauma, pancreatectomy, neoplasia, hemochromatosis.
- Endocrinopathies:- Cushing disease, glucagonoma, pheochromocytoma, aldosteronoma, somatostainoma, hyperthyroidism, acromegaly,
- Drug or chemical induced DM:- glucocorticoid, thyroid hormone, vacor, pentamidine, nicotinic acid, diazoxid, beta –adrenergic agonist.
- Infection induced:- congenital rubella, cytomegalovirus infection,
- Uncommon forms of immune mediated diabetes:- ‘’stiff – man’’ syndrome
- Genetic syndromes:- Down syndrome, Klinefelter syndrome, wolfram syndrome, Turner syndrome, Fredereich ataxia, Prader- willi
syndrome, Huntington chorea. Laurence – Moon Biedl syndrome.

Neonatal Diabetes: Can be Transient – recurrent. Transient – without recurrence. Or Permanent.

Epidemiology: TIDM is the commonest endocrine metabolic disorder of childhood and adolescence. Boys and girls are equally affected.
There is no apparent correlation with socioeconomic status. Peaks of presentation occur in two age groups 5-7yrs and 10-19yrs of age. A
large number of cases are presenting between 1 & 2 yr of age. The first peak may correspond to the time of increased exposure to infectious
agent coincident with the beginning of school. The second peak may correspond to the pubertal growth spurt induced by gonadal steroids &
increased pubertal growth hormone secretion. These possible causes and effect relationships remain to be proved.

In 2011, globally total number of children aged 0 – 14years was 1.9billion. Total number with type 1 diabetes was 490.1million. Number of
newly diagnosed per year is 77.8thousand. Annual increase in incidence 3.0%.
A wide variation exist in worldwide incidence of T1DM with the highest occurring in Finland (45/100,000) and lowest in China, Japan and
south America. In Port Harcourt southern Nigeria a hospital based prevalence of 1.6/1000 was reported in children <16years. In Sudan, a rate
of 0.95/1000 was reported among children 7 -14years in a population based study. In Nigeria, a community based study of 0.3% per 1000. In
the USA there is a prevalence of 1.9/1000 among school aged children. With a range of 1 case/1,430 children @5yrs and 1/360 @16yrs.

Pathogenesis: T1ADM, is a chronic, T – cell mediated autoimmune disease. Results from autoimmune destruction of the pancreatic islets
cells. Genetic predisposition and environmental factors, lead to initiation of an autoimmune process against the pancreatic islets. This results
in progressive destruction of beta – cells with loss if insulin secretion and eventual development of T1DM. TIBDM results from a non
immune mediated destruction of islet cells.
Summary of the pathogenesis can be viewed in series of stages:-
Stage 1: genetic susceptibility. Stage 2: triggers (environmental factors) Stage 3: Autoimmunity.
Stage 4: Loss of insulin secretion. Stage 5: Overt diabetes.

Genetic susceptibility: For Caucasians with T1ADM, the most common DM associated haplotypes are DR3 and DR4. >90% of patients
with T1ADM have one or both alleles. Highest risk DR4 haplotypes have:-DRB1*0401, DRB1*0402, DRB1*0405. They have
DQB1*0302. Low risk:-DRB1*0301 and DQB1*0303. DR3, always conserved with DRB1*03 combined with DQA1*0501 and
DQB1*0201.
T1AD is itself heterogeneous, with several forms of immune mediated diabetes with known genetic causes as part of autoimmune
syndromes. Over 25 gene loci has been associated with it. The most important genes are located within the MHC HLA class 11 region of
chromosome 6p21 accounting for ~50% of cases. MHC(HLA genes) play a major role in determining disease susceptibility. The most
important determinants of T1ADM are the HLA DQ AND DR alleles. Tremendous spectrum of diabetic risk is associated with different DR3
and DR4 genotypes.
212
Compiled by Obasi. D. C. Chinedu

Highest risk genotype has both DR4/DR3, DQB1*0302/DQB1*O201. Protective HLA molecules include:-
Moderate :-DRB1*0403.
Dominant :- DQB1*0602-Most common
DQA1*0201 with DQB1*0303 and DRB1*1401
All provide dramatic protection.

TRIGGERS
- Puberty - Chemicals - Season
Putative Envt Factors: Congenital Rubella, Viruses (Enteroviruses, Rotaviruses), Dietary (Bovine milk, Gliadin, Cereal), Hygiene
Hypothesis (Infections protect) Pinworms.

Autoimmunity:
Immunologic markers: Autoantibodies — the islet-cell antibodies (ICA), Insulin autoantibodies (IAA), Antibodies to glutamic
acid decarboxylase (GAD), Antibodies to tyrosine phosphatase-like proteins such as insulinoma associated protein (IA-2, ICA512).
Zinc transporter 8(ZT8)

Insulin autoantibodies are usually the 1st to appear in children followed from birth for the development of T1ADM. Once seen at such a
young age, there is high risk of developing additional anti- islet AA. >90% of children developing T1ADM <5yrs, have IAA while <50% of
those developing it after 12yrs, have IAA. There is very high risk >75% of progression to Dm in those that have 2 or more AAs.

Loss of insulin secretion: At present, beta cell mass is not readily measured in man. It is not possible to absolutely define progression of
beta cell loss. The best evidence for progressive loss of beta cells function comes from studies of insulin and c- peptide secretion. Thus c –
peptide is an important indication of remaining beta cell function.
Following the onset of diabetes, c – peptide secretion progressively declines until for most patient with T1ADM, it becomes undetectable
associated with true insulin dependence.
Similarly, 1st phase insulin secretion following bolus of glucose on intravenous glucose tolerance testing is progressively lost for relatives
followed to development of T1DM.

Overt diabetes: Apparently preceded by the anti – islet autoantibodies for years. Deterioration in glucose tolerance is noted more than one
year prior to onset of diabetes. Majority of individuals identified to be diabetic following autoantibody testing are found to have diabetic 2hrs
plasma glucose or OGTT (>200mg) rather than fasting hyperglycaemia. May present with transient hyperglycaemia.

Pathogenesis: What does this mean? An individual inherits a combination of genes that determines their risk
This genetic predisposition is then triggered or dampened by environmental factors. Autoimmune destruction of the pancreatic islet cells.
Loss of insulin secretion with eventual development of T1ADM.

Pathophysiology: T1DM is a progressive low insulin catabolic state in which feeding does not reverse but exaggerates. In moderate
insulinopinia, glucose utilization by muscles and fat cells decreases with post prandial hyperglycaemia. At a lower level, gluconeogenesis
and glycogenolysis ensures and fasting hyperglycaemia begins.

Hyperglycaemia produces: - Glycosuria when renal threshold exceed 180mg/dl;10mMOL/l of glucose.


- Osmotic diuresis. - Loss of calories and electrolytes and persistent dehydration.
- Physiologic stress with hypersecretion of stress hormones: epinephrine, cortisol, growth hormone and glucagon
- They further worsen the hyperglycaemia and Hyperlipidemia.
- The hormonal interplay of insulin and excess glucagon, shunts the FFA into ketone body formation.
- Accumulation of keto acids results in metabolic acidosis (diabetic ketoacidosis DKA)
- With progressive dehydration, acidosis, hyperosmolality and diminished cerebral oxygen utilization, consciousness becomes impaired and
patient ultimately becomes comatose.
Modes of Presentation of T1D: - Polyuria, nocturia, bed wetting - Excessive thirst - Weight loss -
Fatigue - Polyphagia or depressed appetite - Persistent mycotic infection
Ketosis ± Acidosis: Abdominal pain, Nausea, vomiting. Tachypnea and deep respiration. “Ketone smell”. Lethargy, obtundation, coma.
Asymptomatic: Fortuitous discovery of glycosuria (beware of renal diabetes)

213
Compiled by Obasi. D. C. Chinedu

Natural history of T1DM: There are four distinct stages:-


1. Preclinical beta cell autoimmunity with progressive defect in insulin secretion.
2. Onset of clinical diabetes
3. Transient remission ‘’honeymoon period’’
4. Established diabetes associated with acute and chronic complications and decreased life expectancy.

Diagnosis of Diabetes: ADA Expert Committee


NORMAL IMPAIRED DIABETES

FASTING < 100 mg% 100-125 ≥126 mg%


(5.6 mM) (7 mM)

ORAL GTT <140 mg% 140-199 ≥200 mg%


(2 hours) (7.8 mM) (11.1 mM)
75g or 1.75g/kg
After determining hyperglycaemia, evaluate for DKA(ketonuria and electrolyte abnormality). Get a baseline HBA1c; could be valuable in
estimating the duration of hyperglycaemia and used to compare subsequent values to determine treatment effectiveness. Transglutaminase
IgA and total IgA, antithyroid peroxidase, antithyroid antibodies , TSH and FT4.

Treatment: Management of T1DM primarily focuses on:


- Adequate insulin replacement - Matched with food intake - Exercise
Insulin: regimen varies with age, acceptation by patient, stages of diabetes, doctor’s preferences and economical considerations.
Diet: ± 50 % of CHO, ± 30 % of lipids, ± 20 % of proteins of RDC. Day time fractioning varies according to insulin regimen.
Exercise should be regular. Psychological support

Treatment goals: ISPAD guidelines: HbA1c < 7.5 % - Quality of life - Low risk for hypoglycemia - Weight

Insulin Therapy: Phamacokinetics


INSULIN BRAND NAME ONSET OF PECK ACTION DURATION
PREPARATION ACTION OF
ACTION

LISPRO HOMALOG 5-15 MINUTES 1-2 HOURS 4-6HOURS

ASPART NOVOLOG 5-15MINUTES 1-2HOURS 4-6HOURS

GLULISINE APIDRA 10-15MINUTES 1-2HOURS 4-6HOURS

HUMAN REGULAR HUMILIN - 30-60 MINUTES 2-4HOURS 6-10HOURS


R/NOVOLOLIN-
R

214
Compiled by Obasi. D. C. Chinedu

HUMAN NPH/LENTE HUMILIN- 1-2HOURS 4-8HOURS 10-20HOURS


N/NOVOLIN-N

ULTRALENTE HUMILIN –U 2-4 HOURS UNPREDICTABLE 16-20HOURS

GLARGINE LANTUS 1-2HOURS FLAT ~24HOURS

DETEMIR LEVEMIR 0.8 – 2HOURS FLAT ~24HOURS

TREATMENT
INSULIN REGIMEN DEFINITION DOSE CALCULATION

BASAL INSULIN Insulin requirement to suppress 40-50% of total daily dose


hepatic glucose production
between meals

BOLUS Insulin requirement to maintain Consist of the sum of two components:-Insulin:


INSULIN(PRANDIAL) normal glucose disposal after carbohydrate ratio=500/Total starting dose
eating. 2.Correction factor=1500/TSD –Correction
factor for young children(1800/TSD)

Most newly diagnosed Are started at 0.25 -0.5/kg/ day.


T1DM adolescents
may need more.

Recent advancements: Whole pancreas transplantation. Islet cell transplantation. Engineered cells stem cells manipulated genetically to
produce insulin.
Techniques to protect b cells from autoimmune attack – Immunotherapy
Alternate non invasive routes for insulin administration. Chemical alteration of insulin molecule. Artificial pancreas
Adding c peptide to insulin. Implantable insulin pumps
New blood glucose meters
Table 3. The American Diabetes Association Guidelines for the Target Gluose and HbA1C Levels by Age
Age Group (years) Target Blood Glucose HbA1C
(%)
Before Meals Bedtime-Overnight
Toddlers and Preschoolers: 0-6 100-180 110-200 7.5-8.5
School Age: 6-12 90-180 100-180 <8
Adolescents and Young Adults: 13-19 90-130 90-150 <7.5*

Complications: Acute: Hypoglycaemia, Hyperglycaemia, DKA.


Long term: Microvascular complications: Retinopathy, Nephropathy, Neuropathy.
- Macro vascular disease - Increased risk of infection - Injection sits hypertrophy

HYPOGLYCAEMIA: Most frequent complication of T1D. Severe prolonged hypoglycemia may cause permanent CNS damage,
particularly in young children (< 6 years). May be symptomatic or asymptomatic: “Hypoglycemic unawareness”. Important limiting factor to
achieve near-normal hypoglycemia

Symptoms of Hypoglycaemia
Autonomic: BG ≤ 3.5-4mmole: Hunger. Anxiety. Irritability. Pallor. Tremulousness. Heart palpitation. Throbbing pulse. Numbness in the
lips, finger & tongue. Cold sweating.
Neuroglycopenia (40 – 60 mg%): Headache/Mental confusion. Visual disturbances. Behavioural changes/Impaired thinking. Slurred
speech. Incoordination/Tiredness. Feeling warm or hot. Difficulties with hearing. Seizures/Coma.

DKA: Creates a life-threatening medical emergency. Is the most important cause of mortality and severe morbidity in children with
diabetes, particularly at the time of first diagnosis. Early recognition and careful management are essential if death and disability are to be
avoided.

215
Compiled by Obasi. D. C. Chinedu

Diagnosis: 3 cardinal features: - Hyperglycaemia - >300mg/dl - Ketonuria >5mmol/l


- Metabolic acidosis: bicarbonate < 15mmol/l

Clinical features: Severe dehydration, shock. Frequent vomiting. Polyuria despite dehydration. Weight loss in spite of good intake. Acetone
breath – (Kussmaul respiration) deep and rapid. Altered sensorium
Signs of raised intracranial pressure – bradycardia, HT, anisocoria

Management of DKA: Treatment: Expansion of intravascular volume, Correction of deficit of fluid and electrolytes
Insulin therapy. Monitoring and Follow up

Prediction of T1DM: Individuals at risk for T1DM can be identified by a combination of genetic, immunologic and metabolic markers. The
most informative locus is the HLA class 11 which confers about half of the total genetic risk. H/e it has a low positive predictive value (PPV)
when used in the general population. The immunologic markers have become the mainstay of T1DM prediction effort. The autoantibodies
provide a practical readout of B-cell autoantibodies.

RISK: 2 – 3 fold increase risk when HLA DR4 OR DR3 is inherited. 7 -10 fold increased risk if both are inherited.
Siblings: 5 – 6 %. Mother: 3 – 4 %. Father: 6 %. Monozygotic Twins: 30 – 50 %. General Population: 0-4 %.

Prevention of T1DM: No proven therapy to prevent progression to T1DM or to halt beta cell loss after presentation with DM has been
found. Obstacle to finding a plausible preventive strategies include: treatment dilemma, Ethical issues surrounding prediction, Selection of
population at risk and treatment strategies. Finding new preventive agents.

Prevention: Several agents have been tried with limited success. These include:-
low doses of subcutaneous insulin, Oral insulin,
Modifying CD3 monoclonal antibody to reduce cytokine release. This only transiently delayed loss of C- peptide.

216
Compiled by Obasi. D. C. Chinedu

MACRONUTRIENTS AND MICRONUTRIENT DEFICIENCIES


Macronutrients: are nutrients that are needed in large quantities in the body for energy generation and tissue repairs.

(a) Carbohydrates: Make up the bulk of calories supplied in the diet. Major natural sources of carbohydrates include tubers, root crops
(yams, cassava, potatoes and cereals (maize, rice, sorghum, millet and wheat). They supply the complex carbohydrates that provide more
fibres, minerals, and vitamins than the simple carbohydrates. Examples of simple carbohydrates: refined sugar and confectionaries.
It is recommended to consume the simple carbohydrates together with other sources of minerals and vitamins in order to make our meals
balanced – balanced diet.
Types:
Monosaccharides: fructose, glucose, galactose Disaccharides: lactose, sucrose Polysaccharides: starchy foods – hydrolysis

Digestion: By salivary amylase, pancreatic amylase to the intestine – maltose, maltotriose, and α–dextrins.
The deficiency of the oligosaccharidases allows the intra-luminal accumulation of lactose and sucrose resulting in diarrhoea, flatulence and
bloating (e.g. lactose intolerance which can be 10 or 20).

(b). Proteins: Make up the bulk of tissues which are the major structural components of cells and are known as body – building foods.
Needed for growth and development, cellular repairs, tissue building and production of metabolic and digestive enzymes. During periods of
sub-optimal dietary protein consumption, the body uses the protein previously stored in muscles to meet the metabolic demands of the body,
resulting in Muscle Wasting. Proteins are made up of amino acids. Animal sources of protein contain high quality, easily digestible proteins
with most of the essential amino acids. Plant proteins are less easily digested and contain fewer essential amino acids.

(c). Lipids: Lipids in the form of fats and oils, constitute a concentrated source of high energy.
Important for the supply of essential fatty acids which must be present in foods.
Pattern of lipid consumption varies with geographical variations in dietary intakes. e.g. lipids constitute <10% of total energy intake per day
(developing countries) compared to 40% in high income countries.
Natural sources: butter, margarine, vegetables, oils and bacon fat. Other sources: Milk, nuts, avocados, cheese, egg yolks.
These essential fully acids are: Linolenic and linoleic acids.
Linoleic acids and linolenic acids are not synthesized in human body and must be supplied in food for growth, skin integrity, regulation of
cholesterol metabolism and immuno-modulation. The recommended ratio of linoleic acid to linolenic acid in diet is 4-6:1.
Lipids are less readily digested or absorbed and generates x2 as much energy as other macronutrients (e.g. lipids after digestion – generates
9.1kcal/g compared to 4Kcal/g of lactose), making tendency to accumulate excess weight gain/obesity.

d). Fibres: Materials present in food substances (cellulose, hemi- cellulose and lignin) of plant origin and they reach the colon intact.
Recommended Dietary Allowance for fibire= 0.5g/kg-30g/kg/day.
Desired effects: ↑ed fibre intake: helps to achieve early satiety and limit the amount of calories ingested and restricts simple sugar intake.
Prevents constipation. Limited information about inflammatory bowel disease, IBD conditions.

MICRONUTRIENTS AND MINERALS


The term micronutrients include two main classes of nutrient substances required in the diet in very small amounts; the essential organic
micronutrients (vitamins) and the essential inorganic micronutrients (trace elements).
Vitamin and mineral deficiencies may complicate malnutrition arising from underlying disease or inadequate diet.
In children with restricted diets (e.g. vegetarians) a detailed dietary assessment by an experienced dietician may help identify likely vitamin
or mineral deficiencies. Children with cow milk allergy maintained on diary free diets, for example, are likely to require calcium
supplements unless they are drinking adequate volumes of calcium- containing milk substitute. Fat soluble vitamin supplementation may be
necessary in any chronic condition where there is impairment of fat digestion or absorption.
Plasma concentration of vitamins and trace elements do not always accurately reflect tissue stores and should be interpreted with caution,
particularly when paediatric reference ranges are not well defined (e.g. selenium). Plasma zinc falls during an acute phase response, whereas
plasma copper increases.

Vitamin deficiency
1. Vitamin A: Xerophthalmia, cornea becomes dry and hazy and may progress to necrosis and scarring (keratomalacia), occasionally to
perforation. Major cause of preventable childhood blindness worldwide. Subclinical deficiency associated with increased mortality:
Sources: Vegetables (carrots), fish oils, liver, fortified milk.
Assessment: Plasma retinol, Retinol binding protein.

2. Vitamin B1 (Thiamine): The deficiency syndrome (beriberi), is associate with polished rice diet. Acute cardiomyopathy @ few months
of age. Hoarseness, aphonia, encephalopathy, apathy, drowsiness, convulsions.
Sources: Germ of cereals, pulses, yeast, nuts, milk, liver, pork, seeds. Assessment: Red cell transketolase, blood thiamine level.

3. Vitamin B2 (Riboflavin): Usually associated with other nutritional defects, rather than occurring by itself.
Angular stomatitis, fissuring of lips, naso-labial seborrhoea, magneta coloured tongue.
Sources: liver, milk, eggs, vegetables, cheese. Assessment: Red cell glutathione reductase, urinary riboflavin (< 30mcg/day).
217
Compiled by Obasi. D. C. Chinedu

4. Folic acid: Nutritional deficiency mainly in developing countries may occur in prematurity, malignant diseases and its treatment,
chronic haemolytic anaemia, malabsorption, drugs (e.g. methotrexate, anticonvulsants) & vitamin B12 deficiency. Megaloblastic anaemia
on blood film, macrocytosis, neutropenia with hyper-segmentation of polymorph nuclei, thrombocytopaenia.
Supplementation in pregnancy reduces risk of neural tube defects.
Source: Green vegetables, liver, fortified cereals, dried beans, orange juice, kidney.
Assessment: Low serum folate, low RBC folate levels, elevated homocysteine levels with normal methylmalonic acid levels.

5. Vitamin B6 (Pyridoxine): Deficiency is rare: reported in association with use of infant formula deficient in pyridoxine. Convulsions:
abnormal EEG. Consider in any newborn with persistent scizures.
Features in children include weakness, depression, stomatitis, diarrhoea, and dermatitis.
Sources: animal products (meat, poultry, liver, kidney), milk, some nuts, seeds, whole grains, beans.
Assessment: Whole blood concentrating, plasma pyridoxal phosphate level.

6. Nicotinic acid (Niacin): Deficiency causes: Pellegra (children eating maize diet, toddles with kwashiorkor).
Child usually of school age: Symmetrical desquamating pigmented dermatitis affecting exposed areas of skin.
Dementia and diarrhea.
Sources: meat, fish, yeast, fortified enriched foods.
Assessment: low urinary excretion levels of niacin and metabolities (in 24hr urine).

7. Vitamin B12: Deficiency may occur in infants of strict vegetarians, vitamin B12 deficiency mothers, feeding with unfortified artificial
milks. Pernicious anaemia (intrinsic factor deficiency associated with autoantibodies against gastric parietal cells causing vitamin B12
malabsorption) may occur in older children. Complication of ileal resection may take years to become apparent. Pallor, fatigue, glossitis.
Sub-acute combined degeneration of spinal cord (diminished tendon reflexes, loss of vibration sense, ataxia, extensor plantar response).
Sources: animal products (liver, meats, fish, shellfish, poultry, eggs, cheese), milk.
Assessment: low serum Vitamin B12 (if <400mcg/ml, consider checking methylmalonic acid and homocysteine levels.

8. Biotin: Scaly dermatitis and hair loss.


Sources: liver, lentils, peanuts, mushrooms, chicken, while wheat, eggs, intestinal bacteria.
Assessment: Decreased urinary Biotin level (in 24hr urine).

9. Vitamin C: Scurvy: rarely before 6 months of life. Associated with extremely limited dietary intake (e.g. in a child with severe
neurological handicap) or tube feeding with special formula. Petechial haemorrhage into the skin, impaired growth, irritability, painful joints
with pseudo paralysis.
Radiologically may be mistaken for rickets: Long bones show thinning of cortex, ‘eggshell’ calcification around epiphysis; periosteal
elevation; occasionally epiphyseal separation.
Sources: Fresh fruits and vegetables (particularly citrus fruits). Assessment: Leucocyte vitamin C, Plasma vitamin C level.

10. Vitamin D: Rickets: Impaired bone formation and growth. Decreased calcium absorption, low plasma calcium, raised ALP level. Raised
PTH metabolizes calcium from bone, but also leads to phosphaturia and hypophoshataemia ultimately PTH effect on bone is impaired and
plasma calcium falls. Hypotonia and impaired linear growth in infancy, delayed closure of anterior fontanelle, prominent forehead.
Rarely, symptomatic hypocalcaemia (e.g. stridor, seizures). Swelling of costochordral junctions (‘Rachitic rosary’).
Bowing of tibia in weight bearing children, swelling over growing end of long bones (e.g. wrists). Coxa vara, kyphoscoliosis, pelvic
deformity in long standing cases.
Radiologically poor mineralization, delayed development of epiphyses, cupping, fraying and splaying of metabolites; radiolucent transverse
bands (Looser’s zones).
Sources: Fish oil, vegetable oil, fortified diary products; skin synthesis.
Assessment: Serum calcium, phosphate, ALP, Serum 25 – OH – Vitamin D level.

11. Vitamin E: Deficiency may be seen in preterm infants and children with malabsorption. Haemolytic anaemia in the preterm. Progressive
peripheral neuropathy, ataxia, and retinopathy in severe, prolonged deficiency.
Sources: vegetable oils, dried beans, sunflower seeds, wheat germ, dark leafy green.
Assessment: Serum (alpha) tocopherol.

12. Vitamin K: Haemorrhagic disease of newborn (breast fed infants @ increased risk). Gastrointestinal haemorrhage or bleeding from cord,
intracranial haemorrhage. Prevented by routine prophylactic vitamin K administration after birth (usually multiple oral doses).
Sources: Green vegetables, gut flora. Assessment: Prothrombin time (PT), serum phylloquinone, INR, PIVKA -1.

Mineral Deficiency
Calcium: Absorbed from proxiomal small bowel. Nutritional deficiency in isolation very unusual. Absorption affected by other nutrients e.g.
fat mal-absorption.
Sources: Diary products, calcium – set tofu, calcium fortified beverages.
Assessment: Bone density; serum or ionized calcium.
218
Compiled by Obasi. D. C. Chinedu

Phosphorus: Isolated deficiency very unlikely. Prolonged treatment with aluminium or magnesium hydroxide bind to phosphorus resulting
in non-absorption. Deficiency leads to osteoporosis or rickets.
Sources: Diary products, soda containing phosphoric acid. Assessment: Serum phosphorus.

Magnesium: Deficiency may be encountered in protein – energy malnutrition (PEM), after small – bowel resection or with protracted
diarrhoea. Rare selective inability to absorb magnesium managed by magnesium supplementation.
Sources: Leafy green vegetables, whole grains, nuts. Assessment: Serum magnesium, 24hr urine magnesium (most accurate).

Trace element deficiency


Iron: Deficiency from poor intake (common), impaired absorption or excessive losses.
Symptoms includes: Anaemia, pallor, tiredness, loss of appetite increased infection, impaired development, pica.
Sources: meats, fish, poultry. Assessment: Serum ferritin, TIBC, Serum iron, Hb/HCT/PCV, Zinc protoporphyrin, serum iron.

Zinc: Growth retardation, hypogonadism, hepatosplenomegaly, and anaemia. Delayed wound healing, pica, diminished taste. Symmetrical
peri-orificial erythematious rash. May occur in preterm newborn receiving parenteral nutrition (particularly after bowel resection).
Source: Red meat, sea food, whole grains, fortified cereals.
Assessment: Plasma zinc, leukocyte zinc, ALP.
Copper: May occur in infants with PEM or mal-absorption. (or during TPN with inadvertent omission of take elements). Hypochromic
anaemia unresponsive to iron, neutropenia. Skeletal changes can resemble scurvy or non – accidental injury.
Sources: Organ meats, sea foods, nuts, seeds, cocoa products, whole grains.
Assessment: Plasma copper, caeruloplasmin

Fluoride: Important for reducing risk of dental caries.


Assessment: urine excretion of fluoride.

Iodine: Hypothyrodism with poor growth and development.


Assessment: Serum T4, T3, TSH

Chromium: Deficiency may impair glucose tolerance, weight loss, neuropathy.


Assessment: Plasma chromium, Glucose tolerance.

Cobalt: Essential component of vitamin B12.

Manganese: Role in human metabolism uncertain. Deficiency in animals results in growth impairment, neonatal ataxia, chondrodystrophy,
impaired fertility.
Assessment: Plasma manganese.

Molybdenum: Essential component of xanthine oxidase.


Assessment: urine xanthine, plasma molybdenum.

Selenium: Deficiency: implicated in Keshan disease (cardiomyopathy) and Kashin – Beck disease (osteoarthropathy), but other
environmental factors likely to be important. Skeletal myopathy and pseudo-albinism described during parenteral nutrition without selenium.
Food sources: meat, seafood, cereals, grains, diary.
Assessment: plasma selenium, red cell glutathione peroxidase.

VITAMIN SUPPLEMENTATION FOR INFANTS AND YOUNG CHILDREN


Precise daily requirements for vitamins are not well established but supplementation is regarded as important in early life (children < 4yr).
The foetus acquires vitamins from its mother with fat soluble supplies being transferred towards the end of pregnancy (3 rd trimester).
Breast milk from mother with adequate nutritional status supplies sufficient amount of all vitamins other than vitamins K and D
respectively. The UK Department of Health recommends either a single intramuscular (i.m.) dose of 1mg vitamin K @ birth to newborn
babies or an alternative oral regimen of three 2mg doses during the first 6-8 weeks. Infant formula is fortified with vitamin K. Dark skin
and low sunlight exposure increase the risk of vitamin D deficiency.
Risk factors for vitamin deficiency should be identified in the child and in the diet. In the temperate countries there is strong evidence that
vitamin D deficiency exists. Data in our setting sparse. In addition large surveys have suggested that – 50% of children may have a
suboptimal vitamin A intake, although clinical deficiency is not seen.
The key step in ensuring adequate vitamin status in children are giving vitamin K @ birth, promoting a healthy balanced diet in early life and
maintaining a low threshold for giving supplementary vitamin D (5mcg/day; 200iu).

USE OF VITAMIN AND MINERAL SUPPLEMENTS


This may also be required in children with a poor diet, those on therapeutic diets, and to replace increased losses.
Expert dietetic assessment is appropriate and vitamin and mineral supplementations should go together with dietary advice. Children with
eating difficulties or those on vegan or vegetarian diet may be deficient in iron and zinc.
219
Compiled by Obasi. D. C. Chinedu

Children on exclusion diets for food allergy and low protein diets for inborn errors of metabolism or renal disease may need comprehensive
supplementation.
Diets excluding or reducing specific items such as milk, or fructose or sucrose may need supplementation with calcium or vitamin C
respectively. Low fat diets should routinely be supplemented with fat soluble vitamins.
A complete trace elements and vitamin supplement is needed for children being fed a modular diet.

Nutrient Intakes for Vitamins (Units/Day)


VITAMIN RNI RNI RNI Tolerable upper intake

0-6months 7-12months 1-3years

A(mcg) 350 350 400 800mcg/day (1-3yrs)

D(mcg) 8.5 7 7 25mcg/day (0-24 months)

E(mg) 0.4mg/g PUFA 0.4mg/g PUFA 0.4mg/g PUFA 10mg/100kcal Formula

K(mcg) 10 10 10 - (not given)

B1 (Thiamine)(mg) 0.2 0.2/0.3 0.5 -

B2 (riboflavin)(mg) 0.4 0.4 0.6 -

Niacin (equivalents mg) 3 4/5 8 2mg/day (1-3yr)

B6 pyridoxine (mg) 0.2 0.3/0.4 0.5 -

B12 (mcg) - not given - - 7.5/100kcal

Pantothenate (mg) 1.7 1.7 1.7 1.2/100kcal

Folic acid (mcg) 50 50 70 200mcg/day (1-3yr)

C (mg) 25 25 30 30mg/100kcal

Water: Is a primary component of the body representing about 70% of the body weight. It is necessary for body functions including
digestion and absorption and certain metabolic processes. Water regularly leaves the body through sweating, excretion, and breathing and
therefore must be replaced through food digestion and absorption.
Total body water (TBW) as a percentage of body weight varies with age. The fetus has very high TBW, which gradually decreases to
approximately 75% of birthweight for a term infant.
TBW is divided b/w 2 main components- ICF & ECF. The ECF is further divided into plasma water and interstitial fluid. The plasma water
is 5% of body weight.
The volume of plasma water can be altered by pathologic conditions, including dehydration, anaemia, polycythaemia, heart failure, abnormal
plasma osmolality, and hypoalbuminaemia.
The interstitial fluid, normally 15% of body weight, can increase dramatically in diseases associated with oedema, such as heart failure,
protein losing enteropathy, liver failure, nephrotic syndrome, and sepsis. An increase in interstitial fluid also occurs in patients with ascites or
pleural effusion.

Practice questions
1). Because of her own success with a low carbohydrate diet, the mother of an overweight 9yr old boy asks about low carbohydrate diet for
her son. As you discuss this with her, you point out that the greatest percentage of calories in the normal diet of the school-age child comes
from which of the following?
A. Carbohydrate B. proteins C. fat D. cholesterol E. whey

2). At a 2-month well child visit, a mother states her family has a history of cow milk protein allergies and wonders if soy formula would be
better for her baby. Which of the following statements regarding soy formula is correct?
A. Infants fed exclusively with soy formula display growth comparable to infants fed cow milk protein formula.
B. The protein in soy formulas is essentially nonallergic, and clinically significant soy protein hypersensitivity is extremely rare.
C. soy formula is most useful in children with well- documented, severe, gastrointestinal allergic reactions to cow milk protein.
D. soy formula should not be used by patients with a family history of coeliac disease.
E. infants fed soy formula should receive supplemental dietary calcium.

220
Compiled by Obasi. D. C. Chinedu

BEHAVIOURAL, SPEECH AND LANGUAGE DISORDERS IN CHILDREN


Combination of: PROF NGOZI C. OJINNAKA (020) and Dr. Uwaezuoke (019)

Learning objectives:
To understand the common behavioural disorders in children
To have a good knowledge of possible causes of speech and language disorder in any age group
To appreciate other associated features of the above disorders
To understand management of these disorders

Introduction: Autism spectrum disorder (ASD) is a pervasive neurodevelop-mental disorder characterized by social difficulties,
communicative deficits, and inappropriate repetitive behaviour.
Autism-autos means self (withdraws within self) and spectrum means it can be mild, moderate and severe in manifestation. It is the most
common lifelong and most frequent neurodevelopmental disorder.

The symptoms of ASD become apparent in early childhood, at around 2 to 3 years of age, and persist throughout an individual’s lifespan.
There is significant increase in prevalence of ASD with 1 in 68 American children being diagnosed with ASD. More common in boys.
Occurs in all races.

Risk factors/Causes: can be Genetic or Environmental


Prenatal: Prematurity, low birth weight, Advanced maternal age, Sodium valproate during pregnancy, Perinatal asphyxia, Congenital
infections (Rubella and encephalitis), Toxic exposure (fetal-alcohol syndrome).
Maternal medical conditions such as DM, obesity, hypertension and assisted reproductive techniques.
Post natal: Heavy metal exposure; Overdose of antibiotics etc

CLINICAL FEATURES/DSM-5 (2013)


1. Deficient social communication and interaction (all 3)
a) Marked deficit in nonverbal and verbal social communication
b) Lack of social reciprocity
c) Poor development and maintenance of peer relationships
2. Restricted repetitive patterns of behavior, interests and activities (at least 2)
a) Stereotyped motor or verbal behaviors or unusual sensory behaviors
b) Excessive routines & ritualized patterns of behavior
c) Restricted, fixated interests

3. Clinically significant, persistent, present since early childhood

Some Standardized Behavioral Diagnostic Tests


1. Childhood Autism Rating Scale – CARS (Schopler et al., 1980)
2. Autism Diagnostic Interview – ADI (Lord et al., 1989)
3. Autistic Diagnostic Observation Schedule – ADOS (Lord et al., 1989)
4. Modified Checklist for Autism in Toddlers -- M-CHAT (Robin et al., 1999)

Physical/neurologic features (None present in all the cases):


 Abnormal head growth curve
 Physical abnormalities/symptoms
 Motor findings
 Atypical sensory responses
 Sleep problems
 Language abnormalities

FREQUENT SENSORY FINDINGS: hyper- & hypo-sensitivity


 Touch
 Pain, temperature
 Proprioception
 Vestibular
 Audition
 Vision
 Taste
 Smell

221
Compiled by Obasi. D. C. Chinedu

SLEEP PROBLEMS
 Difficulty falling asleep
 Difficulty staying asleep
 Need for less sleep time
 Need for excessive sleep
 Inadequate circadian entrainment

Epilepsy in autism:
 Related to the severity, location, type of brain pathology/cognitive level
 Related to type of language disorder
 Rare in high functioning children
 Peaks in early childhood and in adolescence
 Rarely the cause of autistic regression

INTERVENTIONS: Discourage use of medical/dietary treatments that have no reasonable rationale. Urgent need to evaluate efficacy of
medical and educational interventions in well studied subgroups of individuals

TREATMENT OPTIONS:

SPEECH AND LANGUAGE DISORDERS IN CHILDREN- BY DR UWAEZUOKE N.A.

What are speech disorders?


Difficulty in producing speech sounds correctly, or fluently or problems with phonation (voice). Examples:
 Childhood apraxia of speech
 Dysarthria
 Stuttering
 Voice
 Orofacial myofunctional disorders
 Speech sound disorders: articulation and phonology processes

What are language disorders? Inability or poor ability of forming sentences or understanding others, sharing thoughts and ideas
completely. This may be receptive or expressive. Examples:
 Preschool language disorders
 Language-based learning disabilities (reading, spelling and writing)
 Selective MUtism

Childhood Apraxia of speech: Arises from poor coordination of speech at the level of the brain. Affected children have no dysarthria
(weakness of the tongue, lips and jaw). The child knows what he wants to say but has difficulty pronouncing the word. First words are late
and some words may be missing. Simplifies words i.e. uses easier pronunciation to replace complex ones. May have eating problems as well

With older children: Can understand better than he can talk. Imitated speech is clearer than spontaneous speech, although he has difficulty
in imitating speech. Has more difficulty saying longer words and phrases than shorter ones. Disorder is worse with anxiety

Causes: - Arises in the CNS - Genetic disorders or syndromes


- Stroke or brain injury
Note! It is not outgrown except treated
Treatment: Practice speech and watch self in the mirror (visual) or tactile as well as auditory feedback (multisensory feedback)
 Repeat syllables, words and sentences three to five times weekly
 Aim to improve coordination of muscle movement for speech

DYSARTHRIA: Motor speech disorder follows impaired movement of the muscles used for speech production – lips, tongue, vocal folds or
diaphragm. Symptoms includes:
 Slurred, mumbled speech
 Slow rate of speech
 Limited jaw, lip, tongue movement
 Change in voice quality

222
Compiled by Obasi. D. C. Chinedu

Causes: - CP - Muscular dystrophy - Brain injury - Stroke - Tumour


- Huntington’s disease - Multiple schlerosis - Parkisonism

Orofacial myofunctional disorders: Tongue moves forward in an exaggerated way during speech and/or swallowing. “Tongue thrust”
swallow is seen in infancy, but decreases with age and then disappears. They most often sound like /s/, /z/ = sh, zh. \Some becomes shome,
zone becomes zhone. Tongue protrusion may press against the teeth and hinder teeth growth and alignment.
Causes: - Familial - Allergies - Enlarged tonsils and adenoids - Excessive tongue sucking
Diagnosis is made by a team of dentists, orthodentists, pediatrician and SLP

Stuttering: Affects fluency of speech. Starts in childhood and may last throughout life. First appears at between two and half and four
years. More common in boys. Among elementary school pupils, it is less common but has a ratio of 3:1 (male: female). It is characterized by
disfluencies. May arise during specific assignments.
Circumlocution- rearranging words in a sentence. Participation restriction and pretense to forget words

Signs and symptoms: - Repitition of words or parts of words - Out of breath


- Blocked speech - Sound prolongation

Risk factors: Family history - Lasted >=6mths - Other speech or language disorders
- Strong fear about stuttering on the part of the child

Treatment
Behavioural: Control breathing - Rate of speech - Use of short phrases and sentences - Educate listener

Causes: Unknown. Genetics- 2 to 5 yrs dev of grammer


Once stuttering has started, other factors may cause more disfluencies.
Frustration of the child. Negative response from listeners

Language disorders
 Pre-school language disorder
 Language-based learning disability
 Selective mutism

Pre-sch. L.d: 3-5yrs may have problems understanding and talking. Difficulty understanding gestures. Following directions is difficult.
Cannot answer questions. Cannot easily identify objects and pictures. Tough taking turns when talking with others.
Expressive: Asking questions, naming objects, using gestures, putting words into sentences,
learning songs and rhythms knowing how to commence and keep a conversation

Causes: - Hearing loss - Autism - LBW - Premature birth


- Intellectual disabilities - Syndromes- down, fragile x, fetal alcohol - Stroke
- Family history - Brain injury - Tumours - CP
- Poor nutrition - FTT

223
Compiled by Obasi. D. C. Chinedu

NMJ DISORDERS
Disorders of Neuromuscular Junction: These are a common cause of fluctuating fatigue and muscle weakness without sensory deficits.
NMJ disorders results from destruction or absence of one or more key protein involved in neurotransmission. These disorders can be
Disorders of: post –synaptic receptors eg Myasthenia Gravis, LEMS
pre-synaptic release of Acetylcholine eg Botulism
Breakdown of Acetylcholine within the synapse (drugs, neurotoxic chemicals)

Myasthenia Gravis: An acquired autoimmune disorder caused by anti-body mediated blockade of neuromuscular transmission resulting in
skeletal muscle weakness. The antibodies are formed against the niconitic acetylcholine post synaptic recepors at the NMJ of the skeletal
muscles.
Signs and Symptoms:
 Fluctuating fatigue / muscle weakness that worsens with activity and improves with rest. This is the hall mark.
 Weakness progresses from mild to more severe over weeks or months, with exacerbations and remissions.
 Ptosis and extra ocular muscle weakness – earliest and most consistent sign
 Dysphagia, facial weakness and changes in facial expression, feeding difficulties especially in infants.
 Poor head control, muscle weakness – fingers, limb girdle
 Rapid fatigue of muscles e.g. an upward gaze > 30-90 secs, raise arms > 1-2 minutes
 Progressive and life threatening because of involvement of respiratory muscles: difficulty swallowing, shortness of breath
 Infants born to myasthenic mothers may have a transient neonatal myasthenic syndrome secondary to placentally transferred anti-ACh
receptor antibodies.

Triggers of Exacerbation: - Warm weather - Surgery - Immunization


- Emotional stress - Menstruation - Intercurrent illness (eg, viral infection)
- Tapering of immunosuppression - Pregnancy and postpartum period
- Worsening of chronic medical illnesses (cardiac, renal, autoimmune, etc.)
- Medications (eg, aminoglycosides, ciprofloxacin, clindamycin, phenothiazines, chlorpromazine, diazepam, halothane, ketamine, lidocaine,
non-depolarizing neuromuscular blocking agents, chloroquine, procaine, lithium, phenytoin, beta-blockers, procainamide, calcium channel
blockers, d-penicillamine, high-dose prednisone, magneisum)

Classification: The Myasthenia Gravis Foundation of America Clinical Classification divides MG into 5 main classes and several
subclasses:
Class I: Any ocular muscle weakness; may have weakness of eye closure; all other muscle strength is normal
Class II: Mild weakness affecting other than ocular muscles; may also have ocular muscle weakness of any severity
Class IIa: Predominantly affecting limb, axial muscles, or both; may also have lesser involvement of oropharyngeal muscles
Class IIb: Predominantly affecting oropharyngeal, respiratory muscles, or both; may also have lesser or equal involvement of limb, axial
muscles, or both
Class 111: Moderate weakness affecting other than ocular muscles, may also have ocular weakness of any severity.
Class IV: Severe weakness affecting other than ocular muscles; may also have ocular muscle weakness of any severity
Class IVa: Predominantly affecting limb, axial muscles, or both; may also have lesser involvement of oropharyngeal muscles
Class IVb: Predominantly affecting oropharyngeal, respiratory muscles, or both; may also have lesser or equal involvement of limb, axial
muscles, or both. Use of a feeding tube without intubation places the patient in class IVb.
Class V: Defined by the need for intubation, with or without mechanical ventilation, except when used during routine postoperative
management.

Laboratory Investigations: Anti Ach antibodies – may be present


Electromyography – Decremental response is seen in response to repetitive nerve stimulation
Chest X-ray – usually shows an enlarged thymus ECG – usually normal
Other serological tests eg ANA, if positive, then other diagnosis should be explored.
Tensilon/Edrophonium Test: A clinical test for myasthenia gravis.
Administration of a short-acting cholinesterase inhibitor, (edrophonium chloride) Ptosis and ophthalmoplegia improve within a few
seconds, and fatigability of other muscles decreases. The drug blocks the breakdown of acetylcholine and temporarily increases the levels
of acetylcholine at the neuromuscular junction.
Precaution: An intravenous infusion should be started to enable the administration of medications in the event of an adverse reaction. ECG
monitoring during test is recommended.
A dose of atropine sulfate (0.01 mg/kg) should be available in a syringe, ready for intravenous administration at the bedside.

Treatment:
Mild: Oral AchE inhibitors – pyridostigmine. Oral steroids in cases of pure ocular MG
Moderate: Admit the patient. Increase the dose of pyridostigmine or subcut Neostigmine.
Steroids
Immunosupressants (Azathioprines, Methotrexate, cyclosporine): low frequency of spontaneous remission and high incidence of side effects

224
Compiled by Obasi. D. C. Chinedu

of steroids.
Plasmapharesis: IV Immunoglobulin, Thymomectomy

Neonates with transient maternally transmitted myasthenia gravis require cholinesterase inhibitors for only a few days or occasionally for a
few weeks.

Myasthenic crisis: This is a complication of myasthenia gravis characterized by worsening of muscle weakness resulting in respiratory
failure that requires intubation and mechanical ventilation. Can be the first presentation in an undiagnosed myasthenic.

Management involves: - ABC - Steriods - IVIG - Plasmapheresis

Congenital myasthenic syndrome: A group of conditions characterized by muscle weakness that worsens with physical exertion. It is
inherited in an autosomal recessive pattern, rarely autosomal dominant. Usually begins early in childhood. Presents with delayed
developmental milestones

Complications: Prolonged paralysis following Anaesthesia/use of neuromuscular blocking drugs and other drugs like Aminoglycosides.

BOTULISM: Results from ingestion of the toxin from the organism clostridium botulinium. A gram negative bacillus. Incubation period is
short
Clinical features include nausea, vomiting, diarrhoea
Cranial nerve involvement: diplopia, dysphagia, weak suck, facial weakness, and absent gag reflex.
Generalized hypotonia and weakness, respiratory failure

Investigations: EMG: Neuromuscular blockade with repetitive nerve stimulation.

Management: Respiratory support may be required for days or weeks until the toxin is cleared from the body.
No specific antitoxin is available.

MUSCULAR DYSTROPHIES
BISI-ONYEMAECHI A.I

Introduction: Dystrophy: Abnormal growth


Criteria; Must be a primary myopathy, Must be genetic, Must be progressive. Degeneration and death of muscle fibre occurs at some stage.

Types of muscle dystrophies:


- Duchenne - Becker - Emery Drefius - Myotonic - Limb girdle
- Facioscapulohumeral - Congenital

Duchenne/Becker Muscle Dystrophy: Most common affecting all races and ethnic groups. Inheritance is X-linked recessive(Xp21) - > in
boys. 30% from new mutations. Incidence is about 1:3600 live born males.
Poor head control first to be noticed, other milestones may be normal. Hip girdle weakness – lordosis. Gowers sign is present at 3 years.
Confinement to a wheel chair at 7 years. Scoliosis usually following confinement
Wasting of thigh muscles, Calf muscle, tongue and forearm hypertrophy. Distal and extraocular muscles are usually preserved.
Cardiomyopathy and intellectual impairment is a constant feature. Higher prevalence of epilepsy.
Death – around 18 years, usually from respiratory failure, HF, aspiration and pneumonia

HYPERTROPHY is secondary to Fat infiltration, Hypertrophy of some muscle fibres and Proliferation of collagen

Laboratory features: Serum creatine kinase is markedly increased (15,000-30,000IU/l).


EMG features of myopathy but non-specific. Muscle biopsy is diagnostic.
PCR. 2D-ECHO, CXR, and ECG needed for management
Prenatal diagnosis possible at 12 weeks

Muscle biopsy:
 Endomysial connective tissue proliferation,
 Scattered degenerating and regenerating myofibers,
 Myofibre calcification

225
Compiled by Obasi. D. C. Chinedu

 Foci of mononuclear inflammatory cell infiltrates because of muscle fiber necrosis,


 Mild architectural changes in functional muscle fibers.

Treatment: Symptomatic: Antibiotics for pulmonary infections


Good nutritional status, Ca and F supplementation in confined patients
Appropriate physiotherapy +- steroids
Becker follows a milder protracted course.

Emery Dreifuss: X –linked recessive. Clinical features begin in middle childhood. Elbow and ankle contraction, muscle wasting in the
scapulohumeral distribution. Nil muscle hypertrophy or facial involvement
Severe cardimyopathy (conduction defects, VF). Normal Intellectual function
Laboratory: Slight elevation of creatine kinase. Ms biopsy: non specific fibronecrosis and fibrosis
Treatment is supportive, pacemakers may be required.

Myotonic muscular dystrophy: Inheritance is autosomal dominant. Affects both skeletal and smooth muscle.
A multisystem disease: gut, uterus, cataract, dysmorphic, intellectual impairment.
Facies: inverted v upper lips, thin cheeks, scalloped temporalis, wasting of the muscles of the hand, neck and tongue
Distal muscles are more affected. Slowly progressive.
Myotonia and slurred speech present.
Constipation and encopresis.
Endocrine abnormalities: hypothyroidism, diabetes, testicular atrophy
Laboratory: Normal or slightly elevated creatine kinase. Hormonal evaluation. Muscle biopsy not required
Treatment: if symptomatic and supportive.

Limb Girdle ms dystrophy: Autosomal recessive and autosomal dominant.


Affects shoulder and limb girdle. Atrophy and weakness of the distal muscles. Calf hypertrophy
Usually present in mid/late childhood. Rarely involves facial and bulbar muscles
Heart and intellectual function also sparingly involved.

Faciohumeralscapulo ms dystrophy: Autosomal dominant. Facial and scapular muscle most affected. Winging of scapular prominent.
Wasting of triceps, biceps, hips and thigh muscles.
Contractures are rare. No calf hypertrophy
Treatment is supportive.

Congenital (misleading term): Autosomal recessive. Severe disease at birth


Contractures, athrygryposis, and gross hypotonia present.
Thin muscle mass and poor head control. Sucking, pharyngeal and extraocular muscles spared.
Fukayama type has severe cardiomyopathy and brain malformation.
IQ is affected in other types.
Treatment is supportive

226
Compiled by Obasi. D. C. Chinedu

PAEDIATRICS SEPSIS/ ANTIBIOTICS STEWARDSHIP


UJUNWA FA

Outline
- Introduction - Definition of terms - Sepsis, the current trend - Etiology
- Pathophysiology - Clinical manifestations - Diagnosis - Treatment
- Complications - Antibiotic stewardship concept

Introduction: Sepsis is a leading cause of death, morbidity and expenditure, contributing to one third to half of the deaths in hospitalized
patients. Management of sepsis is quite challenging requiring early recognition, management of infection, antibiotic stewardship,
haemodynamic issues and other organ dysfunctions.

Definition of Terms:
Sepsis is a clinical syndrome that complicates severe infection and is characterized by Systemic Inflammatory Response Syndrome, immune
dysregulation, microcirculatory derangements and end organ dysfunction.
SIRS + proven or suspected infection.?
SIRS is an inflammatory cascade that is initiated by the host in response to infection with bacteria, virus, fungi and protozoa. It occurs when
the host defense system does not adequately recognize or clear the infection.
In this syndrome tissue remote from original site of insult display the cardinal signs of inflammation.
Including but not limited to vasodilatation, increased microvascular vascular permeability, leucocyte accumulation.
Important factor in sepsis is immune dysregulation. Usually with an increase in pro-inflammatory and anti-inflammatory mediators.
Severe sepsis: Presence of sepsis combined with organ dysfunction.
Septic Shock: severe sepsis and the persistence of hypotension, or hypoperfusion for > 1hr despite adequate fluid resuscitation or
requirement of inotropic agent or vasopressors. Sepsis with cardiovascular dysfunction. It occurs when there is two of the following
prolonged capillary refill time, oliguria, metabolic acidosis or increased lactate despite administration of more than 40ml/kg of isotonic saline
in one hour.
MODS; Multiple Organ Dysfunction Syndrome

Current concept: The former used to be the old classification and definitions but Sepsis Surviving Campaign update 2016; Sepsis -3
recognizes Sepsis and septic shock only (third international consensus definitions for sepsis and septic shock); using SOFA (Sequential organ
failure assessment score) though on going validation in children (ACCP).
Thus sepsis is currently defined as when the body’s response to infection causes life threatening organ dysfunction.
Septic shock is sepsis that results in tissue perfusion with vasopressor- requiring hypotension and elevated lactate levels. Since the disease is
a continuum differentiating severe sepsis and septic shock is usually clinically difficult

SOFA: - Increased respiratory rate - Altered mental status - Decreased systolic blood pressure
- Deranged creatinine and biluribin - Platelet abnormalities, Pao2/Fio2 - Glasgow coma score
- Mean arterial blood pressure or requirement of vasopressors
Thus defining sepsis as life threatening organ dysfunction caused by dysregulated host response to infection.

Epidemiology: Sepsis is a frequent cause of death in the developing world. The risk of sepsis is inversely related to age in children.
Neonates are at the highest risk 1-10 per 1000live births. No sex predilection except for urosepsis.
More common in patients that has hbSS. Mortality for paediatrics sepsis ranges from 9%to 35%. Almost half of neonatal deaths is caused
by sepsis. In Nigeria Case fatality for paediatrics sepsis ranges from 21.5-40%

Aetiology: Sepsis may develop as a complication of localised infection or may follow colonization and mucosal invasion by virulent
pathogens.
Patient at risk of sepsis include: neonates, preterm, infants admitted in hospital especial ICU infants, children with serious injuries, chronic
antibacterial therapy, malnourished children, steroid therapy, chronic medical illness immunocompromised patients, transplant, malignancy
on chemotherapy, congenital immune deficiencies, splenic dysfunction, congenital heart diseases, burns, indwelling devices
Myriad of bacteria, viruses, fungi and parasites. Among the bacteria causes, there are age related causes:
Neonates- Streptococcus agalactiae, E. coli, Listeria monocytogenes
Late neonatal infections are usually due to staphlococcus auerus, coagulase negative staph, E.coli, klebsiella spp, Enterobacter,spp,
Pseudomonas.
Infancy: H influenza type b, Strep .pnuemoniae Salmonella spp, Neisseria meningitidis
Childhood: The same organisms that cause sepsis in infancy causes it in childhood. However the presence of encapsulated organism
becomes less as the child’s immune system matures

Other pathogens include: Rickettsia rickettsia, Candida, Aspergillus (fungi) esp in immunocompromised patients in which the polymicrobial
sepsis occurs.

Pathogenesis of sepsis and organ dysfunction syndrome: The host immune response through the actions of humoral and cellular immune
system and reticuloendothelial system prevents the body from developing sepsis in response to breaches to the host defense system. Host
227
Compiled by Obasi. D. C. Chinedu

response to infection is usually through production of inflammatory highly toxic mediators especially cytokines. If this cascade is
uncontrolled, SIRS occurs with subsequent organ and cellular dysfunction as a result of microcirculatory system dysfunction.
Series of events ensues which include; complement activation, endothelial injury, changes in the cardiovascular tone, platelet aggregation
and obstruction of the capillary beds.
In sepsis SIRS is caused by infectious agent or toxins.
The outcome depends on the intricate interplay of upregulating and downregulating cytokines and inflammatory cells and the direct effects of
the insult itself. The earliest manifestation of SIRS is a traid temperature alteration, tachypnea, and tachycardia. If it identified early
subsequent cascade can be mitigated. if not it will progress to organ damage. 2 out of the 4 SIRS MUST be met and 1 out of the two must
be either a or b
a. Increase temp of 38.5 degrees or decrease <36.5
b. Leukocyte count elevated or depressed or >10% immature neutrophils
c. Tachycardia >2SD above normal for age in the absence of external stimuli or drugs or bradycardia (<1yrold) in absence of vagal
stimulation, drugs or congenital heart disease.
d. Tachypnea Respiratory rate> 2SD above the normal for age or need for mechanical ventilation

Cellular response: Decreased oxygen delivery-decreased oxidative phosphorylation. - Anaerobic metabolism


- Glycogen depletion - Lactate production and fatty acid - Decreased ATP production
- Prostaglandin production.

Biochemical response: - Production of arachidonic acid metabolites - Myocardial depressant factors


- Endogenous opiates activation of complement system as well as production of many other mediators: decrease sympathetic activity and
release of vasoactive mediators that cause increase capillary permeability, vasodilatation and platelet aggregation.
- Myocardial depressant factors, TNF and some interleukins cause myocardial depression via direct injury and also by an intracardiac
increase in nitric oxide synthase.

Thus the clinical manifestation of sepsis and septic shock are mediated through the inflammatory cascade due to stimulation of these pro
inflammatory and antiinflammatory mediators by the TNF and interleukins.
These inflammatory cascacade is initiated by toxins or superantigens of gram positive organisms or endotoxin (lipopolysaccharide)
glycoprotein components of cell wall of gram negative bacteria as well as fungi.
In sepsis the body has compensatory mechanisms to maintain blood pressure through increased heart rate and peripheral vasoconstriction
when these compensatory fails, hypotension ensues. This is a late sign in infants.

Septic shock is a combination of 3 classic shock: - Hypovemic - Cardiogenic - Distributive shock


the degree of manifestation is variable
In children cold shock which involves decreased cardiac output and elevated systemic vascular resistance is more common while
warm shock is more common in adult.

Clinical manifestation: - Alteration in temperature - Tachycardia - Tachypnea


- Toxic appearance - Distended abdomen - Jaundice.
- Altered mental status (confusion, irritability, agitation, lethargy, coma, hypotension(late sign) cold extremities.
- Delayed capillary refill time
- Cutaneous lesion: purpura, petechiae, diffuse erythema, ecchymosis, ecthyma, gangrenosum, symmetric peripheral gangrene.
- The patient may have signs/symptoms of local infection eg rales in pneumonia, neck stiffness in meningitis etc.
Investigations: - Complete blood count - Electrolyte and liver function tests - Coagulation profile
- Etiology specific serology - Acute phase reactants - Culture of fluids, CSF, pleural fluid, urine abscess
- Chest xray - Ultra sound, echocardiography

Differential diagnosis: Localised infections such as UTI, Pericarditis, bacteral meningitis, Paediatric adrenal insufficiency.

Diagnosis: Diagnosis depends on clinical presentation with evidence of SIRS and proven or suspected infection.
Laboratory abnormalities include thrombocytopenia, prolonged prothrombin and partial thromboplastin time, elevated fibrin products,
Anaemia, elevated WBC count with neutrophilia, left shift of WBC, vacuolation of neutrophils, neutropenia.
Electrolyte derangements, hyperglycaemia or hypoglycaemia, hypocalcaemia, hypoalbuminaemia, hyperlactaemia, low serum bicarbonate
Deranged liver function test and renal function test in MODS. Decrease PaO2 and increased PCO2
Culture of the offending organism
CT SCAN, MRI. these later investigations depend on the clinical context.
Other specific cultures or serology eg viral studies, fungal or protozoa serological studies

Treatment: Treatment of sepsis involves diligent antibiotic stewardship. Early recognition and treatment is important in reduction of
mortality and morbidity. Choice of antimicrobial is dependent on the predisposing risk factors and clinical situation. Bacterial resistance
patterns in the community should also be considered when selecting the optimal antimicrobial therapy.
Empiric antimicrobial therapy should be based on pathogens most frequently encountered in each age group, history of comorbidities,
clinical syndrome, gram stain data, local resistance.
228
Compiled by Obasi. D. C. Chinedu

Broad spectrum bactericidal synergistic antimicrobial agent should be promptly administered to the patient with septic shock or
identified sepsis within the first hour. This has been shown to improve mortality.
Stabilization and correction of metabolic, circulatory and respiratory derangements

Suggested regimens:
Most children with septic shock should receive antibiotic coverage for methicillin resistant staph aureus.
Coverage for enteric organism when there is genitourinary or gastrointestinal sources.
Treatment for pseudomonas when neutropenic or immunocompromised.
Children 28days & above: the common antibiotics used: vancomycin and 3rd generation cephalosporin
Genito urinary/consider an aminoglycoside in addition or metronidazole or clindamycin when there is gastrointestinal source. Review with
culture and sensitivity results.

Early goal directed therapy: - Early antibiotic administration and effective de-escalation
- Oxygen supplementation - Infection source control - Haemodynamic stabilization
- Fluid resuscitation - Correction of coagulopathy/hb - Ionotropic /vasopressor support
- Monitoring of fluid responsiveness and end organ dysfunction
Adjunctive therapy- steroid for fluid refractory catecholamine resistant shock and suspected and proven adrenal insufficiency
Others: Telipressin, Inhaled nitric oxide, Pentoxifylline, IVIG, ECMO- extracorporal membrane oxygenation

Prognosis: Mortality depends on the site of infection pathogen, MODS and host immune response.
Complications include SHOCK, MODS. Neurological deficit, Disabilities.
Death; severe sepsis and septic shock causes about 30-50% mortalities

Prevention: Hand washing, Immunisation, Prophylaxis for specific agents and individuals. Prevention of noscomial infection

ANTIBIOTIC STEWARDSHIP: It is a coordinated program that promotes the appropriate use of antimicrobials to improve patient
outcome, reduce antimicrobial resistance and decreases spread of infection caused by multi resistance strains. It stems from problem of
antimicrobial resistance and rising multi resistant strains, stagnant development of antibiotics. With increasing importance of antibiotics in
clinical practice.
Careful and responsible management of something entrusted to one’s care. Once there is indication for antimicrobial use, it must be given
right, doing it right. Right drug, time, dose, duration, route.

Need: 50% of antimicrobial use is inappropriate. More than 70% of intervention used to improve antimicrobial use in the hospital had
beneficial effects.

Inappropriate use: - Polypharmacy - Prolonged duration for prophylaxis or not indicated


- Treatment of colonizers - Asymptomatic bacteriuria - Treating viral infections with antibiotics

Goals: - Optimal clinical outcomes - Minimize toxicity and adverse drug reaction
- Limit selection of resistant strains - Reduce cost

Recommendations:
Multidiscpilinary approach, everybody in the health team. Physician, epidemiologist, infectious disease physician. Microbiologist,
Pharmacist Collaboration.
Administrative /leadership support

ASP: - Education - Formulary - Restriction/ preauthorization - Selective reporting


- Audit and intervention feedback - Clinical pathways and guidelines - De-escalation therapy
- Dose optimization - Parenteral to oral conversion
- Health care information technology - survillance

Antibiotic resistance is a global problem. ASP is the key in combacting this. Physicians are key to this strategies.
Treatment of infections should be based on most likely organisms following local resistant pattern
We need to protect available antibiotics while waiting for the new ones.

229
Compiled by Obasi. D. C. Chinedu

OBSTRUCTIVE LUNG DISEASES IN PEDIATRICS


By Dr Adaeze Ayuk

OBJECTIVES/LEVELS OF REASONING EXPECTED


 Case scenario – Level 1 to 3 reasoning
 Understanding obstructive vs. restrictive lung disease
 Asthma as an obstructive lung disease

Introduction:
Lung conditions: obstructive lung disease and restrictive lung disease.
They share the same main symptom: shortness of breath with exertion.

What is Restrictive Lung Disease? Conditions that cause stiffness in the lungs parenchyma or stiffness of the chest wall, weak muscles, or
damaged nerves/diaphragmatic paralysis causing restriction in lung expansion.
Patients cannot fully fill their lungs with air. Their lungs are kept from fully expanding.
Examples: Interstitial lung disease (idiopathic pulmonary fibrosis), Sarcoidosis, Obesity (including obesity hypoventilation syndrome),
Scoliosis, Neuromuscular disease, (muscular dystrophy, amyotrophic lateral sclerosis (ALS)

What Is Obstructive Lung Disease? They result from narrowing of the airways, leading to difficulty exhaling all the air from the lungs
(exhaled air comes out more slowly than normal).
Examples: Chronic obstructive pulmonary disease COPD (emphysema and chronic bronchitis)
Asthma, Bronchiectasis, Cystic fibrosis, Bronchiolitis.

ASTHMA IN CHILDREN
Outline: - What asthma is - Asthma phenotypes/wheezing phenotypes -Pathophysiology
- How to make a diagnosis of asthma - How to classify chronic state asthma
- How to classify exacerbation episodes of asthma (acute asthma) - How to manage asthma

What is asthma? Asthma is like a clinical “syndrome”. Asthma is a chronic inflammatory disorder of the airways.
In susceptible individuals, this inflammation causes recurrent episodes of wheezing, breathlessness, chest tightness and coughing,
particularly at night or early in the morning.
These episodes are usually associated with widespread but variable airflow obstruction that is often reversible either spontaneously or with
treatment.

Pathogenesis:

Broad asthma classification-atopic etiology index


 Allergic or atopic asthma - IgE mediated
 Non-allergic or non-atopic asthma – multifactorial:

Mechanisms in Allergic asthma: With exposure to a trigger, a cascade of cellular (T) responses result in:

230
Compiled by Obasi. D. C. Chinedu

- Increased mucus production - Mucosal swelling - Bronchial muscle contraction


Early acute effect: Hyper responsiveness and Obstruction
Late acute effect: Recurrence of symptoms in 4-12 hours
Chronic effect: Airway remodeling

Pathophysiologic/Anatomic alterations of the lungs during “asthma attack”


 Smooth muscle constriction of bronchial airways (bronchospasm) – chest tightness, wheeze
 Excessive production of thick, whitish, tenacious bronchial secretions - cough
 Hyperinflation of alveoli (air-trapping) – fast breathing, blood gas(ABG) derangements
 Mucus plugging and, in severe cases, atelectasis – hypoxemia, cyanosis, cough

Asthma Predictive Index (API) - Diagnosing asthma in young children: > 4 episodes/yr of wheezing lasting more than 1 day affecting
sleep in a child with one MAJOR or two MINOR criteria

Major criteria: - Parent with asthma - Physician diagnosed atopic dermatitis


Minor criteria: - Physician diagnosed allergic rhinitis - Eosinophilia (>4%)
- Wheezing apart from colds

Asthma diagnosis: Clinical diagnosis supported by the certain historical, physical and laboratory findings
History of episodic symptoms of airflow obstruction (e.g. breathlessness, wheezing, COUGH)-response to therapy
Physical examination: wheeze, hyperinflation etc –bedside/chest assessment.
Laboratory: usually more specific. Exclude other possibilities

Bedside physical examination: Increased respiratory rate, Use of accessory muscles of respiration
Substernal/intercostal retractions (hoovers sign) - Increased anteroposterior chest diameter (barrel chest)
Cyanosis - Pulsus paradoxus - Decreased blood pressure during inspiration

Respiratory rate/HR
Resp Rate: <1: 30-40 1-2: 25-35 2-5: 25-30 5-12: 20-25 >12: 15-20
Heart Rate: <1: 110-160 1-2: 110-150 2-5: 95-140 5-12: 80-120 >12: 60-120

Chest assessment (IPPA)/ Findings: Prolonged expiratory phase Decreased tactile and vocal fremitus
Hyperresonant percussion Loss of cardiac dullness Diminished breath sounds
Diminished heart sounds Wheezing/rhonchi

Laboratory findings
Sputum examination: Eosinophils, Charcot-Leyden crystals (break down product of eosinophils)
Casts of mucus from small airways (called Kirschman spirals)
Blood examination: >4% Eosinophils, Ig E levels: Total, specific(elevated), Blood gas analysis
Skin prick allergy test
Chest radiography (when indicated)
Lung function test: PEFR Spirometry-FEV1, FVC, FEV25-75, FEV/FVC
Provocation test- Exercise test, methacholine/histamine
Others: exhaled nitric oxide (eNO)

Arterial Blood Gas derangement: Mild to Moderate Asthma Episode


 Acute alveolar hyperventilation with hypoxemia
pH PaCO2 HCO3- PaO2

   (Slightly) 
Arterial Blood Gases: Severe Asthmatic Episode (Status Asthmaticus)
 Acute ventilatory failure with hypoxemia
pH PaCO2 HCO3- PaO2
   (Significantly) 

Clinical classification of asthma: Classification of asthma: clinical severity & control.


 CHRONIC (stable) STATE

231
Compiled by Obasi. D. C. Chinedu

 Severity of EXARCEBATION (acute asthma).

Check Symptoms frequency


Step 4: Continual symptoms Step 3: Daily symptoms
Step 2: Symptoms > than twice weekly Step 1: Symptoms < than twice weekly
Check Timing/severity
- Day time - Night time - Need for inhaler - Activity limitation
- PEFR - ER admission frequency

Asthma diagnosis using lung function testing

Assessment of asthma severity


SEVERITY DAY TIME NIGHT TIME EXACERBATIO PEF/FEV1 PEF
LEVEL SYMPTOM SYMPTOM NS VARIABILIT
S S Y

MILD > Once per week > Twice per May affect At least 80% 20%-30%
PERSISTENT but not month but activity and predicte
every day not every sleep d
week

MODERATE Daily > Once per week At least twice per 60%-80% > 30%
PERSISTENT week; predicte
restrict d
activity or
affect sleep

SEVERE Continual Frequent Frequent; restrict ≤ 60% > 30%


PERSISITEN activity predicte
T d

Questions asked to determine asthma control


1) How many times/week do asthma symptoms (cough, wheeze, SOB) affect you during the day?
2) How many times/week do asthma symptoms disturb your sleep?
3) How many times/week do you use your relievers?
4) Has asthma caused time off work/school or interfered with your usual activities?
5) Have you needed to attend as an emergency since your last visit/over the last year?

Control classification: Classified (using several tools): - Well controlled - Partly controlled - Uncontrolled

232
Compiled by Obasi. D. C. Chinedu

Classification further determines the lowest amount of medication required

Validated tools for control classification: - ACT - cACT - ATAQ - ACQ

ACUTE ASTHMA EXACERBATION CLASSIFICATION


- Mild - Moderate - Severe (previously called status asthmaticus) - Life threatening

ASTHMA MANAGEMENT: 4 PRONGS


- Environmental Manipulation - Partnership in asthma management
- Pharmacotherapy (depending on if chronic or acute asthma) - Immunotherapy**
MANAGING THE LEVELS

CLINICAL ASSESMENT for classification of acute asthma

1. Environmental Manipulation: Identify precipitating trigger factors. Counsel both parents and children. Avoidance e.g. elimination of
house dust and house dust mite (rugs), avoidance of fried oil/cigarette smoking/ETS exposure, discourage families with asthmatic children
from acquiring pets or operating poultry farm.
Consider removal of patients from environment if efforts at removal of allergens from the environment fail.
2. Partnership in Asthma Management: Establish good partnership with patient, parent and teacher. Train the child/parent to recognize the
signs and symptoms of asthma. Instruct parents/patients on the use of drugs/equipment for asthma control e.g. inhaler, spacer devices, peak
flow meters, charts, asthma action plan (AAP) recordings, asthma diary. Encourage self-referral (to parents, doctors).
3. Pharmacotherapy: Chronic stable state - depends on control status judged by symptoms/signs in past 4 weeks – usually with controller
medication (ICS).
Acute exacerbation– treat according to classification of mild moderate or severe usually with oral steroids, inhaled brochodialators and other
medications in a step wise manner (GINA)

INITIAL MANAGEMENT OF CHILDREN WHO HAVE ACUTE ASTHMA


TREATMENT REQUIRED MILD EPISODE MODERATE EPISODE SEVERE/LIFE-THREATENING

233
Compiled by Obasi. D. C. Chinedu

Hospital admission Probably not required Probably required Yes, consider intensive care
necessary

Supplementary O2 Probably not required May be required Required. Monitor SaO2,


Monitor SaO2 Arterial Bloodgases maybe Req

Salbutamol (100 µg per 4-6 puffs (children< 6 6 puffs (children< 6 yrs) 6 puffs (children< 6 yrs) or 8-12
puff) yrs) 8-12 puffs (children ≥ 6yrs) puffs (children ≥ 6yrs) every
8-12 puffs (children ≥ If initial response is 20mins for 3 doses in 1st
6yrs) inadeq, repeat at 20 hour. If episode is life-
Review in 20mins (X 3) mins interval for threatening, use continuous
further 2 doses; then nebulized salbutamol. If no
give every 1-4 hrs response, bolus IV
salbutamol (15µg/kg) over
10mins, then 1µg/kg/min

Additional Ipratropium (20 Not necessary Optional depending on 2 puffs (children< 6 yrs) or 4
µg per puff) response puffs (children ≥ 6yrs) every
20mins for 3 doses in 1st
hour or use nebulized
ipratropium

IN ALL CASES START INITIAL STEROID DOSE AS REQUIRED

Systemic Yes (consider) Oral prednisolone (1mg/kg Oral prednisolone (1mg/kg/dose)


corticosteroids daily for up to 3 days) daily for up to 5 days
Methylprednisolone IV (1mg/kg)
every 6hrs on day 1, every 12hrs
on day 2, then daily

Magnesium No No MgSO4 50% 0.1mL/kg (50mg/kg) IV


over 20mins, then 0.06 mL/kg/h
(30mg/kg/h); target serum 1.5-
2.5 mmol/L

Aminophylline No No Only in ICU; loading dose 10mg/kg;


maintenance 1.1mg/kg/h if < 9
yrs or 0.7mg/kg/h if ≥ 9yrs

Chest radiograph Not necessary unless Not necessary unless focal Necessary if no response to initial
focal signs signs present therapy or pnuemothorax is
present suspected

Observations Observe for 20mins Observe for 1 hour after last Arrange for admission to hospital
after dose dose

Acute asthma - Discharge plan: Continue Beta 2 agonist 4 hourly as necessary. Consider prednisolone for up 3 days. Advise to come back
if not controlled on above treatment.
Provide a written asthma action plan (& diary). Review regular treatment. Check inhaler technique
Arrange regular follow up

Chronic asthma (step wise)

234
Compiled by Obasi. D. C. Chinedu

When it is NOT LIKELY asthma

In conclusion we now know:


- What asthma is - Asthma phenotypes/wheezing phenotypes - How to make a diagnosis of asthma
- How to you classify chronic state asthma - How to classify asthma exacerbations
- How to manage asthma (chronic and acute)

235
Compiled by Obasi. D. C. Chinedu

OTHER DISORDERS OF THE RENAL SYSTEM (Congenital malformations, Enuresis, Diabetes Inspidus &
Hypertension)

Development of the Renal System: The renal and genital system both develop from the intermediate mesoderm, a collection of cell at the
back of the foetal abdominal cavity. Both drain into the same space – foetal cloaca.
Form- Pronephros - cervical region
Mesonephros - below regression
Metanephros - pelvic region.- final adult kidney – start fxn 2nd trimester.

Congenital Anomalies: Congenital anomalies of the urinary system are not uncommon is clinical practice. Many of these are asymptomatic
and go unsuspected until there is complication resulting in symptom.

Anomalies of the kidney: can be


- Anomalies of Number - Anomalies of structure - Anomalies of location - Anomalies of renal vessels

Anomalies of number
A) Bilateral renal agenesis (Potter’s syndrome): male > female; Hx- oligohydramnious, SGA stillborn.
Characteristic potter’s facie with low set or malformed ear, prominent epicanthal fold, bird-like nose, small receding chin. There are
usually associated genital anomalies, pulmonary hypoplasia, and abnormalities of extremities. Not compatible with life. No known effective
management
B) Unilateral renal agenesis: Solitary kidney. Common in male. Asymptomatic. Associated with other system congenital anomalies.
Compatible with normal life span.
C) Supernumerary kidney: rare, with extra kidney. small and dysplastic.

Anomalies of kidney structure: Unilateral – No Asymptomatic


Bilateral result in progression deterioration in renal function and End Stage Renal Failure and require renal replacement therapy.
Aplasia – small with no renal tissue and atretic ureter
Hypoplasic – small size but structure is normal. Unilateral compatible with normal life but patient should be monitored for life, Bilateral,
polyuria with dilute and salt wasting with progressive deterioration in renal failure to end stage.
Dysplastic: Mal-development, disorganization and persistence of embryonic element, the smooth muscle and cartilage. It may be normal,
small or cystic in size.
The prognosis depend on the amount of functional renal tissue and presence of other anomalies like PUV or Vesico-Ureteric reflex

A) Multicystic Dysplastic Kidney: Commonest form of cystic kidney disease. Kidney tissue is replaced with non – communicating cysts
of varying sizes. Usually unilateral but abnormality of the opposite kidney are common. Treatment not necessary, kidney usually
involutes and shrinks.
B) Familial and hereditary Cystic Dysplasia.

4) Infantile Polycystic Kidney disease – AR. Born with large kidney and not compatible with life.
5) Adult Polycystic kidney disease – AD. Present after the 2nd decade but may present early. An unusual associated with intracranial berry
aneurysm. Present with Abdominal pain, haematuria, recurrent febrile episode, hypertension, bilateral abdominal mass. Deterioration in renal
function till the fourth decade of life.
Dx- IVU, Renal angiography, CT, MRI or USS
TRX- Correction of electrolytes derangement and treatment of hypertension. Manage Renal Failure
6) Juvenile Nephronopthisis AR. Small kidneys with cysts in the renal medulla of both kidneys. Pts present with polyuria, polydysia or
CRF. -Rx-Renal Replacement therapy.
7) Oligomeganephronia: Small kidneys with reduce number of nephrons which are abnormally large.
Presentation: Polyuria, Vomiting, dehydration, salt wasting, concentration defect and proteinuria
There is deterioration in renal failure to End Stage Renal Failure

Anomalies of location
A) Horseshore kidney; Infection, Stone formation B)- Ectopic kidney located in the pelvis or abdomen
Anomalies of Uninary tract
a) Duplicate of the ureters: common in girls. It may drain into the Urethra or vagina. Reflux, hydroureter, hydronephrosis, dribbling
recurrent infection and enuresis.
Diagnosis: IVU RX – surgical repair
b) Posterior Urethral Valve: obstruction mucosal fold on the floor of the posterior urethra. commonest cause of obstruction uropathy in
male children

Anomalies of the Bladder: Agenesis, duplication, patent urachus and extrophy


Vesical extrophy. The bladder is exposed due to absence of abdominal wall muscle and skin infection is common.
Miscellaneous condition:

236
Compiled by Obasi. D. C. Chinedu

Prime-belly syndrome: This is a triad of absent abdominal muscle, underscended tests and urinary tract dilatation.

Obstructive Uropathy: The obstruction lesion are mostly congenital but may occasionally be acquired. Complete reversal of the effect of
obstruction can be achieved by early treatment but a long standing obstruction lead to changes that are usually irreversible or only partially
reversible. The resultant impairment of renal function is detrimental to normal growth and development and may be life threatening.
Obstruction uropathy often present with non special feature and high level of clinical suspicion is necessary for it detect.

Pathophysiology: The effect of obstruction anywhere in the urinary tract are predictable with proximal holdup of urine resulting in stasis
which leads to dilatation, infection and increase pressure within the obstructed system.
There are reversible in the initial stages but lead to deterioration of renal function if the obstruction persists over a long period of time.
Cause: 1. Posterior Urethral valve in boys 2. Phimosis 3. Meatal stenosis
4. Stricture 5. Renal calculi 6. Neuropathic bladder 7. Ureterocele
8. Megaureter 9. Pelviureteric junction obstruction

Posterior Urethral Valves: PUV are the most common cause of Obstructive Uropathy in boys. Dilatation of posterior urethra and
hypertrophy of the urinary bladder follow.
Hx -dribbling of urine, weak stream, recurrent UTT, straining during micturition. Recurrent UTI, FTT, ESRD
Dx –VCUG. VUR and bilateral hydro-ureter and nephrosis may be present.
Laboratory tests- evaluate RF.

Pelvireteric Junction (PUJ) obstruction: The condition is characterized by an anatomical obstruction of the flow of urine from the renal
pelvis into the ureter.
Hx -flank mass upper abdomen or pain UTI. Gross haematuria, stone formation and deterioration RF
(Dietl’s Crisis)
DX: Ultrasandography and DTPA renogram
RX: Pyeloplasty

ENURESIS: Enuresis is defined as the involuntary passing of urine after the age of 5 years or inability to achieve bladder control
resulting in early complete evacuation of the bladder at a wrong place and time at least twice a month after the fifth year of life.
As a rule the bed will be soaking wet as against incontinence, which is loss of urine without normal emptying of the bladder. Bladder
control is usually attained between the age of one and five years. Up to the eleventh year enuresis is twice as common in boys as it is in girls
but there after the incidence is similar or slightly higher in girls.

Classification: Enuresis may be diurnal, nocturnal or both. Primary or Secondary


Primary Enuresis: when the child has never been dry.
Secondary Enuresis: occurs when after being dry for at least six months and started bed wetting.

AETIOLOGY: The cause of enuresis is always functional, in contrast to the cause of incontinence which may be organic or functional.
These are multi factoral and identification of the cause help in the choice of management
(1) MATURATIONAL DELAY- A delayed in the development of fine and gross motor skill may occur in some normal children especially
boys. exaggerated with stressful life events and anxiety.
(2) FAMILY HISTORY AND GENETICS: The risk of enuresis is as high as 40 percent in child with positive history in one parent and 70
percent if both parent had enuresis. The mode of inheritance appears to be autosomal dominant with reduced penetrance modulated by
environmental factor and these genes.
(3) SLEEP FACTOR; Deep sleepers and poor wake up signals from the full bladder. Instead of complete arousal, the switch only gets the
child to light sleep.
(4) LOW BLADDER CAPACITY: A low function bladder capacity has been compared with estimated bladder capacity. Calculated using
the age based formula.
(5) ADH: Lack of Circadian rhythm or impaired response of the kidney to anti diuretic Hormone may be a cause of enuresis especially
nocturnal enuresis.

MANAGEMENT: Based on history and initial examination, children with uncomplicated enuresis require no further evaluation.
Good Hx: Family history (70 percent). Anxiety, depression and stressful life events. Social disadvantage,
Other developmental problems
Investigation: USS and VCUG is the reserve for patient with suspected neurological or urological dysfunction.
Invasive procedure like Uroflowmetry, with or without pelvic floor and abdominal muscle EMG, cystometry and other urodynamic study is
reserved for patient with voiding disorder.

TREATMENT: Adequate clinical assessment is essential to determine the type and seventy of the condition and the appropriate mode of
treatment. The choice of treatment will depend upon the main concerns expressed by the family. A combination of a behaviour management
237
Compiled by Obasi. D. C. Chinedu

programme and medication is most effective.


A) Treatment of organic causes Rx UTI, DM DI
(B) Voiding Training
(C) Behavioural Therapy
(D) Motivational therapy
(E) Pharmaco therapy
(F) Psychotherapy

RENAL TUBULAR ACIDOSIS (RTA): This disorder comprises a group of tubular transport defects characterized by inability to
appropriately acidify the urine with resultant metabolic acidosis.
Pathophysiology: The underlying abnormality consists of an impairment of bicarbonate re-absorption (Type II) or excretion of H+ ions
(Type I) or a combination of both; and exists as three types in children.
The plasma bicarbonate level is low & the children have metabolic acidosis with normal anion gap in all the type.

Type I (Distal RTA): This is an important cause of severe rachitic deformities, failure to thrive and hypokalaemia complication in children;
there is also weakness, polyuria and nephrocalcinosis.
Lab finding: The urine is alkaline (pH > 5.5) despite system acidosis. (Blood pH < 7.2),
Hyper calcuria and hypocitraturia are present; serum KT is normal or low.
RX: Correct the acidosis with NaCO3 2-3 mEq/kg/day
CAUSES OF RTA I(Distal)
-Primary: Genetic, sporadic
-Secondary: #SLE, sjogren syndrome, #Sickle cell anaemia, #Obstructive Uropathy, #reflux nephropathy, #Nephrocalcinosis,
#Amphotericin, B. toxicity, #Prematurity and LBW.

Type II Proximal RTA: Isolated type II RTA is very rare, so it is often seen as a component of primary or secondary Fanconi syndrome.
CLINICAL FEATURES: Failure to thrive, vomiting and muscle weakness
Lab Findings: - Blood pH and bicarbonate level are low. - The urine pH may fall below 5.5.
- Normal Ca+ and citrate excretion.
Diagnosis is made by calculation the fractional excretion of bicarbonate.
Treatment: 2-5mEq/kg/day of Bicarbonate and restriction of sodium intake.
CAUSES OF RTA II (Proximal)
Isolated defect: -Primary: genetic, Sporadic
-Secondary (rare): Drug like carbonic anhydrase, inhibitor (acetazolamide)
Multiple tubular dysfunctions (Fanconi syndrome): -Primary: genetic, sporadic
-Secondary: cystineosis, lower syndrome, wilson’s disease, tyrosinemia, galactosemia, fructosaemia mitochondrial disorders, Vitamin D
dependent rickets, drugs e.g lead, ifosfamide, aminoglycosides, cisplatin, Amyloidosis, renal transplantation

TYPE IV RTA (HYPERKALEMIC): Mineralocorticoid deficiency. Can be;


+ Aldosterone deficiency without renal disease + Addison’s disease + Isolated aldosterone deficiency
+ Aldosterone deficiency in chronic kidney dx + Interstitial nephritis, nephrocalcinosis
+ Aldosterone resistance.

DIABETES INSIPIDUS (DI): This is spectrum of disorders with similar clinical features of polyuria and polydipsia in the absence of
osmotic diuresis as result of deficiency of Anti diuretic hormones(ADH) or tubular unresponsiveness to the hormones. Can be Central or
Nephrogenic.

Central DI (CDI): Secondary to a complete defect in ADH secretion in the posterior pituitary gland resulting in the inability of
concentrating urine appropriately despite normal renal function.
Pathoplysiology: Increase urine output, hypernatraemia and dehydration
Clinical Features; Polyuria, polydipsia, weight loss, growth failure
- patient generally prefer water to other fluid - There may be neurologic or visual complaints
- Feature may not be obvious in the infant who demonstrates instability, frequent feed requirement, unexplained fever, constipation, failure to
thrive and delayed development.
- There is a significant risk of hypernatraemia dehydration
LAB FINDINGS - Urine S.G <1.010, low urine osmolarity - Normal to high serum sodium concentration
- Water deprivation Test: Positive i.e. persistence of dilute urine with osmolarity less than that of plasma
- A rise in serum sodium to > 145mEq/l - A rise of serum osmolarity to > 290 MOSM/kg
- Weight loss of 3% - 5% - Other: skill radiograph, CT, MRT

Treatment: Desmopressin (dDAVP) is given intranasally once or twice daily

238
Compiled by Obasi. D. C. Chinedu

NEPHROGENIC DIABETES INSIPIDUS: There distal tubular unresponsiveness to ADH resulting in inability to concentrate the urine
hyposthenuria and polyuria.
Aetiology
Primary: Rare X-linked recessive disorder with profound effect in males although females may be mildly affected. In most families, the
defect is caused by a mutation in the vasopressin receptor. Autosomal dominant and recessive DI has also been described in which aquaporin
(The renal water channel) on chromosome 12q is defective
Secondary: More common and often less severe than primary. Obstruction Uropathy, Chronic renal failure, Sickle cell disease, Drug toxicity
CLINICAL FEATURE: Present in the first week of life in the severe form
-Polyuria, polydipsia, failure to thrive
Chronic dehydration: the degree of dehydration may be underestimated because the child continues to make urine.
-Fever, instability -Poor feeding are also common

LABORATORY FINDINGS - Hypernatraemia, hyperchloraemia


- Urine Osmolarity < 200 mOsm/kg in the present of serum osmolarity > 300 mOSm/kg
- ADH level are normal, and there is no response to exogenously administered vasopressin

Treatment 1) Maintenance of adequate fluid intake 2) Salt (sodium) restriction: 1mEq/kg/day


3) Administration of hydrochlorothiazide at a dose of 2 – 4 mg/kg/day. (Encourage proximal tubular Na + and water re-absorption)
4) Amiloride 20mg/m2 has an additive effect.

Differential Diagnosis - Psychogenic water drinking - Impaired thirst mechanism


- Diabetes mellitus

Hypertension: What is a normal BP in children...... normogram


(Age in years X 3) + 100= SBP 95th C. Blood pressure increase with age, Ht & gender
The increase in blood pressure with age is largely due to increase in height and weight. A normal value of blood pressure for age is < 90th
percentile. Pre-hypertension BP -values between 90 – 95th percentiles.
A systolic or diastolic BP values > 95th percentile should be repeated on at least 2 more occasions to confirm the diagnosis of
hypertension.
All children with elevated systolic or diastolic BP valves > 95 th percentiles need evaluation.
BP> 99th percentile is defined as severe hypertension and need more prompt evaluation and treatment.

MEASUREMENT OF BLOOD PRESSURE: This is done by direct intra-arterial measurement through non invasive method of
(1) Oscillometry (2) Mercury sphygmomanometery
(3) Ambulatory Blood Pressure Monitoring (ABPM).

CAUSES OF HYPERTENSION IN CHILDREN


SECONDARY CAUSES
A Renal parenchymal disease: Chronic GN, Reflux nephropathy, obstructive Uropathy
- Polycystic kidney disease, renal dysplasia
B Renal Tumors: - Wilms’ tumor, (Nephroblastoma) hemangiopericytoma
C Renovascular Disease: Idiopathic Aortoarteritis (Takayasu’s disease)
- Renal artery stenosis, renal artery thrombosis
D Cardiovascular disease: Coarctation of aorta
E Endocrine disease: Pheochromocytoma, neuroblastoma, primary hyperaldosteronism
- Cushing syndrome, congenital adrenal hyperplasia

PRIMARY CAUSES: Essential hypertension especially in adolescent

CLINICAL FEATURES: -Asymptomatic: headache, abdominal pain, nausea; vomiting and weight loss may be present.
Severe HBP- sensorial impairment, visual disturbance, focal neurological deficits, seizure, and heart failure, which may be the presently
feature. These occur at lower blood pressure compare to adult and in acute situation with rapid increase in the blood pressure.
In sick neonate blood pressure measurement is pertinent since hypertension manifest with feature suggesting sepsis, intracranial haemorrhage
or cardiopulmonary instability.

Evaluation of Sustained Hypertension


(A) Preliminary Investigation
- Urinalysis: cell, cast, protein, 24hrs protein excretion
- Urine culture
- Blood, urea creatinine, electrolytes, Bicarbonate, uric acid, calcium, fasting cholesterol, triglycerides
- Chest x-ray film
- ECG, ECHO, Adbominopelvic ultrasound
- Funds examination
239
Compiled by Obasi. D. C. Chinedu

- 99m TC – DMSA scan

(B) Additional Investigation: Depend on suspected underlying cause


Glomerulonephritis: Serum C3, C4, ASO. Autoantibodies (ANA, Anti-dsDNA, ANCA). Renal biopsy
Reflux Nephropathy: Micturating cystourethrogram. Dimercaptosuccinic (DMSA) renal scan. Intravenous Urogram
Renovascular disease: Doppler ultrasound. Captopril primed isotype scan (DTPA or MAG-3). Renal angiography or digital subtraction
angiography. Renin sampling from renal veins and interior vena cava
Phenochromocytoma: Urine and plasma catecholamines. Plasma calcitonin, parathormone. I¹²³-meta – lodobenzlguanidine (MIBG) scan,
CT, MRI. Arterriography and caval catecholamine sampling.
Coarctation of the aorta: Echocardiogran, angiography.
Other endocrine causesL Urine steroid profile. Plasma aldosterone, cortisol, DOC. Dexemethasome/ACTH test. Peripheral plasma renin
and aldosterone. Spot urine catecholamines.

TREATMENT: 2o HBP commonest in children and the treatments depend on the cause and degree of hypertension. Surgical treatment is
included in coarctation of aorta and most form of renal vascular hypertension such as renal artery stenosis, renal aneurysm are often cured by
revascularization surgical procedure and open or percutaneous transluminal angioplasty.

ANTIHYPERTENSIVE DRUG THERAPY: Children with symptomatic and/ or secondary hypertension, and those with features of target
organ damages require therapy with antihypertensive medication.
Persistent HBP despite adequate non pharmacologic measure in essentials HBP will also need antihypertensive.

Five classes of drug are most suitable in children in the management of HBP.
A) ACE inhibitors and reception blocker
B) B-Adrenoceptor blockser
C) Calcium Antagonists
D) Diuretic:
E) Adrenoceptor Blocker

OTHER DRUG: Centrally acting vasodilators- Hydralazine, Minoxidil, Clonidine.


Methyldopa.

The goal for anti-HBP: reduction of pressure below the 90th percents and initial therapy should be with single drug. Like calcium, channel
blocker, one of the initial medications is ineffective in lowering the blood pressure. It is necessary to use a drug from a different class.

240
Compiled by Obasi. D. C. Chinedu

THE FLOPPY INFANT


DR. UWAEZUOKE NDUBUISI

STUDY OBJECTIVES
 To understand the concept of floppy infant
 Be able to identify one in clinical setting
 Understand the underlying cause and the treatment/ mgmt. for each cause.
 Prognosticate for each disease entity.
 To understand the subtypes of each entity and how they are inherited so the learner can apply it to counseling and treatment.

Outline: - Introduction - Aetio-pathogenesis - Types


Mode of inheritance of each - Modern treatment if any - Prevention
Complications - Management

Introduction: Hypotonia and hyperextensibility= Ragdoll. Reduced active movement and no resistance to passive movement. Delay in
developing motor milestones. Slips through the hands of the examiner when held at the axilla.
Ventral suspension leads to droping hand legs and curved back. Suck and withdrawal from pain may be impaired.

Aetiopathogenesis: - LMNL - UMNL


Cerebral causes; downs syndrome, kernicterus, asphyxia, cns infections
Peripheral nvs; GBS, benign congenital hypotonia, Spinal muscular atrophy, polio
Neuromuscular diseases; Myaestenia gravis, progressive muscular dystrophy.

Down syndrome: Trisomy 21. 1:750 LB. Phenotype include


1. Dysmorphic changes which vary btw patients. 2. Mental retardation
3. Neurologic problem; hypotonia, alzheimer dz. 4. CHD
5. Leukemia 6.Immunological defects

Live span is shorter by 10 to 20 yrs esp female. Maternal age 30-35 yrs is a cause but poorly understood.
Triplication of part or all of chromosome 21. 3 types
 Free trisomy in 95%
 Translocation in 5%
 Mosaicism (2-4% of free trisomy)

Mechanism: Non dysjunction: of maternal chrsm 21; age dependent, occurs in meiosis I or II
Translocation: not age dependent, may involve maternal or paternal chromosomes
SOD 1 gene causes behavioural and learning difficulties, sequencing of long arm of X21 on going with interest in ncRNA(>200 nucleotides)
ncRNAs regulates protein coding genes as well as epigenetic changes

Spinal muscular atrophies (PNV): Degenerative dz of the anterior horn cell od spinal nv and motor nuclei of Cranial nv. Inherited as X-
linked recessive, few are dominant involves 5q 11-13 deletion. Degeneration starts inuterine in foetal life and continues into childhood with
involvement of axons and muscles. Severity varies with type
Infantile SMA type 1. most severe (WERDNIG HOFFMAN DISEASE)
TYPE 2 LATE INFANTILE, seen from 6 months
TYPE 3 JUVENILE TYPE (KULGELBERG-WALENDER)

Werdnig Hoffman (clinical features): Weakness, hypotonia, paucity of mvt, frog leg posture, abducted hip on a flex knee. Paradoxical
breathing from inter-coastal mzl weakness and normal diapgram, fibrillation of tongue, no deep tendon reflex, club foot and contractures at
birth, arthrogyposis complex in 10% of NB at birth. Swallowing may be difficult resulting in aspiration pneumonia, intact sensation,
sphincters are spared, normal mental development, extra-occular muscles are spared and facial muscles are intact.
Prognosis; 7mnths to 7 years
DX; mainly clinical. EMG suggest denervation. CK normal to raised/
Motor nv conduction velocity is normal while sensory nerve velocity may be decreased.
Muscle biopsy various stages of degeneration
DNA PCR from chorionic villi to buccal smear and blood.
Rx; physiotherapy, feeding tube, contracture correction

BENIGN CONGENITAL HYPOTONIA(pnv): NEWBORN hypotonia progressively improves. Most will be normal by age 10 years. Acq
motor milestone is delayed. normal intelligence. Hypotonia is due to CNS apraxia on gamma loop. Normal tendon reflex but shoulder
dislocation is common along with rare ventral spinal column which may result in cord compression.

Myasthenia gravis (NMD): Neuromusuclar jnx disorder resulting in weakness of skeletal muscles and fatigue.

241
Compiled by Obasi. D. C. Chinedu

To generate action potential(AP) which drives muscular movement


Preaxonal release of ACh--- binds to ACh receptors in muscle -------generates AP in the mzl and this propagated.------the ACH has to be
washed away if not the muscle gets into tetany or spasm. To wash away the ACh acetyl cholinesterase(AChE) hydrolyses the receptor ACh
bond to free the receptor for future activity
In MG auto antibodies(aAb) erode the receptors and so decreasing binding space for Ach. There is decreased postsynaptic receptors. When
these Igg Ab. cross placenta they cause symptoms in-utero.
Affects mainly striated voluntary muscles supplied by the cranial in childrden.
3 forms are recognised in children
1. Transient neonatal MG: Ab cross the placenta and bind to the babies Ach receptors blocking them temporarily.
CF, floppiness, difficulty swallowing, ptosis, reduced movement, poor suck, diminished reflexes, breathing difficulty, dysphagia etc.
Prognosis, wears off in 2 to 4 weeks as maternal Ab wanes.
Rx; supportive eg feeding tube, positioning, resp. support
2. Persistent congenital MG: Child is born with symptoms of MG but mother does not have the disease. It is most probably genetic,
familial but mother not a sufferer.
Eyelid and extraocular muscles more common involved and last for life.
3. Juvenile MG: Most common type. 6x more common in females. First starts abt 10 yrs. Extraocular muscle weakness is the most common
symptom, facial muscle weakness, limb girdle weakness is fairly common.
Myaestenia crisis generalised mzl weak with intercurrent infection or surgery

Diagnosis of MG: Hx of muclular weakness dt PROGRESSES with the day. Family hx of muscular weakness esp in the mother or sibling.
Progressive fatigue following muscular contraction.
Response to anti-cholinaseterase eg IV EDROPHONIUM CHLORIDE (TENSILON TEST) 0.2mg/kg don’t exceed 10mg. Musclar
strength increase last for 2min is confirmatory. Don’t use in neonate arrythmia
IM NEOSTIGMINE (0.04-0.08mg/kg) maybe used
EMG decreased response with repeated NV stimulation.

Treatment of MG:
Oral neostegmine 0.04mg/kg q 4-6 hrly
Oral pyridostigmine is an alternative 4 -10mg q 4hrly.
EBT in transient type
Steriods and IV Immunoglobulin
Thymectomy
Plasmapharesis in MG crisis.

Question
A 2yr old was noted to have delayed developmental milestone the mother had noted ptosis in the child, she was excessively weak in the first
month of life esp on the first day of life. What is the most likely dianosis considering the fact that there is no similar history of muscle
weakness in the mother.

Discuss the wednig hoffman disease as a cause of floppy infant syndrome.

242
Compiled by Obasi. D. C. Chinedu

DIAGNOSTIC PROCEDURES IN PAEDIATRIC PRACTICE


Dr Igbokwe O

Objective: To outline and describe the important procedures used in paediatric practice that aid in the diagnosis and treatment of clinical
conditions

Procedures in Paediatric practice


Diagnostic: Investigations to help arrive at a diagnosis. May be invasive or non invasive
Therapeutic: Relief of distressful situation
Both

Requirements: - Procedure room and good lighting - Restraint - Consent

Diagnostic Procedures: A process followed in making a medical diagnosis. Diagnostic evaluation of patients with suspected or known
disease involves the use of both invasive and non-invasive procedures.
- Invasive: May lead to tissue damage - Non-invasive: No tissue damage

Different diagnostic procedures and


systems involved (See table)

Common Procedures: -
Venepuncture - Intraosseous access
- Umbilical vein access
- Lumbar puncture -
Thoracocentesis -
Suprapubic aspiration - Exchange
blood transfusion

Venepuncture:
Indications: Collection of blood
samples for investigations. Infusion
of drugs and fluids
Instruments: Straight needles (25G,
23G, 21G, 19G). Cannulae (24G,
22G, 20G, 18G), Stylets, Guide wire
Containers: Vacutainers,
Bottles (Heparinized,
fluoride, nonheparinized)
Tourniquet
Materials for aseptic
procedures (Alcohol swabs,
Masks, Gloves:
sterile and non
sterile, Gowns)

Peripheral vein access:


Equipment
Aseptic materials: Straight/
butter fly needle/cannula, Containers/instrument
Procedure: Wear gloves, Clean site, Prepare sample bottles, Ensure skin taut/apply tourniquet.
Place needle over the site of a vein, Insert and advance needle (Cannula: advance tube and withdraw stylet)
Pin Prick: Sites (Capillary: Finger or Heel)- Materials and procedure
Materials: Lancet (capillary vessels), Straight needle.
Procedure: Clean site with spirit, Sharp prick.

Intraosseous access: Indications: Rapid cardiovascular access

243
Compiled by Obasi. D. C. Chinedu

Contraindications: Fracture of the target bone. Osteogenesis imperfecta


Complications: Osteomyelitis, Extravasation >> compartment syndrome/necrosis, Growth failure
Requirements: Cleansing lotion e.g. chlorhexidine /hibitane - Intraosseous needle with trochar
- Lidocaine (local anaesthetic) - 5ml syringe - 10ml syringe - sterile dressing pack
- IV fluids to be administered - gauze and securing tape

Procedure: The skin is cleaned and a small amount of Lidocaine is injected into the skin with a small hypodermic needle. This local
anesthetic is allowed to continue to infiltrate down into the periosteum of the bone.
With a scalpel make a 0.5cm cut in the skin 1 cm below and 1 cm medial to tibial tuberosity. The intraosseous needle is inserted at 90
degrees to the skin. The needle is advanced until a "give" is felt. This occurs when the needle penetrates the cortex of the bone.
The trochar is removed at this point. Position is confirmed by aspirating blood using the 5 ml. syringe. The needle is secured with a sterile
gauze and a tape. Fluid boluses are given with the syringes by gentle pushes.

Practice points: - Temporary procedure - Blood can be aspirated for sampling e.g. blood culture.
- Maintain as an iv access but fluid should be given by bolus.
Medications can be given IO (Beware of extravasation)

Lumbar Puncture: Indications


Diagnostic - CSF collection for analysis (Infections – Meningitis, Malignancy- staging of Burkitts lymphoma, CNS diseases – Guillaine – Barre
Syndrome)
Therapeutic - Infusion of drugs (Steroids, Antibiotics, Cytotoxic drugs, Spinal anaesthesia)

Contraindications
Absolute: CNS herniation, Acute neurological abnormality, Cardiopulmonary instability (CCF)
Relative: Suspected local mass lesion (spinal dysraphism, rash, ulcers), Bleeding disorders (platelet < 50000)

Procedure:
Instruments
Aseptic Procedure: Materials (Alcohol swab, gloves and gowns)
Lumbar puncture needle: (< 8 years; 22G short needle, > 8yrs; adult size needle).
Manometer tube (2/3 way tap). Containers
Anesthesia: Local (Topical, Injectable).
Position: Lateral or sitting position (site L3/4 or L4/5). Backed arched to extreme lordosis. Using a line from one anterior superior iliac spine
to the other as land mark, locate L4. Place the spinal needle between inter-vertebral spaces L3 and L4. Angle the needle @ 15 to 30 degrees
aiming for the umbilicus. The needle is on the horizontal plane with the back. Advance the needle until there is a give. If there is a flow allow
the fluid to flow up the tubing into the manometer (to measure opening pressure). Collect CSF for analysis.
Important issues
Site children: L3/4
Needles < 8 years use 22G: adolescents; adult size needle (Dura distance: 10kg 1.5mm/kg. 10-40kg 1.0mm/kg)
No flow: Check needle position (angle).
Bloody tap: Position of needle
Slow flow of CSF: Babies flow slow, Rotate needle to align bevel with flow.
Complications:
- Localized back pain: Minor or Severe (epidural spinal hematoma) - Infections: Meningitis
- Headache: CSF leakage - Acquired epidural cord tumors - Coning
Complications

Thoracocentesis: Indications: - Drainage of pleural effusions. - Diagnostic evaluation of pleural effusion.


- Therapeutic removal of small pneumothorax
Contra-indications: - Skin infections at site e.g. Herpes zoster - Bleeding and use of anticoagulants
- Mechanical ventilation

Material: Straight needle, Butterfly needle, Cannula. Aseptic materials (Alcohol swab, mask, glove and gown)
Local anesthesia. Chest drain (Rigid drain and pigtail >> therapeutic).

Important points: Thoracocentesis will not treat the underlying cause

244
Compiled by Obasi. D. C. Chinedu

Pneumothorax: Emergency: insert the needle/ cannula at the 2nd intercostal space on the mid clavicular line
If the effusion is massive, Do not drain too fast. Hypovolaemia may result.

Procedure:
Patient positioning: Sitting upright with arms support on table or facing the mother. Lateral position with side of effusion lower.
Location of Effusion:
Manual Procedure: Ideal: 1-2cm below onset of dullness. 5-7 th intercostal space mid-scapular or posterior axillary line. Pneumothorax (2 nd
intercostal space mid clavicular). Chest X-Ray and ultrasound guidance

Removal of Pleural effusion for diagnosis: Anesthetize the area, Going over the rib (5-7th) located. Push the needle over the rib advancing
gradually. Apply negative pressure in the syringe while advancing. Once in the pleural space a “pop” will be felt and fluid will enter the
syringe as negative pressure is maintained. Remove effusion into syringe.
Remove the needle and apply bandage to area

Complications
Minor: Pain, Cough, Localized infection.
Major: Pneumothorax, Haemothorax, Intra abdominal organ injury (liver or spleen), Air embolism, Laceration of the lung.

Supra pubic aspiration: Indications: Inability to void on request and there is need for urine specimen for diagnosis or exclusion of UTI
Contraindications: - Bleeding diathesis, - Abdominal distension, - Massive organomegaly.

Equipment: Assistant (preferably not parent). Specimen jar for urine. 23G needle (25G for premature). 3ml or 5 ml syringe. Anesthesia
(Topical, Distraction, Sucrose). Volumetric ultrasound.
Procedure
Caution: Never undo nappy before specimen bottle is ready. Do SPA before venepuncture or lumbar puncture
Different procedures: With and without ultrasound guidance. 50% success without ultrasound
Procedure: Without ultrasound: History of voiding > 30 minutes prior. Pre hydration. Dull supra pubic percussion (ALL will increase
chance). Assistant to hold the infant supine with legs extended, Ready to catch urine. Clean the area with alcohol. Identify insertion points
(Midline, Lower abdominal creases).
Insert the needle perpendicular to skin and aspirate as needle is advanced. If no success withdraw to just underneath the skin and reenter
advancing away from the pelvis. Apply dressing afterwards
Complications:
Uncommon: Macroscopic hematuria, Bladder hematoma, Bladder hemorrhage, Intestinal perforation, Anaerobic bacteremia or abscess.

Umbilical Catheterization: Indications:


Immediate access for resuscitation of the newborn
Investigative procedure: Cardiac catheterization, Blood for analysis
Therapeutic: Infusion of drugs and IV fluids, Exchange blood transfusion.
Contraindications: Gastroschisis, Omphalocele.
Complications: Risk of necrotising enterocolitis, Thromboembolism, Accidental hemorrhage.

Practice points in procedure


UAC insertion: Fill the vessel with 0.9% saline. Length: Weight (kg) x 3 + 9cm
UVC insertion: Prepare patient, Position child (Secure the limbs with tapes in a “spread eagle” supine position).
Length: Estimate distance (from umbilical stump to shoulder or ½ the UAC distance)
Vein: Carefully clean the cord and surrounding skin, Place an umbilical tie at the base of umbilicus to control bleeding. Cut the cord
horizontally about 1-2 cm above the skin. Identify the vein, usually there are 3 vessels, the vein is the larger and the thinner. Grasp the sides
of the stump with artery forceps.
If needed use the tip of a forceps to dilate the lumen of the vessel. Insert the tube into the lumen in a cephalad direction up to the estimated
length. Aspirate the tube and if there is smooth blood flow secure the tube with a ligature.

Exchange blood transfusion: Indications


Infants: Removal of toxins, Bilirubin, Antibody toxins, Sepsis, Coagulopathies
Child: Increased hemoglobin level (polycythemia). Severe anemia with heart failure. Sickle cell anemia
Contraindications: Unstable hemodynamic state (Heart failure).

245
Compiled by Obasi. D. C. Chinedu

Complications:
Catheter related: Emboli, clot or air. Thrombosis. Local hemorrhage. Risk of NEC, infection
Hemodynamic: Bleeding
Metabolic: Hypoglycemia, Hypocalcaemia, Hyperkalemia, Hypothermia, Metabolic acidosis.

Techniques:
‘Push and pull’: Removal of aliquots of blood from UV and
replacement with equal amount of donor blood
Continuous: Removal of blood through umbilical arterial with
simultaneous infusion of blood through the vein

Blood requirements: Heparinized blood, CPD blood. O negative


blood, HIV, Hepatitis, CMV: negative. Fresh whole blood and PCV
55%.
Site: UV, UA, Femoral vein. Other peripheral veins
Equipment: Standard EBT tray, Catheters (5FG (smaller infants),
8 FG (bigger infants), Improvised, Feeding tubes). 3 way tap (With
fluid lines), Meter rule (Measurement of CVP), Containers. Syringes (2ml, 5ml, 10ml, 20ml). Other materials (Recording sheet, calcium
gluconate (10%), dextrose).
Procedure: Requires an assistant (nurse preferable). Sterile procedure: Venous or arterial access
Blood grouping & X-matching: Mother’s, baby’s, donor’s.
Calculate volume of blood & rate: Volume: 80-85mls per kg. Rate: total time 1-2hrs (10mls/2 minutes, 5ml/kg/ 3minutes).
Volume withdrawn: < 1500 gm - 5 cc, 1500 - 3000 gm - 10 cc, > 3000 gm - 15 cc. Decrease the volume if not well tolerated.
Monitoring:
Pre-transfusion: Blood: FBC, UEC, Bilirubin, Glucose
Vitals: Temperature, Pulse, Respiratory rate, BP
Transfusion: Cyclical & continuous
Cycles: Take out determined aliquot. Infuse about 0.5ml heparinized blood. Take the aliquots at the predetermined rate. Out from vein, Into
the effluent bag, From blood bag, Into the vein. Mix blood in bag at intervals.
Monitor at intervals: Temperature, pulse, respiration (15 minutes), Blood glucose (5 minutes), Blood calcium. Patient’s response and
physical status.
Calcium gluconate: 1ml of 10% at every 100ml, Hypocalcaemia.
Glucose: Hypoglycemia

At conclusion: Replace initial aliquot withdrawn – top up transfusion.


Post EBT monitor: Blood; FBC, UEC, Bilirubin, Glucose, Calcium
NPO: 3-6 hours post transfusion, Retain IV line with dextrose

246
Compiled by Obasi. D. C. Chinedu

Dr. Otetunde

RENAL FAILURE: renal failure refers to a state of sudden or progressive and irreversible or reversible loss of renal function. It
could be:
a) Acute Kidney Injury (AKI) - Chronic Renal Failure - AKI on CKD

ACUTE KIDNEY INJURY(AKI): Acute Kidney Injury is the sudden deterioration in renal function caused by injury to the kidneys. The
spectrum of injury varies from mild to severe and results in inability of the kidneys to adequately excrete nitrogenous wastes and maintain
fluid and electrolyte homeostasis.
Resulting in retention of nitrogenous wastes and biochemical derangement. This manifests as a rise in serum creatinine with or without
reduced urine output: Oliguria (urine Volume < 0.5ml /kg/h) or anuria(< 0.1ml/kg/d) is the most prominent feature. Although urine
output may be normal or only slightly reduced (non-oliguric ARF). eg. ARF associated with nephrotoxic drugs. Increase blood level
of urea and creatinine suggest the diagnosis in such case.
When it occurs in a pre-existing renal disease is called AKI on CKD.
AKI comprises of 1.5–2% of all paediatric in-pt at major hospitals. 20-40% of Paediatric intensive care unit.

The Acute Dialysis Quality Initiative (ADQI) group


The Acute Dialysis Quality Initiative (ADQI) group, proposed the RIFLE criteria for diagnosing and classifying AKI based on the decline
in renal function and/or the urine output of the patient. The decline in renal function is estimated by calculating the decrease in Glomerular
Filtration Rate (GFR) from a baseline.
RIFLE is an acronym used for the different grades of AKI. RIFLE stands for (R- Risk, I-Injury, F-Failure, L-Loss of kidney function, E-
End-stage kidney disease).
Schwartz formula (ml/min) = (eCCl); = k x Height (cm) / Serum Creatinine (mg/l) OR
= k x Height (cm) x 88.4 / Serum Creatinine (mmol/l)
Where k= 0.33 For low birth weight children. 0.45 For full term infants and children less than 1yr. 0.55 For children and adolescent girls, 0.7
For adolescent boys
Children with no known baseline serum creatinine are assumed to have an estimated eGFR of 100ml/min/1.73m2.
Any child with eGFR less than 35 ml/min/1.73m2 is considered to have pRIFLE – F Class

PAEDIATRIC RIFLE CRITERIA (pRIFLE)


CLASS Change in eCCL (by Schwartz) Urine Output

Risk (R) Decrease by 25% <0.5 ml/kg/hr for 8 hours

Injury (I) Decrease by 50% <0.5 ml/kg/hr for 16 hours

Failure (F) Decrease by 75% <0.3 ml/kg/hr for 24 hours


Or <35ml/1.73m2/min OR Anuria for > 12 hours

Loss (L) Failure> 4 weeks

End Stage Renal Disease Failure> 3 months


(E)

CAUSES OF AKI: it could be - pre-renal, - Intrinsic (renal) and - post-renal

PRE-RENAL: Pre-renal failure is renal insufficiency due to inadequate renal blood flow.
Hypovolemic pre-renal failure, if treated early, responds to a fluid challenge with resumption of normal urine output and resolution of
azotemia.

CAUSES OF PRE- RENAL AKI


- Acute gastroenteritis with dehydration and shock is a frequent cause of AKI in the part of the world
- Burns. - Blood loss - Shock - Fulminant hepatitis
- Congestive heart failure - Hepato-renal syndrome - Reye syndrome

CAUSES OF INTRINSIC: Intrinsic Renal Disease. Glomerular


1) Acute GN (APSGN) 2) Cresentic GN (Rapidly Progressive GN) 3) Acute tubulointerstitial Nephritis
4) Acute tubular Necrosis 5) other Prolonged pre–renal insult 6) Intravascular haemolysis (G6PD)
7) Sepsis with multiorgan failure, 8) Nephrotoxic agent, 9) Snake bite, other envenomation,
10) P faciparum malaria, 11) leptospirosis
CAUSES OF POST RENAL: Post-renal ARF occurs from obstruction in the urinary collecting system in the absence of renal
247
Compiled by Obasi. D. C. Chinedu

parenchymal disease. It is potentially reversible when the urinary tract obstruction is alleviated.
NB: Both pre and post renal condition can, if prolonged, lead to parenchymal injury to the kidneys (intrinsic)

CAUSES: - Bilateral obstruction at pelvi ureteric junction, - Both ureteric Obstruction,


- bladder outlet and urethral Obstruction by calculi, blood clots and pus debris .

INDICES FOR DIFFERENTIATING PRE-RENAL FROM INTRINSIC


Pre-renal Intrinsic
Urinary Na MEq/l < 20 > 40
Urinary Osmolality (mosm/kg) > 500 < 300
Blood Urea/cr. ratio > 20:1 < 20:1
Urine to plasma osmolality > 1.5 < 0.8 – 1.2
Fractional excretion of Na <1 >1

FeNa% = Urine Na x Serum Cr x 100


Serum Na x Urine Cr

CLINICAL FEATURE
Symptoms: 1) Oliguria 2) Generalized weakness and dizziness
3) Ureamic symptoms- Anorexia, nausea, vomiting, pruritus, drowsiness, apathy, hiccup, convulsion, coma
4) There may be epigastic pain, body swelling

SIGNS: - Pallor - Dehydration - Tachypnoea - Hypotension/hypertension


- Mental clouding and confusion - Asterixis - Pericardial rub

MANAGEMENT
A) Urine Tests: Urinalysis with SG, urine m/c/s. also Urine Osmolality, Na, Fractional Excretion of Na
B) Blood Test: Serum e/u/cr, FBC, Blood culture, Blood gases, Peripheral Blood smear feature of micro angiopathy haemolysis,
thrombocytopenia,and reticulocytosis indicate HUS. Other test like ASO and C3 level as indicated.
(C) Radiology - Plan abdominal x-ray, - CXR, - Abd USS - CT scan - IVU
- Pylograph, renal arterograph and venograph.
(D) OTHERS: - Electrocardigraphy (ECG) - Immunlogical study - Antinuclear antibody
- Anti neutrophil cytoplasma antibody
- Anti streptolysin o ASO titer

TX: Removal and treatment of offending agents (causes)


- History evaluation - Treatment of infection - Blood transfusion
- Fluid replacement -Treatment of Hypertension - Correction of renal artery stenosis
- Surgical relief of obstructive uropathy

MX of Common Conditions causing AKI


(1) Pre-Renal ARF: Administer crystalloid, Colloid infusion. STOP diuretic, NASIDS, ACE inhibition. Inotropic for (cardiac failure).
Pressor agents, and antibiotics for sepsis.
(2) Acute Tubular Necrosis: supporting care, discontinue drugs or toxin, treat cause of circulatory failure.
(3) GN: supporting care if post infectious, antibody if associating with shunt infection or endocarditis, consider immune suppressive
medication.
(4) Hemolytic Ureamic Syndrome: supporting care, limited role for plasma infusion or plasma exchange
(5) Vasculitis: Immuno suppressive medication.
(6) Acute interstitial Nephritis: Discontinue offending drugs and consider steroid therapy
(7) Renal artery or venous occlusion: Anticoagulant, consider thrombolysis or surgical Arterial occlusion.
(8) Intrarenal Obstruction-Rx: Discontinue offending drugs, Alkalin diuresis for rhabdomyolasis, haemoglobinuria, or urate Nephropathy
(9) Urinary Tract Obstruction -Bladder catheter or nephrostomy. Radiologic/surgical treatment of obstruction

SUPPORTIVE OR NON PHARMACOLOGICAL TREATMENT


(1) Dietary advice: ↓Protein intake - Salt intake restriction. ↑↑ Caloric intake- Dietary Vitamin D supplement
- Avoid food and drinks with high K+.
(2) Daily weighing of pt
(3) Daily Serum E/U/Cr
(4) Fluid intake restriction with fluid input and output chart, strict in the olguric phase

PHARMACOLOGICAL TREATMENT
(1) Diuretics- If urine output is less than 0.5ml/kg/hr in pre-renal after two hours and there is no sign of intravascular volume deficit , 2–
3mg/kg of frusemide is administration. If no urine output occur in the following hour, intrinsic renal failure is strong suspected.
248
Compiled by Obasi. D. C. Chinedu

(2) Dopamine Infusion, in low dosage 1-3micg/kg/min/ cause renal vasodilation and increase renal perfusion and has been used in pt with
incipient ARF, not routinely recommendation. AM
(3) Others- H2 blocker in stress gastritis. Anti BP drugs if HBP. Antibiotics. Heamatinics – in blood loss

COMPLICATION: (1) Fluid Overload -Rx Fluid restriction


(2) Pulmonary oedema /pericardial effusion: Oxygen, dopamine 5–10mg/kg/min with Minimum fluid, frusemide 2 – 4mg/kg and monitor
by CVP line, dialysis with hypertonic glucose.
(3) Hypertension: symptomatic - Nitropruside, 0.5 – 8mg/kg/min infusion , frusemide 2-4mg/kg
Asymptomatic– Amlodipine 0.3-0.5mg/kg oral. Maintenance therapy with oral-Amlodipine, hydralizin or atenolol.
(4) Metabolic Acidosis
(5) Hyperkalaemia: Acute emergency-Rx: Calcium gluconate 10% 0.5-1ml/kg (5 – 10 mins) OR Salbutamol 5 – 10mg nebulizer
less emergency-a) NaCo3 (8.4%) 1–2ml/kg (5-20mins) (b) Glucose 50%, 0.5 – 1/kg with 0.1 – 0.2IU/kg insulin
(c) Ca resonium 1g/kg/day (d) Heamodialysis
(6) Hyponatremia: Fluid restriction, if sensorial alteration or seizure Rx- 3% saline 6-12ml/kg over 30-90 mins
(7) Anaemia: pack red cell 10ml/kg, intradialytically or consider EBT
(8) High serum phosphate: phosphate binders (calcium carbonate, acetate and Dietary Phosphate restriction)

INDICATION FOR DIALYSIS IN ARF


(1) Ureamia – Altered senserum, abnormal behavior, seizure, nausea, pericarditis.
(2) Hyperkalaemia: K > 6.5 m Eq/L , K 5.5 – 6.5 m Eq/L with ECG change
(3) Hyponatrema: Na < 120 – 125 meq/l esp if symptomatic
(4) Metabolic acidosis: PH < 7.2 despite NaHCO3 therapy, or NaHCO3 administration not feasible because of fluid overload.
(5) Hypercatabolic state – marked tissue injury, crush-syndromes, burns, sepsis, tumor lysis syndrome.

PROGNOSIS/FOLLOW: Urinalysis – proteinuria - Persistence Hypertension


- 3 Months check, 6 months, l year as the case may be
- Abd U/S - Serum e/cr/u - Other – infection

CHRONIC KIDNEY DISEASE (CKD). CHRONIC RENAL FAILURE


Chronic (CKD) is defined as irreversible deterioration of renal function, which gradually progress to end stage renal disease ESRD. It is
characterized by a fall in glomerular filtration rate GFR below 80mls/min/1.75m3 (100ml) for over 3 months and often accompanied
by other biochemical abnormality.
In Nigeria, about 2.5 children per million of the population develop CKD per years. There are studies suggesting an increase in the incidence
of CRF in Nigeria. Early diagnosis and management of some cases of chronic kidney disease may prevent or retard the progression of the
disease into ESRF.
The term chronic renal failure and End stage renal disease denote advanced renal damage for which renal replacement is the only
option.
Chronic Kidney diseases are condition that has potential to cause progressive loss of kidney function and eventually lead to ESKD. All
patients with CRF have CRD, but not all of those who have CRD are in CRF.
CKD is used to describe patient with kidney damage or decreased level of renal function for three months or more irrespective of the
underlying condition, and is defined as the presence of kidney damage or GRF below 60ml/min/1.73m2

STAGE OF CHRONIC KIDNEY DISEASE


This is based on the level of GFR, which is estimated from the level of serum Cr and height using the Schwartz formular. The classification
was proposed by the National Kidney Foundation Disease Outcome Quality initiative
eGFR = k x Ht/serum creatinine.
where eGFR is the estimated glomerular filtration rate expressed in ml/min/1.73m2
K is constant as follows; 0.33 for low birth weight infants less than 1 year, 0.45 for term infants less than 1 year, 0.55 for children and
adolescent girls, and 0.7 for adolescent males.
The constant is determined by comparing the length/serum creatinine ratio against measured glomerular filtration rate. Height or length of
the child is measured in centimeters while serum creatinine measured in steady state is expressed in milligrams per deciliter (mg/dl).

Classification of Stage
Stage GFR (ml/min/1.73m2) Description

1 90 Kidney damage with normal GFR

2 60-89 Kidney damage with wild reduction of GFR

3 30-59 Moderate reduction of GFR

4 15-29 Severe reduction of GFR

249
Compiled by Obasi. D. C. Chinedu

5 <15 END STAGE (DIALYSIS)

RISK FACTORS FOR CKD


- VUR associated with recurrent urinary infection and renal scaring - Obstructive uropathy
- Prior history of acute Nephritis or Nephrotic syndrome - Children with history of AKI in pre natal period
- Family history of poly cystic kidney or genetic renal condition - Renal dysplasia or hypoplasia
- Low birth weight infant - Prior history of Henoch Schonlein purpura
- Presence of diabetes, hypertension - Systemic lupus Erythematosis and vasculitis

CAUSES OF CKD:
- Chronic Glomerulonephropathies constitutes the most cause of CKD in childhood, especially in the older child.
- Chronic pyelonephritis, associated with severe urinary tract malfunction like severe vesico-uretric reflux and obstruction uropathies such as
posterior urethral valves in boys are significant cause of CRF.

CAUSES OF CRF
- Renal dysplasia is often suggested by concomitant presence of other congenital abnormalities.
- Other renal cause of CRF include haemolytic ureamic syndrome, congenital single kidney, bilateral renal hypoplasia, bilateral wilm’s tumor
and cystinosis.

Clinical features of CKD: Advance Renal failure develops when the GFR is well below 30% of normal. At this point, the kidneys cannot
regulate volume and solute composition.
End-Stage Renal Disease (ESRD): ESRD occurs when the GFR is below 15% of normal. The hallmark of renal pathology at this stage is
the marked reduction of healthy nephrons with a tremendous number of scarred ones together with a remarkable reduction in renal
capillaries. Atrophy and fibrosis of the renal tubules with a total reduction in the mass of the kidneys.

SYSTEMIC MANIFESTATION: The clinical feature of CKD include: -anorexia, -failure to thrive, -growth retardation, -fatigability, -
Anaemia, -hypertension and osteodystrophy.
The late feature, with extreme reduction of GFR, are; - itching, -severe acidosis, -hyperkalaemia,
- left venticular failure, - pulmonary oedema, -peri–cardial effusion, - altered sensorum.
The possibility of Late stage of CKD should be considered when one or more of these manifestation are present without an obvious
cause.

Other manifestation include; - lack of energy, - increase sleep, - poor school performance, - platelet dysfunction
- depressed cell mediated immunity, - Gastric ulceration, - severe itching.

PATHOHYSIOLOGY AND METABOLIC DISTURBANCE


In CRF there is derangement of the function of the body resulting in;
- acid-base disturbance, - disturbance of salt and water metabolism, - various metabolism bone disorder
- Anaemia - central nervous system, - Cardiovascular, - Endocrine,
- gastrointestinal and Immunological disturbances as well as disturbances of nitrogen, carbohydrate and lipid metabolism

PATHOHYSIOLOGY AND METABOLIC DISTURBANCE


1) Anaemia: Normchromic,normocytic aneamia occur in CRF and is due primarily to reduction in the synthesis of erythropoietin by the
diseased kidney.
Other causes of anaemia are; Haemolysis due to uremia. Chronic blood loss from bleeding the gastrointestinal tract, and skin and mucosa
membrane as result of bleeding disorder especially abnormal platelet homeostasis
2) Gastrointestinal: Acute gastrointestinal ulceration, peptic ulcer disease, idiopathy ascitis and petechial haemorrhage from the gut, have
been described with uraemic syndrome.
3) Endocrine: Abnormal metabolism of the sex hormones with absence of the pubertal growth spurt, delayed menarche and puberty with
absent of secondary sexual characteristics has been frequently described in CRF. There also, is low level of serum thyroxine and abnormally
elevation fasting growth hormone level, which explain the glucose intolerance in uraemia.
4) Acid-Base Disturbance: A drop in GFR to about 20 – 30 ml/min/1.73 result in retention of fixed acid and a state of metabolism acidosis
occur with increase in urinary excretion of calcium. There is depression of serum bicarbonate reabsorption in the proximal tubule, with a
small percentage of the filtered load reabsorbed in the distal portion of the nephron. Failure to reabsorbed the filtered bicarbonate completely
lead to urinary loss of this buffer and elevated urinary PH and limitation in the ability of the kidney to execute net acid and result in
metabolism acidosis.
5) Cardiovascular: Mechanism of uraemic heart disease include fluid overload from salt and water retention, system arterial hypertension .
other are anaemia, uraemia, electrolytes derangement ↑K, ↓Na, ↓Ca , ↑Mg.
Peri carditis with or without peri cardial effusion is frequently observed in patient with CRF and acute left ventricular failure and pulmonary
oedema is a well known presenting feature in children with CRF. Majority of children with CRF has abnormal ECG finding &
cardiovascular complication is cause of death on majority of cases.
6) Neurological complication: This is called Uraemic Encephalopathy and consist of various neurological pretention.

250
Compiled by Obasi. D. C. Chinedu

Non-specific general depression of cerebral function, fatigue, listlessness, drowsiness, poor concentration at school, to frank psychosis,
hallucination, seizure and coma with decorticate posture.
Focal neurological sign may occur but are rare; more often being associated with hypertension encephalopathy or intracranial haemorrhage.
The exact pathophysiology of this cerebral dysfunction is unknown.
The degree of dysfunction is not commensurate with level of urea and Cr in the blood probably other biochemical metabolism in the blood
are responsible.
Peripheral neuropathy is a common manifestation of CRF in adult with restless leg syndrome, characterized by peculiar creeping, prickling
and tingling sensation in the lower limb. Often worse in the night, other symptom is burning feet syndrome. Full blown peripheral
neuropathy may subsequent develop with loss of vibration position, touch and pain sensation and loss of deep tender reflex.
In children, frank peripheral neuropathy is rare but defect in nerve condition velocity are common.
7) Immunology: Suppression of normal immune mechanical form a part of the uraemic syndrome. Various degree of lymphopenia, delayed
hypersensitivity reaction, abnormal polymorphonuclear, and monocyte function as well abnormal inflammably responses have been
described.
8) Renal osteodystrophy
MANAGEMENT: 1) Serum e/u/cr: Ur level is serum generally above 6mg% while serum Cr is also 100micmol/di. If serum Ur is
abnormally high for an elevation serum Cr due to inability to eat.
2) Serum HCO3 is often low 3) Calculation of the eGFR
4) Serum calcium may be low, normal or high

Other Investigation: - CXR – cardiomegaly - MCUG - IVU


- X-ray of Hand and wrist will show the earliest radiological changes. If present, will subperiosteal erosion of the radial aspect of the middle
phalange.
- Total skeletal survey is unnecessary in detecting Ros.

Treatment:
Conservative Management: When GFR is above 15ml/min/1.73. Occasional when patient are receiving RRT.
 Oliguric or anuric pt should receive fluid equal to their insensible loss plus the urine out put in 24hrs to prevent fluid overload, i.e. fluid
restoration.
 If serum bicarbonate is 15mm/L or less, sodium bicarbonate may be given in a dose of 1ml/kg/day
 Avoid excessive salt intake with resultant fluid overload
 1.25 dihydoxy cholecalciferol or the synthezised ,1ά hydoxy – cholecalciferol In dose of 10 – 15mg/kg/day
 Anti HBP, particularly β blocker (if there is no cardiac failure). Hydralazin – control HBP. ACEI useful in renal Hypertension.
Phosphate binder e.g. calcium carbonate.
 Low protein diet to prevent hyperfiltration and have been used to delay the onset of end stage renal esp in adult but in children, taking
into consideration the growth most children will require 2g/kg/d of high molecular value.
 The use of Recombinat human erythropoietin therapy has markedly diminished the need to frequent blood transfusion in children with
CRF in whom often iron therapy does not correct the anaemia.
 Avoidance of drugs, there are nephrotoxic e.g. genticin

Renal Replacement Therapy: When ESRF set in, renal replacement therapy must be given to prevent impeding death. This should be done
when GFR is less than 10ml/min/1.75.
Dialysis: - PD - HAEMODIALYSIS
Renal Transplantation

Dialysis: Chronic Ambulatory Peritoneal Dialysis (CAPD), which is prefer by most centres and can be done in very small children aged less
than 2 years in whom renal transplantation is difficulty
Advantage: Better growth rates less restriction diet and fluid allowance, freedom to attend activities, less requirement for blood transfusion.
Other peritoneal dialysis
Automated PD: - Continuous Cycling Peritoneal Dialysis (CCPD) - Intermittent Peritoneal

Heamodialysis: especially in abdominal surgery. Required either a creation of an artero – venus shunt or a fistula or vascular access. This is
difficulty, if the child is young with small vessel

Renal Transplantation
Renal Transplantation is the goal to most children and cadaver or living donor can be used. Immunosuppressive therapy such as MMF,
Azathioprine, Prednisolone or cyclosporin A, Tacrolimus For life is mandatory to prevent host grafting rejection.

251
Compiled by Obasi. D. C. Chinedu

RENAL REPLACEMENT THERAPY(RRT) IN CHILDREN


By PROF H. U OKAFOR

Learning Objectives: - Definition of RRT - Types of RRT - Indications for RRT - Complications
- Advantages /Disadvantages
Outline: - Definition - Introduction - Principles of Dialysis - Types of Dialysis - Renal transplantation

Pre-Test
1. what are the various forms of RRT in children. 2. What is the preferred mode of RRT in children
3. What are the basic physiological principles of hemofiltration. 4. Name 2 advantages of Peritoneal dialysis in children.
5. Name 2 major complications of renal transplantation in children. 6. Give 2 indications for RRT in children.

Definition
Renal replacement therapy is a term used for life-supporting treatments of patients with renal failure. Replaces the functional activities of the
kidney. It can be applied intermittently or continuously; extracorporeal or Para-corporeal.
Renal transplantation is the ultimate form of RRT
In Chronic Renal Failure (CRF), the dialytic forms of these treatments do not cure renal failure; they are used as palliative treatments. But
early dialysis in acute renal failure may have favorable outcomes and bring resolution of renal failure.

Definition: Renal Transplantation


Renal Transplantation is the ultimate form of Renal Replacement Therapy (RRT). It is a surgical procedure to place a healthy kidney from a
live or deceased donor into a person with End stage Renal Disease (ESRD).

Introduction
Renal replacement therapy (RRT) supports the kidneys or replaces the non-endocrine functions of the kidneys in patients with renal failure. It
could also be applied occasionally in some forms of poisoning as well as other conditions where the function of the kidneys maybe
overlabored. It could be dialytic or non-dialytic.
Techniques of the dialytic form of RRT include: - hemodialysis - hemofiltration
- Combination of hemofiltration and hemodialysis - peritoneal dialysis.
All modalities exchange solute and remove excess fluid from the blood, using dialysis and filtration across semi-permeable membranes.

History of Dialysis: Dr John Abel & Dr WJ Kolff –vivi-diffusion 1913 in animals. In 1950s, it was put to clinical use. Modern apparatus
was eventually developed by Dr Alwall . 1923 Dr Tracy Putnam PD in cats. 1940 clinical use by Dr Jacob Fine -4 cases. Quinton catheter
1964; Tenckhoff 1968 catheter.
CRRT Dr Scribner in 1960s(CAVH);CVVHD in adults Dr Cannaud 1988,in children 1990 Dr Yorgin .

Principles of Dialysis: Serum solute (eg, Na, Cl, K, HCO3, Ca, Mg, phosphate, urea, creatinine, uric acid) diffuses passively between fluid
compartments across semi-permeable membrane, down a concentration gradient (diffusive transport) dragging water along.
Principles of Hemofiltration: Serum water passes between compartments down a hydrostatic pressure gradient, dragging solute with it
(convective transport). The two processes are often used in combination (hemodiafiltration).

METHODS: Dialysis and filtration can be performed intermittently or continuously.


Continuous therapy is used exclusively for acute renal failure especially in critically ill children; benefits over intermittent therapy are
improved tolerability as a result of slower removal of solute and water.
All forms of RRT except peritoneal dialysis require vascular access; continuous techniques require a direct arterio-venous or veno-
venous circuit.

Peritoneal dialysis: The peritoneum acts as a semi-permeable membrane facilitating concentration driven movement of water and small
molecular solutes on either side of the membrane. (Diffusion).
Haemodialysis: Blood passes through artificial cellophane membranes which allow passage of water and solutes.

Technique of PD: Acute dialysis is often begun with 2.5% solution. Dialysate must be warmed before infusion.
K+(2-4meq/l) is eventually added to the dialysate when serum potassium levels normalize. Antibiotics are either given parenterally or added
to the dialysate. Gentamycin or cephalosporin are used for this purpose.

Advantages of PD: Although each type of dialysis can be utilized for both acute and chronic conditions, peritoneal dialysis is preferred
for acute conditions depending on the availability of facilities and state of patient.
This is because it is easily available and requires less expertise.
Haemodialysis is better for chronic conditions.
Chronic haemodialysis and indeed continuous forms of peritoneal dialysis are reserved for children with end stage renal failure.

General Indications for RRT (Dialysis): - Acute renal failure with azotemia - Chronic renal failure with uremia GFR<15ml/min.

252
Compiled by Obasi. D. C. Chinedu

- Severe Metabolic acidosis - Accidental poisoning - Intractable cardiac failure - Severe hyperkalemia.
- Dyselectrolytemia - Hepatic coma - Reye’s syndrome
- Respiratory distress syndrome - Fluid overload - Severe sepsis

Dialysis Setup

The commonest indications for dialysis in children:


- Acute renal failure - Accidental poisoning - Chronic renal failure
- Fluid overload and electrolyte/acid base abnormalities in critically ill patients.

Indications for HD: Renal insufficiency or failure (acute or chronic) with any of the following that cannot otherwise be controlled:
- Fluid overload - Hyperkalemia - Hypercalcemia - Metabolic acidosis
- Pericarditis - Uremic symptoms
- GFR < 15 mL/min/1.73 m2 BSA (ESRD or CKD 5) - Poisoning
CONTRAINDICATION: Uncooperative or hemodynamically unstable patient

Indications for Peritoneal dialysis: Same indications as for hemodialysis (except for poisonings) in patients who:
- Have inadequate vascular access - Young children - Prefer self-therapy

Contraindications To Peritoneal Dialysis


Absolute: - Loss of peritoneal function or adhesions that limit dialysate flow.
- Recent abdominal wounds, - Abdominal fistulas, - Patient's condition not amenable to dialysis
- Abdominal wall defects that prevent effective dialysis or increase infection risk (eg, irreparable inguinal or diaphragmatic hernia, bladder
extrophy),

Contraindications to Peritoneal Dialysis


Relative: - Abdominal wall infection - Frequent episodes of diverticulitis - Ischemic colitis - Morbid obesity
- Inability to tolerate large volumes of peritoneal dialysate, inflammatory bowel disease - Peritoneal leaks

OTHER CONTRAINDICATIONS TO PD: - Severe undernutrition - Abdominal surgery


- Peritonitis - Abdominal malignancies - Omphalocele - Gastrochisis - Diaphragmatic hernia
Choice of Techniques: The choice of technique depends on multiple factors; This includes:
- The primary need (e.g, solute or water removal or both).
- Underlying indication (e.g, acute or chronic failure, poisoning). - Vascular access.
- Hemodynamic stability. - Availability. - Local expertise. - And patient preference.

Technique of Peritoneal Dialysis: Peritoneal dialysis is based on two interrelated transport mechanisms:
Diffusion and convection.
Diffusion refers to movement of solutes down an electro-chemical concentration gradient while convection refers to movement of solutes
along a fluid flux. The peritoneal membrane functions as a dialyzing membrane and the surface area in the child is larger than in
adults (522cm2/kg versus 285cm2/kg).

Peritoneal Dialysis Fluids: The peritoneal dialysis fluids can either be Isotonic or hypertonic and the concentration of dextrose which ranges
from 2-4% determines the tonicity of the fluids. The fluids do not contain potassium, urea and creatinine while other electrolytes like sodium
are found in the same concentration as in plasma.
Hence in acute renal failure with azotemia the retained nitrogenous waste products and potassium, diffuse out of the blood vessels into the
dialysate in the peritoneal cavity.

Peritoneal Dialysis: modes of delivery: There are various modes of peritoneal dialysis namely intermittent / continuous;
automated/manual
 Continuous ambulatory peritoneal dialysis (CAPD).
 Intermittent peritoneal dialysis which could be short-term or long term.
 Continuous cyclical peritoneal dialysis (CCPD) whereby an automated device is employed for fluid exchanges.

253
Compiled by Obasi. D. C. Chinedu

Complications of Peritoneal dialysis.


 Peritonitis due to both gram positive (50%) and gram negative organisms (20%) while fungal infections constitute <2%.
 Other complications of ESRD like anemia and growth retardation are not peculiar to PD patients. However, many children receiving
CPD have been observed to have progressive renal osteodystrophy despite treatment with various Vitamin D preparations.

Hemofiltration vs Hemodialysis: Like hemodialysis, in hemofiltration one achieves movement of solutes across a semi-permeable
membrane. However, solute movement with hemofiltration is governed by convection rather than by diffusion. With isolated
hemofiltration, dialysate is not used. Instead, a positive hydrostatic pressure drives water and solutes across the filter membrane from the
blood compartment to the filtrate compartment, from which it is drained. Solutes, both small and large, get dragged through the membrane at
a similar rate by the flow of water that has been engineered by the hydrostatic pressure.
So convection overcomes the reduced removal rate of larger solutes (due to their slow speed of diffusion) seen in hemodialysis. Can be
combined with hemodialysis in which case it is termed hemodiafiltration.
Can be continuous as in ICU settings or intermittent.

Continous Renal Replacement Therapy (CRRT)


Hemofiltration is most commonly used in an intensive care unit setting, where it is either given as 8-12 hours treatments, so called SLEF
(slow extended hemofiltration) or as CHF (continuous hemofiltration also sometimes called continuous veno-venous hemofiltration
(CVVH)) or Continuous Renal Replacement Therapy (CRRT).
Hemodiafiltration (SLED-F or CHDF or CVVHDF) also is widely used in this fashion. Continuous hemofiltration (CHF) or
hemodiafiltration (CHDF).

Hemodialysis
Technique:
Vascular access: This is usually gained using special paediatric arterio-venous cannula which may have single or double lumen. These are
inserted into the radial artery or a forearm vein in older children; brachial artery /cephalic vein in younger children, femoral artery
and long saphenous vein in children weighing less than 10kg. The subclavian and jugular veins may be used for both temporary and
permanent access.
Arterio-venous fistulae may be created for chronic dialysis but these are associated with complications like cardiac overload and limb
hypertrophy.

Complications of Hemodialysis
a) Seizures which are attributed to dialysis disequilibrium syndrome and results from cerebral edema caused by generation of idiogenic
osmoles in the brain due to rapid lowering of blood urea nitrogen(BUN).
b) Hemodynamic complications are the most common especially hypertension followed by hypotension.
c) Gastrointestinal symptoms like epigastric pain and hematemesis have been documented
d) Catheter dislodgement.
e) Arterial or venous thrombosis
f) Infections like hepatitis and retro-viral infections could occur, although some measures have been adopted in most centres to avert this.
This includes screening patients and isolating machines used for patients with positive results. The incidence of hepatitis in children on
dialysis is about 10% which is similar to that in adult patients.
g) Diet related complications: indiscretions in dietary intake can predispose to volume overload, hyperkalemia hypertension and
pulmonary oedema. These may be prevented by strict adherence to dietary restriction.
h) Psychosocial problems arise both with the family and the patient. For the family and relations this is mainly due to the financial and
physical stress of looking after the patient. For the patient it ranges from depression to behavioural disorders associated with chronic
illness and the fact that they seem different from others with limited activity. So it is usually desirable to consult a child psychologist
who should be a member of the management team. However, patients are encouraged to live a near normal life as possible.

Renal Transplantation: This has generally been regarded as the preferred mode of treatment in children with end stage renal disease
because it affords the child the opportunity of living a near normal life. Despite this, a large proportion of children require chronic dialysis
either before or in between transplants.
In North America paediatric transplants account for more than 80% of all transplants.

Indications for Renal transplantation


All children with ESRD are potential candidates for a renal transplant although pre-emptive transplant is a modality reserved for children
only. In this case the patients do not undergo a period of chronic dialysis before the transplant.
Growth failure in patients with chronic renal insufficiency

Contraindications to Renal Tx
 HIV infection
 Pre-existing malignancy although there has been a report of successful transplants without recurrences in patients with wilms tumour.
 Children with severe neurological dysfunction
 Potential recurrence of primary disease e.g oxalosis, cystinosis, focal segmental glomerulosclerosis

254
Compiled by Obasi. D. C. Chinedu

Pre-transplant preparation: Prior to renal transplantation, problems caused by chronic renal failure should be addressed e.g malnutrition,
anemia etc as well as correction of urinary tract anomalies which may have caused an obstructive uropathy.
A Good history and physical examination are mandatory. Investigations include:
- complete blood count (cbc) - platelet count - clotting studies - serum urea and electrolytes
- liver function tests - lipid profile, - Serum calcium and phosphate levels,
- Parathyroid hormone levels - Urinalysis and microscopy culture and sensitivity,
- Blood grouping, - Human Leukocyte Antigen (HLA)typing, and cross match with donors
- Screening for panel reactive antibodies(PRA) - ABO grouping and crossmatching with donors specimen,
- Screening for hepatitis & HIV screening - Serological tests for CMV, EBV, VZV, and MMR titres.
- Radiological tests VCUG, CXR, bone age
- Dentist, social worker and nutritionist consult.
- Vaccination for: DPT, OPV, MMR, HIB, Pneumovax, Varivax.

Pre-Renal Transplant: There are two types of donors; cadaveric and live donors. About 50% of transplants in children are from live
donors. Live donors maybe either related or non-related to patients. In children most donors are live related & usually parents or siblings.
But in all circumstances the histocompatibility tests and anti-HLA antibody screening are done to reduce chances of rejection.

Immunosuppression: This is often necessary in order to reduce the chances of graft rejection. It is divided into two phases; induction and
maintenance.
Induction: This is commenced either before the transplantation or intra-operatively. Drugs used include steroids, Azathioprine,
cyclosporin and tacrolimus. The latter two drugs are quite toxic. The use of anti-lymphocyte antibodies and monoclonal antibodies directed
at subsets of lymphocytes(OKT3) have also been tried.
Maintenance: For maintenace cyclosporine, tacrolimus are the main drugs used while steroids, Mycophenolate mofetil (MMF) and
Azathioprine are used as adjuncts.
Due to the toxicity of these drugs serum levels are monitored regularly and viral infections are common complications of the resultant non-
specific immunosuppression.

Renal Transplant-Surgical technique: This depends on the weight of the child.


 For children <15kg the donor renal vein is anastomosed to the recipient vena cava while the renal artery goes to the aorta.
 For those >15kg the transplant is extraperitoneal like in the adult. The ureter is implanted to the recipients bladder and renal vein goes
to common iliac or external iliac while renal artery goes to common iliac artery or internal iliac artery.
The transplant kidney is placed in the iliac fossa or abdomen and the renal artery is anastomosed either to the internal iliac or hypogastric
artery and renal vein to hypogastric vein.
Ureteral implantation is by ureteroneocystostomy.
Kidneys from adult donors can be transplanted into young children and these are usually placed in the abdomen with anastomosis of renal
vessels to the aorta and inferior vena cava.

Complications of renal Tx
Immediate:
Graft dysfunction could be caused by acute tubular necrosis, graft thrombosis and obstruction and urinary leakage.
Graft rejection: This is diagnosed from clinical and biochemical evidence. Fever is the most common clinical finding suggesting acute
rejection associated with hypertension, increased size and tenderness of allograft. This is a serious complication and manifest in three forms
viz
Hyperacute: This occurs <24hrs after transplant.
Acute: This occurs <6months post transplant and is subdivided into: Incipient- biochemical derangement only
Moderate-biochemical and clinical Severe-requiring reinstitution of dialysis.
A significant decrease in urine output usually heralds or accompanies severe rejection.
Chronic- occurs >6months post transplant with slow deterioration of function with or without nephrotic syndrome.

Late: • Infections: viral and bacterial • Hypertension • Hyperlipidemia • Lymphoproliferative disorders


• Malignancies • Growth retardation which depends on bone age prior to transplant.

Case Study: The mother of a 7yr old male child admitted into wd.6B with a diagnosis of abdominal burkhitts Lymphoma reported that her
son made 150mls of urine in the past 24hrs.An urgent S/E/U/C was done and results are as follows:
Na-130mmol/L; HCO3-12mmol/L; K+-6mmol/L; Ca-2.2mmol/L;
Phosphate-7mmol/L; Urea -15mmol/L; Cr -985micrommole/L
- What will be your next line of action?

Post-Test
1. what are the various forms of RRT in children. 2. What is the preferred mode of RRT in children
3. What are the basic physiological principles of hemofiltration. 4. Name 2 advantages of Peritoneal dialysis in children.
5. Name 2 major complications of renal transplantation in children. 6. Give 2 indications for RRT in children.

255
Compiled by Obasi. D. C. Chinedu

NEONATAL INFECTIONS/SEPSIS
Prof Herbert A Obu, MBBS, FWACP (Paed)

OUTLINE
 Introduction/synonyms/definitions
 Epidemiology/risk factors/aetiologic agents
 Modes of transmission/pathogenesis/classification
 Clinical manifestations/diagnosis/differential diagnosis
 Treatment
 Prevention
 Prognosis

SYNONYMS: Neonatal septicaemia, Sepsis neonatorum, Neonatal sepsis syndrome.

Infections are important causes of morbidity and mortality in newborns worldwide. This is even more so in developing or resource poor
countries for a variety of reasons. Preterm delivery, increased period of stay in hospital, use of various devices, increase in number of
admissions, etc. further add to the increase in the burden of these conditions.

DEFINITIONS:
Neonate: newborn child from age 0–28 days (or within 1st month of life)
Infection: the invasion and multiplication of microorganisms such as bacteria, viruses, and parasites that are not normally present within the
body. It may cause no symptoms and be subclinical, or it may cause symptoms and be clinically apparent. may remain localized, or it may
spread through the blood or lymphatic vessels to become systemic.
Sepsis is a clinical syndrome of life-threatening organ dysfunction caused by a dysregulated response to infection.
In septic shock, there is critical reduction in tissue perfusion; acute failure of multiple organs, including the lungs, kidneys, and liver, can
occur.

Older definitions: Sepsis: infection in association with features of systemic inflammatory response syndrome (sirs)
Sepsis: clinical condition of bacteraemia with associated constitutional (or systemic) features. Now known that microbial agents other than
bacteria can elicit sirs and thus can cause sepsis. Note that sirs can arise from non-infective causes.
Bacteraemia: presence of bacteria in blood stream which may be transient

Features of SIRS: temp instability, resp dysfunction, cardiac dysfunction, perfusion abnormalities, increased capillary permeability, DIC,
etc. Multiple organ dysfunction may follow if tissue injury continues unchecked

EPIDEMIOLOGY: Causes 18-35% of neonatal deaths in Nigeria and other parts of Africa. 30% of neonatal admissions at the RVH in The
Gambia. Among 4 commonest causes of admissions in the NBSCU of UNTH in a recent review. Up to 10% neonates have infections within
1st month of life (US figures). About 2% of foetuses infected in utero.

RISK FACTORS: Maternal peripartum pyrexia due to chorioamnionitis, UTIs, other illnesses
- Prolonged rupture of the foetal membranes - Difficult, prolonged labour (and difficult delivery)
- Instrumental deliveries - Invasive procedures and excessive handling
- Prematurity/low birth weight - Congenital malformations
- Long hospital stays - Socioeconomic status
- Congenital immunodeficiency states - Aspiration syndromes

AETIOLOGIC AGENTS: Bacteria, viruses, protozoa, fungi, chlamydiae and rickettsiae have all been implicated.
Common bacterial agents in our environment include:
Gram positive organisms such as Staph. aureus, Staph. epidermidis, B-haemolytic strep group A, group B streptococci and Listeria
monocytogenes
Gram negatives: E. coli, Klebsiella species, Proteus spp. and Pseudomonas aeruginosa
MODES OF TRANSMISSION:
- Transplacental: TORCHS (or STARCH) infections - Vertical transmission throu infected amniotic fluid
- Direct infection as baby passes through infected birth canal
- Postnatal infections (Community acquired, Hospital acquired

PATHOGENESIS: Infectious agents involved vary from place to place and over time. Emergence of antibiotic resistance has further
complicated the management of neonatal sepsis. Bacterial colonization of personnel, equipments and devices in NICUs are associated with
nosocomial infection. infants with lower birth weight and infants who are less mature have an increased susceptibility to these organisms.
Intrinsic properties of some organisms eg. adherence to iv catheters and shunts, formation of biofilms, elaboration of toxins, etc. favour
infection by organisms.
256
Compiled by Obasi. D. C. Chinedu

Important host factors include impaired cellular immunity, humoral immunity, and barrier function of the newborn. The more premature the
baby is the more the immuno-compromise.
There may also be a genetic association.

CLASSIFICATION:
Early onset: age at onset from birth to 7 days; usually acquired from mother b/4 or during birth. Of newborns with early-onset sepsis,
85% present within 24 hours, 5% present at 24-48 hours, and a smaller percentage present within 48-72 hours. Onset is most rapid in
premature neonates.
Common agents: Group B Streptococcus (GBS), E.coli, CONS, H. influenzae, Listeria monocytogenes
Late-onset: age at onset after 7 days to 30days (4-90 days in some literature); usually acquired after delivery
acquired from the caregiving environment
Common organisms in late-onset: Coagulase-negative Staphylococcus, S.aureus, E.coli, Klebsiela, Pseudomonas, Enterobacter.

CLINICAL MANIFESTATIONS: Presentations are often subtle and non-specific; high index of suspicion needed to make an early
diagnosis and institute appropriate therapy. Observed features seen, esp. in bacterial sepsis, include: - Fever/hypothermia
- Refusal of feeds/poor feeding - Excessive crying/fretfulness
- Decreased activity/unusual quietness - Irritability - Poor skin colour/skin mottling
- FTT - Diarrhoea, vomiting, constipation, abd distension
- Grunting resp, apnoea, resp distress, cyanosis - Jiterriness, seizures, decreased prim reflexes
- Pallor, jaundice, bleeding - Others

Diagnosis: Clinical manifestations. BP, heart rate, and oxygen monitoring.


- CBC with differential, electrolyte panel and creatinine, lactate
- Invasive central venous pressure (CVP), PaO2, and central venous oxygen saturation (ScvO2) readings
- Cultures of blood, urine, and other potential sites of infection, including wounds in surgical patients
- Good history is very important. Thorough physical examination
Lab. Diagnosis:
- Complete blood counts, with diff, blood films, ESR, screen for malaria, etc.
- Absolute neutrophil count less than 1750/mm3
- Total WBC less than 7500/mm3 or more than 40,000/mm3
- I/T ratio more than 0.3
- CRP more than 1.5mg/dl
- Cultures (pan-culture) – blood, urine, CSF, umbilical swab, wound swab, ear swab, etc
- Other studies depending on symptomatology: platelets, PT, PTT, electrolytes, urea and creatinine, LFTs
- Viral studies, fungal studies, special studies

DIFFERENTIAL DIAGNOSIS: - Transient tachypnoea of the newborn - Meconium aspiration syndrome


- Intracranial haemorrhage - Cyanotic congenital heart disease - Perinatal asphyxia
- Respiratory distress syndrome - Bronchopulmonary dysplasia

Treatment (bacterial sepsis):


Specific treatment: Antibiotics: Empirical tx needed pending bacterial cultures and requires broad spectrum coverage. Common
combinations include: ampicillin/gentamicin, cloxacillin/gentamicin, cefuroxime/gentamicin
In the critically ill, third generation cephalosporins can be added or used in place of ampicillin/penicillins.
Duration of treatment b/w 10 &14 days; can be longer in some cases
Supportive/adjunct treatment: Transfusions: blood, EBT, fresh frozen plasma. Crystalloids for volume expansion
Pressor agents – dopamine, dobutamine for hypotension or hypoperfusion (or both)
Sodium bicarbonate for acidosis. Anticonvulsants for seizures
Adjunct immunotherapies: Intravenous immunoglobulins (IVIG). Treatment with granulocyte colony stimulating factor and granulocyte
macrophage stimulating factor. Recombinant cytokines.

PREVENTION/CONTROL:
General measures: socioeconomic well being, safe water, environmental sanitation, health education, antenatal/delivery services, etc.
Hospital measures: aseptic techniques for procedures, sterilize equipments, minimal handling of at-risk babies, separate linens & utensils
for babies, hand washing with soap and water, enough space for infants in the wards

Sepsis: key points


- Sepsis and septic shock are increasingly severe clinical syndromes of life-threatening organ dysfunction caused by a dysregulated
response to infection.
- An important component is critical reduction in tissue perfusion, which can lead to acute failure of multiple organs, including the lungs,
kidneys, and liver.
257
Compiled by Obasi. D. C. Chinedu

- Early recognition and treatment is the key to improved survival.


- Resuscitate with IV fluids and sometimes vasopressors titrated to optimize central venous oxygen saturation (ScvO2) and preload, and to
lower serum lactate levels.
- Control the source of infection by removing catheters, tubes, and infected and/or necrotic tissue and by draining abscesses.
- Give empiric broad-spectrum antibiotics directed at most likely organisms and switch quickly to more specific drugs based on culture
and sensitivity results.

NECROTISING ENTEROCOLITIS (NEC): Severe disease. Mainly affecting prem.


Predisposing factors include: asphyxia, sepsis, umbilical cathetrization, polycythaemia.
Paramount factors in pathogenesis: gut ischaemia, action of anaerobic bacteria, free radical injury
Organisms isolated from affected infants: E.coli, Pseudomonas, Klebsiella

Clinical features: features of sepsis above, abd distension, diminished bowel sounds, blood in stool, etc.
Xray features: thickened bowel walls, air in the bowel walls (pneumatosis intestinalis), portal gas.
Mgt; NPO, TPN, parenteral antibiotics, surgery
Mortality high, gut perforation is common

OTHER NEONATAL INFECTIONS: - Neonatal meningitis - Conjuctivitis (neonatal ophthalmia)


- Umbilical sepsis - Skin infections - Others

SKIN INFECTIONS: Most skin infections caused by Staph. aureus/Strept. Pyogenes. Most common mode of presentation are in form of
pustules/furuncles which may be solitary or in clusters. More severe forms are rarer and include bullous impetigo, staph. Scalded skin
syndrome, necrotising fascitis, etc
Solitary lesions require cleaning with antiseptic solutions only. More extensive lesions need systemic antibiotics.

OPHTHALMIA NEONATORUM: Sticky eyes common in the neonatal period; usually seen on day 3-4 and requires no more than
cleaning with saline or sterile water.
More troublesome discharge due strept/staph infection and may respond to topical antibiotics. Purulent discharge with swelling of eye lids
within the first 48 hours may be gonococcal in origin.
Requires prompt diagnosis (gram stain &culture) and immediate treatment to prevent eye loss
Drugs of choice are parenteral penicillin g or 3rd gen. cefalosporins if penicillin resistant.
Chlamydia trachomatis eye infn may present similarly but at 1-2wks of age; requires tx with oral erythromycin for 2 wks.

NEONATAL MENINGITIS: Common.


Organisms mainly gram neg organisms: Klebsiella, E.coli, Proteus spp., Pseudomonas aeruginosa.
Strept pneum. and Staph aureus are less common
Features akin to those of NNS; may include bulging fontanels, among others
LP must be done to obtain CSF for microbiologic studies
Treatment requires parenteral antibiotics, preferably iv, for 2 wks or more depending on the organism.

UMBILICAL INFECTION: Umbilicus usually dries and separates within 1st few days of life. The umbilical stump may become infected
with inflammation of the surrounding skin. Systemic antibiotics indicated in the treatment

HSV INFECTIONS: Uncommon. Occurs as an ascending infection or during passage through an infected canal.
Commoner in prems. Occurs first 4 wks of life.
May prsent with localised skin or eye lesions or with diseminated dse or encephalitis.
High mortality associated with diseminated disease while encephalilitis is associated with sign. Morbidity

258
Compiled by Obasi. D. C. Chinedu

DIABETIC KETOACIDOSIS
Dr. UGO CHIKANI

Outline:
- Epidemiology - Pathophysiology - Diagnosis
- TREATMENT - Complications - CHALLENGES - Solutions

Definition: Hyperglycemia (blood glucose >11 mmol/L [200 mg/dL]). Venous pH <7.3 or serum bicarbonate <15 mmol/L. Ketonemia* or
ketonuria.
Clinical manifestations of diabetic ketoacidosis: Dehydration. Tachypnea (deep, sighing (Kussmaul) respiration).
Nausea, vomiting, and abdominal pain that may mimic an acute abdominal condition.
Confusion, drowsiness, progressive obtundation, and loss of consciousness.

Epidemiology of DKA: Is a problem of type 1 diabetes (T1DM). Occurs in 15-70% of patients at onset of T1DM. More common in
children < 5Yrs of age. Rates inversely co-relate with regional incidence of T1DM. 1 – 10% per patient/year in established T1DM.
Pathophysiology (What’s wrong): - Elevated blood glucose - Dehydration
- Altered electrolytes - Metabolic acidosis (ketosis).

Clinical features (What you see):


 High lab blood glucose, glucose meter reading or urine glucose, polyuria, polydypsia
 Sunken eyes, dry mouth, ↓ skin turgor, ↓ perfusion (shock rare)
 Irritability, change in level of consciousness
 Acidotic breathing, nausea, vomiting, abdominal pain, altered level of consciousness

Severity: The severity of DKA is categorized by the degree of acidosis:


Mild: venous pH <7.3 or serum bicarbonate <15 mmol/L Moderate: pH <7.2, serum bicarbonate <10 mmol/L
Severe: pH <7.1, serum bicarbonate <5 mmol/L HHS, formerly referred to as hyperosmolar non-ketotic

Risk factors: Risk factors for DKA in newly diagnosed patients include;
- Younger age - Delayed diagnosis
lower socioeconomic status, and residence in a country with a low prevalence of type 1 diabetes mellitus (T1DM).
Risk factors for DKA in patients with known diabetes include:
- omission of insulin for various reasons, limited access to medical services, and unrecognized interruption of insulin delivery in patients
using an insulin pump.
- Absolute insulin deficiency OR Stress, infection or insufficient insulin

Subsequent release of Counterregulatory Hormones: ↑ Glucagon ↑ Cortisol ↑ Catecholamines ↑ Growth Hormone.


↑ FFA to liver ↑ Ketogenesis ↑ Lipolysis Acidosis ↓ Alkali reserve

Pathophysiology of Diabetic Ketoacidosis: - Acidosis: ↓ Alkali reserve - ↑ Proteolysis


- ↓ Protein synthesis - ↓Glucose utilization - ↑ Gluconeogenic substrates - ↑ Glycogenolysis
- Glucosuria (osmotic diuresis) - ↑ Lactate Hyperosmolarity - Decreased fluid intake
- Hyperglycemia : ↑ Gluconeogenesis or - Impaired renal function: Loss of water and electrolytes

Diagnosis – clinical: - Dehydration - Rapid, deep (Kussmaul) respiration


- Nausea, vomiting, abdominal pain - Reduced level of consciousness
- Increased leukocyte count with left shift - Fever (only when infection present)
Diagnosis – laboratory: - Serum glucose > 11mmol/L - PH < 7.3 or HCO3 < 15meq/L
- Ketonemia and ketonuria

Mild Moderate severe

PH < 7.3 <7.2 <7.1

HCO3(meq/L) <15 <10 <5


Haemogram, BS for malaria, urine and blood cultures, serum electrolytes– to rule out other causes of illness

MANAGEMENT: - Correction of shock - Correction of dehydration - Correction of hyperglycaemia


- Correction of deficits in electrolytes - Correction of acidosis - Treatment of infection
- Treatment of complications

259
Compiled by Obasi. D. C. Chinedu

MANAGEMENT – FLUID (restore circulatory volume):


SHOCK is rare in paediatric DKA. 0.9% saline or Ringers at 20mls/kg boluses – repeated until perfusion is re-established. More commonly
10-20ml/kg/h for 1-2hours
Subsequent replacement: 0.9% saline or Ringer’s lactate over 4 – 6 hours

Maintenance Fluid: Fluid replacement is with 0.45% saline or normal saline. Clinical assessments for volume depletion are subjective. Use
5-7% for moderate and 7-10% in severe dehydration. Aim to replace deficit evenly over 48 hours. Achieved by adding remaining deficit to
maintenance volume/48h

INSULIN: DKA is caused by reduction in insulin and an increase in counter- regulatory hormones. Start insulin only after initial volume
expansion (1 – 2h) and establishing K+ status. Add 50units of rapid acting (regular) insulin to 50ml of 0.9% saline to make 1unit/ml.
Start 0.1unit/kg/h if newly diagnosed or blood glucose >15mmol/l. Start 0.05unit/kg/h if blood sugar < 15mmol/l
During insulin therapy blood glucose falls by 2-5mmol/L/h. To avoid a more rapid fall, add 5% dextrose to 0.45% or 0.9% saline once blood
sugar is 14mmol/L(250mg/dl).
DKA is resolved when PH>7.3 or serum HCO3>15mmol/l. Aim to keep blood sugar around 11mmol/l until resolution of DKA

POTASSIUM: Total body potassium deficit is 3-6mmol/kg. Replacement is thus required regardless of serum potassium measurement.
If hypokalemic begin potassium at time of initial volume expansion and before insulin (20meq/L)
If [K+] normal begin potassium after initial volume expansion while starting insulin (40meq/L)
If hyperkalaemic differ potassium until patient has passed urine

BICARBONATE: Trials have shown no benefit of routine HCO3 administration AND its use causes:
Dissociation curve of oxygen to shift to the left worsening tissue hypoxia.
Immediate increase in CO2, which easily crosses the blood brain barrier into brain cells causing a paradoxical increase in cellular acidosis.
This increases the risk of brain edema.
Exceptions are severe acidosis (PH<6.9) or resuscitation for shock with catecholamines

BICARBONATE-OXYGEN DISSOCIATION

Bicarbonate administration
Causes a shift of the
Oxygen disosciation
Curve to the left

MONITORING: - Hourly blood glucose - Hourly fluid input & output


- Neurological status at least hourly - Electrolytes 2 hourly after start of IV therapy
- Monitor ECG for T-wave changes.

Complications of therapy: • Inadequate rehydration • Hypoglycemia • Hypokalemia


• Hyperchloremic acidosis • Cerebral oedema

COMPLICATIONS
1. Cerebral oedema – accounts for 60-90% of all DKA deaths. Clinical features: Headache, Slowing of heart, Rising blood pressure, Altered
neurological status (irritability, drowsiness, age inappropriate incontinence).
Cranial nerve palsy – III, IV, VI, Decreasing oxygen saturation
Definitive diagnosis is via brain CT scan.
Tx: Start treatment as soon as it is suspected. Elevate head of bed to 30 degrees
Mannitol: 0.5g -1g/kg over ½ hour. Repeat in 2-4 hours if indicated or
3% saline: 3-5ml/kg

Challenges in diagnosis
Differential diagnosis: Pneumonia, asthma. Malaria.
Blood glucose measurement: Low index of suspicion. Glucometers. Strips.

Solution:
- Sensitisation: measure blood glucose in all critically ill children to rule out hypoglycaemia – will also identify hyperglycaemia
260
Compiled by Obasi. D. C. Chinedu

- Provide point of care glucose measurement


- Training: CDiC

Treatment:
Fluids: Availability of appropriate fluids. Fluid regimes based on adult care
Insulin: Most facilities have insulin for acute in-patient use. Challenges of transitioning to long term therapy
Advise on procurement of appropriate fluids. Use of treatment algorithms to aid fluid and insulin therap
Management of DKA

Dehydration K+ Replacement Acidiosis (Insulin)


)Therapy)
start insulin 1 hour after initial fluid repl
First one hour
Serum K+ >3MMOL/L IV insulin infusion 0.1u/kg/hr.
DKA in shock, Volume K+<3MMOL/L
rapidly restore
Add 40mmol/l in the Add 50 units to 500mls of N/S giving 1
depleted but Give 20mmol/l
circulatory not in shock--
maintenance fluid in unit ------10mls
volume with of KCL with the the 2nd hour with
10 mL/kg first hour rapid Using soluset the number of drops=the
isotonic saline in commencement of
infused over 30
20 mL/kg bolus infusion before insulin. Alternate weight of the patient. Eg wt of 30kg
to 60 minutes; giving insulin
ASAP. 20mmol/l of kcl =3units =30mls/1 hr=Volume/time
Reassess
R/A – Repeat with 20mmol/l of
2nd bolus if need
Use N/S or 0.45% depending on Na kpo4 =no of drops =30drops .
level Expect blood glucose drop
Give K+ throughout IV fluid therapy. of 2-5mmo/l/hr.
2ND hour Max-0.5mmol/kg/hr.
If further drop of
Add remaining deficit +90mls/kg to Goal is to maintain serum K+ 4-5mmol/l BG100mg/dl , reduce insulin
the maintenance fluid for 48hours.
to 0.05-0.025iu/kg/hr
1st 10kg=100mls/kg
2nd 10kg=1000mls+50mls/kg
3rd 10kg=1500mls+20mls /kg Resolution of symptoms If Iv assess not possible +uncomplicated DKA

Transition
(pt fully conscious/stable).
Liberal fluid intake
Add 5% D if BG <250mg/dl or Give rapid acting insulin 0.3unit/kg followed by
>5mmol/lkg/hr. hourly 0.1u/kg or 2-3hourly- 0.15-0.2unit /kg
If further drop <100mg/dl, , increase Short acting regular insulin can be given 4 hourly
to 10-12%D calculate 24 hour dose @0.8-1 unit /kg/in 6 DD.

Monitoring:
Staffing: High dependency staffing ratios
Blood glucose: Glucose strips. Manpower
Electrolytes: Limited laboratory capabilities. Costs.

261
Compiled by Obasi. D. C. Chinedu

PERINATAL ASPHYXIA AND NEONATAL RESUSCITATION


Dr. ILOH KENECHUKWU K

OBJECTIVES
- Definition of asphyxia - Causes/predisposing factors - Physiology of asphyxia
- Organ/system manifestations - Management

Definition: Condition in the foetus/newborn where there is reduced O 2 saturation (hypoxaemia) and increased acid in the blood (acidaemia)
from carbon dioxide retention and lactic acid accumulation. Perinatal asphyxia is a condition of impaired blood gas exchange that, if
persistent, leads to progressive hypoxia and hypercapnia.
Criteria for diagnosis include: - Profound fetal acidemia (pH<7.0)
- Depression at birth with Apgars of 0-3 for more than 5 minutes
- Evidence of encephalopathy - Multi-system organ dysfunction

Causes/ Predisposing factors: 50 – 70% of babies who would require assistance at birth may be predicted based on risk factors
Maternal (prenatal) factors:
- Infections - chorioamnionitis etc - Lungs - pneumonia, asthmatic attack etc
- Cardiac - arrythmias, heart failure etc - Uterus - hypertonia, malformations, rupture.
- Vascular - anaemia ( haemoglobinopathy, Nutritional, Leukaemias, Diabetes, Hypertension, hypotension)
- Others - drugs; narcotics , anaesthetics, alcohol, magnesium sulphate, tranquillizers.
- Prolonged obstructed labour.

Placental problems: Abruptio, placenta previa, placenta insufficiency.

Foetal factors: - Prematurity, postmaturity,


- Developmental anomalies eg. Diaphragmatic hernia, pulmonary hypoplasia, choanal atresia.
- Cord problems eg knotting, compression, prolapse - Infections eg congenital pneumonia

Physiology of Asphyxia:
10 apnoea 20 apnoea
 Respirations cease  Respirations cease
 Heart rate decreases  Heart rate decreases
 Blood pressure maintained  Blood pressure decreases/circulatory collapse
 Response to stimulation  No response to stimulation
 The whole process may take up to 20mins
Foetus usually recovers (clinical & biochemical) between episodes, if transient and infrequent.
If baby is born during such episodes, may be apnoeic but responds promptly to resuscitation, May have few or no sequelae.
Prolonged episodes (about 1hr) or foetus does not recover between episodes, there is;
 Hypotension & profound acidaemia. Multiorgan damage including the brain
 Usually require prolonged resuscitation/have delayed onset of spontaneous respiration
 Severe neonatal illness/ permanent sequelae even with prompt resuscitation

The principal manifestation of hypoxic-ischemic cerebral injury is encephalopathy. CNS manifestations are variable and involves over
time;
Birth -12 hours – deep stupor/coma, respiratory failure or periodic breathing, diffuse hypotonia, intact pupillary and oculomotor response
and subtle or focal seizures by 6-12 hours. Generalized tonic seizures may occur in preterm infants.
12-24 hours – level of alertness may appear to improve, but seizures, apnea and marked jitteriness develop. Term infants may present with
weakness of the proximal upper limbs, while preterm have lower extremity weakness.
24-72 hours – The consciousness level worsens leading to deep stupor and coma. Pupillary and oculomotor disturbance develop due to
brainstem involvement. Death most often occur during this stage.
After 72 hours – Mild to moderate stupor may persist, but overall level of alertness improves. Diffuse hypotonia may persist or hypertonia
can become evident. Feeding difficulties become obvious due to abnormal sucking, swallowing and tongue movement.

Effects of severe asphyxia


CNS - Cerebral oedema (cytotoxic & vasogenic), SIADH, ICH, NE resulting in seizures, hypotonia, mental retardation, learning disabilities,
speech disorders, cerebral palsy (spastic quadriplegia or dyskinetic).
Respiratory - respiratory distress, type II RDS, persistent pulmonary hypertension, MAS.
CVS - left ventricular dysfunction, tricuspid incompetence, PDA, cardiogenic shock.
Renal - bladder atony, acute tubular necrosis, acute cortical necrosis.
Digestive System – paralytic Ileus (Increased risk of NEC, Feeding intolerance), liver dysfunction.
Haemopoietic - Marrow suppression, Thrombocytopenia, DIC, increased risk of infection.
Metabolic - Hypo or hyperglycaemia, hyponatraemia, hypocalcaemia, acidosis.

262
Compiled by Obasi. D. C. Chinedu

Skin – Dusky appearance, mottling

Grading of encephalopathy (Sarnat &Sarnat modified by Levene)


Grade I Grade II Grade III

Irritability

Hyperalert Lethargy Comatose

Mild hypotonia Marked abnormality in tone Severe hypotonia

Poor suck Unable to feed PO Seizures + Failure to maintain spont respiration


No seizures Prolonged seizures or no seizures, isoelectric EEG
Low 10min Apgar score more specific but less sensitive predictor of death or disability.

Assessment of baby
 Anticipation is the key to good care – adequate preparation (man and material)
 Maternal and foetal monitoring before and during labour
 FHR, use of cardiotocograph to monitor variability in FHR and foetal scalp blood pH
 At delivery, assess heart rate and respirations, colour (pulse oximetry where available) ,
 APGAR SCORING

Apgar score; Has the potential for diagnostic misclassification


Score 0 1 2

Colour Pale Or Centrally Peripheral Cyanosis Completely Pink


Cyanosed

Heart Rate Absent <100b/Min >100b/Min

Reflex Non Grimace Cry


Irritability

Muscle Tone Flaccid Some Flexion Well Flexed

Respiration Absent Weak, Irregular Regular

Interpretation:
Score > 7 -well
1 – 3@ 1min or < 5 @ 5 min is severe asphyxia and risk of sequelae is high:
Delayed onset of spontaneous respiration
Signs of multiorgan dysfunction
Clinical signs of encephalopathy may be present even with acceptable scores
Score of 5 or less @ 10mins predicts death or major neurologic disability

Management: Resuscitation in delivery room


A: keep warm, clear airway, dry/stimulate, reposition
B: ensure ventilation (Bag and mask), oxygen or room air – term infants, b/w room air & 100% O 2
C: ensure circulation - chest compressions, adrenalin as necessary
Give volume expanders if in shock & other drugs as indicated
Post resuscitation – stabilization, monitoring and appropriate intervention (STABLE)

Symptomatic and Supportive management: - Fluid and electrolyte - Anti-seizure agents


Feeding – NPO for 72hrs - Blood transfusion - Dialysis
Ventilatory support - Inotropes

263
Compiled by Obasi. D. C. Chinedu

NEONATAL JAUNDICE
Dr. Obumneme-Anyim Ijeoma

LEARNING OBJECTIVES
- Definition of terms - Metabolism of bilirubin - Types of Jaundice
- Differentiate between physiological and pathological jaundice - State the causes of hyperbilirubinemia
- Discuss the complication of hyperbilirubinemia - Therapeutic management

Definition of terms
Jaundice: Yellow or greenish staining of the skin and sclera as a result of abnormally high blood level of bilirubin.
Neonatal hyperbilirubinemia: Elevation of the bilirubin level in the newborn’s blood resulting in yellowish staining of skin and sclera.
Normal bilirubin level in the neonate is less than 5mg/dl. Jaundice occurs when levels are more than 5mg/dl. Kernicterus is a
pathological staining of the brain tissues (basal ganglia, hippocampus, geniculate bodies, cranial nerve nuclei and sometimes the
cerebellum).

BACKGROUND: Jaundice is the most common condition that requires medical attention in newborns. Approximately,85% of all term
neonates and most preterm neonates develop clinical jaundice. In most infants, it reflects a normal transitional phenomenon. Unconjugated
bilirubin is neurotoxic and can cause death in newborns and lifelong neurologic sequelae in infants who survive. Every jaundice must be
observed/investigated.
Normal adult bilirubin level is <1mg/dl and jaundice develop when serum bilirubin exceeds 2mg/dl. In neonates, jaundice appears at
> 5mg/dl. Jaundice progresses in a cephalo-caudal direction. Jaundice which occurs in the first 24hours of birth must be investigated and
treated as an emergency. Jaundice persisting for more than 14days must be investigated.

SOURCES OF BILIRUBIN: Bilirubin is derived from the breakdown of heme-containing proteins in the RES. A normal newborn produces
about 6-10mg bilirubin/kg/day.{adults: 3-4/kg/day}. The major heme-containing protein is the RBCs contributing 75% of bilirubin
production. One gram of haemoglobin produces 34mg of unconjugated bilirubin. 25% of bilirubin come from myoglobin, cytochromes,
catalase, peroxidase and free heme.

BILIRUBIN METABOLISM: Heme is converted to Biliverdin via (heme oxygenase) which is further converted by biliverdin Reductase to
Bilirubin. Carbon monoxide and Iron are produced in the process which are (excreted) and (re-used) respectively.
Transport: bound to albumin and transported to the liver.
Uptake: dissociates from albumin, bound to cytoplasmic Ligandin(Y protein) for transport into the smooth endoplasmic reticulum.
Conjugation: bilirubin is bound to glucoronic acid which is the water soluble form by uridine
diphosphogluconurateglucoronosyltransferase(UGT). Conjugation can be to mono or diglucuronides. Both are water soluble and are excreted
into bile canaliculi via a concentration gradient.
Excretion: Conjugated bilirubin from the biliary tree enters the GI tract and is eliminated in the stool. Conjugated bilirubin is converted to
the unconjugated form by intestinal enzyme β-glucuronidase when the gut transit time is prolonged and then delivered back to the liver
for re-conjugation in a process called entero-hepatic circulation.
Intestinal bacteria prevent this process by converting Conjugated bilirubin to urobilinoids which are not substrates for β-glucuronidase

Classification of Jaundice
Transient jaundice is common in the neonates. It does not always signify a disorder unlike in the adults.
It can therefore be classified as physiological or pathological.
Jaundice can also be classified as conjugated (direct hyperbilirubinemia) or unconjugated (indirect hyperbilirubinemia).

MECHANISMS OF PHYSIOLOGICAL JAUNDICE

264
Compiled by Obasi. D. C. Chinedu

Increased Bilirubin production: - Increased RBCs volume per kilogram.


-Decreased RBCs survival (90 days) preterms; 40days -Increased ineffective erythropoiesis
-Increased turn over of non-hemoglobin heme proteins
Increased entero-hepatic circulation: - high level of β-glucuronidase - decreased intestinal bacteria
-decreased gut motility - poor evacuation of meconium
-preponderance of bilirubin mono glucuronide
Defective uptake: -decreased ligandin - binding of ligandin by other ions.
Defective conjugation due to decreases UGT activities
Decreased hepatic excretion of bilirubin.

PATHOLOGICAL JAUNDICE: Jaundice is pathological when


- It is present in the first 24hours of birth. - Rate of increase in bilirubin is >5mg/dl/day or 0.2-0.5mg/dl/hour.
- Total bilirubin level is >95th percentile for age in hours based on a nomogram
- Jaundice persisting for more than 14days. - Direct bilirubin (unconjugated fraction) >2mg/dl or 20% of the total.

UNCONJUGATED HYPERBILIRUBINEMIA
Problem: An infant’s indirect bilirubin is 10mg/dl. (Total and direct bilirubin is measured in blood. The indirect fraction is calculated.
(TSB –CB =UCB)
Important questions: Age.
Is the infant being breastfed? Breastfeeding jaundice (early) and breast milk jaundice (late).
Is the infant dehydrated
Ethnicity: common in Nigerian, native American Indians, Mediterranean, Sephardic Jews.

Breastfeeding jaundice: In exclusively breastfed infants. Appears at 24-48 hours of age. Peaks by 5-15days. Disappears by the 3 rd week.
Related to inadequate breastfeeding. Treatment is proper and adequate breastfeeding.
Breastmilk jaundice: Occurs in 2-4% of exclusively breastfed babies. Serum bilirubin >10mg/dl beyond 3-4weeks.
Due to agents in breast milk which inhibit bilirubin conjugation in the liver or deconjugate bilirubin in the gut. May require
phototherapy. Usually declines over time.

UNCONJUGATED HYPERBILIRUBINEMIA: Derived mainly from hemoglobin metabolism. Needs to be conjugated before it can be
excreted in bile, stool or urine. It is never present in urine.

CAUSES OF UNCONJUGATED HYPERBILIRUBINEMIA


Overproduction of bilirubin due to: -ABO, RH isoimmunization.
-Hereditary spherocytosis, elliptocytosis, somatocytosis -G6PD deficiency
-Other red cell enzyme deficiencies
-Extravascular blood like petechiae, hematomas, pulmonary, cerebral or occult hemorrhage
-Polycythemia -Increased enterohepatic circulation.
Sepsis like: - TORCH - Intra uterine infections; syphilis, hepatitis.
-Respiratory distress syndrome - Hypoxia-ischemia - Infant of diabetic mother
Medications e.g: Penicillins, oxytocin, sulphonamides, vitamin K.

Visual estimation of jaundice: Face = 5mg/dl Upper trunk = 10mg/dl


Abdomen = 12mg/dl Palms/soles >15mg/dl.

Important Investigations: Serum bilirubin, total and direct. - Blood group (baby and mother) and Coombs test
FBC, differentials and smear (observe RBC morphology) - Reticulocyte count, G6PD status,
Others include; Albumin level, hemoglobin electrophoresis.

Management
a. Phototherapy: Bilirubin absorption of visible light in the blue region of the spectrum converts albumin bound unconjugated bilirubin
through Photo-oxidation, Photo-isomerization and Structural isomerization into products which become water soluble and secreted in
bile and excreted in stool and urine.
Effective phototherapy requires: - Full illumination of patients body surface area
- Irradiance level (energy output) ≥30µW/cm²/nm¹ - Waveband 460-490nm (blue-green region)
- Position infant 20cm below the device.
Indication for phototherapy: - Rapidly increasing jaundice - While preparing for EBT
- Serum bilirubin of15mg/dl at presentation. - Post exchange blood transfusion.

Side-effects of phototherapy: - Skin rashes, dehydration, - Bronze baby syndrome


- Retinal degeneration has been reported in animal studies.

265
Compiled by Obasi. D. C. Chinedu

- Contraindicated in congenital erythropoietic porphyria; exposure could cause severe bullous eruption & death.

b. Exchange blood transfusion EBT: This is the fastest method of reducing bilirubin level in the blood. It is indicated when the risk of
acute bilirubin encephalopathy is significant, when there is ongoing hemolysis or when the rate of rise in bilirubin level indicates it could
get to 25mg/dl within 48hours. As well as during high level of bilirubin and early signs of encephalopathy.
Guideline for EBT: -Generally, type O Rh negative blood is used for ABO or Rh incompatibility.
-Donor blood must always be crossmatched with maternal serum. -Donor blood should be warmed to 37ºC
-Fresh whole blood (not more than 4 days) should be used.
- Double volume exchange replaces 85% of the circulating RBCs and decreases TSB by 50% of the pre exchange level.

Side-effects: - Hypocalcemia - Hypomagnesemia - Hypoglycemia - Metabolic acidosis


- Hyperkalemia - Thromboembolism - Infections - Hemolysis - Graft versus Host disease
- Bleeding from thrombocytopenia,deficient clotting factors - Hypothermia, hyperthermia
- Necrotising enterocolitis

c. Other modalities:
Pharmacologic: -Phenobarbital(2.5mg/kg/day). Useful in Gilberts syndrome and as an adjunct therapy in exaggerated neonatal jaundice.
Increases the production of Ligandin and UGT. Takes 3-7 days to become effective.
Metalloporphyrins: inhibits heme oxygenase.
Albumin: decreases the pool of unbound bilirubin and reduces toxicity.

BILIRUBIN TOXICITY
Acute bilirubin encephalopathy: clinical manifestation of bilirubin toxicity seen in the neonatal period. Divided into 3 phases
-Early phase: hypotonia,lethargy,poor suck, high-pitched cry.
-Intermediate phase: hypertonia of extensor muscles (opisthotonus, rigidity, oculogyric crisis and retrocolis), irritability, fever and seizures.
(most infant die at this stage. Survivors develop chronic bilirubin encephalopathy).
Advanced phase: pronounced opisthotonus, shrill cry, apnea, seizures and death.

UCB crosses the disrupted BBB as free bilirubin or bound to albumin. (factors that disrupt BBB include hyperosmolarity, asphyxia, acidosis,
hypercarpnia, hypothermia and hypoglycemia).
Kernicterus is a pathological diagnosis, it refers to the yellow staining of the brain cells with evidence of neuronal injury. seen in basal
ganglia, cerebellar nuclei, hippocampus.
Chronic bilirubin encephalopathy (kernicterus) is marked by (cerebral palsy) athetosis, sensineural deafness, upward gaze paralysis, dental
hyperplasia and intellectual deficits.

CONJUGATED HYPERBILIRUBINEMIA
Conjugated hyperbilirubinemia is defined as a measure of direct reacting bilirubin >1mg/dl, if the TSB is ≤5.0mg/dl or >20% of TSB. It is
the biochemical marker of cholestasis and a sign of hepatobilliary dysfunction.
It is always pathological. It affects about 1:2500 infants.
Less common than unconjugated hyperbilirubinemia.

Causes:
 Metabolic conditions like galactosemia, Crigler-Najjar syndrome type 1 and 2, Gilbert disease, Tyrosinosis, Lucey-Driscoll syndrome.
 Endocrine disorders like hypothyroidism, hypopituitarism, infant of diabetic mother, anencephaly
 Obstructive disorders like biliary atrsia, choledochal cyst, Dubin-Johnson syndrome, Rotor syndrome, cystic fibrosis and α1-antitrypsin
deficiency and total parenteral nutrition.

266
Compiled by Obasi. D. C. Chinedu

267
Compiled by Obasi. D. C. Chinedu

URINARY TRACT INFECTIONS (UTIs) AND OBSTRUCTIVE UROPATHY.

Introduction
UTI: A common bacterial infection in children with occasional remnant degrees of morbidity even after the period of acute illness. When left
untreated may result in serious and life-threatening kidney problems.

UTIs - Urethritis (infection limited to the urethra). Cystitis (inflammation of the bladder 20 to infections),
Pyelonephritis (infection of the renal parenchyma, calyces and renal pelvis).
Renal abscess (intra renal or peri renal).

Epidemiology of UTI in children.


UTI is the commonest dx of Genito-Urinary System. It is the 2nd commonest childhood infection worldwide. Age incidence of UTI (before
11yrs): 3-5% in girls and 1% in boys.
Average age of infection: 3 yrs in girls (coinciding with the period of toilet training) and 1 yr in boys.
By 1 yr the male:female ratio is 2.8-5.4:1.0, and after 1 yr, then the ratio becomes 1:10.

Predisposing factors: Presence of a short urethra predisposes girls to this condition, thus the higher prevalence in them after the 1 st yr of life
A 10-20 fold increased risk for UTI in uncircumcised males infants < 1yr of age. Presence of the foreskin encourages periurethral
bacterial colonization.
Major risk factors are from obstruction to urine flow and urine stasis. Anatomic abnormalities, Nephrolithiasis, Renal tumor, indwelling
catheter, Ureteropelvic junction obstruction, Megaureter, Extrinsic compression etc.

Others are:
- Vesicoureteral reflux (VUR) which predisposes to chronic infections and renal scarring.
- Tight and dirty underwears - Pin worm infestations
- Constipation which can cause voiding dysfunction - Sexual activity in girls b/cos of incomplete bladder emptying
- Neuropathic bladder b/cos of incomplete bladder emptying - Toilet training esp. wiping from back-front in girls
- Prolonged hospital stay– common nosocomial infection in children - Bubble bath

Aetiology
Commonest causative organism for UTI: E.coli in all age groups except in hospitalized children (Nosocomial infections). Accounts for 75-
80% of UTIs in all children and 75-90% in girls.

Other gram negative organisms causing UTI: - Klebsiella, - Pseudomonas - Proteus and - Citrobacter.
Gram positive causative organisms: Cocci (Enterococci, Staph.aureus and epidermidis).
Viral causes: Adenoviruses associated more with cystitis.

Common bacterial causes of nosocomial UTI: - Klebsiella(47.3%), - Staphylococcus aureus (31.6%)


- E. coli (10.5%) - Pseudomonas(5.3%) and - Citrobacter(5.3%).

PATHOGENESIS: UTI B4 infancy often results from a haematogenous spread. Most UTIs esp in females after 1 yr are ascending
infections through the urethra, often involving bowel flora. In ascending infections esp. in the uncircumcised males, bladder infection arises
from the urethra which is highly colonized by the bacterial flora beneath the prepuce and close to the anus. The infection in some cases may
progress to the kidneys resulting in pyelonephritis.

Pathogenesis of UTI Factors involved:


Loss of the anti reflux protective action of the simple and compound papillae.
Encouragement of intra renal reflux: some compound papillae located at the upper and lower poles of the kidneys allow intrarenal reflux
which bring in infected urine.
Presence of fimbrie or pilli on the surface of the infecting bacteria: they attach themselves to the walls of the urinary tract. (These
fimbriae are commonly seen in E.coli which possesses 2 different types- types I and II)
Initiation of immunological and inflammatory response: resulting in renal injury and scarring.

Clinical Manifestations: The symptoms and signs of UTI vary markedly with age as seen in the table (although the 1 st UTI may be
asymptomatic)
- Unexplained fever (rectal temp. >38.50C) esp in young patients
- In Patients of all ages with fever and cong. anomalies of the urinary tract, a urine sample should be collected.

At infancy, all boys with such symptoms as: - Straining at micturition,


- Dribbling or a poor urine stream i.e voiding intermittently should be investigated for urethral obstruction from
PUV (Posterior urethral valves), urethral meatal stenosis, ureterocoele, or bladder diverticula.

268
Compiled by Obasi. D. C. Chinedu

Frequency of occurrence (%) by Age

Symptom or Sign 0-1 mo 1-24mo 2-5yr 5-12yr


Failure to thrive 53 36 7 0
Jaundice 44 0 0 0
Vomitting 24 29 16 3
Diarrhoea 18 16 0 3
Fever 11 38 57 50
Convulsions 2 7 9 5
Irritability 0 13 7 0
Changes in urine 0 9 13 0
Haematuria 0 7 16 8
Frequency, Dysuria 0 4 34 41
Enuresis 0 0 27 29
Abdominal pain 0 0 23 0
Flank pain 0 0 0 0

Classification of symptoms with type of UTI


Cystitis: - Dysuria, urgency, frequency, - Supra pubic pain,
- Incontinence, malodorous urine. - Does not usually cause fever nor result in renal injury.

Clinical Pyelonephritis: - Abdominal/flank pains, - fever, - malaise, - Nausea and vomiting


- Jaundice in neonates and occasionally diarrhea. - Poor feeding,
- Irritability, weight loss or poor weight gain in some newborns or infants.
- Symptoms indicate bacterial involvement of the upper URTract.

Asymptomatic bacteruria: Referring to children with positive urine culture but no clinical manifestations of infections. Occurring
mostly in girls. Usually benign and not causing renal injury.

Diagnosis / Laboratory Investigations


There is a need to make a proper and correct dx in patients with suspected UTI. Delays may result in risks of renal scarring and long term
complications. Thus, any child with temperatures > 38.50C regardless of the cause of the fever must have the following investigations carried
out:
Urinalysis / Microscopy / Culture / Sensitivity
Urine Nitrite test: Quite reliable when positive and based on the action of the presence of urine nitrate to nitrite converting bacteria.
Requires urine to be left for at least 1 hour in the bladder without voiding to enable bacterial action.
Urine culture - Gold standard for diagnosis. Dip stick slide technique can be used; however, its result is controversial in terms of accuracy.
Renal function tests - most often may not be affected in uncomplicated UTIs except in cases suggestive of bilateral obstruction or
parenchymal reduction.
Blood cultures - especially in neonates since septicaemia is a close differential in this group.
Other tests to rule out other causes of fever:
- Blood film for Malaria parasite - ESR , Sputum for AFB
- FBCount. - CXR

Methods of Urine Collection


- Mid-stream urine in older children/adults - Clean catch in younger children - ideal for urine analysis.
- Bag urine can also be used where above is not possible however, the bag must be sterile. A negative result is reliable but when positive,
urine samples must be collected via a clean catch or supra pubic tap or catherization for confirmation.
- Urine collected from the catheter - Supra pubic tapped urine.

Sample Storage: Collected samples must be kept in sterile containers and chilled to 40C. Examined in the lab within 2hrs Or kept in the
fridge at 40C and examined in the next 24hrs. All collected urine samples could also be preserved in appropriate dose of Boric acid (which
preserves the pus cells and bacteria).
For urine culture, sterile urine samples are inoculated on some media e.g MacConkey agar, blood agar and incubated at 37 0C or room temp.
Isolated organisms are then gram stained, and tested for drug sensitivity

Imaging: The aim is to identify anatomical abnomalities which may predispose the child to recurrent UTI.

269
Compiled by Obasi. D. C. Chinedu

Criteria for selection:


 All children with UTI should have some form of imaging after the 1 st proven infection.
 All boys and all children below 5 yrs of age
 All patients with voiding dysfunction
 All school-age girls with recurrent UTI to identify those with VUR, obstruction and other urinary abnormalities.

Imaging / Radiological methods


Plain abdominal X-ray: can exclude presence of radio-opaque renal stones and lumbo-sacral defects.
Ultrasonography: Clearly defines masses, makeout renal agenesis, hydronephrosis, renal stones, cystic kidneys and renal tumors.
Voiding cystourethrogram(VCUG): carried out when the urine is sterile. Indicated in children <5yrs, with febrile UTI, school-aged girls
with 2 or more UTIs and any male with UTI. Commonest finding is VUR in =40% of pts.
It outlines the bladder and urethra and determines degree of VUR. Best method for establishing presence of a reflux. Test is to be delayed
2-6wks to allow inflammation subside.
Excretory urography (IVU): More detailed view of the upper urinary tract than ultra sound. Partial assessment of renal function by
indicating a delay in the excretion of the contrast agent.
Radionuclide renal imaging: Provides both anatomic and functional information about the kidney without requiring much irradiation.
Determines the shape, size and location of the kidneys. Detects renal obstruction
Measures function and blood flow. Determines contribution of specific kidney to renal function.
Computed Tomography Scan (CT scan): Useful for upper urinary tract in detecting renal scarring.
Magnetic Resonance Imaging (MRI): Serving almost the same purpose as the CT scan.

Differential Diagnosis/ Treatment: 1) Malaria 2) Lobar pneumonia


3) Severe upper respiratory tract infection (pharyngitis, acute otitis media)
4) Septicaemia especially in newborns
5) Other conditions which may present with fever in the child. 6) Tuberculosis

Treatment: This could be medical or surgical. Treatment should be started immediately with empirical antibiotics which most of these
organisms are sensitive to prior to outcome of the lab results.
This prompt action helps in preventing the onset of other long term complications such as renal scarring, hypertension and End stage renal
failure.
Where symptoms are mild, treatment could wait till the results are out.

Common drugs used include:


Trimethoprim-sulfamethoxazole (Septrin etc): 7mg/kg/day of trimethoprim orally or IV 12hrly x 7-10 days. Usually good in E.coli or
other gram negative bacteria except Pseudomonas.
Nitrofurantoin: 5-7mg/kg/day in 3-4 divided doses also effective against Klesiella –Enterobacter species.
Amoxicillin: 50mg/kg/day given in 3 divided doses is also quite effective.
Ampicillin with Gentamicin or Kanamycin for 10-14 days could be given in cases of suspected UTI in neonates because of the possibility
of an underlying septicaemia.
Parenteral Ceftriaxone or Ampicillin with an aminoglycoside eg Gentamicin (3-5mg/kg/day) in appropriate divided doses could be used in
cases of febrile UTIs.
Ciprofloxacin: 10mg/kg /day in 2 divided doses could be given in resistant UTI for children above 17 yrs of age.

Specific antibiotics should be started immediately the lab results are out.
Supportive treatment like rehydration and caloric replacement is important.
Repeat Urine culture 1 week at the end of the course of antibiotic treatment is important to ensure that the urine remains sterile. This can be
done again periodically for 1-2 yrs even if the child is asymptomatic.
Surgery should be performed on those with underlying structural abnormalities in their urinary tract.

Complications
1) Recurrent UTI resulting from underlying anatomical abnormalities e.g Vesico-ureteral reflux (VUR)
2) Renal scarring in 20% 3) Hypertension in 10% and
4) End stage renal failure in another 10% of the patients.

A UTI recurrence rate of 30-40% is estimated. 75-80% of UTI occurs in whites and 50% in Black American children. Girls are more affected
within the 1st 3yrs after the previous infection. Recurrence in girls is mostly due to re-infection from bowel flora than from relapse.

PREVENTION: This involves both 10 and 20 prevention of UTI.


10 Prevention: Involves ways of preventing UTI in the 1st place e.g - Proper toilet training
- Prevention of constipation - Treatment of underlying voiding dysfunctions
- Proper wiping techniques - Appropriate male circumcision at birth
2 0 prevention: Entails ways of ensuring that recurrences are not encountered by;
- Prophylactic use of antibiotics in children with cases of recurrences (1/3 rd of therapeutic dose).
270
Compiled by Obasi. D. C. Chinedu

- Detection and correction of urinary tract abnormalities in children with underlying obstructive uropathies.

Prognosis in children with UTI is generally good.

OBSTRUCTIVE UROPATHY
Definition: Obstruction to normal urine flow. Could be congenital or acquired, partial or complete.
It can also be located at the upper urinary tract or the lower urinary tract.
Congenital anomalies are commonest causes of urine obstruction in children and are commoner in males than in females.
It is important that this is noted early enough to avoid progressive damage to the bladder and upper urinary tract.

Causes of Obstructive Uropathy


A) Lower urinary tract obstruction:
1) Posterior Urethral valves 2) Urethral strictures 3) Anterior Urethral valves
4) Urethral polyps 5) Meatal stenosis 6) Tumors

B) Upper urinary tract obstruction- Commonest site is the ureteropelvic junction.


1) Functional intrinsic ureteropelvic junction obstruction
2) Ureterocoele 3) Ureteric stenosis 4) Aberrent vessels
5) Ureteric valve 6) Fibrous bands

Clinical Manifestations: Complaints vary with severity of the obstruction, Very severe cases = Diagnosis usually obtained very early in
infancy.

Clinical features include: - Neonatal ascites - Abdominal masses - Poor urinary stream
- Crying on micturition - Dribbling of urine - Diarrhoea/vomiting

In mild to moderate obstruction, the child may present later in childhood with the following:
- Failure to thrive - Feeding problems - Pyrexia - Loin pain
- Pyuria - Intermittent haematuria which may follow minor abdominal trauma at any age suggesting a possible bil.
hydronephrosis
Pathogenesis: Chronic urinary tract obstruction can lead to permanent damage to the urinary tract through the following pathway:
Intravesical obstruction-  Pressure effects  to changes in the bladder, such as trabeculation, cellule formation, diverticula, bladder wall
thickening, and finally detrusor muscle decompensation.
Progressive back pressure on the ureters and kidneys  cause hydroureter and hydronephrosis. The ureter becomes dilated and tortuous,
with the inability to adequately propel urine forward. Hydronephrosis can cause permanent nephron damage and renal failure.
Urinary stasis along any portion of the urinary tract increases the risk of stone formation and infection, and, ultimately, upper urinary tract
injury. Urinary tract obstruction can cause long-lasting effects to the physiology of the kidney, including its ability to concentrate urine.

Diagnosis of Obstructive Uropathy:


Hx of hydraminos (polyhydraminos or oligohydraminos) is very important for congenital obstructive uropathy and pre-natal diagnosis is
useful. Most often done by pre-natal ultrasound scanning.
Other diagnostic tools include: Ultrasound (including other children), micturating cystoureterogram, IVU, Retrograde Pyelography,
Arteriography, Isotopic renal scan and CT scan.
Urine examination for presence of Pus cells, Red blood cells, and bacteria.
Full blood count that may show presence of degrees of anaemia in the presence of uraemia, and /or high WBCs in cases with urine infection.
Massive proteinuria is not always encountered.
Renal function tests with emphasis on creatinine and urea levels which would indicate the degree of renal damage.

Renal Imaging studies


Ultrasonography: Ultrasonography of the kidneys and bladder is a useful imaging modality as an initial study.
Ultrasonography is sensitive in revealing renal parenchymal masses, hydronephrosis, distended bladder, and renal calculi.
Computerized Tomography Scan (CT scan): Provides anatomic detail. Provides information on the urinary tract, presence of any
retroperitoneal or pelvic pathologic condition that can affect the urinary tract via direct extension or external compression. A CT scan can
identify or rule out any other intra-abdominal processes that can cause presenting symptoms (eg, appendicitis, cholecystitis, diverticulitis,
abdominal aneurysms, and ovarian cysts).
Intravenous pyelography: Performed in patients with a normal creatinine value (<1.5 mg/dL) for visualization of the upper urinary tract.
Provides both anatomical and functional information. Superior to CT scan in revealing small urothelial upper tract lesions.
Radionucleotide studies: Determines the differential function of the kidney. Demonstrates the concentrating ability, excretion, and drainage
of the urinary tract.
Magnetic resonance imaging: Delineates specific tissue planes for surgical planning. Useful in evaluating the presence or extent of a renal
vein or inferior vena cava thrombus in cases of renal tumors. MRI does not reveal urinary stones well so is not often used as a first-line test.
271
Compiled by Obasi. D. C. Chinedu

Retrograde urethrography: Radiographic dye is injected into the urethral meatus via Foley’s catheter at the distal urethra. Fluoroscopy is
used to visualize the entire urethra for stricture or any abnormalities. This test can be particularly useful in working up lower urinary tract
trauma.
Nephrostography: Performed in patients who have a nephrostomy tube in place. Radiographic dye is injected antegrade through the
nephrostomy tube. With fluoroscopy, any abnormalities or filling defects in the renal pelvis or ureter are visible. Can be safely performed
even in patients with IVP contrast allergies.
Cystoscopy: Cystoscopy is the placement of a cystoscope through the urethral meatus and passing through the urethra into the bladder. Any
abnormalities in the urethra, prostatic urethra, bladder neck, and bladder can be visualized. Can be performed in the office or in the operating
room.
Cystoscopy with retrograde pyelography: Retrograde pyelography is performed in the operating room with a cystoscope in the bladder.
Radiographic dye is injected into each ureteral orifice. Then, with the use of fluoroscopy, any ureteral or renal pelvis filling defects or
abnormalities can be visualized. The contrast load does not interfere with renal function and can be used in patients with elevated creatinine
levels. It can also be used in patients with an IVP dye allergy because the contrast remains extravascular.

MANAGEMENT OF OBSTRUCTIVE UROPATHY: This involves both medical and surgical Tx. However
Surgery is the definitive management to achieve complete relief of obstruction.
Medical treatment include:
1) Pain relief with analgesics such as Codone, Hydrocodone, PCM, NSAIDS etc.
2) Use of appropriate antibiotics after urine is collected for culture.
3) Correction of uraemia
4) Nutritional Rehabilitation

Surgical Care: An important aspect of management of urinary tract obstruction and is dependent on the type and site of the obstruction
present. E.g
Lower urinary tract obstruction (bladder, urethra) can be relieved with the following:
Urethral catheterization: A urethral catheter (size 8F-24F) is a flexible external catheter that extends from the bladder through the urethra.
It can be left indwelling, or, done at intermittent intervals.
If blood is present at the urethral meatus after pelvic trauma and suspicion of urethral injury exists, a urologist should be consulted prior to
catheter placement.
Retrograde urethrography needs to be performed to rule out urethral injury.
Suprapubic tube or catheter: If a Foley catheter cannot be passed, a suprapubic tube can be placed percutaneously on the lower anterior
abdominal wall, approximately 2 finger-breadths above the pubic symphysis.

Upper urinary tract obstruction (ureter, kidney) can be relieved with the following:
Ureteral stent: A ureteral stent is a flexible tube that extends from the renal pelvis to the bladder. It can be placed during cystoscopy to
relieve obstruction along any point in the ureter. A ureteral stent generally needs to be changed every 3 months.
Nephrostomy tube: A nephrostomy tube is a flexible tube that is placed through the back directly into the renal pelvis. If a ureteral stent
cannot be placed cystoscopically in a retrograde fashion, a percutaneous nephrostomy tube can be inserted for relief of hydronephrosis. If
needed, a ureteral stent can then be passed in an antegrade fashion through the nephrostomy tube tract.

Other surgical procedures carried out for urinary tract obstruction include;
 Ablation of the valves only in uncomplicated posterior urethral valves (PUVs).
 Pyeloplasty in Ureteropelvic junction obstruction where the renal function is good or moderately impaired.
 Nephrostomy and later pyeloplasty in cases of severe renal function impairment.
 Nephrectomy where all else have failed.

Complications: The following are complications of obstructive uropathy: - Pain


- Post –obstructive polyuria with electrolyte imbalances (see causes in table 1)
- Infection, including cystitis (bladder infection), pyelonephritis (kidney infection), abscess formation, and urosepsis
- Urinary extravasation with urinoma formation - Urinary fistula formation - Renal insufficiency or failure
- Bladder dysfunction 2 0 to a defunctionalized bladder - Vesical calculi formation

Physiopathological state
Tubulopathy caused by acute renal Biochemical and immunological factors
failure

Damage to the Sodium overload Atrial natriuretic factor

272
Compiled by Obasi. D. C. Chinedu

dilution-concentration capacity

Decreased sodium Hyperuricemia, Prostaglandins, prostacyclins,


reabsorption hypercreatininaemia thromboxane A2, endothelins,
Hyperkalemia, metabolic
Leakage of potassium, magnesium and phosphorus Macrophage Infiltrate
acidosis
Decrease in H+ ions secretion in urine Dilutional hyponatraemia Increase in PAF and cytokines

Insensitivity to ADH

Outcome and Prognosis: The prognosis of urinary tract obstruction depends on:
- The cause, - Location, - Degree,
- Duration of obstruction, as well as Presence of a urinary tract infection.
Thus, the longer the duration of obstruction, greater severity of obstruction and the presence of a concomitant infection usually results to a
worse prognosis. The prognosis is favorable if the renal function is normal, the infection is cleared, and the obstruction is relieved on time.

273
Compiled by Obasi. D. C. Chinedu

CARDIOPULMONARY RESUSCITATION (CPR)


BY DR. KEN ADIELE

OUTLINE: - Introduction - Indications for CPR - The Procedure - Contraindications


- Complications - Follow up

INTRODUCTION: Cardiopulmonary arrest (CPA) occurs when a patient’s heart and lungs stop functioning. In
children, often begins as a primary respiratory failure unlike in adults where it is often primarily cardiac.
Cardiopulmonary resuscitation (CPR) refers to basic airway management, artificial ventilation, and chest compressions to provide oxygen
and circulation to core organs: the heart, brain, and lungs.
The scope includes the initial establishment and/or maintenance of airway, breathing, circulation, defibrillation and related emergency care.

INDICATIONS: Newly born, neonate, infant, or child of any age who is apneic and pulseless
Newly born with a heart rate less than 60 beats/min and not improving after standard newborn care
Neonate, infants and children with a heart rate less than 60 beats/min and poor perfusion, including drowning, shocking, sudden infant death
syndrome, severe asthma, infection & CHD.
In adults due to cardiac arrest from heart attack; drowning and electric shock.

EQUIPMENT REQUIRED: - Mouth-to-mask device - Bag-mask device, infant or child


- Airway adjuncts - Appropriate mask sizes

PROCEDURE OF CPR: The steps of CPR is summarised in the initials DRS ABCD.
D – Danger
R – Response (any?)
S – Send for help
A – Airway
B – Breathing
C - CPR
D - Defibrillation
NB: For Infants(<1yr) and small children(1-8yrs), follow the same steps of doing CPR on adults with minor changes

Position a child or adult on a hard surface.


Position a neonate or infant on a hard surface or on the forearm of the rescuer with the hand supporting the head.

ASSESS RESPONSIVENESS. IF UNRESPONSIVE; ASSESS BREATHING

 Open airway using either the headtilt/chin-lift maneuver (medical patient) or modified jaw thrust maneuver (trauma patient) to achieve a
neutral position
 Manipulation of the head to keep the airway in a neutral position is essential for effective ventilation.
 A towel roll under the shoulders of the infant or small child may help maintain neutral head position.
 Continuously assess effectiveness of CPR by ensuring chest rise and feeling for a palpable pulse every 2 minutes.

Headtilt/chin-lift maneuver. Remove any obvious obstructions, such as loose teeth or vomitus

274
Compiled by Obasi. D. C. Chinedu

VENTILATION PROCEDURE ASSESS PULSE

REPOSITION HEAD & CONT VENTILATION REPOSITION HEAD & CONT VENTILATION

Parameters for BLS Resuscitation for Health Care Providers


AGE COMPRESSIONS C:V Ratio DEPTH HAND PLACEMENT FOR
(MIN) COMPRESSION

Newly born 120 3:1 1/3rd depth of 2 fingers at lower 1/3 of sternum, 1 finger
(<1day) chest below nipple line, or 2 thumbs at
midsternum with hands encircling chest

Neonate (1- 100 30:2 1/3rd -1/2 depth of 2 or 3 fingers at lower 1/3 of sternum,
28days) 15:2 chest 1 finger below nipple line
One rescuer 2 thumbs at midsternum with hands
Two rescuers encircling chest

Infants (1-12 100 30:2 1/3rd -1/2 depth of 2 fingers at midsternum, 1 finger below
months) 15:2 chest nipple line
One rescuer 2 thumbs at lower 1/2 of sternum with
Two rescuers hands encircling chest

275
Compiled by Obasi. D. C. Chinedu

Child (1-8 yrs) 100 30:2 1/3rd -1/2 depth of Heel of 1 or 2 hands at lower 1/2 of
One rescuer 15:2 chest sternum
Two rescuers (do not push on xiphoid process)

Child over 8 yrs 100 30:2 1.5-2.5 inches Heel of one hand, other hand on top, at
One or two lower 1/2 of sternum between nipples
rescuers

C:V= COMPRESSION TO VENTILATION RATIO


AGE RATE OF RESCURE BREATHS PER MIN RESCURE BREATHS PER MIN WITH
COMPRESSION WITHOUT COMPRESSION COMPRESSION WITH ADVANCED AIRWAY
(MINS)

Newly born (<1day) 120 30 30

Neonate (1-28days) and 100 12-20 8-10


Infants(1-12 months)

Child (1-8 yrs) 100 12-20 8-10

Child over 8 yrs 100 10-12 8-10


One or two rescuers

NEWBORNS & INFANTS OLDER CHILDREN ADULTS

Summary of chest compression


 The depth of chest compressions should be approximately one third to one half the depth of the chest.
 Compressions should be deep enough to produce a palpable brachial, femoral, or carotid pulse. Push hard and fast and release
completely to allow chest to fully rise.
 Use the two-rescuer technique when possible.
 Reassessment: Check pulse after approximately every 5 compression-ventilation cycles

DEFIBRILLATION: INDICATIONS FOR DEFIBRILLATION:


 Ventricular Fibrillation (VF) –Asynchronized mode
 Pulseless Ventricular Tarchycardia (VT)- Asynchronized mode.
 Supraventricular tarchycardia (SVT) with shock and no vascular access rapidly available – (use Synchronized mode)
 VT with shock and unresponsiveness but with pulse and no vascular access rapidly available (use synchronized mode)
 Atrial Fibrillation or atrial flutter with shock (use synchronized mode)

Procedure: Attach an AED (Automated external defibrillator) if one is available and follow it’s voice prompts.
Keep doing CPR while the AED is being attached.
Stand clear and don’t touch the patient when the AED shock is being given. Touching the collapsed person while the AED shock is being
given stops it from working properly & puts your safety at risk.
Appropriate Electrical Charge for Countershock
DYSRHYTHMIA MODE CHARGE

Ventricular fibrillation Defibrillation 2 J/Kg , then 4 J/Kg, then 4 J/Kg as

276
Compiled by Obasi. D. C. Chinedu

Ventricular tarchycardia without a (Asynchronized mode) needed.


pulse Then 4 J/Kg after CPR and each dose of
medication

Ventricular tarchycardia with Synchronized mode 0.5-1.0 J/Kg. Repeat as needed.


pulse
SVT; Atrial fibrillation and Atrial
flutter with shock

AED APPLICATION

CONTRAINDICATIONS: Contraindication to CPR:


Newly born, infant, child or adult with effective perfusion (palpable central or peripheral pulse) & Breathing normally.

Contraindication to AED application: Conscious patient with good perfusion

COMPLICATIONS: COMPLICATIONS OF CPR:


- Coronary vessel injury - Diaphragm injury - Hemopericardium
- Hemothorax - Interference with ventilation - Liver injury
- Myocardial injury - Pneumothorax - Rib fractures
- Spleen injury - Sternal fracture

COMPLICATIONS: COMPLICATIONS OF AED:


 Ineffective delivery of countershock because of failure to charge, improper positioning on the chest, incorrect paddle size, or improper
conduction medium
 Burns on the chest wall
 Failure to “clear” before voltage discharge, leading to electrical shock of a team member or bystander
 Rhythm complications – tarchydysrhythmia or bradycardia & heart block
 Myocardial damage or necrosis
 Cardiogenic shock
 Embolic phenomena

FOLLOW UP
 Admit the patient for some period for observation
 Monitor the pulse-rate & rhythm; blood pressure; Oxygen saturation; responsiveness
 Take a good history and some investigations to address any underlying primary health condition
 Counsel caregiver or patient on the situation and see at intervals in the clinic to avoid a repeat occurrence of such cardiopulmonary
arrest.

SAMPLE QUESTIONS
1. Write short note on CPR.
2. Outline the steps involved in CPR and discuss the precautions for effective ventilation and chest compression in different age groups?
3. What is AED? Outline different types available?
Discuss the indications, procedure of application and complications of AED?

277
Compiled by Obasi. D. C. Chinedu

CHD –INCIDENCE, MORBIDITY AND MORTALITY


Dr. Chinawa

Btw. 7-10/1000 live birth. 25-30% born with CHD die in 1st 4wks of life – when early corrective treatment is unavailable. 15% of neonatal
deaths attributable to CHD

 LUTH – Okoromah CA, Ekure EN, Ojo OO, et al. Niger Postgrad Med J 2008; 15: 82 – 85: VSD – 41%, ASD – 20.2%, TOF –
11.8%, AVCD – 7%, PDA – 5.7%
 Benin – Ibadin MO, Sadoh WE, Osarogiagbon WO. Nig J Paediatr 2005; 32: 29 – 32: VSD – 55%, TOF – 28.6%, ASD – 12.8%, PDA –
4.1%

EPIDEMIOLOGY OF CHD: Seasonal variation – none. Residential environment – no effect. Gender: equal for all CCVM. Male at greater
risk for certain defects (TGA, AS, other L. heart defects (CoAo, HLHS, Bicuspid aortic valve, interrupted aortic arch), TOF, Truncus
arteriosus.

Race – Correa-Villansenor et al.(1991) BWIS. Am J Epidemiol 134:394 – 402: Some studies suggest no ethnic differences. Others suggest
predominance of whites in subgroups of defects – Ebstein’s anomaly, AS, CoAo, TGA, PA (possible socio-economic factors.
Prenatal care – presence or lack of ANC is not a factor in risk of CCVM
Environmental risk factors: - prenatal exposure to cold temperature - Maternal exposure to misc. solvents and hair dyes
- Maternal exposure to auto repair works
Drug exposure – diazepam, corticosteroid, phenothiazines, gastrointestinal drugs, paternal exposure to cocaine
Maternal diabetes – TOF, Truncus arteriosus, DORV
Maternal reproductive history: 3 prior pregnancies + genetic risk factors + increased no. of miscarriages increase odds ratio
Previous premature births and induced abortions increase odds ratio in those without genetic risk factors.
Genetic risk factor – pre-occurrence of a CCVM in the family is single largest determinant in BWIS data set.
Syndromic associations:
Marfans syndrome – underlying molecular defect in fibrillin – correlated with pathogenesis of changes in aortic wall & MV
In BWIS 27.7% of all cases had either a chromosomal anomaly, heritable syndrome or additional major organ system defect. –
Down’s syndrome most frequent – 9%
Abnormalities of situs or looping, branchial arch defect
Metabolic genetic syndrome: - Williams syndrome - Holt-Oram syndrome
Possible gene locus may exist with some of the above.

CHD - FURTHER EVALUATION: - Echocardiography - Cardiac catheterization


- MRI - Holter monitoring - Exercise ECG - Electrophysiologic studies

CHD – TREATMENT MODALITIES


MEDICAL: - Anti-failure medication - Pharmacologic closure of PDA - Anti-arrhythmic drug therapy
- Electrical cardioversion of malignant arrhythmias - Anti-coagulation therapy for in-situ prosthetic valves.

INTERVENTIONAL TECHNIQUES: - Device closure of defects – PDA, ASD


- Balloon angioplasty and valvuloplasty - Embolization - Revascularization with stents

VSD: In the normal heart, the right and left chambers are completely separated from each other by a wall of muscle called a septum. The
right atrium is separated from the left atrium by the atrial septum and the right ventricle is separated from the left ventricle by the ventricular
septum. It is normal for all infants to be born with a small opening between the two atria, which usually closes within the first few weeks of
life. Normally there is no opening between the two ventricles, but some infants are born with these called ventricular septal defects.
Ventricular septal defects are among the most common congenital heart defects, occurring in 0.1 to 0.4 percent of all live births and making
up about 20 to 30 percent of congenital heart lesions.
Ventricular septal defects occur in many locations and sizes. The ventricular septum is made up of different types of tissue, with one part
composed of mainly muscle and another part made of thinner, fibrous tissue.
Small ventricular septal defects rarely cause problems. A physician usually discovers these by noticing an extra heart sound called a murmur
on a routine physical exam. This murmur is often not present in the first few days of life.
Therefore, if the small ventricular septal defect does not close, the child should continue to be seen by a cardiologist for occasional checkups.
Large ventricular septal defects can cause problems, often developing gradually in the first few months of life.

Before birth, the pressure on the right side of the heart is equal to pressure on the left side of the heart.
As soon as the baby takes its first breath, the pressure in the lungs and the right side of the heart starts to decrease. This process is slow and
usually takes about 2-4 weeks for the pressure in the lungs to reach normal level
In the first 1 to 2 weeks of life, babies with large ventricular septal defects may do very well. But as the pressure in the right side of the heart
278
Compiled by Obasi. D. C. Chinedu

decreases, blood will start to flow to the path of least resistance (i.e., from the left ventricle through the ventricular septal defect to the right
ventricle and into the lungs). This will gradually lead to symptoms of congestive heart failure and must be treated.
Ventricular septal defects never get bigger and sometimes get smaller or close completely. This is why when a baby is diagnosed with a
ventricular septal defect, most cardiologists will not recommend immediate surgery but will closely observe the baby and try to treat
symptoms of congestive heart failure with medication to allow time to determine if the defect will close on its own.

Ventricular septal defects have a very characteristic murmur(Explain?). However, a murmur is often not heard at birth.

Babies who do have moderate or large ventricular septal defects with excessive blood flow to the lungs will have signs of congestive heart
failure. The most important sign will be the baby's growth. Babies who have significant congestive heart failure will have failure to thrive
and will have difficulty maintaining a normal weight gain in the first few months of life.

80% are located at the membranous part. 20% in muscular or endocardial cushion aspect of the septum.
Swiss cheese defects (Multiple defects in the muscle). Symptoms from 3-6 months

These defects are in a posteroinferior position, anterior to the septal leaflet of the tricuspid valve.
The magnitude of intracardiac shunts is usually described by the Qp : Qs ratio. If the left-to-right shunt is small (Qp : Qs <1.75 : 1),
the cardiac chambers are not appreciably enlarged and the pulmonary vascular bed is probably normal. If the shunt is large (Qp :
Qs >2 : 1), left atrial and ventricular volume overload occurs, as does right ventricular and pulmonary arterial hypertension. The
main pulmonary artery, left atrium, and left ventricle are enlarged.

CLINICAL FEATURES: Depends on the size of VSD. Depends on Pulm to systemic pressure ratio.
Could be asymptomatic. Always as heart failure when pulm resistance falls. Mumur is always PSM (Explain)

INVESTIGATION: - CXR - 2D ECHO - ECG - CATHERTERIZATION


TREATMENT: MEDICAL, SURGICAL.

PDA: Commonest in neonatal period. 5-10% of all congenital heart disease. Associated with an aetiology
Prematurity, rubella, Birth asphyxia.
Pathology: Ductus arteriosus is derived from the six aortic arch. Closes within 3 days of life. When it remains patent, communicates
between the main pulmonary artery and aorta

CLINICAL FEATURES: Similar to VSD.


- Bounding and collapsing pulse - Continuous murmur
- Large p2 in pulmonary hypertension
INVESTIGATION: - CXR - 2 D ECHO
TREATMENT:
Medical: Indomethacin
SURGERY: Surgical ligation and Division

TOF: Commonest cyanotic cardiac disease in neonatal period. Occurs in about 8-10% of all cardiac defects. M=F
4 Components.
ANATOMICAL: Pulmonary stenosis & VSD
PHYSIOLOGICAL: Right ventricular hypertrophy & Overriding of the aorta.

VSD are always large. Located at the membranous part of the septum. Aorta straddles the septum. Receiving blood from both ventricles.

Pulmonary stenosis (PS) is usually infundubular. PS determines the right ventricular hypertrophy. Also the left to right shunt. At times
there may be bidirectional shunt giving rise to PINL TETRALOGY.

CLINICAL FEATURES: - Clubbing - Cyanosis -Ejection systolic murmur


Tet spells: Hypoxia-cyanosis-squatting-convulsion-loss of consciousness and even death (When PS is Severe)
NB Characteristic Systolic murmur is drowned

INVESTIGATION: - CXR= BOOT SHAPED (what is this boot shaped heart?) - ECG= R axial deviation
- 2 D ECHO

TREATMENT: - ABC - Morphine - Knee chest position - Oxygen in metabolic acidosis


- Saline pharesis or plasma pharesis (Formular: Simply 10mils/kg)
Propranolol (MOA? Reduces infundubular spasm)
Surgical: BT(Blalock-Taussig) shunt (Distal end of SA to PA)

279
Compiled by Obasi. D. C. Chinedu

OSTEOMYELITIS
DR KEN ADIELE.

OUTLINE: - Definition/introduction - Epidermiology - Classification - Pathogenesis


- Microbiology - Clinical features - Investigations /diagnosis - Differential diagnosis
- Management (treatment) - Follow up - Chronic osteomyelitis
- Osteomyelitis in children with sickle cell anaemia

EPIDERMIOLOGY: Commoner in children than adults. More in children living in the tropics /developing countries. 50% of cases in under
5 children. 33% of all cases is seen in children less than 2yrs of age. Equal sex predilection in neonates. Beyond neonatal age boys > girls
(2:1) – prepubertal
Mostly in neonates, no precipitating factor seen; but can follow common procedures. In older children – 33% of cases due to preceeding
trauma; other could follow surgical procedures.

CLASSIFICATION
Duration of symptoms: Acute– < 2 weeks, Sub-acute– 2 weeks to 3 months or Chronic– more than 3 months
Histopathologic finding: Acute or Chronic
Source of infecting organism: Haematogenous or Exogenous

PATHOGENESIS: Affects mainly long bones – 50% occur in lower limbs (tibia & fibula); 25% in upper limbs (humerus). Multiple bone
affectation commoner in neonates and children with SCA.
 Metaphysis supplied with loop vessels that end in sinosoids >> sluggish flow + few phagocytes + low o2 tension >> high bacterial
growth (infection) >> inflamation + release of proteolytic enzymes + free oxygen radical + cytokines >> further decrease in oxygen
tension & low ph >> osteolysis + tissue destruction >> oedema + high pressure >>>
 Tissue necrosis + breakdown of bone structure + matrix removal >> suppuration >> pus tracking through the haversian system and
volkmann canal to the subperiostum >> subperiosteal abcess.
 Increasing pressure >> compromise of both periosteal & endosteal blood supply >> bone necrosis (sequestrum) + new bone formation
around (involucrum)

NB: transphyseal vessels seen in neonates and infants >> crossing of infection from metaphysis to epiphysis to the joint space >> septic
arthritis.
Unlike in older children- no transphyseal vessels + avascular growth plate that acts as a barrier

MICROBIOLOGY
Staphylococcus Aureus (MRSA & MSSA): By far the commonest causative organism worldwide.
Respiratory Pathogens like: Streptococcus Pyogenes & S. Pneumoniae.
Haemophilus Influenzae type b: For unknown reason causes more of septic arthritis
Salmonella species: Common cause of Osteomyelitis in developing countries and amongst Children with Sickle cell anaemia
Kingella Kingae: Increasing frequency and mostly seen in children less than 4yrs of age.

CLINICAL FEATURES:
Symptoms: Fever – very high in cases of MRSA. - Bone pain (back pain for spine).
- Limping or inability to walk
Signs: - Visible redness and swelling around the long bones especially the lower limbs
- POSITIVE POINT TENDERNESS. - Pain on rectal examination for sacral involvement
- Calcaneal may be very insidious with delay in seeking medical attention

INVESTIGATIONS /DIAGNOSIS:
 C-Reactive Protein (CRP) - > 20mg per liter
 Procalcitonin level (Both are very sensitive and suitable for diagnosis and follow-up of patient’s treatment)
 Erythrocyte sedimentation rate (ESR) - > 20mm per hour + FBC
 Plain Radiograph (X-ray) taken 2-3 wks after onset of symptoms = shows “rat bite” in bone ie periosteal reaction and rule out
pathological fracture as well as bone malignancies (eg. Ewing’s Sarcoma, Osteosarcoma).

Other imaging studies include:


 Scintigraphy (Radio-isotope bone scan)
 Computed tomography Scan (CT-Scan) – cumbersome and risk of radiation exposure
 MRI – ‘Best’> very sensitive for difficult diagnosis. May require anesthesia in children
 Ultrasound – Minor importance but could pick up fluid in joint space suggestive of Septic arthritis

To determine the causative organism (Pivotal):

280
Compiled by Obasi. D. C. Chinedu

 Blood culture
 Bone aspirate culture – more sensitive with better yield of up to 40%.
NB: K. kingae require special culture method or PCR for it’s isolation and should be actively looked for especially in young children (< 4yrs)

DIFFERENTIAL DIAGNOSIS: - Trauma - Fracture - Bone tumours - Neuroblastoma


- Soft tissue infections eg cellulitis, pyomyosities - Acute leukaemias

MANAGEMENT (TREATMENT): Initial treatment is often empirical using the known epidemiology of causative organism and local
sensitivity pattern, while waiting form culture & sensitivity result and reviewing the patient’s clinical state.
The most relevant antibiotic, must have acceptable side-effect profile when given orally (as large and frequent dosing may be required); good
bone penetration via oral route & effective as monotherapy >> Clindamycin and first generation cephalosporines met these requirements.

Clindamycin and Vancomycin are very effective against MRSA but not for K. Kingae.
Beta-lactam antibiotics (eg Co-Amoxyclav) = effective for K. Kingae & respiratory pathogens – S. pyogene & S. pneumonia.
Amoxycillin & Ampicillin = for beta-lactamase –negative strain of H. influenza type B.
2nd or 3rd generation Cephalosporins = beta-lactamase –positive H. influenza type B.
NB: Consider H. influenza in an unimmunized child <4yrs with bone & joint affectation (Septic arthritis).
For Clindamycin resistance in patients with MRSA, Vancomycin is used and in very rare cases, more expensive linezolid is used for
resistance to Vancomycin.
NB: In a small survey, old & cheap Trimethoprim – Sulfamethoxazole combination (Septrin) was shown to be effective for MRSA –
It’s recommendation still controversial.

For Osteomyelitis due to Salmonella = 3rd generation Cephalosporins like Cefotaxime or Ceftriaxone or Fluoroquinolone. If above drugs are
unaffordable, the cheap and old Chloramphenicol should be used.

Other ancillary treatments include:


 NSAIDs – to control pain & fever
 Anticoagulants
 Steroids = controversial

FOLLOW UP: Monitoring with CRP while on therapy (Both in and out patient)
After therapy, follow up for at least 1 year to monitor for growth disturbances, limb length etc

CHRONIC OSTEOMYELITIS: (7.8% in Gambia- Paediatric surgical admissions; 6.7% in Malawi-all orthopedic procedures). Physical,
Psychological & socioeconomic morbidity
Read: Pathophysiology, Clinical features, Treatment, Complications and Follow-up

OSTEOMYELITIS in Children with Sickle Cell Anaemia: Pathogen – Salmonella & Staph aureus.
Involvement of Diaphysis, small bones, flat bones, multiple symmetrical bone affectation
Distinguish clinically between Osteomyelitis and Vaso-occlusive crisis + FBC, CRP
NB: In acute condition X-ray, bone scan and MRI are unable to differentiate both.
Treat for 6-8 weeks (prolonged).

281
Compiled by Obasi. D. C. Chinedu

ACUTE BRONCHIOLITIS
Dr. ADAEZE C. AYUK

OUTLINE/OBJECTIVES: - BRONCHIOLITIS - What it is; how it is diagnosed


 Clinical features and clinical progression - Management of bronchiolitis
 SOME ASPECTS OF FOREIGN BODY INHALATION

Introduction: Bronchiolitis is a viral infection of the lower respiratory tract. It is one of the most substantial health problem for infants
worldwide. It is a common cause of hospitalization in young children. Associated with chronic respiratory symptoms in adulthood. May be
associated with significant morbidity or mortality.

Definition: Acute infectious inflammation of the bronchioles resulting in wheezing and airways obstruction in children less than 2 years old
Age: Up to 12 months: Onset with an upper respiratory tract infections. First episode of respiratory distress with: crackles and/or
wheezing.
Children <2 years of age: Viral upper respiratory tract infection followed by increased respiratory effort and wheezing. Symptoms:
rhinorrea, cough, wheezing, and respiratory distress

COMMON Organisms involved: Typically caused by viruses


– RSV-most common – Parainfluenza (usually type 3) – Human Metapneumovirus
– Influenza – Rhinovirus – Coronavirus – Human bocavirus
Occasionally associated with Mycoplasma pneumonia infection

Pathophysiology: RSV infects ciliated respiratory epithelial cells by binding of the protein G to receptors. Direct cytotoxic action of the
virus leads to air trapping airway obstruction.

Epidemiology: Peak incidence: between 3 months and 6 months of age. Incubation period - 5 days. Around 70% of all infants will be
infected with RSV or a respiratory virus in their first year of life. Of these, 22% will developed a symptomatic disease. 2-3% will be admitted
to hospital in their first 12 months of life. 2% will require intubation and mechanical ventilation.
Transmission is frequent person to person. Direct contact through big secretion droplets, Touching infected material with the hands.

CLINICAL FEATURES: Begins with upper respiratory tract symptoms/prodrome: nasal congestion, rhinorrhea, mild cough, low-grade
fever.
Progresses in 3-6 days: Rapid respirations, chest retractions, wheezing. Increased work of breathing evidenced by Subcostal, intercostal
and supraclavicular recessions and nasal flaring.
Fever; (High fever uncommon -30% of patients). In the presence of high fever careful evaluation for other causes.
Poor feeding: Many infants have feeding difficulties and it is often the reason for hospital admission

EXAMINATION FINDINGS: Tachypnea: 80-100/min in infants or 30-60/min in older children


- Prolonged expiratory phase of respiration - Rhonchi/wheezes and crackles
- Possible dehydration - Possible cyanosis or apnea

Clinical progression: Usually self-limited. Symptoms may last for weeks but generally back to baseline by 28 days. Symptoms of upper
airway infection: 1-3 days. Peak respiratory symptoms and dehydration: 3-5 days.
Symptoms improve over 2-5 days but wheezing often persists for over a week. Cough resolves in 90% of infants within 3 weeks. Disruption
in feeding and sleeping patterns may persist for 2-4 weeks.
In infants > 6 months, average hospitalization: 3-4 days

Risk factors for severe disease: - Age <3 months - Significant comorbidities
- Prematurity (< 35 weeks gestational age) - Haemodynamically significant congenital heart disease
- Chronic lung disease - Neuromuscular diseases - Immunodeficiency - Atopy
- Social factors - No breast feeding - Parental smoking
- Number of siblings/nursery or day-care attendance - Low Socioeconomic status
Why these Children are at Highest Risk for Severe RSV

282
Compiled by Obasi. D. C. Chinedu

Bronchiolitis Severity Score:

DIAGNOSIS: Clinical diagnosis based on history and physical exam. Supported by CXR showing: hyperinflation, flattened diaphragms, Air
bronchograms, peribronchial cuffing, patchy infiltrates or atelectasis.

Admission criteria
 Oxygen saturation <92% in room air: Hospitalize at SaO2 92-94%; if clinical assessment/phase of the illness/social factors = poor
 Inadequate oral fluid intake (<50-75% of usual volume)
 Severe respiratory distress (grunting, marked chest recession, respiratory rate of more than 70 breaths/min)
 Apnoea (observed or reported)
 Risks factors for severe bronchiolitis present

DIFFERENTIAL DIAGNOSIS: - Viral-triggered asthma - Bronchitis or pneumonia


- Chronic lung disease - Foreign body aspiration - Vascular rings
- Gastroesophageal reflux/dysphagia leading to aspiration - Congenital heart disease or heart failure
- Bronchomalacia - Complete tracheal rings or other anatomical abnormalities

Management of bronchiolitis: - Supportive care - Pharmacologic therapy - Ancillary evaluation


Supportive treatment: Fluid (nasogastric or intravenous). - Suctioning (superficial and not frequent).
Oxygen: high-flow nasal cannula HFNC/CPAP especially if SaO2 <94%. - Feeding

Medical treatment (discontinue if not helpful): Trial with beta2 adrenergic medication (Albuterol or Epinephrine)
Clinical features -significant wheezing, older infants and atopic history
- Trial with hypertonic saline
- OCS – especially in moderate to severe disease and also if difficult to wean off oxygen

Others:
RIBAVIRIN: Not routinely recommended due to modest effectiveness and cost. May be useful in infants with confirmed RSV at risk for
more severe disease. Must be used early in the course of the illness
ANTIBIOTICS: Not useful in routine bronchiolitis. Should be used if there is evidence of concomitant bacterial infection like; Positive
urine culture, Acute otitis media, Consolidation on CXR.

NONSTANDARD THERAPIES
Heliox: Mixture of helium and oxygen that creates less turbulent flow in airways to decrease work of breathing. Only small benefit in limited
patients
Anti-RSV preparations RSV-IGIV or Palivizumab: No improvement in outcomes
Surfactant: May decrease duration of mechanical ventilation or ICU stay

Discharge criteria: - Patient is clinically stable - Patient is taking adequate oral fluids/feeds
- Patient has maintained oxygen saturation over 94% in room air for 4 hours, including a period of sleep

283
Compiled by Obasi. D. C. Chinedu

FOREIGN BODY ASPIRATION


Epidemiology: Sex: boys > girls (2:1). Age <3 years old: 50%. Nuts/peanuts - half of all foreign body aspirations in children.
Choking deaths related to: Toys: 68%, Latex Balloons: 29%, Marble balls: 19%.
Object aspirated known in <40%

Distribution of Foreign Body in airway; Right main bronchus in adults. Higher variability in young children
Larynx 1-5%, Trachea 5-15%, L Main Bronchus 30-35%, R Main Bronchus 30-40%, L Lobar Bronchus 5-15%, R Lobar Bronchus 5-15%.

Pathophysiology: At the larynx or trachea obstruction by foreign object can cause immediate asphyxia and death
Beyond the carina, (depends on the patient's age and physical position at the time of the aspiration) – inevitably causes obstruction.
Obstruction may change position/migrate distally, may cause obstruction at that point. May induce inflammation, edema, cellular infiltration,
ulceration, and granulation tissue formation. Eg Leakage of battery contents (Toxic effect, Pressure necrosis, Electrolytic reaction and
mucosal burn or airway obstruction).

Consequences OF AIRWAY OBSTRUCTION: Distal to the obstruction, there is air trapping leading to
- local emphysema - atelectasis - hypoxic vasoconstriction - post-obstructive pneumonia
- volume loss - necrotizing pneumonia - abscess - suppurative pneumonia
- bronchiectasis
NB: The likelihood of complications increases after 24-48 hours, thus quick removal of the foreign body should be considered as urgent

Diagnosis: through History, Physical Exam, Radiography, Bronchoscopy, CT/Fluoroscopy and/or Tracheotomy.
History of choking (FB aspiration may not be witnessed in children)
Classic history: Choking episode followed by coughing spells

Symptoms and SIGNS: - Cough - Possible blood streak in sputum - Dyspnea/Tachypnea


- Wheezing/Rhonchi (unilateral) - Stridor - Decreased breath sounds (affected side)
- Cyanosis - SpO2/ABG abnormalities - Somnolence
- Recurrent/non-resolving LRTI – if not detected on time

Chest x-ray (Inspiratory/ expiratory film) (AP/LATERAL): Normal in 20- 40% of cases. Most are radiolucent (food particles). Air-trapping
on expiration.
Can see: Atelectasis, Infiltration, Consolidation and/or Hyperinflation

Treatment: Emergency treatment – if history is suggestive of recent FB ingestion or signs of choking noted. If stable – watch and wait
(non-invasive management – SABA, INO2).
- Rigid/flexible bronchoscopy if expertise available. - Chest physiotherapy – best avoided
- Surgical bronchotomy/segmental resection/lobar resection – where indicated.
Antibiotics for secondary infections. Steroids for inflammation. Treat complications

EMERGENCY MANAGEMENT for choking from fb aspiration: Use Heimlich Maneuver.

Rule of thumb: Perform bronchoscopy if more than one of the following is suggestive
- History - Physical Examination - Radiography
NB: Bronchoscopic evaluation is warranted on the basis of a positive history alone (Digoy GP et al. 2008 )

Complications: - Bronchiectasis - Pneumonia/bronchitis - Subcutaneous Emphysema


- Pneumothorax / pneumomediastinum - Granulation tissue and hemorrhage
- Cartilage destruction - Airway compromise - Death
Those related to bronchoscopy procedure

284
Compiled by Obasi. D. C. Chinedu

JUVENILE RHEUMATOID ARTHRITIS (PEDIATRIC RHEUMATOLOGY)


DR. S. N. UWAEZUOKE, MBBS (NIG) FWACP

OUTLINE: - Definition - Classification - Clinical Features


- Diagnostic Steps (Overview) - Management

DEFINITION: An inflammatory disorder (collagen/connective tissue disorder) characterized by chronic synovitis which occurs with extra-
articular manifestations.
The arthritis is characterized by chronic non-suppurative synovitis lasting more than 3 months. The major criterion for its diagnosis is a
persistent (at least 6 wks in duration) arthritis of one or more joints with the exclusion of other causes of arthritis.

DIAGNOSTIC CRITERIA:
Onset under 16 years of age
Minimum duration of 3 months or duration of > 6 weeks (hence synonym- juvenile chronic arthritis)

CLASSIFICATION: - Systemic (still’s disease) - Polyarticular (5 or more joints)


- Pauciarticular or oligoarticular (4 or fewer joints)

CLINICAL FEATURES OF SYSTEMIC JRA (STILL’S DISEASE)


- Intermittent high spiking fever - Generalized joint pains and myalgia
- Usual joints: knee, wrist & ankles - Usual age of onset: less than 5 years (f=m. After 5 years, f > m)
- Maculopapular rash - Lymphadenopathy - Anemia (normocytic, normochromic)
- Weight loss - Pericarditis and pleurisy

LABORATORY FEATURES: - Very high erythrocyte sedimentation rate (>100)


- Polymorhonucleocytosis - IGM rheumatoid factor negative

POLYARTICULAR JRA: F:M = 8:1. Presents at any age. Any joint affected (usually wrists & hands, knees & ankles). Weight loss, fever
and malaise may occur

SUB-CLASSIFICATION OF POLYARTICULAR JRA:


Rheumatoid Factor +VE: ANA-positive in 75%. Late childhood onset. >50% progress to severe chronic arthritis
Rheumatoid Factor -VE: ANA-positive in 25%. Better prognosis. Only 10% develop severe chronic arthritis

PAUCIARTICULAR JRA: COMMONEST TYPE; KNEE & ANKLE JOINTS USUALLY INVOLVED
TYPE I: Young girls. ANA-positive. Iridocyclitis (chronic uveitis) in 50%
TYPE II: Older boys. Usually hip or knee involvement. Ankylosing spondylitis a usual morbidity later. Family history in 50%.
TYPE III: Older girls. Knee joint usually involved. ANA and RHF-negative

See images of the following in material: Swan neck deformity, Knee joint involvement, Poly-articular jra, Severe chronic arthritis from jra,
Mono-articular presentation, Ankylosing spondylitis from JRA PAUCIARTICULAR TYPE II

DIAGNOSTIC STEPS
Historical data and signs- articular and extra-articular manifestations are common in jra. Hence,it is important to distinguish primary joint
disease from non-articular rheumatism as most children with rheumatic disorders present with joint pain or stiffness
True articular pain tends to be diffuse and increases in intensity with motion in all possible directions while pain caused by non-articular
conditions tends to be localized on one side of the joint
Differentiating arthritis from arthralgia is also important. The classical signs of inflammation such as redness, warmth, swelling, and
tenderness characterize arthritis. Only pain is noted in arthralgia
Laboratory evaluation- children with JRA are usually investigated using haematolgy, immunology and imaging tests

HAEMATOLOGY TESTS: - Complete(full)blood count- leucocytosis with shift to the left of neutrophils
- Anemia of chronic diseases
- Erythrocyte sedimentation rate(esr)- valuable as an index of disease activity and the success of treatment, although it is a nonspecific test
signifying an inflammatory process

IMMUNOLOGICAL TESTS:
- Anti-nuclear antibodies(ANA)- made up of circulating gammaglobulins directed against several known and unknown antigens. Positive in
285
Compiled by Obasi. D. C. Chinedu

sle,dermatomyositis, scleroderma and other non-rheumatic conditions


- Anti-double stranded dna (ds DNA)- highly specific for SLE. Positive in 40-50% of SLE patients and is almost negative in other
conditions.
- Rheumatoid factors- these are abnormal antibodies which react with immunoglobulin g. Found in systemic lupus erythematosus,
scleroderma, dermatomyositis and JRA.

IMAGING STUDIES: Plain x-ray of the joints-useful for prognosis and follow-up treatment (for instance chronic joint disease is a
complication of systemic onset jra and poly-articular jra).
- Echocardiography may be useful in detecting pericarditis

MANAGEMENT OF JRA
Drug therapy-aspirin and other NSAIDS (naproxen, ibuprofen).
Alternatives include gold salts, d-penicillamine and hydroxychloroquine
Other anti-inflammatory drugs: steroids (prednisolone/ACTH), immunosuppressives

General measures: patient/parental counselling


Adequate nutrition
Balanced rest and physical exercise
Psychotherapy/psychosocial support
Physiotherapy
Occupational therapy (vocational counselling)
4 –monthly opthalmologic evaluation for young girls with pauci-articular JRA (ANA +ve) to look for early iridocyclitis

DIFFERENTIAL DIAGNOSIS
 SEPTIC ARTHRITS
 SLE
 HEMOPHILIA
 RHEUMATIC FEVER
 SARCOIDOSIS
 DERMATOMYOSITIS

PROGNOSIS: 70% achieve normal function. 50% resolve completely. Mortality from infection or amyloidosis occur in 2-7%

286
Compiled by Obasi. D. C. Chinedu

THE UNCONSCIOUS CHILD- Dr. B. O. Edelu

287
Compiled by Obasi. D. C. Chinedu

288
Compiled by Obasi. D. C. Chinedu

289
Compiled by Obasi. D. C. Chinedu

290
Compiled by Obasi. D. C. Chinedu

291
Compiled by Obasi. D. C. Chinedu

292
Compiled by Obasi. D. C. Chinedu

293
Compiled by Obasi. D. C. Chinedu

294
Compiled by Obasi. D. C. Chinedu

VIRAL HAEMORRHAGIC FEVER


Dr. Ujunwa

Introduction: This are a group of severe febrile illnesses caused by RNA Families that are associated with bleeding.

Ebola VIRUS DISEASE: The Deadly Virus

Introduction: Ebola virus disease (EVD) is acute viral haemorrhagic fever. It is caused by infection with a virus of the genus Ebolavirus.
One of the most virulent viral diseases known to humankind. It is a severe, often fatal disease in humans and nonhuman primates (such as
monkeys, gorillas, and chimpanzees). Most fatal viral hemorrhagic fever.

EVD outbreaks have a case fatality rate of up to 25-90%. These outbreaks occur primarily in remote villages in Central and West Africa,
near tropical rainforests. The virus is transmitted to people from wild animals and spreads in the human population through human-to-human
transmission. Fruit bats are considered to be the natural host of the Ebola virus.

Natural Host: Fruit Bat species: - Hypsignathus monstrosus - Epomops franqueti - Myonyteris torquata

Historical perspective: First appeared in Africa 1976. In Nzara Sudan and Yambuku in DRC.
“African Hemorrhagic Fever”: acute, mostly fatal disease. causes blood vessel “bursting”. systemic ( affects all organs/tissues) as well as
humans and nonhuman primates.

WHO Update 1st August 2014: From March 2014 when the current outbreak started in Guinea it has spread 3 other West African countries.
As of 4th August 2014 a total of 1711 cases has been attributed to EVD including 932 deaths. Between 2 and 4 August 2014, 108 new cases
and 45 deaths were recorded. Sierre Lone with a total of 691 cases has the highest number of cases. Mortality EVD in Guinea stands at 363

Ebola Taxonomy: Scientific Classification: Order: Mononegavirales Family: Filoviridae


Genus: Ebola like viruses Species: Ebola
Genus Ebolavirus is 1 of 3 members of the Filoviridae family. Genus Ebolavirus comprises 5 distinct species:
- Bundibugyo ebolavirus (BDBV) - Zaire ebolavirus (EBOV) - Reston ebolavirus (RESTV)
- Sudan ebolavirus (SUDV) - Taï Forest ebolavirus (TAFV).
BDBV, EBOV, and SUDV have been associated with large EVD outbreaks in Africa, whereas RESTV and TAFV have not. The RESTV
species, found in Philippines and the People’s Republic of China, can infect humans, but no illness or death in humans from this species has
been reported to date

Ebola virus is filamentous molecule about 800 to 14000 nm long. It has a uniform diameter of 80 nm. The virus contains a helical
nucleocapsid and is enveloped by a helical capsid. Each virion contains a single-stranded, non-segmented, negative-sense viral RNA.

Transmission of Ebola Virus via:


Direct contact: Blood, saliva, vomitus, sweat, stool, urine
Unsterilized equipments
Burial ceremonies
Handling of infected forest antelope, gorillas, bats
Airborne transmission: limited evidence of human-human

Men who have recovered from the disease can still transmit the virus through their semen for up to 7 weeks after recovery from illness

PATHOGENESIS: Ebola virus enters the host cell and replicates in the cytoplasm within 6 hours. It targets host immune defense system
and leads to severe immunosuppression. It also targets blood coagulation system
Infectious dose: 1 to 10 organisms are sufficient to cause infection in humans. Incubation period is 2 to 21 days.

Clinical features: • Fever • Headache • Joint and muscle aches • Weakness • Diarrhea • Vomiting • Stomach pain
• Lack of appetite • Rash • Red Eyes • Hiccups • Cough • Sore throat • Chest pain • Difficulty breathing • Difficulty swallowing • Internal
and external bleeding

As the disease progresses it can cause: - Multiple organ failure - Severe bleeding - Jaundice - Delirium - Seizures
- Shock - Coma

Complications: - Asymmetric arthralgias - Headache - Amenorrhea - Hearing loss - Orchitis


- Parotitis - Bulimia

Differential diagnosis: - Malaria - Typhoid fever - Shigellosis - Cholera - Leptospirosis


- Streptococcal pharyngitis - Bacterial conjunctivitis - Relapsing fever - Hepatitis
- Other viral hemorrhagic fevers
295
Compiled by Obasi. D. C. Chinedu

Alert: Anyone presenting with fever and signs of bleeding such as:
. Bleeding of the gums . Bleeding from the nose . Red eyes . Bleeding into the skin
. Bloody or dark stools . Vomiting blood . Other unexplained signs of bleeding
Whether or not there is a history of contact with a suspected case of EVD.

Suspected: Anyone living or deceased with a history of fever AND 3 of the following symptoms:
. Headache . Vomiting . Loss of appetite . Diarrhea . Weakness or severe fatigue
. Abdominal pain . Generalized muscle or joint pain . Difficulty swallowing
. Difficulty breathing . Hiccups

Probable: Anyone living or deceased with:


. Contact with a suspected case of EVD and . A history of fever, with or without signs of bleeding

Confirmed: Any patient who met the clinical case definition confirmed by Antigen, IgM, IgG, Culture, or PCR-positive laboratory results.

Diagnosis: Diagnosis of EVD is difficult due to nonspecific features, Many differential diagnosis and High risk of spread.

Investigations: - Antigen capture ELISA - IgM and IgG ELISA - Polymerase chain reaction
- Culture - Immunohistochemistry
Full blood count: Leucopenia, Lymphopenia, Neutrophilia, Thrombocytopenia.
Liver function test: Elevated AST and ALT, Elevated bilirubin,
Blood Urea Nitrogen: Elevated creatinine, Elevated urea

Treatment: No Standard Treatment available. Patients receive supportive therapy via:


- Isolation and barrier nursing - Maintaining fluids and electrolytes balance
- Maintaining oxygen status and blood pressure - Adequate nutrition
- Treat complicating infections

Prevention: - Public enlightenment - Setting up of designated hospitals - Patient isolation


- Rapid investigation - Barrier nursing - Use of personal protective equipment
- Infection-control Measures (complete equipment and area sterilization, Use of universal precaution including hand washing)
- Strict follow up and quarantine of contacts - Medical staff training
- Quick and appropriate disposal of corpse - Reduce contacts with high-risk infected animals
- Avoid area of known outbreaks

LASSA FEVER:
Lassa fever, also known as Lassa hemorrhagic fever (LHF). A type of viral hemorrhagic fever caused by the Lassa virus. Many of those
infected by the virus do not develop symptoms

EPIDEMIOLOGY: Around 300,000 people are infected annually, with up to 5,000 deaths per year. It is relatively common in parts of West
Africa where the multimammate rat is common. Particularly Guinea, Liberia, Nigeria and Sierra Leone.

MULTIMAMMATE RAT: common African rodents of the genus Mastomys that have 12 rather than the usual five or six mammae on each
side

AETIOLOGY: Lassa virus specifically spreads to humans from a rodent. This is called natal multimammate mouse (Mastomys natalensis).
In these rodents, infection is in a persistent asymptomatic state. The virus is probably transmitted by contact with the feces or urine of
animals accessing grain stores in residences.
Humans usually become infected with Lassa virus from exposure to excreta of infected Mastomys via direct exposure (to the excreta) of rat.
Lassavirus may also be spread between humans through direct contact with the blood, urine, faeces, or other bodily secretions of a person
with Lassa Fever. Infection in humans typically occurs by exposure to animal excrement through the respiratory or gastrointestinal tracts.
Inhalation of tiny particles of infectious material (aerosol) is believed to be the most significant means of exposure. (???)
Virus may be spread by contaminated medical equipment, such as re-used needles. Sexual transmission of Lassa virus has been
reported. Incubation Period is usually about 10 days but can range from 6-21 days.

CASE DEFINITION
a) Suspected case- Illnesses with onset of Fever, with or without sore throat and at least one of the following signs: bloody diarrhea,
bleeding from gums, bleeding into skin (purpura), bleeding into eyes and urine.
b) Confirmed case- A suspected case with laboratory confirmation (positive IgM antibody or viral isolation) or epidemiological link to
confirm cases or outbreaks.
296
Compiled by Obasi. D. C. Chinedu

The virus is still present in the urine for between three and nine weeks. After infection, and it can be transmitted in semen for up to three
months after becoming infected.

Gastrointestinal tract: Nausea, Vomiting (bloody), Diarrhea (bloody), Stomach ache, Constipation, Dysphagia (difficulty swallowing),
Hepatitis.
Cardiovascular system: Pericarditis, Hypertension, Hypotension, Tachycardia (abnormally high heart rate)
Respiratory tract: Cough, Chest pain, Dyspnoea, Pharyngitis, Pleuritis
Nervous system: Encephalitis, Meningitis, Unilateral or bilateral hearing deficit, Seizures.

Lassa fever infections are difficult to distinguish from other viral hemorrhagic fevers such as Ebola and Marburg and from more
common febrile illnesses such as malaria. The virus is excreted in urine for 3–9 weeks and in semen for three months.

Marburg virus is a hemorrhagic fever virus of the Filoviridae family. A member of the species Marburg marburgvirus,
genus Marburgvirus. Marburg virus (MARV) causes Marburg virus disease in humans and nonhuman primates, a form of viral hemorrhagic
fever

TRANSMISSION via: Exposure to urine or faeces of infected Mastomys rats.


Direct contact with the blood, urine, faeces, or other bodily secretions of a person infected with Lassa fever.
Nosocomial: contaminated medical equipment, such as re-used needles.
Sexual transmission of Lassa virus has been reported.

DIAGNOSIS: Clinical diagnosis is often difficult, especially early in the course of the disease. Definitive diagnosis requires testing that is
available only in reference laboratories through:
 Reverse transcriptase polymerase chain reaction (RT-PCR) assay
 Antibody enzyme-linked immunosorbent assay (ELISA)
 Antigen detection tests
 Virus isolation by cell culture.
TREATMENT: The only known specific treatment for Lassa Fever is Ribavirin. Effective if given within the first six days of illness. It
should be given intravenously for ten (10) days. Intravenous Ribavirin treatment should start as soon as a diagnosis of Lassa Fever is made.
Give a single “loading dose” of 33mg per Kg body weight. Then give a dose of 16mg/ Kg body weight every 6 hours for 4 days. Then give
8mg/Kg every 8 hours for 6 days.
Complete treatment course for individual patient clearly laying out the correct amount to give each dose. Once started, a Ribavirin treatment
should not be discontinued until the 10 days course is complete.

Supportive treatment should include; Paracetamol, Vitamin K (Phytamenadione), Heamacel, Ringers lactate, Quinine lactate, Quinine
injection, and antibiotics- start by I.V.
If patient is severely anaemic, consider transfusion.
Prophylaxis: Tablets: Adults 500mg every 6 hours X 5 days
Children > 5 years: 500mg every 12 hours X 5 days
Adverse Effects of Ribavirin: Major side-effect is a reversible, most-often mild anemia. Women who are pregnant or planning to become
pregnant are advised not to take Ribavirin

Prevention of Lassa Fever: High Index of suspicion for Lassa Fever should be entertained by health workers to improve case detection.
UNIVERSAL PRECAUTION must be practiced in the management of suspected or confirmed cases as listed below:
 Appropriate personal protective clothings (hand gloves, facemasks, eye goggles and overall) must be worn when attending to cases.
 Hands must be washed after each with patient or contaminated materials. They must first be rinsed in disinfectant and then washed
with soap and water.
 Instruments and Dressings – Each patient must have an individual thermometer labelled with the patient’s name and kept in a receptacle
containing disinfectant.
 The stethoscope and the sleeve of the syphgmomanometer must be decontaminated between each use by rinsing them in disinfectant
solution.
 Bed covering – The use of a plastic sheet is essential to avoid the contamination of mattresses. They must be large enough to cover the
entire mattress, be waterproof, and be thoroughly disinfected after the discharge or the death of patients.

297
Compiled by Obasi. D. C. Chinedu

HELMINTHIASIS AND OTHER PARASITIC INFECTIONS


DR KEN ADIELE.

Outline: - Definition / Introduction - Epidemiology - Classifications


- Parasites of the Intestines - Parasites of the liver - Other parasitic infections
- Conclusion

Definition/Introduction: Helminthiasis, also known as worm infection, is any macro parasite disease of humans and other animals in
which a part of the body is infected with parasitic worms, known as helminths.
Has a heavy burden especially in school-age children of West African subregion and the tropics generally. A group of Helminths were
targeted in a project launched by some world leading Pharmaceutical companies and NGOs in 2012 known as “London Declaration on
Neglected Tropical Diseases” aimed to control and eradicate some of these worm infections & others.

Epidemiology: A quarter of the human population worldwide estimated by WHO as 2billion & another source 3.5 billion people are infected
with soil-transmitted helminths. Breakdown of the figure shows 249 million with schistosomiasis, 56 million people with food-borne
trematodiasis, 120 million with lymphatic filariasis, 37 million people with onchocerciasis, and 1 million people with echinococcosis.
In 2014, only 148 people were reported worldwide to have dracunculiasis (Guinea worm) due to successful eradication campaign for that
particular helminth, which is easier to eradicate than other helminths as it is transmitted only by drinking contaminated water which was
squarely addressed.
School-age children most vulnerable due to high mobility and lower standard of hygiene. Most children from developing nations will have at
least one infestation. Multi-species infections are very common.

Classifications
A. Can be classified in terms of mode of transmission as:
Soil-transmitted helminths, including Ascaris lumbricoides (the most common worldwide), Trichuris trichiura, Necator
americanus, Strongyloides stercoralis and Ancylostoma duodenale
Hymenolepis nana, Taenia saginata, Enterobius, Fasciola hepatica, Schistosoma mansoni, Toxocara canis
Toxocara cati
Feco-oral helminths (Flukes)
Infected drinking water helminth (Dracunculiasis)

B. In terms of body morphology:


Roundworm infection (nematodiasis): Filariasis (Wuchereria bancrofti, Brugia malayi infection)
Onchocerciasis (Onchocerca volvulus infection)
Some Soil-transmitted helminthiasis – this includes ascariasis (Ascaris lumbricoides infection), trichuriasis (Trichuris infection),
and hookworm infection (includes necatoriasis and Ancylostoma duodenale infection)
Trichostrongyliasis (Trichostrongylus spp. infection), Dracunculiasis guinea worm infection)
Baylisascaris (raccoon roundworm, may be transmitted to pets livestock and humans)

Tapeworm infection (cestodiasis): Echinococcosis (Echinococcus infection)


Hymenolepiasis (Hymenolepis infection) Taeniasis/cysticercosis (Taenia infection)
Coenurosis (T. multiceps, T. serialis, T. glomerata, and T. brauni infection)

Trematode infection (trematodiasis – Flukes): Amphistomiasis (amphistomes infection)


Clonorchiasis (Clonorchis sinensis infection) Fascioliasis (Fasciola infection)
Fasciolopsiasis (Fasciolopsis buski infection) Opisthorchiasis (Opisthorchis infection)
Paragonimiasis (Paragonimus infection) Schistosomiasis/bilharziasis (Schistosoma infection)

Acanthocephala infection: Moniliformis infection

C. In terms of the part of the human body they infect:


- Parasites of the Intestines (small & large)
- Parasites of the Liver
- Other site parasitic infections
Parasites of the Intestines: The small intestine plays host to a wide range of parasites. Whether or not this causes symptoms depends on:
 The virulence, load & multiplicity of the parasite
 The host immunity (determined by the nutritional status & intercurrent infections e.g HIV/AIDs, TB etc)
 The environment: e.g poor water supply, poor sanitation, poor living condition, local habits of allowing children play barefoot in the
sand.

Symptoms: Vague and includes- abdominal pain, diarrhoea, nausea, weight loss, flatulence, dysentery sometimes , purities ani, expelling
worm in stool & intestinal obstruction /distention. May also be asymptomatic.
Diagnosis: by Demonstration of the egg or larva in stool sample. Sensitive & Specific parasite antigen test in stool eg. ELISA & Fluorescein
298
Compiled by Obasi. D. C. Chinedu

labelled monoclonal test especially for G. lamblia.

ASCARIASIS: Mode of transmission :– Ingestion & inhalation of eggs


 Clinical features: - Biliary obstruction with jaundice; Intestinal obstruction; wheeze; Appendicitis
 Diagnosis:- Egg in stool, worm vomitted or expelled in stool
 Treatment:- Piperazine; Thiabendazole

2. Gardiasis:
Mode of transmission:– Ingestion of contaminated food, drink & water; Orogenital contact; person to person
Clinical features: - Asymptomatic; anorexia; nausea; abdominal cramps; explosive foul smelling flatulence; bulky foul smelling stool with
undigested food, steatorrhoea; malabsorption; weight loss; bronchial asthma
Diagnosis:- Demonstration of parasite in stool, duodenum & other histologic materials
Treatment:- Metronidazole

3. Strongyloides:
Mode of transmission :– Skin penetration from soil contact; accidental innoculation; ano-oral; person-to-person
Clinical features:- Heavy parasite burden in intestine; flatulence; abd pain; foul watery stool; puritis ani;
NB: Wheeze; dyspnoea; urticarial pruritis
Diagnosis:- Demonstration in formalin-concentrated stool
Treatment:- Thiabendazole, Benzimidazole & Ivermectin

4. CRYPTOSPORIDIUM: Mode of transmission :– Faeco-oral; person-to-person contact; auto-infection


Clinical features: - Chronic diarrhoea; malabsorption; weight loss
Diagnosis:- Oocytes in stool & duodenal biopsy material
Treatment:- Fluid replacement; Spiramycin on trial.

Parasites of the liver: Parasites of the liver either inhabit the liver, pass through the liver in course of normal development, or carried to the
liver for destruction. Extent of the disease depends on the type of parasite (Virulence); the burden and stage of infestation; and the host
immunity.
Liver disease by the parasites could result in the following pathological changes: reticuloendothelial hyperplasia (Malaria); bile duct
obstruction (Ascaris); granuloma & periportal fibrosis (Schistosomiasis); necrosis or cyst formation (Amoeba)

1. MALARIA: This has 4 species that infect man and the liver (P. falciparium, P. ovale; P. vivax & P. malariae). Liver involvement could
vary from minimal derangement in liver function to hepatic encephalopathy
Mode of transmission :– Inoculation by mosquito
Clinical features: - Fever, tender hepatomegaly, ± splenomegaly; ± Jundice; sequestration of parasite infested RBCs shows characteristic
histological changes = fatty change, hyperplastic kupffer cells, portal tract inflammation, bile duct proliferation, sinusoidal congestion &
haemozoin deposition.

Diagnosis:- Involves demonstration of the parasite; at least 3 fold rise in transaminases in 2 serial samples taken 24hrs apart ± conjugated
hyperbilirubinaemia; absence of clinical/ serological evidence of viral hepatitis & response to antimalarial drugs or histological changes in
malaria on autiopsy
Treatment:- Standard ACT and supportive care

2. AMOEBIC LIVER ABSCESS: Caused by Entamoeba histolytica. Exist in 2 forms – cyst stage (Infective) & trophozoite stage (causes
invasive disease). Commonest extra intestine manifestation is Amoebic liver abscess. This is more common in adults (10-50%) than children
(1-5%). The abscess cavity comprise of central area of hepatic cell necrosis surrounded by a ring of congested liver tissue. Could vary from
pinpoint in size to up to replacing up to 80% of normal liver parenchyma. May be solitary or multiple. Right lobe of the liver more
commonly involved.
Mode of transmission: Ingestion of contaminated food & water
Clinical features: - Insidious onset of symptoms, fever is the most common symptom in 99% of cases; Hepatomegaly (18-63%) with point
tenderness over the liver below the rib or in the intercoastal space; past history of dysentery; rarely jaundice (< 10% of cases); pulmonary
symptom (mainly cough & pluritic chest pain ) in 18-26% of cases
Diagnosis:- Need to differentiate from Pyogenic liver abscess and hydatid cyst of the liver. Chest radiograph shows typical elevation and
hypomotility of the right dome of the diaphragm; aspirate- thick pus (anchovy paste) that rarely yield the parasite; elevated titres of indirect
hemagglutination (IHA) in the presence of clinical ± characteristic ultrasound or CT-scan findings to rule out old from recent infection
Complication:- Rupture of the abscess to the peritoneum (left lobe) or to the thorax (Upper right lobe) giving pleural & pericardial effusion
Treatment:- Medical – Metronidazole or Thinidazole; rarely surgery - in failure of medical mgt or in large abscess (> 5-7cm) at site with
potential to rupture

3. Schistosomiasis: Prevalent in tropical and sub-tropical areas with high burden of poor communities, unsafe drinking water and poor
sanitary conditions. Caused by a trematode blood fluke of the genus Schistosoma & family Schistosomatidae. Six species infect humans
known as; Schistosoma haematobium, S. mansoni, S. japonicum, S. intercalatum, S. malayensis and S. mekongi. Apart from S. haematobium
299
Compiled by Obasi. D. C. Chinedu

which very rarely do, others cause infection of the intestine and liver. Chronic infection result in significant morbidity and mortality from
granuloma formation in the organs
Mode of transmission :– Skin penetration by infective larvae (cercariae) released by fresh water snail, after human contamination of fresh
water source.
Pathogenesis:- The degree of infection depends on the parasite (virulence, dose & duration) and the host immunity factors. May take
between 3 months to as long as 5-15 yrs for hepatic lesions to develop following exposure to the worm.
Eggs of specie eg S. mansoni from gut enter the veins and drained via the portal vein to the microvasculature of the liver. This cause
moderate type 1 HSR >> Type 4 HSR – recruitment of eosinophils, granuloma formation and fibrogenesis of the liver >> marked portal &
perilobular fibrosis & angiogenesis >> severe portal fibrosis >> greatly enlarged fibrotic portal tract (symmers pipe stem fibrosis) >> Portal
hypertension >> bleeding oesophageal varices. NB: Normal liver cell & lobular architecture is preserved.
Clinical features: - May be asymptomatic; abdominal pain ; Jaundice ; fever
Diagnosis:- Microscopy to identify the characteristic egg; serology using PCR for Schistosoma antigen confirms infection at any stage.
Complication:- Post sinusoidal cirrhosis characterized by extensive scarring, centrizonal necrosis & regeneration nodules(possible at 5yrs of
age)
Treatment:- Praziquantel (single dose ± repeat dose); in highly endemic regions, institute periodic mass retreatment (PMR) to limit the
burden.

4. Ascariasis: This is the most common helminthic infection in humans. Adult worm found in the SI has high predisposition to explore
small openings. Sometimes (rarely) they migrate into the bile & pancreatic ducts. Hepatobiliary ascariasis result in biliary tract obstruction.
Eggs laid in the biliary tree & degenerating adult worms result in granulomatous reaction
Clinical features: Non-colicky abdominal pain; Jaundice (20% of cases); fever; nausea & vomiting –less frequent.
Diagnosis: Abdominal ultrasound – xteristic tubular, echogenic non-shadowing structures, with thin, longitudinal, central sonolucent line.
Worm movement could be observed. ERCP = useful for both diagnostic & therapeutic use. The worms appear as smooth, linear filling
defects with tapering ends in the ducts.
Treatment: Mainly conservative (antihelmintics,antibiotics & antispasmodics); endoscopic extraction and surgery

5. Hydatid cyst of the liver: Endemic in Mediterranean, Middle East & North Africa. Caused by Enchinococcus granulosus (Dog
tapeworm). Affect lung & liver in children. Present with Fever, abdominal masses and hepatomegaly
Treatment - Surgery

6. Liver flukes
Hepatobiliary flukes include – Fasciola hepatica; F. gigantica; Opisthorchis viverrini, O. felineus & Clonorchis sinensis
Endemic & of public health importance in East Asia, East Europe, Africa & Latin America
Mode of transmission: Eating of undercooked or raw fish & other aquatic food
Clinical features: Abd pain, liver enlargement & more importantly, cholangiocarcinoma
Treatment: Triclabendazole & Praziquantel

Other parasitic infections:


 Lung flukes
 Brain flukes- with CNS manifestations

Conclusion: Helminthiasis & it’s control is of public health importance. Mass deworming programmes initiated by WHO are important in
endemic regions especially in school age children could improve school attendance by 25% + other benefits.

300
Compiled by Obasi. D. C. Chinedu

ALLERGIC DISORDERS
Dr. Eze JN

Learning Objectives: - To list the common allergic disorders in childhood.


- To know the epidemiology and aetio-pathogenesis, modes of presentation and possible complications of common allergic disorders
(allergic asthma, allergic rhinitis/rhinosinusitis, atopic/contact dermatitis, insect bite allergies, allergic conjunctivitis, urticaria/angioedema;
anaphylaxis)
- To be able to diagnose each of these conditions and their morbidities and possibly identify their trigger
- Be able to effectively treat and minimize complications of any allergic disorders

Outline: - Introduction - Epidemiology - Aetio-pathogenesis - Clinical features - Case review


- Diagnosis - Complications - Treatment/Follow-up - Prevention - References

Introduction: Increasing prevalence of allergic diseases have been documented worldwide in both developed and developing countries.
Notable allergic disorders include allergic asthma; rhinitis/rhinosinusitis; anaphylaxis; drug, food, and insect bite allergy; atopic/contact
dermatitis (eczema); and urticaria (hives) and angioedema.

Epidemiology & Aetiopathogenesis: Allergic disorders are heterogeneous and arise from gene-environmental interactions. Human genetics
accounts for increase susceptibility for allergic disease, phenotypes and sub-phenotypes, natural history, severity, & response to treatments.
Phosphorylation and methylation and a large number of micro RNAs, among other factors explain a proportion of the gene-environmental
interactions. Sensitization (IgE antibodies) to foreign proteins in the environment plays an important role in disease manifestation in asthma,
rhinitis and atopic eczema. These are proteins derived from pollens, molds, dust mites and cockroaches. Indoor and outdoor pollution have
been implicated in increased prevalence of asthma and allergic diseases. AR is also a risk factor for asthma
NB: Hygiene Theory

Let’s take closer look:


Case review: A 6 year old girl, first diagnosed with asthma at age of 3 years. Had an average of one exacerbation per month for the first
year. Progressed to an average of 2 to 3 exacerbation episodes every month by the second year. She also had concomitant allergic rhinitis and
conjunctivitis.
Medications include low dose inhaled corticosteroid/long acting β agonist (ICS/LABA) combination, leukotriene receptor antagonist (LTRA)
and short acting β agonist (SABA) as needed. Also on Oral antihistamine when needed

ASTHMA: Asthma is a chronic inflammatory airway disease that results in airway hyper-responsiveness to noxious stimuli; leading to
bronchoconstriction.
Atopy- the genetic predisposition to develop IgE- mediated sensitivity to common aeroallergens is the strongest identifiable predisposing
factor for asthma in children. Symptoms vary over
time and in intensity (wheeze, shortness of breath,
cough and chest tightness/chest pain) and may
reverse spontaneously or following treatment
Allergic asthma is associated with a past and/or
family history of allergic disease (eczema, allergic
rhinitis, or food or drug allergy. It is characterized by
eosinophilic airway inflammation and responds to
inhaled corticosteroid (ICS) treatment.

Allergic Rhinitis: Rhinitis is the inflammation of the


nasal membranes commonly caused by Aeroallergens.
Common; up to 40% of children; occurs in all ages,
onset usually before age 20. Occurs in all races;
highest prevalence in Africa and Latin America. Boys
more affected than girls
Pathophysiology of AR: IgE- Mediated
inflammatory response. 3 notable phases:
- Sensitization phase - The early response (4-8
hours) - Late phase response (hours to days)

Clinical Features: AR is characterized by a symptom complex which may be acute or chronic. May cause significant morbidity, not life
threatening. Usually co-exists with asthma
History: Sneezing; itching (nose, eyes, throat and ears); nasal discharge; congestion; anosmia; red itchy eyes; eye swelling, tearing, facial
pain, ear pain
Systemic symptoms- headache, fatigue; drowsiness; malaise; irritability; weakness. reduced appetite. difficulty concentrating
Other aspects of history- Possible triggers; response to medications; co-morbid conditions; environmental/occupational (indoor/outdoor)
exposures; family history of allergies; effect on quality of life
301
Compiled by Obasi. D. C. Chinedu

Important Co-morbidities- sinusitis, nasal polyposis, conjunctivitis, otitis media with effusion, upper respiratory infections, and sleep
disorders

Signs: - Nasal crease - Dennie-Morgan lines - Allergic Shiners - Conjunctival Injection


- Oropharyngeal cobblestoning - Dental malocclusion (overbite) - High arched Palate
- Postnasal drip - Boggy nasal and pale nasal turbinates - Deviation/Perforation of the nasal septum

Diagnosis: Clinical. Can be confirmed by any of the following;


- Allergic skin tests - Radioallergosorbent test (RAST) - Nasal provocation (allergen challenge)
- Total serum IgE; Total blood Eosinophil count - Radiography- Plain X-ray: Sinuses, Lateral neck; CT/MRI

Differential Diagnosis: - Bacterial Sinusitis - Vasomotor rhinitis

Complications/Co-morbidites: - Acute or chronic sinusitis - Otitis media - Sleep disturbance or apnoea


- Dental problems (overbite) - Palatal abnormalities - Eustachian tube dysfunction
- Nasal polyps - Asthma exacerbation

Treatment of AR: Environmental control measures (Avoid known allergies; 1-2 weekly linen laundering). Counseling and follow-up
Pharmacotherapy: Oral or intra nasal antihistamines (Second gen. - Loratidine, Cetrizine preferable)
- Nasal decongestants –pseudoephedrine (Drawback: Rebound congestion after 3-5 days of use)
- Intranasal steroid- such as Fluticasone nasal spray - Intranasal cromolyn
- Intranasal anticholinergic – Ipratropium - Short course oral steroid - LRTA – Montelukast
Adjunctive/palliative therapy: - Nasal lavage with warm saline - Saline spray/drop
- Inhalation of warm mist
Immunotherapy: - (Desensitization)- Repeated high dose allergen shots
- Recombinant humanized monoclonal anti IgE antibody- Omalizumab

ATOPIC/CONTACT DERMATITIS
Case review: A 15yr old male, first seen in 2014 with symptoms in keeping with asthma and related atopic diseases (AR and AD). Had
extensive itchy hyper-pigmented maculopapular rash (limbs, face, neck) with flexural lichenification and Dennie Morgan's fold. Aggravated
by contact with grass, leather materials.
Initial management: 5% SSA ointment, aqueous cream, cetirizine and Dove soap. Asthma symptoms were controlled but AD continued to
worsen over the years with secondary bacteria infection.
Other medications- Oral antibiotics; twice daily soaking of feet in 1:20,000 dilution of potassium permanganate.
Symptoms have continued to wax and wane over these years. Patient advised to use drops of olive oil in bath water
Use of immuno-modulators being considered

Introducing AD: Atopic dermatitis is a chronic, inflammatory skin disorder; common in children. AD reflects cutaneous manifestation of a
systemic disorder. Is often the initial step in the “allergic march” (the sequential development of allergic disease manifestation in early
childhood).

Pathophysiology of AD: Complex & multifactorial- breach in continuity/barrier of the skin and immune dysregulation are implicated. They
include: Skin barrier dysfunction, Alteration in cell mediated immune responses, IgE mediated hypersensitivity, Environmental
factors and infectious agents.
Mutations within the FILAGGRIN Gene, transglutaminase, keratins and intercellular proteins which encodes a structural protein essential for
skin barrier formation are the basis for the skin barrier abnormalities.
Altered epidermal lipids (ceramide) and antimicrobial peptides such cathelicidins are associated with staphylococcal colonization of the skin
due to trans epidermal water loss, pH alteration and dryness of the skin.
Defective innate immune responses also contribute to increased bacterial and viral infection in the patient.
This interplay of factors leads to T-cell responses in the skin. Calcineurin is a T-cell activator. Effect of Th2-generated cytokines further
aggravate atopic dermatitis

Acquired pH-dependent increases in serine protease activity likely accounts for the precipitation of atopic dermatitis following the use of
neutral-to-alkaline soaps. Prolonged exposure to a reduced environmental humidity, as occurs in radiant-heated homes in temperate climates
during the winter, is also a well known risk factor

Clinical Manifestation of AD: The clinical spectrum of AE is wide, ranging from mild forms such as Pityriasis alba (dry de-pigmented
patches) to major forms with erythrodermic rash.
The eczema is polymorphic including acute (oozing, crusted eroded vesicles or papules on erythematous plaques); sub-acute (thick, pale
excoriated plaques); and chronic (lichenified, slightly pigmented, excoriated plaques) forms.
Notable Features are Skin rash, redness, swelling, dryness, scratch marks, lichenifications and skin thickening.

302
Compiled by Obasi. D. C. Chinedu

**The extent of skin involvement, intensity and presence of sleeplessness and itch is used in scoring disease severity**

Infantile Eczema: Present with facial and patchy or generalized body eczema. Lesions generally first appear on the cheeks. Characterized
by dry and erythematous skin with papulo-vesicular lesions. Scratching results in crusty erosions. The term “milk crust” or “milk scurf”
refers to the yellowish crusts on the scalp in infants, resembling scalded milk. The infant is uncomfortable and becomes restless and
agitated during sleep due to persistent pruritus. In about 50% of patients lesions heal by the end of the second year of life; in some cases they
gradually lose their original exudative character and turn into chronic lesions, characterized by lichenification.

In childhood: from 18 to 24 months onwards. Common eczema sites include flexural areas (ante-cubital fossae, neck, wrists, and
ankles), the nape of the neck, dorsum of the feet and the hands. They can either develop from the preceding neonatal phase or arise de
novo. Rashes usually begin with papules that become hard and lichenified with inflammatory infiltration when they are scratched.
Frequent licking of the lips may lead to small, painful cracks in the perioral skin. Frequent scratching and manipulation of the affected skin
causes destruction of melanocytes, resulting in areas of hypo-pigmentation when the inflammation subsides (post-inflammatory hypo-
pigmentation). The eczema may disappear completely for a long phase, leaving sensitive, dry skin.

Adolescents and Adults: AE may relapse during puberty or adulthood. As in the childhood phase, localized inflammation with
lichenification of the flexural areas is the most common pattern in adolescents and adults
Predominant sites are the neck, upper chest, large joint flexures, and backs of the hands. Facial skin is usually affected on the forehead,
eyelids, and perioral region.

Management of AD: Optimal management of AD requires a multifaceted approach aimed at healing and protecting the skin barrier and
addressing the complex immuno-pathogenesis of the disease.

General measures: - Avoidance of triggers - Use of moisturizers - Bleach baths - Wet wrap therapy
**Bathing Practices with plain warm water for 10 to 15min, followed by quick pat and drying; then apply topical medications and
moisturizers within 2-3mins. This helps to reduce the evaporative losses and drying effect**
- Reduction in the pH of stratum corneum alone (hyperacidification). - Applications of serine protease inhibitors
- General moisturization measures – aqueous cream

Specific tx: Basic therapy with emollients (lotions, creams, ointments, sprays)- E45, Aveeno, Eucerine, Alba.
- Specific lipid replacement therapy - Treatment of Bacterial (staphylococcal) colonization
- Corticosteroids: topical hydrocortisone, Betamethasone fluticasone
- Calcineurin inhibitors : pimecrolimus, tacrolimus
- Biologic agents : Dupilumab, a human monoclonal antibody which blocks IL-4 and -13.
- Immunosuppressives : Cyclosporine(CSA) Azathioprine(AZA), & Methotrexate (MTX)
- UV-B, UV-A1, Balneo radiation - PDE-4 INHIBITORS : Cipamphylline.

Insect Bite Allergy: Sting-induced large local (LL) or systemic allergic reaction (anaphylaxis).
An LL reaction is defined as a reaction larger than 10 cm in diameter which lasts over 24 hours in which the signs and symptoms are
confined to tissues contiguous with the sting site.
Common manifestation in children: papular rashes on the extremities and exposed areas of the body

Allergic Conjunctivitis: Allergic conjunctivitis is a distinct clinical entities, ranging from mild but disturbing forms due to IgE sensitization
to aeroallergens; to forms of kerato-conjunctivitis where the severe allergic inflammation becomes difficult to treat, with corneal
involvement and may lead to permanent ocular damage and even loss of vision.

Clinical Features: Clinical forms include: - Seasonal allergic conjunctivitis - Vernal conjunctivitis
- Persisitent or perennial allergic conjunctivitis - Atopic Keratoconjunctivitis
Seasonal and Perenial Allergic Conjunctivitis are mild diseases very often associated with rhinitis. They are easily controlled by adequate
anti-allergic treatment.
On the other hand, Vernal Keratoconjunctivitis (occur alone or more frequently associated with asthma
Atopic Keratoconjunctivitis (typically associated with atopic eczema) are rare but severe clinical entities
Involvement of the cornea (vernal and atopic keratoconjunctivitis) is difficult to treat and may eventually cause impairment of visual
function.

Symptoms: Itching – which is typical of allergic conjunctivitis, distinguishing it from other forms of red eye
- Redness - Tearing and photophobia

Signs: - Conjuctival ejection - Brownish discoloration of the sclera - Eye Discharge


- Papillae at the tarsal area - Cobble stoning around the cornea

Treatment of Allergic Conjunctivitis: - Systemic and topical antihistamines: H1-receptor blockers- eg. Antazoline (active ingredient in
Sparsallerg eye drop)
303
Compiled by Obasi. D. C. Chinedu

- Use of topical ophthalmic steroids (indicated for) corneal involvement in VKC and AKC: Bexamethasone, 1% Prednisolone.

Urticaria and Angioedema: The term urticaria is derived from the Latin name for stinging nettle (Urtica urens). Common names are hives
or wheals. Urticaria is a heterogeneous group of disease sub-type characterized by wheals (fleeting elevations of the skin lasting
approximately 24 hours) and/or angioedema (deeper swellings of skin and mucus membranes which can last up to 72 hours). The urticacae
or wheals can occur anywhere on the body.

Categories of Urticaria: Three major categories exist:


a) Spontaneous occurrence of wheals, associated with acute and chronic urticaria
b) Wheals and angioedema elicited by particular stimuli, and in particular physical urticarias: and
c) Other urticarial disorders such as exercise-induced urticaria.

Notable Triggers of Urticaria: Acute urticaria is defined by a maximum duration of 6 weeks. In children, the majority of acute urticaria
cases persist for 3-7 days and are primarily associated with viral upper respiratory infections. In approximately 10%, acute urticaria is
caused by reactions to drugs;
Food is a rare cause, accounting for approximately 1% of cases, but can lead to severe and life-threatening reactions

Chronic urticaria is characterized by the spontaneous appearances of wheals for 6 weeks or longer. An exact etiology is not evident for
many patients. Factors eliciting chronic urticaria are diverse and include:
- Autoimmune mechanisms and may be exacerbated by Pseudo-allergic reactions to food and/or Inflammatory or infectious diseases
Autoimmune reactions are caused by spontaneously occurring antibody that can activate histamine-containing cells in the skin to initiate an
acute response, plus a more persistent “late phase” reaction with individual urticarial lesions lasting from 4-36 hours
**Diagnostic and therapeutic procedures for chronic urticaria can be complex and referral to a specialist is often required**

Mechanisms of Urticaria
Group Subgroup Definition

Spontaneous urticaria Acute spontaneous urticaria Spontaneous wheals and/or angioedema < 6 weeks
Chronic spontaneous urticaria Spontaneous wheals and/or angioedema > 6 weeks

Physical urticaria Cold contact urticaria Eliciting factor: cold objects/ air/ fluids/ wind
Delayed pressure urticaria Eliciting factor: vertical pressure
(wheals arising with a 3 – 12 h latency)
Heat contact urticaria Eliciting factor: localized heat
Solar urticaria Eliciting factor: Ultraviolet (UV) and/or visible light
Urticaria factitia / Eliciting factor: mechanical shearing forces
dermographic urticaria (wheals arising after 1 – 5 min)
Vibratory urticaria / Eliciting factor: vibratory forces
angioedema

Other urticaria Aquagenic urticaria Eliciting factor: water


Cholinergic urticaria Elicitation by increase of body core temperature due to
physical exercise, spicy food
Contact urticaria Elicitation by contact with urticariogenic substance

Treatment of Urticaria: The diagnosis & treatment of urticaria requires consideration of all the complex & individually different triggering
factors as well as the underlying disease mechanisms. Management strategies include:
- Avoidance of triggers
- Symptomatic relief: H1-type oral antihistamines are the most preferred drugs to induce symptom relief.
- 2nd generation antihistamines are preferred over older 1st generation antihistamine molecules. Eg., Cetirizine, Levocetirizine,
fexofenadine, loratadine, and desloratadine
- 1st generation H1 antihistamines: hydroxizine, diphenhydramine and chlorpheniramine. Side effects include: behavioral changes-
irritability, hyperactivity; sedation; rarely- dry mouth and arrhythmias
For chronic refractory urticarial: Leukotriene antagonists like montelukast in combination with H1 antihistamines.
- H2 blockers like Ranitidine in combination with H1 antihistamines.
- Other treatment options for chronic refractory urticaria: methotrexate, cyclosporine, immunoglobulins or omalizumab
Short course oral steroids may be used for acute but not chronic urticaria

304
Compiled by Obasi. D. C. Chinedu

ANAPHYLAXIS: Anaphylaxis is defined as “an acute and potentially lethal multi-system allergic reaction in which some or all of the
following signs and symptoms occur”. Anaphylaxis includes both allergic and non-allergic etiologies.
“Allergic anaphylaxis” is immunologically mediated and involves IgE, IgG and immune complexes
“Non-allergic anaphylaxis” refers to anaphylaxis from whatever non-immunologic cause and replaces the term “anaphylactoid”.

Pathophysiology of Anaphylaxis: The mechanism of allergic anaphylaxis usually represents an immunological reaction, most often an
immunoglobulin E mediated allergy.
The symptoms of allergic anaphylactic reactions are caused by release of different mediators (e.g. histamine, prostaglandins, leukotrienes,
tryptase, platelet-ac- tivating factor, cytokines, chemokines) from mast cells and basophil granulocytes
Specific antibodies of other classes can trigger similar complement-dependent symptoms through the formation of circulating immune
complexes (immune complex anaphylaxis)

The mechanisms of non-allergic anaphylaxis comprise G protein-induced; - Direct release of vasoactive mediators,
- Direct activation of the complement system, - Interactions with the kallikrein-kinin system,
- Interactions with arachidonic acid metabolism - as well as Psychoneurogenic reflex mechanisms.

Anaphylaxis can be an explosive, potentially fatal event which can affect any organ system. Manifestations are usually rapid in onset and
appear in most instances within minutes to an hour of exposure to the offending agent.
If the agent is injected, the reaction usually begins within minutes. After ingestion, there can be a longer time interval between exposure to
the culpable agent and the onset of the reaction; usually within 2 hours.

Signs and Symptoms: Cutaneous: >90 of cases


Urticaria (hives) and Angioedema (localized swellings beneath the skin, most commonly on the lips and eyes)
Flush, Pruritus (itch) without rash
Respiratory: Dyspnea (shortness of breath), Wheeze, Cough
Upper airway angioedema (e.g. swelling in throat) - Rhinitis (runny nose, nasal congestion)
Dizziness, syncope (loss of consciousness), hypotension (low blood pressure):
Abdominal: Nausea, Vomiting, Diarrhea, Cramping pain
Miscellaneous: Headache, Substernal pain, Seizure

Management of anaphylaxis
Agent Dose and route of administration Comments

Epinephrine 1:1000 0.2-0.5 mg IM thigh (adult); 0.01 mg/kg [up to Give immediately and repeat every 5-15
0.3 mg] min as needed. Monitor for toxicity.
IM thigh (child)
Volume expansion- 1-2 litres rapidly IV in adults Rate is titrated to pulse and blood
Normal saline (5-10ml/kg in first 5 min); 30ml/kg in first hr for pressure.
children Establish IV access with the largest
catheter possible.
Use administration sets that permit rapid
infusions. Monitor for volume
overload
Antihistamines 25-50 mg IV (adults); Second-line agents; H1 and H2 agents
e.g., diphenhydramine 1 mg/kg IV up to 50 mg (children) may be more effective than H1
e.g., ranitidine 1 mg/kg (adults) agents alone; oral doses might
12.5 – 50 mg infused over 10 min (children) suffice for milder episodes.
Corticosteroids 1-2 mg/kg/day IV No role in acute anaphylaxis
e.g. methlyprednisone 0.5 mg/kg/day PO
e.g., prednisone

A flash back at our Learning Objectives: To list the common allergic disorders in childhood.
To know the epidemiology and aetio-pathogenesis, modes of presentation and possible complications of common allergic disorders
- allergic asthma, allergic rhinitis/rhinosinusitis, atopic/contact dermatitis, insect bite allergies, allergic conjunctivitis
- urticaria/angioedema; anaphylaxis
To be able to diagnose each of these conditions and their morbidities and possibly identify their trigger
Be able to effectively treat and minimize complications of any allergic disorders

305
Compiled by Obasi. D. C. Chinedu

IMMUNOLOGIC/ALLERGIC DISORDERS (OVERVIEW)


DR ADAEZE C. AYUK

OBJECTIVES: - To understand what constitutes the body's immune system


- To understand what can go wrong with the immune system - To have an overview of some immunological disorders

OUTLINE: - Introduction - Immune system: - Blood cells of the immune system - Organs of the immune system
- Types of Immunological disorders: - Hypersensitivity reactions (body over-reacts)
- Autoimmune disorders (reacts to the wrong substances) - Immunodeficiency disorders (not react when it should)
- Management - Conclusion

CLASSIFICATION OF

The immune system function

Overview of immunologic Disorders: The immune system may over-react to antigens – Hypersensitivity reaction
It may react to self proteins - Autoimmune disease. It may under-react as with – immunodeficiency
Sometimes there may be an interplay of more than one mechanism

A. HYPERSENSITIVITY REACTIONS
Hypersensitivity refers to an immune system response to an antigen beyond what is considered normal. This immunological response to the
antigen (or allergen) leads to tissue damage rather than immunity. Hypersensitivity reactions are divided into 4 types;
- Type I - Type II - Type III - Type IV
Type I, II and III is Antibody Mediated while Type IV is Cell Mediated

306
Compiled by Obasi. D. C. Chinedu

Classification of Hypersensitivity reactions

Type I (Immediate) Hypersensitivity reactions: Commonly called allergy. Mediated by IgE antibodies in response to stimulation by an allergen
e.g. House dust, Pollens, Cosmetics, Insects, Clothing and Drug
Exposure to an allergen can occur by: ingestion, inhalation, injection, direct contact. The outcome reaction may be localized to a specific
target tissue or organ (allergic rhinitis, asthma) or can be systemic (anaphylaxis)

Type II reactions - Cytotoxic hypersensitivity reactions: IgG or IgM antibody-mediated. The immunoglobulins react with cell-surface
antigens to activate the complement system and produce direct damage of the cell surface. E.g - Transfusion reactions(ABO, Rh) and
hemolytic disease of the newborn (HDN)

Type III reactions - Immune-complex reactions: Antigens combines with antibody within circulation and form immune complexes.
Wherever in the body the complexes are deposited, they activate compliment system. Polymorphonuclear cells are attracted to the site
resulting in local inflammation and tissue injury within 3 - 10 hours after exposure to the antigen(e.g., Hypersensitivity Pneumonitis,
Glomerulonephritis, Rheumatoid Arthritis, Systemic Lupus Erythematosus). The reaction may be general (e.g.serum sickness) or may
involve individual organs including or other organs.
Systemic lupus erythematosus: Autoantibodies against DNA result in immune complex formation. Trigger unknown. Glucocorticoids
(steroids) will help reduce inflammation

Type IV - Delayed hypersensitivity reaction: A function of T Lymphocytes, not antibody -CELL MEDIATED. It starts hours (or days)
after contact with the antigen and often lasts for days. Activated T Lymphocytes = Release of cytokines and macrophage activation = T-cell
mediated cytotoxicity. It is the principal pattern of immunologic response to variety of intracellular microbiologic agents (Mycobacterium
Tb, Viruses, Fungi, Parasites). Also in contact dermatitis, graft rejection.

B. AUTOIMMUNE DISORDERS
An autoimmune disorder is a condition that occurs when the immune system mistakenly attacks and destroys healthy body tissue. Aetiology
is unknown.
One theory - some microorganisms (such as bacteria or viruses) or drugs may trigger changes in susceptible genes that such individual more
likely to get autoimmune disorders.

An autoimmune disorder may result in: - Destruction of one or more types of body tissue
- Abnormal growth of an organ - Changes in organ function
An autoimmune disorder may affect one or more organ or tissue types. E.g
- Blood vessels - Connective tissues - Endocrine glands such as the thyroid or pancreas
- Joints - Muscles - Red blood cells - Skin

Examples of autoimmune-related disorders: - Addison's disease - Dermatomyositis - Graves disease


- Celiac disease (gluten-sensitive enteropathy) - Hashimoto's thyroiditis - Multiple sclerosis
- Myasthenia gravis - Rheumatoid arthritis - Sjogren syndrome
- Systemic lupus erythematosus (SLE) - Type I diabetes

INVESTIGATIONS: - Antinuclear antibody ANA tests - Autoantibody tests - CBC


- C-reactive protein (CRP) - Erythrocyte sedimentation rate (ESR)

TREATMENT: This depends on the specific autoimmune disease. The goals of treatment are to:
- Reduce symptoms - Control the autoimmune process - Maintain the body's ability to fight disease
Replacement hormones may be required - thyroid supplements, vitamins B12, insulin injections.
Anti-inflammatory/immunosuppressive agents - corticosteroids (such as prednisone) and non-steroid drugs- azathioprine,
cyclophosphamide, mycophenolate, tacrolimus.
307
Compiled by Obasi. D. C. Chinedu

C. IMMUNODEFICIENCY DISORDERS
Immunodeficiency disorders: These involve malfunction of the immune system, resulting in infections that develop and recur more
frequently, are more severe, and last longer than usual. Immunodeficiency disorders impair the immune system’s ability to defend the
body against foreign or abnormal cells/disease pathogens that invade or attack it (such as bacteria, viruses, fungi, and cancer cells). As a
result, unusual bacterial, viral, or fungal infections or lymphomas or other cancers may develop.
Types of immunodeficiency disorders
Primary: These disorders are usually present at birth and are usually hereditary (most are X-linked). They typically become evident during
infancy or childhood.
Secondary: These disorders generally develop later in life and often result from use of certain drugs or from another disorder, such as
diabetes or human immunodeficiency virus (HIV) infection. They are more common than primary immunodeficiency disorders.

Primary immunodeficiency disorders – classification: Classified by which part of the immune system is affected – if it affects:
Humoral immunity, which involves B cell lymphocytes - that produces antibodies (immunoglobulins)- commonest
Cellular immunity, which involves T cell lymphocytes OR
Both humoral and cellular immunity
Phagocytes - Complement proteins (Innate Immunity)
The affected component of the immune system may be missing, reduced in number, or abnormal and malfunctioning.

Secondary immunodeficiency disorders: Most commonly due to drug effect (mainly immunosuppressants)
- Immunosuppressants are used to intentionally suppress the immune system. E.g Corticosteroids, Chemotherapy and radiation therapy
- Following certain diseases (HIV, lymphoid malignancy) - Malnutrition

Symptoms of immunodeficiency disorder


 Recurrent infections such as recurrent chest infections (typically sinusitis, bronchitis, middle ear infections, or pneumonia), and skin
infections (pyoderma, Viral warts).
 Recurrent fevers, chronic diarrhoea and failure to thrive.
 Other symptoms vary depending on the severity and duration of the infections.
If Primary immunodeficiency occurs as part of a syndrome with other symptoms it is often more easily recognized eg DiGeorge syndrome
(low-set ears, small receding jawbone, wide-set eyes).

Table of primary immune disorders


Part of the Immune System That Is Affected Disorder
 Common variable immunodeficiency
 Deficiency of a specific antibody
Humoral immunity: B cells (lymphocytes) and their
(immunoglobulin), such as IgA deficiency
production of antibodies
 Transient hypogammaglobulinemia of infancy
 X-linked agammaglobulinemia
 Chronic mucocutaneous candidiasis
Cellular immunity: T cells (lymphocytes)  DiGeorge syndrome
 X-linked lymphoproliferative syndrome
 Ataxia-telangiectasia
Combined humoral and cellular immunity: (Problems  Hyperimmunoglobulinemia E syndrome
with B and T cells  Severe Combined Immunodeficiency SCID
 Wiskott-Aldrich syndrome
 Chronic granulomatous disease (CGD)
Phagocytes: Problems with the movement or killing  Chédiak-Higashi syndrome (rare)
activity of these cells  Cyclic neutropenia
Leukocyte adhesion defects

 Complement component 1 (C1) inhibitor deficiency


Complement proteins: Deficiency of complement (hereditary angioedema)
proteins  C3 deficiency
 C5, C6, C7, C8, and/or C9 deficiency


CONCLUSION: We now know what constitutes the body's immune system. We now understand what can go wrong with the immune
system. We now have an overview of some immunological disorders.
DIGEORGE SYNDROME: Primary immunodefficiency associated with absence of thymus and parathroid glands. CATCH-22: C-cardiac
abnormalities A-abnomal facies T.. T-cell deffiency
C- cleft palate H- hypocalcemia Chromosome 22 and FTT

308
Compiled by Obasi. D. C. Chinedu

CHILD SURVIVAL STRATEGIES AND SD GOALS


BY DR ILOH, O.N

Outline: - Introduction - Definition of Terms - Components of CSS - Child survival Indicators


- Sustainable development goals

Introduction: Worldwide, nearly 6.6 million under-five children die yearly, translating to about 18,000 under-five deaths every day. About
50% of under-five child deaths occur in only five countries of the world, namely, India, Nigeria, Democratic Republic of the Congo,
Pakistan, and China. Two of these countries, India and Nigeria, account for more than one-third of global under-five mortality, contributing
22 percent and 13 percent, respectively.

Commonest causes of death in children less than 5 years: - Malnutrition (53%) - Neonatal causes (37%)
- Pneumonia (19%) - Diarrhoea (17%) - Malaria (8%)
- Vaccine preventable diseases eg: Measles (4%) - HIV/AIDS (3%)

Worried about the alarming under-five mortality rates in developing countries, the WHO in collaboration with UNICEF and the World Bank
IN 1982, developed a set of evidence-based interventions which, when properly implemented, would reduce under-five mortality. This
package of interventions was named child survival strategies.

What is Child Survival? Concentrated efforts by governments, the United Nations, organizations, and communities to use effective, low-
cost solutions to protect children from illness during their first five years of life.

Originally, there were four child survival strategies, namely, growth monitoring, oral rehydration therapy, breastfeeding, and
immunization, giving the acronym “GOBI”. With time, some other interventions like family planning, female education, food
supplementation, were added, giving the acronym GOBIFFF
Subsequently more interventions were further added namely environmental sanitation, essential drugs, treatment of common ailments and
health education(EETH). This has led to the current acronym of GOBIFFFEETH

COMPONENTS OF THE CSS: - G-growth Monitoring /Promotion - O-Oral Rehydration Therapy


- B-Breast feeding - I-Immunization - F-Family Planning - F-Female Education
- F-Food supplementation/Fortification - E-Environment sanitation - E-Essential drugs
- T-Treatment of common childhood diseases - H-Health Education

Growth Monitoring: An essential component of health care for all children. Monitoring a child’s growth helps to confirm a child’s healthy
growth and development, or identify early a potential nutritional or health problem.
Growth monitoring is defined as the process of following the growth rate of a child in comparison to a standard by periodic, frequent
anthropometric measurements in order to assess growth adequacy and identify faltering early.
Growth Monitoring is a screening tool to diagnose nutritional, chronic systemic and endocrine disease at an early stage.

Aims of Growth Monitoring


Assessment Role: It allows for early detection of growth faltering and helps to initiates effective action in response to abnormal patterns of
growth.
Educational Role: It serves as a guide in teaching parents and the communities on how nutrition, physical activity, genetics and illness can
affect growth.
Promotional and motivational role: It provides regular contact with primary health care services and facilitates their utilization. It helps to
reduce the risk of death and leads to early referral for conditions manifested by growth disorders. It also serves to encourage parents whose
children are growing well to keep up the good work.

Indicators for Growth Monitoring: - Weight - Height/Length - Head circumference


- Mid upper arm circumference (MUAC)

Oral Rehydration Therapy: It offers a simple and cost effective first step in the treatment of dehydration. Includes;
- Standard osmolarity ORS - Low osmolarity ORS - SSS

Breast Feeding: Exclusive breastfeeding for the first 6months of life and continuation of breastfeeding up to 2 years of age. Breast feeding
has both direct and indirect roles in improving child survival.
Benefits to the mother- Lactational amenorrhoea, Psychological gains
Benefits to the child – Immune protection, Emotional development, Nutrition (micro and macro nutrients)
Benefits to the family – Less use of resources (cheaper)

Immunisation: Vaccination is widely recognized as one of the most powerful and cost-effective public health tools. Expanded Programme
on Immunisation was introduced in 1978 with a vision of eradicating all the six killer diseases, which are polio, measles, diphtheria,
whooping cough, tuberculosis, and tetanus. Renamed the National Programme on Immunisation in 1997. Yellow fever and Hepatitis B
309
Compiled by Obasi. D. C. Chinedu

were later added and more recently, injectable polio vaccine, haemophilus influenza vaccine, rotavirus vaccine and also the pneumovax.

Family Planning: With an unmet need for family planning of 18 percent (13% for spacing, 5% for limiting births) and a contraceptive
prevalence rate (CPR) of 9 percent, Nigerians have more children than planned and at shorter than desired birth intervals. Infant and child
deaths have been found to be on average, twice as high when the interval between births is less than two years.
These intervals pose substantial mortality and nutritional risks for children. Intervals of at least 36 months are associated with the lowest
mortality and morbidity levels. Improved maternal survival and hence better child welfare

Challenges to Family Planning (in Nigeria)


- Poor budgetary allocations for FP/RH activities which would aid to subsidize the cost of FP.
- Socio-cultural factors, - Religious beliefs, - Low educational levels,
- Poverty, - Misinformation, - Poor spousal communication.

Female Education: Women’s education is a key factor in reducing infant and child mortality. Educating a girl child educates the Nation. An
educated female will marry at a later age, have better health seeking behaviour, and avoid harmful cultural practices. Her children will tend
to be better nourished and thus enjoy better health. An educated mother is empowered to get a good paying job which will impact positively
on the wellbeing of her family.

Food Supplementation/ Fortification:


Food fortification is the process of adding micronutrients to foods or condiments that are consumed regularly by the population, such as
flour, sugar, salt and cooking oils. These programmes are extremely effective in preventing micronutrient deficiencies at minimal cost.
Examples of added micronutrients are: Vitamin A, Iron, Zinc, Iodine.

Environmental Sanitation & Safe Water: Many of the diseases that lead to increased morbidity and mortality of children under five are
largely related to the unavailability of safe water, unhygienic behaviours, poor sanitary facilities, and poor housing conditions. Other causes
are;
- Overcrowding – Acute respiratory Infections and Vaccine preventable diseases
- Poor refuse disposal and unsafe water – Diarrhoea, Cholera and Typhoid
- Poor drainage systems – Mosquitoes and Malaria - Bushes and empty cans – Malaria

Essential Drugs: These are the medicines that satisfy the priority health care needs of the population. They are medications which people
should have access to at all times in sufficient amounts, in the appropriate dosage forms, with assured quality and adequate information for
use and at a price the individual and the community can afford.

Treatment of common ailments: Common ailments contribute to mortality and morbidity. They increase the chances of malnutrition.
Examples: Malaria, Diarrhoea, Pneumonia, Helminthiasis.

Health education: This underlies all the above listed steps, prevention of illness much better than cure.
Health education is any combination of learning experiences designed to help individuals and communities improve their health, by
increasing their knowledge or influencing their attitudes.

SUSTAINABLE DEVELOPMENT GOALS (SDGs): The sustainable development goals (SDGs) are a new, universal set of goals, targets
and indicators that UN member states will be expected to use to frame their agendas and political policies over the next 15 years. The SDGs,
otherwise known as the Global Goals, build and expand on the Millennium Development Goals (MDGs), eight anti-poverty targets that the
world committed to achieving by 2015. The MDGs, adopted in 2000, aimed at an array of issues that included slashing poverty, hunger,
disease, gender inequality, and access to water and sanitation.
Enormous progress has been made on the MDGs, showing the value of a unifying agenda underpinned by goals and targets. Despite this
success, the indignity of poverty has not been ended for all. The goals made no mention of human rights and did not specifically address
economic development. While the MDGs, in theory, applied to all countries, in reality they were considered targets for poor countries to
achieve, with finance from wealthy states.

The sustainable goals go much further than the MDGs, addressing the root causes of poverty and the universal need for development that
works for all people.
The SDGs, unanimously adopted by the UN’s 193 Member States at a historic summit on 25th September 2015, addresses the needs of
people in both developed and developing countries, emphasising that no one should be left behind. The 17 Sustainable Development Goals
with169 targets and 230 indicators were launched and adopted by the 193 Heads of Governments including Nigeria and were set to be
implemented by January 2016.

THE SD GOALS ….
GOAL 1. End poverty in all its forms everywhere
GOAL 2. End hunger, achieve food security and adequate nutrition for all, and promote sustainable agriculture
GOAL 3. Attain healthy life for all at all ages
310
Compiled by Obasi. D. C. Chinedu

GOAL 4. Provide equitable and inclusive quality education and life-long learning opportunities for all
GOAL 5. Attain gender equality, empower women and girls everywhere.
GOAL 6. Secure water and sanitation for all for a sustainable world
GOAL 7. Ensure access to affordable, sustainable, and reliable modern energy services for all
GOAL 8. Promote strong, inclusive and sustainable economic growth and decent work for all
GOAL 9. Promote sustainable industrialization
GOAL 10. Reduce inequality within and among countries
GOAL 11. Build inclusive, safe and sustainable cities and human settlements
GOAL 12. Promote sustainable consumption and production patterns
GOAL 13. Promote actions at all levels to address climate change.
GOAL 14. Attain conservation and sustainable use of marine resources, oceans and seas
GOAL 15. Protect and restore terrestrial ecosystems and halt all biodiversity loss.
GOAL 16. Achieve peaceful and inclusive societies, rule of law, effective and capable institutions.
GOAL 17. Strengthen and enhance the means of implementation and global partnership for sustainable development.

GOAL 3: ATTAIN HEALTHY LIFE FOR ALL AT ALL AGES: Most of the sustainable development goals are health related directly
or indirectly but the SINGLE goal that relates most with health is goal 3.
Target 8: Reduce by three quarters, between 2015 and 2030, the under-five mortality rate and to the barest minimum5 by 2040.
Target 9: Reduce by three quarters, between 2015 and 2030, the maternal mortality ratio and to the barest minimum by 2040.
Target 10: Achieve, by 2030, universal access to reproductive health and overall health care.
Target 11: Reduce annual new infections of HIV/AIDS by three quarters in 2030 and ensure universal treatment for HIV/AIDS patients by
2040
Target 12: By 2020, reverse the incidence of malaria and other major diseases and ensure that deaths caused by these diseases are reduced
by a half in 2030.
Target 13: By 2040, to have achieved a universal access to health care delivery, especially in the rural areas.
Target 14: By 2040, to have achieved a significant improvement in the lives of at least 200 million slum dwellers and improved social
protection.

311
Compiled by Obasi. D. C. Chinedu

CHILD ABUSE AND NEGLECT


Dr Onukwuli V.O

Definition: All forms of physical, emotional act or neglect resulting in actual or potential harm to a child’s health, survival, development or
dignity in the context of a relationship of responsibility, trust and power. This can be either by acts of commission or omission.

It is of various broad sub-types: -Physical abuse -Sexual abuse -Emotional abuse


-Child labour / trafficking -Neglect and neglectful treatment

Physical Abuse - act or acts committed toward a child by a care-giver which produce(s) physical harm or which has/have the potential for
producing harm in the child.
Sexual Abuse – a form of physical abuse in which an adult or a significantly older child (more than 5 years older) uses a child for sexual
gratification
Emotional abuse – deliberate actions or inactions that fail to provide a developmentally appropriate and supportive environment, or care-
giver’s actions which can have adverse effect on the emotional health and development of the child. This includes restrictions of movement,
belittling, denigrating, scape- goating, threatening, scaring, discriminating against, ridiculing or other forms of non-physical treatment or
forms of hostile or rejecting treatment.
Child labour - use of children to accomplish non-age related tasks and/or for working long hours for cheap or no remuneration.
Child Trafficking – Business where children are moved as a good for gain, by those involved in the business, for them to be used
exploitatively or for commercial purposes. Can be domestic/internal or international/external (export of the illegal services of child labour).
Neglect or Negligent Treatment – Deliberate failure to provide for child health, education and emotional development, nutrition, shelter or
safe living conditions when the care-giver is in a position to do so for the child. This is clearly different from poverty.

Importance / Significance of Child Abuse


a) The physical pain and tissue injury from physical trauma can heal in time.
b) But the actual and potential harm from psychological and medical consequences may persist through adulthood.
c) Child abuse has been associated with learning difficulties, anxiety, substance abuse, teenage pregnancy, STIs/HIV/AIDS, psychiatric
sequel, low self esteem, and even suicide attempts.
d) Many children who experienced child abuse who grow up into adulthood may have skewed view of life and exhibit abnormal
interpersonal relationships including abnormal use of power and authority
e) Problems of child and adolescent maltreatment reaches well beyond the victims.

Epidemiology: Over 40 million children below age 15 are being abused severely worldwide. With demonstrable significant initial and long-
term harm. It is a global public health problem.
Generally, under reported. When reported, it is presented as a problem of poverty because most of the cases reported are from the lower
socioeconomic class. Evidence abounds to show that it cuts across the society
Poverty, however, is an important fuel or predisposing factor in many situations and may include poverty of the mind.
Sex of the abused child varies with the type of maltreatment. In child sexual abuse, girls are more frequently abused than boys. More boys
are involved in child labour. Cultural physical abuses like scarification marks including tribal marks, has no sex predilection but may be
more in males when used as a bill of passage as a test of maturity.

Use of children as soldiers and war aides is usually seen among males. Child trafficking has sex predilection depending on the uses the child
is to be put to. Females dominate the child prostitution and domestic servant scene
Those in child labour as plantation workers and drug peddlers are mainly 12 to 14 year-old males

Forms of child maltreatment/abuse: a. Domestic b. International


a. Domestic child Abuse: Can be intra-familial or extra-familial. Intra-familial occurs when maltreatment takes place within the family,
including the nuclear family. Abused child may be a biological or an adopted child of the family, or a member of the extended family system.
Extra-familial involves situations when children not related to the family get to live with the family as house-helps or domestic servants for
an agreed symbiotic relationship built on trust.

b. International child abuse: A large scale business across the borders of a country. Involves middle men and women, the barons who move
children from one location to another often across transit camps until they get to their prearranged destination outside the country of origin.

Child Trafficking is of two types:


Internal child trafficking: Children are moved from one location to another to end users in the same country for the same purpose of
exploitative child labour with little or no remuneration.
External child trafficking: Children are moved from one location to another across the borders of the country for the same exploitative
purposes.

Causes of child maltreatment/abuse- Predisposing factors


 Lack of or poor legal framework and weak policy implementation

312
Compiled by Obasi. D. C. Chinedu

 Ignorance or lack of knowledge about the true details of this social problem.
 High levels of illiteracy, unemployment and generally poor standards of living
 Poverty
 Limited capacity of customs and immigration agencies which give way to very porous borders
 Skewed value system being gulped by the young people for the perpetration of this and other social ills.
 Increasing quest for materialism among youths aggravated by peer pressure.
 Poor reporting and monitoring of cases by law enforcement agencies
 High rates of school drop-out coupled with long closures of higher institutions.
 Abuse of the common practice of placement and fostering along with weakened extended family system.
 Desperation of poor and illiterate parents with large families, ignorant of the impact of child trafficking
 Non-provision of social amenities e.g. tap water, electricity that enhances good living.
 The role of the middle men serve as the main fuel which propel this form of ‘modern slave trade’.
-They collect the children with different guises and deliver them to end users directly or indirectly
-Transient camps exist which are used locally and internationally
-As the only connection the children have, they collude with them and whisk them from one location to another locally.
-Some who are entrusted with care of children use them for money making
-Socio-cultural and religious practices facilitates easy movement of children across the borders

Vulnerable and Protective factors in child maltreatment


Most cases of physical, emotional, sexual abuse and neglect occur as a result of complex interaction of personal and socio-economic
difficulties. Several factors can make the child more vulnerable to child abuse/maltreatment and/or neglect. Again, there are different
protective factors which are those elements that appear to modify, improve or change the likelihood of future harm to the child; they
constitute the strengths of the child and the family

The vulnerability factors should always be weighed with protective factors in handling child abuse cases. These factors can be divided into
four levels: Individual, Relationship, Community and Social factors.

Individual factors:
a(i) Vulnerability factors in the caregiver(s): Increased risk of child abuse is associated with one who:
a) Was abused or neglected as a child
b) Has difficulty bonding with the child due to certain problems surrounding child’s birth
c) Does not show nurturing characteristics towards the child
d) Displays a lack of knowledge/ awareness of child development or has an unrealistic expectations that hinders his/her understanding of
the child’s needs and behaviours.
e) Responds to a perceived child’s misbehaviour with inappropriate, excessive or violent behaviour.
f) Approves of physical punishment of children
g) Is suffering from physical or mental health problems or cognitive impairment that interferes with the ability to be a good parent
h) Demonstrates lack of self control when angry i) Is addicted to drugs/substances
i) Is involved in criminal activity j) Is isolated in the community for any reason
j) Demonstrates feelings of low self esteem and inadequacy k) Exhibits poor parental skills
k) Does not provide safe environment for health and development
l) Is experiencing financial difficulty

a(ii)Vulnerability factors in the child: The following does not mean that the abused child is responsible for his/her maltreatment but that
the child may be more difficult to parent because he/she:
a) Is an unwanted baby and/or failed to fulfil the parent’s expectations and desires e.g. sex, appearance , congenital anomalies etc
b) Is a high-need infant who was born prematurely, cries constantly or has a chronic illness
c) Cries persistently and cannot be soothed easily, or has poor intellectual capacity
d) Has physical features like facial abnormalities or resemblance with a detested relative, which may be a risk to positive parenting bonding
e) Is showing symptoms of mental ill-health
f) Demonstrates personality or temperament traits that are perceived by the parents as challenging
g) Is part of a multiple birth situation that has increased the demands on the parents skills and supports esp. when there are already many
others and worse still if is of the undesired sex
h) Has a sibling that is demanding of parental resources and attention e.g. When there are several children of close age range
i) A child who exhibits or is exposed to dangerous or serious behavioural problems like violence, criminal behaviour, self-abusive
behaviours etc

b. Protective factors: Individual characteristics in both parents and children that may act as protective factors against child maltreatment:
a) Appropriate/secure attachment to the primary care-giver (one of the strongest protective factors against child maltreatment).
b) High self esteem c) Good physical and mental health; positive interpersonal skills
c) Having good intellectual capacity
d) Knowledge and awareness of the child’s physical and psychological developmental abilities at a given age

313
Compiled by Obasi. D. C. Chinedu

e) High parental sensitivity to the child’s needs in all areas of development

Relationships: Family composition may be different according to some unique circumstances or local societal norms. Different family set up
exists e.g. Single mothers/fathers, orphans led by siblings, same sex couples etc
a. Vulnerability factors here include:
a) Lack of adequate housing
b) Physical developmental and/or mental health issues of any family member
c) Family breakdown e.g. in marital problems d) Violence in the family
d) Gender roles and roles in marriage that are disrespectful of one or more persons in the household
e) Being isolated in the community
f) Lack of support network to assist with stressful or difficult situations
g) Experiencing discrimination h) Involving in criminal or violent systems in the community

b. Protective Factors
a) Supportive relationship between family members
b) Consistent routine and rules; the ability to resolve problems amicably and mobilise help during stressful times

Community:
a. Vulnerability Factors: Characteristics of community environment that increases the risk for child maltreatment include:
a) Tolerance of violence and sexual inequality in the community and how they are featured in the media
b) Lack of services to provide families and institutions with support and help to meet specialised needs
c) High levels of unemployment
d) Poverty and its association with limited social capital
e) Transient neighbourhoods
f) A local drug trade
g) Weak policies and programmes that contribute to or facilitate the occurrence of child abuse and neglect

b. Protective Factors: Availability of and easy access to the following community supports as well as others contribute to the prevention of
child maltreatment.
a) Services, supports and information about prenatal care and parenting
b) Programmes that reach out to vulnerable and isolated families e.g. home visiting and mentoring programmes
c) Affordable quality child care and education
d) Extended family members and friends in close proximity
e) Safe, well supervised places for children and adolescents to meet and play as they acquire basic healthy habits
f) Participation in extra curricular activities and events
g) Local community initiatives that support the prevention of child maltreatment and violence

Societal Factors
a. Vulnerability factors
a) Social , economic, health and education policies that contribute to inadequate and unstable standards of living and inequity
b) Social and cultural norms that support or glorify violence towards others
c) Weak sanctions against child maltreatment
d) A lack of partnership between sectors to build and share preventive services

b. Protective Factors: Broad societal factors that encourage a climate of valuing and protecting children include:
a) Legislation to protect children
b) Training and job creation programmes
c) Options without financial repercussions for parents who must care for an ill child
d) Laws that enforce fair wages, unemployment benefits, parental leave and financial support during leave
e) Public awareness campaigns on the rights and support for children
f) Resources to support research for constructive solutions and effective preventive approaches.

Presentation /Manifestations
 Recognising and diagnosing child abuse or maltreatment and neglect is one of the most difficult intellectual and emotional exercise and
challenge.
 Requires high index of suspicion and experience to overcome the initial impulse to cover up.
 Non accidental injuries should be considered first by anybody working with children until proven otherwise
 Child abuse especially physical abuse or non accidental injury is actually a symptom of disordered or dysfunctional parenting
 Prompt diagnosis and appropriate intervention can provide possible cure early enough.
 From the point of child walking into the consulting room, to case presentation and interaction with parents/guardians, the doctor or other
health workers should pay attention to every information and observation as a medical detective
 Every information or observation is important

314
Compiled by Obasi. D. C. Chinedu

The following classic pointers should be taken seriously though they may occasionally be false alarms:
a) Delay in seeking medical attention, if at all
b) Vague and inconsistent story of the “accident”
c) The account of the accident may not be compatible with the observed injury
d) Abusing parents will often show abnormal affect and tend to be preoccupied with their own problems
e) Parent’s behaviour exhibit hostility
f) Abnormal interaction between the child and the parents. Child may be withdrawn or frightened often with obvious failure to thrive
Every utterance by the child should be noted. Private interview with him should be in a safe place preferably on admission to remove the fear
of being returned to the custody of the abusing parent immediately.
Presence of another member of staff is important and private interviews with the child and with the parents or suspects is encouraged.

Important note should be taken of the past medical history, family and social history and the milestones. There must be a sharp sense of
observation to catch any tell-tale signs like pain being suppressed, scars, bruises at different ages, any form of communication between
parents and child etc.
Anthropometric measurements should be documented as well as facies and suspicious marks on the child, in writings with diagrams and
preferably with a camera (coloured camera still better).

In case of child sexual abuse, the examining doctor should be familiar with the anatomy of the adolescent and the pre-adolescent genitalia.
Multiple examination of the genitalia should be avoided as this could tantamount to professional/clinical sexual abuse. Thus every health
worker that needs the information should be able to gather at short notice for the clinical examination together

In other forms of physical abuse, common marks or scars of different types of trauma should be sought. It include cigarette or hot iron burns,
spiral and/or multiple fractures, bruises from flogging, obvious malnutrition in a child well provided for while the child minder looks well
fed and robust.
Neighbour’s observation that child is forever crying when parents are not at home. Mysterious death of the baby when parents are out.
History of similar complaints in children earlier cared for in the past by same child minder.

Investigations: To be done whether on in-patient or out-patient basis


Full blood count, including genotype and blood group.
X-rays. Microscopy, culture and sensitivity of swabs. HIV screening etc
Psychological tests
Non laboratory investigations include fact hunting with the employment of social welfare workers, the police, neighbours, school nurse, child
psychologists, psychiatrist etc

Treatment: Mainstay is counselling basically beyond the early medical management. Other forms of therapy also exist including play
therapy. Application of social learning theory to treatment where self –management of behaviour is at the core. This approach can be quite
rewarding.
The client /perpetrator is seen as being responsible for the development of solutions and coping strategies to forestall recurrences and
surmount future influences or provocations.

Prevention: To effectively manage a case of child abuse including the short term and long term complications cannot be compared with the
cost of prevention. Urgent need for awareness creation and education of the populace on child abuse and neglect. For those who lack
socially acceptable means for getting what they seek, the best mode of prevention is to combine deterrents with the cultivation of more
functional alternatives. The eyes of people should be opened to the ills that accompany this social malady.
The following will go a long way in addressing the issue if well handled; Establishment of the:
- National Society for the Prevention of cruelty to Children (NSPCC)
- Child Protection Services in different areas - Child watch monitors
- Schools anti child abuse watch clubs
- Provision of basic social amenities like free and compulsory basic education up to Junior Secondary School – JSS3, free flowing tap water
available to all, mechanised farming, laundry etc
- Establishment of Creches and Day Care centres in strategic positions
- Social Service programmes for both the children and the adolescents e.g. Establishment of Youth Centres for youth development under
supervision

Limitations and challenges:


 Weak data bases- the actual size of the problem is not known, just estimates. Essentially due to under reporting and recognition
 Inadequate provision of basic social services
 Weak enforcement of law in prosecuting criminals
 Rehabilitation of girls repatriated from trafficked places
 Law against girls who return as adults – they left as children and return as adults
 Follow up on outcomes from sensitisation of policy makers

315
Compiled by Obasi. D. C. Chinedu

 Strengthening of UBE scholarship, skills development and job creation

In the legal Framework:


 Nigerian Criminal law has several provisions protecting children and youth from harm and sexual exploitation
 Government and several State Houses of Assembly have passed or are in the process passing laws to protect children.
 Some of these laws include banning of prostitution, children working during school hours, child marriage and female circumcision,
street hawking/begging, anti trafficking and child labour bill

Recommendations
 Massive enlightenment and sensitization programmes with Media involvement
 Community mobilisation for education and involvement and ownership of the programme
 Appointment of children’s Commissioner to safeguard the rights of children among other things
 Research, Monitoring and Evaluation in child Maltreatment
 Serious poverty alleviation programmes
 Strengthening law enforcement agencies
 Encourage co-ordination of Development agencies at county level
 Sanitization and empowerment of the legal system and law enforcement agencies
 Developing the populace to appreciate appropriate value system different from the prevailing materialism
 Including Paediatricians in various policy -making groups in all issues that concern children and adolescents
 Paediatricians should be advocates and initiators of group formation to wield strong enough political pressure that has the potential to
yield good results.

316
Compiled by Obasi. D. C. Chinedu

SCHOOL HEALTH PROGRAM


Dr. Bisi-Onyemaechi

Outline: - Definition - Objectives - Components - Administration - Personnel


- Record keeping - Evaluation - Roles - The adolescent and SHP
- legal aspects of SHP - Terms in SHP - The Nigerian situation

Objectives of the lecture: Comprehensive understanding of SHP - Importance of SHP


- Who is involved in SHP - The Nigerian SH situation

THE SCHOOL HEALTH PROGRAMME: School health programme (SHP) is the group of co-ordinated activities which contributes to
the understanding, maintenance and improvement of the health of the school population. It is multidisciplinary and involves inter-sectoral
cooperation, but revolves around the school teacher.

Objectives: A child spends a good part of his life at school exposed to a wide variety of environmental, physical, emotional and social
influences. The objectives of SHP include:
 To detect early deviation from normal health, especially those which will hinder the child from benefiting fully from his education.
 To maintain the child in an optimal state of health throughout his/her course of instruction, and to treat promptly all ill children and
restore good health as soon as possible.
 To inculcate into the child healthy habits which the child should maintain throughout their life by personal efforts.
 To teach the child to be always in harmony with himself or herself, his or her family, community, state, his nation and world as well as
teaching them to submit to discipline and to respect their elders, teachers and all those in authority over them.
Vary from locality to locality in relation to local needs. Requires multidisciplinary effort and an interdisciplinary team. The SHP has a great
potential for fostering sustainable human and community development.

COMPONENTS OF THE SCHOOL PROGRAMME: - School health services


- Health instruction, - Healthful school environment - School-Home-Community Relationship

SCHOOL HEALTH SERVICES: Deals with the maintenance of the health of school children in order to identify and treat the ill and
handicapped school child. The specific services include;
- school medical examination, - school clinics, - school meals & food hygiene,
- control of communicable diseases and - play activities.

1a. School medical examination: To identify those children with defects which may adversely affect their education. offset these adverse
effects, by ensuring early and continued treatment and/or providing special educational facilities.
Every child should undergo routine medical examination three times while at school; at entry, midway through school and at completion.
The most common defects often found during these medical examinations include visual defects, hearing defects, stunting of growth,
malnutrition, dental caries and skin diseases.
Routine medical examination is therefore an opportunity to identify defects in school children and correct them early.

1b. Health clinics: This is done at two levels:


Minor ailments: involves treatment of simple cuts bruises, slight fevers and minor skin infections.
Treatment is provided with First Aid Kits. Sick bays and health workers are provided in bigger schools.
Specialist Clinics: these are specialized services and may be provided for school children to take care of dental, visual and hearing problems.
usually by referral to specialist hospitals, occasionally maybe available in the school

1c. School meals and food hygiene: involves the provision of essential nutrients for school (vitamin A, iron, iodine etc). Usually achieved by
the provision of one nutritious meal a day while in school. Schools meals have been reported to:
1. Cause an increase of about 25% in school enrollment
2. reverse low academic performance of children in public schools.
3. Teachers also reported regular and punctual attendance to school,
4. reduction in truancy and absenteeism,
5. increase retention and participation in curricular activities.

1d. Control of communicable diseases: Involves instituting measures against outbreak of communicable diseases in schools eg head lice,
diarrhea etc. Isolation of children with communicable disease by sending them home to reduce the spread in school is usual however a health
room in school is preferable, in order to also reduce the spread at home and community.

1e. Play activities: promote mental, physical and psychological developments through healthy competition. Examples include School youth
clubs, girls guide, boys brigade and boys scout. May involve excursions to places of educational interest.
HEALTH INSTRUCTION: Refers to the classroom instructional activities or the process of delivering the health lessons thereby
facilitating student learning. Involves teaching of Personal hygiene, nutrition education identification of simple diseases, safety education
317
Compiled by Obasi. D. C. Chinedu

lessons and first aid.


Objectives include provision of health knowledge, enhancement of wellness behaviors, facilitation of healthful relationships and enabling
children/students make responsible decisions.

HEALTHFUL SCHOOL ENVIRONMENT: This involves everything in the school surroundings that affect the physical, mental and
psychological well-being of the pupils. This can be grouped into the three basic components; the physical, biological and social
environments.
Physical environment; school buildings, water supply, refuse disposal, school premises, and equipment.
The biological environment; living things in the environment, e.g. viruses, bacteria, fungi, protozoa, helminths, plants and animals, some of
which cause diseases.
The social environment represents the situation of a child as a member of the school and the activities forming part of the school
programme. This includes the child/teacher relationship, child/child relationship, head teacher/teacher/child relationship, school
clinic/kitchen/canteen, transportation and recreational facilities.
There should be periodic inspection of the school environment; classroom, assembly halls, dormitories, toilets, refuse management system,
water source and kitchens. The health sector, the education authorities, ministry of works & housing and the social services department
should be involved. Co-operation and co-ordination of services between these different sectors will ensure a healthy environment for the
child to grow and develop maximally in.

School-Home-community relationship: The co ordination of what the child learns and experiences in school and their equivalent in the
home and community

Administration of school health programme:


 The ministry of health, with active participation of the ministry of education. As a component of primary health care. Federal, state and
local government.
 Federal develops policy. State develops policy and manpower and also provides infrastructure. Local government: day to day
implementation and supervision.
 Good record keeping is very essential for maintaining a first class health level in schools.
 Evaluation involves monitoring the implementation of components and assessing how effective they are in achieving the objectives of
the programme.

Manpower needs is multidisciplinary and inter sectoral. Medical officers especially those with public health or paediatric bias. Experienced
community nurses, health sisters, CHEWs, environmental health superintendents
Health educators. Nutritionists and dieticians. Child psychologists, psychiatrists from both ministry of health/education. Sports and physical
recreation experts. Teachers and school health masters.
Parents-Teachers Association.

RECORD – KEEPING: Essential for effective school health in schools. Health records should be available, cumulative and tranferable.
Registers should be kept of all defects – visual, hearing, and mental and intellectual problems. Reasons for absenteeism should also be
documented.
Notifiable diseases records and records of health bills should be kept.

EVALUATION: Evaluation involves monitoring the implementation of components and assessing how effective they are in achieving the
objectives of the programme. Achieved using an adapted school health programme evaluation scale:
 Used to assess the different components of the school health programme.
 Each component is scored based on specific grades.
 Scores are collated at the end.
 Total maximum score-152
 Minimum acceptable score-103
 Actual score-? (relate to above scores

COLLATION OF SCORES
Component of SHP Maximum Score Minimum Acceptable score Actual Score

School Health Care Services 42 15

School Health Instruction 55 31

Healthful School Environment 65 52

Total scores 162 98 ?

318
Compiled by Obasi. D. C. Chinedu

INTERSECTORAL CO-OPERATION/COLLABORATION:
One of the principles of PHC is inter – sectoral cooperation, and the same level of collaboration is required for school health amongst the
under listed sectors of any given nation:
- Planning and establishment - Environment and sanitation - Communication
- Agriculture - health - Water - Power - Housing
- Education - Transportation - Commerce and Industry - Youths, sports, culture
- Social organization: Non-governmental organizations
- Tiers of Government: THE SCHOOL HEALTH DISTRICT

ROLE OF THE PHYSICIAN IN SHP


 Works out policy and organizes and supervises health-care delivery in and out of schools
 Prescribes standards for the school health programme
 Motivates teacher-educators to comply to the set curricula and standards on SHP
 Treats sick children ,arranges specialist care when necessary
 Organizes routine medical examination and appropriate referral when necessary
 Trains and monitors progress of teachers in relation to school health
 Evaluates the standard of school health practice in schools.
 Serves as an advocate for the community’s health by sensitizing all the stakeholders and vital functional sectors to their roles in the
promotion, maintenance, improvement and conservation of the community’s health.

THE ROLES OF THE SCHOOL TEACHER IN SHP:


The teacher, especially the head-teacher is the catalyst in the implementation of the SHP.
 Bringing a child’s condition to the attention of parents and authorities, by performance of periodic health appraisals on all school
children, continuous alertness over the health needs of pupils in the class, initiating specific, screening procedures to identify disabilities
and faltering growth and development.
 Follow-up of referrals to ensure that children receive the appropriate needed care.
 Counseling parents on various means for obtaining professional services.
 Seeking financial and other aid to obtain corrective action for the children’s needs;
 Implementing the orders of children’s physicians, when such children are in school;
 Helping children who are not directly under the care of physicians solve their problems, through counseling, first aid and getting access
to emergency help as the case may be.
 Understanding and empathizing with children’s problems in such a way to make the school more enjoyable and effective.
 Adapting the SHP to the needs of the specific child.
 Being an advocate for health of the children and the community through provision of leadership, education of the parents, members of
the governing board of the school and the LGA legislature and executive, towards ensuring their appreciation of the value of the SHP.

THE FACILITATORY ROLE OF THE HEAD-TEACHER


The head teacher is the key person in the school health programme because of his professional training as a teacher and the administrative
head of the school. His attitude to health matters affects the members of the school community.
 Provides the necessary leadership to carry out a successful programme.
 Sees to the provision of sufficient funds for a programme that meets the needs of all students.
 Designates one staff person to be responsible for directing the programme and provides sufficient time from other school responsibilities
to oversee the SHP
 Supports the programme and encourages all staff to participate actively.
 Schedules and supports in-service sessions for the purpose of perfecting health observation techniques and informal counseling
procedures, and
 Favourably interprets the purpose of the programme to the community and especially before the Parent-teachers Association and Board
of Education.

THE ADOLESCENT AND SHP: At puberty, children have psychological problems and are particularly vulnerable of infections, stress,
drug addiction and alcoholism.
The adolescent may feel shy to seek information from the parents/other family members and family doctor. The school teacher must be
equipped with adequate skill and knowledge to handle the problems of the adolescent;
E.g. counseling a teenager on sexuality and contraception, maturity, sexual activity substance abuse etc.

LEGAL ASPECTS OF SCHOOL HEALTH PROGRAMME: The child is under the care of the school authorities and the class teacher.
 The school has the responsibility to provide a healthy emotional and physical environment for its students.
 The child is not expected to suffer any damage while at school.
 Parents and guardians are expected to meet their financial moral obligations to the school
 The child is bound by the rules and regulations of the school.
Problems may arise due to inadequacy of facilities, sub-optimal work environment, lack of knowledge regarding child care, wrong attitude or

319
Compiled by Obasi. D. C. Chinedu

poor motivation. When any problem arises, what needs to be established is whether the teacher's action was taken.
-In the interest of the child;
-In the interest of other children in school;
- In the overall interest of the school, within the ambit of the school regulation and societal norms and devoid of any prejudice or malice.
Legal consideration uphold the notion that a child needs to be catered for by the school authority, while within the confines of the school-
system.

Health promoting school: This is a school that


Fosters health and learning with all its resources
Engages all (health and education officials, communty etc) to make the school a healthy place
Strive to provide a healthy environment, health services, outreaches, social support and recreation
Implement policies that respects individuals wellbeing and dignity.

Child friendly school: Assures every child of a safe, emotionally secure and psychologically enabling environment.
Is friendly and welcoming
Attends to the health and safety needs of the children
Provides equal opportunities and democratic procedures for boys and girls
Healthy psychosocial environment:
- Provide friendly, rewarding and supportive atmosphere - Supports cooperation and active learning
- Forbids physical punishment and violence - Zero tolerance to bullying, harassment and discrimination
- Connects school and home life involving parents
- Promotes equal opportunities and participation in decision making

The Nigerian SHP Problem: Efforts at addressing the issue of school health program in Nigeria have remained largely at policy level, with
minimal implementation.
 Where any implementation has been attempted, the emphasis has been outside, rather than within the schools.
 School age children (age range, 6-14) constitute about 23 percent of the population of the average Nigerian community. In
underdeveloped countries with high infant and under-five mortality rates, the school-age child is a survivor of the gains of the child survival
strategies. The quality of life of school children must be improved to attain their full potential and develop into educated and productive
adults.
Despite the effort of communities, nations and NGOs, there’s still High incidence of parasitic, infectious and skin diseases. School absence
for medical reasons, high incidence of road traffic accidents and infectious diseases,

Malaria, anaemia and meningitis are leading causes of hospital admission of school-age children in a referral hospital. The tip of the iceberg,
a large proportion of the morbidity and mortality in school-age children is largely preventable.
Schools have been identified, as sites of injuries in children therefore be planned to provide safety, sanitation and comfort.

However, since accident cannot be completely eliminated, provision should also be made for facilities to handle emergencies and minor
ailment at school, to minimize loss of valuable school hours by children.
With frequent episodes of illnesses and limited access to health facilities, schools are likely to record frequent episodes of absenteeism
affecting the child’s educational progress. Several studies have shown that where medical facilities are in close proximity to schools, children
to receive prompt attention and report back to class, with high school attendance rate recorded.

Suggested Interventions
 Reinforcing and expanding the SHP in the curricula and the training of doctors, other health care personnel, as well as all level of
teachers.
 Providing supervised in-training experiences for trainee teachers and health-care personnel.
 Making SHP a vital component of barrier assessment of trainee teachers and health-care givers to give signals to students of its
importance.
 Re-orientation of all practicing physicians, nurses, paramedics, and teachers on the value of, and their role in SHP, as part of continuing
education. This may involve the use of LGA level workshops for teachers, doctors, nurses, sanitation and utilities engineers and all key
personnel for the success of SHP.
 Provision of incentives for inter-sectoral career development in the area of SHP.
 Deliberate enlightenment of the key policy makers and implementation in the legislative, executive and judicial arms of government.
 In Nigeria, almost every small community has primary school, communities without health centres should use the primary school as a
centre for primary health care delivery, for both pupils and the community.
 Reinforcement of the community health extension system and redeployment of some of the currently under-utilized health extension
workers to school where they can help with primary health care delivery.
 Appropriate funding for the SHP.
 Other measures of intervention would include eradication of hunger, through an all embracing agrarian policy that involves school
pupils, serving soilders, all unemployed youths, as well as retired civil servants in productive farming activities.

320
Compiled by Obasi. D. C. Chinedu

321
Compiled by Obasi. D. C. Chinedu

322
Compiled by Obasi. D. C. Chinedu

323
Compiled by Obasi. D. C. Chinedu

324
Compiled by Obasi. D. C. Chinedu

325
Compiled by Obasi. D. C. Chinedu

326
Compiled by Obasi. D. C. Chinedu

327
Compiled by Obasi. D. C. Chinedu

328
Compiled by Obasi. D. C. Chinedu

329
Compiled by Obasi. D. C. Chinedu

330
Compiled by Obasi. D. C. Chinedu

331
Compiled by Obasi. D. C. Chinedu

332
Compiled by Obasi. D. C. Chinedu

333
Compiled by Obasi. D. C. Chinedu

334
Compiled by Obasi. D. C. Chinedu

335
Compiled by Obasi. D. C. Chinedu

CARDIOVASCULAR SYSTEM

1. In mitral incompetence
a) The pulse is collapsing 12) A 5yr old Nigerian child born in Britain is admitted to the ER with a 1day hx of
b) The characteristic murmur is diastolic in timing high fever (temp.40c) and an episode of convulsion. He returned to Nigeria with his
c) The murmur is maximal at the left sternal edge parents 3mnths earlier
d) A left ventricular enlargement occur a) The fever is the most likely cause of the convulsion
e) Rheumatic fever is the commonest cause in childhood b) The fact that the child was born in Britain makes meningitis an unlikely dx
c) Cerebral malaria is a probable dx
2) The common cyanotic CHD include d) The finding of malaria parasite in bld film rules out meningitis
a) VSD e) Lumbar puncture should be done only if convulsion is recurrent
b) ASD
c) TOF 13) In TOF
d) TGA a) Heart failure is rare
e) b) Cyanosis in infancy is uncommon
c) There is biventricular hypertrophy
3) Cardiac lesion of TOF include d) Iron def. anemia can occur
a) Aortic stenosis e) Sodium bicarbonate may be indicated in its mx
b) Pulmonary stenosis
c) ASD 14) The ff are feature of a large PDA
d) VSD a) Continous murmur
e) b) Normal or raised diastolic pressure
c) Biventricular heart failure
4. The following combinations are associated with congenital heart disease d) Loud p2
a) Maternal diabetes mellitus e) Normal sized lt ventricle
b) Rubella infection after the 16th week of pregnancy
c) Marfan's syndrome 15) A wide pulse pressure is usually found in
d) a) Sickle cell haemoglobinopathy
e) b) AV malformation
c) MR
5. In TOF d) AR
a) Failure to thrive is a common feature e) Pb poisoning
b) Cyanosis usually appears during the second year of life
c) The lung field in the chest X-ray shows increased vascular markings 16) In patients with moderately severe pericardial effusion
d) There is evidence of right atrial hypertrophy in the ECG a) The heart sounds are often muffled
e) Apical pan systolic murmur is typical b) The pulse volume is increased by the associated tachycardia
c) The central venous pressure is usually elevated
6) Wide pulse pressure is often a feature of d)Cardiac contour is globular because the myocardium is damaged
a) Venous hum e) M mode ECG is diagnostic
b) Mitral incompetence 17) The apex beat
c) VSD a) is located in the 4th left intercostal space in most infants
d) PDA b) is diffuse in volume overload situations
e) Thyrotoxicosis c) Has a tapping character in patients with mitral stenosis
d) is best felt with the patient lying supine
7) Typical features of TOF include e) is located in the right hemithorax only in patients with CHD
a) Presentation in the first year of life
b) Squatting 18) Signs of digoxin toxicity include
c) Single soft S2 a) Sinus bradycardia
d) Heart failure b) Nodal tachycardia
e) Good response to hyperoxic test c) Ventricular coupling
d) Unilateral gynaecomastia
8) The ff are true of cardiac failure in the infant e) Constipation
a) Bronchopneumonia is a common precipitating factor
b) The absence of pedal edema rules out the diagnosis 19) Endomyocardial fibrosis
c) There is usually associated raised JVP a) Is a common CHD in Nigerian children
d) Tachycardia is a common feature b) The age of occurrence is usually in the first 5 years of life.
e) The vaccine treatment is indicated irrespective of etiology c) Pericardial effusion is characteristic
d) Bilateral pedal edema evolving to generalized oedema is a feature
9) PDA e) A mid-diastolic murmur maximal at the apex is characteristic
a) Is a benign condition which rarely causes heart failure
b) May be caused by intrauterine infection 20) In CHD
c) The diastolic pressure is usually normal a) The commonest structural heart defect seen in 35% of fetuses exposed to rubella
d) Examination of the peripheral pulse is of no dx value in the first trimester include PDA, VSD and pulmonary stenosis
e) The characteristic continuous murmur may be caused by other defect b) VSD is the commonest form of CHD
c) TOF is 80% likely if a neonate is born cyanosed
d) PDA causes a systolic and diastolic murmur
10) Sign of CCF in an infant of six months e) High VSD has a xteristic ECG pattern
a) Tachycardia
b) Pedal edema 21) The ff are seen in px with hyperkalemia
c) Raised JVP a) Seizure
d) Hepatomegaly b) Cardiac arrhythmia
e) Crepitation in lung bases c) Tall p wave on ECG
D) Broad u wave on ECG
11) In TOF e) Bizarre qrs complexes
a) Growth retardation is a common feature
b) Cyanosis usually appear during the 2nd year of life 22) Concerning coarctation of the aorta
c) The lung field in a chest x-ray show increased vascular markings a) It is usually preductal
d) There is evidence of left ventricular hypertrophy in ECG b) Postductal type is associated with hypertension in the upper limb and normal bld
e) Subacute bacterial endocarditic is a common complication pressure in the lower limb

336
Compiled by Obasi. D. C. Chinedu
c) May be associated with high renin hypertension b) heart failure is a common feature
d) Tx of choice in uncomplicated cases is surgery in the 1st mnth of life c) the ECG shows evidence of LVH
e) Hypertension may persist after surgery d) cardiac catheterization in the neonate is of diagnostic value only
e) total correction is not possible in the infant
23) The ff are true of Steven Johnson syndrome
a) it is a very severe of erythema multiforme 32) Hypotonic dehydration in infants
b) the mucus membrane are not involved a) acidosis is seldom marked
c) the skin lesion consist mainly of vesicles and bullae b) correction is by infusion of full-strength
d) it can be precipitated by anti TB drugs c) loss of skin turgor is usually present
e) steroid therapy should only be given in a few cases d) occurs more commonly in well-nourished than in malnourished ones
e) is associated with ileus
24) Complications of large lt to rt shunt 33) Causes of ecchymoses in infants include
a) recurrent chest infection a) sickle cell disease
b) mental retardation b) maternal ITP
c) CVA c) hemorrhagic disease of the newborn
d) kwashiorkor d) DIC
e) pericardial effusion e) severe birth asphyxia

25) A fullterm , 2day old boy develops petechiae, and is found to have a platelet 34) Acute rheumatic heart dx in Nigerian children
count of 2000/mm3. The child is otherwise healthy and has no other physical a) is etiologically related to streptococcal throat infection
findings. The mother’s platelets count is normal. The likely diagnosis b) commonly result in mitral regurgitation
a) maternal autoimmune thrombocytopenia c) occurs in infancy
b) isoimmune thrombocytopenia d) is assoc. with a raised ESR
c) sepsis e) is rare
d) congenital infection
e) DIC

26) Ff are compatible with diagnosis of isolated PS 35) Common causes of heart failure are
a) normal nutritional status a) iron deficiency anemia
b) pink mucosae b) rheumatic heart disease
c) heart failure c) EMF
d) loud pulmonary closure sound d) myocarditis
e) ejection systolic murmur loudest at the lt sternal edge e) malnutrition

27) Ff may be indicated in the tx of infective endocarditis


a) 36) Anterior fontanelle
b) a) is a useful indicator of dehydration
c) b) is always tense in meningitis
d) c) usually closes during infancy
e) d) late closure is common in rickets
e) is pulsatile in a neonate
28) Concerning tricuspid atresia
a) px most often present after the 1st yr of life 37) A known diabetic 2 year old male child was admitted at midday into the
b) presence of heart failure makes the diagnosis unlikely OTCHEW in coma with acidotic respiration. The mother gave the history that the
c) no chest x-ray feature is characteristic child has had fever and anorexia for one day. He had refused both his dinner the
d) ECG shows rt axis deviation night before admission and his breakfast the morning of admission. The mother
e) surgery of choice is mustard's operation gave him insulin as usual that morning and the child vomited soon after before
gradually losing consciousness
29) The ff statements are true of VSD a) The child is in diabetic coma because he has acidotic breathing
a) a loud means that the murmur is loud??? b) Diabetic ketoacidosis is ruled out if urine testing reveals the presence of ketones
b) a precordial bulge signifies rt ventricular enlargement but no sugar
c) a child who is asymtomatic is in infancy is still likely to go into heart failure c) hypoglycemic therapy due to insulin therapy was unlikely since the child must
before the age of 2yr have vomited the drug
d) the chest x-ray may be normal d) acidotic breathing may occur in acute malaria fever
e) pulmonary artery banding requires cardiopulmonary bypass e) intravenous injection of 20 -25 mls of concentrated glucose solution is a rapid
and safe method of differentiating between hypoglycemic and hyperglycemic coma
30) Ff are useful in confirming a diagnosis of acute rheumatic fever
a) bld culture 38) A 3 year old child with TOF will have the ff features
b) ESR a) central cyanosis
c) x-ray of affected joints b) cardiomegaly
d) ECG c) digital clubbing
e) skin snip d) pulmonary plethora
e)polycythaemia
31) Ff are true of TGA
a) it presents with cyanosis at birth

337
Compiled by Obasi. D. C. Chinedu
I. CARDIOVASCULAR SYSTEM

History a. TGA is a possibility


b. There is right sided hypertrophy
Station: A Patient who squats has history of breathlessness for 4 years and c. Rheumatic disease is a differential
cyanosis for 2 years. The diagnosis is

a. Autosomal recessive disease Station : Chest X-ray showing globular heart. The differentials include
b. Creatinine phosphate kinase is useful in the diagnosis
c. The condition will likely show superior axis deviation on ECG a. Pleural effusion
d. The condition is associated with respiratory acidosis b. Pericardial effusion
c. Endomyocardial fibrosis
d. Tetralogy of Fallot
Station : Take a relevant history of 9-month old child with complaint of
breathlessness, cough, failure to gain weight adequately. Station : From the
above Pictures

a. Patient’s name is Oluranti Joseph Station : Picture of finger clubbing. Station : From the station above
b. Patient has had previous episode of cough associated with
fever The feature can be found in
c. Patient has been cyanosed since birth
a. Hypoplastic heart syndrome
d. A large VSD is a likely diagnosis
b. TGA
e. TOF is a likely diagnosis
c. TOF
Examination
d. ASD
Station : Examine the praecordium, Station e. Tricuspid atresia

a. the apex beat is in the 6th ICS AAL


b. There is apical thrill radiating to the axilla Station : Picture of a patient in Squatting Position
c. The diagnosis is query VSD
a. Tricuspid atresia is a differential
d. Heave (LSII)
b. Superior ais deviation is found on the ECG
e. Systolic murmur is heart at LLSE
c. The patient may heart failure
d. Creatinine phosphate kinase level is important
Station : Cardiovascular examination

a. There is central cyanosis


b. There is left lower sternal edge thrill
Station : Picture of a child in knee chest position
c. There is heaving apex beat
d. There is respiratory distress
This 6-year old become restless after struggling with her sister over
a toy
Investigation

Station: X-ray showing enlarged heart. This is chest x-ray of a 7-year old girl
a. A PCV of 68% is expected
with recurrent chest infection
b. A PCV of 18% is expected
c. The condition can be acquired as a result of maternal infection
a. ECG is indicated
during pregnancy
b. Acute rheumatic heart disease is a likely diagnosis
d. Urgent surgery is required
c. TGA is a likely diagnosis
e. The mother should be assured that her younger ones are
d. There is evidence of left ventricle enlargement
unlikely to suffer from the same condition
f. ECG shows LVH
Station : Chest X-ray & Loud P2 etc

338
Compiled by Obasi. D. C. Chinedu
NEONATOLOGY
11) The following measures are appropriate in the resuscitation of a neonate born
1. At the age of eight months a baby can be expected to: with Apgar score of 2 at 1 min
a) Roll over from front to back a) Oxygen administraion by face
b) Sit up with a straight back b) Emptying of the gastric content
c) Pick a small bead between thumb and finger c) Subcutaneous adrenaline 0.1cc/kg
d) Say up to five word clearly d) 50% dextrose 0.5cc/kg diluted and infused into the umbilical vein
e) Feed himself with a spoon e) Intramuscular curamin

2. Convulsion in the first week of life is characteristic of 12) Vomiting in the newborn can be caused by
a) Hypocalcaemia a) Malrotation &volvulus
b) Post maturity b) Intestinal atresia
c) Craniopharyngioma c) Meconium ileus
d) Hypomagnesaemia d) Sepsis
e) Birth asphyxia e) All of the above

3. Both human breast milk and cow's milk can be used for feeding infants but they 13) Children with juvenile diabetes
differ in that a) Usually have abrupt onset of symptom
a) Human breast milk contains more carbohydrate/100mlof milk than cow's milk b) May be treated with oral hypoglycemic agent
b) Human milk is richer in protein than cow c) Need not be concern with the degenerative complication of the dx
c) cow's milk has more calcium than human d) Have marked variation in their insulin requirement
d) Cow's milk contains less vitamin c than human milk e)
e) Human milk is poorer in vit.D than cow's milk 14) Breast fed infant should receive supplemental
a) Vit.A
4. Hypotonic dehydration in infants b) Vit.D
a) Occurs more commonly in malnourished ones c) Vit.K
b) is associated with irritability and hypertonic d) Fluoride
c) Loss of skin turgor is usually present e) Iron
d) Acidosis is often marked
e) Correction is by infusion of dextrose solution 15) The most common anemia in children
a) Sickle cell anemia
5. The following may be responsible for hypoglycemia in a newborn baby b) Thalassemia
a) small-for-gestational-age baby c) Iron deficiency
b) Beckwith syndrome d) Congenital spherocytosis
c) Biliary atresia e)
d) prematurity
e) Glycogen storage disease 16) The most common cause of death in children is
a) Malnutrition
6. NS in Nigeria- the following are usual features b) Malignancy
a) onset before age of 6years c) Accidents
b) Associated with P.falciparium infection d) Congenital defect
c) Persistent vomiting e)
d) Normal cholesterol level
e) Good response to steroids and cyclophoshamide 17) The average caloric requirements for infants during the first year of life
a) 80-90kcal/kg
7 The following are features of neonatal tetanus b) 100-120kcal/kg
a) Convulsion c) 120-130kcal/kg
b) Apnoiec attack d) 90-100kcal/kg
c) Difficulty in sucking e)
d) Stiffening out attack
e) Acquisition of life long immunity against tetanus 18)
a) Human milk is poorer in vit.D than cow's milk
8) Neonatal hypoglceamia b) The neonates head increase by 0.5cm weekly if the baby was delivered at term
a) Is bld glucose of less than 30mg/100ml during the first 3 days of life in both c) Metabolic acidosis may lead to failure to thrive in the preterm infant
fullterm & low birth weight infants d) Indomethacin is useful in medical mx of PDA
b) Most neonates with hypoglceamia are asymtomatic e) Aminophylline is useful in the TX of apnoea of prematurity
c) Is more common in preterm than in small for gestational age infants
d) Can be definitely diagnosed from the symptoms 19)0-hour old infant delivered at term presented with marked respiratory distress.
e) Is treated with dextrose 5% There was no prolonged rupture of membranes and the baby weighed 1.0kg
a) Pneumothorax may occur
9) Causes of respiratory distress in the newborn include b) Parenteral antibiotics are indicated
a) Pneumothorax c) The bld glucose may be less than 30mg/dl
b) Congenital diaphragmatic hernia of the morgagni type d) Meconium aspiration is an unlikely cause of the respiratory distress
c) TOF e) Early feeding is contraindicated
d) Persistence of fetal circulation
e) Hypothermia 20) Pertussis
a) Young infant are nut passively protected by the transplancental transfer of
10) Reflexes normally present at birth include antibody even if mother has been infected in the past
a) Moro b) The cough of pertussis last at least 3 months
b) Tonic neck c) Epistaxis and conjunctival hemorrhage may complicate the illness
c) Parachute d) Reactivation of an old TB focus may occur
d) Rooting e) Malnutrition may follow an attack
e) Palmar grasp
21) The following are with within normal limit for a one year old infant
339
Compiled by Obasi. D. C. Chinedu
a) Weight 6.0kg d) The phenomenon may be due to hemorrhagic disease of the newborn
b) Length 75cm e) Administration of haematinics is a preferred method of treatment
c) Head circumference 52cm
d) Pulse rate 72/min 32) The ff can be acquired transplacentally
e) Respiratory rate 25/min a) Rubella
b) Opthalmia neonatorum
22) Bone pain is a common feature of the ff c) Group B beta-hemolytic septicemia
a) Sickle cell disease d) Syphilis
b) Juvenile rheumatoid arthritis e) NEC
c) Acute leukemia
d) Neuroblastoma 33) In neonatal tetanus
e) Malaria a) 20000 units of ATS should be administered IV stat
b) Crystalline penicillin is administered IM six hourly for one week
23) Adequate nutritional requirements of a low birth weight infant includes c) The common regimen for sedation consists of phenobarbitone, chlorpromazine
a) 100 - 120 calories /kg/day and chlordiazepoxide
b) 3 - 4 mg/kg /day of protein d) Death from aspiration pneumonia is common
c) 400 - 800 I.U. of vitamin D e) There is an inverse relationship between the incubation period an the case fatality
d) 100 ml/kg/day of fluids rate
e) 1 mg vitamin K daily
34) Major causes of perinatal death include
24) Septicemia in the early newborn period is a) Asphyxia neonatorum
a) Not associated with subnormal temperature patterns b) Septicemia
b) Most frequently due to Gram-negative organisms c) Bronchiolitis
c) Best treated with large doses of amoxicillin d) Infective enteritis
d) Frequently causes jaundice e) CP
e) Is common sequelae to rupture of membranes more than 24 hrs before delivery
35) In a six year old with sickle cell haemoglobinopathy
25) Neonatal tetanus a) A haematocrit of 23% indicates a need for urgent blood transfusion
a) Commonest portal of entry is the umbilicus b) The hand and foot syndrome is likely to occur frequently
b) The earlier the onset the worse prognosis c) Anemia with reticulocyte response is more ominous than anemia without
c) the most important clinical feature is convulsion reticulocyte response
d) Immunization of the mother during help to reduce incidence d) Skin petechiae would be an uncommon finding
e) Onset of symptoms is usually the 4th week of life e) Presence of cardiac murmur is usually indicative of the presence of an associated
rheumatic heart disease
26) Septicemia in the early newborn period is
a) Often associated with normal temp. pattern 36) A 15 month old boy presents with a haematocrit of 10%.
b) Most frequently due to gram -ve organism a) Hb genotype should be determined before blood transfusion
c) Best TX with large dose of ampicillin b) He should be transfused with whole blood at 25 mls/kg body weight
d) Frequent cause of jaundice c) He should be transfused with packed red cells at 15 mls/kg body weight
e) An important cause of neonatal mortality d) IV 8.4% sodium bicarbonate 1 ml/kg body weight is desirable
e) the most likely cause of the anemia is malaria
27) Hematemesis in childhood may be due to
a) Acute leukemia 37) The ff values are normal for the associated ages
b) Anaphylactoid purpura (henoch schonlein) a) Haematocrit of 55% at birth
c) Aspirin ingestion b) haematocrit of 35% at 3 months
d) Staphylococcus endotoxin c) Platelet count of 250,000 per cubic mm at age 2 yrs
e) Ingestion of iron d) MCHC of 32 % at age 3 yrs
e) Eosinophil count of 5% in a 7 yr old rural dweller
28) Vomiting in the 1st week of life is due to the ff
a) Pyloric stenosis 38 Causes of thrombocytopenia include
b) Duodenal stenosis a) Birth asphyxia
c) Hiatus hernia b) Salicylate poisoning
d) Intussusception c) H S purpura
e) tracheao-oesophageal fistula d) Gram -ve septicemia
e) ALL
29) The recommendation of the baby friendly hospital initiative include
a) mother and the newborn baby resting 3hr before breastfeeding commences 39) The ff features are compatible with the dx of neonatal septicemia in a 1 week
b) Exclusively breastfeeding infant for more than six months old neonate
c) Supplementing breast milk with water only from time to time a) PROM for 48hrs in the mother
d) Storing express breast milk for more than 24hr if mother is away b) bulging anterior fontanelle
e) Colostrum should not be thrown away c) resp. distress
d) absolute WBC count of 10,000/m3
e) immature neutrophil count of 10%
30) Common causes of neonatal jaundice in Nigeria include 50) The ff statements are true
a) Rh isoimmunization a) SGA are more likely to be polycythemic when compared with preterm ones
b) Septicemia b) jaundice is more common in SGA compared to preterm ones
c) Prematurity c) congenital malformations are commoner in SGA than preterm ones
d) ABO incompatibility d) hypoglycemia is more common in preterm than SGA
e) Hypothyroidism e) weight loss in the 1st week of life is greater among SGA than preterm

31) A two day old infant is discovered to have a haematocrit of 30% 40) Mx of a 3 day old neonate with jaundice may include
a) Circumcision is a likely etiological factor a) phenobarbitone
b) It is useful to know if the infant is one of a set of twins b) glucose water
c) A lumbar puncture is a relevant investigation c) EBT
340
Compiled by Obasi. D. C. Chinedu
d) ampiclox syrup
e) phototherapy88) Reflexes normally present at birth include
a) parachute 43) Anterior fontanelle
b) stepping a) is a useful indicator of dehydration
c) tonic neck b) is always tense in meningitis
d) rooting c) usually closes during infancy
e) gag d) late closure is common in rickets
e) is pulsatile in a neonate
41) Daily requirements in the first month of life in a full term baby
a) water: 75 ml/kg/day 44) A known diabetic 2 year old male child was admitted at midday into the
b) calories: 100 kg/day OTCHEW in coma with acidotic respiration. The mother gave the history that the
c) Vitamin D: 1000 units/day child has had fever and anorexia for one day. He had refused both his dinner the
d) Protein: 2g/kg/day night before admission and his breakfast the morning of admission. The mother
e) potassium: 4 mEq/kg/day gave him insulin as usual that morning and the child vomited soon after before
gradually losing consciousness
42) A 3kg baby was delivered by emergency lower segment caesarean section to a a) The child is in diabetic coma because he has acidotic breathing
16yr old primigravida at term after a labour which lasted 48hrs. The liquor was b) Diabetic ketoacidosis is ruled out if urine testing reveals the presence of ketones
meconium stained. At5min of age his hrt rate was 96/min. The baby was gasping but no sugar
and there was no response to the nasal catheter. He was also floppy and pale. The c) hypoglycemic therapy due to insulin therapy was unlikely since the child must
ff are true have vomited the drug
a) the apgar score is 2 d) acidotic breathing may occur in acute malaria fever
b) endotracheal intubation after adequate suctioning is indicated for the e) intravenous injection of 20 -25 mls of concentrated glucose solution is a rapid
administration of oxygen and safe method of differentiating between hypoglycemic and hyperglycemic coma
c) IV administration of alkali is the 1st priority
d) Pneumothorax is a possible complication
e) seizure within the 1st 24hr will be unusual

NEONATOLOGY

Station : Clerk a 24-hour old neonate with neonatal jaundice

Station : From the station above

a. There is difficulty in breathing


b. There is poor feeding
c. The patient is at risk of septicaemia
d. She had PROM 5 days before birth
e. There is constipation and diarrhea

Station : Picture of cephalhaematoma + 4-way tap valve

Station : From the station above

a. It is caput succedaneum
b. He has risk of unconjugated hyperbilirubinaemia
c. He has a risk of severe anaemia
d. The instrument is used in routine EBT
e. The instrunment is use in EBT

Station : Phototherapy set-up

a. Retinal damage is a complication


b. Diarrhoea is a known complication
c. Circulatory collapse is a complication
d. Is for only LBW
e. Supplies heat to the baby

341
Compiled by Obasi. D. C. Chinedu
NEUROLOGY

1. Petit mal
a) Occurs predominantly in infants of 3 to 12months 11) A leg paralysis as a result of poliomyelitis will show
b) Attacks are associated with loss of muscle tone and fall a) Wasting
c) Respond to pyridoxine therapy b) A positive babinski response
d) EEG has the characteristic c) Exaggerated reflexes
e) Mental retardation is rare d) Diminished tone
e) Glove and stocking anaesthesia
2) Poliomyelitis
a) The infection is transmitted through inhalation of infectious droplets 12) The ff are uncommon features of cerebral malaria
b) It is caused by an RNA virus a) Cortical blindness
c) The vaccine used in Nigeria consist of killed virus b) Metabolic alkalosis
d) In the acute phase patient may present with signs & symptoms of meningitis c) Retinal hemorrhage
e) The virus attack the posterior column of the spinal cord d) Papilloedema
e) Hypoglycemia
3) Miscellaneous
a) Erb's palsy result from damage to c5 &c6 roots 13) Intracranial calcification in childhood occur in
b) Myocardial damage is a likely complication of severe birth asphyxia a) CMV
c) The paraplegia associated with spinal TB is usually of the spastic variety b) Congenital rubella syndrome
d) Crytorchidism is one of the components of prune belly syndrome c) TB meningitis
e) Burkitt’s lymphoma is an important cause of death in pre school age in Nigeria d) Congenital toxoplasmosis
e) Craniopharyngioma
4) A 4yr old boy has a severe seizure disorder and mental retardation; the following
are true 14) TB meningitis
a) A facial hemangioma is a pointer to the diagnosis a) Is a benign dx
b) Intracranial calcification may be present b) is typically associated with CSF polymorph pleocytosis
c) He may have one of the neurocutaneous syndromes c) is as common as pyogenic meningitis
d) CT skull is essential for diagnosis d) Steroids are useful in mx
e) The prognosis is poor e) is sometimes associated with choriodal tubercles

5) Meningitis in children is most frequently caused by 15) The ff are associated with intracranial hemorrhage in newborn
a) Staphylococcus aureus a) prematurity
b) H.influenza b) Birth asphyxia
c) N.meningococcus c) IV NaCO3
d) Strep. pneumoniae d) Neonatal seizure
e) e) Hydrocephalus

6) With regard to meningitis 16) The ff conditions are ass. With Down’s syndrome
a) Common aetiological agents in the newborn are gram -ve organism a) Leukemia
b) In pyogenic meningitis pending culture result the drug of choice is i.m penicillin b) Hydrocephalus
& streptomycin c) CHD
c) The CSF is usually turbid in TB meningitis d) A large tongue
d) The meninges may be the primary site of tuberculous infection e) Brushfield spots
e) The CSF glucose in aseptic meningitis is always normal
17) Common causes of MR in Nigeria include
a) Phenylketonuria
7) Cerebral palsy b) Tetanus
a) Is a progressive disorder of posture & movement c) Cerebral birth trauma
b) Results from injury to developing brain d) Down’s syndrome
c) Bilirubin encephalopathy is the commonest cause in Ibadan e) Kernicterus
d) Spastic diplegia occur commonly in low birth weight infant
e) Dyskinesia results from involuntary movement and changes in muscle tone 18. Classical feature of meningitis in 1 month old infant include
a) Positive kernig sign
8) The ff are constant features of Down's syndrome b) Neck rigidity
a) Mental retardation c) Excessive crying
b) Simian creases d) Vomiting
c) Hypotonia e) Convulsion
d) Congenital heart disease
e) Translocation of chromosome 21 19. The anterior horn cell is the site of damage in
a) G-B syndrome
9) An upper limb paralysis as a result of poliomyelitis will show b) Paralytic polio
a) Wasting c) Hypertrophic muscular dystrophy
b) Hypotonia d) MG
c) Hypertonia e) Tetanus
d) Fasciculations
e) Brisk reflexes 20) Petit mal
a) Occurs predominantly in infants of 5-12 month
10) Klumpke's paralysis results from damage to b) Attacks are associated with loss of muscular tone and fall
a) Radial nerve c) Responds to pyridoxine therapy
b) Nerve roots C8 and T1 d) EEG has the characteristics of hypsarrythmia
c) Nerve roots C5 and C6 e) Mental retardation is rare
d) Phrenic nerve
e) Facial nerve 21) The ff are true of birth trauma
342
Compiled by Obasi. D. C. Chinedu
a) the swelling due to cephal haematoma usually crosses the midline 31) Epilepsy
b) a sternomastoid tumor usually occurs after breech delivery and can be seen within a) hypsarrhythmia is a typical finding in infantile spasm
a few hrs after birth b) ethosuximide is a drug of choice in the tx of psychomotor epilepsy
c) subcutaneous fat necrosis should be excised to prevent calcification and infection c) petit mal usually has its onset in the 2nd decade of life
d) facial nerve injury is usually of upper motor neuron type d) the prognosis for infantile spasm is good
e) in erb's palsy there is damage to the 6 &7 cervical nerve and is managed by e) febrile convulsion can occur at the onset of the fever
physiotherapy

22) Myasthenia Gravis


A) Uncommon in childhood 32) Drug of choice
b) curable in about 25%of affected children a) in status epilepticus is phenobarbitone
c) tx with pyridostigmine bromide b) in status asthmaticus is subcutaneous adrenaline
d) not assoc. with other immunological dx c) in petit mal epilepsy is phenytoin
e) dx clinically by the demonstration of progressive weakness on repetitive or d) in meningococcal meningitis is penicillin G
sustained muscular contraction e) in giardiasis is mepacrine

23) In duchenne muscular dystrophy


a) Creatinine phosphokinase at birth of 100.u/l is suggestive
b) Muscle biopsy shows degeneration of muscle fibres
c) The inheritance is by AR
d) Cardiomyopathy is a cause of death
e) Is a diff. diagnosis of Cerebral Palsy

24) Tuberous sclerosis features include


a) seizure disorders
b) intracranial calcification
c) capillary hemangioma
d) hydrocephalous
e) hypsarrythmia on EEG

25) Complications of lumbar puncture


a) headache
b) vomiting
c) injury to the spinal cord
d) meningitis
e) sciatic nerve injury

26) The ff differential diagnoses could be considered in a 6 year old boy with
generalized seizures
a) head trauma
b) petit mal epilepsy
c) chronic renal failure
d) cerebral malaria
e) sickle cell disease

27) Facial palsy may occur in the ff conditions


a) following forceps delivery
b) chronic mastoiditis
c) cerebellar tumour
d) bacterial meningitis
e) poliomyelitis

28) Reflexes normally present at birth include


a) parachute
b) stepping
c) tonic neck
d) rooting
e) gag

29) Concerning poliomyelitis


a) the virus is a DNA virus
b) it is transmitted by the faeco-oral route
c) the virus travels along axons in the posterior column of the spinal cord
d) the paralysis is usually asymmetrical
e) proximal muscle groups are more often affected than distal muscle groups

30) Myasthenia gravis


a) is an autosomal recessive dx
b) the pathological lesion affects the motor end plate
c) Edrophonium is the drug of choice for tx
d) Most patients die before the age of 20 years
e) Diagnosis is majorly clinical

343
Compiled by Obasi. D. C. Chinedu
Full Term Baby Pre-term Baby

CENTRAL NERVOUS SYSTEM 1. WBC in CSF Approx. 20/mm³ 30/mm³

2. Protein in 150mg/dl 100mg/dl


CSF
History

Examination
Station
Station : Examine the motor system of the patient’s lower limb
CSF Result

 Age – 3 months
Station : From the station above  Sex - Male
 Appearance – Turbid
a. Thereis hypotonia bilaterally  WBC - 485/mm³
b. Muscle fasciculation is noticed Lymphocytes – 90%
c. There is bilateral ankle clonus
d. There is Babinski reflex Polymorphs – 10%
e. There is brisk deep tendon reflex
 RBC – 64%
 Protein - > 1000mg/dl
Station : Examination of big head (Note look at the back and the limbs)
 Culture – Gram +ve diplococci

Station : The follwing are features of an untreated pyogenic meningitis


Station : Do a motor system examination of lower limb on this patient

a. WBC of 4 in CSF
Station : Concerning the patient you examined
b. Protein – 80mg/dl
c. RCC – 1,000
d. Sugar
a. There is fasciculation
b. There is sustained ankle clonus on the right
c. There is hypotonia Station : Examine this CSF analysis result
d. There is brisk tendon reflex over the knees
Turbid

WBC - > 2.0cells/high per film (hpf)


Investigation
Gram stain – negative diplococcus

Culture – No organism
Station
Protein 75mg/dl
CSF Result
CSF Sugar – 25mg/dl
o Age – 2months
o Female
RBC – 100/hpf
o Appearance – slightly hazy
o WCC – 22cells/mm³
- Lymphocyte –
80% Station : From the station above
- Polymorphs –
20% a. The diagnosis is tuberculous meningitis
o Red blood cells – 80/mm³ b. The result is not useful because the lumbar puncture was
o Protein – 50mg/100ml traumatic
c. Sugar level is normal
d. Protein is normal
What is the diagnosis? – Partially treated meningitis/Viral meningitis e. There has been a previous antibiotic use

Laboratory report : showing meningitis


Station

CSF Result
a. Report is suggestive of viral meningitis
b. Report is suggestive of pyogenic meningitis
344
Compiled by Obasi. D. C. Chinedu
c. The CSF sugar is normal
d. It suggests partially treated bacterial meningitis
Gram –ve diplococcus is a cause Picture

Station

Station : This is the CSF picture of a child who presented with seizures and altered A picture of a boy that was asked to look up
sensorium blood sugar is 90mg% and CSF sugar is 50mg%
a. He demonstrate Bell’s sign
WBC > 25cells/mm³
b. He demonstrates biparietal bossing
Polymorphs – 20%
c. He demonstrates bilateral 7th cranial nerve palsy
Lymphocytes – 80% involving upper motor neurone

Protein 120mg% d. This condition can be congenital

Culture : Negative e. Patient usually has a single palmar crease

a. Gullain Barre Syndrome is a likely diagnosis Station : Picture of patient with Facial nerve palsy (Congenital ptosis)
b. Pyogenic meningitis is likely
c. Partially treated pyogenic meningitis is likely
d. Acute viral meningitis is likely
a. Bell’s sign is on the left
b. It is UMN lesion
c. There is biparietal bossing
d. May be congenital
Instruments

Station : Vim Silverman Renal Biopsy pack Station : Picture of a child with erb’s palsy

a. The lesion is at C7,8, and T1


b. Physiotherapy is the modality of treatment
Station

a. This is Vim Silverman renal biopsy pack Station : Picture of a child who was asked to close his eyes. Examiner’s finger
b. It can be used for liver biopsy pulled the right upper lid up.
c. The needle is Mengihin needle
d. Clotting profile is a prerequisite for its use a. There is right upper motor neuron lesion
e. Blood must be crossmatched before the instrument is used b. There is right oculomotor nerve lesion
f. Evaluation of liver function is advisable
g. Its use is contraindicatedby urea >95mg/ml
h. Its use is contraindicated in hyperlidaemia.
i. Gross haematuria is a complications of its use

345
Compiled by Obasi. D. C. Chinedu
ENDOCRINOLOGY
c) Normal alkaline phosphatase
1 Recognised causes of delayed bone age include: d) Delayed closure of the anterior fontanelle
a) Hypopiturtarism
b) Primary hypothyroidism
c) Congenital adrenal hypoplasia 9) Juvenile hypothyroidism presenting at the age of 5yrs may have the ff features
d) Prolonged corticosteroid therapy a) Goitre
e) Tuberculosis b) Deafness
c) Full and bounding pulse
2 The adrenogenital syndrome in early childhood d) Dwarfism
a) Occurs only in males e) Warm tremulous hands
b) May result from an autosomal recessive trait
c) May cause virilization 10) Neonatal hypoglceamia
d) May cause hypernatremia a) Is define as bld sugar of 40mg or less during the 1st 72hrs of life
e) b) Is frequent in the small for dates
c) Rare in babies of diabetic mother
3 The ff are characteristic of vitamin D deficient rickets d) Is common in asphyxiated babies
a) High serum calcium e) Has good overall prognosis with minimal TX
b) Low serum phosphorus
c) Low alkaline phosphatase 11) The ff objectives are attainable in the long term mx of IDDM
d) Delayed closure of the anterior fontanelle a) Elimination of polyuria and polydipsia
e) Hypertonia b) Normal growth and development
c) Continuously normal bld glucose level
4 Features of congenital hypothyroidism in this environment include d) Prevention of microangiopathy
a) Prolonged physiological jaundice e) Maintenance of normal bld lipid level
b) Meconium ileus
c) Polycythaemia 12) In nutritional rickets
d) Tachycardia a) serum calcium levels are always low
e) Cold mottled skin b) plasma vit.D assay can assist dx
c) Hyperparathyroidism is one of the causes
5 Glycosuria may be a feature of d) renal dx has no part to play to in its causation
a) Brain damage e) the bone deformities are irreversible
b) Diabetes insipidus
c) Renal tubular acidosis 13) The ff are true concerning hormones
d) Febrile convulsions a) protein hormones are synthesized mainly on demand
e) Diabetes mellitus b) steroid hormones have diff. molecular precursor
c) thyroid hormones are excreted from storage by pinocytosis
6 The ff are present in 21-hydroxylase enzyme def. of CAH. d) CAMP has a role to play in the synthesis of adrenal cortical hormones
a) Hypertension e) some hormones are excreted in their active form
b) Hypokalemia
c) Acidosis 14) The ff enzymes have been implicated in the adrogenital syndrome
d) Hyponatreamia a) 21- hydroxylase def.
e) Hypochloraemia b) peroxidase enzyme
c) 11-hydroxylase def
7 Genu valgus d) 7dehydrocholesterol dehydrogenase def.
a) Blount’s dx is a strong diff. e) 3-beta hydroxysteroid dehydrogenase def.
b) Wedge osteotomy on the lateral side is the TX of choice
c) May result from inadequate of vit. D 15) Ff are true of IDM
d) Serum calcium may be normal a) it is an autoimmune dx
e) Daily 10,000units of vit.D orally may be helpful in the mx of renal rickets b) antibodies have been demonstrated against glutamic acid decarboxylase enzyme
c) px are ketosis resistant
8. Ff are characteristic of vit.D def. rickets d) it is commoner in West African blacks than American blacks
a) Low serum calcium e) it is commoner in obese females
b) High serum phosphates
e) Hypotonia

Investigation

Station : X-ray of genu varum - Ricket

ENDOCRINOLOGY

Station : From the station above

History a. There is metaphyseal fraying


b. Ricket is a likely diagnosis
c. ALP will be reduced
d. Blount disease is the most likely diagnosis
Examination

Station

346
Compiled by Obasi. D. C. Chinedu
Laboratory report c. Splinting is part of management
d. Surgery of the calf muscle is important in the management
Calcium – 10mg/dl e. Surgery has poor prognosis in this case.

Alkaline phosphatase – 504iu/L


Station
Phosphate – 3.0mg/dl
A picture of a knock knee

a. Alkaline phosphatase is the upper limit of normal


b. Ricket is a likely diagnosis a. Diagnosis is genu varum deformities
b. Intermalleolar malleolar distance must be monitored
c. X–ray of the wrist is recommended
Station : Thyroid function test of a 5-year boy (Hypothyrosidsm) d. Physiological form of this deformity is arrested between 1 -2
years
TSH – e. Prematurity is a predisopising factor
T3 –
Station : Picture of Genu valgum
T4 –
a. It is genu varum
b. Prematurity is a known cause
c. Physiological form is between 1 – 2years
a. Tyrosine should be given to treat
d. Intercondylar distance is measured
b. Mental retardation is likely
c. The child should be treated for life
Station : Picture of a child with wind-swept deformity i.e right leg bow and left
leg k-shaped.

The differential diagnoses are


Pictures

Station : Picture of 2 sisters with bilateral varum deformities


a. Blount’s disease
a. Both child have bilateral genu valgus
b. The shorter child have enlargement of the wrist b. Nutritional rickets
c. Genu varum
c. Prematurity may be a predisposing factor
d. Achondroplasia
d. 400i.u daily vitamin is therapeutic
e. The aetiology may be genetic
Station : A Picture showing baby with a big round face, skin fold under jaw,
Station mouth open

Picture of a child with bilateral talipes equinovarus


a. Is likely to have mental retardation
b. Cystic hygroma is a possible differential
Station c. Constipation is usual
d. Is likely to have high-pitched cry
a. It is bilateral equinovalgus e. Propylthiouracil is used in management
b. It is bilateral talipes equinovarus

..

347
Compiled by Obasi. D. C. Chinedu
ONCOLOGY
b) May cause polyuria
1) The following are appropriately matched c) Skull x-ray usually show calcification in area of tumor
a) Cyclophosphamide /alopecia d) May present as pituitary gigantism
b) ampicillin/diarrhoea e) Is a rapidly growing fatal malignant tumor
c) carbimazole/agranulocytosis
d) Prednisolone/gastric erosion 8. Peak incidence of the ff condition are as ff
e) Vincristine/peripheral neuropathy a) Measles 3-12months
b) Burkitt’s 3-5 years
2. Neuroblastoma may present with c) Kwashiorkor 2-3 years
a) Abdominal mass d) Retinoblastoma 2-1 years
b) Unilateral exo e) NS 5-7 years
c) Hematuria
d) Lowered VMA level 9) Calcification on a plain radiograph of the abdomen is seen in
e) a) Tuberculous mesenteric lymphadenitis
b) Nephroblastoma
3) Nephroblastoma is c) Acute pancreatitis
a) Sometimes bilateral d) Rhabdomyosarcoma
b) Associated with hemihypertrophy e) Disseminated retinoblastoma
c) Associated with aniridia
d) 10) The ff are true of Wilm's tumor
e) a) It is an embryonal tumor
b) It is sometimes inherited in an autosomal dominant manner
4) Nephroblastoma c) it is frequently bilateral
a) Haematuria may be a presenting feature d) IVU shows downward displacement of the affected kidney
b) Occurs commonly in the preschool age e) With surgery alone the 5 year survival rate is 80%
c) The abdominal mass usually crosses the midline
d) High level of VMA in the urine is diagnostic 11) Concerning Burkitt’s lymphoma
e) Surgery in combination with chemotherapy is the treatment a) It is the common children malignancy in Ibadan
b) Its doubling time is about 72hrs
5) Childhood malignancy c) It is assoc. with t(8:14) abnormality
a) Bilateral retinoblastoma is usually inherited as an autosomal recessive trait d) It may be complicated by CRF
b) Surgery is the TX of choice in early retinoblastoma e) It is found more commonly in well nourished children
c) ALL of childhood carries a better prognosis than AML
d) Commonest site of occurrence of rhabdomyosarcoma is the GUT 12) The ff are seen in px with hyperkalemia
e) R-S cells are characteristic of Hodgkin’s lymphoma a) Seizure
b) Cardiac arrhythmia
6) The ff are drugs known to cause BM depression c) Tall p wave on ECG
a) Aspirin D) Broad u wave on ECG
b) Chlorabucil e) Bizarre qrs complexes
c) Chloramphenicol
d) Phenytoin sodium 13) Craniopharyngioma
e) Cyclophosphamide a) arise from the posterior pituitary gland
b) are the commonest intracranial tumours in children
7) Craniopharyngioma cause c) may compress the optic chiasma
a) Causes optic atrophy d) may become calcified
e) respond well to radiotherapy
ONCOLOGY Picture of a child with bilateral proptosis

Picture Station
Station 1 : Picture of an abdominal tumour (Wilm’s tumour) a. ? AML
b. ALL
a. It is an autosomal recessive inheritance c. Retinoblastoma
b. It wil cause acute abdominal obstruction d. Neuroblastoma
c. Recurrent sucking of the abomen is curative e. Nephroblastoma
d. The diagnosis is necrotizing enterocolitis
e. Surgery is curative Station
Picture of an enlarged neck. The likely diagnosis is
Station 2 a. Cysitc hygroma
Picture (slide) – showing starry and sky appearance b. Atypical mycobacterium
c. ALL
Station 3 : Picture a a child with right jaw swelling and right neck swelling d. Hodgkin Lymphoma
e. CML
The differentials are
Station : Picture of a child with jaw tumour, hair fluffiness and mass wasting
a. Tuberculosis
b. Mumps
c. Burkitt’s Lymphoma a. Burkitt lymphoma is likely if the child is presenting 9 months
d. Hodgkin’s Lymphoma after the symptoms
e. Reactioning lymphadenopathy b. The child should be hydrated before commencing
management
c. There is evidence of wasting
Station

348
Compiled by Obasi. D. C. Chinedu
Station 6 : Haematological report
1. HAEMATOLOGICAL Take a critical at this
o Neutrophil – 20%
Station 1 : Haematological report o Lymphocyte – 70%
o Monocyte – 4%
o Eosinophil – 1%
PCV – 22% Anisocytosis - ++
o Blast – 15%
Neutrophil – 45% Poikilocytosis - ++
o Platelet – 82,000
Eosinophil – 4% Polychromasia - ++
o WBC – 80,000
Basophil – 1% Target cell -
+ Station 7: From the station above
Lymphocytes – 48% Platelet – 170,000 a. AML is the likely diagnosis
Monocytes – 2% b. ALL is the likely diagnosis
c. Patient is likely to die from intracranial haemorrhage
Station 2 : With regards to the station above d. Bone marrow aspiration will benefit the patient
a. Physiological anaemia is a possibility e. There is increased risk of septicaemia
b. Reticulocyte count is expected to be high
c. There may be thrombocytopaenia Station 8 : Haematological report
d. It may be due to allergy o Blast – 2%
e. Aplastic anaemia is a differential o WBC – Marked increase, 98%
The differentials are
Station 3 : Laboratory result a. AML
o Age – 2years b. ALL
o PCV – 21% c. Septicaemia
o WCC – 17,900 d. Malaria
o Platelet – 623,000
o Monocytes - ++ Station 9
o Hypochromia - ++ Microhaematocrit reader with PCV (26%)
o Toxic granulations - The Laboratory report of a 4-year old female
o Neurtrophils - 52%
Conjugated bilirubin – 0mg/dl
o Lymphocytes - 41%
Total bilirubin – 22.0mg/dl
o Monocytes - 7%
Station 10
What is the diagnosis?
a. The PCV in the previous station is 28%
b. A 2 volume EBT is indicated
Station 4 : Malarial parasite blood film
c. Biliary atresia is a likely diagnosis
- ++++ (11-100 HPF), ++++ (1 – 10HPF)
d. G6PD is a likely diagnosis
- Hyperparasitaemia (250,000 parasite/ml or 25% of RBC) e. Phototherapy is the mode of treatment

Q. Contraindications to the use of Quinine as loading dose. Station 11


Glass slide – showing malarial parasite
Station 5 : Haematological report
o PCV - Reduced Station 12 : Read this PCV
o Platelet - Reduced
o WBC – Reduced
a. The PCV is 50 ± 3
b. This is likely in a 2-month old infant
a. Hookworm infestation c. This is likely in a 3-year old boy
b. Acute blood loss d. This is unusual in Tetralogy of Fallot
c. Patient could have used chloramphenicol e. This is unusual in chronic renal failure
d. Patient will benefit from steroids

349
Compiled by Obasi. D. C. Chinedu
GENETICS

1) The ff are autosomal recessive inheritance d) obesity


a) Galactosaemia e) polyuria
b) G6PD deficiency
c) Sickle cell dx 6) The ff enzymes have been implicated in the adrogenital syndrome
d) Congenital extrahepatic biliary obstruction a) 21- hydroxylase def.
e) Sydenhan's chorea b) peroxidase enzyme
c) 11-hydroxylase def
2) Turner’s syndrome d) 7dehydrocholesterol dehydrogenase def.
a) Negative Barr body e) 3-beta hydroxysteroid dehydrogenase def.
b) Hypoplastic nail
c) Cubitus valgus deformity 7) Congenital malformation has been assoc. with the use of
d) Short stature a) the use of diethylstilbestrol
e) Amenorrhea b) thalidomide tx during pregnancy
3) The ff conditions are ass. With Down’s syndrome c) maternal rubella in the 2nd trimester
a) Leukemia d) a positive family hx
b) Hydrocephalus e) mental retardation
c) CHD
d) A large tongue 8) In oesophageal atresia
e) Brushfield spots a) assoc. tracheo-oesophageal fistula worsens the prognosis
b) a hx of polyhydramnios is unusual
4) The ff conditions are assoc. with immune def. c) there is delayed passage of meconium
a) PEM d) cyanosis during feeding is common
b) ataxia telangiectasia e) px are better nursed in the head down position
c) downs syndrome
d) HIV syndrome 9) Complications of cleft palate include
e) cyclosporine A therapy a) recurrent otitis media
b) malpositioning of the teeth
5) Features of klinefelter's syndrome c) speech defect
a) indeterminate sex d) hearing loss
b) persistence of childhood voice e) meningitis
c) short stature

GENETICS Station : Chromosomal karyotyping

a. This is buccal smear


b. Hypotonia is a constant feature
Station c. The sex of the child is male
d. The child is likely to have mental retardation
A picture of child with obliterated limb growing (Syndactyly) e. The child is likely to have hydrocephalus

a. Thalidomide is a likely aetiological agent


b. Has a 50% chance of recurrence Station
c. It could be a complication of amniotic band
d. It is a chromosomal abnormality Picture of cleft lip/palate

a. The aetiology hae autosomal recessive inheritance


Station : Chromosomal studies showing 45X b. There will an associated speech defect if the mother starts
talking to the child early
a. Patient will be 75cm at birth c. If not treated early, malnutrition could be a complication
b. Has Webbed neck d. Chest infection is a complication
c. Has lymphoedema of feet e. A prosthesis should be added for speech F - Is not for speech,
d. Has Single Palmar Simian Crease but for feeding)
e. Has Gonadal dysgenesis

Station : Picture of amputated limb


Station : Picture of a child with cleft lip and palate
a. Thalidomide can cause it
a. There is cleft lip only b. Omphalocoele
b. There is cleft lip and palate c. Amniotic band
c. The child has 95% chance of severe mental retardation d. Chromosomal abnormality
d. The child is predisposed to recurrent chest infections e. 50% recurrence likely
e. Surgery is done after 3 months of life

350
Compiled by Obasi. D. C. Chinedu
RESPIRATORY SYSTEM

1 Bronchial asthma: 11) Common causes of laryngeal obstruction includes


a) is associated with recurrent rather than chronic persistent cough a) Foreign body
b) Is inherited in a Mendelian fashion b) Measles
c) May be associated with flexural eczema c) Retropharyngeal abscess
d) Rarely occurs in infancy d) Tetanus
e) Requires laboratory investigation for diagnosis e) Acute necrotizing pharyngitis

2 LTB can be differentiated from epiglottitis by the following features of the former 12) A 1yr old infant weighing 10kg requires /day
a) Very abrupt onset a) 1000calories
b) Isolation of a virus in a trachea aspirate b) 600-800I.U OF VIT.A
c) Fixed circumferential subglottic narrowing on the lateral X-ray of neck C) 300-500I.UOF VIT.D
d) Inspiratory stridor d) 15mg of vit C
e) Dramatic response to antibiotic therapy E) 6OO-8OOML OF WATER

3 The following are features of lobar pneumonia in infants 13) Lobar pneumonia
a) Chest pain a) Tactile fremitus is diminished
b) Trachea deviation b) Breath sounds are bronchial over affected lobe
c) Bronchial breadth sound allover the chest c) Percussion note is dull
d) Grunting respiration d) Trachea is displaced to the opposite
e) Vomiting e) Air bronhogram may be seen in the consolidated areas

4 Acute epiglottitis
a) Caused by RSV 14) Stridor is a prominent feature of the ff
b) Caused by H.influenza a) Croup
c) Is a benign infection requiring only supportive therapy b) Foreign body in left main bronchus
d) Is a life threatening infection c) Retropharyngeal abscess
e) d) Acute epiglottitis
e) Laryngomalacia
5 Pneumothorax
a) May complicate pertussis infection 15) In empyema thoracis
b) a) There may be no mediastinal shift
c) b) Percussion note is dull on the contralateral side
d) c) The breath sounds are bronchovesicular on the affected side
e) d) A diagnostic needle aspiration via the 2nd intercostal space anteriorly is often
indicated
6. Childhood tuberculosis e) The haematocrit may be low on account of hemorrhage
a) BCG immunization offers absolute immunity
b) Cough may not be a symptom in the pulmonary form 16) Causes of paratracheal shadowing on a chest radiograph include
c) Treatment with oral rifampicin is indicated in TB meningitis a) TB
d) Affectation of the lumbosacral region in TB spine is likely to give rise to b) Hodgkin's lymphoma
paraplegia c) ALL
e) Duration of treatment with anti-TB drugs is for 6 months d) Lung abscess
e) Hyaline membrane diseases
7) Common feature of whooping cough are 17) The ff are true of primary pulmonary TB
a) inc. period of 14- 21 days a) 80% of affected children will have primary complex on presentation
b) Infective in the catarrhal and early paroxysmal stage b) Miliary shadowing is the commonest X-ray finding
c) Mortality is high in any age group c) Cavitation does not occur
d) Is an important cause of marasmus d) It causes digital clubbing in 50% of the patients
e) Penicillin is a drug of choice e) The infective organism can be recovered from the sputum in 25% of the patients

8. Lung abscess in childhood 18. A 2yr old infant presents with a 4day hx of fever, cough and breathlessness,
a) is uncommon in childhood and a pustule on the rt foot. A chest x-ray brought from the referral doctor shows
b) Respond favorably to medical therapy patches of consolidation and pneumatoceles
c) Often associated with finger clubbing a) A bld culture is indicated
d) H.influenza is the most frequently cultured organism from the sputum b) A lumbar is superfluous
e) c) The haematocrit should be checked
d) Antibiotics therapy should consist of penicillin and streptomycin
9) A three year old girl presented with fever, cough and breathlessness. On e) A sudden increase in the respiratory rate is likely to be due to Pneumothorax
examination there was dullness on percussion and diminished air entry over the RT
lower lobe 19) With respect to febrile seizure
a) This child has lobar pneumonia a) It occurs in children age 6mnth -5yrs
b) Empyema is not likely because of the dull percussion note b) Seizure is focal in the simple variety
c) Pulmonary TB is the likely dx c) Seizure rarely last more than 15 mins.
d) Pleural tap is an important part of mx d) Meningitis is a common in our environment
e) A preceding hx of measles is likely e) Long term prophylaxis with antibiotics is advisable after the 1st episode

10) Concerning pulmonary TB 20) The ff pneumonias are likely to form multiple lung abscesses
a) It is compatible with a -ve tuberculin test a) Chlamydia
b) Absent chest signs rules out its dx b) Mycoplasmal
c) It is a cause of bronchiectasis c) Staphylococcal
d) It is confined to the low socio economic class d) Klebsiella
e) Supraclavicular lymphadenopathy is highly suggestive of this dx e) Streptococcal

351
Compiled by Obasi. D. C. Chinedu

21) ARI 25) Ff conditions can cause resp acidosis


a) Is the 2nd commonest cause of under 5 mortality worldwide a) bronchopneumonia
b) Malnourished children with severe ALRI have a mortality rate 2-3 times higher b) status asthmaticus
than well-nourished children c) atelectasis
c) Most deaths are due to URTI d) resp.collapse
d) In Ibadan staphy. and Klebsiella are the predominant bacteria pathogens of ALRI e) RDS
e) Viruses are rare cause
26) Pleural effusion is a complication of
22) Bronchiolitis a) nephrotic syndrome
a) It is common after the age of two months b) pulmonary TB
b) Bronchodilator is useful in the TX c) Burkitt’s lymphoma
c) Arbovirus virus is the predominant aetiological agent d) aspiration pneumonia
d) Heart failure is a common complication e) kerosene poisoning
e) Antibiotics are not useful in the mx
27) Ff organism causes pneumonia in infants
23) A hyperimmune response can be reflected clinically in TB by the appearance of a) pneumocystis carinii
a) Erythema nodosum b) Candida albicans
b) Phlycternular conjunctivitis c) staph. epidemidis
c) Pterygium d) H influenza
d) Pleural effusion e) strep. pneumonia
e) Lung cavities
28) Nasopharyngitis
24) The ff are common types of TB in children in Ibadan a) causes difficulty in feeding in infant
a) TB adenitis b) can be caused by grp A streptococcus
b) TB of the hip c) may be differentiated from allergic rhinitis by the presence of neutrophil in the
c) spinal TB nasal smear in the later
d) miliary TB d) liberal fluid are contraindicated
e) psoas abscess e) is routinely tx with antibiotics

I. RESPIRATORY SYSTEM Station 9


X-ray showing pleural effusion - homogenous (white) opacity with loss
History of costophrenic angle, trachea is shifted
Station 1 : Clerk cough in a 9 month old ( See the Examiner’s list on page ) a. Lung aspiration is required to make a diagnosis i.e to identify the
aetiological agent (False it is pleural aspirate)
Station 2 : From the station above b. Pneumothorax is a complication
a. The informant is the mother c. Percussion note on the affected side is hyperresonant
b. There is history of weight loss d. Tactile fremitus will be reduced on the affected side
c. The cough is of 3 weeks duration
d. Pulmonary tuberculosis is the most likely diagnosis Station 13 : An X-ray of chest and abdomen
e. ESR is diagnostic a. There is pneumothorax
b. There is pneumoperitoneum
Examination of the Chest c. NEC is a likely cause
Station 3 : Do an examination of the respiratory system on this patient. Count
respiratory rate for 30s. Do not percuss. Station 14 : Chest X-ray
a. The trachea is deviated to the left
Station 4 : Concerning the patient you just examined b. Features seen are suggestive of tuberculosis
c. There is pleural effusion on the left
a. There is apical flattening on the right
d. There is consolidation on the right upper lobe
b. The respiratory rate is 50 – 55/min
c. There is gibbus at L1 – L2
d. There are crepitations over the right lower lung zone Pictures in Respiratory System
Station 10: Picture of the back of a child with gibbus – Tuberculous spine
Investigations & Pictures in Respiratory System
Station 5 : Look at this x-ray Station 11: With regards to the picture in station 1
Station 6 a. Cervical spine is affected
a. Trachea is central b. Differential diagnosis include Histoplasmosis duboisii
b. Patient has upper lobe consolidation infection
c. There is associated pneumothorax c. Psoas abscess is a complication
d. Closed thoracotomy tube drainage is important d. BCG is required
e. Blood culture is important e. Thiazine for 6 months is therapeutic

Station 7 : X-ray of the chest Station 12 :


Station 8 a. The gibbus is located at L1 – 2
b. Prednisolone therapy is indicated
a. Needle aspiration with under water seal is important.
c. The child has quadriplegia
b. There is right mediastinal shift
d. Hypotonia is a feature
c. There is left pneumothorax
e. Hyperreflexia is a feature
d. There is cardiomegaly
f. Is likely to be due to tuberculosis
e. Cyanosis treatment with oxygen is mandatory

352
Compiled by Obasi. D. C. Chinedu
HEPATOLOGY

1. The following features are in favour of congenital biliary atresia in diffentiating


this condition from neonatal hepatitis 8) A 3 month old girl presents with abdominal distention. The diff diagnoses should
a) Family history of previous affected sib include
b) Appearance of pale stool with the first ten days of life a) Congenital pyloric obstruction
c) Presence of urobilinogen in urine b) Imperforate anus
d) History of isoimmune hemolytic disease c) Oesophageal atresia
e) Raised serum transaminases d) Hirschsprung's disease
e) Gastroschisis
2) Biliary atresia
a) The presence of conjugated hyperbilirubinaemia is diagnostic 9) A 5day old fullterm infant weighing 3.0kg is admitted with jaundice, total
b) About 5% of patient having EBT will later develop biliary atresia bilirubin is 17.0mg/dl with a conjugated fraction of 1.0mg/dl. MX will include
c) Most affected babies are small for gestational age a) Immediate EBT
d) The incidence is related to paternal age b) Oral phenobarbitone and instruction for bilirubin checked after 48hrs
e) Cholestyramine may be beneficial after surgery c) Phototherapy
d) Glucose drinks and ampiclox syrup
3) Jaundice in the newborn may be caused by e) Reassuring the mother that it is probably physiological jaundice
a) Hepatitis
b) ABO bld grp incompatibilities 10) Neonatal hyperbilirubinaemia can be made potentially more dangerous by
c) Congenital sherocytosis a) acidosis
d) Gallstones b) low serum albumin
e) c) low level of FFA
d) hypoxia
4) A child with pyloric stenosis usually present e) hyperthermia
a) Over the age of 6 months
b) With growth failure 11) In oesophageal atresia
c) With seizure a) assoc. tracheo-oesophageal fistula worsens the prognosis
d) With hypochloremic alkalosis b) a hx of polyhydramnios is unusual
e) c) there is delayed passage of meconium
d) cyanosis during feeding is common
5) e) px are better nursed in the head down position
a) porto-enterostomy is the TX of choice in all cases of conjugated
hyperbilirubinaemia 12) Concerning percutaneous liver biopsy
b) Hirschsprung's dx may present as failure to thrive a) it is contraindicated in the presence of ascites
c) In Blount’s dx serum calcium is usually normal b) hemorrhage is the commonest complication
d) Chronic diarrhoea may predispose to kwashiorkor c) Pneumothorax is a possible complication
e) Anorectal manometry may aid the dx of congenital megacolon d) accuracy of diff biliary atresia from neonatal hepatitis is 100%
e) antibiotics prophylaxis is necessary in px with cardiovascular dx
6. In hypertrophic pyloric stenosis
a) Bilious projectile vomiting is characteristic 13) G6PD def
b) Hypokalemia is a metabolic presentation a) affect only negroes
c) Atropine methyl nitrate is a useful therapy b) is commoner in males than females
d) Myomectomy is the surgical TX of choice c) is a primary result of a defective enzyme in the Hexose Monophosphate Shunt
e) Can result in visible peristalsis from rt to left d) is a common cause of neonatal jaundice in Ibadan
e) causes coombs -ve hemolytic anemia
7) The ff are relevant in the investigation of an infant with prolonged conjugated
hyperbilirubinuria 14) Biliary atresia cause
a) Blood culture a) acholuric jaundice
b) Peroxide haemolysis test b) is associated with pale stools
c) Screen for hepatitis B surface antigen c) hepatocellular fxn is unaffected
d) Oral cholecystography d) clubbing of the digits is an early feature
e) Abdominal ultrasonography e) rickets is a common complication

353
Compiled by Obasi. D. C. Chinedu
o Protein – normal
o Enzymes – Increased
o Urobilinogen – Increased
o Bilirubinuria – Increased
GASTROINTESTINAL SYSTEM & HEPATOLOGY

Station : Bilirubin estimation of a premature very low birth weight

History Bilirubin : Total – 15mg%, Conjugated – 0.1mg%

Station : Clerk loose stool & Jaundice PCV – 30%

G6PD status – Normal

Station : From the above

a. The duration of the loose stool is 3 days a. Single volume EBT is indicated
b. Duration of jaundice is 10days b. Blood culture is indicated
c. Jaundice has been persistent c. Early phototherapy will prevent the need for EBT
d. There is a positive family history d. ABC incompatibility is a likely cause
e. Peroxidase test is indicated.

Station : A 4-day old infant was rushed into the Emergency with significant
jaundice, take a detailed history. Station : 5-day old neonate (full term). Read the PCV and bilirubin results

Station : Station : From the above

Concerning the patient you just clerked, a. The child has severe anaemia
b. The child is polycythaemic and require erythrophoresis
a. The delivery was done at a mission house c. One volume EBT is indicated
b. ABO incompatibility is likely d. Two volume EBT is indicated
c. G6PD deficiency is likely
d. The child is the first of the parents
e. The jaundice started on the 2nd day life Station

Stool M/C/S
Station : Clerk this patient with history of jaundice, oliguria and fever.
o Age – 1 year
o Sex - Female
o Pale brown uniformed mucoid bloody stool
Station : A 6-year old boy presents with jaundice, pallor, body swelling, olguria, o RBC - ++
all of 4-day duration. Take a detailed history to arrive at diagnosis. o WBC - ++
o NO protozoa, ova or cyst
o Culture – Negative (No Salmonella or shigella)

Physical Examination
What is the diagnosis?

Pictures
Investigations

Station : Liver function test

Picture of viral hepatitis

354
Compiled by Obasi. D. C. Chinedu
NEPHROLOGY

1. Chronic renal failure 11) Pertaining to renal replacement therapy in children


a) Polyuria may occur a) The typical prescription call for haemodialysis once or twice per week
b) Hypertension is uncommon b) Haemodialysis is possible in infants as small as 5kg
c) Hypocalcaemia may occur c) Anemia is common with peritoneal dialysis than with haemodialysis
d) Associated anemia is microcytic & hypochromic d) Continous ambulatory peritoneal dialysis is more user friendly than continous
e) Pleural rub may be heard on auscultation cycling peritoneal dialysis
e) The usual glucose content for the isotonic dialysate for peritoneal dialysis is 3.5%
2. Common causes of renal tract obstruction in children are
a) Phimosis 12) The ff lab results are typical of chronic renal failure
b) Nephrolithiasis a) Haematocrit of 20 %
c) Posterior urethral valve b) Serum K+ 2.5 mEq/L
d) Congenital urethral stricture c) Plasma Ca++ 90mg/dL
e) d) Plasma inorganic phosphorus 3 mg/dL
e) Urinary specific gravity of 1010
3) In AGN
a) Circulating immune complexes can be demonstrated 13) Nephrotic syndrome
b) Plasma infusion is a useful adjunct to treatment a) The commonest cause in Nigeria is P. falciparum
c) Heart failure is a possible complication b) Steroids are the drugs of choice in patients with poorly selective proteinuria
d) AntiDNAse is suggestive of recent throat infection c) Hypoproteinemia is a constant finding
e) Hyperkalemia is a common finding d) Quick acting diuretics may be dangerous because of hypovolaemia
e) Spontaneous recovery is common in the idiopathic type
4) Enuresis
a) Is night time bed wetting beyond the age of 5 14) AGN
b) Measurement of bld glucose is indicated a) Hx of preceding sore throat is usual in Nigerian children
c) Urine microscopy is essential before starting tx b) Haemoglobinuria is common
d) Is often associated with petit mal seizure c) Hypertension is a rare occurrence
e) Drug use has no value d) Has a better prognosis than NS
e) The mainstay of TX is bed rest
5) Concerning the composition of fluids used in rehydration
a) Darrow's solution contains 36mmol/l of potassium 15) The ff are true regarding fluid and electrolyte composition in a child
b) Normal saline contains 154meq/l of sodium a) Dehydration in excess of 15% in childhood is compatible with life
c) 8.4% of NaCO3 contains meq/ml of NaCO3 b) Seizures in hypertonic dehydration is common
d) Home made salt sugar solution contains 15-79mmol of sodium and 15mmol of c) Release of ADH is stimulated by an increase in the osmolality
sodium bicarbonate per litre d) The normal plasma osmolality is between 285 - 295 mOsm/kg water
e) Standard oral rehydration salt solution contains 2ommol of potassium chloride e) Isotonic dehydration is the commonest type of dehydration in this environment
and 30mmol of sodium bicarbonate per litre 16) Ascites
a) Fluid thrill is always present
6) Portal hypertension b) Occur in portal hypertension
a) Propanolol is useful in its mx c) In EMF the fluids is an exudates
b) Impalpable spleen makes the diagnosis unlikely d) In abdominal TB the protein content of the fluid is more than 3g/100ml
c) May result from sickle cell anemia e) Is excluded by dullness to percussion in the central part of the abdomen and in
d) Variceal injection sclerotherapyis the TX of choice the flanks
e) May present with pulmonary hemorrhage
17) Hypertension
7) Congenital nephrosis is characterized by a) May occur in early childhood
a) Hypertension b) is often associated with AGN
b) Elevated ASO titer c) May be associated with Cushing syndrome
c) Albuminuria d) Is a known complication of penicillin therapy
d) Hypoproteinemia e) Known complication include renal failure
e)
18) The ff drugs are nephrotoxic in large doses
8) The following are increased in conc. in the serum of nephrotic syndrome patients a) Methicillin
a) Factor B b) Tetracycline
b) Fibrinogen c) Streptomycin
c) Anti thrombin 3 d) Kanamycin
d) Transferrin e) Penicillin
e) Vit.D binding protein
19) The ff values are indications for peritoneal dialysis
a) Urea of 200mg
9) Match: b) Potassium of eight meq/litre
a) ark-upmark syndrome/hypotension c) Bicarbonate of 8meq/litre
b) Beckwith wiedemann syndrome/wilm's tumor d) Sodium of 145meq/litre
c) Potter's syndrome/polyhydramnios e) Chloride of 110meq/litre
d) AR PCKDx/liver fibrosis
e) Barter’s syndrome/hyperkalemia 20) Posterior urethral valves
a) Are found in both sexes but are commoner in boys
10) Concerning HIV nephropathy in children b) A hx of poor urine stream is obtainable in a majority of patients
a) It may manifest before AIDS develops c) Often causes unilateral hydronephrosis
b) HIV associated glomerulosclerosis has a striking predilection for black d) Are best treated by placement of a prosthetic valve
c) Children with focal segmented sclerosis have a better outcome e) Diuresis may occur in the early post-operative period
d) Enlarged kidneys for age & height are consistently found at late stage of the dx
e) Proteinuria is the earliest and the most consistent finding 21) Common causes of hypertension in childhood include
355
Compiled by Obasi. D. C. Chinedu
a) Acute nephritis b) turners
b) Chronic nephritis c) barters
c) 11-hydroxylation defects in CAH d) klinefelter’s
d) Coarctation of the aorta e) pheochromocytoma
e) Renal artery stenosis
30) Ff are true of 24 hr urine collection
22) Hypoglycemia is a common problem in a) it can be started anytime of the morning provided it ends at the same time the ff
a) Low birth weight infants day
b) Marasmic kwashiorkor b) the preservative placed in the container is 5 of sulphuric acid
c) Sickle cell disease c) it can be used to measure creatinine clearance
d) Bronchopneumonia d) the container must be clear and colourless
e) Cerebral malaria e) it gives more quantitative than qualitative analysis

23) The ff are important in the management of chronic renal failure 31. Ff are true of fluid and electrolyte
a) High protein diet in view of their stunted growth a) 2/3 of TBW is ECF
b) Calcitriol b) hormones necessary for control include ADH, renin, angiotensin & aldosterone
c) Calcium carbonate c) can be disrupted by head injury
d) Recombinant human erythropoietin d) insensible water loss is 800mls / day
e) glucose/insulin therapy e) alkalosis can result from persistent vomiting

24) Clues in the dx of CH in the newborn 32) Ff may predispose to UTI


a) Large fontanelle a) female sex
b) Poor feeding and crying b) renal calculi
c) Prolonged neonatal jaundice c) ambiguous genitalia
d) Raised thyrotropin level at 48hrs of age d)
e) Eyelid swelling e)

25) In chronic pyelonephritis


a) There are dilated calyces, cortical scarring unevenly contracted kidney 33) Which of the ff are true of congenital hypertrophic pyloric stenosis?
b) Congenital bladder outlet obstruction is a cause a) presenting symptom is constipation
c) Red cell casts are typical urine finding b) more frequent in females
d) Intravenous pyelogram is an important inv. c) more frequent in the firstborn of the family
e) Hypertension is frequently associated d) metabolic alkalosis is a common complication
e) barium studies are mandatory for the diagnosis
26) Indications for peritoneal dialysis in px with ARF are
a) Uncontrollable hyperkalemia 34) Common causes of persistently elevated blood pressure in children include
b) Serum Na less than 130meq /l a) Cushing's syndrome
c) Fluid overload with pulmonary edema b) Essential hypertension
d) Moderate metabolic acidosis c) Renal disease
e) Hallucination d) coarctation of the aorta
e) thyrotoxicosis
27) the ff factors are important in the pathogenesis of renal osteodystrophy in CRF
a) defective calcium absorption from the gut
b) defect in 25 hydroxylation of vit.D 35) Portal hypertension
c) persistent metabolic acidosis a) does not occur in childhood
d) primary hyperparathyroidism b) may be a long-term complication of EBT
e) defective calcitonin production c) commonest predisposing factor in children in UCH is biliary cirrhosis
d) splenomegaly is a constant finding
28) Polyuria occur in the ff condition e) always carries a poor prognosis
a) DM
b) eosinophil granuloma 36) The ff findings help to distinguish nephrotic syndrome from kwashiorkor
c) hypocalcaemia a) proteinuria
d) CRI b) hypercholesterolemia
e) Cushing syndrome c) hypoproteinemia
d) ascites
29) Ff syndromes have renal involvement e) pallor
a) downs

Examination
URINARY SYSTEM Investigation
History Station : Laboratory result
Station : History taking - Nephrotic syndrome o Na - 140
o K - 4.7
o Cl - 106
Station : From above
o HCO3 - 20
a. Did he recently have blood transfusion
o Protein – 4.8g/dl (Albumin = 1.8)
b. Is he G6PD deficiency
o Cholesterol – 301
c. Symptoms are due to drug reaction to sulphonamide
o Triglycerides – 450
d. Is the cough and breathlessness due t bronchopneumonia
o Creatinine – 0.8%
e. Does he have ARF?

356
Compiled by Obasi. D. C. Chinedu
What one investigation would you do to confirm the diagnosis? - Nephrotic e. IgA Nephropathy
syndrome
What is the likely diagnosis? Station : Urinalysis
o Age – 6.5years
Station : Electrolytes & Urea o Sex - male
o Age - 11years o Glucose – Negative
o Sex - male o RBC - ++++
o Na - 120 o pH - 6.5
o K - 3.2 o Protein - ++
o Cl - 98
o HCO3 - 13 Differential diagnosis
o Protein - 7.1 i. Acute glomerulonephritis
o Albumin - 19 ii. Nephritic Nephrotic
o Creatinine – 3.0 iii. Urinary tract infection
o Urea – 218 iv. Papillary necrosis in HbSS
o ALP - 489
o Ca - 9.2 Station : Urinalysis
o Phosphate – 5.9 o Age – 11 years
o Sex – male
What is the diagnosis? – Chronic renal failure o RBC - +++
o Protein - ++
Station : Electrolytes & Urea result
Na – normal What is the likely diagnosis? – AGN
K - Increased Mention investigations to confirm the diagnosis.
Phospahate – Increased
Calcium - Decreased Pictures
Bicarbonate – normal Station
Urea – Increased Picture of a child with anasarca
Creatinine – Increased a. HIV is a known aetiological agent
b. P. malariae offers a good prognosis
Station c. Protein selectivity of 10 is a good prognosis
a. Salbutamol can be used to correct the electrolyte derangement d. Group A B-hemolytic streptococcus is an aetiological
b. Calcium carbonate can be used to treat one of the biochemical agent
derangements e. Haemolytic Uraemic Syndrome is the commonest
c. Encephalitis is a complications cause.
d. Pericarditis is a complications
e. Haemolytic Uraemic Syndrome is the commonest cause in this Station : Prune Belly Syndrome
environment. a. Nasogastric suctioning can help
b. Necrotising Enterocolitis is a differential diagnosis
Station : Electrolytes and Urea result c. It is autosomal recessive
o Na – 130 d. Can cause acute gastrointestinal obstruction
o K – 2.6 e. Surgical resection is curative
o HCO3 -14
o Cl – 80 Station : The following are common causes of haematuria
o Urea – 65mg% a. Haemoglobinuria
b. Schistosomiasis
a. Glucose and insulin can be used in management of the above c. Wlim’s tumour
b. Patient may have paralytic ileus d. Berger’s disease
c. Serum HCO3 is normal e. Alkaptonuria
d. There is hypernatraemia
e. Report is inkeeping with gastroenteritis in a malnourished child Station : Picture of a child with omphalocoele, septic looking
a. Omphalitis
Station : Sample of urine with blood b. Omphalocoele
Common causes of this condition in this environment include c. Gastroschisis
b. Haemoglobinuria d. The child is likely to be low for gestational age
c. Schistosomiasis e. The child is likely to suffer from hypoglycaemia
d. Alkaptonuria

357
Compiled by Obasi. D. C. Chinedu
INFECTIONS
11) Poisoning
1. Congenital rubella: a) In alkaline diuresis, the pH must be about 8
a) Has an incubation period of 7-10 days. b) Nalorphine may be used as an antidote for salicylate poisoning
b) May be complicated by polyarthralgia. c) Desferrioxamine is a specific chelator of iron
c) Rarely causes deafness. d) Subsequent esophageal stricture and pyloric stenosis may occur with iron and
d) Is an indication for termination if it occurs in the first two months of pregnancy. strong alkali poisoning
e) May cause prolonged jaundice. e) Hypoglycemia may result from cow urine poisoning

2. if a child in the ward's develops measles, the following action are appropriate 12) Bullous impetigo
a) Close the wards to all admissions for one week a) Is not contagious
b) Actively immunized all the other patients against measles b) Intramuscular streptomycin is an effective antibiotic
c) Give gamma globulin to all patients who have not been immunized or had c) The most likely organism is Klebsiella
measles d) Affects both the epidermal and dermal layers of the skin
d) Forbid visiting by the parents until the rash has gone e) Affects only infants
e) Give prophylactic antibiotics to all contacts at home
13) UTI
3. Bloody stools occur in a) Has an equal sex incidence in children under the age of 1yr
a) Constipation b) May be symptomless
b) Intussusception c) Should be considered in the inv. of obscure fever
c) Typhoid fever d) Congenital anomalies of the urinary tract are non predisposing factor
d) Tropical sprue e) Is essentially a benign condition
e) Campylobacter enteritis
14) Common cause of severe anemia in Nigeria children
4. Miscellaneous: the following are true a) Leukemia
a) The silent infection of congenital rubella is commoner b) Malaria
b) Severe psychomotor retardation occurs in congenital toxoplasmosis c) Septicemia
c) CMV can not be transmitted by ingestion of infected breast milk d) Kwashiorkor
d) Genital herpes infection is an indication for delivery by caesarean section e) Empyema
e) The CSF is normal in congenital syphilis
15) Congenital rubella syndrome
5. The following are true of cerebral malaria in Ibadan a) Can be dx at birth
a) It is less common in marasmic children b) Microcephaly is an invariable feature
b) Hypoglycemia is an index of poor prognosis c) Is due to an arbovirus
c) Quinine is the drug of first choice d) Tends to occur in subsequent siblings
d) The demonstration of crescentic gametocytes is essential for diagnosis e)
e) The use of mannitol i.v. increase the chance of survival
16) UTI
6) The following are features of neonatal tetanus a) Is common in the M than F
a) Convulsion b) Cause failure to thrive especially in newborn
b) Apnoiec attack c) Is diagnosed by the finding of bacterial colony count of 50,000/ml of midstream
c) Difficulty in sucking urine specimen
d) Stiffening out attack d) When recurrent is an indication for IVU
e) Acquisition of life long immunity against tetanus e) Is commonly caused by virus

7) The congenital rubella syndrome include 17) Rheumatoid arthritis in childhood


a) Cataracts a) May present with very high fever
b) Deafness b) is not likely if splenomegaly occur
c) CHD c) Corticosteroid is the drug of 1st choice
d) Microcephaly d) May affect atlanto axial joint
e) e) Often cause stunting of growth

8) The TORCH infection include 18) The ff are correct


a) Toxoplasmosis a) P.falciparium 24hrs
b) Syphilis b) P.ovale 96hrs
c) Rubella c) P.malariae 72hrs
d) d) P.vivax irregular
e) e)

9) Newborns may be particularly susceptible to gram -ve infection because they 19) Constipation is not a feature of
have a) Congenital megacolon
a) Low level of IgM b) Typhoid fever
B) High level of IgM c) Abdominal TB
c) Maternal IgG d) Down's
d) No maternal IgG e) Sporadic cretinism
e)
20) Prolonged diarrhoea is a recognized feature of
10) The ff are causes of steatorrhea a) Measles
a) Coeliac disease b) Lactate deficiency
b) Giardiasis c) Nutritional marasmus
c) Tropical splenomegaly syndrome d) Typhoid
d) Kwashiorkor e) Chronic pancreatitis
e) Fibrocystic disease of the pancreas
21) Common causes of constipation include
358
Compiled by Obasi. D. C. Chinedu
a) Poor feeding methods b) elliptocytosis
b) NEC c) ovalocytosis
c) Rectal stenosis d) G6PD def.
d) Abdominal Burkitt's lymphoma e) sickle cell trait
e) Renal tubular acidosis
32) Ff are recognized complication of pertussis infection
22) Measles a) meningitis
a) Is confined to children in the low socio economic grp b) atelectasis
b) The typical rash appear on the 2nd day of the fever c) pleural effusion
c) Koplik spots are best seen on the pharyngeal mucosa d) bronchiectasis
d) Encephalitis is a common complication e) PEM
e) Second attacks are very uncommon
33) Infantile gastroenteritis
23) The ff are recognized complication of acute malaria in the 1st year of life a) is often caused by the rotavirus
a) Hyperpyrexia b) antibiotics tx with cotrimoxazole and metronidazole is often indicated
b) Diarrhoea c) isotonic dehydration is a more common complication than hypertonic
c) Convulsion dehydration in this environment
d) Jaundiced d) may be successfully mx with oral rehydration therapy
e) Metabolic acidosis e) breastfeeding is contraindicated

24) Which of the ff are correctly associated 34) In treating neonatal infections
a) Encephalitis - pertussis a) chloramphenicol may cause circulatory collapse
b) Adenitis - tetanus b) ampicillin is the drug of choice for staph. sepsis
c) Osteomyelitis - TB c) sulphonamide increase the risk of kernicterus
d) Paralysis - polio d) erythromycin may be conveniently given intravenously
e) - measles e) gentamicin may cause nephrotoxicity

25) Pertussis infection 35) Atopic dermatitis


a) Does not affect neonates a) pruritus is marked
b) Incubation period is 4 - 6 weeks b) there is tendency to remission at 3-5yr of age
c) the most infectious phase is the catarrhal phase c) there may be a genetically determine predilection
b) May be complicated by bronchiectasis d) secondary bacterial infection may occur
e) May present with apnea e) topical tx is contraindicated

26) The ff are true of cerebral malaria in Ibadan


a) It is more common in marasmic children 36) Dermatitis
b) Hypoglycemia is an index of poor prognosis a) about 50%of cases of atopic dermatitis resolve by the age of 2yr
c) Quinine is the drug of first choice b) there is no pruritus in infantile atopic dermatitis
d) The demonstration of crescentic gametocytes is essential for diagnosis c) napkin dermatitis due to moniliasis should be tx with exposure and painting with
e) The use of dexamethasone improves chances of survival 1% aq. sol. of gentian violet
d) topical corticosteroid cream may be effective in the tx of seborrheic dermatitis
e) in atopic dermatitis a strong family hx of other atopic manifestation may be
27) In the mx of acute diarrhoea present
a) Assessment of dehydration at the beginning of therapy is usaually enough
b) The recommended rate of administration of oral rehydration solution is 15ml/kg
hr
c) Oral rehydration is not contraindicated in the presence of glucose malabsortion
d) Hyponatreamia dehydration should be TX with hypotonic saline solution
e) Chemotherapy agents are essential adjunct to TX of most doses of diarrhea

28) Major factor causing anemia in the tropics


a) Malaria especially p. vivax
b) Viral and bacterial infection
c) Iron def.
d) Folate def.
e) The hbnopathies

29) The ff are potential complication of tetanus


a) Hypertension
b) Vertebral fracture
c) Ileus
d) Retention of urine
e) Myositis ossificans

30) Concerning measles


a) Transmission is by droplets or by contact with sick children during
exanthematous stage only
b) 2 antigenic type of the virus are known
c) Quarantine is a very helpful prophylaxis
d)
e) When complicated by giant cell pneumonia, cloxacillin and gentamicin are useful

31) The ff genetic traits may protect against malaria


a) membranocytosis
359
Compiled by Obasi. D. C. Chinedu

OTHERS e) Barter’s syndrome/hyperkalemia

1. Kwashiorkor: 12) Match:


a) Hypothermia is a recognized complication a) DM/polygenic
b) Edema is mainly due to protein losing enteropathy. b) neurofibromatosis/autosomal recessive
c) Measles is a recognized precipitant c) Duchenne muscular dystrophy /sex linked recessive
d) The incidence is highest in the first two month of life d) G6PD deficiency/sex linked dominant
e) The birth of a second child to the mother may be a contributory factor e) Achondroplasia….

2 The following programmes would enhance the realization of health for all by 13) A typical sickle cell patient has
the year 2000 in Nigeria a) Has distended abdomen
a) Nomadic education b) Has koilonychia
b) Better life for rural women c) Has macrocephaly
c) A national food policy d) Is pale &jaundice
d) Supply of contraceptive pills at subsidized rates e) Benefits from steroid therapy
e) Tetanus toxoid i.m. each term to all primary 6 school girls
14) A healthy 6month old infant presented with a sudden onset of colicky
3) Breast milk abdominal pain and vomiting
a) Is rich in vit.C a) A sausage shaped mass concave towards the umbilicus may be palpated on
b) Is contraindicated in galactosaemia abd examination
c) Is produced in response to prolactin release b) Mx should include passage of ng tube for decompression
d) Is low in phosphorus compared with cow's milk c) Dehydration may complicate the illness
e) Supplies about 67 calories per 100cc d) Red currant jelly occurs in 2/3 of such cases
e) A laparotomy for reduction is the method of choice
4. Reye's syndrome
a) May result from aspirin ingestion 15) Complication of veinous umbilical exchange bld transfusion include
b) Is usually anicteric a) Abdominal distension
c) Is a form of hepatic failure b) Hyperkalemia
d) May be associated with influenza type b virus c) Metabolic acidosis
e) Is not accompanied by coma d) Cardiac arrhythmias
e) Portal hypertension
5) Poisoning in childhood
a) Alkaline diuresis is a means of enhancing elimination of phenobarbitone 16) Routine immunization is given by the following routes
b) Haematuria is a feature of kerosene poisoning a) Measles/intradermal
c) Hyperpyrexia is a feature of salicylate poisoning b) polio/oral
d) Hypoglycemia may complicatepoisoning by cow's urine c) Triple vaccine /intramuscular
e) Desferrioxamine is used for TX of iron poisoning d) Small pox /intradermal
e) BCG/intradermal
6) An infant with frequent pulmonary infection may have
a) Cystic fibrosis 17) Childhood m & m in Ibadan
b) Hypogammaglobulinemia a) Perinatal mortality rate is an accurate index of under-five clinic services in a
c) Chronic granulomatous dx community
d) b) The commonest cause of perinatal mortality is prematurity
e) c) Gastroenteritis is a significant cause of m&m in the 5-8 year grp
d) EPI is an effective control measure in reducing neonatal mortality rate
7) Sickling of the RBC occur e) Poisoning is a significant cause of morbidity in the 1 - 3 year age group
a) In eight % of black population
b) Only in sickle cell anemia but not in sickle cell trait 18)
c) In thalassemia and sickle cell trait occurring together a) Significant hypocalcaemia is defined as total serum calcium of less than
d) In congenital spherocytosis 7mg/100ml
e) b) Breast milk protein is more???
c) The normal neonatal blood volume is 80 -90 ml/kg
8) Sex linked dx include d) Erb-Duchenne's palsy results from damage to the C5 and C6 nerve roots
a) Hemophilia e) Erythrophoresis is contraindicated in neonatal polycythaemia
b) Cystic fibrosis
c) Sickle cell anemia 19) The following are common causes of coma in children in Ibadan
d) Galactosaemia a) Meningitis
e) b) Brain tumor
c) Cerebral malaria
9) Children with nutritional rickets are likely to have d) DM
a) A prolonged hx of breast feeding e) Tetanus
b) Growth failure
c) Craniotabes 20) Poisoning in childhood
d) all of the above a) Peak incidence is in the age group 5 - 8
e) b) Salicylate poisoning may cause an initial respiratory alkalosis
c) Syrup of ipecac may encourage elimination of the poison by inducing
10) Iron deficiency anemia is commonly found in diarrhoea
a) Premature infant d) In kerosene poisoning gastric washout is contraindicated
b) Adolescent e) Alkaline diuresis is useful in salicylate poisoning
c) Adolescent girl
d) Infant fed with whole milk 21) The ff are live vaccines
e) a) Rabies
b) Pertussis
11) Match: c) Polio
a) ark-upmark syndrome/hypotension d) Measles
b) Beckwith wiedemann syndrome/wilm's tumor e) BCG
c) Potter's syndrome/polyhydramnios
d) AR PCKDx/liver fibrosis

360
Compiled by Obasi. D. C. Chinedu

22) Characteristic findings in uncomplicated congenital hypertrophic pyloric


stenosis include 33) Major causes of death in the 1st month of life are
a) Anorexia a) Malaria
b) Marasmus b) Measles
c) Dehydration c) prematurity
d) Metabolic acidosis d) Anemia
e) Mass in epigastrum e) Birth asphyxia

23) Cyanosis 34) The ff statements about childhood mortality in Nigeria are true
a) The extremities are blue and warm in peripheral cyanosis a) Perinatal mortality rate = no. of stillbirths + 1st wk deaths /1000 live births
b) Is not clinically recognized until reduced HB exceeds 5gm/100ml b) Infant mortality rate = no. of deaths in preschool children / 1000 live births
c) Central cyanosis is due to deficient oxygenation of HB c) About 50% of infant deaths occur in the first month of life
d) Inhalation of 100% oxygen helps to differentiate between pulmonary and d) About 25% of infant deaths occur on the first day of life
cardiac causes e) There is an association between perinatal mortality and maternal age
e) Is an unusual feature of EMF
35) Causes of failure to thrive
24) Causes of metabolic alkalosis are a) TB
a) Prolonged bout of vomiting b) Chronic obstructive uropathy
b) Status asthmaticus c) Maternal deprivation
c) Severe malaria infection d) Congenital pyloric stenosis
d) CRF e) Low birth weight
e) Salicylate poisoning
36) Childhood morbidity and mortality in Ibadan
25) Uncommon feature of kwashiorkor include a) Perinatal mortality is an accurate index of under-5 clinic services in the
a) Ascites community
b) Growth retardation b) The commonest cause of perinatal mortality is prematurity
c) Pedal edema c) Gastroenteritis is a significant cause of perinatal mortality in the 5 - 8 year
d) Splenomegaly age group
e) Dyspigmentation of the skin d) EPI is an effective control measure in reducing neonatal mortality rate
e) Poisoning is a significant cause of morbidity in the 1-3 year age group
26) Oesophageal atresia
a) Associated tracheao-oesophageal fistula worsens prognosis 37) Neonatal hypoglycemia
b) Aspiration pneumonia is a major complication a) Is frequent in the small for age
c) Cyanosis on feeding occur b) Is defined as bld sugar of 40mg /100ml or loss during the 72hrs of life
d) Hx of polyhydramnios is usual c) May complicated EBT
e) Delayed passage of meconium is characteristic d) Has a good overall prognosis even with minimal tx
e) May manifest as apnoeic attacks
27) In eczema
a) Acute lesions are usually dry thickened and scaly with lichenification 38) The ff are rich in folate
b) Commonest type in childhood is atopic dermatitis a) Dark green vegetable leaves
c) Allergic contact dermatitis is a T cell mediated hypersensitivity reaction b) Bananas
d) Prolonged use of hydrocortisone ointment is without complication c) Mangoes
e) A strong family hx of atopic manifestation is present d) Eggs
e) Fish
28) The ff are causes of steatorrhea
a) Coeliac dx 39) Common causes of diarrhoea in children
b) Giardiasis a) Dissacharidase def.
c) TSS b) Gluten sensitivity
d) Kwashiorkor c) Complication of systemic infection such as UTI
e) Fibrocystic dx of the pancreas d) Allergy to cow milk protein
e) Injudicious use of antibiotics
29) In marasmic kwashiorkor
a) Lethargy is the usual form of presentation 40) Hazards of bld transfusion include
b) There is weight for age deficit less than 60% with edema a) Circulatory overload
c) Hypothermia is a frequent occurrence b) Febrile reaction which are common with plastic storage bags
d) gram-ve septicemia may occur c) DIC
e) d) Shock
e) Haemoglobinuria
30) Peak incidence of the ff condition are as ff
a) Measles 3-12months 41) Which of the ff are side effects of salbutamol, a beta stimulant
b) Burkitt’s 3-5 years a) Tremor
c) Kwashiorkor 2-3 years b) Tachycardia
d) Retinoblastoma 2-1 years c) Headache
e) NS 5-7 years d) Hyperglycemia
e) Hypokalemia
31) Rheumatoid arthritis in childhood
a) May present with very high fever 42) Ff conditions and clinical pictures are properly matched
b) is not likely if splenomegaly occur a) meckel's diverticulum/constant colicky abdominal pain
c) Corticosteroid is the drug of 1st choice b) ulcerative colitis/red currant jelly stool
d) May affect atlanto axial joint c) shigellosis/acute mucoid diarrhea
e) Often cause stunting of growth d) intussusception/palpable abdominal mass
e) rectal polyps/recurrent painless bleeding
32) A 1yr old infant weighing 10kg requires /day
a) 1000calories 43) Ff are diff. of arthritis in this environment
b) 600-800I.U OF VIT.A a) sickle cell dx
C) 300-500I.UOF VIT.D b) rheumatic fever
d) 15mg of vit C c) TB
E) 6OO-8OOML OF WATER d) still's dx

361
Compiled by Obasi. D. C. Chinedu

e) perthe's dx b) hypokalemia
c) hypocalcaemia
44) Ff dx conditions and diagnostic procedures are properly matched d) hyponatreamia
a) acute hepatocellular damage serum aspartate transferase e) hypochloraemia
b) bone dx serum acid phosphatase
c) skeletal muscle dx serum creatine kinase 54) The ff are correct
d) G6PD def. serum bilirubin a) 150 - 180 ml of blood is required per kg body weight for full EBT
e) galactosaemia serum glucose 1 phoshate uridyl kinase b) haematocrit of 20% is an indication for blood transfusion in sickle cell
disease
45) Complications of phototherapy c) a cretin is an active child
a) diarrhea d) most Nigerian babies walk by 12 months
b) increased insensible water loss e) neuroblastoma usually damages the kidney
c) bronze baby syndrome
d) skin rashes 55)
e) increased platelet turnover a) penicillin is a drug of choice in empyema
b) ethosuximide is a drug of choice in temporal lobe epilepsy
46) BCG c) actinomycin D is a drug of choice in neuroblastoma
a) live attenuated bovine vaccine d) surgery is not the primary treatment for Tb lymphadenopathy
b) can be given prophylactically only e) normal saline is the fluid of choice for treating severe dehydration in a
c) tuberculin skin test is positive 1/52 after BCG marasmic 3 year old child
d) cannot be given to someone with febrile illness
e) 56) Cataracts may be a feature of
a) congenital syphilis
47. The ff statements concerning childhood morbidity and mortality are true b) toxoplasmosis
a) perinatal period is defined as 28 weeks of gestation and above + the first 7 c) rubella infection in infancy
days of life d) galactosaemia
b) neonatal mortality rate is the number of neonatal deaths per 1000 of all births e) gargoylism
c) infant mortality rate is the number of infant deaths per 1000 of all births
d) diarrhoea is an important cause of death in Ibadan 57) Drug of choice
e) Burkitt's lymphoma is an important cause of death in the preschool age in a) in status epilepticus is phenobarbitone
Ibadan b) in status asthmaticus is subcutaneous adrenaline
c) in petit mal epilepsy is phenytoin
48) In Nigeria chemotherapy against malaria d) in meningococcal meningitis is penicillin G
a) should be provided using halofantrine e) in giardiasis is mepacrine
b) should be given during tx of kwashiorkor
c) should be given to all children under 5 yr of age
d) should be given during tx of ALL
e) should not be given to pregnant women because it will reduce malaria
antibody titre in their babies

49) Bone marrow hypoplasia occurs in


a) folate def. anemia
b) iron def anemia
c) fanconi syndrome
d) parvovirus B19 infection
e) Blackfan-Diamond syndrome

50) The ff are known to be associated with sickle cell anemia in Nigerian
children
a) Hb S+C
b) avascular necrosis of the humeral head
c) zinc deficiency
d) toe clubbing
e) microfinger

51) A known sickle cell anemia patient is admitted with sever drowsiness, rectal
temperature of 39.5 C, PCV of 18%, deep jaundice and generalized hypotonia
and absent deep tendon reflexes on the left side. This child
a) may have had a cerebrovascular accident
b) may have cerebral malaria
c) needs urgent packed cell transfusion
d) needs daily penicillin and streptomycin injections
e) is likely to have bone marrow erythroid hyperplasia

52) A 3 year old child presents with fever, malaise, bleeding gums and
splenomegaly. His PCV was 24%, WBC was 28,000 and platelet count was
20,000.
a) The blood cell count suggests a diagnosis of acute leukemia
b) normal white cell count would have ruled out acute leukemia
c) corticosteroids are used for inducing remission in acute leukemia
d) in acute leukemia, lumbar puncture is not indicated unless there are
neurological signs
e) the thrombocytopenia is due to consumption coagulopathy

53) A 1 year old child weighing 8 kg presenting with diarrhoea and vomiting
will most likely have the ff biochemical abnormalities
a) acidosis

362
Compiled by Obasi. D. C. Chinedu

2000 resit Exams 9. Differential diagnosis of a 4yr old child with cyanosis and reduced
1. Indication for oxygen therapy in supportive management of effort tolerance includes
pneumonia includes
a) Tetralogy of Fallot
a) Sao2 <90% b) Pulmonary stenosis
b) Tachypnoea > 20/min for the age c) Tricuspid atresia
c) Central cyanosis d) Ventricular septal defect
d) Severe lower chest wall indrawing e) Asthma
e) Grunting

10. Predisposing factors to infective endocarditis


2. The following is/are appropriate in a 2 year old with asthma
a) Male Sex
a) MDI only b) Haemophilia
b) MDI + VSD c) Rheumatic heart disease
c) MDI +VSD + Mouthpiece d) Dental caries
d) MDI + VSD + Facemask e) Corrective heart surgery
e) Nebulizer + Face mask

11. Concerning Diabetes Mellitus in children


3. Concerning bronchiectasis
a) It is associated with hyperthyroidism
a) Tb and Pertussis are common causes b) It can present for the first time as DKA
b) Antibiotics is the main stay of treatment c) OGTT is mandatory in making a diagnosis
c) Symptoms are similar to that of lung abscess d) Can be caused by prolonged steroid therapy
d) Cor Pulmonale is a late complication e) Prolonged insulin use can lead to a cure
e) Type III (saccular) has the best prognosis

12. Concerning Vit. D and Calcium metabolism


4. Bronchiolitis
a) Deficient calcium absorption leads to secondary
a) Wheeze is present hyperparathyroidism
b) Poor prognostic indicator is underlying congenital heart defect b) Deficient Vit. D leads to craniotabes in infancy
c) Has poor prognosis, morality is >10% c) Treatment of hypocalcaemia requires the use of IV 10% Ca
d) Adenovirus is the commonest cause Gluconate /ml/kg to prevent complications
e) Diazepam syrup is indicated when the patient is restless d) Vit. D deficiency is a common cause of BLOUNT’S Disease
e) The use of calcium supplement is essential in the management of
nutritional rickets
5. The following are differentials of a military picture on Chest X-ray

a) Pneumocystic Carini Pneumonia 13. Concerning Congenital Adrenal Hyperplasia


b) Eosinophilic Pneumonia
c) Chlamydia Pneumonia a) Most common manifestation is ambiguous genitalia
d) Sarcoidosis b) Salt loosing syndrome occurs in 11 Hydroxylase deficiency
e) Chicken Pox Pneumonia c) It is a cause of short stature
d) Hypertension does not occur
e) 20,22 Desmolase deficiency is not compatible with life
6. Common Causes of Mortality in Under 5 age group include

a) Measles 14. Seizures


b) Malaria
c) Pneumonias a) Complex febrile seizure has a duration of between 10 – 15 mins
d) Gastroenteritis b) Recurrent febrile seizure is an indication for developing epilepsy
e) Malnutrition in later life
c) EEG is necessary for definitive diagnosis
d) West syndrome has a poor prognosis
7. The following are correctly matched e) Neurofibromatosis is inherited as Autosomal recessive

a) Single, soft pulmonary closure sound – Severe pulmonary Stenosis


b) Loud pulmonary closure sound – Pulmonary Hypertension 15. Concerning Pott’s disease
c) Ejection systolic murmur loudest @RUSE – Aortic Incompetence
d) Continuous murmur @LUSE – Ventricular Septal Defect a) flaccid paralysis is a feature
e) Diastolic apical Murmur – Mitral incompetence b) most commonly affected is the cervical vertebrae
c) damage starts at the spinous process
d) steroid therapy is essential
8. The following measures are appt. in the treatment of e) gibbus disappears following treatment
hypercyanotic spell

a) Cardiac position 16. Concerning GBS


b) Oxygen by face mask
c) Oral propanolol, 1mg/kg/dose a) paralysis is often symmetrical
d) Antibiotics b) preceding viral infection is typical
e) Plasma Exchange c) clear cut sensory level exists on the trunk
d) CSF protein 800mg/dl and WBC of 2cells/mm is suggestive
e) Plasmaphoresis is indicated when respiratory arrest is imminent.

363
Compiled by Obasi. D. C. Chinedu

17. Common causes of Cerebral Palsy in Nigeria include 25. Concerning UTI

a) Severe Birth Asphyxia a) All confirmed cases must be investigated


b) Severe Neonatal Jaundice b) Staph. Aureus is the commonest cause in Ibadan
c) Pyogenic meningitis c) Presence of an indwelling catheter implies a complicated case
d) Intrauterine factors d) Co-trimoxazole is the 1st line drug in Ibadan
e) Poliomyelitis e) Bowel training is a preventive measure

18. Concerning Malaria 26. in a 3 month old infant, the ff are correct

a) Artesunate is a 2nd line drug in the treatment of uncomplicated a) Jaundice is less common in breastfed infants than those on
malaria formula feeds
b) Chemoprophylaxis is indicated in children who recover from b) Breastmilk contains more Vit D than cow milk
cerebral malaria c) Incidence of URTI is lower in breastfed children
c) Oral quinine is a drug of choice used in cerebral malaria d) Breast milk is the best source of Fe
d) Severe malaria is common in malnourished children e)
e) Acute renal failure is a known complication

27.
19. HbSS
a) SGA looses 5-10% bodyweight and grows rapidly
a) Incidence in Nigeria is about 3% b) Clavicular fracture in one arm may affect its participation in
b) Fluid restriction is indicated in vaso-occlusive crisis Moro’s Reflex
c) Cellulose –acetate paper is used to confirm diagnosis c) Lecithin-Sphingomyelin ratio is about 1.5 in preterm
d) Pyrimethamine is indicated for prophylaxis d) Term infants gain 30mg per day
e) Daily folic acid/iron is indicated e) OFC increases by 0.8-1cm weekly in preterm

20. In Tumor Lysis Syndrome 28. Concerning AGN

a) Fluid administration at daily maintenance value is adequate a) it is caused by Group B B-haemolytic streptococcus
b) Allopurinol is useful in preventing it b) ASO titre is higher in pharyngeal related AGN than in pyoderma
c) Uric acid of 11mg/dl is in keeping c) It has a better prognosis than Nephrotic syndrome
d) IV NaHCO3 is used in its treatment d) Proteinuria of more than one year is normal
e) Can occur in Acute Leukaemia e) There is a transient hypocomplimentaemia

21. Concerning Fluid Administration 29. Concerning chronic renal failure

a) Darrow’s solution contains 36mmol/L a) Renal replacement therapy is indicated when GFR is 30ml/kg or
b) Home made salt and sugar solution contains 45mmol/L of K and less
20mmol/L of NaHCO3 b) CCPD more user friendly than CAPD
c) SSS contains 40-90mmol of NaHCO3 c) The best phosphate binder is aluminium hydroxide
d) Normal saline contains 154meq/L d) Hypertension increases rate of progression
e) 8.4% of NaHCO3 contains 1meq/L e) Haemodialysis is indicated once per week

22. HIV 30. The following are usual causes of sustained hypertension in
under-10
a) In a 12month old child ,western blot can be used to diagnose
b) Risk of mother to child transfer is about 35 – 50% in Africa a) Chronic glomerulonephritis
c) Measles vaccine is contraindicated in a child born to a HIV mother b) Chronic pyelonephritis
d) Breastfeeding is contraindicated c) Obstructive uropathy
e) CD4:CD8 ratio is <1 d) Essential hypertension
e) Autosomal dominant polycystic kidney disease

23. Leukaemia
31. Match the following
a) Acute is commoner than chronic in children
b) Down’s syndrome increases predisposition a) Toxoplasmosis – conjugated hyperbilirubinemia
c) Abnormal bleeding is common in leukaemia b) Rubella – microcephaly
d) Blasts in peripheral film is diagnostic c) Syphilis – Hutchinson’s tooth
e) Fever occurs in most cases d) CMV – Scattered Intracranial calcification
e) Herpes – mental retardation

24. Burkitt’s lymphoma


32. Miscellaneous
a) Commoner in the maxilla than in the mandible
b) M>F a) Cortical thickening is seen on plain radiograph in Chronic
c) There is generalised lymphadenopathy in endemic type osteomyelitis
d) Modified Ziegler’s regimen used in the treatment b) Limited barium swallow is useful in a 3month old child with TOF.
e) Prognosis is better in abdominal than in the jaw form

364
Compiled by Obasi. D. C. Chinedu

c) Tuberculosis of the lung can present with Pleural effusion on plain b) About 10% progress to chronic state
chest radiograph c) Primary Liver Cell Carcinoma is a complication
d) Ultrasound can be used to outline the lungs d) Case fatality in Fulminant hepatitis is 80%
e) Boot shaped heart is seen on plain chest radiograph in VSD e) It is faeco-oral in transmission

33. Meconium Aspiration syndrome

a) it occurs in preterm babies


b) Broad spectrum antibiotics is indicated
c) hyperinflation of the lungs occurs till school age
d) Features of bronchopneumonia are seen on chest radiograph
e) Pneumomediastinum is a complication

34. The following are causes of unconjugated jaundice and severe


anaemia in a 4day old.

a) G6PD Deficiency
b) Septicaemia
c) ABO incompatibility
d) Fracture of the femur
e) Infant of a Diabetic mother

35. Concerning Measles

a) Koplik spots are diagnostic


b) the fever crashes at the onset of the rash
c) Laryngotracheobronchitis is a complication
d) Stevens – Johnson syndrome is a differential diagnosis
e) Parenteral drugs are contraindicated in the acute phase

36. Causes of vomiting and diarrhoea in the first year of life include

a) Furunculosis
b) Malaria
c) UTI
d) Bronchopneumonia
e) Pytiriasis Vesicolor

37. The following are multifactorial in aetiology

a) Cleft lip
b) Sickle cell Anaemia
c) Congenital dislocation of the hip
d) Exomphalos
e) Achondroplasia

38. The following are teratogenic

a) Phenytoin
b) Chloroquine
c) Paracetamol
d) Thalidomide
e) Mercury

39. Histological features in chronic active hepatitis include

a) Mononuclear cell infilteration


b) Piecemeal necrosis
c) Bridging necrosis
d) Expansion of the portal tracts
e) Rosette formation

40. Concerning Acute Hepatitis B

a) Presence of AntiHBsAg indicates a resolution

365
Compiled by Obasi. D. C. Chinedu

MCQ PAPER 1 MCQ PAPER 2


1-Concerning gastroenteritis (in the UK):
1-Concerning bronchiolitis: A. It can be fatal.
A. Epidemics often occur during winter. B. The most common cause is Salmonella spp. infection.
B. A chest radiograph usually shows a poorly inflated chest. C. Broad-spectrum antibiotics are usually indicated.
C. Poor feeding is an indication for treatment with ribavirin. D. A degree of lactose intolerance may follow gastroenteritis.
D. The disease is more benign in infants born prematurely. E. Blood pressure is a sensitive indicator of dehydration.
E. Respiratory syncytial virus is the most common causative virus.

2-Skin peeling occurs in the following conditions:


2-The following are single gene disorders: A. Infectious mononucleosis.
A. Cystic fibrosis. B. Kawasaki disease.
B. Type 1 diabetes mellitus. C. Scarlet fever.
C. Duchenne muscular dystrophy. D. Toxic shock syndrome.
D. Marfan syndrome. E. Erythema infectiosum (slapped cheek disease).
E. Turner syndrome.

3-The risk of neonatal jaundice is increase by: 3-In neonatal RDS (respiratory distress syndrome):
A. Prematurity. A. Surfactant is useful in the treatment.
B. Trisomy 21. B. It is rare in infants below 28 weeks gestation.
C. Elective caesarean section. C. Antenatal steroids are beneficial.
D. Congenital hypothyroidism. D. Maternal opiate abuse increases the risk.
E. Cephalahaematoma. E. Maternal diabetes increases the risk.

4-In the emergency treatment / rescusitation of infants:


A. Tension pneumothorax is diagnosed by chest x-ray. 4-In children with cerebral palsy:
B. Intraosseous needles must not be used under 1 year old. A. All have a degree of learning impairment.
C. Oxygen should be administered in anaphylaxis. B. Birth asphyxia is the most common cause.
D. Cardiac arrest is usually secondary to hypoxia. C. Hand preference before 12 months can be a useful sign.
E. Cardiac compressions are contraindicated when the heart rate is 4 D. The spastic form is the most common clinical type.
E. Management should adopt a multidisciplinary approach.

5-Recognised complications of measles include:


A. Encephalitis. 5-The following conditions are haemaglobinopathies:
B. Febrile convulsions. A. G6PD deficiency.
C. Diarrhoea. B. von Willebrand disease.
D. Infertility in males. C. Spherocytosis.
E. Corneal ulceration. D. Thalassaemia.
E. Sickle cell disease.
6-Concerning asthma:
A. Inhaled steroids are useful in the treatment.
B. Wheeze on auscultation is pathognomonic. 6-Theses neuromuscular disorders are "genetic diseases":
C. It is seldom a cause of hospital admission. A. Duchenne muscular dystrophy.
D. Exacerbation of symptoms can occur with exercise. B. Myotonic dystrophy.
E. Sufferers should be excluded from sport at school. C. Spinal muscular atrophy.
D. Guillain-Barre syndrome.
E. (Typical) Myasthenia gravis.
7-The following can be the cause of a non-blanching rash:
A. Idiopathic thrombocytopenic purpura.
B. Systemic lupus erythematosis. 7-Concerning oncological disease in children:
C. Henoch-Schönlein purpura. A. Acute myeloid leukaemia is the most common leukaemia.
D. Haemolytic uraemic syndrome. B. Most brain tumours are metastases from other primaries.
E. Meningococcal sepsis. C. Wilm's tumour can present with haematuria.
D. Neuroblastoma can present in the first month of life.
E. Squint can be the presenting feature of retinoblastoma.
8-Regarding nocturnal enuresis:
A. Is more common in girls.
B. Can be a presenting feature of diabetes melitus. 8-In pyloric stenosis:
C. Can be a presenting feature of diabetes insipidus. A. Is more common in boys.
D. Emotional disturbance can be a cause. B. Surgery is the usual treatment.
E. Desmopressin is used in the treatment. C. Infants often have a metabolic acidosis.
D. Constipation can be a presenting feature.
9-Concerning childhood immunisations (in the UK): E. A barium meal is usually performed to confirm the diagnosis.
A. BCG vaccine should be given to children with HIV.
B. A history of febrile convulsions is a contraindication.
9-The following can cause failure to thrive:
C. Meningococcal C vaccine is routinely offered.
A. Coeliac disease.
D. Polio vaccine is usually administered orally.
B. Cystic fibrosis.
E. A fever Hib vaccine means futher Hib vaccinations are
C. Emotional neglect.
contraindicated.
D. Gastro-oesophageal reflux.
E. Patau syndrome.
10-The following congenital heart lesions are "cyanotic":
A. Ventricular septal defect. 10-The following are causes of polyhydramnios:
B. Atrial septal defect. A. Maternal diabetes mellitus.
C. Patent ductus arteriosus. B. Potter syndrome.
D. Tetralogy of Fallot. C. Anencephaly.
E. Transposition of the great arteries. D. Oesophageal atresia.
E. Polycystic kidneys.

366
Compiled by Obasi. D. C. Chinedu

1-Concerning brain tumours in children:


MCQ PAPER 3 A. A sixth cranial nerve palsy can be the presenting feature.
B. The majority are metastatic from other tumors.
1-The following congenital abnormalities are deformations: C. They are the most common solid organ tumor.
A. Coarctation of the aorta. D. Brain stem glioma usually present with personality change.
B. Haemangioma. E. Medulloblastomas can metastasize to the spine.
C. Talipes.
D. Cleft lip.
E. Imperforate anus. 2-In depression in childhood:
A. A family history is common.
2-Regarding anorexia nervosa: B. Suicide attempts are likely to be successful.
A. Is never fatal. C. Sleep is typically impaired.
B. Occurs exclusively in females. D. Is more common in pre-pubertal boys than pre-pubertal girls.
C. Amenorrhoea can be a feature. E. Antidepressant medication is ineffective.
D. Growth hormone is typically elevated.
E. Distorted body image is common.
3-Risk of developmental dysplasia of the hips is increased by:
3-Systemic lupus erythematosus (SLE): A. Male sex.
A. Is more common in females. B. Asian ethnic origin.
B. Psychosis is a recognized manifestation. C. Positive family history.
C. There is usually an associated thrombocytosis. D. Oligohydramnios.
D. Is more common in Caucasians than Africans. E. Breech presentation.
E. Autoantibodies against double-stranded DNA are frequently found.
4-The following exhibit autosomal dominant inheritance:
4-In children with eczema: A. Gilbert syndrome.
A. Topical steroids are the first-line treatment. B. Becker muscular dystrophy.
B. IgE is raised in most cases. C. Turner syndrome.
C. Parental atopy is a positive risk factor. D. Tuberous sclerosis.
D. In infants the extensor surfaces are mostly affected. E. Cystic fibrosis.
E. Herpes simplex can cause serious infection.
5-Compared with cow's milk, human breastmilk contains:
5-Concerning congenital pseudohypoparathyroidism:
A. Less sodium.
A. Inheritance is autosomal dominant.
B. Less calcium.
B. Short 4th metacarpals are characteristic.
C. Less protein.
C. Parathyroid hormone levels are usually increased.
D. Less fat.
D. Calcium levels are decreased.
E. Less carbohydrate.
E. Urine analysis is helpful in confirming the diagnosis.

6-The following are inherited in an X-linked manner: 6-Regarding cystic fibrosis:


A. Duchenne muscular dystrophy. A. It is most common in sub-Saharan Africa.
B. Haemophilia A. B. The most common mutation is called delta-F508.
C. Cystic fibrosis. C. The responsible gene is located on chromosome 14.
D. Sickele cell disease. D. Can present in neonates with intestinal obstruction.
E. Hunter syndrome. E. Can be investigated by measuring sweat electrolytes.
7-Concerning chickenpox:
A. The incubation period is 7 days. 7-An 8 month old child would be expected to:
B. Aspirin is a useful anti-pyretic. A. Roll from back to front.
C. Transmission is usually by contact or airborne. B. Sit unsupported.
D. Encephalitis is a recognized complication. C. Transfer objects between hands.
E. There is no effective vaccine. D. Build a tower of 3 blocks.
E. Have full head control.
8-Regarding Kawasaki's disease:
A. Conjunctivitis is a recognized feature.
B. It is caused by a spirochaete organism. 8-The following are recognised causes of clubbing:
C. It is a cause of coronary artery aneurysms. A. Cystic fibrosis.
D. Fever is not a typical feature. B. Familial.
E. Is associated with profound thrombocytopaenia C. Bacterial endocarditis.
D. Fallot's tetralogy.
9-The following cause a microcytic anaemia: E. Ulcerative colitis.
A. Ulcerative colitis.
B. Pernicious anaemia.
C. Methotrexate therapy. 9-The following are causes of short stature:
D. Folate deficiency. A. Klinefelter syndrome.
E. ß-Thalassaemia. B. Marfan syndrome.
C. Emotional deprivation.
10-A normal two year old child would be expected to: D. Hypothyroidism.
A. Drink from a cup. E. Achondroplasia.
B. Build a tower of 5 blocks.
C. Copy a circle.
D. Kick a ball. 10-Concerning systemic lupus erythematosus (SLE):
E. Hop on one foot. A. It causes a characteristic "butterfly rash".
B. Is more common in males.
C. Arthralgia is a rare feature.
MCQ PAPER 4 D. Maternal SLE can cause heart block in the neonate.
E. Anti-nuclear antibodies are usually present.

367
Compiled by Obasi. D. C. Chinedu

Pediatrics Questions
1.The following are true concerning isolated pulmonary stenosis :
MCQ PAPER 5 a) patients are usually cyanosed
b) patients often present in heart failure
1-Concerning epilepsy in children: c) pulmonary artery pressure is high
A. There is an association with cerbral palsy.
d) right ventricular pressure is high
B. Most forms of childhood epilepsy require lifelong treatment.
e) dialated pulmonary artery on chest X-ray usually indicates that
C. Infantile spasms may respond to corticosteroid treatment.
D. Consciousness is usually impaired in absence seizures. stenosis is the at the valvar level
E. A post-ictal hemi-paresis may occur.
2.The following investigations are useful in distinguishing between rheumatic
fever and infective endocarditis:
2-The following are common findings in Trisomy 21: a) Full blood count
A. Learning difficulties. b) Erythrocyte sedimentation rate
B. Protruding tongue. c)Blood culture
C. Congenital heart defects. d)Chest X-ray
D. Single transverse palmar crease. e)Echocardiography
E. Hypertonia.
3.The following features are suggestive of decompensation in a 1 yr old boy
3-Regarding rheumatic fever: with ventricular septal defect:
A. It occurs following a staphylococcal infection. a) weight of 5kg
B. May present with chorea (a movement disorder). b)Respiratory rate of 68/min
C. Erythema marginatum is a "major" diagnostic criterion. c) Excessive sweating
D. It requires treatment with immunoglobulin. d) Loud (4/6) murmurs
E. Renal impairment is the most serious long-term risk. e) hepatomegaly of 2 cm

4-UK infants are routinely immunised against: 4.Features of transposition of the great arteries include :
A. Meningococcus type B. a)Cyanosis from birth
B. Diphtheria. b)Heart failure
C. Cholera. c)Egg on side appearance on chest X-ray
D. Polio.
d) Right axis deviation on ECG
E. Pertussis.
e)Good response to medical treatment
5-These neuromuscular disorders are inherited conditions:
A. Duchenne muscular dystrophy. 5.Features highly sensitive as predictors of pneumonia in a 12mnt old child
B. Becker muscular dystrophy. with cough and nasal catarrh include;
C. Spinal muscular atrophy Type 1 (Werdnig-Hoffmann disease). a) Respiratory rate of 40 breath per min
D. Guillain-Barre Syndrome. b)Lower chest wall indrawning
E. Dystrophia myotonica. c)intercoastal indrawing
d)Flaring alae nasae
6-Concerning ADHD (attention deficit hyperactivity disorder): e) Temperature 40 degree celcius
A. Impulsive behaviour is a feature.
B. Is associated with a higher rate of expulsion from school. 6.Concerning bronchiectasis
C. Is associated with a higher rate of teenage pregnancy.
a)Cylindrical type is the most severe
D. Is associated with a higher rate of tic disorders.
b)Cough is worse on waking up in the morning
E. May respond to treatment with stimulants such as amphetamines.
c)Lung abcess is a complication
7-The following are causes of jaundice: d)It is incurable
A. Prune belly syndrome. e)Chest physiotherapy offers no meaningful treatment
B. Wilson's disease. 7.The following are associated with poor prognosis in bronchiolitis
C. Pityriasis rosea. a)Post maturity
D. Alagille Syndrome. b)Respiratiory rate over 80/min
E. Alpha-1-antitrypsin deficiency. c)Low birth weight
d)Presence of expiratory rhonchi
8-The following drugs used in asthma are beta-agonists: e)Underling congenital
A. Salmeterol.
B. Salbutamol. 8.The following are correctly matched
C. Ipratropium bromide.
a)Streptomycin hepatotoxicity
D. Fluticasone.
b)Rifampicin Ataxia
E. Terbutaline.
c)Isonizaid pheripheral neuritis
9-Neonates in the UK are routinely screened for the following: d)Thiacetazone Optic nerve damage
A. Hirschprung's disease. e)Ethambutol pschycosis
B. Phenylketonuria.
C. Fragile X syndrome. 9.The following statements are true of larynotracheobronchitis
D. Hypothyroidism. a)Occur most commonly in children less than 3 yrs of age
E. Congenital rubella infection. b)It is the least common form of group
c)Dexamethazone is useful in its treatment
10-Regarding the condition of patent ductus arteriosus (PDA): d)Lateral radiograph of the neck shows sub-glottic narrowing
A. Chest CT is the optimum radiological investigation. e)The commonest aethiological agent is respiratory syncytial
B. May cause a "collapsing pulse".
C. May be treated by open surgical ligation.
10. Typical neurological finding in tuberculosis of the spine with cord
D. May be treated medically using indomethacin.
E. May be treated by catheter trans-venous umbrella occlusion. compression include
a)Hypertonia
b)Hyperflexia
c)Normal muscle power

368
Compiled by Obasi. D. C. Chinedu

d)Absent ankle clonus 20.In kwashoikor;


e)Loss of sensation over the area supplied by the nerve which arise adjacent to a)Oedema is mainly due to a protein losin anteropathy
the affected vertebra b)the incident is highest in the first year of life
c)the birth of a second child to the mother may be a contributory factor
11.The complication of pertusis may usually include the d)impaired cognitive function is a recognized long term complication
a)malnutrition e)constipation is a common problem
b)Intracerebral haemorrage
c)Sinusitis 21.Infantile gastroenteritis ;
d)Splenic rupture a)generous milk feeds are essential in the early state of the disease to make up
e)Acute tubular necrosis for losses in the stools.
b)drowsiness out of proportion to the degree of clinical dehydration suggest
12.The following disorders result in hypotonia hypertonic dehydration
a)Cerebral palsy secondary to birth asphaxia c)marked sunken eyes and diminished skin turgor suggest that dehydration is
b)Poliomyelitis about 5%
c)Duchene muscular dystrophy d)isolation of pathogenic E.coli is an indication for antibiotic treatment
d)Wernig Hoffman’s disease e)convulsions may be avoided by rapid correction of hypernatraemic
e)Erb’s palsy dehydration

13.A 3 day old boy presents with 3 day history of fever ,repeated seizure and 22.Viral hepatitis in childhood
coma in the previous hour. Cerebrospinal fluid examination was a.occurs in 10% of children before the age of 10years
remarkable only for sugar level of 10mg/dl(blood sugar 25mg/dl) b.majority are anicteric
a)Cerebral malaria c.commonest cause in Nigeria is hepatitisB virus
b)Pyogenic meningitis d.If due to hepatitis A,it may progress to chronic liver disease
c)TB meningitis e.Hepatitis C virus is the commonest cause of transfusion related hepatitis
d)Febrile combustion
e)Typoid psychosis 23.Hypotonic dehydration in infants
a.may cause convulsion because of cerebral oedema
14.The following seizure type are well matched with drugs of first choice for b.occurs commonly in malnourished children
management c.gives a characteristic doughy feel to the skin ]
a)Absence seizure; clonazepam d.is complicated by metabolic alkalosis
b)Complex partial seizure; clonazepam e.should be corrected by infusion of dextrose solution
c)Generalized tonic clonic seizure; phenobarbitone
d)Infantile spasm; Adenocorticotropic Hormone 24.Hypothyroidism is characterized by the following ;
e)Febrile seizures. a.Excessive sleeping
b.Poor grades in school
15.With respect to cerebral palsy c.Dry crease skin
a)Common causes in this environment includes post natal viral encephalitis d.persistent diarrhea
b) Mental retardation occurs in all forms of the disease e.Tachycardia
c)Diagnosis is not possible until the age of 1 year
d)Diagnosis is not definite in the absence of skull X –ray 25.Diabetes Mellitus in children;
e)The main stay of management is physiotherapy a.May follow prolonged steroid therapy
b.Intake of highly refined sugars should be encouraged
16.The initial investigation of a 4year old asymptomatic girl presenting with a c.Level of glycosylated haemoglobin (HbAIc) should be measured monthly
PCV of 2% in this environment should include d.Performance of an oral glucose tolerance test(OGTT) is mandatory to
a)Full blood count confirm diagnosis
b)Blooc film for malaria parasite are typical e.May be associated with hyperthyroidism
c)Heamoglobin genotype
d)Serum iron level 26.The following signs of rickets are correctly matched with their meanings:
e)G6PD screening a.genu varum : knock-knees
b.genu valgum : bowing of the legs
17.The following haematological parameters are typical of haemolytic process c.rachitic rosaries :coate condral enlargement of the ribs
a)Reticulocyte count 6% d.caput quadratum :macrocephaly
b)Anisocytosis e.craniotabes : widened anterior fontnelle
c)polychromasia
d)microcytosis 27.The following endocrine condition can be diagnosed by by urinalysis
e)hypersegmentation of neutropils a.phaeochromocytoma
b.Diabetes insipidus
18.With respect to HIV infections in children c.Androgenital syndrome
a)The commonest route of transmission is through contaminated instrument d.Diabetes mellitus
b)breast feeding is a recognized mode of transmission e.Rickets
c)positive antibody titre in a 7 months old child is indicaticative of infection
d)Initial presentation at the age of 3 to 4 years is unusual 28.The following are well matched :
e)Accepted treatment schedule includes the combination of 2 protease a.apnea of prematurity : theophline therapy
inhibitor drugs b.oesophageal atresia : Gliohydraminos
c.trisomy 21 : single palmar crease
19.The following are indications of severity in malaria; d.congential hypothyroidism : tachycardia
a)Fever e.disseminated intravascular coagulation
b)Acute cortical necrosis
c)parasitaemia 29.Features of neonatal septieaemia include
d)Cereral malaria a. those of respiratory distress
e)Severe anaemia b.scleroma
c.those of menigeal irritation

369
Compiled by Obasi. D. C. Chinedu

d.conjugated hyperbilurubinaemia b.When found in abdomen it is usually not across the midline
e.polycythaemia c.an intravenous urogram shews distortion of the calyceal architecture
d.It may cause paraplegia
30.A low birth weight baby: e.Urinary vanillymandelic acid is usually elevated
a.weighs less than 2500gm at birth irrespective of the gestational age
b.is small for gestational age when his weight falls below the 3rd percentile 39.The following are true concerning leukaemia
c.is more prone to persistence of the fetal circlution if he is SGA a.Anaemia is common
d.may result from chromosomal abnormality b.Thrombocytopenia is rare in acute leukaemia
c.White blood cell count of 140,000/cm of blood is associated with a poor
31.The following are true: prognosis in acute lymphoblastic leukaemia
a.Anaemia of prematurity is an exaggeration of a normal physiologic process d.diagnosis is confirmed by the presence of blast in peripheral blood
b.The lowest haematocrit level is reached earlier in a term than a preterm e.there is increase risk of leukaemia in Down’s syndrome
neonate
c.seizure may indicate the presence of polycythaaemia in a new born 40.The following epidermiological rates are correctly stated :
d. preterm appropriate for gestational age low birth weight babies have higher a.Incident rate = number of new cases in a stated period * 1000
incidence of polycythaemia than term low birth weight population at risk
e.The treatment of choice in a symptomatic neonate with polycythaemia is b.Prevalence rate = number of current cases at a specified time * 1000
plasmaphoresis population at risk
c.Case fatality rate = number of death ascribed to specific disease * 1000
32.The following are correctly matched: number of reported cases of the specified disease
a.Thrombocytosis and disseminated intravascular coagulaopathy(DIC) d.Infant mortality = number of deaths in infants under 1 year * 1000
b.Thrombocytopenia and sepsis without disseminated intravascular number of live births during the year
coagulopathy e.Mortality rate = number of deaths * 1000
c.Haemolytic anaemia and recticulocyte count above 6% mid-year population
d.Prolonged PT,PTTK and haemorragic disease of the newborn
e.Prolonged PT,PTTK and idiopathic thrombocytopenia 52.With respect to febrile convulsions
a.It occurs in children aged 6months to 5 years
33.The following congential disorders are well-matched with conditions they b.Seizure is focal in the simple variety
are usually associated with: c.Seizure rarely lasts more than 15minutes
a.potters syndrome : pulmonary hypoplasia d.Meningitis is common cause in our environment
b.Horse –shoe kidney : wilms’ tumour e.Long term prophylaxis with anticonvulsant is advisable after the first
c.prune belly syndrome : Deafness episode
d.Posterior ureathral valves: Cystic renal dysplasia
e.Autosomal recessive polycystic kidney : Liver fibrosis disease 53.A 5 day-old full term male infant weighting 3.0kg is admitted with
jaundice.Total bilirubin is 17.0mg/dl with a conjugated
34.Concerning HIV nephropathy in children the following are true: fraction of dl management with include
a.It manifest before AIDS develops a.Immediate exchange blood transfusion
b.It most commonly manifests as membranous nephropathy b.Oral phenobarbitone and instruction for check after 48hrs
c.HIV associated glomerulosclerosis has a striking predilection c.Phototherapy
d.Enlarged kidneys for age and height are constantly found at early and later d.Glucose drinks and Ampiclox
stages of the disease e.Reassuring the mother that it is probably phycological jaundice
e.Nephrotic syndrome is its most common manifestation
54.Petit mal:
a.Occurs predominantly in infants of 5-12 months
35.Pertaining to renal replacement therapy in children: b.Attacks are associated with loss of muscular tone and fall
a.It is commensed when GFR is 25/min/1.75m2 c.Respond to pyridoxine therapy
b.The ideal prescription calls for dialysis once or twice a week d.Electroencephalogram has the characteristics of hypsarrhythmia
c.It is possible in infancy e.Mental retardation is rare
d.Continous ambulatory peritoneal dialysis (CAPD) more user –friendly than
continous cycling peritoneal dialysis(CCPD) 55.Child hood morbidity and mortality in Ibadan
e.Dysequilibrium syndrome is less frequent with haemodialysis than a.Perinatal mortality rate is an accurate index of under five clinic services in a
peritoneal dialsis community
b.The commonest cause of perinatal mortality is prematurity
36.Concerning fluid and electrolyte management in childhood : c.Gastroenteri is a significant cause of morbidity and mortality in the 5-8 year
a.The ideal fluid for maintaining a newborn baby is normal saline group
b.Narrow’s solution contains less sodium per litre than ringer’s lactate d.EPI is an effective control measure in reducing neonatal mortality rate
c.The sodium bicarbonate in the salt sugar solution is adequate for preventing e.Poisoning is a significant cause of morbidity in the 1-3 year age group
metabolic acidosis in a child with acute watery diarrhea
d.Hypernatraemic dehydration is corrected very rapidly to prevent 56.Endocardial Fibrosis:
cerebrovascular events a.is a common congenital heart disease in Nigeria children
e.A 9kg infant with severe dehydration will normally require a total of b.Age of occurance is usually the first 5years of life
1,500mls of fluid in the first 24 hours c.Pericardial effusion is characteristic
d.Bilateral pedal oedema evolving to generalized oedema is a feature
37.The following statements relate to childhood urinary tract infection(UTI) e.A mid line diastolic murmur maximal at the apex is characteristic
a.UTI is commoner in females than in males at 2 years of age
b.Escherichia Coli is the commonest cause of UTI world wide 57.Adequate nutritional requirement of a 1/52-old low birth weight infant
c.Vesico ureteric reflux is present in 90% of cases of UTI includes intake of:
d.gentamicin is contraindicated in the treatment of UTI a.100-120 calories /kg/day
e.pyelonephritis is adequately treated with a 5-day course of antibiotics b.3-4gm/kg/day of protein
c.400 i.u/day of fluids
38.The following statements are true concerning neuroblastoma d.100ml/kg/day of fluids
a.The most common site of presentation is abdominal e.1mg vitamin k daily

370
Compiled by Obasi. D. C. Chinedu

67.Major factors causing anaemia in the tropics are :


58.The following are associated with autosomal recessive mode of a.Malaria especially p.vivax
inheritance; b.Viral and bacterial infections
a.Adult poycystic kidney disease c.Iron deficiency
b.Neurofibromatosis d.Folate deficiency
c.Achondroplasiamia e.The haemoglobinopathies
d.Haemophila
e.Glucose -6-phospate dehydrogenase 68.Corncering AIDS(Acquired Deficiency Syndrome )
a.The cause is HTLV
59.Hypogleaemia is a common problem in ; b.It can be cured with zidovuline
a.Low birth weight infants c.It is a common cause of morbidity and mortality in Nigeria children
b.Marasmic kwashiorkor d.It can be transplacentally acquired by the fetus
c.Sickle cell disease e.Haemophilines have a higher incidence
d.Bronchopneumonia
e.Cerebral malaria 69.Diabetes mellitus in children
a.Obesity is a usual predisposing factor
60.The following are important in the management of chronic renal failure b.Usually requires insulin for control
a.High protein diet in view of their stunted growth c.Ketoacidosis is a common mode of presentation
b.Calcitriol d.Potassium added to the IV fluid is important in the management of
c.Calcium carbornate ketoacidiosis
d.Recombinant Human Erythropoeitin e.Periodical monitoring of the glycosylated haemoglobin helps to ascertain
e.Glucose/insulin control

61.The following complications of immunization are appropriately matched; 70.In chronic Pyelonephritis :
a.Encephalitis – Perussis a.The are dialated calyces , cortical scarring unevening kidneys
b.Adenitis – Tetanus b.Congenital outlet obstruction
c.Osteomyelitis – Tuberculosis c.Red cell casts are typical urine findings
d.Vaccinia – Measles d.Intravenous pyelogram is an important investigation
e.Paralysis - Polio e.Hypertension is a frequently associated

62.In congenital Heart Disease 71.Indications for peritoneal dialysis in patients with acute renal failure are:
a.The commonest structural heart defects in 35% of fetuses exposed to rubella a.Uncontrollable hyperkalemia
in the first trimester include PDA,VSD & PS b.Serum sodium less than 130meq/L
b.VSD is the commonest form of congenital heart disease c.Fluid overload with pulmonary oedema
c.Tetralogy of Fallot is 80% likely if a neonate is born cyanosed d.Moderate metabolic acidosis
d.PDA causes a systolid and diastolic murmur e.Hallucinations
e.High VSD has characteristic ECG patterns
72.The following are potential complications of tetanus
63.The following pneumonias are likely to form multiple lung abscesses a.Hypertension
a.Chamydial b.Vertebral fracture
b.mycoplasma c.Ileus
c.staphyloccocal d.Retension of urine
d.klebsielial e.Myositis ossificans
e.streptococeal
73.Concerning meales
64.Acute respiratory infections(ARI) a.Transmission is by droplets or by contact with sick children during
a.It is the commonest cause of under 5 mortality worldwide exanthamatous stage only
b.malnourished children with severe acute lower respiratory tract infection b.2 antigenic type of virus are known
(ALRI) have a mortality risk two-to-two times higher c.In haemorragic type bleeding occurs into the brains
than that of well nourished children d.Quarantine is a helpful prophylaxis
c.most of the deaths are due to upper respiratory tract infection (URI) e.When complicated by giant cell pneumonia , cloxacillin and gentamicin are
d.In Ibadan Staphyococcus aureus and Klebsiella pneumonia are the useful
predominant bacterial pathogens of ALRI
e.Viruses are rare causes 74.Neonatal hypoglycaemia
a.Is frequent in the small-for-date
65.Bronchiolitis: b.May manifest as apnoeic attacks
a.It is common after the age of 2 years c.Is defined as blood sugar of 40mg/100ml or less during the first 72hours of
b.Bronchodialator is useful in the treatment life
c.Adenovirus is the predominant aethiological agent d.May complicate EBT
d.Antibiotics are not useful in the management e.Has overall prognosis even with minimal treatment
c.Heart failure is a common complication
75.Septicaemia in the early newborn period
66.In the management of acute diarrhea a.May be associated with subnormal temperature pattern
a.assessment of dehydration at the beginning of therapy is usually enough b.Most frequently due to gram negative organisms
b.The recommended rate of administration of oral rehydrating solution(ORS) c.Best treated with large doses of amoxicillin
is 15mls/kg/hr d.May be an indication for EBT with fresh whole blood
c.Oral rehydration is not contraindicated in the presence of glucose e.May be complicated by disseminated intravascular coagulopathy
malabsortion
d.Hyponateamic dehydration should be treated with hypotonic saline solution 76.In Duchenne Muscular dystrophy
e.Chemotherapeutic agents are essential adjunts to treatment of most cases of a.Creatinine phosphokinase at birth of 1000u/L is suggestive
diarrhea b.Muscle biopsy shows degeneration of muscle fibres
c.The inheritance is autosomal recessive

371
Compiled by Obasi. D. C. Chinedu

d.Cardiomyopathy is a cause of death


e.Is a differential diagnosis of cerebral palsy

77.The following are rich in folate :


a. Dark green vegetable leaves
b Bananas
c. Mangoes
d. Eggs
e. Fish

78.Common causes of diahorrea in children are:


a. Dissachardase deficiency
b. Gluten sensitivity
c. Complication of systemic infection such as urinary tract infections
d Allergy to cow’s milk protein
e. Injudicious use of antibiotics

79.Hazzards of blood transfusion include:


a. Circulatory overload
b. Febrile reactions which are commoner with plastic storage bags
c. Disseminated intravascular coagulopation
d. Shock
e. Haemoglobinuria

80.In congenital heart disease


a. The commonest structural heart defects seen in 35% of fetuses exposed
rubella in the first trimester include PDA , VSD & PS
bVSD is the commonest form
c.Tetralogy of Fallot is 80% likely if a neonate is born cyanosed
d. PDA causes a systolic and diastolic murmur
e. High VSD has characteristic ECG patterns

372
Compiled by Obasi. D. C. Chinedu

1. Congenital rubella:
a) Has an incubation period of 7-10 days. 10. Bloody stools occur in
b) May be complicated by polyarthralgia. a) Constipation
c) Rarely causes deafness. b) Intussusception
d) Is an indication for termination if it occurs in c) Typhoid fever
the first two months of pregnancy. d) Tropical sprue
e) May cause prolonged jaundice. e) Campylobacter enteritis

2. Recognised causes of delayed bone age include: 11. The following features are in favour of
a) Hypopiturtarism congenital biliary atresia in diffentiating this
b) Primary hypothyroidism condition from neonatal hepatitis
c) Congenital adrenal hypoplasia a) Family history of previous affected sib
d) Prolonged corticosteroid therapy b) Appearance of pale stool with the first ten days
e) Tuberculosis of life
c) Presence of urobilinogen in urine
3. Kwashiorkor: d) History of isoimmune hemolytic disease
a) Hypothermia is a recognized complication e) Raised serum transaminases
b) Edema is mainly due to protein losing
enteropathy. 12. Miscellaneous: the following are true
c) Measles is a recognized precipitant a) The silent infection of congenital rubella is
d) The incidence is highest in the first two month commoner
of life b) Severe psychomotor retardation occurs in
e) The birth of a second child to the mother may be congenital toxoplasmosis
a contributory factor c) CMV can not be transmitted by ingestion of
infected breast milk
4. At the age of eight months a baby can be expected d) Genital herpes infection is an indication for
to: delivery by caesarean section
a) Roll over from front to back e) The CSF is normal in congenital syphilis
b) Sit up with a straight back
c) Pick a small bead between thumb and finger 13. LTB can be differentiated from epiglottitis by
d) Say up to five word clearly the following features of the former
e) Feed himself with a spoon a) Very abrupt onset
b) Isolation of a virus in a trachea aspirate
5. if a child in the ward's develops measles, the c) Fixed circumferential subglottic narrowing on the
following action are appropriate lateral X-ray of neck
a) Close the wards to all admissions for one week d) Inspiratory stridor
b) Actively immunized all the other patients against e) Dramatic response to antibiotic therapy
measles
c) Give gamma globulin to all patients who have not 14. The following are true of cerebral malaria in
been immunized or had measles Ibadan
d) Forbid visiting by the parents until the rash has a) It is less common in marasmic children
gone b) Hypoglycemia is an index of poor prognosis
e) Give prophylactic antibiotics to all contacts at c) Quinine is the drug of first choice
home d) The demonstration of crescentic gametocytes is
essential for diagnosis
6. Convulsion in the first week of life is e) The use of mannitol i.v. increase the chance of
characteristic of survival
a) Hypocalcaemia
b) Post maturity 15. Hypotonic dehydration in infants
c) Craniopharyngioma a) Occurs more commonly in malnourished ones
d) Hypomagnesaemia b) is associated with irritability and hypertonic
e) Birth asphyxia c) Loss of skin turgor is usually present
d) Acidosis is often marked
7. Bronchial asthma: e) Correction is by infusion of dextrose solution
a) is associated with recurrent rather than chronic
persistent cough 16. The following may be responsible for
b) Is inherited in a Mendelian fashion hypoglycemia in a newborn baby
c) May be associated with flexural eczema a) small-for-gestational-age baby
d) Rarely occurs in infancy b) Beckwith syndrome
e) Requires laboratory investigation for diagnosis c) Biliary atresia
d) prematurity
8. Both human breast milk and cow's milk can be used e) Glycogen storage disease
for feeding infants but they differ in that
a) Human breast milk contains more 17. In TOF
carbohydrate/100mlof milk than cow's milk a) Failure to thrive is a common feature
b) Human milk is richer in protein than cow b) Cyanosis usually appears during the second year
c) cow's milk has more calcium than human of life
d) Cow's milk contains less vitamin c than human c) The lung field in the chest X-ray shows increased
milk vascular markings
e) Human milk is poorer in vit.D than cow's milk d) There is evidence of right atrial hypertrophy in
the ECG
9. The following combinations are associated with e) Apical pan systolic murmur is typical
congenital heart disease
a) Maternal diabetes mellitus 18. Petit mal
b) Rubella infection after the 16th week of a) Occurs predominantly in infants of 3 to 12months
pregnancy b) Attacks are associated with loss of muscle tone
c) Marfan's syndrome and fall
d) c) Respond to pyridoxine therapy
e) d) EEG has the characteristic

373
Compiled by Obasi. D. C. Chinedu

e) Mental retardation is rare b) Myocardial damage is a likely complication of


severe birth asphyxia
19. NS in Nigeria- the following are usual features c) The paraplegia associated with spinal TB is
a) onset before age of 6years usually of the spastic variety
b) Associated with P.falciparium infection d) Crytorchidism is one of the components of prune
c) Persistent vomiting belly syndrome
d) Normal cholesterol level e) Burkitt’s lymphoma is an important cause of death
e) Good response to steroids and cyclophoshamide in pre school age in Nigeria

20. Chronic renal failure 29) Causes of respiratory distress in the newborn
a) Polyuria may occur include
b) Hypertension is uncommon a) Pneumothorax
c) Hypocalcaemia may occur b) Congenital diaphragmatic hernia of the morgagni
d) Associated anemia is microcytic & hypochromic type
e) Pleural rub may be heard on auscultation c) TOF
d) Persistence of fetal circulation
21. Common causes of renal tract obstruction in e) Hypothermia
children are
a) Phimosis 30) In AGN
b) Nephrolithiasis a) Circulating immune complexes can be demonstrated
c) Posterior urethral valve b) Plasma infusion is a useful adjunct to treatment
d) Congenital urethral stricture c) Heart failure is a possible complication
e) d) AntiDNAse is suggestive of recent throat
infection
22 In mitral incompetence e) Hyperkalemia is a common finding
a) The pulse is collapsing
b) The characteristic murmur is diastolic in timing 31) Enuresis
c) The murmur is maximal at the left sternal edge a) Is night time bed wetting beyond the age of 5
d) A left ventricular enlargement occur b) Measurement of bld glucose is indicated
e) Rheumatic fever is the commonest cause in c) Urine microscopy is essential before starting tx
childhood d) Is often associated with petit mal seizure
e) Drug use has no value
23) The following are appropriately matched
32) A 4yr old boy has a severe seizure disorder and
a) Cyclophosphamide /alopecia mental retardation; the following are true
b) ampicillin/diarrhoea a) A facial hemangioma is a pointer to the diagnosis
c) carbimazole/agranulocytosis b) Intracranial calcification may be present
d) Prednisolone/gastric erosion c) He may have one of the neurocutaneous syndromes
e) Vincristine/peripheral neuropathy d) CT skull is essential for diagnosis
e) The prognosis is poor
24) The following are features of lobar pneumonia in
infants 33) The following programmes would enhance the
a) Chest pain realization of health for all by the year 2000 in
b) Trachea deviation Nigeria
c) Bronchial breadth sound allover the chest a) Nomadic education
d) Grunting respiration b) Better life for rural women
e) Vomiting c) A national food policy
d) Supply of contraceptive pills at subsidized rates
25) The following are features of neonatal tetanus e) Tetanus toxoid i.m. each term to all primary 6
a) Convulsion school girls
b) Apnoiec attack
c) Difficulty in sucking 34) Breast milk
d) Stiffening out attack a) Is rich in vit.C
e) Acquisition of life long immunity against tetanus b) Is contraindicated in galactosaemia
c) Is produced in response to prolactin release
26) Neonatal hypoglceamia d) Is low in phosphorus compared with cow's milk
a) Is bld glucose of less than 30mg/100ml during the e) Supplies about 67 calories per 100cc
first 3 days of life in both fullterm & low birth
weight infants 35) Reye's syndrome
b) Most neonates with hypoglceamia are asymtomatic a) May result from aspirin ingestion
c) Is more common in preterm than in small for b) Is usually anicteric
gestational age infants c) Is a form of hepatic failure
d) Can be definitely diagnosed from the symptoms d) May be associated with influenza type b virus
e) Is treated with dextrose 5% e) Is not accompanied by coma

27) Poliomyelitis 36) Biliary atresia


a) The infection is transmitted through inhalation a) The presence of conjugated hyperbilirubinaemia is
of infectious droplets diagnostic
b) It is caused by an RNA virus b) About 5% of patient having EBT will later develop
c) The vaccine used in Nigeria consist of killed biliary atresia
virus c) Most affected babies are small for gestational
d) In the acute phase patient may present with signs age
& symptoms of meningitis d) The incidence is related to paternal age
e) The virus attack the posterior column of the e) Cholestyramine may be beneficial after surgery
spinal cord
37) Reflexes normally present at birth include
28) Miscellaneous a) Moro
a) Erb's palsy result from damage to c5 &c6 roots b) Tonic neck
c) Parachute

374
Compiled by Obasi. D. C. Chinedu

d) Rooting a) In eight % of black population


e) Palmar grasp b) Only in sickle cell anemia but not in sickle cell
trait
38) Poisoning in childhood c) In thalassemia and sickle cell trait occurring
a) Alkaline diuresis is a means of enhancing together
elimination of phenobarbitone d) In congenital spherocytosis
b) Haematuria is a feature of kerosene poisoning e)
c) Hyperpyrexia is a feature of salicylate poisoning
d) Hypoglycemia may complicatepoisoning by cow's 48) Vomiting in the newborn can be caused by
urine a) Malrotation &volvulus
e) Desferrioxamine is used for TX of iron poisoning b) Intestinal atresia
c) Meconium ileus
39) Concerning the composition of fluids used in d) Sepsis
rehydration e) All of the above
a) Darrow's solution contains 36mmol/l of potassium
b) Normal saline contains 154meq/l of sodium 49) Children with juvenile diabetes
c) 8.4% of NaCO3 contains meq/ml of NaCO3 a) Usually have abrupt onset of symptom
d) Home made salt sugar solution contains 15-79mmol b) May be treated with oral hypoglycemic agent
of sodium and 15mmol of sodium bicarbonate per litre c) Need not be concern with the degenerative
e) Standard oral rehydration salt solution contains complication of the dx
2ommol of potassium chloride and 30mmol of sodium d) Have marked variation in their insulin
bicarbonate per litre requirement
e)
40) Portal hypertension
a) Propanolol is useful in its mx 50) Breast fed infant should receive supplemental
b) Impalpable spleen makes the diagnosis unlikely a) Vit.A
c) May result from sickle cell anemia b) Vit.D
d) Variceal injection sclerotherapyis the TX of c) Vit.K
choice d) Fluoride
e) May present with pulmonary hemorrhage e) Iron

41) The following measures are appropriate in the 51) The congenital rubella syndrome include
resuscitation of a neonate born with Apgar score of a) Cataracts
2 at 1 min b) Deafness
a) Oxygen administraion by face c) CHD
b) Emptying of the gastric content d) Microcephaly
c) Subcutaneous adrenaline 0.1cc/kg e)
d) 50% dextrose 0.5cc/kg diluted and infused into
the umbilical vein 52) The TORCH infection include
e) Intramuscular curamin a) Toxoplasmosis
b) Syphilis
42) Jaundice in the newborn may be caused by c) Rubella
a) Hepatitis d)
b) ABO bld grp incompatibilities e)
c) Congenital sherocytosis
d) Gallstones 53) Newborns may be particularly susceptible to gram
e) -ve infection because they have
a) Low level of IgM
43) The adrenogenital syndrome in early childhood B) High level of IgM
a) Occurs only in males c) Maternal IgG
b) May result from an autosomal recessive trait d) No maternal IgG
c) May cause virilization e)
d) May cause hypernatremia
e) 54) Sex linked dx include
a) Hemophilia
44) Meningitis in children is most frequently caused b) Cystic fibrosis
by c) Sickle cell anemia
a) Staphylococcus aureus d) Galactosaemia
b) H.influenza e)
c) N.meningococcus
d) Strep. pneumoniae 55) The most common anemia in children
e) a) Sickle cell anemia
b) Thalassemia
45) The common cyanotic CHD include c) Iron deficiency
a) VSD d) Congenital spherocytosis
b) ASD e)
c) TOF
d) TGA 56) Congenital nephrosis is characterized by
e) a) Hypertension
b) Elevated ASO titer
46) An infant with frequent pulmonary infection may c) Albuminuria
have d) Hypoproteinemia
a) Cystic fibrosis e)
b) Hypogammaglobulinemia
c) Chronic granulomatous dx 57) The most common cause of death in children is
d) a) Malnutrition
e) b) Malignancy
c) Accidents
47) Sickling of the RBC occur d) Congenital defect

375
Compiled by Obasi. D. C. Chinedu

e) b) HIV associated glomerulosclerosis has a striking


predilection for black
58) The average caloric requirements for infants c) Children with focal segmented sclerosis have a
during the first year of life better outcome
a) 80-90kcal/kg d) Enlarged kidneys for age & height are
b) 100-120kcal/kg consistently found at late stage of the dx
c) 120-130kcal/kg e) Proteinuria is the earliest and the most
d) 90-100kcal/kg consistent finding
e)
69) Pertaining to renal replacement therapy in
59) Neuroblastoma may present with children
a) Abdominal mass a) The typical prescription call for haemodialysis
b) Unilateral exo once or twice per week
c) Hematuria b) Haemodialysis is possible in infants as small as
d) Lowered VMA level 5kg
e) c) Anemia is common with peritoneal dialysis than
with haemodialysis
60) Nephroblastoma is d) Continous ambulatory peritoneal dialysis is more
a) Sometimes bilateral user friendly than continous cycling peritoneal
b) Associated with hemihypertrophy dialysis
c) Associated with aniridia e) The usual glucose content for the isotonic
d) dialysate for peritoneal dialysis is 3.5%
e)
70) Match:
61) Children with nutritional rickets are likely to a) DM/polygenic
have b) neurofibromatosis/autosomal recessive
a) A prolonged hx of breast feeding c) Duchenne muscular dystrophy /sex linked recessive
b) Growth failure d) G6PD deficiency/sex linked dominant
c) Craniotabes e) Achondroplasia….
d) all of the above
e) 71) With regard to meningitis
a) Common aetiological agents in the newborn are
62) Iron deficiency anemia is commonly found in gram -ve organism
a) Premature infant b) In pyogenic meningitis pending culture result the
b) Adolescent drug of choice is i.m penicillin & streptomycin
c) Adolescent girl c) The CSF is usually turbid in TB meningitis
d) Infant fed with whole milk d) The meninges may be the primary site of
e) tuberculous infection
e) The CSF glucose in aseptic meningitis is always
63) Acute epiglottitis normal
a) Caused by RSV
b) Caused by H.influenza 72)
c) Is a benign infection requiring only supportive a) Human milk is poorer in vit.D than cow's milk
therapy b) The neonates head increase by 0.5cm weekly if the
d) Is a life threatening infection baby was delivered at term
e) c) Metabolic acidosis may lead to failure to thrive
in the preterm infant
64) Cardiac lesion of TOF include d) Indomethacin is useful in medical mx of PDA
a) Aortic stenosis e) Aminophylline is useful in the TX of apnoea of
b) Pulmonary stenosis prematurity
c) ASD
d) VSD 73) Cerebral palsy
e) a) Is a progressive disorder of posture & movement
b) Results from injury to developing brain
65) A child with pyloric stenosis usually present c) Bilirubin encephalopathy is the commonest cause
a) Over the age of 6 months in Ibadan
b) With growth failure d) Spastic diplegia occur commonly in low birth
c) With seizure weight infant
d) With hypochloremic alkalosis e) Dyskinesia results from involuntary movement and
e) changes in muscle tone

66) The following are increased in conc. in the 74)0-hour old infant delivered at term presented
serum of nephrotic syndrome patients with marked respiratory distress. There was no
a) Factor B prolonged rupture of membranes and the baby weighed
b) Fibrinogen 1.0kg
c) Anti thrombin 3 a) Pneumothorax may occur
d) Transferrin b) Parenteral antibiotics are indicated
e) Vit.D binding protein c) The bld glucose may be less than 30mg/dl
d) Meconium aspiration is an unlikely cause of the
67) Match: respiratory distress
a) ark-upmark syndrome/hypotension e) Early feeding is contraindicated
b) Beckwith wiedemann syndrome/wilm's tumor
c) Potter's syndrome/polyhydramnios 75) Pertussis
d) AR PCKDx/liver fibrosis a) Young infant are nut passively protected by the
e) Barter’s syndrome/hyperkalemia transplancental transfer of antibody even if mother
has been infected in the past
68) Concerning HIV nephropathy in children b) The cough of pertussis last at least 3 months
a) It may manifest before AIDS develops c) Epistaxis and conjunctival hemorrhage may
complicate the illness

376
Compiled by Obasi. D. C. Chinedu

d) Reactivation of an old TB focus may occur e) Good response to hyperoxic test


e) Malnutrition may follow an attack
85) The following are common causes of coma in
76) A typical sickle cell patient has children in Ibadan
a) Has distended abdomen a) Meningitis
b) Has koilonychia b) Brain tumor
c) Has macrocephaly c) Cerebral malaria
d) Is pale &jaundice d) DM
e) Benefits from steroid therapy e) Tetanus

77) A healthy 6month old infant presented with a 86) Bone pain is a common feature of the ff
sudden onset of colicky abdominal pain and vomiting a) Sickle cell disease
a) A sausage shaped mass concave towards the b) Juvenile rheumatoid arthritis
umbilicus may be palpated on abd examination c) Acute leukemia
b) Mx should include passage of ng tube for d) Neuroblastoma
decompression e) Malaria
c) Dehydration may complicate the illness
d) Red currant jelly occurs in 2/3 of such cases 87) Pneumothorax
e) A laparotomy for reduction is the method of a) May complicate pertussis infection
choice b)
c)
78) Complication of veinous umbilical exchange bld d)
transfusion include e)
a) Abdominal distension
b) Hyperkalemia 88) Childhood tuberculosis
c) Metabolic acidosis a) BCG immunization offers absolute immunity
d) Cardiac arrhythmias b) Cough may not be a symptom in the pulmonary form
e) Portal hypertension c) Treatment with oral rifampicin is indicated in TB
meningitis
79) The following are with within normal limit for a d) Affectation of the lumbosacral region in TB spine
one year old infant is likely to give rise to paraplegia
a) Weight 6.0kg e) Duration of treatment with anti-TB drugs is for 6
b) Length 75cm months
c) Head circumference 52cm
d) Pulse rate 72/min 89) The ff are true of cardiac failure in the infant
e) Respiratory rate 25/min a) Bronchopneumonia is a common precipitating factor
b) The absence of pedal edema rules out the
80) Routine immunization is given by the following diagnosis
routes c) There is usually associated raised JVP
a) Measles/intradermal d) Tachycardia is a common feature
b) polio/oral e) The vaccine treatment is indicated irrespective
c) Triple vaccine /intramuscular of etiology
d) Small pox /intradermal
e) BCG/intradermal 90) The ff are causes of steatorrhea
a) Coeliac disease
81) Childhood m & m in Ibadan b) Giardiasis
a) Perinatal mortality rate is an accurate index of c) Tropical splenomegaly syndrome
under-five clinic services in a community d) Kwashiorkor
b) The commonest cause of perinatal mortality is e) Fibrocystic disease of the pancreas
prematurity
c) Gastroenteritis is a significant cause of m&m in 91) Nephroblastoma
the 5-8 year grp a) Haematuria may be a presenting feature
d) EPI is an effective control measure in reducing b) Occurs commonly in the preschool age
neonatal mortality rate c) The abdominal mass usually crosses the midline
e) Poisoning is a significant cause of morbidity in d) High level of VMA in the urine is diagnostic
the 1 - 3 year age group e) Surgery in combination with chemotherapy is the
treatment
82)
a) Significant hypocalcaemia is defined as total 92) Poisoning in childhood
serum calcium of less than 7mg/100ml a) Peak incidence is in the age group 5 - 8
b) Breast milk protein is more??? b) Salicylate poisoning may cause an initial
c) The normal neonatal blood volume is 80 -90 ml/kg respiratory alkalosis
d) Erb-Duchenne's palsy results from damage to the c) Syrup of ipecac may encourage elimination of the
C5 and C6 nerve roots poison by inducing diarrhoea
e) Erythrophoresis is contraindicated in neonatal d) In kerosene poisoning gastric washout is
polycythaemia contraindicated
e) Alkaline diuresis is useful in salicylate
83) Wide pulse pressure is often a feature of poisoning
a) Venous hum
b) Mitral incompetence 93) The ff are live vaccines
c) VSD a) Rabies
d) PDA b) Pertussis
e) Thyrotoxicosis c) Polio
d) Measles
84) Typical features of TOF include e) BCG
a) Presentation in the first year of life
b) Squatting 94) The ff lab results are typical of chronic renal
c) Single soft S2 failure
d) Heart failure a) Haematocrit of 20 %

377
Compiled by Obasi. D. C. Chinedu

b) Serum K+ 2.5 mEq/L b) Nerve roots C8 and T1


c) Plasma Ca++ 90mg/dL c) Nerve roots C5 and C6
d) Plasma inorganic phosphorus 3 mg/dL d) Phrenic nerve
e) Urinary specific gravity of 1010 e) Facial nerve

95) The ff are constant features of Down's syndrome 4) Poisoning


a) Mental retardation a) In alkaline diuresis, the pH must be about 8
b) Simian creases b) Nalorphine may be used as an antidote for
c) Hypotonia salicylate poisoning
d) Congenital heart disease c) Desferrioxamine is a specific chelator of iron
e) Translocation of chromosome 21 d) Subsequent esophageal stricture and pyloric
stenosis may occur with iron and strong alkali
96) The ff are characteristic of vitamin D deficient poisoning
rickets e) Hypoglycemia may result from cow urine poisoning
a) High serum calcium
b) Low serum phosphorus 5) Bullous impetigo
c) Low alkaline phosphatase a) Is not contagious
d) Delayed closure of the anterior fontanelle b) Intramuscular streptomycin is an effective
e) Hypertonia antibiotic
c) The most likely organism is Klebsiella
97) Nephrotic syndrome d) Affects both the epidermal and dermal layers of
a) The commonest cause in Nigeria is P. falciparum the skin
b) Steroids are the drugs of choice in patients with e) Affects only infants
poorly selective proteinuria
c) Hypoproteinemia is a constant finding 6) The ff are true regarding fluid and electrolyte
d) Quick acting diuretics may be dangerous because composition in a child
of hypovolaemia a) Dehydration in excess of 15% in childhood is
e) Spontaneous recovery is common in the idiopathic compatible with life
type b) Seizures in hypertonic dehydration is common
c) Release of ADH is stimulated by an increase in
98) Adequate nutritional requirements of a low birth the osmolality
weight infant includes d) The normal plasma osmolality is between 285 - 295
a) 100 - 120 calories /kg/day mOsm/kg water
b) 3 - 4 mg/kg /day of protein e) Isotonic dehydration is the commonest type of
c) 400 - 800 I.U. of vitamin D dehydration in this environment
d) 100 ml/kg/day of fluids
e) 1 mg vitamin K daily 7) Features of congenital hypothyroidism in this
environment include
99) An upper limb paralysis as a result of a) Prolonged physiological jaundice
poliomyelitis will show b) Meconium ileus
a) Wasting c) Polycythaemia
b) Hypotonia d) Tachycardia
c) Hypertonia e) Cold mottled skin
d) Fasciculations
e) Brisk reflexes 8) Glycosuria may be a feature of
a) Brain damage
100) Septicemia in the early newborn period is b) Diabetes insipidus
a) Not associated with subnormal temperature c) Renal tubular acidosis
patterns d) Febrile convulsions
b) Most frequently due to Gram-negative organisms e) Diabetes mellitus
c) Best treated with large doses of amoxicillin
d) Frequently causes jaundice 9) The ff are autosomal recessive inheritance
e) Is common sequelae to rupture of membranes more a) Galactosaemia
than 24 hrs before delivery b) G6PD deficiency
c) Sickle cell dx
d) Congenital extrahepatic biliary obstruction
e) Sydenhan's chorea

10) Neonatal tetanus


a) Commonest portal of entry is the umbilicus
PAST QUESTIONS b) The earlier the onset the worse prognosis
1) AGN c) the most important clinical feature is convulsion
a) Hx of preceding sore throat is usual in Nigerian d) Immunization of the mother during help to reduce
children incidence
b) Haemoglobinuria is common e) Onset of symptoms is usually the 4th week of life
c) Hypertension is a rare occurrence
d) Has a better prognosis than NS 11) Septicemia in the early newborn period is
e) The mainstay of TX is bed rest a) Often associated with normal temp. pattern
b) Most frequently due to gram -ve organism
2) Characteristic findings in uncomplicated c) Best TX with large dose of ampicillin
congenital hypertrophic pyloric stenosis include d) Frequent cause of jaundice
a) Anorexia e) An important cause of neonatal mortality
b) Marasmus
c) Dehydration 12) Hematemesis in childhood may be due to
d) Metabolic acidosis a) Acute leukemia
e) Mass in epigastrum b) Anaphylactoid purpura (henoch schonlein)
c) Aspirin ingestion
3) Klumpke's paralysis results from damage to d) Staphylococcus endotoxin
a) Radial nerve e) Ingestion of iron

378
Compiled by Obasi. D. C. Chinedu

13) Cyanosis 23) PDA


a) The extremities are blue and warm in peripheral a) Is a benign condition which rarely causes heart
cyanosis failure
b) Is not clinically recognized until reduced HB b) May be caused by intrauterine infection
exceeds 5gm/100ml c) The diastolic pressure is usually normal
c) Central cyanosis is due to deficient oxygenation d) Examination of the peripheral pulse is of no dx
of HB value
d) Inhalation of 100% oxygen helps to differentiate e) The characteristic continous murmur may be caused
between pulmonary and cardiac causes by other defect
e) Is an unusual feature of EMF
24) Turner’s syndrome
14) Causes of metabolic alkalosis are a) Negative Barr body
a) Prolonged bout of vomiting b) Hypoplastic nail
b) Status asthmaticus c) Cubitus valgus deformity
c) Severe malaria infection d) Short stature
d) CRF e) Amenorrhea
e) Salicylate poisoning
25) Oesophageal atresia
15) Uncommon feature of kwashiorkor include a) Associated tracheao-oesophageal fistula worsens
a) Ascites prognosis
b) Growth retardation b) Aspiration pneumonia is a major complication
c) Pedal edema c) Cyanosis on feeding occur
d) Splenomegaly d) Hx of polyhydramnios is usual
e) Dyspigmentation of the skin e) Delayed passage of meconium is characteristic

16) UTI 26) TB meningitis


a) Has an equal sex incidence in children under the a) Is a benign dx
age of 1yr b) is typically associated with CSF polymorph
b) May be symptomless pleocytosis
c) Should be considered in the inv. of obscure fever c) is as common as pyogenic meningitis
d) Congenital anomalies of the urinary tract are non d) Steroids are useful in mx
predisposing factor e) is sometimes associated with choriodal tubercles
e) Is essentially a benign condition
27) Sign of CCF in an infant of six months
17) A leg paralysis as a result of poliomyelitis a) Tachycardia
will show b) Pedal edema
a) Wasting c) Raised JVP
b) A positive babinski response d) Hepatomegaly
c) Exaggerated reflexes e) Crepitation in lung bases
d) Diminished tone
e) Glove and stocking anaesthesia 28) Lung abscess in childhood
a) is uncommon in childhood
18) The ff are uncommon features of cerebral malaria b) Respond favorably to medical therapy
a) Cortical blindness c) Often associated with finger clubbing
b) Metabolic alkalosis d) H.influenza is the most frequently cultured
c) Retinal hemorrhage organism from the sputum
d) Papilloedema e)
e) Hypoglycemia
29) In TOF
19) Intracranial calcification in childhood occur in a) Growth retardation is a common feature
a) CMV b) Cyanosis usually appear during the 2nd year of
b) Congenital rubella syndrome life
c) TB meningitis c) The lung field in a chest x-ray show increased
d) Congenital toxoplasmosis vascular markings
e) Craniopharyngioma d) There is evidence of left ventricular hypertrophy
in ECG
20) Common cause of severe anemia in Nigeria e) Subacute bacterial endocarditic is a common
children complication
a) Leukemia
b) Malaria 30) A 5yr old Nigerian child born in Britain is
c) Septicemia admitted to the ER with a 1day hx of high fever
d) Kwashiorkor (temp.40c) and an episode of convulsion. He returned
e) Empyema to Nigeria with his parents 3mnths earlier
a) The fever is the most likely cause of the
21) Common feature of whooping cough are convulsion
a) inc. period of 14- 21 days b) The fact that the child was born in Britain makes
b) Infective in the catarrhal and early paroxysmal meningitis an unlikely dx
stage c) Cerebral malaria is a probable dx
c) Mortality is high in any age group d) The finding of malaria parasite in bld film rules
d) Is an important cause of marasmus out meningitis
e) Penicillin is a drug of choice e) Lumbar puncture should be done only if convulsion
is recurrent
22) Congenital rubella syndrome
a) Can be dx at birth 31) A three year old girl presented with fever,
b) Microcephaly is an invariable feature cough and breathlessness. On examination there was
c) Is due to an arbovirus dullness on percussion and diminished air entry over
d) Tends to occur in subsequent siblings the RT lower lobe
e) a) This child has lobar pneumonia

379
Compiled by Obasi. D. C. Chinedu

b) Empyema is not likely because of the dull b) Exclusively breastfeeding infant for more than
percussion note six months
c) Pulmonary TB is the likely dx c) Supplementing breast milk with water only from
d) Pleural tap is an important part of mx time to time
e) A preceding hx of measles is likely d) Storing express breast milk for more than 24hr if
mother is away
32) Vomiting in the 1st week of life is due to the e) Colostrum should not be thrown away
ff
a) Pyloric stenosis 41) The ff are associated with intracranial
b) Duodenal stenosis hemorrhage in newborn
c) Hiatus hernia a) prematurity
d) Intussusception b) Birth asphyxia
e) tracheao-oesophageal fistula c) IV NaCO3
d) Neonatal seizure
33) The ff are present in 21-hydroxylase enzyme def. e) Hydrocephalus
of CAH.
a) Hypertension 42)
b) Hypokalemia a) porto-enterostomy is the TX of choice in all
c) Acidosis cases of conjugated hyperbilirubinaemia
d) Hyponatreamia b) Hirschsprung's dx may present as failure to
e) Hypochloraemia thrive
c) In Blount’s dx serum calcium is usually normal
34) In eczema d) Chronic diarrhoea may predispose to kwashiorkor
a) Acute lesions are usually dry thickened and scaly e) Anorectal manometry may aid the dx of congenital
with lichenification megacolon
b) Commonest type in childhood is atopic dermatitis
c) Allergic contact dermatitis is a T cell mediated 43) Genu valgus
hypersensitivity reaction a) Blount’s dx is a strong diff.
d) Prolonged use of hydrocortisone ointment is b) Wedge osteotomy on the lateral side is the TX of
without complication choice
e) A strong family hx of atopic manifestation is c) May result from inadequate of vit. D
present d) Serum calcium may be normal
e) Daily 10,000units of vit.D orally may be helpful
35) Childhood malignancy in the mx of renal rickets
a) Bilateral retinoblastoma is usually inherited as
an autosomal recessive trait 44) The ff are causes of steatorrhea
b) Surgery is the TX of choice in early a) Coeliac dx
retinoblastoma b) Giardiasis
c) ALL of childhood carries a better prognosis than c) TSS
AML d) Kwashiorkor
d) Commonest site of occurrence of rhabdomyosarcoma e) Fibrocystic dx of the pancreas
is the GUT
e) R-S cells are characteristic of Hodgkin’s 45) In marasmic kwashiorkor
lymphoma a) Lethargy is the usual form of presentation
b) There is weight for age deficit less than 60%
36) In TOF with edema
a) Heart failure is rare c) Hypothermia is a frequent occurrence
b) Cyanosis in infancy is uncommon d) gram-ve septicemia may occur
c) There is biventricular hypertrophy e)
d) Iron def. anemia can occur
e) Sodium bicarbonate may be indicated in its mx 46) Craniopharyngioma cause
a) Causes optic atrophy
37) The ff are drugs known to cause BM depression b) May cause polyuria
a) Aspirin c) Skull x-ray usually show calcification in area of
b) Chlorabucil tumor
c) Chloramphenicol d) May present as pituitary gigantism
d) Phenytoin sodium e) Is a rapidly growing fatal malignant tumor
e) Cyclophosphamide
47) In hypertrophic pyloric stenosis
38) Concerning pulmonary TB a) Bilious projectile vomiting is characteristic
a) It is compatible with a -ve tuberculin test b) Hypokalemia is a metabolic presentation
b) Absent chest signs rules out its dx c) Atropine methyl nitrate is a useful therapy
c) It is a cause of bronchiectasis d) Myomectomy is the surgical TX of choice
d) It is confined to the low socio economic class e) Can result in visible peristalsis from rt to left
e) Supraclavicular lymphadenopathy is highly
suggestive of this dx 48) The ff conditions are ass. With Down’s syndrome
a) Leukemia
39) UTI b) Hydrocephalus
a) Is common in the M than F c) CHD
b) Cause failure to thrive especially in newborn d) A large tongue
c) Is diagnosed by the finding of bacterial colony e) Brushfield spots
count of 50,000/ml of midstream urine specimen
d) When recurrent is an indication for IVU 49) Peak incidence of the ff condition are as ff
e) Is commonly caused by virus a) Measles 3-12months
b) Burkitt’s 3-5 years
40) The recommendation of the baby friendly hospital c) Kwashiorkor 2-3 years
initiative include d) Retinoblastoma 2-1 years
a) mother and the newborn baby resting 3hr before e) NS 5-7 years
breastfeeding commences

380
Compiled by Obasi. D. C. Chinedu

50) Ff are characteristic of vit.D def. rickets e) Chloride of 110meq/litre


a) Low serum calcium
b) High serum phosphates 71) Common causes of laryngeal obstruction includes
c) Normal alkaline phosphatase a) Foreign body
d) Delayed closure of the anterior fontanelle b) Measles
e) Hypotonia c) Retropharyngeal abscess
d) Tetanus
51) Common causes of MR in Nigeria include e) Acute necrotizing pharyngitis
a) Phenylketonuria
b) Tetanus 72) A 1yr old infant weighing 10kg requires /day
c) Cerebral birth trauma a) 1000calories
d) Down’s syndrome b) 600-800I.U OF VIT.A
e) Kernicterus C) 300-500I.UOF VIT.D
d) 15mg of vit C
52) Ascites E) 6OO-8OOML OF WATER
a) Fluid thrill is always present
b) Occur in portal hypertension 73) Lobar pneumonia
c) In EMF the fluids is an exudates a) Tactile fremitus is diminished
d) In abdominal TB the protein content of the fluid b) Breath sounds are bronchial over affected lobe
is more than 3g/100ml c) Percussion note is dull
e) Is excluded by dullness to percussion in the d) Trachea is displaced to the opposite
central part of the abdomen and in the flanks e) Air bronhogram may be seen in the consolidated
areas
53) The ff are feature of a large PDA
a) Continous murmur 74) Neonatal hypoglceamia
b) Normal or raised diastolic pressure a) Is define as bld sugar of 40mg or less during the
c) Biventricular heart failure 1st 72hrs of life
d) Loud p2 b) Is frequent in the small for dates
e) Normal sized lt ventricle c) Rare in babies of diabetic mother
d) Is common in asphyxiated babies
54) Hypertension e) Has good overall prognosis with minimal TX
a) May occur in early childhood
b) is often associated with AGN 75) Major causes of death in the 1st month of life
c) May be associated with Cushing syndrome are
d) Is a known complication of penicillin therapy a) Malaria
e) Known complication include renal failure b) Measles
c) prematurity
55) Rheumatoid arthritis in childhood d) Anemia
a) May present with very high fever e) Birth asphyxia
b) is not likely if splenomegaly occur
c) Corticosteroid is the drug of 1st choice 76) The ff statements about childhood mortality in
d) May affect atlanto axial joint Nigeria are true
e) Often cause stunting of growth a) Perinatal mortality rate = no. of stillbirths +
1st wk deaths /1000 live births
66) The ff are correct b) Infant mortality rate = no. of deaths in
a) P.falciparium 24hrs preschool children / 1000 live births
b) P.ovale 96hrs c) About 50% of infant deaths occur in the first
c) P.malariae 72hrs month of life
d) P.vivax irregular d) About 25% of infant deaths occur on the first day
e) of life
e) There is an association between perinatal
67) Juvenile hypothyroidism presenting at the age of mortality and maternal age
5yrs may have the ff features
a) Goitre 77) Common causes of neonatal jaundice in Nigeria
b) Deafness include
c) Full and bounding pulse a) Rh isoimmunization
d) Dwarfism b) Septicemia
e) Warm tremulous hands c) Prematurity
d) ABO incompatibility
68) Constipation is not a feature of e) Hypothyroidism
a) Congenital megacolon
b) Typhoid fever 78) A two day old infant is discovered to have a
c) Abdominal TB haematocrit of 30%
d) Down's a) Circumcision is a likely etiological factor
e) Sporadic cretinism b) It is useful to know if the infant is one of a
set of twins
69) The ff drugs are nephrotoxic in large doses c) A lumbar puncture is a relevant investigation
a) Methicillin d) The phenomenon may be due to hemorrhagic disease
b) Tetracycline of the newborn
c) Streptomycin e) Administration of haematinics is a preferred
d) Kanamycin method of treatment
e) Penicillin
79) The ff can be acquired transplacentally
70) The ff values are indications for peritoneal a) Rubella
dialysis b) Opthalmia neonatorum
a) Urea of 200mg c) Group B beta-hemolytic septicemia
b) Potassium of eight meq/litre d) Syphilis
c) Bicarbonate of 8meq/litre e) NEC
d) Sodium of 145meq/litre

381
Compiled by Obasi. D. C. Chinedu

80) In neonatal tetanus a) Sinus bradycardia


a) 20000 units of ATS should be administered IV stat b) Nodal tachycardia
b) Crystalline penicillin is administered IM six c) Ventricular coupling
hourly for one week d) Unilateral gynaecomastia
c) The common regimen for sedation consists of e) Constipation
phenobarbitone, chlorpromazine and chlordiazepoxide
d) Death from aspiration pneumonia is common 90) The ff are relevant in the investigation of an
e) There is an inverse relationship between the infant with prolonged conjugated hyperbilirubinuria
incubation period an the case fatality rate a) Blood culture
b) Peroxide haemolysis test
81) Major causes of perinatal death include c) Screen for hepatitis B surface antigen
a) Asphyxia neonatorum d) Oral cholecystography
b) Septicemia e) Abdominal ultrasonography
c) Bronchiolitis
d) Infective enteritis 91) A 3 month old girl presents with abdominal
e) CP distention. The diff diagnoses should include
a) Congenital pyloric obstruction
82) Stridor is a prominent feature of the ff b) Imperforate anus
a) Croup c) Oesophageal atresia
b) Foreign body in left main bronchus d) Hirschsprung's disease
c) Retropharyngeal abscess e) Gastroschisis
d) Acute epiglottitis
e) Laryngomalacia 92) Prolonged diarrhoea is a recognized feature of
a) Measles
83) In empyema thoracis b) Lactate deficiency
a) There may be no mediastinal shift c) Nutritional marasmus
b) Percussion note is dull on the contralateral side d) Typhoid
c) The breath sounds are bronchovesicular on the e) Chronic pancreatitis
affected side
d) A diagnostic needle aspiration via the 2nd 93) Common causes of constipation include
intercostal space anteriorly is often indicated a) Poor feeding methods
e) The haematocrit may be low on account of b) NEC
hemorrhage c) Rectal stenosis
d) Abdominal Burkitt's lymphoma
84) Causes of paratracheal shadowing on a chest e) Renal tubular acidosis
radiograph include
a) TB 94) Calcification on a plain radiograph of the
b) Hodgkin's lymphoma abdomen is seen in
c) ALL a) Tuberculous mesenteric lymphadenitis
d) Lung abscess b) Nephroblastoma
e) Hyaline membrane diseases c) Acute pancreatitis
d) Rhabdomyosarcoma
85) The ff are true of primary pulmonary TB e) Disseminated retinoblastoma
a) 80% of affected children will have primary
complex on presentation 95) The ff are true of Wilm's tumor
b) Miliary shadowing is the commonest X-ray finding a) It is an embryonal tumor
c) Cavitation does not occur b) It is sometimes inherited in an autosomal
d) It causes digital clubbing in 50% of the patients dominant manner
e) The infective organism can be recovered from the c) it is frequently bilateral
sputum in 25% of the patients d) IVU shows downward displacement of the affected
kidney
86) A wide pulse pressure is usually found in e) With surgery alone the 5 year survival rate is
a) Sickle cell haemoglobinopathy 80%
b) AV malformation
c) MR 96) Posterior urethral valves
d) AR a) Are found in both sexes but are commoner in boys
e) Pb poisoning b) A hx of poor urine stream is obtainable in a
majority of patients
87) In patients with moderately severe pericardial c) Often causes unilateral hydronephrosis
effusion d) Are best treated by placement of a prosthetic
a) The heart sounds are often muffled valve
b) The pulse volume is increased by the associated e) Diuresis may occur in the early post-operative
tachycardia period
c) The central venous pressure is usually elevated
d) Cardiac contour is globular because the 97) Common causes of hypertension in childhood
myocardium is damaged include
e) M mode ECG is diagnostic a) Acute nephritis
b) Chronic nephritis
88) The apex beat c) 11-hydroxylation defects in CAH
a) is located in the 4th left intercostal space in d) Coarctation of the aorta
most infants e) Renal artery stenosis
b) is diffuse in volume overload situations
c) Has a tapping character in patients with mitral 98) In a six year old with sickle cell
stenosis haemoglobinopathy
d) is best felt with the patient lying supine a) A haematocrit of 23% indicates a need for urgent
e) is located in the right hemithorax only in blood transfusion
patients with CHD b) The hand and foot syndrome is likely to occur
frequently
89) Signs of digoxin toxicity include

382
Compiled by Obasi. D. C. Chinedu

c) Anemia with reticulocyte response is more ominous 6) The ff objectives are attainable in the long term
than anemia without reticulocyte response mx of IDDM
d) Skin petechiae would be an uncommon finding a) Elimination of polyuria and polydipsia
e) Presence of cardiac murmur is usually indicative b) Normal growth and development
of the presence of an associated rheumatic heart c) Continuously normal bld glucose level
disease d) Prevention of microangiopathy
e) Maintenance of normal bld lipid level
99) A 15 month old boy presents with a haematocrit
of 10%. 7) Causes of failure to thrive
a) Hb genotype should be determined before blood a) TB
transfusion b) Chronic obstructive uropathy
b) He should be transfused with whole blood at 25 c) Maternal deprivation
mls/kg body weight d) Congenital pyloric stenosis
c) He should be transfused with packed red cells at e) Low birth weight
15 mls/kg body weight
d) IV 8.4% sodium bicarbonate 1 ml/kg body weight is 8) A 2yr old infant presents with a 4day hx of
desirable fever, cough and breathlessness, and a pustule on
e) the most likely cause of the anemia is malaria the rt foot. A chest x-ray brought from the referral
100) The ff values are normal for the associated doctor shows patches of consolidation and
ages pneumatoceles
a) Haematocrit of 55% at birth a) A bld culture is indicated
b) haematocrit of 35% at 3 months b) A lumbar is superfluous
c) Platelet count of 250,000 per cubic mm at age 2 c) The haematocrit should be checked
yrs d) Antibiotics therapy should consist of penicillin
d) MCHC of 32 % at age 3 yrs and streptomycin
e) Eosinophil count of 5% in a 7 yr old rural
dweller e) A sudden increase in the respiratory rate is
likely to be due to Pneumothorax

9) With respect to febrile seizure


a) It occurs in children age 6mnth -5yrs
b) Seizure is focal in the simple variety
c) Seizure rarely last more than 15 mins.
d) Meningitis is a common in our environment
e) Long term prophylaxis with antibiotics is
advisable after the 1st episode

10) A 5day old fullterm infant weighing 3.0kg is


PAST QUESTIONS admitted with jaundice, total bilirubin is 17.0mg/dl
1) Causes of thrombocytopenia include with a conjugated fraction of 1.0mg/dl. MX will
a) Birth asphyxia include
b) Salicylate poisoning a) Immediate EBT
c) H S purpura b) Oral phenobarbitone and instruction for bilirubin
d) Gram -ve septicemia checked after 48hrs
e) ALL c) Phototherapy
d) Glucose drinks and ampiclox syrup
2) Classical feature of meningitis in 1 month old e) Reassuring the mother that it is probably
infant include physiological jaundice
a) Positive kernig sign
b) Neck rigidity 11) Petit mal
c) Excessive crying a) Occurs predominantly in infants of 5-12 month
d) Vomiting b) Attacks are associated with loss of muscular tone
e) Convulsion and fall
c) Responds to pyridoxine therapy
3) The anterior horn cell is the site of damage in d) EEG has the characteristics of hypsarrythmia
a) G-B syndrome e) Mental retardation is rare
b) Paralytic polio
c) Hypertrophic muscular dystrophy 12) Childhood morbidity and mortality in Ibadan
d) MG a) Perinatal mortality is an accurate index of
e) Tetanus under-5 clinic services in the community
b) The commonest cause of perinatal mortality is
4) Measles prematurity
a) Is confined to children in the low socio economic c) Gastroenteritis is a significant cause of
grp perinatal mortality in the 5 - 8 year age group
b) The typical rash appear on the 2nd day of the d) EPI is an effective control measure in reducing
fever neonatal mortality rate
c) Koplik spots are best seen on the pharyngeal e) Poisoning is a significant cause of morbidity in
mucosa the 1-3 year age group
d) Encephalitis is a common complication
e) Second attacks are very uncommon 13) Endomyocardial fibrosis
a) Is a common CHD in Nigerian children
5) The ff are recognized complication of acute b) The age of occurrence is usually in the first 5
malaria in the 1st year of life years of life.
a) Hyperpyrexia c) Pericardial effusion is characteristic
b) Diarrhoea d) Bilateral pedal edema evolving to generalized
c) Convulsion oedema is a feature
d) Jaundiced e) A mid-diastolic murmur maximal at the apex is
e) Metabolic acidosis characteristic

383
Compiled by Obasi. D. C. Chinedu

14) Hypoglycemia is a common problem in c) Oral rehydration is not contraindicated in the


a) Low birth weight infants presence of glucose malabsortion
b) Marasmic kwashiorkor d) Hyponatreamia dehydration should be TX with
c) Sickle cell disease hypotonic saline solution
d) Bronchopneumonia e) Chemotherapy agents are essential adjunct to TX
e) Cerebral malaria of most doses of diarrhea

15) The ff are important in the management of 24) Major factor causing anemia in the tropics
chronic renal failure a) Malaria especially p. vivax
a) High protein diet in view of their stunted growth b) Viral and bacterial infection
b) Calcitriol c) Iron def.
c) Calcium carbonate d) Folate def.
d) Recombinant human erythropoietin e) The hbnopathies
e) glucose/insulin therapy
25) Clues in the dx of CH in the newborn
16) Which of the ff are correctly associated a) Large fontanelle
a) Encephalitis - pertussis b) Poor feeding and crying
b) Adenitis - tetanus c) Prolonged neonatal jaundice
c) Osteomyelitis - TB d) Raised thyrotropin level at 48hrs of age
d) Paralysis - polio e) Eyelid swelling
e) - measles
26) In chronic pyelonephritis
17) Pertussis infection a) There are dilated calyces, cortical scarring
a) Does not affect neonates unevenly contracted kidney
b) Incubation period is 4 - 6 weeks b) Congenital bladder outlet obstruction is a cause
c) the most infectious phase is the catarrhal phase c) Red cell casts are typical urine finding
b) May be complicated by bronchiectasis d) Intravenous pyelogram is an important inv.
e) May present with apnea e) Hypertension is frequently associated

18) The ff are true of cerebral malaria in Ibadan 27) Indications for peritoneal dialysis in px with
a) It is more common in marasmic children ARF are
b) Hypoglycemia is an index of poor prognosis a) Uncontrollable hyperkalemia
c) Quinine is the drug of first choice b) Serum Na less than 130meq /l
d) The demonstration of crescentic gametocytes is c) Fluid overload with pulmonary edema
essential for diagnosis d) Moderate metabolic acidosis
e) The use of dexamethasone improves chances of e) Hallucination
survival
28) The ff are potential complication of tetanus
19) In CHD a) Hypertension
a) The commonest structural heart defect seen in 35% b) Vertebral fracture
of fetuses exposed to rubella in the first trimester c) Ileus
include PDA, VSD and pulmonary stenosis d) Retention of urine
b) VSD is the commonest form of CHD e) Myositis ossificans
c) TOF is 80% likely if a neonate is born cyanosed
d) PDA causes a systolic and diastolic murmur 29) Concerning measles
e) High VSD has a xteristic ECG pattern a) Transmission is by droplets or by contact with
sick children during exanthematous stage only
20) The ff pneumonias are likely to form multiple b) 2 antigenic type of the virus are known
lung abscesses c) Quarantine is a very helpful prophylaxis
a) Chlamydia d)
b) Mycoplasmal e) When complicated by giant cell pneumonia,
c) Staphylococcal cloxacillin and gentamicin are useful
d) Klebsiella
e) Streptococcal 30) Neonatal hypoglycemia
a) Is frequent in the small for age
21) ARI b) Is defined as bld sugar of 40mg /100ml or loss
a) Is the 2nd commonest cause of under 5 mortality during the 72hrs of life
worldwide c) May complicated EBT
b) Malnourished children with severe ALRI have a d) Has a good overall prognosis even with minimal tx
mortality rate 2-3 times higher than well-nourished e) May manifest as apnoeic attacks
children
c) Most deaths are due to URTI 31) In duchenne muscular dystrophy
d) In Ibadan staphy. and Klebsiella are the a) Creatinine phosphokinase at birth of 100.u/l is
predominant bacteria pathogens of ALRI suggestive
e) Viruses are rare cause b) Muscle biopsy shows degeneration of muscle fibres
c) The inheritance is by AR
22) Bronchiolitis d) Cardiomyopathy is a cause of death
a) It is common after the age of two months e) Is a diff. diagnosis of Cerebral Palsy
b) Bronchodilator is useful in the TX
c) Arbovirus virus is the predominant aetiological 32) The ff are rich in folate
agent a) Dark green vegetable leaves
d) Heart failure is a common complication b) Bananas
e) Antibiotics are not useful in the mx c) Mangoes
d) Eggs
23) In the mx of acute diarrhoea e) Fish
a) Assessment of dehydration at the beginning of
therapy is usaually enough 33) Common causes of diarrhoea in children
b) The recommended rate of administration of oral a) Dissacharidase def.
rehydration solution is 15ml/kg hr b) Gluten sensitivity

384
Compiled by Obasi. D. C. Chinedu

c) Complication of systemic infection such as UTI


d) Allergy to cow milk protein 43) The ff are common types of TB in children in
e) Injudicious use of antibiotics Ibadan
a) TB adenitis
34) Hazards of bld transfusion include b) TB of the hip
a) Circulatory overload c) spinal TB
b) Febrile reaction which are common with plastic d) miliary TB
storage bags e) psoas abscess
c) DIC
d) Shock 44) The ff genetic traits may protect against
e) Haemoglobinuria malaria
a) membranocytosis
35) A hyperimmune response can be reflected b) elliptocytosis
clinically in TB by the appearance of c) ovalocytosis
a) Erythema nodosum d) G6PD def.
b) Phlycternular conjunctivitis e) sickle cell trait
c) Pterygium
d) Pleural effusion 45) The ff are true concerning hormones
e) Lung cavities a) protein hormones are synthesized mainly on demand
b) steroid hormones have diff. molecular precursor
36) Which of the ff are side effects of salbutamol, c) thyroid hormones are excreted from storage by
a beta stimulant pinocytosis
a) Tremor d) CAMP has a role to play in the synthesis of
b) Tachycardia adrenal cortical hormones
c) Headache e) some hormones are excreted in their active form
d) Hyperglycemia
e) Hypokalemia 46) Features of klinefelter's syndrome
a) indeterminate sex
37) Concerning Burkitt’s lymphoma b) persistence of childhood voice
a) It is the common children malignancy in Ibadan c) short stature
b) Its doubling time is about 72hrs d) obesity
c) It is assoc. with t(8:14) abnormality e) polyuria
d) It may be complicated by CRF
e) It is found more commonly in well nourished 47) The ff enzymes have been implicated in the
children adrogenital syndrome
a) 21- hydroxylase def.
38) The ff are seen in px with hyperkalemia b) peroxidase enzyme
a) Seizure c) 11-hydroxylase def
b) Cardiac arrhythmia d) 7dehydrocholesterol dehydrogenase def.
c) Tall p wave on ECG e) 3-beta hydroxysteroid dehydrogenase def.
D) Broad u wave on ECG
e) Bizarre qrs complexes 48) The ff features are compatible with the dx of
neonatal septicemia in a 1 week old neonate
39) Concerning coarctation of the aorta a) PROM for 48hrs in the mother
a) It is usually preductal b) bulging anterior fontanelle
b) Postductal type is associated with hypertension c) resp. distress
in the upper limb and normal bld pressure in the d) absolute WBC count of 10,000/m3
lower limb e) immature neutrophil count of 10%
c) May be associated with high renin hypertension
d) Tx of choice in uncomplicated cases is surgery in 49) the ff factors are important in the pathogenesis
the 1st mnth of life of renal osteodystrophy in CRF
e) Hypertension may persist after surgery a) defective calcium absorption from the gut
b) defect in 25 hydroxylation of vit.D
40) The ff conditions are assoc. with immune def. c) persistent metabolic acidosis
a) PEM d) primary hyperparathyroidism
b) ataxia telangiectasia e) defective calcitonin production
c) downs syndrome
d) HIV syndrome 50) The ff statements are true
e) cyclosporine A therapy a) SGA are more likely to be polycythemic when
compared with preterm ones
41) In nutritional rickets b) jaundice is more common in SGA compared to
a) serum calcium levels are always low preterm ones
b) plasma vit.D assay can assist dx c) congenital malformations are commoner in SGA than
c) Hyperparathyroidism is one of the causes preterm ones
d) renal dx has no part to play to in its causation d) hypoglycemia is more common in preterm than SGA
e) the bone deformities are irreversible e) weight loss in the 1st week of life is greater
among SGA than preterm
42) The ff are true of birth trauma
a) the swelling due to cephal haematoma usually 51) The ff are true of Steven Johnson syndrome
crosses the midline a) it is a very severe of erythema multiforme
b) a sternomastoid tumor usually occurs after breech b) the mucus membrane are not involved
delivery and can be seen within a few hrs after c) the skin lesion consist mainly of vesicles and
birth bullae
c) subcutaneous fat necrosis should be excised to d) it can be precipitated by anti TB drugs
prevent calcification and infection e) steroid therapy should only be given in a few
d) facial nerve injury is usually of upper motor cases
neuron type
e) in erb's palsy there is damage to the 6 &7 52) Complications of large lt to rt shunt
cervical nerve and is managed by physiotherapy a) recurrent chest infection

385
Compiled by Obasi. D. C. Chinedu

b) mental retardation e) it gives more quantitative than qualitative


c) CVA analysis
d) kwashiorkor
e) pericardial effusion 61. Ff are true of fluid and electrolyte
a) 2/3 of TBW is ECF
52) A fullterm , 2day old boy develops petechiae, b) hormones necessary for control include ADH,
and is found to have a platelet count of 2000/mm3. renin, angiotensin & aldosterone
The child is otherwise healthy and has no other c) can be disrupted by head injury
physical findings. The mother’s platelets count is d) insensible water loss is 800mls / day
normal. The likely diagnosis e) alkalosis can result from persistent vomiting
a) maternal autoimmune thrombocytopenia
b) isoimmune thrombocytopenia 62) Ff conditions and clinical pictures are properly
c) sepsis matched
d) congenital infection a) meckel's diverticulum/constant colicky abdominal
e) DIC pain
b) ulcerative colitis/red currant jelly stool
53) Myasthenia Gravis c) shigellosis/acute mucoid diarrhea
A) Uncommon in childhood d) intussusception/palpable abdominal mass
b) curable in about 25%of affected children e) rectal polyps/recurrent painless bleeding
c) tx with pyridostigmine bromide
d) not assoc. with other immunological dx 63) Ff are diff. of arthritis in this environment
e) dx clinically by the demonstration of progressive a) sickle cell dx
weakness on repetitive or sustained muscular b) rheumatic fever
contraction c) TB
d) still's dx
54) Mx of a 3 day old neonate with jaundice may e) perthe's dx
include
a) phenobarbitone 64) Ff dx conditions and diagnostic procedures are
b) glucose water properly matched
c) EBT a) acute hepatocellular damage serum aspartate
d) ampiclox syrup transferase
e) phototherapy b) bone dx serum acid phosphatase
c) skeletal muscle dx serum creatine kinase
55) Polyuria occur in the ff condition d) G6PD def. serum bilirubin
a) DM e) galactosaemia serum glucose 1 phoshate uridyl
b) eosinophil granuloma kinase
c) hypocalcaemia
d) CRI 65) Ff may predispose to UTI
e) Cushing syndrome a) female sex
b) renal calculi
56) Congenital malformation has been assoc. with the c) ambiguous genitalia
use of d)
a) the use of diethylstilbestrol e)
b) thalidomide tx during pregnancy
c) maternal rubella in the 2nd trimester 66) Ff are recognized complication of pertussis
d) a positive family hx infection
e) mental retardation a) meningitis
b) atelectasis
57) Neonatal hyperbilirubinaemia can be made c) pleural effusion
potentially more dangerous by d) bronchiectasis
a) acidosis e) PEM
b) low serum albumin
c) low level of FFA 67) In oesophageal atresia
d) hypoxia a) assoc. tracheo-oesophageal fistula worsens the
e) hyperthermia prognosis
b) a hx of polyhydramnios is unusual
58) Ff syndromes have renal involvement c) there is delayed passage of meconium
a) downs d) cyanosis during feeding is common
b) turners e) px are better nursed in the head down position
c) barters
d) klinefelter’s 68) Infantile gastroenteritis
e) pheochromocytoma a) is often caused by the rotavirus
b) antibiotics tx with cotrimoxazole and
59) Ff are true of IDM metronidazole is often indicated
a) it is an autoimmune dx c) isotonic dehydration is a more common
b) antibodies have been demonstrated against complication than hypertonic dehydration in this
glutamic acid decarboxylase enzyme environment
c) px are ketosis resistant d) may be successfully mx with oral rehydration
d) it is commoner in West African blacks than therapy
American blacks e) breastfeeding is contraindicated
e) it is commoner in obese females
69) In treating neonatal infections
6o) Ff are true of 24 hr urine collection a) chloramphenicol may cause circulatory collapse
a) it can be started anytime of the morning provided b) ampicillin is the drug of choice for staph.
it ends at the same time the ff day sepsis
b) the preservative placed in the container is 5 c) sulphonamide increase the risk of kernicterus
of sulphuric acid d) erythromycin may be conveniently given
c) it can be used to measure creatinine clearance intravenously
d) the container must be clear and colourless e) gentamicin may cause nephrotoxicity

386
Compiled by Obasi. D. C. Chinedu

c) x-ray of affected joints


70) Atopic dermatitis d) ECG
a) pruritus is marked e) skin snip
b) there is tendency to remission at 3-5yr of age
c) there may be a genetically determine predilection 80) Ff conditions can cause resp acidosis
d) secondary bacterial infection may occur a) bronchopneumonia
e) topical tx is contraindicated b) status asthmaticus
c) atelectasis
71) Complications of cleft palate include d) resp.collapse
a) recurrent otitis media e) RDS
b) malpositioning of the teeth
c) speech defect 81) Pleural effusion is a complication of
d) hearing loss a) nephrotic syndrome
e) meningitis b) pulmonary TB
c) Burkitt’s lymphoma
72) Concerning percutaneous liver biopsy d) aspiration pneumonia
a) it is contraindicated in the presence of ascites e) kerosene poisoning
b) hemorrhage is the commonest complication
c) Pneumothorax is a possible complication 82) Ff organism causes pneumonia in infants
d) accuracy of diff biliary atresia from neonatal a) pneumocystis carinii
hepatitis is 100% b) Candida albicans
e) antibiotics prophylaxis is necessary in px with c) staph. epidemidis
cardiovascular dx d) H influenza
e) strep. pneumonia
73) G6PD def
a) affect only negroes 83) BCG
b) is commoner in males than females a) live attenuated bovine vaccine
c) is a primary result of a defective enzyme in the b) can be given prophylactically only
Hexose Monophosphate Shunt c) tuberculin skin test is positive 1/52 after BCG
d) is a common cause of neonatal jaundice in Ibadan d) cannot be given to someone with febrile illness
e) causes coombs -ve hemolytic anemia e)

74) Complications of phototherapy 84) Tuberous sclerosis features include


a) diarrhea a) seizure disorders
b) increased insensible water loss b) intracranial calcification
c) bronze baby syndrome c) capillary hemangioma
d) skin rashes d) hydrocephalous
e) increased platelet turnover e) hypsarrythmia on EEG

75) Ff are compatible with diagnosis of isolated PS 85) Complications of lumbar puncture
a) normal nutritional status a) headache
b) pink mucosae b) vomiting
c) heart failure c) injury to the spinal cord
d) loud pulmonary closure sound d) meningitis
e) ejection systolic murmur loudest at the lt e) sciatic nerve injury
sternal edge
86) The ff differential diagnoses could be
76) Ff may be indicated in the tx of infective considered in a 6 year old boy with generalized
endocarditis seizures
a) a) head trauma
b) b) petit mal epilepsy
c) c) chronic renal failure
d) cerebral malaria
d) e) sickle cell disease
e)
87) Facial palsy may occur in the ff conditions
77) Concerning tricuspid atresia a) following forceps delivery
a) px most often present after the 1st yr of life b) chronic mastoiditis
b) presence of heart failure makes the diagnosis c) cerebellar tumour
unlikely d) bacterial meningitis
c) no chest x-ray feature is characteristic e) poliomyelitis
d) ECG shows rt axis deviation
e) surgery of choice is mustard's operation 88) Reflexes normally present at birth include
a) parachute
78) The ff statements are true of VSD b) stepping
a) a loud means that the murmur is loud??? c) tonic neck
b) a precordial bulge signifies rt ventricular d) rooting
enlargement e) gag
c) a child who is asymtomatic is in infancy is still
likely to go into heart failure before the age of 89) Concerning poliomyelitis
2yr a) the virus is a DNA virus
d) the chest x-ray may be normal b) it is transmitted by the faeco-oral route
e) pulmonary artery banding requires cardiopulmonary c) the virus travels along axons in the posterior
bypass column of the spinal cord
d) the paralysis is usually asymmetrical
79) Ff are useful in confirming a diagnosis of acute e) proximal muscle groups are more often affected
rheumatic fever than distal muscle groups
a) bld culture
b) ESR 90) Myasthenia gravis

387
Compiled by Obasi. D. C. Chinedu

a) is an autosomal recessive dx d) diarrhoea is an important cause of death in


b) the pathological lesion affects the motor end Ibadan
plate e) Burkitt's lymphoma is an important cause of death
c) Edrophonium is the drug of choice for tx in the preschool age in Ibadan
d) Most patients die before the age of 20 years
e) Diagnosis is majorly clinical

91) Daily requirements in the first month of life in


a full term baby
a) water: 75 ml/kg/day
b) calories: 100 kg/day
c) Vitamin D: 1000 units/day
d) Protein: 2g/kg/day
e) potassium: 4 mEq/kg/day
PAST QUESTIONS
92) Which of the ff are true of congenital 1) Acute rheumatic heart dx in Nigerian children
hypertrophic pyloric stenosis? a) is etiologically related to streptococcal throat
a) presenting symptom is constipation infection
b) more frequent in females b) commonly result in mitral regurgitation
c) more frequent in the firstborn of the family c) occurs in infancy
d) metabolic alkalosis is a common complication d) is assoc. with a raised ESR
e) barium studies are mandatory for the diagnosis e) is rare

93) Common causes of persistently elevated blood 2) Dermatitis


pressure in children include a) about 50%of cases of atopic dermatitis resolve by
a) Cushing's syndrome the age of 2yr
b) Essential hypertension b) there is no pruritus in infantile atopic
c) Renal disease dermatitis
d) coarctation of the aorta c) napkin dermatitis due to moniliasis should be tx
e) thyrotoxicosis with exposure and painting with 1% aq. sol. of
gentian violet
94) Craniopharyngioma d) topical corticosteroid cream may be effective in
a) arise from the posterior pituitary gland the tx of seborrheic dermatitis
b) are the commonest intracranial tumours in e) in atopic dermatitis a strong family hx of other
children atopic manifestation may be present
c) may compress the optic chiasma
d) may become calcified 3) A 3kg baby was delivered by emergency lower
e) respond well to radiotherapy segment caesarean section to a 16yr old primigravida
at term after a labour which lasted 48hrs. The
96) Nasopharyngitis liquor was meconium stained. At5min of age his hrt
a) causes difficulty in feeding in infant rate was 96/min. The baby was gasping and there was
b) can be caused by grp A streptococcus no response to the nasal catheter. He was also
c) may be differentiated from allergic rhinitis by floppy and pale. The ff are true
the presence of neutrophil in the nasal smear in the a) the apgar score is 2
later b) endotracheal intubation after adequate suctioning
d) liberal fluid are contraindicated is indicated for the administration of oxygen
e) is routinely tx with antibiotics c) IV administration of alkali is the 1st priority
d) Pneumothorax is a possible complication
97) Ff are true of TGA e) seizure within the 1st 24hr will be unusual
a) it presents with cyanosis at birth
b) heart failure is a common feature 4) Epilepsy
c) the ECG shows evidence of LVH a) hypsarrhythmia is a typical finding in infantile
d) cardiac catheterization in the neonate is of spasm
diagnostic value only b) ethosuximide is a drug of choice in the tx of
e) total correction is not possible in the infant psychomotor epilepsy
c) petit mal usually has its onset in the 2nd decade
98) Hypotonic dehydration in infants of life
a) acidosis is seldom marked d) the prognosis for infantile spasm is good
b) correction is by infusion of full-strength e) febrile convulsion can occur at the onset of the
c) loss of skin turgor is usually present fever
d) occurs more commonly in well-nourished than in
malnourished ones 5) In Nigeria chemotherapy against malaria
e) is associated with ileus a) should be provided using halofantrine
b) should be given during tx of kwashiorkor
99) Causes of ecchymoses in infants include c) should be given to all children under 5 yr of age
a) sickle cell disease d) should be given during tx of ALL
b) maternal ITP e) should not be given to pregnant women because it
c) hemorrhagic disease of the newborn will reduce malaria antibody titre in their babies
d) DIC
e) severe birth asphyxia 6) Bone marrow hypoplasia occurs in
a) folate def. anemia
200) The ff statements concerning childhood b) iron def anemia
morbidity and mortality are true c) fanconi syndrome
a) perinatal period is defined as 28 weeks of d) parvovirus B19 infection
gestation and above + the first 7 days of life e) Blackfan-Diamond syndrome
b) neonatal mortality rate is the number of neonatal
deaths per 1000 of all births 7) The ff are known to be associated with sickle
c) infant mortality rate is the number of infant cell anemia in Nigerian children
deaths per 1000 of all births a) Hb S+C

388
Compiled by Obasi. D. C. Chinedu

b) avascular necrosis of the humeral head b) cardiomegaly


c) zinc deficiency c) digital clubbing
d) toe clubbing d) pulmonary plethora
e) microfinger e) polycythaemia

8) A known sickle cell anemia patient is admitted 14) A 1 year old child weighing 8 kg presenting with
with sever drowsiness, rectal temperature of 39.5 C, diarrhoea and vomiting will most likely have the ff
PCV of 18%, deep jaundice and generalized hypotonia biochemical abnormalities
and absent deep tendon reflexes on the left side. a) acidosis
This child b) hypokalemia
a) may have had a cerebrovascular accident c) hypocalcaemia
b) may have cerebral malaria d) hyponatreamia
c) needs urgent packed cell transfusion e) hypochloraemia
d) needs daily penicillin and streptomycin
injections 15) The ff are correct
e) is likely to have bone marrow erythroid a) 150 - 180 ml of blood is required per kg body
hyperplasia weight for full EBT
b) haematocrit of 20% is an indication for blood
9) A 3 year old child presents with fever, malaise, transfusion in sickle cell disease
bleeding gums and splenomegaly. His PCV was 24%, WBC c) a cretin is an active child
was 28,000 and platelet count was 20,000. d) most Nigerian babies walk by 12 months
a) The blood cell count suggests a diagnosis of e) neuroblastoma usually damages the kidney
acute leukemia
b) normal white cell count would have ruled out 16)
acute leukemia a) penicillin is a drug of choice in empyema
c) corticosteroids are used for inducing remission b) ethosuximide is a drug of choice in temporal lobe
in acute leukemia epilepsy
d) in acute leukemia, lumbar puncture is not c) actinomycin D is a drug of choice in
indicated unless there are neurological signs neuroblastoma
e) the thrombocytopenia is due to consumption d) surgery is not the primary treatment for Tb
coagulopathy lymphadenopathy
e) normal saline is the fluid of choice for treating
10) Common causes of heart failure are severe dehydration in a marasmic 3 year old child
a) iron deficiency anemia
b) rheumatic heart disease 17) Cataracts may be a feature of
c) EMF a) congenital syphilis
d) myocarditis b) toxoplasmosis
e) malnutrition c) rubella infection in infancy
d) galactosaemia
11) Anterior fontanelle e) gargoylism
a) is a useful indicator of dehydration
b) is always tense in meningitis 18) Drug of choice
c) usually closes during infancy a) in status epilepticus is phenobarbitone
d) late closure is common in rickets b) in status asthmaticus is subcutaneous adrenaline
e) is pulsatile in a neonate c) in petit mal epilepsy is phenytoin
d) in meningococcal meningitis is penicillin G
12) A known diabetic 2 year old male child was e) in giardiasis is mepacrine
admitted at midday into the OTCHEW in coma with
acidotic respiration. The mother gave the history 19) The ff findings help to distinguish nephrotic
that the child has had fever and anorexia for one syndrome from kwashiorkor
day. He had refused both his dinner the night before a) proteinuria
admission and his breakfast the morning of b) hypercholesterolemia
admission. The mother gave him insulin as usual that c) hypoproteinemia
morning and the child vomited soon after before d) ascites
gradually losing consciousness e) pallor
a) The child is in diabetic coma because he has
acidotic breathing 20) Biliary atresia cause
b) Diabetic ketoacidosis is ruled out if urine a) acholuric jaundice
testing reveals the presence of ketones but no sugar b) is associated with pale stools
c) hypoglycemic therapy due to insulin therapy was c) hepatocellular fxn is unaffected
unlikely since the child must have vomited the drug d) clubbing of the digits is an early feature
d) acidotic breathing may occur in acute malaria e) rickets is a common complication
fever
e) intravenous injection of 20 -25 mls of 21) Portal hypertension
concentrated glucose solution is a rapid and safe a) does not occur in childhood
method of differentiating between hypoglycemic and b) may be a long-term complication of EBT
hyperglycemic coma c) commonest predisposing factor in children in UCH
is biliary cirrhosis
13) A 3 year old child with TOF will have the ff d) splenomegaly is a constant finding
features e) always carries a poor prognosis
a) central cyanosis

389

S-ar putea să vă placă și